Você está na página 1de 382

Punsalan v. vda. De Lacsamana [G.R. No. L-55729. March 28, 1983.

] First Division, Melencio-Herrera (J): 5 concur Facts: Antonio Punsalan, Jr., was the former registered owner of a parcel of land consisting of 340 m2 situated in Bamban, Tarlac. In 1963, Punsalan mortgaged the land to PNB (Tarlac Branch) for P10,000.00, but for failure to pay said amount, the property was foreclosed on 16 December 1970. PNB (Tarlac Branch) was the highest bidder in said foreclosure proceedings. However, the bank secured title thereto only on 14 December 1977. In the meantime, in 1974, while the property was still in the alleged possession of Punsalan and with the alleged acquiescence of PNB (Tarlac Branch), and upon securing a permit from the Municipal Mayor, Punsalan constructed a warehouse on said property. Punsalan declared said warehouse for tax purposes for which he was issued Tax Declaration 5619. Punsalan then leased the warehouse to one Hermogenes Sibal for a period of 10 years starting January 1975. On 26 July 1978, a Deed of Sale was executed between PNB (Tarlac Branch) and Lacsamana over the property. This contract was amended on 31 July 1978, particularly to include in the sale, the building and improvement thereon. By virtue of said instruments, Lacsamana secured title over the property in her name (TCT 173744) as well as separate tax declarations for the land and building. On 22 November 1979, Punsalan commenced suit for "Annulment of Deed of Sale with Damages" against PNB and Lacsamana before the CFI Rizal, Branch XXXI, Quezon City, essentially impugning the validity of the sale of the building as embodied in the Amended Deed of Sale. The CFI dismissed the case on the ground of improper venue on 25 April 1980, finding that the warehouse allegedly owned and constructed by the plaintiff on the land of the PNB situated in the Municipality of Bamban, Province of Tarlac, which warehouse is an immovable property pursuant to Article 415 (1) of the New Civil Code; and, as such the action of the plaintiff is a real action affecting title to real property which, under Section 2, Rule 4 of the New Rules of Court, must be tried in the province where the property or any part thereof lies. Punsalan filed a Motion for Reconsideration of the Order, which the Court denied on 1 September 1980. Hence, the petition for Certiorari. The Supreme Court denied the petition without prejudice to the refilling of the case by Punsalan in the proper forum; with cost against the petitioner. 1. Buildings are always immovable under the Code Buildings are always immovable under the Code. A building treated separately from the land on which it stood is immovable property and the mere fact that the parties to a contract seem to have dealt with it separate and apart from the land on which it stood in no wise changed its character as immovable property. 2. Annulment or rescission of sale of real property does not operate to efface the objective of recovering real property Even if one does not directly seek the recovery of title or possession of the property, his action for annulment of sale and his claim for damages are closely intertwined with the issue of ownership of the building which, under the law, is considered immovable property, the recovery of which is petitioner's primary objective. The prevalent doctrine is that an action for the annulment or rescission of a sale of real property does not operate to efface the fundamental and prime objective and nature of the case, which is to recover said real property. It is a real action. 3. Lack of allegation of improper venue does not warrant case to proceed as it also require other indispensable party The contention that the case should proceed as the respondent failed to allege improper venue and, therefore, issues had already been joined, is untenable. An indispensable party exist besides the parties in the Amended Contract of Sale, the validity of which is being questioned. It would be futile to proceed with the case against one respondent alone.

Lopez v. Orosa [G.R. Nos. L-10817-18. February 28, 1958.] En Banc, Felix (J): 10 concur. Facts: Enrique Lopez is a resident of Balayan, Batangas, doing business as Lopez-Castelo Sawmill. Sometime in May 1946, Vicente Orosa, Jr. invited Lopez to make an investment in the theatre business (Plaza Theatre, Inc.). Although Lopez expressed his unwillingness to invest in the business, he agreed to supply the lumber necessary for the construction of the proposed theatre and at Orosa's behest and assurance that the latter would be personally liable for any account that the said construction might incur, Lopez further agreed that payment therefor would be on demand and not cash on delivery basis. Pursuant to said verbal agreement, Lopez delivered the lumber for the theater on 17 May 1946, up to 4 December of the same year. The Plaza Theatre was erected on a piece of land with an area of 679.17 m2 formerly owned by Vicente Orosa, Jr., and was acquired by the corporation on 25 September 1946. The total cost of the materials amounted to P62,255.85, of which Lopez was paid only P20,848.50, thus leaving a balance of P41,771.35. Orosa and Belarmino Rustia, corporation president, promised Lopez to obtain a bank loan to satisfy the balance, to which assurance Lopez had to accede. Unknown to him, however, as early as November 1946, the corporation already got a loan for P30,000 from the PNB with the Luzon Surety Company as surety, and the corporation in turn executed a mortgage on the land and building in favor of said company as counter-security. As the land at that time was not yet brought under the operation of the Torrens System, the mortgage on the same was registered on 16 November 1946, under Act 3344. Subsequently, when the corporation applied for the registration of the land under Act 496, such mortgage was not revealed and thus OCT O-391 was correspondingly issued on October 25, 1947, without any encumbrance appearing thereon. Vicente Orosa, Jr. executed, on 17 March 1947, an alleged "deed of assignment" of his 420 shares of stock of the Plaza Theater, Inc., at P100 per share or with a total value of P42,000 in favor of the creditor, and as the obligation still remained unsettled, Lopez filed on 12 November 1947, a complaint with the CFI Batangas (Civil Case 4501, later R-57) against Vicente Orosa Jr. and

Plaza Theatre, Inc., praying that defendants be sentenced to pay him jointly and severally the sum of P41,771.35 with legal interest from the filing of the action; that in case defendants fail to pay the same, that the building and the land covered by OCT O-391 owned by the corporation be sold at public auction and the proceeds thereof be applied to said indebtedness; or that the 420 shares of the capital stock of the Plaza Theatre, Inc., assigned by Vicente Orosa, Jr., to said plaintiff be sold at public auction for the same purpose; and for such other remedies as may be warranted by the circumstances. Plaintiff also caused the annotation of a notice of lis pendens on said properties with the Register of Deeds. The surety company, in the meantime, upon discovery that the land was already registered under the Torrens System and that there was a notice of lis pendens thereon, filed on 17 August 1948, or within the 1-year period after the issuance of the certificate of title, a petition for review of the decree of the land registration court dated 18 October 1947, in order to annotate the lights and interests of the surety company over said properties. Opposition thereto was offered by Lopez, asserting that the amount demanded by him constituted a preferred lien over the properties of the obligors; that the surety company was guilty of negligence when it failed to present an opposition to the application for registration of the property; and that if any annotation of the rights and interest of said surety would ever be made, same must be subject to the lien in his favor. The two cases were heard jointly and in a decision dated 30 October 1952, the lower Court held that Orosa and the Plaza Theatre, Inc., were jointly liable for the unpaid balance of the cost of lumber used in the construction of the building and the plaintiff thus acquired the materialman's lien over the same; the lien being merely confined to the building and did not extend to the land on which the construction was made. Plaintiff tried to secure a modification of the decision in so far as it declared that the obligation of therein defendants was joint instead of solidary and that the lien did not extend to the land, but same was denied by order of the court of 23 December 1952. The matter was thus appealed to the Court of Appeals, which affirmed the lower court's ruling, and then to the Supreme Court. The Supreme Court affirmed the decision appealed from, with costs against appellant. 1. Building is separate and distinct from land While it is true that generally, real estate connotes the land and the building constructed thereon, it is obvious that the inclusion of the building, separate and distinct from the land, in the enumeration of what may constitute real properties could mean only one thing that a building is by itself an immovable property (cf. Leung Yee v. Strong Machinery). In the absence of any specific provision of law to the contrary, a building is an immovable property, irrespective of whether or not said structure and the land on which it is adhered to belong to the same owner. 2. Article 1923 (5); Lien charged to property for which credit was made A close examination of Article 1923 (5) of the Civil Code reveals that the law gives preference to unregistered refectionary credits only with respect to the real estate upon which the refection or work was made. This being so, the inevitable conclusion must be that the lien so created attaches merely to the immovable property for the construction or repair of which the obligation was incurred. In the case at bar, the lien for the unpaid value of the lumber used in the construction of the building attaches only to said structure and to no other property of the obligors. Thus, the materialman's lien could be charged only to the building for which the credit was made or which received the benefit of refection, the interest of the mortgagee over the land is superior and cannot be made subject to the said materialman's lien. Prudential Bank v. Panis [G.R. No. L-50008. August 31, 1987.] First Division, Paras (J): 4 concur. Facts: On 19 November 1971, Fernando A. Magcale and Teodula Baluyut Magcale secured a loan of P70,000.00 from Prudential Bank. To secure payment of this loan, the Magcales executed in favor of Prudential Bank a deed of Real Estate Mortgage over a 2-storey, semi-concrete residential building with warehouse space (total area of 263 sq.m.); and granting upon the mortgagee the right of occupancy on the lot where the property is erected. A rider is also included in the deed that in the event the Sales Patent on the lot is issued of Bureau of Lands, the Register of Deeds is authorized to hold the Registration until the mortgage is cancelled or annotate the encumbrance on the title upon authority from the Secretary of Agriculture and Natural Resources, which title with annotation release in favor of the mortgage. The Real Estate Mortgage was registered under the Provisions of Act 3344 with the Registry of Deeds of Zambales on 23 November 1971. Subsequently, the Magcales secured an additional loan from Prudential Bank, secured by another deed of Real Estate Mortgage registeed with the Registry of Deeds in Olongapo City, on 2 May 1973. On 24 April 1973, the Secretary of Agriculture issued Miscellaneous Sales Patent 4776 over the parcel of land, possessory rights over which were mortgaged to rudential Bank, in favor of the Magcales. On the basis of the Patent, and upon its transcription in the Registration Book of the Province of Zambales, OCT P-2554 was issued in the name of Fernando Magcale, by the Ex-Oficio Register of Deeds of Zambales, on 15 May 1972. For failure of the Magcales to pay their obligation to the Bank after it became due, the deeds of Real Estate Mortgage were extrajudicially foreclosed. Consequent to the foreclosure was the sale of the properties mortgaged to the bank as the highest bidder in a public auction sale conducted by the City Sheriff on 12 April 1978. The auction sale was held despite written request from the Magcales through counsel, dated 29 March 1978, for the City Sheriff to desist from going with the scheduled public auction sale. The issue was raised to the CF Zambales and Olongapo City which, on 3 November 1978, declared the deeds of Real Estate Mortgage as null and void. The bank filed a motion for reconsideration on 14 December 1978, which the court denied on 10 January 1979 for lack of merit. Hence, the petition. The Supreme Court modified the decision of the CFI Zambales & Olongapo, declaring that the Deed of Real Estate Mortgage for P70,000.00 is valid but ruling that the Deed of Real Estate Mortgage for an additional loan of P20,000.00 is null and void, without prejudice to any appropriate action the Government may take against private respondents.

1. Building separate and distinct from the land In the enumeration of properties under Article 415 of the Civil Code of the Philippines, it is obvious that the inclusion of 'building' separate and distinct from the land, in said provision of law can only mean that a building is by itself an immovable property. (Lopez vs. Orosa, Jr., et al., L-10817-18, Feb. 28, 1958; Associated Inc. and Surety Co., Inc. vs. Iya, et al., L-10837-38, May 30, 1958). 2. Building can be mortgaged apart from the land it is built; possessory rights may be validly transferred in a deed of mortgage While a mortgage of land necessarily includes, in the absence of stipulation of the improvements thereon, buildings; still a building by itself may be mortgaged apart from the land on which it has been built. Such a mortgage would be still a real estate mortgage for the building would still be considered immovable property even if dealt with separately and apart from the land (Leung Yee vs. Strong Machinery Co., 37 Phil. 644). Possessory rights over said properties before title is vested on the grantee, may be validly transferred or conveyed as in a deed of mortgage (Vda. de Bautista vs. Marcos, 3 SCRA 438 [1961]). 3. A valid real estate mortgage may be constituted on the building erected on the land belonging to another The original mortgage was executed (19 November 1971) before the issuance of the final patent (24 April 1972) and before the government was divested of its title to the land (15 May 1972), an event which takes effect only on the issuance of the sales patent and its subsequent registration in the Office of the Register of Deeds (Visayan Realty Inc. vs. Meer, 96 Phil. 515; Director of Lands vs. De Leon, 110 Phil. 28; Director of Lands vs. Jurado, L-14702, May 23, 1961; Pea, "Law on Natural Resources", p. 49). In the case at bar, it is evident that the mortgage executed by Magcale on his own building which was erected on the land belonging to the government is to all intents and purposes a valid mortgage. 4. Public land act and RA 730 not violated in first mortgage As to restrictions appearing to the Magcales title; Sections 121, 122 and 124 of the Public Land Act refer to land already acquired under the Public Land Act or any improvement thereon. Section 2 of RA 730 refers to encumbrance or alienation before the patent is issued because it refers specifically to encumbrance or alienation on the land itself and does not mention anything regarding the improvements existing thereon. Both have no application to the assailed mortgage in the case at bar; as the former, the mortgage was executed before such eventuality, and the latter, it does not encumber nor alienate the land. 5. Mortgage made after issuance of Sales Patent an OCT prohibited; Estoppel does not give validating effect to a void contract As regards the second mortgage executed, such mortgage executed after the issuance of the sales patent and of the Original Certificate of Title, falls squarely under the prohibitions stated in Sections 121, 122 and 124 of the Public Land Act and Section 2 of RA 730, and is therefore null and void. Even if the title was voluntary surrendered to the bank for the mortgage to be annotated without the prior approval of the Ministry of Natural Resources; in pari delicto may not be invoked to defeat the policy of the State neither may the doctrine of estoppel give a validating effect to a void contract. Indeed, it is generally considered that as between parties to a contract, validity cannot be given to it by estoppel if it is prohibited by law or is against public policy (19 Am. Jur. 802). It is not within the competence of any citizen to barter away what public policy by law seeks to preserve (Gonzalo Puyat & Sons, Inc. vs. De los Amas and Alino, supra; Arsenal vs. IAC, 143 SCRA 54 [1986]). Such does not, however, preclude new contracts that may be entered into in accordance with the requirements of the law. Any new transaction, however, would be subject to whatever steps the Government may take for the reversion of the land in its favor. Leung Yee v. Strong Machinery [G.R. No. L-11658. February 15, 1918.] First Division, Carson (J): 5 concur, 3 took no part. Facts: The "Compaia Agricola Filipina" bought rice-cleaning machinery from the machinery company, and executed a chattel mortgage thereon to secure payment of the purchase price. It included in the mortgage deed the building of strong materials in which the machinery was installed, without any reference to the land on which it stood. The indebtedness secured by this instrument not having been paid when it fell due, the mortgaged property was sold by the sheriff, in pursuance of the terms of the mortgage instrument, and was bought in by the machinery company. The mortgage was registered in the chattel mortgage registry, and the sale of the property to the machinery company in satisfaction of the mortgage was annotated in the same registry on 29 December 1913. On 14 January 1914, the "Compaia Agricola Filipina" executed a deed of sale of the land upon which the building stood to the machinery company, but this deed of sale, although executed in a public document, was not registered and made no reference to the building erected on the land and would appear to have been executed for the purpose of curing any defects which might be found to exist in the machinery company's title to the building under the sheriff's certificate of sale. The machinery company went into possession of the building at or about the time when this sale took place, that is to say, the month of December 1913, and it has continued in possession ever since. At or about the time when the chattel mortgage was executed in favor of the machinery company, the "Compaia Agricola Filipina" executed another mortgage to Leung Yee upon the building, separate and apart from the land on which it stood, to secure payment of the balance of its indebtedness to Leung Yee under a contract for the construction of the building. Upon the failure of the mortgagor to pay the amount of the indebtedness secured by the mortgage, Leung Yee secured judgment for that amount, levied execution upon the building, bought it in at the sheriff's sale on or about the 18 December 1914, and had the sheriff's certificate of sale duly registered in the land registry of the Province of Cavite. At the time when the execution was levied upon the building, the machinery company, which was in possession, filed with the sheriff a sworn statement setting up its claim of title and demanding the release of the property from the levy. Thereafter, upon demand of the sheriff, Leung Yee executed an indemnity bond in favor of the sheriff in the sum of P12,000, in reliance upon which the sheriff sold the property at public auction to the plaintiff, who was the highest bidder at the sheriff's sale. The current action was instituted to recover possession of the building from the machinery company. The Court gave judgment in favor of the machinery company, relying upon Article 1473 and the fact that the company had its title to the building registered prior to the date of the registry of plaintiffs certificate. Hence the appeal.

The Supreme Court affirmed the judgment with costs against the appellant. 1. Building separate from land does not affect character as real property; Registry of chattel mortgage does not affect character of the building and the machineries installed therein The Chattel Mortgage Law contemplates and makes provision for mortgages of personal property; and the sole purpose and object of the chattel mortgage registry is to provide for the registry of "Chattel mortgages," mortgages of personal property executed in the manner and form prescribed in the statute. The building of strong materials in which the machinery was installed was real property, and the mere fact that the parties seem to have dealt with it separate and apart from the land on which it stood in no wise changed its character as real property. It follows that neither the original registry in the chattel mortgage registry of the instrument purporting to be a chattel mortgage of the building and the machinery installed therein, nor the annotation in that registry of the sale of the mortgaged property, had any effect whatever so far as the building was concerned. 2. Possession before sheriffs sale, not Article 1473 (on good faith), controlling as to ownership of property The ruling cannot be sustained on the ground of Article 1473, second paragraph, but on the ground that the agreed statement of facts discloses that neither the purchase of the building by plaintiff nor his inscription of the sheriff's certificate of sale in his favor was made in good faith, and that the machinery company must be held to be the owner of the property under the third paragraph of the above cited article of the code, it appearing that the company first took possession of the property; and further, that the building and the land were sold to the machinery company long prior to the date of the sheriff's sale to the plaintiff. 3. Good faith an essential requisite of inscription of property in registry, even if not mentioned unlike in possession and title; Construction should not defeat the purpose of law Even if Article 1473 of the Civil Code require "good faith," in express terms, in relation to "possession" and "title," but contain no express requirement as to "good faith" in relation to the "inscription" of the property in the registry, it remains an essential requisite of registration as it could not have been the intention of the legislator to base the preferential right secured this article of the code upon an inscription of title in bad faith. Such an interpretation placed upon the language of this section would open wide the door to fraud and collusion. The public records cannot be converted into instruments of fraud and oppression by one who secures an inscription therein in bad faith. The force and effect given by law to an inscription in a public record presupposes the good faith of him who enters such inscription; and rights created by statute, which are predicated upon an inscription in a public registry, do not and cannot accrue under an inscription "in bad faith," to the benefit of the person who thus makes the inscription. 4. Construction of Article 1473 as to issue of good faith It is always to be understood on the basis of the good faith mentioned in the first paragraph; therefore, it having been found that the second purchasers who record their purchase had knowledge of the precious sale, the question is to be decided in accordance with the following paragraph. Although article 1473, in its second paragraph, provides that the title of conveyance of ownership of the real property that is first recorded in the registry shall have preference, this provision must always be understood on the basis of the good faith mentioned in the first paragraph; the legislator could not have wished to strike it out and to sanction bad faith, just to comply with a mere formality which, in given cases, does not obtain even in real disputes between third persons. 5. Bad faith: One cannot claim acquisition of title in good faith if knowledgeable of defect or lack of title One who purchases real estate with knowledge of a defect or lack of title in his vendor cannot claim that he has acquired title thereto in good faith as against the true owner of the land or of an interest therein; and the same rule must be applied to one who has knowledge of facts which should have put him upon such inquiry and investigation as might be necessary to acquaint him with the defects in the title of his vendor. A purchaser cannot close his eyes to facts which should put a reasonable man upon his guard, and then claim that he acted in good faith under the belief that there was no defect in the title of the vendor. His mere refusal to believe that such defect exists, or his willful closing of his eyes to the possibility of the existence of a defect in his vendor's title, will not make him an innocent purchaser for value, if it afterwards develops that the title was in fact defective, and it appears that he had such notice of the defect as would have led to its discovery had he acted with that measure of precaution which may reasonably be required of a prudent man in a like situation. 6. Test of good faith Good faith, or the lack of it, is in its last analysis a question of intention; but in ascertaining the intention by which one is actuated on a given occasion, the Court is necessarily controlled by the evidence as to the conduct and outward acts by which alone the inward motive may, with safety, be determined. So it is that "the honesty of intention," "the honest lawful intent," which constitutes good faith implies a "freedom from knowledge and circumstances which ought to put a person on inquiry," and so it is that proof of such knowledge overcomes the presumption of good faith in which the courts always indulge in the absence of proof to the contrary. "Good faith, or the want of it, is not a visible, tangible fact that can be seen or touched, but rather a state or condition of mind which can only be judged of by actual or fancied tokens or signs." (Wilder vs. Gilman, 55 Vt., 504, 505; Cf. Cardenas vs. Miller, 108 Cal., 250; Breaux-Renoudet, Cypress Lumber Co. vs. Shadel, 52 La. Ann., 2094-2098; Pinkerton Bros. Co. vs. Bromley, 119 Mich., 8, 10, 17.) Bicerra v. Teneza [G.R. No. L-16218. November 29, 1962.] En Banc, Makalintal (J): 10 concur. Facts: The Bicerras are supposedly the owners of the house (PhP 20,000) built on a lot owned by them in Lagangilang, Abra; which the Tenezas forcibly demolished in January 1957, claiming to the owners thereof. The materials of the house were placed in the custody of the barrio lieutenant. The Bicerras filed a complaint claiming actual damages of P200, moral and consequential damages amounting to P600, and the costs. The CFI Abra dismissed the complaint claiming that the action was within the exclusive (original) jurisdiction of the Justice of the Peace Court of Lagangilang, Abra. The Supreme Court affirmed the order appealed. Having been admitted in forma pauperis, no costs were adjudged.

1. House is immovable property even if situated on land belonging to a different owner; Exception, when demolished A house is classified as immovable property by reason of its adherence to the soil on which it is built (Article 415, paragraph 1, Civil Code). This classification holds true regardless of the fact that the house may be situated on land belonging to a different owner. But once the house is demolished, as in this case, it ceases to exist as such and hence its character as an immovable likewise ceases. 2. Recovery of damages not exceeding P2,000 and involving no real property belong to the Justice of the Peace Court The complaint is for recovery of damages, the only positive relief prayed for. Further, a declaration of being the owners of the dismantled house and/or of the materials in no wise constitutes the relief itself which if granted by final judgment could be enforceable by execution, but is only incidental to the real cause of action to recover damages. As this is a case for recovery of damages where the demand does not exceed PhP 2,000 and that there is no real property litigated as the house has ceased to exist, the case is within the jurisdiction of the Justice of the Peace Court (as per Section 88, RA 296 as amended) and not the CFI (Section 44, id.) Evangelista v. Alto Surety [G.R. No. L-11139. April 23, 1958.] En Banc, Concepcion (J): 9 concur Facts: On 4 June 1949, Santos Evangelista instituted Civil Case No. 8235 of the CFI Manila (Santos Evangelista vs. Ricardo Rivera) for a sum of money. On the same date, he obtained a writ of attachment, which was levied upon a house, built by Rivera on a land situated in Manila and leased to him, by filing copy of said writ and the corresponding notice of attachment with the Office of the Register of Deeds of Manila, on 8 June 1949. In due course, judgment was rendered in favor of Evangelista, who, on 8 October 1951, bought the house at public auction held in compliance with the writ of execution issued in said case. The corresponding definite deed of sale was issued to him on 22 October 1952, upon expiration of the period of redemption. When Evangelista sought to take possession of the house, Rivera refused to surrender it, upon the ground that he had leased the property from the Alto Surety & Insurance Co., Inc. and that the latter is now the true owner of said property. It appears that on 10 May 1952, a definite deed of sale of the same house had been issued to Alto Surety, as the highest bidder at an auction sale held, on 29 September 1950, in compliance with a writ of execution issued in Civil Case 6268 of the same court (Alto Surety & Insurance vs. Maximo Quiambao, Rosario Guevara and Ricardo Rivera)" in which judgment, for the sum of money, had been rendered in favor of Alto Surety. Hence, on 13 June 1953, Evangelista instituted an action against Alto Surety and Ricardo Rivera, for the purpose of establishing his title over said house, and securing possession thereof, apart from recovering damages. After due trial, the CFI Manila rendered judgment for Evangelista, sentencing Rivera and Alto Surety to deliver the house in question to Evangelista and to pay him, jointly and severally, P40.00 a month from October, 1952, until said delivery, plus costs. On appeal, the decision was reversed by the Court of Appeals, which absolved Alto Surety from the complaint, upon the ground that, although the writ of attachment in favor of Evangelista had been filed with the Register of Deeds of Manila prior to the sale in favor of Alto Surety, Evangelista did not acquire thereby a preferential lien, the attachment having been levied as if the house in question were immovable property, although, in the opinion of the Court of Appeals, it is "ostensibly a personal property." As such, the Court of Appeals held, "the order of attachment . . . should have been served in the manner provided in subsection (e) of section 7 of Rule 59," of the Rules of Court. Evangelista filed an appeal by Certiorari with the Supreme Court. The Supreme Court reversed the decision of the Court of Appeals, and another one entered affirming that of the CFI Manila, with the costs against Alto Surety & Insurance Co. 1. House is not personal, but immovable property The house is not personal property, much less a debt, credit or other personal property not capable of manual delivery, but immovable property. As explicitly held, in Laddera vs. Hodges (48 OG 5374), "a true building (not merely superimposed on the soil) is immovable or real property, whether it is erected by the owner of the land or by a usufructuary or lessee. This is the doctrine in Leung Yee vs. Strong Machinery Company, 37 Phil., 644. The opinion that the house of Rivera should have been attached in accordance with subsection (c) of said section 7, as "personal property capable of manual delivery, by taking and safely keeping in his custody", for it declared that "Evangelista could not have validly purchased Ricardo Rivera's house from the sheriff as the latter was not in possession thereof at the time he sold it at a public auction is untenable. 2. House may be considered personal property in a deed of chattel mortgage, but view is limited to parties Parties to a deed of chattel mortgage may agree to consider a house as personal property for purposes of said contract (Luna vs. Encarnacion, 48 OOG 2664; Standard Oil Co. of New York vs. Jaramillo, 44 Phil., 630; De Jesus vs. Juan Dee Co., Inc., 72 Phil., 464). However, this view is good only insofar as the contracting parties are concerned. It is based, partly, upon the principle of estoppel. Neither this principle, nor said view, is applicable to strangers to said contract. 3. House is not personal property even if subject to chattel mortgage; Rules of execution strict on the character of the properties as real and personal notwithstanding agreement of parties The rules on execution do not allow, and should not be interpreted as to allow, the special consideration that parties to a contract may have desired to impart to real estate as personal property, when they are not ordinarily so. Sales on execution affect the public and third persons. The regulation governing sales on execution are for public officials to follow. The form of proceedings prescribed for each kind of property is suited to its character, not to the character which the parties have given to it or desire to give it. The regulations were never intended to suit the consideration that parties, may have privately given to the property levied upon. Enforcement of regulations would be difficult were the convenience or agreement of private parties to determine or govern the nature of the proceedings (Manarang v. Ofilada) 4. House of mixed material levied upon on execution is real property The house of mixed materials levied upon on execution, although subject of a contract of chattel mortgage between the owner and a third person, is real property within the purview of Rule 39, section 16, of the Rules of Court as it has become a permanent fixture on the land, which is real property. (Manarang v. Ofilada, citing various sources).

5. Foregoing consideration apply to conditions for levy of attachment The foregoing considerations apply, with equal force, to the conditions for the levy of attachment, for it similarly affects the public and third persons. 6. Evidence; Conclusion, that lack of proof that the writ or notice of attachment to have been served leads to such not having been served, is inaccurate Considering that neither the pleadings, nor the briefs in the Court of Appeals, raised an issue on whether or not copies of the writ of attachment and notice of attachment had been served upon Rivera; that the defendants had impliedly admitted that Rivera had received copies of said documents; and that, for this reason, evidently, no proof was introduced thereon, the Court held that the finding of the Court of Appeals to the effect that said copies had not been served upon Rivera is based upon a misapprehension of the specific issues involved therein and goes beyond the range of such issues, apart from being contrary to the admission by the parties, and that, accordingly, a grave abuse of discretion was committed in making said finding, which is, furthermore, inaccurate. Associated Insurance & Surety v. Iya [G.R. Nos. L-10837-38. May 30, 1958.] En Banc, Felix (J): 9 concur Facts: Spouses Adriano and Lucia A. Valino were the owners and possessors of a house of strong materials constructed on Lot 3, Block 80 of the Grace Park Subdivision in Caloocan, Rizal, which they purchased on installment basis from the Philippine Realty Corporation. On 6 November 1951, to enable her to purchase on credit rice from the NARIC, Lucia Valino filed a bond (P11,000.00; AISCO Bond 971) subscribed by the Associated Insurance & Surety Co. and as counter-guaranty therefor, the Valinos executed an alleged chattel mortgage on the aforementioned house in favor of the surety company, which encumbrance was duly registered with the Chattel Mortgage Register of Rizal on 6 December 1951. It is admitted that at the time said undertaking took place, the parcel of land on which the house is erected was still registered in the name of the Philippine Realty Corporation. Having completed payment on the purchase price of the lot, the Valinos were able to secure on 18 October 1958, a certificate of title in their name (TCT 27884). Subsequently, however, or on 24 October 1952, the Valinos, to secure payment of an indebtedness in the amount of P12,000.00, executed a real estate mortgage over the lot and the house in favor of Isabel Iya, which was duly registered and annotated at the back of the certificate of title. Later, Lucia A. Valino failed to satisfy her obligation to the NARIC, the surety company was compelled to pay the same pursuant to the undertaking of the bond. The surety company demanded reimbursement from the Valinos, who failed to do so. The company foreclosed the chattel mortgage over the house as a consequence. A public sale was conducted thereafter by the Provincial Sheriff of Rizal on 26 December 1952, wherein the property was awarded to the surety company for P8,000.00, the highest bid received therefor. The surety company then caused the said house to be declared in its name for tax purposes (Tax Declaration 25128). Sometime in July 1953, the surety company learned of the existence of the real estate mortgage over the lot covered by TTC 26884 together with the improvements thereon; thus, said surety company instituted Civil Case 2162 with the CFI Manila naming Adriano and Lucia Valino and Isabel Iya, the mortgagee, as defendants. On the other hand, on 29 October 1953, Isabel Iya filed a civil action against the Valinos and the surety company (Civil Case 2504 with CFI Manila) praying for a decree of foreclosure of the land, building and improvements thereon to be sold at public auction and the proceeds applied to satisfy the demands; this pursuant to the contract of mortgage as the Valinos have failed to pay interest for more than 6 months already; the surety company included as it claims to have an interest on the residential house covered by said mortgage. The two cases were jointly heard upon agreement of the parties, who submitted the same on a stipulation of facts, after which the Court rendered judgment dated 8 March 1956, holding that the chattel mortgage in favor of the Associated Insurance & Surety was preferred and superior over the real estate mortgage subsequently executed in favor of Isabel Iya. It was ruled that as the Valinos were not yet the registered owner of the land on which the building in question was constructed at the time the first encumbrance was made, the building then was still a personalty and a chattel mortgage over the same was proper. However, as the mortgagors were already the owners of the lot at the time the contract with Isabel Iya was entered into, the building was transformed into a real property and the real estate mortgage created thereon was likewise adjudged as proper. The residential building was, therefore, ordered excluded from the foreclosure prayed for by Isabel Iya, although the latter could exercise the right of a junior encumbrancer. The spouses Valino were ordered to pay the amount demanded by said mortgagee or in their default to have the parcel of land subject of the mortgage sold at public auction for the satisfaction of Iya's claim. The Supreme Court reversed the decision of the lower court, recognized Isabel Iya's right to foreclose not only the land but also the building erected thereon, and ordered that the proceeds of the sale thereof at public auction (if the land has not yet been sold), be applied to the unsatisfied judgment in favor of Isabel Iya. The decision however is without prejudice to any right that the Associated Insurance & Surety may have against the Valinos on account of the mortgage of said building they executed in favor of said surety company. Without pronouncement as to costs. 1. Nature of property encumbered is the decisive factor in determination of preferential right The decisive factor in resolving the issue as to which of these encumbrances should receive preference over the other is the determination of the nature of the structure litigated upon, for where it be considered a personalty, the foreclosure of the chattel mortgage and the subsequent sale thereof at public auction, made in accordance with the Chattel Mortgage Law would be valid and the right acquired by the surety company therefrom would certainly deserve prior recognition; otherwise, appellant's claim for preference must be granted. 2. Building always immovable While it is true that generally, real estate connotes the land and the building constructed thereon, it is obvious that the inclusion of the building, separate and distinct from the land, in the enumeration of what may constitute real properties (Art. 415, new Civil Code) could only mean one thing: that a building is by itself an immovable property. Moreover, and in view of the absence of any specific provision to the contrary, a building is an immovable property irrespective of whether or not said structure and the land on which it is adhered to belong to the same owner." (Lopez vs. Orosa).

3. Building cannot be divested of character as realty when constructed on land belonging to another A building certainly cannot be divested of its character of a realty by the fact that the land on which it is constructed belongs to another. To hold it the other way, the possibility is not remote that it would result in confusion, for to cloak the building with an uncertain status made dependent on the ownership of the land, would create a situation where a permanent fixture changes its nature or character as the ownership of the land changes hands. 4. Execution of a chattel mortgage over a building invalid and a nullity As personal properties could only be the subject of a chattel mortgage (Section 1, Act 3952), the execution of the chattel mortgage covering a building is clearly invalid and a nullity. While it is true that said document was correspondingly registered in the Chattel Mortgage Register, this act produced no effect whatsoever for where the interest conveyed is in the nature of a real property, the registration of the document in the registry of chattels is merely a futile act. Thus, the registration of the chattel mortgage of a building of strong materials produce no effect as far as the building is concerned (Leung Yee vs. Strong Machinery Co., 37 Phil., 644). 5. No right acquired by chattel mortgage creditor who purchases real properties in an extrajudicial foreclosure sale A mortgage creditor who purchases real properties at an extrajudicial foreclosure sale thereof by virtue of a chattel mortgage constituted in his favor, which mortgage has been declared null and void with respect to said real properties, acquires no right thereto by virtue of said sale (De la Riva vs. Ah Keo, 60 Phil., 899). Navarro v. Pineda [G.R. No. L-18456. November 30, 1963.] En Banc, Paredes (J): 8 concur Facts: On 14 December 1959, Rufino G. Pineda and his mother Juana Gonzales (married to Gregorio Pineda), borrowed from Conrado P. Navarro, the sum of P2,550.00, payable 6 months after said date or on 14 June 1959. To secure the indebtedness, Rufino executed a document captioned "Deed of Real Estate and Chattel Mortgages ", whereby Juana Gonzales, by way of Real Estate Mortgage hypothecated a parcel of land, belonging to her, registered with, the register of Deeds of Tarlac, under TCT 25776, and Rufino G. Pineda, by way of Chattel Mortgage, mortgaged his 2-story residential house, having a floor area of 912 sq.m., erected on a lot belonging to Atty. Vicente Castro, located at San Roque, Tarlac, Tarlac; and 1 motor truck, registered in his name, under Motor Vehicle Registration Certificate A-171805. Both mortgages were contained in one instrument, which was registered in both the Office of the Register of Deeds and the Motor Vehicles Office of Tarlac. The Pinedas failed to pay the mortgage debt when it became due. They were granted an extension up to 30 June 1960, but they likewise failed to pay on the said day. They were granted another extension up to 30 July 1960, but they likewise failed and refused to pay. On 10 August 1960, Navarro filed a complaint for foreclosure of the mortgage and for damages, which consisted of liquidated damages in the sum of P500.00 and 12% per annum interest on the principal, effective on the date of maturity, until fully paid. On 24 February 1961, the lower court dismissed the complaint with regards to Gregorio Pineda; ordering Juana Gonzales and spouses Rufino Pineda and Ramona Reyes to pay Conrado Navarro the sum of P2,550 with 12% compounded interest plus P500 as liquidated damages and the cost of the suit from 14 June 1960 within 90 days from receipt of the copy of the decision, else the properties mentioned in the deed of real estate and chattel mortgage be sold to realize said mortgage debt in accordance with the pertinent provisions of Act 3135 and Article 14 of Act 1508; and ordering Rufino Pineda and Ramona Reyes to deliver the personal properties to the Provincial Sheriff of Tarlac immediately after the lapse of 90 days in default of such payment. The judgment was appealed directly to the Supreme Court, questioning the lower courts decision in holding the deed of real estate and chattel mortgages appended to the complaint valid, notwithstanding that the house of Rufino Pineda was made subject of the chattel mortgage for the reason that it is erected on a land that belongs to a third person. The Supreme Court affirmed the decision appealed from, with costs against appellants. 1. Building is immovable property Article 415 of the New Civil Code, in classifying a house as immovable property, makes no distinctions whether the owner of the land is or is not the owner of the building; the fact that the land belongs to another is immaterial, it is enough that the house adheres to the land; that in case of immovables by incorporation, such as houses, trees, plants, etc; the Code does not require that the attachment or incorporation be made by the owner of the land, the only criterion being the union or incorporation with the soil. A building is an immovable property, irrespective of whether or not said structure and the land on which it is adhered to, belong to the same owner (Lopez vs. Orosa, Leung Yee vs. Strong Machinery Co.). 2. Lower Courts Decision predicated on the doctrine of estoppel and not only on the ground that the house mortgaged was erected on the land which belonged to a third person The trial court did not predicate its decision declaring the deed of chattel mortgage valid solely on the ground that the house mortgaged was erected on the land which belonged to a third person, but also and principally on the doctrine of estoppel, in that "the parties have so expressly agreed" in the mortgage to consider the house as a chattel "for its smallness and mixed materials of sawali and wood". In the present case, Rufino Pineda grouped the house with the truck which is inherently a movable property. The house which was not even declared for taxation purposes was small and made of light construction materials: G.I. sheets roofing, sawali and wooden walls and wooden posts; built on land belonging to another. Clearly, the house in question was treated as personal or movable property, by the parties to the contract themselves. 3. Property may have a character different from that imputed to it by parties in a contract, but such property remains in its actual character as regards third persons who are not parties in the contract; Summary of relevant cases In construing Arts. 334 and 335 of the Spanish Civil Code (corresponding to Arts. 415 and 416, N.C.C.), for purposes of the application of the Chattel Mortgage Law, it was held that under certain conditions, a property may have a character different from that imputed to it in said articles. It is undeniable that the parties to a contract may by agreement, treat as personal property that

which by nature would be real property (Standard Oil vs. Jaranillo). "There can not be any question that a building of mixed materials may be the subject of a chattel mortgage, in which case, it is considered as between the parties as personal property. The matter depends on the circumstances and the intention of the parties. Personal property may retain its character as such where it is so agreed by the parties interested even though annexed to the realty". (42 Am. Jur. 209-210, cited in Manarang, et al. vs. Ofilada, et al.). The view that parties to a deed of chattel mortgage may agree to consider a house as personal property for the purposes of said contract, is good only insofar as the contracting parties are concerned. It is based, partly, upon the principles of estoppel (Evangelista vs. Alto Surety). In a case, a mortgaged house built on a rented land, was held to be a personal property not only because the deed of mortgage considered. it as such, but also because it did not form an integral part of the land (Evangelista vs. Abad, [CA]), for it is now well settled that an object placed on land by one who has only a temporary right to the same, such as a lessee or usufructuary, does not become immobilized by attachment (Valdez vs. Central Altagracia, cited in Davao Sawmill vs. Castillo). Hence, if a house belonging to a person stands on a rented land belonging to another person, it may be mortgaged as a personal property if so stipulated in the document of mortgage. It should be noted, however, that the principle is predicated on statements by the owner declaring his house to be a chattel, a conduct that may conceivably estop him from subsequent claiming otherwise (Ladera, et al. vs. C.W. Hodges, et al., [CA]). The doctrine, therefore, gathered from these cases is that although in some instances, a house of mixed materials has been considered as a chattel between the parties and that the validity of the contract between them, has been recognized, it has been a constant criterion nevertheless that, with respect to third persons, who are not parties to the contract, and specially in execution proceedings, the house is considered as an immovable property (Art. 1431, New Civil Code). 4. Cases cited are not applicable The cases cited by appellants are not applicable to the present case. The Iya cases refer to a building or a house of strong materials, permanently adhered to the land, belonging to the owner of the house himself. In the case of Lopez vs. Orosa, the subject building was a theater, built of materials worth more than P62,000.00 attached permanently to the soil. In these two cases and in the Leung Yee Case, third persons assailed the validity of the deed of chattel mortgages; in the present case, it was one of the parties to the contract of mortgages who assailed its validity. Standard Oil v. Jaramillo [G.R. No. 20329. March 16, 1923.] First Division, Street (J): 6 concur Facts: On 27 November 1922, Gervasia de la Rosa Vda. de Vera was the lessee of a parcel of land situated in the City of Manila and owner of the house of strong materials built thereon, upon which date she executed a document in the form of a chattel mortgage, purporting to convey to Standard Oil Company of New York by way of mortgage both the leasehold interest in said lot and the building which stands thereon. After said document had been duly acknowledged and delivered, Standard Oil caused the same to be presented to Joaquin Jaramillo, as register of deeds of the City of Manila, for the purpose of having the same recorded in the book of record of chattel mortgages. Upon examination of the instrument, Jaramillo opined that it was not chattel mortgage, for the reason that the interest therein mortgaged did not appear to be personal property, within the meaning of the Chattel Mortgage Law, and registration was refused on this ground only. The cause was brought to the Supreme Court upon demurrer interposed by Joaquin Jaramillo, register of deeds of the City of Manila, to an original petition of the Standard Oil Company of New York, seeking a peremptory mandamus to compel the respondent to record in the proper register a document purporting to be a chattel mortgage executed in the City of Manila by Gervasia de la Rosa, Vda. de Vera, in favor of the Standard Oil Company of New York. The Supreme Court overruled the demurrer, and ordered that unless Jaramillo interposes a sufficient answer to the petition for mandamus by Standard Oil within 5 days of notification, the writ would be issued as prayed, but without costs. 1. Jaramillo, register of deeds, does not have judicial or quasi-judicial power to determine nature of document registered as chattel mortgage Section 198 of the Administrative Code, originally of Section 15 of the Chattel Mortgage Law (Act 1508 as amended by Act 2496), does not confer upon the register of deeds any authority whatever in respect to the "qualification," as the term is used in Spanish law, of chattel mortgages. His duties in respect to such instruments are ministerial only. The efficacy of the act of recording a chattel mortgage consists in the fact that it operates as constructive notice of the existence of the contract, and the legal effects of the contract must be discovered in the instrument itself in relation with the fact of notice. Registration adds nothing to the instrument, considered as a source of title, and affects nobody's rights except as a species of notice. Thus, it is duty for the register of deed to accept the proper fee and place the instrument on record, as his duties in respect to the registration of chattel mortgages are of a purely ministerial character; and no provision of law can be cited which confers upon him any judicial or quasi-judicial power to determine the nature of any document of which registration is sought as a chattel mortgage. It may be noted that in an administrative ruling by James Ostrand, Judge of the fourth branch of CFI Manila (9th Judicial District) and later Supreme Court Justice, provided the same position that the Register of Deeds has no authority to pass upon the capacity of the parties to a chattel mortgage which is presented to him for record. The issue where the chattel mortgage is held ineffective against third parties as the mortgaged property is real instead of personal is a question determine by the courts of justice and mot by the register of deeds. 2. Article 334 and 335 of the Civil Code does not supply absolute criterion on distinction between real and personal property for purpose of the application of the Chattel Mortgage Law Article 334 and 335 of the Civil Code supply no absolute criterion for discriminating between real property and personal property for purposes of the application of the Chattel Mortgage Law. Those articles state rules which, considered as a general doctrine, are law in this jurisdiction; but it must not be forgotten that under given conditions property may have character different from that imputed to it in said articles. It is undeniable that the parties to a contract may be agreement treat as personal property that which by nature would be real property; and it is a familiar phenomenon to see things classed as real property for purposes of taxation which on general principle might be considered personal property. Other situations are constantly arising, and from time to time

are presented to the Supreme Court, in which the proper classification of one thing or another as real or personal property may be said to be doubtful. 3. Issue whether interest is in nature of real property not relevant to the issue of placing the document on record in Chattel Mortgage In Leung Yee vs. Frank L. Strong Machinery Co. and Williamson (37 Phil., 644), the Supreme Court held that where the interest conveyed is of the nature of real property, the placing of the document on record in the chattel mortgage register is a futile act. That decision is not decisive of the question before the Supreme Court, which has reference to the function of the register of deeds in placing the document on record. Manarang v. Ofilada [G.R. No. L-8133. May 18, 1956.] En Banc, Labrador (J): 10 concur Facts: On 8 September 951, Lucia D. Manarang obtained a loan of P200 from Ernesto Esteban, and to secure its payment she executed a chattel mortgage over a house of mixed materials erected on a lot on Alvarado Street, Manila. As Manarang did not pay the loan as agreed upon, Esteban brought an action against her in the municipal court of Manila for its recovery, alleging that the loan was secured by a chattel mortgage on her property. Judgment having been entered in Esteban's favor, execution was issued against the same property mortgaged. Before the property could be sold Manarang offered to pay the sum of P277 (amount of the judgment of P250, the interest thereon, the costs, and the sheriff's fees), but the sheriff refused the tender unless the additional amount of P260 representing the publication of the notice of sale in two newspapers be paid also. So the Manarangs brought the suit to compel the sheriff to accept the amount of P277 as full payment of the judgment and to annul the published notice of sale. It is to be noted that in the complaint filed in the municipal court, a copy of the chattel mortgage is attached and mention made of its registration, and in the prayer request is made that the house mortgaged be sold at public auction to satisfy the debt. It is also important to note that the house mortgaged was levied upon at Manarang's request. The case facts do not mention circumstances of the case as decided in the municipal court to elevate the case to the Court of First Instance The Court of First Instance denied the petition, holding that although real property may sometimes be considered as personal property, the sheriff was in duty bound to cause the publication of the notice of its sale in order to make the sale valid or to prevent its being declared void or voidable. The Supreme Court affirmed the judgment appealed, with costs. 1. Building is real property, general rule; exceptions The general principle of law is that a building permanently fixed to the freehold becomes a part of it, that prima facie a house is real estate, belonging to the owner of the land on which it stands, even though it was erected against the will of the landowner, or without his consent. The general rule does not apply (1) where the improvement is made with the consent of the landowner, and pursuant to an understanding either expressed or implied that it shall remain personal property, or (2) to a building which is wrongfully removed from the land and placed on the land of the person removing it. 2. Criterion to determine property as personal or real Among the principal criteria for determining whether property remains personally or becomes realty are annexation to the soil, either actual or construction, and the intention of the parties. 3. Personal property may retain character by agreement Personal property may retain its character as such where it is so agreed by the parties interested even though annexed to the realty, or where it is affixed in the soil to be used for a particular purpose for a short period and then removed as soon as it has served its purpose. 4. Building of mixed materials may be subject of a chattel mortgage A building of mixed materials may be the subject of a chattel mortgage, in which case it is considered as between the parties as personal property. (see Luna vs. Encarnacion 48 OG No. 7, p. 2664; Standard Oil vs. Jaranillo, 44 Phil., 630; and De Jesus vs. Guan Dee Co. 72 Phil., 464). The matter depends on the circumstances and the intention of the parties. 5. Real and personal property are real and personal property as they mean ordinarily under the Rules of Execution The rules on execution do not allow, and should not be interpreted as to allow, the special consideration that parties to a contract may have desired to impart to real estate as personal property, when they are not ordinarily so. Sales on execution affect the public and third persons. The regulation governing sales on execution are for public officials to follow. The form of proceedings prescribed for each kind of property is suited to its character, not to the character which the parties have given to it or desire to give it. The regulations were never intended to suit the consideration that parties, may have privately given to the property levied upon. Enforcement of regulations would be difficult were the convenience or agreement of private parties to determine or govern the nature of the proceedings. 6. House of mixed material levied upon on execution is real property The house of mixed materials levied upon on execution, although subject of a contract of chattel mortgage between the owner and a third person, is real property within the purview of Rule 39, section 16, of the Rules of Court as it has become a permanent fixture on the land, which is real property. (42 Am. Jur. 199-200; Leung Yee vs. Strong Machinery Co., 37 Phil., 644; Republic vs. Ceniza, et al., 90 Phil., 544; Ladera, et al. vs. Hodges [C. A], 48 OG 5374.).

Piansay v. David [G.R. No. L-19468. October 30, 1964.] En Banc, Cocnepcion (J): 10 concur Facts: On 11 December 1943, Conrado S. David received a loan of P3,000 with interest at 12% per annum from Claudia B. Vda. de Uy Kim, and to secure the payment of the same, David executed a chattel mortgage on a house situated at 1259 Sande Street, Tondo, Manila. The chattel mortgage was registered with the Register of Deeds of Manila on 19 December 1948. On 10 February 1953, the mortgaged house was sold at public auction to satisfy the indebtedness to Claudia B. Vda. de Uy Kim, and the house was sold to Claudia B. Vda. de Uy Kim in the said foreclosure proceedings. On 22 March 1954, Vda. de Uy Kim sold the same house to Salvador Piansay for P5,000.00. On 22 November 1949, David mortgaged the said house to Marcos Mangubat. On 1 March 1956, Mangubat filed a complaint against David with the CFI Manila (Civil Case 29078), for the collection of the loan of P2,000. On 24 March 1956, the complaint was amended to include Piansay and Vda. de Uy Kim as party defendants and praying that auction sale of 10 February 1953 and the deed of absolute sale between Vda. De Uy Kim and Piansay be annulled. The Court ordered David to pay Mangubat the sum of P2,000, damages and attorneys fees; but dismissed the case against Uy Kim and Piansay. The case was appealed to the Court of Appeals, which affirmed the decision but setting aside the award of damages in favor of Uy Kim. In the execution of Civil Case 29078, which was affirmed by the Court of Appeals (CA-GR 21797-R), the house which had been bought by Uy Kim at the foreclosure proceedings and sold by her to Piansay was levied upon at the instance of Mangubat. To prevent the sale at public auction of the house in question, Piansay and Uy Kim filed a petition for certiorari and mandamus with preliminary injunction in the CA (CA-GR 28974-R, Claudia B. Vda. de Uy Kim and Salvador Piansay vs. Hon. Judge Jesus Y. Perez, et al.). Acting upon the said petition, the CA denied the petition to lift or discharge the writ of execution on 28 April 1961. Thereupon, or on 31 July 1961, Piansay and Uy Kim instituted the present action at the CFI Manila (Civil Case 47664), against David and Mangubat. After due hearing the lower court issued the order appealed from granting said motion and dismissing the complaint, with costs against the Piansay and Uy Kim. A reconsideration of said order having been denied, Piansay and Uy Kim interposed an appeal directly to the Supreme Court. The Supreme Court affirmed the orders affirmed from, with costs against Piansay and Vda. De Uy Kim. 1. Building and construction partake the nature of the principal thing, i.e. land Since it is a rule in the law that buildings and constructions are regarded as mere accessories to the land (following the Roman maxim omne quod solo inaedificatur solo credit) it is logical that said accessories should partake of the nature of the principal thing, which is the land, forming, as they do, but a single object (res) with it in contemplation of law (Ladera v. Hodges). 2. Act of registering in Chattel Mortgage register of an interest in the nature of real property is futile Tthe registration of the document in the registry of chattels is merely a futile act, where the interest conveyed is in the nature of real property. The registration of the chattel mortgage of a building of strong materials produced no effect as far as the building is concerned (Leung Yee vs. Strong Machinery Co., cited in Ladera v. Hodges). 3. No right acquired by chattel Mortgage creditor who purchase real property at extra-judicial foreclosure sale A mortgage creditor who purchases real properties at an extra- judicial foreclosure sale thereof by virtue of a chattel mortgage constituted in his favor, which mortgage has been declared null and void with respect to said real properties, acquires no right thereto by virtue of said sale'. (De la Riva vs. Ah Kee, cited in Ladera v. Hodges). In the present case, Uy Kim had no right to foreclose the alleged chattel mortgage constituted in her favor, because it was in reality a mere contract of an unsecured loan. It follows that the Sheriff was not authorized to sell the house as a result of the foreclosure of such chattel mortgage. And Uy Kim could not have acquired the house at public auction, she could not have sold it validly to Salvador Piansay. 4. House may be levied upon as it remained the property of mortgage debtor The mere fact that the dispositive part of the decision in Civil Case 29078 states that the complaint is dismissed with respect to defendants Claudia B. de Uy Kim, Leonardo Uy Kim and Salvador Piansay is of no moment because the chattel mortgage executed by David in favor of Claudia B. de Uy Kim might not be annulled but still it did not transmit any right from David to Uy Kim. Piansay in Civil Case 47664 cannot assail the right of Mangubat to levy execution upon the house in question because it had remained the property of defendant David. 5. Chattel mortgage of a house between parties is valid, but does not bind third persons; Plaintiffs have no cause of action Regardless of the validity of a contract constituting a chattel mortgage on a house, as between the parties to said contract (Standard Oil Co. of N.Y. vs. Jaramillo), the same cannot and does not bind third persons, who are not parties to the aforementioned contract or their privies (Leung Yee vs. Strong Machinery Co.; Evangelista vs. Alto Surety; Navarro vs. Pineda). As a consequence, the sale of the house in question in the proceedings for the extra-judicial foreclosure of said chattel mortgage, is null and void insofar as defendant Mangubat is concerned, and did not confer upon Mrs. Uy Kim, as buyer in said sale, any dominical right in and to said house (De la Riva vs. Ah Yee), so that she could not have transmitted to her assignee, Piansay, any such right as against Mangubat. In short, plaintiffs have no cause of action against the defendants herein. Tumalad v. Vicencio [G.R. No. L-30173. September 30, 1971.] En Banc, Reyes JBL (J): 10 concur Facts: On 1 September 1955 Vicencio and Simeon, defendants-appellants, executed a chattel mortgage in favor of the Tumalads, plaintiff-appellees over their house of strong materials located at 550 Int. 3, Quezon Boulevard, Quiapo, Manila, over Lot 6-B and 7-B, Block 2554, which were being rented from Madrigal & Company, Inc. The mortgage was registered in the Registry of Deeds

of Manila on 2 September 1955. The mortgage was executed to guarantee a loan of P4,800.00 received from the Tumalads, payable within one year at 12% per annum. The mode of payment was P150.00 monthly, starting September, 1955, up to July 1956, and the lump sum of P3,150 was payable on or before August, 1956. It was also agreed that default in the payment of any of the amortizations would cause the remaining unpaid balance to become immediately due and payable, the Chattel Mortgage enforceable, and the Sheriff of Manila authorized the Mortgagors property after necessary publication. When Vicencio and Simeon defaulted in paying, the mortgage was extrajudicially foreclosed, and on 27 March 1956, the house was sold at public auction pursuant to the said contract. As highest bidder, the Tumalads were issued the corresponding certificate of sale. On 18 April 1956, the Tumalads commenced Civil Case 43073 in the municipal court of Manila, praying, among other things, that the house be vacated and its possession surrendered to them, and for Vicencio and Simeon to pay rent of P200.00 monthly from 27 March 1956 up to the time the possession is surrendered. On 21 September 1956, the municipal court rendered its decision in favor of the Tumalads. Having lost therein, appealed to the court a quo (Civil Case 30993) which also rendered a decision against them. On appeal, the case was certified to the Supreme Court by the Court of Appeals (CA-G.R. No. 27824-R) for the reason that only questions of law are involved. Plaintiffs-appellees failed to file a brief and this appeal was submitted for decision without it. Nearly a year after the foreclosure sale the mortgaged house had been demolished on 14 and 15 January 1957 by virtue of a decision obtained by the lessor of the land on which the house stood. The Supreme Court reversed the decision appealed from and entered another dismissing the complaint, with costs against plaintiffs-appellees. 1. Answer a mere statement and not evidence; Allegations or averments determines jurisdiction It has been held in Supia and Batiaco vs. Quintero and Ayala that "the answer is a mere statement of the facts which the party filing it expects to prove, but it is not evidence; and further, that when the question to be determined is one of title, the Court is given the authority to proceed with the hearing of the cause until this fact is clearly established. In the case of Sy vs. Dalman, wherein the defendant was also a successful bidder in an auction sale, it was likewise held by the Court that in detainer cases the claim of ownership "is a matter of defense and raises an issue of fact which should be determined from the evidence at the trial." What determines jurisdiction are the allegations or averments in the complaint and the relief asked for. 2. Fraud and deceit renders a contract voidable or annullable, and not void ab initio; Claim of ownership by virtue of voidable contract fails without evidence that steps were made to annul the same Fraud or deceit does not render a contract void ab initio, and can only be a ground for rendering the contract voidable or annullable pursuant to Article 1390 of the New Civil Code, by a proper action in court. In the present case, the charge of fraud, deceit or trickery, the conterntions are not supported by evidence. Further, there is nothing on record to show that the mortgage has been annulled. Neither is it disclosed that steps were taken to nullify the same. Hence, defendants-appellants' claim of ownership on the basis of a voidable contract which has not been voided fails. 3. Buildings as immovable The rule about the status of buildings as immovable property is stated in Lopez vs. Orosa, Jr. and Plaza Theatre, Inc., cited in Associated Insurance Surety Co., Inc. vs. Iya, et al. 16 to the effect that the inclusion of the building, separate and distinct from the land, in the enumeration of what may constitute real properties (art. 415, New Civil Code) could only mean one thing that a building is by itself an immovable property irrespective of whether or not said structure and the land on which it is adhered to belong to the same owner. 4. Deviations allowed, parties treatment of real property as personal property; cases Certain deviations, however, have been allowed for various reasons. In the case of Manarang and Manarang vs. Ofilada, it was held that "it is undeniable that the parties to a contract may by agreement treat as personal property that which by ,nature would be real property", citing Standard Oil Company of New York vs. Jaramillo. In the latter case, the mortgagor conveyed and transferred to the mortgagee by way of mortgage "the following described personal property." The "personal property" consisted of leasehold rights and a building. In the case of Luna vs. Encarnacion, the subject of the contract designated as Chattel Mortgage was a house of mixed materials, and the Court held therein that it was a valid Chattel mortgage because it was so expressly designated and specifically that the property given as security "is a house of mixed materials, which by its very nature is considered personal property." In Navarro vs. Pineda, the Court stated that the view that parties to a deed of chattel mortgage may agree to consider a house as personal property for the purposes of said contract, 'is good only insofar as the contracting parties are concerned. It is based, partly, upon the principle of estoppel' (Evangelista vs. Alto Surety). In a case, a mortgaged house built on a rented land was held to be a personal property, not only because the deed of mortgage considered it as such, but also because it did not form part of the land, for it is now settled that an object placed on land by one who had only a temporary right to the same, such as the lessee or usufructuary, does not become immobilized by attachment (Valdez vs. Central Altagracia, cited in Davao Sawmill vs. Castillo). Hence, if a house belonging to a person stands on a rented land belonging to another person, it may be mortgaged as a personal property as so stipulated in the document of mortgage. It should be noted, however that the principle is predicated on statements by the owner declaring his house to be a chattel, a conduct that may conceivably estop him from subsequently claiming otherwise. (Ladera vs. C.N. Hodges). 5. House treated by parties as chattel; factors to determine In the contract, the house on rented land is not only expressly designated as Chattel Mortgage; it specifically provides that "the mortgagor voluntarily cedes, sells and transfers by way of Chattel Mortgage the property together with its leasehold rights over the lot on which it is constructed and participation;" whcih could only have meant to convey the house as chattel, or at least, intended to treat the same as such, so that they should not now be allowed to make an inconsistent stand by claiming otherwise. Moreover, the subject house stood on a rented lot to which defendants-appellants merely had a temporary right as lessee, and although this can not in itself alone determine the status of the property, it does so when combined with other factors to sustain the interpretation that the parties, particularly the mortgagors, intended to treat the house as personality.

6. Estoppel; Party in chattel mortgage cannot question validity of chattel mortgage entered into Inlike in the Iya cases, Lopez vs. Orosa, Jr. and Plaza Theatreand Leung Yee vs. F. L. Strong Machinery and Williamson, wherein third persons assailed the validity of the chattel mortgage, it is the defendants-appellants themselves, as debtors-mortgagors, who are attacking the validity of the chattel mortgage in this case. The doctrine of estoppel therefore applies to the herein defendantsappellants, having treated the subject house as personalty. 7. Chattel mortgage covered by Act 1508, Chattel Mortgage Law Chattel mortgages are covered and regulated by the Chattel Mortgage Law, Act 1508. Section 14 of this Act allows the mortgagee to have the property mortgaged sold at public auction through a public officer in almost the same manner as that allowed by Act 3135, as amended by Act 4118, provided that the requirements of the law relative to notice and registration are complied with. In the present case, the parties specifically stipulated that "the chattel mortgage will be enforceable in accordance with the provisions of Special Act 3135. 8. Mortgagors entitled to remain in possession without rent within redemption period Nearly a year after the foreclosure sale the mortgaged house had been demolished on 14 and 15 January 1957 by virtue of a decision obtained by the lessor of the land on which the house stood. The CFI sentenced the mortgagors to pay a monthly rent of P200.00 from the time the chattel mortgage was foreclosed until when it was torn down by the sheriff. The Court ruled that the mortgagors were entitled to remain in possession without any obligation to pay rent during the one year redemption period after the foreclosure sale. Section 6 of Act 3135 provides that the debtor-mortgagor may, at any time within one year from and after the date of the auction sale, redeem the property sold at the extra judicial foreclosure sale. Section 7 of the same Act allows the purchaser of the property to obtain from the court the possession during the period of redemption: but the same provision expressly requires the filing of a petition with the proper CFI and the furnishing of a bond. It is only upon filing of the proper motion and the approval of the corresponding bond that the order for a writ of possession issues as a matter of course. No discretion is left to the court. In the absence of such a compliance, as in the instant case, the purchaser can not claim possession during the period of redemption as a matter of right. In such a case, the governing provision is Section 34, Rule 39, of the Revised Rules of Court which also applies to properties purchased in extrajudicial foreclosure proceedings. 9. Rentals received during redemption period credited to redemption price Before the expiration of the 1-year period within which the judgment-debtor or mortgagor may redeem the property, the purchaser thereof is not entitled, as a matter of right, to possession of the same. Thus, while it is true that the Rules of Court allow the purchaser to receive the rentals if the purchased property is occupied by tenants, he is, nevertheless, accountable to the judgmentdebtor or mortgagor as the case may be, for the amount so received and the same will be duly credited against the redemption price when the said debtor or mortgagor effects the redemption. Differently stated, the rentals receivable from tenants, although they may be collected by the purchaser during the redemption period, do not belong to the latter but still pertain to the debtor of mortgagor. The rationale for the Rule, it seems, is to secure for the benefit of the debtor or mortgagor, the payment of the redemption amount and the consequent return to him of his properties sold at public auction. (Reyes vs. Hamada, reiterating Chan v. Espe) 10. Case prematurely filed The period of redemption had not yet expired when action was instituted in the court of origin, and that plaintiffs-appellees did not choose to take possession under Section 7, Act 3135, as amended, which is the law selected by the parties to govern the extrajudicial foreclosure of the chattel mortgage. Neither was there an allegation to that effect. Since plaintiffs-appellees' right to possess was not yet born at the filing of the complaint, there could be no violation or breach thereof. Wherefore, the original complaint stated no cause of action and was prematurely filed. 11. Courts authority to review errors not assigned Even if there was no assignment of error to that effect, the Supreme Court is clothed with ample authority to review palpable errors not assigned as such if it finds that their consideration is necessary in arriving at a just decision of the case. Chua Peng Hian v. CA [G.R. No. 60015. December 19, 1984.] Second Division, Aquino (J): 4 concur, 1 took no part. Facts: Miguel C. Veneracion owned a 2,194 sq.m. lot located at 787 Melencio Street, Cabanatuan City. He leased it in 1948 to Patrick Chua Peng Hian for 10 years. The lease was renewed for another 10 years. Chua constructed on that lot a 2-storey building, the ground floor being the sawmill and the second floor as residence. On 25 May 1968, after the second lease agreement had expired, Veneracion leased to Chua 1,850 square meters of the lot for 3 years or from 1 May 1968 to 1 May 1971 at the monthly rental of P1,500. The new lease contract stipulates that the lease shall terminate automatically without extension and the lessee shall vacate and surrender the premises without any obstruction thereon; that in the event that the Lessee fails to surrender and vacate the leased premises at the expiration of this lease on May 1, 1971, and/or to remove his buildings and improvements, same shall automatically remain as property of the Lessor without the necessity of executing a Deed of Transfer or conveyance of the aforementioned properties; that this document will serve as Deed of Transfer and Conveyance of the above mentioned buildings and improvements in favor of the Lessor; and that the Lessee shall pay the Lessor "compensatory damages" of P20,000 plus attorney's fee of P2,000 should the Lessor seek judicial relief by reason of Lessee's non-fulfillment or violation of the terms of the lease. On 6 February 1969 Veneracion died. After the third lease contract expired or on 1 May 1971, Veneracion's heirs demanded that Chua vacate the premises and pay the accrued rentals. Chua did not comply with their demand. On 11 April 1972, the Veneracions filed in the CFI Nueva Ecija an action for specific performance against Chua. The trial court rendered a decision from which both parties appealed. On 30 October 1980, the Appellate Court rendered judgment ordering Chua and his family to vacate the land in question; to convey the buildings and improvements existing on the land to the Veneracion heirs and to pay the monthly rental of P1,500 from June, 1971 until he delivers possession thereof and the amount of P20,000 as compensatory damages plus P2,000 as attorney's fee. Chua appealed to the Supreme Court.

The Supreme Court affirmed the judgment of the Court of Appeals; with costs against the petitioner. 1. CFI has jurisdiction over issue on rights of parties to a building constructed on land Where the issues raised before the inferior court do not only involve possession of the lot but also the rights of the parties to the building constructed thereon, the Court of First Instance and not the municipal or city court has jurisdiction over the case (Ortigas and Co., Ltd. Partnership vs. Court of Appeals, G.R. No. 52488, July 25, 1981, 106 SCRA 121). 2. CFI has jurisdiction as action for specific performance is not capable of pecuniary The action sought was for specific performance of the stipulations of a lease contract. It was not capable of pecuniary estimation. It was within the exclusive original jurisdiction of the Court of First Instance (De Jesus vs. Garcia, 125 Phil. 965; Lapitan vs. Scandia, Inc., L-24668, July 31, 1968, 24 SCRA 479). 3. Building and improvements on leased land treated as personal property The building and improvements on the leased land may be treated as personal properties (Standard Oil Co. of New York vs. Jaramillo, 44 Phil. 630; Luna vs. Encarnacion, 91 Phil. 531; Manarang vs. Ofilada, 99 Phil. 108; Tumalad vs. Vicencio, L-30173, September 30, 1971, 41 SCRA 143, 152-3). Alienation in the contract of lease of the Lessees improvements is not a disposition of conjugal realty without the wife's consent. 4. Stipulation that Lessor would become owner of improvements is valid The validity of a stipulation that the lessor would become the owner of the improvements constructed by the lessee on the leased land has been sustained (Lao Chit vs. Security Bank & Trust Co. and Consolidated Investment, Inc., 105 Phil. 490; Co Bun Kin vs. Liongson, 100 Phil. 1091). 5. Contract is the law between the parties. The case is governed by the lease contract which is the law between the parties. The four-year extension of the lease made by the trial court and the amount of damages do not merit any serious consideration. Board of Assessment Appeals v. City Treasurer [G.R. No. L-15334. January 31, 1964.] En Banc, Paredes (J): 8 concur, 1 concur in result, 1 took no part. Facts: On 20 October 1902, the Philippine Commission enacted Act 484 which authorized the Municipal Board of Manila to grant a franchise to construct, maintain and operate an electric street railway and electric light, heat and power system in the City of Manila and its suburbs to the person or persons making the most favorable bid. Charles M. Swift was awarded the said franchise on March 1903, the terms and conditions of which were embodied in Ordinance 44 approved on 24 March 1903. Meralco became the transferee and owner of the franchise. Meralco's electric power is generated by its hydro-electric plant located at Botocan Falls, Laguna and is transmitted to the City of Manila by means of electric transmission wires, running from the province of Laguna to the said City. These electric transmission wires which carry high voltage current, are fastened to insulators attached on steel towers constructed by respondent at intervals, from its hydro-electric plant in the province of Laguna to the City of Manila. Meralco has constructed 40 of these steel towers within Quezon City, on land belonging to it. On 15 November 1955, City Assessor of Quezon City declared the aforesaid steel towers for real property tax under Tax Declaration 31992 and 15549. After denying Meralco's petition to cancel these declarations an appeal was taken by Meralco to the Board of Assessment Appeals of Quezon City, which required Meralco to pay the amount of P11,651.86 as real property tax on the said steel towers for the years 1952 to 1956. Meralco paid the amount under protest, and filed a petition for review in the Court of Tax Appeals which rendered a decision on 29 December 1958, ordering the cancellation of the said tax declarations and the City Treasurer of Quezon City to refund to Meralco the sum of P11,651.86. The motion for reconsideration having been denied, on 22 April 1959, the petition for review was filed. The Supreme Court affirmed the decision appealed from, with costs against the petitioners. 1. Definition of pole The word "pole" means "a long, comparatively slender usually cylindrical piece of wood or timber, as typically, the stem of a small tree stripped of its branches; also, by extension, a similar typically cylindrical piece or object of metal or the like". The term also refers to "an upright standard to the top of which something is affixed or by which something is supported; as a dovecote set on a pole; telegraph poles; a tent pole; sometimes, specifically, a vessel's mast." (Webster's New International Dictionary, 2nd Ed. p. 1907.) Poles made of two steel bars joined together by an interlacing metal rod, are called "poles" notwithstanding the fact that they are not made of wood. 2. Steel towers, which is within the term poles, are exempted from taxes under part II, paragraph 9 of Meralcos franchise Paragraph 9 of Meralcos franchise provides that the concept of the "poles" for which exemption is granted, is not determined by their place or location, nor by the character of the electric current it carries, nor the material or form of which it is made, but the use to which they are dedicated. In accordance with the definitions, a pole is not restricted to a long cylindrical piece of wood or metal, but includes "upright standards to the top of which something is affixed or by which something is supported." In the present case, Meralco's steel supports consist of a framework of four steel bars or strips which are bound by steel cross-arms atop of which are cross-arms supporting five high voltage transmission wires and their sole function is to support or carry such wires. 3. Interpretation of poles so as to include towers is not a novelty; US cases The conclusion that the steel supports in question are embraced in the term "poles" is not a novelty. Several courts of last resort in the United States have called these steel supports "steel towers", and they have denominated these steel supports or towers, as electric poles. In their decisions the words "towers" and "poles" were used interchangeably, and it is well understood in that

jurisdiction that a transmission tower or pole means the same thing. (See Steamons v. Dallas Power & Light Co. (Text) 212 S.W. 222, 224; 32-A Words and Phrases p. 365.; Salt River Valley Users' Ass'n. v. Compton 8 p. 2nd. 249-250; and Inspiration Consolidation Cooper Co. v. Bryan, 252 p. 1016) 4. Interpretation should not be restrictive and narrow to defeat the object for which the franchise granted The word "poles", as used in Act 484 and incorporated in the Meralco's franchise, should not be given a restrictive and narrow interpretation, as to defeat the very object for which the franchise was granted. The poles as contemplated thereon, should be understood and taken as a part of the electric power system of the Meralco, for the conveyance of electric current from the source thereof to its consumers. If the respondent would be required to employ "wooden poles," or "rounded poles" as it used to do 50 years ago, it would be a departure to progress in technology. Steel towers, thus, can better effectuate the purposes for which Meralco's franchise was granted. 5. Tax law does not define real property; Article 415 of the Civil Code defines by enumeration The tax law does not provide for a definition of real property; but Article 415 of the Civil Code does, by stating which are immovable property. 6. Steel towers are not immovable property under paragraph 1, 3 and 5 The steel towers or supports do not come within the objects mentioned in paragraph 1, because they do not constitute buildings or constructions adhered to the soil. They are not constructions analogous to buildings nor adhering to the soil. As per description, given by the lower court, they are removable and merely attached to a square metal frame by means of bolts, which when unscrewed could easily be dismantled and moved from place to place. They can not be included under paragraph 3, as they are not attached to an immovable in a fixed manner, and they can be separated without breaking the material or causing deterioration upon the object to which they are attached. Each of these steel towers or supports consists of steel bars or metal strips, joined together by means of bolts, which can be disassembled by unscrewing the bolts and reassembled by screwing the same. These steel towers or supports do not also fall under paragraph 5, for they are not machineries or receptacles, instruments or implements, and even if they were, they are not intended for industry or works on the land. Petitioner is not engaged in an industry or works on the land in which the steel supports or towers are constructed. 7. It is the duty of the City Treasurer to refund; legal technicalities cannot be availed of Indulging in legal technicalities and niceties which do not help the City Treasurer any; for, factually, it was he who had insisted that respondent herein pay the real estate taxes, which Meralco paid under protest. Having acted in his official capacity as City Treasurer of Quezon City, he would surely know what to do, under the circumstances. Thus, he cannot be sustained in his argument that as the City Treasurer is not the real party in interest, but Quezon City, which was not made a party to the suit, notwithstanding its capacity to sue and be sued, he should not be ordered to effect the refund. The question has not been raised in the lower court and, therefore, it cannot properly be raised for the first time on appeal. Inter-regional Development Corporation v. CA [G.R. No. L-39677. July 22, 1975.] First division, Castro (J): 4 concur, 1 on leave Facts: In civil case 8195 in the CFI Iloilo (Branch 3, Inter-Regional Development Corporation vs. Isidro Estrada), for the annulment of sales and damages and involving land ownership, the Court through Judge Castrence Veloso, ordered Jose and Isabel Baez (president and treasurer, respectively, of the Inter-Regional Development Corporation) or their privies to immediately vacate the possession of Lots 1 and 6, Plan Psu-118496 to Isidro A. Estrada, third-party plaintiff, and not to molest, disturb or in any manner interfere with his possession thereof. The corporation appealed; nonetheless, Judge Veloso issued, on 7 May 1973, a partial writ of execution pending appeal, and possession of the two lots was delivered to Estrada, following which the latter, on 25 May 1973, leased the lots to Ricardo Caballero for a term of 10 years, starting with the crop year 1973-74. On July 6, 1973 the corporation filed a special civil action for certiorari with the Court of Appeals, praying that Judge Veloso be enjoined from enforcing the partial writ of execution. Holding that the said Judge did not abuse his discretion in ordering partial execution, the Court of Appeals, on 16 October 1973, dismissed the petition. This decision became final. 5 days before entry of judgment, or on 15 November 1973, the corporation filed civil case 9562, for injunction and damages, against the lessee Caballero to settle the question of ownership of the sugar crop for 1973-74 which the corporation alleged to have planted in good faith on the lands litigated in civil case 8195. On 16 January 1974, Judge Sancho Y. Inserto of the CFI Iloilo (Branch 1) enjoined Ricardo Caballero in Civil Case 9562 (Inter-Regional Development Corporation vs. Ricardo Caballero) from cutting and milling the sugar cane crop which the company asserts to have planted in good faith. Caballero moved for reconsideration, but before the court could act on his motion, he filed a petition for certiorari with the CA. On 30 September 1974 the appellate court promulgated its decision setting aside Judge Inserto's preliminary injunction. The corporation filed a petition for review on certiorari with the Supreme Court. The Court resolved to treat the case as a special civil action upon consideration of the allegations contained, the issues raised, and the arguments adduced. The Court also issued a TRO preventing the CA from executing its decision, and preventing Caballero and his assigns from cutting the sugar cane. The Supreme Court set aside the judgment of the Court of Appeals and the restraining order issued is made permanent, without prejudice, however, to the final outcome in case 9562 of the CFI Iloilo; without costs. 1. Preliminary injunction in case 9562 and the partial writ of execution pending appeal issued in case 8195 are complementary and not interfering with each other; Absent incompatibility, doctrine that equal courts have no power to interfere does not apply

The preliminary injunction issued in case 9562 (by Judge Inserto enjoining Caballero from cutting and milling the sugar cane) and the partial writ of execution pending appeal issued in case 8195 (by Judge Veloso ordering the Baez spouses to vacate and surrender possession of the parcels of land and not to molest or disturb Estrada's, Caballero's lessor, possession thereof) do not interfere with the other; but as a matter of fact the order of Judge Inserto complements that of Judge Veloso. The latter refers to the land itself, the ownership of which was the only issue adjudged in the decision pending appeal, while the former refers to the sugar cane crop standing thereon. Absent any incompatibility between the orders issued by Judges Inserto and Veloso, the doctrine that no court has the power to interfere by injunction with the judgment or decrees of another court of concurrent or coordinate jurisdiction, having equal power to grant the relief, does not apply. 2. Landowner does not ipso facto acquire ownership of what has been planted; good faith is issue True it is that under article 440 of the Civil Code the ownership of property includes the right of accession to everything attached thereto either naturally or artificially, and that under article 415, trees, plants and growing fruits, while they are attached to the land, are immovable property; it is equally true that when a person plants in good faith on land belonging to another, the landowner does not ipso facto acquire ownership of what has been planted; he must first indemnify the planter before he can appropriate the same. In the present case, the petitioner has alleged good faith in planting the sugar cane, thus giving rise to a conflict of rights which poses the issue of the protection of the alleged planter in good faith without causing injustice to the landowner. 3. Gathering crops existing as Estradas right of ownership and possession premature The Court of Appeals erred in holding that the gathering of the crops existing on the land is part of Estrada's right of ownership and possession, as this, in effect prematurely held that the petitioner is a planter in bad faith. The issues as to who planted and whether the planter planted in good faith are the very issues posed in case 9562, which is yet pending. 4. Procedure; Perishability of goods allow the hearing of petition for certiorari even if motion for reconsideration is pending in the lower court The Court of Appeals did not err in entertaining the petition for certiorari even if a motion for reconsideration had not yet been resolved by the Court of First Instance, in view of the urgency of securing a definitive ruling on the sugar cane crop, which is perishable. Sibal v. Valdez [G.R. No. 26278. August 4, 1927.] Second Division, Johnson (J): 5 concur Facts: On 11 May 1923, the deputy-sheriff of the Province of Tarlac, by virtue of a writ of execution in civil case 20203 of the CFI Manila (Macondray & Co., Inc. vs. Leon Sibal), levied an attachment on Leon Sibals 8 parcels of land for the sum of P4,273.93. 2 months later, or on 30 July 1923, Macondray & Co., Inc., bought said parcels of land, at the auction held by the sheriff of the Province of Tarlac. Within 1 year from the sale of said parcels of land, or on 24 September 1923, Sibal paid P2,000 to Macondray for the account of the redemption price of said parcels of land, without specifying which said amount was to be applied. The redemption price of the parcels was reduced to P2,579.97 including interest. On 29 April 1924, the deputy sheriff of the Province of Tarlac, by virtue of a writ of execution in civil case 1301 of the Province of Pampanga (Emiliano J. Valdez vs. Leon Sibal 1.), attached the personal property of Sibal located in Tarlac, among which was included the sugar cane in question in the 7 parcels of land described in the complaint. He also attached Sibals real property in Tarlac, including rights, interest and participation therein, which consists of 11 parcels of land and a house and camarin situated in one of said parcels. On 9-10 May 1924, the deputy sheriff sold at public auction said personal properties to Emiliano J. Valdez, who paid therefor the sum of P1,550, of which P600 was for the sugar cane. On 25 June 1924, 8 of the 11 parcels, including the camarin and the house were bought by Valdez at the auction held by the sheriff for the sum of P12,200. The 3 remaining parcels were released from attachment by virtue of claims presented by Cayugan and Tizon. On the same date, Macondray sold and conveyed to Valdez for P2,579.97 all of its rights and interest in the 8 parcels of land acquired by it in connection with civil case 20203 of the CFI Manila. On 14 December 1924, action was commenced in the CFI of the Province of Tarlac. The plaintiff alleged that the deputy sheriff of Tarlac Province attached and sold to Valdez the sugar cane planted by the plaintiff and his tenants on 7 parcels of land, and that within 1 year from the date of the attachment and sale the plaintiff ordered to redeem said sugar cane and tendered to Valdez the amount sufficient to cover the price paid by the latter, with taxes and interests, and that Valdez refused to accept the money and return the sugar cane to the plaintiff. After hearing and on 28 April 1926, the judge (Lukban) rendered judgment in favor of the defendant holding that the sugar cane in question was personal property and, as such, was not subject to redemption; among others. Hence, the appeal. 1. Paragraph 2, Article 334 of the Civil Code interpreted by the Tribunal Supremo de Espana as that growing crops may be considered as personal property Sugar cane may come under the classification of real property as "ungathered products" in paragraph 2 of article 334 of the Civil Code, which enumerates as real property as "Trees, plants, and ungathered products, while they are annexed to the land or form an integral part of any immovable property." That article, however, has received in recent years an interpretation by the Tribunal Supremo de Espaa, which holds that, under certain conditions, growing crops may be considered as personal property. (Decision of March 18, 1904, vol. 97, Civil Jurisprudence of Spain.) Thus, under Spanish authorities, pending fruits and ungathered products may be sold and transferred as personal property. Also, the Supreme Court of Spain, in a case of ejectment of a lessee of an agricultural land, held that the lessee was entitled to gather the Products corresponding to the agricultural year because said fruits did not go with the land but belonged separately to the lessee. And further, under the Spanish Mortgage Law of 1909, as amended, the mortgage of a piece of land does not include the fruits and products existing thereon, unless the contract expressly provides otherwise.

2. Manresa admits growing crops as personal property Manresa, the eminent commentator of the Spanish Civil Code, in discussing section 334 of the Civil Code, in view of the recent decisions of the Supreme Court of Spain, admits that growing crops are sometimes considered and treated as personal property. 3. Paragraph 2, Article 344 of the Civil Code corresponds to Article 465 of the Civil Code of Louisiana Article 465 of the Civil Code of Louisiana, which corresponds to paragraph 2 of article 334 of the Civil Code, provides: "Standing crops and the fruits of trees not gathered, and trees before they are cut down, are likewise immovable, and are considered as part of the land to which they are attached." 4. Louisiana jurisprudence: Growing crops mobilization by anticipation Standing crops and the fruits of trees not gathered and trees before they are cut down are considered as part of the land to which they are attached,' but the immovability provided for is only one in abstracto and without reference to rights on or to the crop acquired by others than the owners of the property to which the crop is attached. The existence of a right on the growing crop is a mobilization by anticipation, a gathering as it were in advance, rendering the crop movable quoad the right acquired therein. Jurisprudence recognizes the possible mobilization of the growing crop. (Citizens' Bank vs. Wiltz, 31 La. Ann., 244; Porche vs. Bodin, 28 La. Ann., 761; Sandel vs. Douglass, 27 La. Ann., 629; Lewis vs. Klotz, 39 La. Ann., 267.; as cited in Lumber Co. vs. Sheriff and Tax Collector [106 La., 418], c.f. Citizens Bank v. Wiltz [31 La. Ann., 244]) 5. Louisiana jurisprudence: Standing crops as immovable or movable based on owned and leased premises; seizure by creditors Standing crops are considered as immovable and as part of the land to which they are attached, and the fruits of an immovable gathered or produced while it is under seizure are considered as making part thereof, and inure to the benefit of the person making the seizure. But the evident meaning of these articles is, where the crops belong to the owner of the plantation, they form part of the immovable, and where it is seized, the fruits gathered or produced inure to the benefit of the seizing creditor. A crop raised on leased premises in no sense forms part of the immovable. It belongs to the lessee, and may be sold by him, whether it be gathered or not, and it may be sold by his judgment creditors. (Porche vs. Bodin [28 La. An., 761]) 6. Louisiana jurisprudence: Law cannot be interpreted result in absurd consequences If crop necessarily forms part of the leased premises the result would be that it could not be sold under execution separate and apart from the land. If a lessee obtain supplies to make his crop, the factor's lien would not attach to the crop as a separate thing belonging to his debtor, but the land belonging to the lessor would be affected with the recorded privilege. The law cannot be construed so as to result in such absurd consequences. 7. American jurisprudence: growing crops by yearly labor and cultivation personal property The settled doctrine followed in the State of California and other states in connection with the attachment of property and execution of judgment is, that growing crops raised by yearly labor and cultivation are considered personal property. All annual crops which are raised by yearly manurance and labor, and essentially owe their annual existence to cultivation by man, may be levied on as personal property. Crops, whether growing or standing in the field ready to be harvested, are, when produced by annual cultivation, no part of the realty. They are, therefore, liable to voluntary transfer as chattels. It is equally well settled that they may be seized and sold under execution. 8. Valid sale of a thing not yet in existence; thing must be owned by the vendor A valid sale may be made of a thing, which though not yet actually in existence, is reasonably certain to come into existence as the natural increment or usual incident of something already in existence, and then belonging to the vendor, and the title will vest in the buyer the moment the thing comes into existence. (Emerson vs. European Railway Co., 67 Me., 387; Cutting vs. Packers Exchange, 21 Am. St. Rep., 63.) The thing sold, however, must be specific and identified. They must be also owned at the time by the vendor. (Hull vs. Hull, 48 Conn., 250 [40 Am. Rep., 165].) 9. Source of provisions on execution of judgment in Code of Civil Procedure (Act 190); Growing crops are personal property Section 450 and most of the other sections of the Code of Civil Procedure relating to the execution of judgments were taken from the Code of Civil Procedure of California. Section 450 of the Code of Civil Procedure enumerates the property of a judgment debtor which may be subjected to execution, and reads as "All goods, chattels, moneys, and other property, both real and personal, shall be liable to execution." The Supreme Court of California, under section 688 of the Code of Civil Procedure of that state, to which the Code of Civil Procedure was pattered, has held, without variation, that growing crops were personal property and subject to execution. 10. Chattel Mortgage Law recognizes growing crops as personal property Act 1508, the Chattel Mortgage Law, fully recognizes that growing crops are personal property. Section 2 of said Act provides that "All personal property shall be subject to mortgage, agreeably to the provisions of this Act, and a mortgage executed in pursuance thereof shall be termed a chattel mortgage." Section 7 in part provides that "If growing crops be mortgaged the mortgage may contain an agreement stipulating that the mortgagor binds himself properly to tend. care for and protect the crop while growing." The above provisions of Act 1508 were enacted on the assumption that "growing crops" are personal property. 11. Personal property includes ungathered products; Paragraph 2, Article 334 of the Civil Code modified by Act 190 and 1508 Paragraph 2 of article 334 of the Civil Code has been modified by section 450 of Act No. 190 and by Act No. 1508 in the sense that "ungathered products" as mentioned in said article of the Civil Code have the nature of personal property; or that in the sense that, for the purposes of attachment and execution, and for the purposes of the Chattel Mortgage Law, "ungathered products" have the nature of personal property. In other words, the phrase "personal property" should be understood to include "ungathered products." In the case at bar, the sugar cane in question was personal property and was not subject to redemption.

12. Absence from trial and failure to cross-examine lend weight to the evidence presented by the other party The absence of the plaintiff from the trial and his failure to cross-examine the defendant have lent considerable weight to the evidence then presented for the defense. The court has been inclined to to give more weight to the evidence adduced by him than to the evidence adduced by the plaintiff, with respect to the ownership of parcels of land. 13. Lack of evidence of bad faith in planting palay in questioned parcels entitles plaintiff to of the crop There being no evidence of bad faith on the plaintiffs part, in planting the palay in the disputed parcels of land and harvested therefrom 190 cavans, he is therefore entitled to one-half of the crop, or 95 cavans, not 190 cavans as ordered by the lower court. Berkenkotter v. Cu Unjieng [G.R. No. 41643. July 31, 1935.] En Banc, Villa-real (J): 4 concur Facts: On 26 April 1926, the Mabalacat Sugar Company obtained from Cu Unjieng e Hijos, a loan secured by a first mortgage constituted on 2 parcels of land "with all its buildings, improvements, sugar-cane mill, steel railway, telephone line, apparatus, utensils and whatever forms part or is a necessary complement of said sugar-cane mill, steel railway, telephone line, now existing or that may in the future exist in said lots. On 5 October 1926, the Mabalacat Sugar Company decided to increase the capacity of its sugar central by buying additional machinery and equipment, so that instead of milling 150 tons daily, it could produce 250. The estimated cost of said additional machinery and equipment was P100,000. In order to carry out this plan, B. A. Green, president of said corporation, proposed to the B. H. Berkenkotter, to advance the necessary amount for the purchase of said machinery and equipment, promising to reimburse him as soon as he could obtain an additional loan from the mortgagees, Cu Unjieng e Hijos, and that in case B. A. Green should fail to obtain an additional loan from Cu Unjieng e Hijos, said machinery and equipment would become security therefor, said B. A. Green binding himself not to mortgage nor encumber them to anybody until Berkenkotter be fully reimbursed for the corporation's indebtedness to him.. Having agreed to said proposition made in a letter dated 5 October 1926, B. H. Berkenkotter, on 9 October 1926, delivered the sum of P1,710 to B. A. Green, the total amount supplied by him to B. A. Green having been P25,750. Furthermore, B. H. Berkenkotter had a credit of P22,000 against said corporation for unpaid salary. With the loan of P25,750 and said credit of P22,000, the Mabalacat Sugar Co., Inc., purchased the additional machinery and equipment. On 10 June 1927, B. A. Green applied to Cu Unjieng e Hijos for an additional loan of P75,000 offering as security the additional machinery and equipment acquired by said B. A. Green and installed in the sugar central after the execution of the original mortgage deed, on 27 April 1927, together with whatever additional equipment acquired with said loan. B. A. Green failed to obtain said loan. The case facts do not provide for the instance when the case was filed with the lower court, and details of such controversy The CFI Manila dismissed Berkenkotters complaint, with costs. Hence, the appeal. The Supreme Court affirmed the appealed judgment in all its parts, with costs to the appellant. 1. Mortgage include all natural accessions, improvements, growing fruits and rents not collected Article 1877 of the Civil Code provides that mortgage includes all natural accessions, improvements, growing fruits, and rents not collected when the obligation falls due, and the amount of any indemnities paid or due the owner by the insurers of the mortgaged property or by virtue of the exercise of the power of eminent domain, with the declarations, amplifications, and limitations established by law, whether the state continues in the possession of the person who mortgaged it or whether it passes into the hands of a third person. 2. Mortgage includes improvements and fixtures It is a rule, established by the Civil Code and also by the Mortgage Law, with which the decisions of the courts of the United States are in accord, that in a mortgage of real estate, the improvements on the same are included; therefore, all objects permanently attached to a mortgaged building or land, although they may have been placed there after the mortgage was constituted, are also included. (Arts. 110 and 111 of the Mortgage Law, and 1877 of the Civil Code; decision of U.S. Supreme Court in the matter of Royal Insurance Co. vs. R. Miller, liquidator, and Amadeo [26 Sup. Ct. Rep., 46; 199 U.S., 353] as cited in Bischoff vs. Pomar and Compaia General de Tabacos; further cited in Cea vs. Villanueva) 3. Exclusion of machinery from mortgage; necessity of stipulation In order that it may be understood that the machinery and other objects placed upon and used in connection with a mortgaged estate are excluded from the mortgage, when it was stated in the mortgage that the improvements, buildings, and machinery that existed thereon were also comprehended, it is indispensable that the exclusion thereof be stipulated between the contracting parties (Bischoff vs. Pomar and Compaia General de Tabacos; further cited in Cea vs. Villanueva). 4. Machinery has character of real property Article 334, paragraph 5, of the Civil Code gives the character of real property to machinery, liquid containers, instruments or implements intended by the owner of any building or land for use in connection with any industry or trade being carried on therein and which are expressly adapted to meet the requirements of such trade or industry. The installation of a machinery and equipment in a mortgaged sugar central, in lieu of another of less capacity, for the purpose of carrying out the industrial functions of the latter and increasing production, constitutes a permanent improvement on said sugar central and subjects said machinery and equipment to the mortgage constituted thereon (article 1877, Civil Code). 5. Personal security does not alter permanent character of machinerys incorporation with the central The fact that the purchaser of the new machinery and equipment has bound himself to the person supplying him the purchase

money to hold them as security for the payment of the latter's credit, and to refrain from mortgaging or otherwise encumbering them does not alter the permanent character of the incorporation of said machinery and equipment with the central. The fact that B. A. Green bound himself to Berkenkotter to hold said machinery and equipment as security for the payment of the latter's credit and to refrain from mortgaging or otherwise encumbering them until Berkenkotter has been fully reimbursed therefor, is not incompatible with the permanent character of the incorporation of said machinery and equipment with the sugar central of the Mabalacat Sugar Co., Inc., as nothing could prevent B. A. Green from giving them as security at least under a second mortgage. 6. Creditor not vested with right of ownership but only with right of redemption The sale of the machinery and equipment in question by the purchaser who was supplied the money, after the incorporation thereof with the mortgaged sugar central, does not vest the creditor with ownership of said machinery and equipment but simply with the right of redemption. Thus, while the mortgage constituted on said sugar central to Cu Unjieng e Hijos remained in force, only the right of redemption of the vendor Mabalacat Sugar Co., Inc., the sugar central with which said machinery and equipment had been incorporated, was transferred thereby, subject to the right of Cu Unjieng e Hijos under the first mortgage. Ago v. CA [G.R. No. L-17898. October 31, 1962.] En Banc, Labrador (J): 9 concur, 1 took no part Facts: In 1957, Pastor D. Ago bought sawmill machineries and equipments from Grace Park Engineering, Inc., executing a chattel mortgage over said machineries and equipments to secure the payment of a balance of the price remaining unpaid of P32,000.00, which Ago agreed to pay on installment basis. Ago defaulted in his payments and so, in 1958, Grace Park Engineering, Inc. instituted extrajudicial foreclosure proceedings of the mortgage. To enjoin said foreclosure, Ago instituted Special Civil Case 53 in the CFI Agusan. The parties to the case arrived at a compromise agreement and submitted the same in court in writing, signed by Ago and the Grace Park Engineering. Judge Ortiz of the CFI Agusan dictated a decision in open court on 28 January 1959. Still, Ago continued to default in his payments as provided in the judgment by compromise, so Grace Park Engineering filed with the lower court a motion for execution, which was granted by the court on 15 August 1959. A writ of execution, dated 23 September 1959, later followed. The Provincial Sheriff of Surigao, acting upon the writ of execution, levied upon and ordered the sale of the sawmill machineries and equipments in question. These machineries and equipments had been taken to and installed in a sawmill building located in Lianga, Surigao del Sur, and owned by the Golden Pacific Sawmill to whom he had sold them or assigned them in payment of his subscription to the shares of stock of said corporation on 16 February 1959 (a date after the decision of the lower court but before levy by the Sheriff). Thereafter the sawmill machineries and equipments were installed in a building and permanently attached to the ground. Having been advised by the sheriff that the public auction sale was set for 4 December 1959, Ago, on 1 December 1959, filed the petition for certiorari and prohibition with preliminary injunction with the CA. The Court of Appeals on 8 December 1959, issued a writ of preliminary injunction against the sheriff but it turned out that the latter had already sold at public auction the machineries in question as scheduled. Grace Park Engineering was the only bidder for P15,000.00, although the certificate of sale was not yet executed. The CA instructed the sheriff to suspend the issuance of a certificate of sale of the said sawmill machineries and equipment until the final decision of the case. On 9 November 1960, the CA dismissed the petition for certiorari and dissolved the writ of preliminary injunction, with costs against the petitioner. The Supreme Court set aside the decision of the Court of Appeals and declared that the issuance of the writ of execution against the sawmill machineries and equipments purchased by Pastor D. Ago from the Grace Park Engineering, Inc., as well as the sale of the same by the Sheriff of Surigao, are null and void. Costs against Grace Park Engineering, Inc. 1. [?CA] Compromise agreement binding between parties A compromise agreement is binding between the parties and becomes the law between them. (Gonzales vs. Gonzales, GR L-1254 [1948]; Martin vs. Martin, GR L-12439 [1959]). 2. [?CA] Judgment based on a compromise agreement is not appealable and is executory It is a general rule in this jurisdiction that a judgment based on a compromise agreement is not appealable and is immediately executory, unless a motion is filed on the ground of fraud, mistake or duress. (De los Reyes vs. Ugarte, 75 Phil. 505; Lapea vs. Morfe, G.R. No. L-10089, July 31, 1957) 3. Judgment made in open court not real judgment of the court as it has not yet been rendered Section 1 of Rule 35 describes the manner in which judgments shall be rendered, providing that all judgments determining the merits of cases shall be in writing personally and directly prepared by the judge, and signed by him, stating clearly and distinctly the facts and the law on which it is based, and filed with the clerk of the court." The court of first instance being a court of record, in order that a judgment may be considered as rendered it must not only be in writing, signed by the judge, but it must also be filed with the clerk of court. The mere pronouncement of the judgment in open court with the stenographer taking note thereof does not, therefore, constitute a rendition of the judgment. It is the filing of the signed decision with the clerk of court that constitutes rendition. While it is to be presumed that the judgment that was dictated in open court will be the judgment of the court, the court may still modify said order as the same is being put into writing. And even if the order or judgment has already been put into writing and signed, while it has not yet been delivered to the clerk for filing, it is still subject to amendment or change by the judge. It is only when the judgment signed by the judge is actually filed with the clerk of court that it becomes a valid and binding judgment. Prior thereto, it could still be subject to amendment and change and may not, therefore, constitute the real judgment of the court. 4. Dictating judgment in open court is not valid notice of said judgment The mere fact that a party heard the judge dictating the judgment in open court, is not a valid notice of said judgment. If rendition thereof is constituted by the filing with the clerk of court of a signed copy (of the judgment), it is evident that the fact that a party

or an attorney heard the order or judgment being dictated in court cannot be considered as notice of the real judgment. No judgment can be notified to the parties unless it has previously been rendered. The notice, therefore, that a party has of a judgment that was being dictated is of no effect because at that time no judgment has as yet been signed by the judge and filed with the clerk. 5. Rules specific on the service of final orders or judgment Section 7 of Rule 27 expressly require that final orders or judgments be served personally or by registered mail. In accordance with this provision, a party is not considered as having been served with the judgment merely because he heard the judge dictating the said judgment in open court; it is necessary that he be served with a copy of the signed judgment that has been filed with the clerk in order that he may legally be considered as having been served with the judgment. 6. Issuance of writ of execution null and void As the signed judgment not having been served upon the petitioner, said judgment could not be effective upon him who had not received it. It follows as a consequence that the issuance of the writ of execution was null and void, having been issued before petitioner was served, personally or by registered mail, a copy of the decision. 7. Sawmill machineries and equipment are real properties in accordance with Art. 415 (5) By reason of installment in a building, the said sawmill machineries and equipments became real estate properties in accordance with the provision of Art. 415(5) of the Civil Code. It is interpreted similarly to the case of Berkenkotter vs. Cu Unjieng e Hijos, where the Court held that the installation of the machinery and equipment in the central of the Mabalacat Sugar Company for use in connection with the industry carried by that company, converted the said machinery and equipment into real estate by reason of their purpose. In the present case, the installation of the sawmill machineries in the building of the Golden Pacific Sawmill, Inc., for use in the sawing of logs carried on in said building, the same became a necessary and permanent part of the building or real estate on which the same was constructed, converting the said machineries and equipments into real estate within the meaning of Article 415(5) of the Civil Code of the Philippines. 8. Sale made by sheriff void for lack of publication Considering that the machineries and equipments in question valued at more than P15,000.00 appear to have been sold without the necessary advertisement of sale by publication in a newspaper, as required in Section 16 of Rule 39 of the Rules of Court, which requires publication for properties with value above P400, the sale made by the sheriff must be declared null and void. Mindanao Bus v. City Assessor [G.R. No. L-17870. September 29, 1962.] En Banc, Labrador (J): 7 concur, 3 took no part. Facts: Mindanao Bus Company is a public utility engaged in transporting passengers and cargoes by motor trucks in Mindanao; having its main offices in Cagayan de Oro. The company is also owner to the land where it maintains and operates a garafe, a repair shop, blacksmith and carpentry shops; the machineries are place therein on wooden and cement platforms. The City Assessor of Cagayan de Oro City assessed at P4,400 said maintenance and repair equipment. The company appealed the assessment to the Board of Tax Appeals on the ground that the same are not realty. The Board of Tax Appeals of the City sustained the city assessor, so the company filed with the Court of Tax Appeals a petition for the review of the assessment. The Court of Tax Appeal (CTA Case 710) held that the Company was liable to the payment of the realty tax on its maintenance and repair equipment. Hence, the company filed a petition for review with the Supreme Court. The Supreme Court set aside the decision subject of the petition for review and the equipment in question declared not subject to assessment as real estate for the purposes of the real estate tax. Without costs. 1. Machinery made immovable must be essential and principal elements of an industry or works Paragraph 5 of Article 415 of the New Civil Code (previously Article 344, paragraph 5, of the old Civil Code) which provides machinery, receptacles, instruments or implements intended by the owner of the tenement for an industry or works which may be carried on in a building or on a piece of land, and which tend directly to meet the needs of the said industry or works are immovable properties. Movable equipments to be immobilized in contemplation of the law must first be "essential and principal elements" of an industry or works without which such industry or works would be "unable to function or carry on the industrial purpose for which it was established." 2. Machinery essential to industry in Berkenkotter v. Cu Unjieng; Machinery incidental in present case It cannot be said that the incorporation of the machinery and equipment in the central of the Mabalacat Sugar Company was not permanent in character because, as essential and principal elements of a sugar central, without them the sugar central would be unable to function or carry on the industrial purpose for which it was established. Inasmuch as the central is permanent in character, the necessary machinery and equipment installed for carrying on the sugar industry for which it has been established must necessarily be permanent. In contrast, the transportation business could be carried on without the repair or service shop if its rolling equipment is repaired or serviced in another shop belonging to another. 3. Machinery incidental to industry are not immobilized; examples Movables which become immobilized by destination because they are essential and principal elements in the industry are distinguished from those which may not be so considered immobilized because they are merely incidental, not essential and principal. Thus, cash registers, typewriters, etc., usually found and used in hotels, restaurants, theaters, etc. are merely incidentals and are not and should not be considered immobilized by destination, for these businesses can continue or carry on their functions without these equipments. Airline companies use forklifts, jeep-wagons, pressure pumps, IMB machines, etc. which are incidentals, not essentials, and thus retain their movable nature. On the other hand, machineries of breweries used in the manufacture of liquor and soft drinks, though movable in nature, are immobilized because they are essential to said industries; but

the delivery trucks and adding machines which they usually own and use and are found within their industrial compounds are merely incidentals and retain their movable nature. 4. Transportation business not carried on in a building, tenement or specified land; equipment thus are not real estate Aside from the element of essentiality, Article 415 (5) also requires that the industry or works be carried on in a building or on a piece of land. In the case of Berkenkotter vs. Cu Unjieng, the "machinery, liquid containers, and instruments or implements" are found in a building constructed on the land. A sawmill would also be installed in a building on land more or less permanently, and the sawing is conducted in the land or building. In contrast, Mindanao Bus Companys transportation business is not carried on in a building, tenement or on a specified land, so said equipment may not be considered real estate within the meaning of Article 415 (c) of the Civil Code. Davao Sawmill v. Castillo [G.R. No. 40411. August 7, 1935.] En Banc, Malcolm (J): 4 concur Facts: The Davao Saw Mill is the holder of a lumber concession from the Government. It has operated a sawmill in the sitio of Maa, barrio of Tigatu, municipality of Davao, Province of Davao. However, the land upon which the business was conducted belonged to another person. On the land the sawmill company erected a building which housed the machinery used by it. Some of the implements thus used were clearly personal property, the conflict concerning machines which were placed and mounted on foundations of cement. In the contract of lease stipulated that on the expiration of the period agreed upon, or if the Lessee should leave or abandon the land leased, all the improvements and buildings introduced and erected by the Lessee shall pass to the exclusive ownership of the Lessor without any obligation on its part to pay any amount for said improvements and buildings; which do not include the machineries and accessories in the improvements. In another action (Davao Light & Power vs. Davao Saw Mill), a judgment was rendered in favor of Davao Light & Power; a writ of execution issued thereon, and the properties in question were levied upon as personalty by the sheriff. No third party claim was filed for such properties at the time of the sales thereof as is borne out by the record made by the plaintiff herein. Indeed the bidder, which was Davao Light & Power, and the defendant herein having consummated the sale, proceeded to take possession of the machinery and other properties described in the corresponding certificates of sale executed in its favor by the sheriff of Davao. It must be noted that on a number of occasions, Davao Sawmill treated the machinery as personal property by executing chattel mortgages in favor of third persons. One of such persons is the appellee by assignment from the original mortgagees. Instance on how controversy arose cant be found in the case facts. Impliedly, the issue on the character of the properties arose from the consummation of a sale following the execution of the judgment in the other action, Davao Light & Power v. Davao Sawmill The trial judge found that the properties were personal in nature, and as a consequence absolved the defendants from the complaint. The issue was raised in the Supreme Court involving the determination of the nature of said properties. The Supreme Court affirmed the judgment appealed from, with costs against the appellant. 1. Standard Oil ruling key to issue on the character of the property It must be pointed out that Davao Sawmill should have registered its protest before or at the time of the sale of this property. It must further be pointed out that while not conclusive, the characterization of the property as chattels by Davao Sawmill is indicative of intention and impresses upon the property the character determined by the parties. In this connection the decision of the court in the case of Standard Oil Co. of New York vs. Jaramillo ([1923], 44 Phil., 630), whether obiter dicta or not, furnishes the key to such a situation. 2. Immobilization of machinery; when placed in plant by owner Machinery which is movable in its nature only becomes immobilized when placed in a plant by the owner of the property or plant, but not when so placed by a tenant, a usufructuary, or any person having only a temporary right, unless such person acted as the agent of the owner. The distinction rests upon the fact that one only having a temporary right to the possession or enjoyment of property is not presumed by the law to have applied movable property belonging to him so as to deprive him of it by causing it by an act of immobilization to become the property of another. 3. Concrete immobilization of lessees machinery only if lease stipulates transfer of ownership on its termination Concrete immobilization takes place because of the express provisions of the lease which requires the putting in of improved machinery, deprived the tenant of any right to charge against the lessor the cost of such machinery, and it was expressly stipulated that the machinery so put in should become a part of the plant belonging to the owner without compensation to the lessee. Under such conditions the tenant in putting in the machinery was acting but as the agent of the owner in compliance with the obligations resting upon him, and the immobilization of the machinery which resulted arose in legal effect from the act of the owner in giving by contract a permanent destination to the machinery. (Valdes v. Altagracia) Peoples Bank v. Dahican Lumber [G.R. No. L-17500. May 16, 1967.] En Banc, Dizon (J): 8 concur. Facts: On 8 September 1948, Atlantic Gulf & Pacific Company of Manila (ATLANTIC) sold and assigned all its right in the Dahican lumber concession to Dahican Lumber Company (DALCO) for P500,000.00 of which only the amount of $50,000.00 was paid. Thereafter, to develop the concession, DALCO obtained various loans from the People's Bank (P200,000.00, as of 13 July 1950). DALCO also obtained, through the Bank, a loan of $250,000.00 from the Export-Import Bank of Washington D.C.

(through 5 promissory notes of $50,000.00 each, maturing on different dates), executed by both DALCO and the Dahican American Lumber Corporation (DAMCO), a foreign corporation and a stockholder of DALCO, all payable to the BANK. As security, DALCO executed on 13 July 1950 a deed of mortgage covering live parcels of land situated in the province of Camarines Norte, together with all the buildings and other improvements existing thereon and all the personal properties of the mortgagor located in its place of business in the municipalities of Mambulao and Capalonga, Camarines Norte in favor of the BANK. On the same date, DALCO executed a second mortgage on the same properties in favor of ATLANTIC to secure payment of the unpaid balance of the sale price of the lumber concession amounting to the sum of $450,000.00. Both deeds contained a provision extending the mortgage lien to properties to be subsequently acquired by the mortgagor. Both mortgages were registered in the Office of the Register of Deeds of Camarines Norte. In addition thereto DALCO and DAMCO pledged to the BANK 7,296 shares of stock of DALCO and 9,286 shares of DAMCO to secure the same obligations. Upon DALCO's and DAMCO's failure to pay the fifth promissory note upon its maturity, the BANK paid the same to the Export-Import Bank of Washington D.C. and the latter assigned to the former its credit and the first mortgage securing it. Subsequently, the BANK gave DALCO and DAMCO up to 1 April 1953 to pay the overdue promissory note. After 13 July 1950, DALCO purchased various machineries, equipment, spare parts and supplies in addition to, or in replacement of some of those already owned and used by it. Pursuant to the provision of the mortgage deeds quoted heretofore regarding "after acquired properties", the BANK requested DALCO to submit complete lists of said properties but the latter failed to do so. In connection with these purchases, there appeared in the books of DALCO the sum of P452,860.55 as due to Connell Bros. Company (CONNEL) and the sum of P2,151,678.34 to DAMCO. On 16 December 1952, the Board of Directors of DALCO passed a resolution agreeing to rescind the alleged sales of equipment, spare parts and supplies by CONNELL and DAMCO to it. Thereafter, the corresponding agreements of rescission of sale were executed. On 23 January 1953, the BANK, in its own behalf and that of ATLANTIC, demanded that the agreements be cancelled but CONNELL and DAMCO refused to do so. As a result, on 12 February 1953, ATLANTIC and the BANK, commenced foreclosure proceedings in the CFI Camarines Norte against DALCO and DAMCO. Upon motion of the parties, the Court, on 30 September 1953, issued an order transferring the venue of the action to the CFI Manila (Civil Case 20987). On 30 August 1958, upon motion of all the parties, the Court ordered the sale of all the machineries, equipment and supplies of DALCO, and the same were subsequently sold for a total consideration of P175,000.00 which was deposited in court pending final determination of the action. By a similar agreement one half (P87,500.00) of this amount was considered as representing the proceeds obtained from the sale of the "undebated properties" (those not claimed by DAMCO and CONNELL), and the other half as representing those obtained from the sale of the "after acquired properties". After due trial, the Court, on 15 July 1960, rendered Judgment against Dahican Lumber Co, ordering it to pay the bank, ATLANTIC, CONNEL and DAMCO; and adjudicating the half of the sum realized from sale of the properties unto the plaintiffs; and the other half to both the plaintiffs and defendant DAMCO and CONNELL. The Court further ordered that the cost of the receivership shall be borne by the Bank, Atlantic, Connell and DAMCO. On the following day, the Court issued a supplementary decision ordering the sale at public auction of lands object to the mortgages if DALCO fails to pay the bank and Atlantic. Therafter, all the parties appealed. The Supreme Court modified the appealed judgment and affirmed in all other respects. With costs. 1. On after acquired properties, stipulation is clear and valid The stipulation in the Deed of Mortgage states that all property of every nature, buildings, machinery, etc. taken in exchange or replacement by the mortgagor "shall immediately be and become subject to the lien of this mortgage in the same manner and to the same extent as if now included therein". No clearer language could have been chosen. Further, such stipulation is common is neither unlawful nor immoral, its obvious purpose being to maintain, to the extent allowed by circumstances, the original value of the properties given as security. It is only logical in all cases where the properties given as collateral are perishable or subject to inevitable wear and tear or were intended to be sold, or to be used (thus subject to wear and tear); they shall be replaced with others to be thereafter acquired by the mortgagor, such understanding either express or implied. If such properties were of the nature already referred to, it would be poor judgment on the part of the creditor who does not see to it that a similar provision is included in the contract. 2. Registry of chattel mortgage as to affect third party does not apply to the case Though it is the law in this jurisdiction that, to affect third persons, a chattel mortgage must be registered and must describe the mortgaged chattels or personal properties sufficiently to enable the parties and any other person to identify them, such law does not apply to the present case (?reason). 3. Whether old or new civil code to be used as guide irrelevant in this case The mortgages in question were executed on 13 July 1950 with the old Civil Code still in force. There can be no doubt that the provisions of said code must govern their interpretation and the question of their validity. It happens, however, that Articles 334 and 1877 of the old Civil Code are substantially reproduced in Article 415 and 2127, respectively, of the new Civil Code. It is, therefore, immaterial in this case whether we take the former or the latter as guide in deciding the point under consideration. 4. Civil code does not define real property but enumerates them Article 415 does not define real property but enumerates what are considered as such, among them being machinery, receptacles, instruments or replacements intended by the owner of the tenement for an industry or works which may be carried on in a building or on a piece of land, and shall tend directly to meet the needs of the said industry or works. 5. Chattels place in real properties mortgaged within the operation of Art 415 (5), and 2127 of the New Civil Code; Illustrative cases: Berkenkotter v. Cu Unjieng and Cu Unjieng Hijos v. Mabalacat Sugar The chattels, placed in the real properties mortgaged to plaintiffs, came within the operation of Art. 415, paragraph 5 and Art. 2127 of the new Civil Code. This is consistent with previous rulings of the court. (1) In Berkenkotter vs. Cu Unjieng (61 Phil. 663), Article 334 [5] of the old Civil Code gives the character of real property to machinery, liquid containers, instruments or

replacements intended by the owner of any building or land for use in connection with any industry or trade being carried on therein and which are expressly adapted to meet the requirements of such trade or industry; and (2) In Cu Unjieng Hijos vs. Mabalacat Sugar Co. (58 Phil. 439), a mortgage constituted on a sugar central includes not only the land on which it is built but also the buildings, machinery and accessories installed at the time the mortgage was constituted as well as the buildings, machinery and accessories belonging to the mortgagor, installed after the constitution thereof. 6. After acquired properties immobilized; No need for second registration as Chattel Mortgage in light of existing real estate mortgage The "after acquired properties" were purchased in connection with, and for use in the development of its lumber concession and that they were purchased in addition to, or in replacement of those already existing in the premises on 13 July 1950. In law, they must be deemed to have been immobilized, with the result that the real estate mortgages involved therein, which were registered as such, did not have to be registered a second time as chattel mortgages in order to bind the "after acquired properties" and affect third parties. 7. Davao Sawmill case not on all fours; Intent of parties relevant The facts in the Davao Sawmill case are not on all fours with the current case. In the former, Davao Sawmill had repeatedly treated the machinery therein involved as personal property by executing chattel mortgages thereon in favor of third parties, while in the present case the parties had treated the "after acquired properties" as real properties by expressly and unequivocally agreeing that they shall automatically become subject to the lien of the real estate mortgages executed by them. In the Davao Sawmill decision it was stated that the characterization of the property as chattels by the appellant is indicative of intention and impresses upon the property the character determined by the parties. In the present case, the characterization of the "after acquired properties" as real property was made not only by one but by both interested parties. There is, therefore, more reason to hold that such consensus impresses upon the properties the character determined by the parties who must now be held in estoppel to question it. 8. Valdez v. Central Altagracia relevant; Knowledge of suppliers of real mortgage stipulations Valdez vs. Central Altagracia Inc. (225 U.S. 58) was quoted in the Davao Sawmill case, holding that machinery placed on property by a tenant does not become immobilized, yet, when the tenant places it there pursuant to contract that it shall belong to the owner, it then becomes immobilized as to that tenant and even as against his assignees and creditors who had sufficient notice of such stipulation. In the present case, the "after acquired properties" are not disputed to be placed on and be used in the development of its lumber concession, and agreed further that the same shall become immediately subject to the lien constituted by the questioned mortgages. DAMCO and CONNELL, having full notice of such stipulation, must be deemed barred from denying that the properties in question had become immobilized. 9. Connell and Damco not unpaid suppliers; They do not have superior right to lien DAMCO was a stockholder and CONNELL was not only a stockholder but the general agent of DALCO. Further, the report of the auditors and its annexes show that neither DAMCO nor CONNELL had supplied any of the goods of which they respectively claimed to be the unpaid seller; that all items were supplied by different parties. If DALCO still owes them any amount in this connection, as financiers, they can not claim any right over the "after acquired properties" superior to the lien constituted thereon by virtue of the deeds of mortgage under foreclosure. 10. Defense of prematurity does not hold if debtor is insolvent When the law permits the debtor to enjoy the benefits of the period notwithstanding that he is insolvent by his giving a guaranty for the debt, that must mean a new and efficient guaranty, must concede that the causes of action for collection of the notes were not premature. In the present case, plaintiffs, as a matter of grace, conceded an extension of time to pay up to 1 April 1953, months after the action to foreclose was filed 12 February 1953; without debate that DALCO was already insolvent at the date of the time of filing.g 11. Proceeds from the sale of after acquired properties and the undebated properties awarded exclusively to plaintiff In view of the Courts opinion sustaining the validity of the mortgages in relation thereto, that proceeds obtained from the sale of the "after acquired properties" and the "undebated properties" should be awarded exclusively to the plaintiffs in payment of the money obligations secured by the mortgages under foreclosure. 12. Claims for damages supported The law (Articles 1313 and 1314 of the New Civil Code) provides that creditors are protected in cases of contracts intended to defraud them, and that any third person who induces another to violate his contract shall be liable for damages to the other contracting party. Similar liability is demandable under Arts. 20 and 21 which may be given retroactive effect (Arts. 2252-53) or under Arts. 1902 and 2176 of the Old Civil Code. In the present case, the facts clearly show that DALCO and DAMCO, after failing to pay the fifth promissory note upon its maturity, conspired jointly with CONNELL to violate the provisions of the fourth paragraph of the mortgages under foreclosure by attempting to defeat plaintiffs' mortgage lien on the "after acquired properties". 13. Expenses of receivership considered; other damages cannot be computed Considering that the sale of the real properties subject to the mortgages under foreclosure has not been effected, and considering further the lack of evidence showing that the true value of all the properties already sold was not realized because their sale was under stress, The true elements or factors that should determine the amount of damages that plaintiffs are entitled to recover from defendants are not present. However, all the expenses of the Receivership, which was deemed necessary to safeguard the rights of the plaintiffs, should be borne by all the defendants, jointly and severally, in the same manner that all of them should pay to the plaintiffs, jointly and severally, the attorney's fees awarded in the appealed judgment. Burgos v. Chief of Staff, AFP [G.R. No. 64261. December 26, 1984.] En Banc, Escolin (J): 10 concur, 1 took no part

Facts: On 7 December 1982, Judge Ernani Cruz-Pao, Executive Judge of the then CFI Rizal [Quezon City], issued 2 search warrants where the premises at 19, Road 3, Project 6, Quezon City, and 784 Units C & D, RMS Building, Quezon Avenue, Quezon City, business addresses of the "Metropolitan Mail" and "We Forum" newspapers, respectively, were searched, and office and printing machines, equipment, paraphernalia, motor vehicles and other articles used in the printing, publication and distribution of the said newspapers, as well as numerous papers, documents, books and other written literature alleged to be in the possession and control of Jose Burgos, Jr. publisher-editor of the "We Forum" newspaper, were seized. A petition for certiorari, prohibition and mandamus with preliminary mandatory and prohibitory injunction was filed after 6 months following the raid to question the validity of said search warrants, and to enjoin the Judge Advocate General of the AFP, the city fiscal of Quezon City, et.al. from using the articles seized as evidence in Criminal Case Q-022782 of the RTC Quezon City (People v. Burgos). The prayer of preliminary prohibitory injunction was rendered moot and academic when, on 7 July 1983, the Solicitor General manifested that said articles would not be used until final resolution of the legality of the seizure of said articles. The Supreme Court declared the search warrants 20-82(a,b) issued on 7 December 1982 null and void, and granted the writ of mandatory injunction for the return of the seized articles, such articles seized ordered released to the petitioners; without costs. 1. Urgency of constitutional issue raised overrides procedural flaw; Failure to file motion to quash Petitioners, before impugning the validity of the warrants before the Court, should have filed a motion to quash said warrants in the court that issued them. But this procedural flaw notwithstanding, the Court take cognizance of this petition in view of the seriousness and urgency of the constitutional issues raised, not to mention the public interest generated by the search of the "We Forum" offices, which was televised and widely publicized in all metropolitan dailies. The existence of this special circumstance justifies this Court to exercise its inherent power to suspend its rules. In Vda. de Ordoveza v. Raymundo, it was said that "it is always in the power of the court [Supreme Court] to suspend its rules or to except a particular case from its operation, whenever the purposes of justice require it . . . " 2. Laches defined Laches is failure or negligence for an unreasonable and unexplained length of time to do that which, by exercising due diligence, could or should have been done earlier. It is negligence or omission to assert a right within a reasonable time, warranting a presumption that the party entitled to assert it either has abandoned or declined to assert it. 3. Extrajudicial effort negates presumption of abandonment of right to the possession of property Extrajudicial efforts exerted by petitioners negate the presumption that they had abandoned their right to the possession of the seized property, thereby refuting the charge of laches against them. Although the reason given by petitioners may not be flattering to the judicial system, The Court finds no ground to punish or chastise them for an error in judgment in pursuing other remedies, relying on the so-called executive benevolence or largesse (e.g. Letter of presidential friend Fiscal Flaminiano to Col. Balbino Diego, Chief Intelligence and Legal Officer of the PSG). 4. Documents marked as evidence in criminal case do not affect issue on the validity of the warrants The documents seized lawfully belong to Jose Burgos, Jr. and he can do whatever he pleases with them, within legal bounds; such as marking them as evidence in the criminal case. The fact that he has used them as evidence does not and cannot in any way affect the validity or invalidity of the search warrants assailed in the petition. 5. Examination conducted Petitioners objection that there is an alleged failure to conduct an examination under oath or affirmation of the applicant and his witnesses, as mandated by the Constitution as well as Sec. 4, Rule 126 of the Rules of Court, may properly be considered moot and academic, as petitioners themselves conceded during the hearing on 9 August 1983, that an examination had indeed been conducted by the judge of Col. Abadilla and his witnesses. 6. Defect on the address where items are to be seized is typographical error Search Warrants No. 20-82[a] and No. 20-82[b] were used to search two distinct places: No. 19, Road 3, Project 6, Quezon City and 784 Units C & D, RMS Building, Quezon Avenue, Quezon City, respectively. Objection is interposed to the execution of Search Warrant No. 20-82[b] at the latter address on the ground that the two search warrants pinpointed only one place where petitioner Jose Burgos, Jr. was allegedly keeping and concealing the articles listed therein, i.e., No. 19, Road 3, Project 6, Quezon City. The defect pointed out is obviously a typographical error. Precisely, two search warrants were applied for and issued because the purpose and intent were to search two distinct premises. It would be quite absurd and illogical for respondent judge to have issued two warrants intended for one and the same place. 7. Determination whether warrant describes premises to be search with sufficient particularity In the determination of whether a search warrant describes the premises to be searched with sufficient particularity, the executing officer's prior knowledge as to the place intended in the warrant is relevant. This would seem to be especially true where the executing officer is the affiant on whose affidavit the warrant had issued, and when he knows that the judge who issued the warrant intended the building described in the affidavit. And it has also been said that the executing officer may look to the affidavit in the official court file to resolve an ambiguity in the warrant as to the place to be searched. 8. Property seized need not be owned by person against whom the warrant is directed Section 2, Rule 126 of the Rules of Court, enumerates the personal properties that may be seized under a search warrant, such as [a] Property subject of the offense; [b] Property stolen or embezzled and other proceeds or fruits of the offense; and [c] Property used or intended to be used as the means of committing an offense. The rule does not require that the property to be seized should be owned by the person against whom the search warrant is directed. It may or may not be owned by him. Ownership is of no consequence, and it is sufficient that the person against whom the warrant is directed has control or possession of the property sought to be seized was alleged to have in relation to the articles and property seized under the warrants.

9. Machineries remain to be movable property; owner should place the machinery to be immovable Under Article 415[5] of the Civil Code of the Philippines, "machinery, receptables, instruments or implements intended by the owner of the tenement for an industry or works which may be carried on in a building or on a piece of land and which tend directly to meet the needs of the said industry or works" are considered immovable property. In Davao Sawmill Co. v. Castillo, it was said that machinery which is movable by nature becomes immobilized when placed by the owner of the tenement, property or plant, but not so when placed by a tenant, usufructuary, or any other person having only a temporary right, unless such person acted as the agent of the owner. In the present case, petitioners do not claim to be the owners of the land and/or building on which the machineries were placed. The machineries, while in fact bolted to the ground, remain movable property susceptible to seizure under a search warrant. 10. Article IV, Section 3 of the 1973 Constitution; Finding of probable cause required to issue warrant Section 3 provides that no search warrant or warrant of arrest shall issue except upon probable cause to be determined by the judge, or such other responsible officer as may be authorized by law, after examination under oath or affirmation of the complainant and the witnesses he may produce, and particularly describing the place to be searched and the persons or things to be seized. In the present case, a statement in the effect that the petitioner "is in possession or has in his control printing equipment and other paraphernalia, news publications and other documents which were used and are all continuously being used as a means of committing the offense of subversion punishable under PD 885, as amended" is a mere conclusion of law and does not satisfy the requirements of probable cause. Bereft of such particulars as would justify a finding of the existence of probable cause, said allegation cannot serve as basis for the issuance of a search warrant. 11. Probable cause for search defined; Application to searches against newspaper publisher/editor Probable cause for a search is defined as such facts and circumstances which would lead a reasonably discreet and prudent man to believe that an offense has been committed and that the objects sought in connection with the offense are in the place sought to be searched. When the search warrant applied for is directed against a newspaper publisher or editor in connection with the publication of subversive materials, the application and/or its supporting affidavits must contain a specification, stating with particularity the alleged subversive material he has published or is intending to publish. Mere generalization will not suffice. 12. Constitution requires personal knowledge of complainant / witnesses to justify issuance of warrant In mandating that "no warrant shall issue except upon probable cause to be determined by the judge, after examination under oath or affirmation of the complainant and the witnesses he may produce; the Constitution requires no less than personal knowledge by the complainant or his witnesses of the facts upon which the issuance of a search warrant may be justified. In Alvarez v. CFI, it was ruled that "the oath required must refer to the truth of the facts within the personal knowledge of the petitioner or his witnesses, because the purpose thereof is to convince the committing magistrate, not the individual making the affidavit and seeking the issuance of the warrant, of the existence of probable cause." 13. General warrants invalid In Standford v. State of Texas, the search warrant which authorized the search for 'books, records, pamphlets, cards, receipts, lists, memoranda, pictures, recordings and other written instruments concerning the Communist Parties of Texas, and the operations of the Communist Party in Texas," was declared void by the U.S. Supreme Court for being too general. It is that it is not the policy of the government to suppress any newspaper or publication that speaks with "the voice of non-conformity" but poses no clear and imminent danger to state security. 14. Closure of publications in the nature of censorship abhorrent to the freedom of the press As a consequence of the search and seizure, these premises of the Metropolitan Mail and We Forum were padlocked and sealed, with the further result that the printing and publication of said newspapers were discontinued. Such closure is in the nature of previous restraint or censorship abhorrent to the freedom of the press guaranteed under the fundamental law, and constitutes a virtual denial of petitioners' freedom to express themselves in print. Thus state of being is patently anathematic to a democratic framework where a free, alert and even militant press is essential for the political enlightenment and growth of the citizenry. 15. Machines not sequestered under PD 885; Lack of IRR, contrary claim by Marcos and Romulo Sequestration under Section 8 of PD 885, as amended, which authorizes "the sequestration of the property of any person, natural or artificial, engaged in subversive activities against the government and its duly constituted authorities in accordance with implementing rules and regulations as may be issued by the Secretary of National Defense" could not validly be effected in view of the absence of any implementing rules and regulations promulgated by the Minister of National Defense. Further, no less than President Marcos himself denied the request of the military authorities to sequester the property seized as reported in the 10 December 1982 issue of the Daily Express. This was confirmed by Foreign Minister Carlos P. Romulo on 10 February 1982, reiterating Marcos claims, in his letter to US Congressman Tony P. Hall. GSIS v. Calson [G.R. No. L-19867. May 29, 1968.] En Banc, Makalintal (J): 7 concur, 1 on official leave Facts: On 11 April 1957, Calsons Inc. applied for a loan of P2M to appellee to pay the balance of the purchase price of certain parcels of land situated at the corner of Globo de Oro and Elizondo Streets, Quiapo, Manila, and to finance the construction of a 2-story textile market building on said land. The application was approved by appellee's Board of Trustees on 26 August 1957. In connection with said loan appellants executed on 31 October 1957 a promissory note binding themselves jointly and severally to pay appellee the sum of P2M, with interest at the rate of 7% per annum compounded monthly, in 120 equal monthly installments of P23,221.69 each; the first of such due and payable beginning the month following the last release or the month following the expiration of the period for the construction of the building (or within 12 months), whichever is earlier. It was also stipulated that the properties should be free from all liens and encumbrances other than the mortgage itself. The first release in the amount of P819,000.00 was made on 7 November 1957, while the second (and last) release in the amount of P30,000.00 was made on 15 May 1958. The checks covering both releases were drawn in favor of the vendor of the mortgaged properties. In accordance with

the agreement between the parties, the old building standing on the mortgaged properties was insured for P300,000.00 on 1 December 1959. Appellee advanced the sum of P5,628,00 for the annual premium, but appellants failed to reimburse the same. Appellee filed a complaint for the foreclosure of the mortgage with the CFI Manila on 11 August 1958, alleging a number of violations of the mortgage contract, to wit: (1) that the mortgaged properties had not been freed by the mortgagor from certain liens and encumbrances other than the mortgage itself; (2) that without the prior written consent of plaintiff defendants removed and disposed of the complete band sawmill and filing machine which formed part of the properties mortgaged; (3) that Calsons, Inc., failed to submit to appellee evidence showing the reduction of defendant's account on the lot to at least P819,000.00; (4) and that. Calsons, Inc., failed to begin, much less complete, the construction of the supermarket building on the mortgaged properties. On August 11, 1959, plaintiff filed supplemental complaint, which was admitted without opposition. Two additional grounds for the foreclosure of the mortgage were alleged, namely: (1) that defendants failed, despite demands therefor, to pay the amortizations due and payable, including accrued interest and surcharges, on the portion of the loan released to them; and (2) that defendants failed to complete the construction of the textile market building on the mortgaged properties within 12 months from 7 November 1957, the date of the first release of P819,000.00. Judgment was rendered on 3 March 1962 in favor of plaintiff, and defendants brought the appeal directly to the Court in view of the amount involved. The Supreme Court affirmed the judgment appealed from, with costs against appellants. 1. Vendors lien a legal encumbrance, which is effective even if not recorded Even if the two certificates of title covering the mortgaged property do not show any lien or encumbrance thereon other than the mortgage itself; the vendor's lien in favor of the former owners, representing the unpaid balance of P280,000.00 on the purchase price of the lots mortgaged. The lien is a legal encumbrance and therefore effective although not recorded. 2. Appellee not estopped in invoking right to have properties free from vendors lien One of the reasons why appellant Calsons, Inc., applied for the P2M loan was precisely to use part thereof to pay the balance of the purchase price of 5 parcels of land it mortgaged to appellee. And to assure itself that no vendor's lien attached to the said properties, appellee caused the additional conditions to be added to the original terms of the mortgage contract. It turns out in fact that appellants had failed to reduce their account on the lot to P819,000.00, as stipulated in the mortgage contract, since there was still a balance of P280,000 on the purchase price. With respect to the second release of P30,000.00, the check was also drawn in favor of the vendor with the understanding that it would be used to pay the real estate taxes due on said properties and thus remove the corresponding tax lien imposed by law. The steps taken by appellee negate any inference that it agreed to waive its right to have the properties "free from all liens and encumbrances," as provided in the mortgage contract. 3. Appellee cannot be estopped by a commitment made by its agent not reflected in the Agreement Estoppel is invoked by appellants on the basis of a letter dated 28 October 1957, sent by the Manager of appellee's Real Estate Department to the vendor of the properties, to the effect that the balance of the purchase price in the amount of P280,000.00 would be released within six (6) months from the date of the said letter. The commitment of said Manager was not recognized by the Board of Trustees of the appellee as shown by the fact that it was not incorporated in the mortgage contract, which was executed on a later date, 31 October 1957. While the schedule of subsequent releases was clearly defined in the mortgage contract, no mention was made about the said commitment. 4. Machineries are immovables and are included in mortgage; installed by the owner to meet demands of industry or works The mortgage was on the lands "together with all the buildings and improvements now existing or which may hereafter be constructed" thereon. And the machineries were permanently attached to the property, and installed there by the former owner to meet the needs of certain works or industry therein. They were therefore part of immovable pursuant to Article 415 of the Civil Code, and need not be the subject of a separate chattel mortgage in order to be deemed duly encumbered in favor of appellee. 5. Promissory note provides for due date of installments; Failure to pay amortizations a violation of mortgage contract The promissory note executed by the parties clearly provides when the first installment, as well as subsequent ones, would become due, i.e. beginning the month following the last release and/or the month following the expiration of the period for the construction of the textile market building, whichever is earlier and the rest on the 7th day of every month thereafter until the principal of P2M and the interest shall have been fully paid. The mortgage contract provides that the proposed building should be completed within 12 months from the date of the first release. Said release having been made on 7 November 1957, the construction period expired on 7 November 1958; hence, the first installment became due one month thereafter or on 7 December 1958, and the rest on the 7th day of every month thereafter. Appellants' failure to pay the amortizations, interest and surcharges demanded of them by appellee, therefore, constitutes a violation of the mortgage contract and is sufficient ground for the foreclosure of the mortgage. Sergs Products v. PCI Leasing [G.R. No. 137705. August 22, 2000.] Third division, Panganiban (J): 3 concur Facts: On 13 February 1998, PCI Leasing and Finance, Inc. filed a complaint for sum of money, with an application for a writ of replevin (Civil Case Q-98-33500). On 6 March 1998, upon an ex-parte application of PCI Leasing, judge issued a writ of replevin directing its sheriff to seize and deliver the machineries and equipment to PCI Leasing after 5 days and upon the payment of the necessary expenses. On 24 March 1998, the sheriff proceeded to petitioner's factory, seized one machinery with word that the return for the other machineries. On 25 March 1998, petitioners filed a motion for special protective order, invoking the power of the court to control the conduct of its officers and amend and control its processes, praying for a directive for the sheriff to defer enforcement of the writ of replevin. On 6 April 1998, the sheriff again sought to enforce the writ of seizure and take possession of the remaining properties. He was able to take two more, but was prevented by the workers from taking the rest. On 7 April 1998, they went to the CA via an original action for certiorari.

Citing the Agreement of the parties, the appellate court held that the subject machines were personal property, and that they had only been leased, not owned, by petitioners; and ruled that the "words of the contract are clear and leave no doubt upon the true intention of the contracting parties." It thus affirmed the 18 February 1998 Order, and the 31 March 1998 Resolution of the lower court, and lifted the preliminary injunction issued on 15 June 1998. A subsequent motion for reconsideration was denied on 26 February 1999. Hence, the petition for review on certiorari. The Supreme Court denied the petition and affirmed the decision of the Court of Appeals; with costs against petitioners. 1. Petition for review on certiorari is clearly under Rule 45 The petition need not expressly indicate if it is being filed under Rule 45 or Rule 65 of the Rules of Court, as it is clear that the present recourse is under Rule 45; the conclusion of such supported by the title of the Petition, which is "Petition for Review on Certiorari." 2. Error in impleading the Judge as respondent not ground to dismiss the case While the judge should not have been impleaded as a respondent, substantial justice requires that such lapse by itself should not warrant the dismissal of the present Petition. The Court may deems it proper to remove, motu proprio, the name of the Judge from the caption of the case. 3. Writ of replevin issued for recovery of personal property Rule 60 of the Rules of Court provides that writs of replevin are issued for the recovery of personal property only. Section 3 provides that upon the filing of such affidavit and approval of the bonds the court shall issue an order and the corresponding writ of replevin describing the personal property alleged to be wrongfully detained and requiring the sheriff forthwith to take such property into his custody. 4. Machinery immovable properties by incorporation The machinery were essential and principal elements of their chocolate-making industry. Hence, although each of them was movable or personal property on its own, all of them have become "immobilized by destination because they are essential and principal elements in the industry." The machines are thus, real, not personal, property pursuant to Article 415 (5) of the Civil Code. 5. Parties estopped when parties stipulated properties as personal; property thus subject to writ of seizure Contracting parties may validly stipulate that a real property be considered as personal. After agreeing to such stipulation, they are consequently estopped from claiming otherwise. Under the principle of estoppel, a party to a contract is ordinarily precluded from denying the truth of any material fact found therein. Thus, said machines are proper subjects of the Writ of Seizure (compare Tumalad v. Vicencio). 6. Similar cases In Trinidad v. Vicencio, the Court upheld the intention of the parties to treat a house of strong materials as a personal property because it had been made the subject of a chattel mortgage. Applying Tumalad, the Court in Makati Leasing and Finance Corp. v. Wearever Textile Mills also held that the machinery used in a factory and essential to the industry was a proper subject of a writ of replevin because it was treated as personal property in a contract. 7. Third parties acting in good faith not affected by stipulation to consider real property as personal The holding that the machines should be deemed personal property pursuant to the Lease Agreement is good only insofar as the contracting parties are concerned. Hence, while the parties are bound by the Agreement, third persons acting in good faith are not affected by its stipulation characterizing the subject machinery as personal. In the present case, however, there is no showing that any specific third party would be adversely affected. 8. Title to property should be determined at trial; Remedies under Rule 60 either to post a counter-bond or to question the sufficiency of the plaintiffs bond The validity and the nature of the contract are the lis mota of the civil action pending before the RTC. A resolution of the questions whether the Agreement is a loan and not a lease, or whether the Agreement is invalid, therefore, is effectively a resolution of the merits of the case. Hence, they should be threshed out in the trial, not in the proceedings involving the issuance of the Writ of Seizure. As held in La Tondea Distillers v. CA, the Court explained that the policy under Rule 60 was that questions involving title to the subject property should be determined in the trial. In that case, the Court noted that the remedy of defendants under Rule 60 was either to post a counter-bond or to question the sufficiency of the plaintiff's bond. They were not allowed, however, to invoke the title to the subject property. 9. Title to property should be determined at trial; no place in a petition for certiorari under Rule 65 or in a petition for review under Rule 45. The questions whether the Agreement is a loan and not a lease, or whether the Agreement is invalid require a determination of facts and a presentation of evidence, both of which have no place in a petition for certiorari in the CA under Rule 65 or in a petition for review in the Court under Rule 45. 10. Agreement presumed to be valid and binding The Agreement must be presumed to be valid and binding as the law between the parties; as there is nothing on record to show that it has been nullified or annulled. In the present case, petitioners assailed it first only in the RTC proceedings, which had ironically been instituted by respondent. As in the Makati Leasing and Finance case, even granting that he charge is true, such fact alone does not render a contract void ab initio, but can only be a ground for rendering said contract voidable, or annullable pursuant to Article 1390 of the new Civil Code, by a proper action in court.

11. Consequences cannot be blamed on the Court for the petitioners failure to avail of remedies under Section 5, Rule 60 of the Rules of Court Petitioners' arguments, that the seizure will lead to the unemployment of their workers and nullify all efforts to rehabilitate the corporation, do not preclude the implementation of the Writ. Law and jurisprudence support its propriety. Such consequences should not be blamed on this Court, but on the petitioners for failing to avail themselves of the remedy under Section 5 of Rule 60, which allows the filing of a counter-bond. Makati Leasing v. Wearever Textiles [G.R. No. L-58469. May 16, 1983.] Second Division, de Castro (J): 5 concur, 1 concur in result Facts: To obtain financial accommodations from Makati Leasing and Finance Corporation, Wearever Textile Mills, discounted and assigned several receivables with the former under a Receivable Purchase Agreement. To secure the collection of the receivables assigned, Wearever Textile executed a Chattel Mortgage over certain raw materials inventory as well as a machinery described as an Artos Aero Dryer Stentering Range. Upon Wearever's default, Makati Leasing filed a petition for extrajudicial foreclosure of the properties mortgage to it. However, the Deputy Sheriff assigned to implement the foreclosure failed to gain entry into Wearever's premises and was not able to effect the seizure of the machinery. Makati Leasing thereafter filed a complaint for judicial foreclosure with the CFI Rizal (Branch VI, Civil Case 36040). Acting on petitioner's application for replevin, the lower court issued a writ of seizure, the enforcement of which was restrained upon Wearever's filing of a motion for reconsideration. The lower court finally issued on 11 February 1981, an order lifting the restraining order for the enforcement of the writ of seizure and an order to break open the premises of Wearever to enforce said writ. The lower court reaffirmed its stand upon Wearever's filing of a further motion for reconsideration. On 13 July 1981, the sheriff enforcing the seizure order, repaired to the premises of Wearever and removed the main drive motor of the subject machinery. On 27 August 1981, the Court of Appeals, in certiorari and prohibition proceedings filed by Wearever, set aside the Orders of the lower court and ordered the return of the drive motor seized by the sheriff pursuant to said Orders, after ruling that the machinery in suit cannot be the subject of replevin, much less of a chattel mortgage, because it is a real property pursuant to Article 415 of the new Civil Code. The appellate court also rejected the argument that Wearever is estopped from claiming that the machine is real property by constituting a chattel mortgage thereon. A motion for reconsideration was filed by Makati Leasing, which was later denied. Makati Leasing brought the case to the Supreme Court by review by writ of certiorari. The Supreme Court reversed and set aside the decision and resolution of the Court of Appeals, and reinstated the orders of the lower court, with costs against Wearever Textiles. 1. Case not moot and academic with the return of the seized motor When the subject motor drive was returned, it was made unequivocably clear that said action was without prejudice to a motion for reconsideration of the Court of Appeals decision, as shown by the receipt duly signed by Wearever's representative. Considering that Makati Leasing has reserved its right to question the propriety of the CA' decision, the contention of Wearever that the petition has been mooted by such return may not be sustained. 2. Similar case; Estoppel applies to parties as having treated the house as personalty Although there is no specific statement referring to the subject house as personal property, yet by ceding, selling or transferring a property by way of chattel mortgage defendants-appellants could only have meant to convey the house as chattel, or at least, intended to treat the same as such, so that they should not now be allowed to make an inconsistent stand by claiming otherwise. Unlike in the Iya cases, Lopez vs. Orosa, Jr. & Plaza Theatre, Inc. & Leung Yee vs. F.L. Strong Machinery & Williamson, wherein third persons assailed the validity of the chattel mortgage, it is the defendants-appellants themselves, as debtors mortgagors, who are attacking the validity of the chattel mortgage in this case. The doctrine of estoppel therefore applies to the herein defendants appellants, having treated the subject house as personalty" (Tumalad v. Vicencio). One who has so agreed is estopped from denying the existence of the chattel mortgage. 3. Pronouncement on estoppel involving chattel mortgage applies to machinery There is no logical justification to exclude the rule out the present case from the application of the pronouncement in Tumalad v. Vicencio. If a house of strong materials may be considered as personal property for purposes of executing a chattel mortgage thereon as long as the parties to the contract so agree and no innocent third party will be prejudiced thereby, there is absolutely no reason why a machinery, which is movable in its nature and becomes immobilized only by destination or purpose, may not be likewise treated as such. 4. Chattel mortgage treating real property as personal property valid, as long as third parties are not prejudiced The characterization of the subject machinery as chattel is indicative of intention and impresses upon the property the character determined by the parties. As stated in Standard Oil v. Jaramillo, , it is undeniable that the parties to a contract may by agreement treat as personal property that which by nature would be real property, as long as no interest of third parties would be prejudiced thereby. 5. Equity prevents respondent to impugn the efficacy of the chattel mortgage Equity dictates that one should not benefit at the expense of another. Weareverf could not be allowed to impugn the efficacy of the chattel mortgage after it has benefited therefrom. 6. Machinery and Engineering Supplies v. CA not applicable; Tumalad case more in parity with case The case of Machinery and Engineering Supplies, Inc. v. CA, 96 Phil. 70, heavily relied upon by the Court of Appeals, is not applicable to the present case as the nature of the machinery and equipment involved therein as real properties never having been

disputed nor in issue, and they were not the subject of a Chattel Mortgage. Undoubtedly, the Tumalad case bears more nearly perfect parity with the present case to be the more controlling jurisprudential authority. Hongkong & Shanghai Banking v. Aldecoa & Co. [G.R. No. 8437. March 23, 1915.] En Banc, Trent (J): 3 concur, 1 concurs in result, 1 dissents. Facts: The defendants, Joaquin, Zoilo, and Cecilia (all Ibaez de Aldecoa), were born in the Philippines on 27 March 1884, 4 July 1885, and 1887, respectively, the legitimate children of Zoilo Ibaez de Aldecoa and Isabel Palet; the parents being natives of Spain but domiciled in Manila. The firm of Aldecoa & Co., of which Zoilo Ibaez de Aldecoa, the father who died on 4 October 1895, had been a member and managing director, was reorganized in December 1896, and the widow became one of the general or "capitalistic" partners of the firm. The 3 children appear in the articles of agreement as industrial partners. The widow, retaining her Manila domicile, left the Philippines and went to Spain in 1897 due to her health, and did not return until the latter part of 1902. On 31 July 1903, Isabel Palet went before a notary public and executed two instruments, whereby she emancipated her sons, who were 18 years old at that time, with their consent and acceptance. After the execution of said instruments, both Joaquin and Zoilo participated in the management of Aldecoa & Co. as partners by being present and voting at meetings of the partners of the company upon matters connected with its affairs. On 23 February 1906, Aldecoa & Co. obtained from the bank a credit in account current up to the sum of P450,000 upon the terms and conditions set forth in the instrument Exhibit A. Later it was agreed that Isabel Palet and her sons should mortgage, in addition to certain securities of Aldecoa & Co. certain of their real properties as additional security for the obligations of Aldecoa & Co. Thus, on 23 March 1906, the mortgage, Exhibit B, was executed wherein certain corrections in the description of some of the real property mortgaged to the bank by Exhibit A were made and the amount for which each of the mortgaged properties should be liable was set forth. These two mortgages, Exhibits A and B, were duly recorded in the registry of property of the city of Manila on 23 March 1906. The real property mortgaged by Isabel Palet was at her instance, registered under the provisions of the Land Registration, the property subject to the mortgage in favor of the bank, by decree of the land court 8 March 1907; while another property, on 6 November 1906 and at the instance of Isabel Palet and her 3 children, was applied for registration (the undivided of said property being subject to the mortgage in favor of the bank), the application of which was granted by decree of the land court 8 September 1907. On 31 December 1906, Aldecoa & Co. went into liquidation on account of the expiration of the term for which it had been organized, and the intervener, Urquhart, was duly elected by the parties as liquidator, and by resolution dated 24 January 1907, he was granted the authority expressed in that resolution. Additional security for the performance of the obligation in favor of the bank under the terms of contracts Exhibit A and B were made on various dates. On 22 December 1906, Aldecoa & co. mortgaged to the bank the right of mortgage upon real property in the Province of Albay mortgage to it by one Zubeldia. On 31 Mach 1907, Aldecoa & Co., already in liquidation, mortgaged to the bank the right of mortgage upon real estate in the province of Ambos Camarines mortgage to it by one Andres Garchitorena (P20,280.19). On the same date, Aldecoa & Co. further mortgaged the right of mortgage upon real property in the same province mortgage to it by Tremoya Hermanos (P43,117.40) and Liborio Tremoya (P75,463.54). Further, on 30 January 1907, Aldecoa & Co. duly authorized the bank to collect from certain persons and firms any and all debts owing by them to Aldecoa & Co. and to apply all amounts so collected to the satisfaction, pro tanto, of any indebtedness of Aldecoa & Co. to the bank. On the other hand, as the result of the litigation between Aldecoa & Co. and A. S. Macleod, wherein the injunction bond of P50,000 was made by the bank upon the condition that any liability incurred on the part of the bank upon this injunction bond would be covered by the mortgage of 23 February 1906, Aldecoa & Co. became the owner, through a compromise agreement executed on 14 August 1907, of the shares of the Pasay Estate Company Limited, and on 30 August 1907, Urquhart, as liquidator mortgaged to the bank, by way of additional security for the performance of the obligations set forth in Exhibits A and B, the 312 shares of the Pasay Estate Company, Limited, acquired by Aldecoa & Co. On 18 February 1907, Aldecoa & Co. acknowledged an indebtedness of P154,689,20 each to Joaquin and Zoilo Ibaez de Aldecoa, and another P89,177.07 to Cecilia Ibaez de Aldecoa. On 30 September 1908, Joaquin, Zoilo, and Cecilia recovered a judgment in the CFI Manila for the payment of the balance of P155,127.31. On 30 November 1907, Joaquin, Zoilo, and Cecilia instituted an action in the CFI Manila against the bank for the purpose of obtaining a judicial declaration to the effect that the contract whereby Aldecoa & Co. mortgaged to the bank the shares of the Pasay Estate Company recovered from Alejandro S. Macleod, was null and void, and for a judgment that these shares be sold and applied to the satisfaction of their judgment obtained on 30 September 1908. Judgment was rendered by the lower court in favor of the children, but upon appeal the Supreme Court reversed that judgment and declared that the mortgage of the shares of stock in the Pasay Estate Co. to the bank was valid. In 1908, Joaquin, Zoilo, and Cecilia commenced an action against their mother, Isabel Palet, and Aldecoa & Co., in which the bank was not a party, and in September of that year procured a judgment of the CFI annulling the articles of copartnership of Aldecoa & Co., in so far as they were concerned, and decreeing that they were creditors and not partners of that firm. In October 1908, Joaquin and Zoilo instituted an action against the bank for the purpose of obtaining a judgment annulling the mortgages created by them upon their interest in the properties described in Exhibits A and B, upon the ground that the emancipation by their mother was void and of no effect, and that, therefore, they were minors incapable of creating a valid mortgage upon their real property. The CFI dismissed the complaint as to Joaquin upon the ground that he had ratified those

mortgages after becoming of age, but entered a judgment annulling said mortgages with respect to Zoilo. Both parties appealed from this decision and the case was given registry No. 6889 in the Supreme Court. On 31 January 1911, the Bank filed an action against the defendants for the purpose of recovering from Aldecoa & Co., an amount due from the latter as the balance to its debit in an account current with the Company, and to enforce the subsidiary liability of the other defendants for the payment of this indebtedness, as partners of the Company, and to foreclose certain mortgages executed by the defendants to secure the indebtedness sued upon. On 10 August 1912, judgment was entered in favor of the bank, ordering the defendants to pay the sum of P344,924.23 with interest of 7% per annum from date of judgment until fully paid, and the costs; and ordering the foreclosure of the mortgages. Judgment was also entered denying the relief sought by the intervener. All of the defendants and the intervener have appealed. The Supreme Court affirmed the judgment appealed from, and ordered the appellants whose appeals are determined to pay their respective portions of the cost. 1. Complaint not vague nor ambiguous The complaint alleges that a certain specific amount was due from the defendant firm as a balance of its indebtedness to the plaintiff, and this necessarily implies that there were no credits in favor of the defendant firm of any kind whatsoever which had not already been deducted from the original obligation. 2. No evidence supports claim that bank prejudiced Aldecoa by inducing the customers to cease commercial relations There is no evidence to show that there was any inducement made by the bank to prejudice Aldecoa & Co, for its customers to cease their commercial relations with Aldecoa & Co.. It may be possible that some of Aldecoa & Co.'s customers ceased doing business with that firm after it went into liquidation. This is the ordinary effect of a commercial firm going into liquidation. This is especially true for the reason that it was a well known fact that Aldecoa & Co. was insolvent. Furthermore, the bank was expressly empowered to take any steps which might be necessary, judicially or extrajudicially, for the collection of these credits. The real reason which caused the defendant's provincial customers to cease making shipments was due to the fact that the defendant, being out of funds, could not give its customers any further credit. It is therefore clear that the bank, having exercised the authority conferred upon it by the company in a legal manner, is not responsible for any damages which might have resulted from the failure of the defendant's provincial customers to continue doing business with that firm. 3. Court has jurisdiction as bank does not seek to exercise mortgage right on real properties in the provinces The bank is not seeking to exercise its mortgage rights upon the mortgages which the defendant firm holds upon certain real properties in the Provinces of Albay and Ambos Camarines and to sell these properties at public auction in these proceedings; nor does the judgment of the trial court directs that this be done. Before that property can be sold the original mortgagors will have to be made parties. The bank is not trying to foreclose any mortgages on real property executed by Aldecoa & Co. 4. Solidary obligation; Money judgment against the firm and foreclosure judgment against the others It is true that the bank sought and obtained a money judgment against that firm, and at the same time and in the same action obtained a foreclosure judgment against the other defendants. If two or more persons are in solidum the debtors of a third person, and one or more of such debtors mortgage any of their real property situate in the jurisdiction of the court, the creditor, in case his obligation is not paid at maturity, may include all of the solidary debtors in the same suit and secure a joint and several judgment against them, as well as judgments of foreclosure upon the respective mortgages. 5. Extensions does not extinguish the mortgages The contention that the extensions granted to Aldecoa & Co.'s debtors, with the consent and authority of that firm itself, has resulted in extinguishment of the mortgages created by Aldecoa & Co. or of the mortgages created by partners of that company to secure its liabilities to the bank, is untenable. The record shows that all the sureties were represented by Urquhart, the person elected by them as liquidator of the firm, when he agreed with the bank upon the extensions granted to those debtors. The authority to grant these extensions was conferred upon the bank by the liquidator, and he was given authority by all the sureties to authorize the bank to proceed in this manner. 6. Properties Isabel Palet mortgage were not security for performance of her solidary subsidiary obligation but part of the direct obligation of the firm itself Although the court recognized the subsidiary character of the personal liability of Doa Isabel Palet as a member of the firm of Aldecoa & Co. and decreed that as to any deficiency which might result after the sale of the mortgaged properties, execution should not issue against the properties of Doa Isabel Palet until all the property of Aldecoa & Co. shall have been exhausted. The properties mortgaged by Doa Isabel Palet were so mortgaged not merely as security for the performance of her own solidary subsidiary obligation as a partner bound for all the debts of Aldecoa & Co., but for the purpose of securing the direct obligation of the firm itself to the bank. 7. Isabel Palet a personal debtor in solidum with Aldecoa & Co., and not a mere surety The extension of the term which, in accordance with the provisions of article 1851 of the Civil Code produces the extinction of the liability of the surety must of necessity be based on some new agreement between the creditor and principal debtor, by virtue of which the creditor deprives himself of his right to immediately bring an action for the enforcement of his claim. The mere failure to bring an action upon a credit, as soon as the same or any part of it matures, does not constitute an extension of the term of the obligation. In the present case, Doa Isabel Palet is a personal debtor jointly and severally with Aldecoa & Co. for the whole indebtedness of the latter firm to the bank, and not a mere surety for the performance of the obligations of Aldecoa & Co. without any solidary liability. It is true that certain additional deeds of mortgage and pledge were executed by Aldecoa & Co. in favor of the bank as additional security after Aldecoa & Co. had failed to meet its obligation to pay the first installment due under the agreement of 23 February 1906, but there is no stipulation whatever in any of these documents or deeds which can in any way be interpreted in the sense of constituting an extension which would bind the bank to wait for the expiration of any new term before suing upon its claim against Aldecoa & Co.

8. Intervener is not a preferred creditor over the bank The intervener is seeking to have himself declared a preferred creditor over the bank; citing Section 121 of the Code of Civil Procedure which provides that "A person may, at any period of a trial, upon motion, be permitted by the court to intervene in an action or proceeding, if he has legal interest in the matter in litigation, or in the success of either of the parties, or an interest against both." The amount (P21,000) to which the intervener is a creditor of Aldecoa & Co. is not evidenced by a public document, or any document for that matter, nor secured by pledge or mortgage, while the amount due the bank appears in a public instrument and is also secured by pledges and mortgages on the property of Aldecoa & Co., out of which the intervener seeks to have his indebtedness satisfied. It is, therefore, clear that the intervener is not entitled to the relief sought. Further, the amount (P14,000) sought by the intervener as salary represents his salary as liquidator of the firm and not to salary prior to liquidation. 9. Receiver appointed by the court preferred in payment of fees over creditors; Intervener was appointed by members of the Company without approval from the creditor-bank The ruling of the supreme court of Spain in 16 March 1897 was correct to the effect that the fees of a receiver, appointed by the court to preserve property in litigation, must be paid in preference to the claims of creditors. In said ruling the court said that the expense of maintenance of property is bound to affect such persons as have an interest therein, whether they be the owners or creditors of the property; therefore payment for this object has preference over any other debt, since such other debts are recoverable to the extent that the property is preserved and maintained." In the present case, however, Urquhart was elected liquidator by the members of the firm of Aldecoa & Co. without the consent or approval of the bank or of any other creditor. He did not receive his employment by reason of any judicial act. Whatever may be due him for his services as liquidator is due under a contract of employment between himself and the members of the firm of Aldecoa & Co. 10. Intervener not preferred over creditors over the firms personal property; mortgage over real property not personal property Article 1922 of the Civil Code provides that, with regard to specified the personal property of the debtor, the credits for the construction, repair, preservation, or for the amount of the sale of personal property which may be in the possession of the debtor to the extent of the value of the same are preferred. The only personal property of Aldecoa & Co. is 16 shares of the stock of the Banco Espaol-Filipino; 450 shares of the stock of the Compaia Maritima (both items preceding were pledged before the liquidation); 330 shares of the stock of the Pasay Estate Co., Ltd. (which were in the possession of Aldecoa & co or its liquidator for only 1 day); and certain claims against debtors of Aldecoa & Co., mentioned in Exhibit G, which were assigned to the bank on 30 January 1907. As the intervener has been paid for his services up to January 1910, he cannot be declared a preferred creditor of the bank. The only property of Aldecoa & Co. which the liquidator had anything to do with after 1910 was the real estate mortgages mortgaged to the bank as additional security. These mortgages on real property cannot be regarded as personal property, and it is only of personal property that article 1922 speaks of. 11. Plea of another action pending is not sustained if its pendency is set up to defeat another The principle upon which a plea of another action pending is sustained is that the latter action is deemed unnecessary and vexatious (Williams vs. Gaston, 148 Ala., 214; 42 Sou., 552; 1 Cyc. 21; 1 RCL, sec. 1.) but when the pendency of such a suit is set up to defeat another, the case must be the same. There must be the same parties, or at least such as represent the same interest, there must be the same rights asserted, and the same relief prayed for. This relief must be founded on the same facts, and the title or essential basis of the relief sought must be the same. The identity in these particulars should be such that if the pending case had already been disposed of, it could be pleaded in bar as a former adjudication of the same matter between the same parties (Watson vs. Jones, 13 Wall., 679, 715; 20 L. ed., 666). In the present case, the case and the one pending in the Supreme Court are identical; thus the inquiry must therefore proceed to the other requisites demanded by the rule. The former suit is one to annul the mortgages. The present suit is one for the foreclosure of the mortgages. It may be conceded that if the final judgment in the former action is that the mortgages be annulled, such an adjudication will deny the right of the bank to foreclose the mortgages. 12. Test of identity The test of identity, stated in 1 Cyc., 28, is that "a plea of the pendency of a prior action is not available unless the prior action is of such a character that, had a judgment been rendered therein on the merits, such a judgment would be conclusive between the parties and could be pleaded in bar of the second action." This test has been approved, citing the quotation, in Williams vs. Gaston (148 Ala., 214; 42 Sou., 552); Van Vleck vs. Anderson (136 Iowa, 366; 113 N. W., 853); Wetzstein vs. Mining Co. (28 Mont., 451; 72 P., 865). It is applicable, between the same parties, only when the judgment to be rendered in the action first instituted will be such that, regardless of which party is successful, it will amount to res adjudicata against the second action. 13. Judgment declaring the children as creditors and not partners of Aldecoa not binding to the bank It appears that a certified copy of the judgment entered in the former case, wherein it was declared that the children, were creditors and partners of Aldecoa & Co., was offered in evidence. Such evidence was objected to by the bank on the ground that is was res inter alios acta and not competent evidence against the bank or binding upon it in any way because it was not a party to that action. This objection was sustained and the proffered evidence excluded. It was an action in personam and the bank was not a party. The judgment is binding only upon the parties to the suit and their successors in interest (sec. 306, Code of Civil Procedure, No. 2). Involuntary insolvency of Paul Strochecker v. Ramirez [G.R. No. 18700. September 26, 1922.] First Division, Romualdez (J): 7 concur Facts: Half-interest in the business (Antigua Botica Ramirez) was mortgaged with Fidelity & Surety Co. on 10 March 1919, and registered in due time in the registry of property, while another mortgage was made with Ildefonso Ramirez on 22 September 1919 and registered also in the registry. Raised in the lower court, the trial court declared the mortgage of Fidelity & Surety Co. entitled to preference over that of Ildefonso Ramirez and another mortgage by Concepcion Ayala. Ayala did not appeal, but Ramirez did. The Supreme Court affirmed the judgment appealed from with costs against the appellant.

1. Interest in business may be subject of mortgage With regard to the nature of the property mortgaged which is one-half interest in the business, such interest is a personal property capable of appropriation and not included in the enumeration of real properties in articles 335 of the Civil Code, and may be the subject of mortgage. All personal property may be mortgaged. (Sec. 7, Act 1508.) 2. Description of mortgage property sufficient The description contained in the document is sufficient. The law (sec. 7, Act 1508) requires only a description of the mortgaged property shall be such as to enable the parties to the mortgage, or any other person, after reasonable inquiry and investigation, to identify the same. In the case at bar, his half interest in the drug business known as Antigua Botica Ramirez, located at Calle Real Nos. 123 and 125, District of Intramuros, Manila Philippine Islands" is sufficient. 3. Article 1922 (1-3) of the Civil Code applicable only to mortgage property in possession Numbers 1, 2, and 3 of the article 1922 of the Civil Code are not applicable as neither the debtor, nor himself, is in possession of the property mortgaged, which is, and since the registration of the mortgage has been, legally in possession of the surety company (Sec. 4, Act. 1508; Meyers vs. Thein, 15 Phil., 303) 4. Stipulation about personal property not a mortgage upon property In no way can the mortgage executed be given effect as of the date of the sale of the store in question; as there was a mere stipulation about personal security during said date, but not a mortgage upon property, and much less upon the property in question. US v. Carlos [G.R. No. 6295. September 1, 1911.] En Banc, per curiam: 4 concur. Facts: Ignacio Carlos has been a consumer of electricity furnished by the Manila Electric Railroad and Light Company for a building containing the residence of the accused and 3 other residences. On 15 March 1909, representatives of the company, believing that more light is consumed than what is shown in the meter, installed an additional meter on a pole outside Carlos house to compare actual consumption (2,500 kilowatts against 233 kilowatts). Marks on the insulation of the meter points to the use of jumper. Further, the consumption registered in the inside meter is not the reasonable amount for the number of lights installed in Carlos building. On the strength of a search warrant duly served by a police officer, a jumper was found in a drawer of a small cabinet in the room of the defendants house were the meter was installed. In the absence of any explanation for Carlos possession of said device, the presumption raised was that Carlos was the owner of the device whose only use was to deflect the current from the meter. Thus, it was deduced that from 13 February, 1909 and until 3 March 1910, Carlos was found to have taken 2,273 kilowatts of electric current, worth P909.20, the electricity being the property of the Manila Electric Railroad and Light Company, a corporation doing business in the Philippine Islands, without the consent of the owner thereof. Thus, he was charged with the crime of theft. A warrant for the arrest of Carlos was issued by Judge Jenkins on 4 March and placed in the hands of the sheriff. The sheriff's return shows that the defendant gave bond for his appearance. On 14 March, the counsel for the defendant demurred to the complaint claiming the court has no jurisdiction over the person of the accused, and that the facts do not constitute a public offense. The demurrer was overruled, and as defendant refused to plead, a plea of not guilty was entered for him. After due trial, Carlos was found guilty and was sentenced to 1 year, 8 months and 21 days in prison, and was ordered to indemnify Manila Electric Railroad and Light company in the sum of P865.26 with subsidiary imprisonment in case of insolvency; and to pay the costs. From this judgment, defendant appealed. The Supreme Court affirmed the judgment appealed from; with costs against the appellant. 1. US v. Grant and Kennedy; overrule of demurrer on issue involving alleged lack of preliminary investigation The question whether "the court erred in overruling the objection of the accused to the jurisdiction of the court, because he was not given a preliminary investigation as required by law, and in overruling his demurrer is the same as was raised in US v. Grant and Kennedy (18 Phil 122), where after a thorough examination and due consideration, decided adversely to appellant's contention. No sufficient reason is presented why the Court should not follow the doctrine enunciated in that case. 2. Right of ownership of electric current secured by article 517-518 of the Penal Code The right of ownership of electric current is secured by articles 517 and 518 of the Penal Code; the application of these articles in cases of substraction of gas, a fluid used for lighting, and in some respects resembling electricity, is confirmed by the rule laid down in the decisions of the supreme court of Spain January 20, 1887, and April 1, 1897, construing and enforcing the provisions of articles 530 and 531 of the penal code of that country, articles identical with articles 517 and 518 of the code in force in the Philippines (US v. Genato). 3. Test of proper subject of larceny The true test of what is a proper subject of larceny seems to be not whether the subject is corporeal or incorporeal, but whether it is capable of appropriation by another than the owner. It is true that electricity is no longer, as formerly, regarded by electricians as a fluid, but its manifestations and effects, like those of gas, may be seen and felt. There is nothing in the nature of gas used for illuminating purposes which renders it incapable of being feloniously taken and carried away. It is a valuable article of merchandise, bought and sold like other personal property, susceptible of being severed from a mass or larger quantity, and of being transported from place to place (Commonwealth v. Shaw). Electricity, the same as gas, is a valuable article of merchandise, bought and sold like other personal property and is capable of appropriation by another. 4. Illuminating gas subject of larceny even in the absence of statute Further, it is well-settled that illuminating gas may be the subject of larceny, even in the absence of a statute so providing.

(Decisions of supreme court of Spain, January 20, 1887. and April 1, 1897, supra; also (England) Queen vs. Firth, L. R. 1 C. C., 172, 11 Cox C. C., 234; Queen vs. White, 3 C. & K., 363, 6 Cox C. C., 213; Woods vs. People, 222 Ill., 293, 7 L. R. A., 520; Commonwealth vs. Shaw, 4 Allen (Mass.), 308; State vs. Wellman, 34 Minn., 221, N. W. Rep., 385, and 25 Cyc., p. 12, note 10.) 5. No consent by company for defendant to misappropriate electricity The company had a contract with the defendant to furnish him with current for lighting purposes. It could not stop the misappropriation without cutting off the current entirely. It could not reduce the current so as to just furnish sufficient for the lighting of two, three, or five lights, as claimed by the defendant that he used during the most of this times but the current must always be sufficiently strong to furnish current for the thirty lights, at any time the defendant desired to use them. There is no indication that the company wished the electricity to be taken, and no knowledge by the defendant that the company wished him to take the current, and no mutual understanding between the company and the defendant, and no measures of inducement of any kind were employed by the company for the purpose of leading the defendant into temptation, and no preconcert whatever between him and the company: The original design to misappropriate this current was formed by the defendant absolutely independent of any acts on the part of the company or its agents. 6. Consolidation of complaint covering a period beneficial, rather than prejudicial to the defendant None of the essential rights of the defendant were shown to have been prejudiced by reason of the fact that the complaint covered the entire period. If twelve distinct and separate complaints had been filed against the defendant, one for each month, the sum total of the penalties imposed might have been very much greater than that imposed by the court in this case. The covering of the entire period by one charge has been beneficial, if anything, and not prejudicial to the rights of the defendant. The electricity was stolen from the same person, in the same manner, and in the same place. It was substantially one continuous act, although the "jumper" might have been removed and replaced daily or monthly. 7. Continuous act; Analogy of a person stealing gas by means of a pipe A person stole gas for the use of a manufactory by means of a pipe, which drew off the gas from the main without allowing it to pass through the meter. The gas from this pipe was burnt everyday, and turned off at night. The pipe was never closed at its junction with the main, and consequently always remained full of gas. It was held, that if the pipe always remained full, there was, in fact, a continuous taking of the gas and not a series of separate takings. It was held also that even if the pipe had not been kept full, the taking would have been continuous, as it was substantially all one transaction." (Regina vs. Firth, L. R., 1 C. C., 172; 11 Cox C. C., 234. Cited on p. 758 of Wharton's Criminal-Law, vol. 1, 10th ed.) US v. Tambunting [G.R. No. 16513. January 18, 1921.] En Banc, Street (J): 4 concur Facts: On January 1918, Manuel Tambunting and his wife became occupants of the upper floor of the house situated at 443, Calle Evangelista, Manila. In this house the Manila Gas Corporation had previously installed apparatus for the delivery of gas on both the upper and lower floors, consisting of the necessary piping and a gas meter, which last mentioned apparatus was installed below. When the occupants at whose request this installation had been made vacated the premises, the gas company disconnected the gas pipe and removed the meter, thus cutting off the supply of gas from said premises. Upon 2 June 1919, one of the inspectors of the gas company visited the house in question, while Manuel Tambunting was not home, and found that gas was being used, without the knowledge and consent of the gas company, for cooking in the quarters occupied by the Tambuntings. Upon arrival, Tambunting admitted that he was using gas without knowledge of the company for 2-3 months but denied making the connection where the meter used to be installed. Before the institution of the case in the CFI, the accused had been unsuccessfully prosecuted for an infraction of section 504 of the Revised Ordinances of the city of Manila, under a complaint charging that the accused, not being a registered installer of gas equipment, had placed a gas installation in the house at 443, Calle Evangelista. Later, the gas company sued Manuel Tambunting at the CFI Manila; which later found the accused guilty of stealing a quantity of gas belonging to the Manila Gas Corporation, and sentencing him to undergo imprisonment for 2 months and 1 day (arresto mayor) with the accessories prescribed by law; to indemnify the said corporation in the sum of P2, with subsidiary imprisonment in case of insolvency; and to pay the costs. Tambunting appealed. The Supreme Court affirmed the judgment with modification, it being understood that the amount of the indemnity which the accused shall pay to the gas company is P4, instead of P2, with subsidiary imprisonment for one day in case of insolvency; with costs against the appellant. 1. Right of ownership of electric current, gas, fluid used for lighting The right of the ownership of electric current is secured by articles 517 and 518 of the Penal Code; the application of these articles in cases of substraction of gas, a fluid used for lighting, and in some respects resembling electricity, is confirmed by the rule laid down in the decisions of the supreme court of Spain of January 20, 1887, and April 1, 1897, construing and enforcing the provisions of articles 530 and 531 of the Penal Code of Spain, articles identical with above articles 517 and 518. These expressions were used in a case which involved the substraction and appropriation of electrical energy and the court held, in accordance with the analogy of the case involving the theft of gas, that electrical energy could also be the subject of theft (see US v. Genato, 15 Phil., 170, 175; US v. Carlos, 21 Phil., 553) 2. Landmark case as to issue whether taking of gas constitute larceny The taking of gas may constitute larceny has never before been the subject of adjudication in the Supreme Court, but the decisions of Spanish, English, and American courts all answer the question in the affirmative. (See US vs. Carlos, 21 Phil., 553, 560.)

3. Gas has character of personal property, and may be subject of larceny There is nothing in the nature of gas used for illuminating purposes which renders it incapable of being feloniously taken and carried away. It is a valuable article of merchandise, bought and sold like other personal property, susceptible of being severed from a mass or larger quantity and of being transported from place to place. Likewise water which is confined in pipes and electricity which is conveyed by wires are subjects of larceny (Ruling Case Law, Vol. 17, p. 34). 4. Justification of the P2 per month charge The court was justified in fixing the value of the gas at P2 per month, which is the minimum charge for gas made by the gas company, however small the amount consumed. Presumably, no person desiring to use gas at all for domestic purposes can purchase the commodity at a lower rate per month than P2. There was evidence, however, before the court showing that the general average of the monthly bills paid by consumers throughout the city for the use of gas in a kitchen equipped like that used by the accused is from P18 to P20, while the average minimum is about P8 per month. We think that the facts above stated are competent evidence; and the conclusion is inevitable that the accused is at least liable to the extent of the minimum charge of P2 per month. Absolute certainty as to the full amount taken is of course impossible, because no meter was used; but absolute certainty upon this point is not necessary, when it is certain that the minimum that could have been taken was worth a determinable amount. 5. Acquittal in prosecution for violation of city ordinance not bar to prosecution for same offense under the general law of the land Acquittal of the charge of illegal gas installation in violation of Section 504 of the Revised Ordinances of Manila does not bar his prosecution for the offense of theft, since the two offenses are of totally distinct nature. Furthermore, a prosecution for violation of a city ordinance is not ordinarily a bar to a subsequent prosecution for the same offense under the general law of the land. (US vs. Garcia Gavieres, 10 Phil., 694.) Philippine Refining v. Jarque [G.R. No. 41506. March 25, 1935.] En Banc, Malcolm (J): 9 Concur Facts: On varying dates the Philippine Refining Co., Inc., and Francisco Jarque executed three mortgages, denominated as chattel mortgage on the motor vessels Pandan and Zaragoza. The first two mortgages do not have an appended affidavit of good faith, while the third contains such. The third mortgage was subscribed by Jarque and MN Brink (in what capacity the latter signed is not disclosed) and was not registered in the customs house until 17 May 1932, or within the period of 30 prior to the commencement of insolvency proceedings against Jarque. A fourth mortgage was executed by Jarque and Ramon Aboitiz on the motorship Zaragoza and was entered in the chattel mortgage registry of the register of deeds on 12 May 1932, or again within the 30-day period before the institution of insolvency proceedings. A petition was filed with the CFI Cebu on 2 June 1932 in which it was prayed that Francisco Jarque be declared an insolvent debtor, with the result that an assignment of all the properties of the insolvent debtor, was executed in favor of Jose Corominas. The petition on the matter of Jarques insolvency was granted. However, the judge declined to order the foreclosure of the mortgages, but on the contrary sustained the special defenses of fatal defectiveness of the mortgages. The Supreme Court affirmed the judgment, with costs against appellant. 1. Admiralty jurisdiction of court raised to warrant court to sit en banc The case was decided by the court in banc, as a motion was presented by counsel for the appellant in which it was asked that the case be heard and determined by the court sitting in banc because the admiralty jurisdiction of the court was involved, and this motion was granted in regular course. On further investigation it appears that this was an error because the mere mortgage of a ship is a contract entered into by the parties to it without reference to navigation or perils of the sea, and does not, therefore, confer admiralty jurisdiction. (Bogart vs. Steamboat John Jay [1854], 17 How., 399.) 2. Vessels are personal property under civil and common law Vessels are considered personal property under the civil law. (Code of Commerce, article 585.) Similarly under the common law, vessels are personal property. Under the common law, vessels are personal property although occasionally referred to as a peculiar kind of personal property. 3. Chattel mortgage of a vessel, distinguished to chattel mortgage of other personality Since the term "personal property" includes vessels, they are subject to mortgage agreeably to the provisions of the Chattel Mortgage Law. (Act 1508, section 2.) Indeed, it has heretofore been accepted without discussion that a mortgage on a vessel is in nature a chattel mortgage. The only difference between a chattel mortgage of a vessel and a chattel mortgage of other personality is that it is not now necessary for a chattel mortgage of a vessel to be noted in the registry of the register of deeds, but it is essential that a record of documents affecting the title to a vessel be entered in the record of the Collector of Customs at the port of entry. Otherwise a mortgage on a vessel is generally like other chattel mortgages as to its requisites and validity. 4. Chattel mortgage of a vessel without affidavit of good faith is unenforceable against third persons Section 5 of the Chattel Mortgage Law deemed it a requirement to have an affidavit of good faith appended to the mortgage and recorded therewith. The absence of the affidavit vitiates a mortgage as against creditors and subsequent encumbrancers. As a consequence a chattel mortgage of a vessel wherein the affidavit of good faith required by the Chattel Mortgage Law is lacking, is unenforceable against third persons. Rubiso v. Rivera [G.R. No. L-11407. October 30, 1917.] First Division, Torres (J): 5 concur, 1 took no part

Facts: Bonifacio Gelito sold his share in the pilot boat Valentina, consisting of 2/3 interest therein, to the Chinaman Sy Qui, the co-owner of the other 1/3 interest in said vessel; wherefore this vendor is no longer entitled to exercise any action whatever in respect to the boat in question. Gelito was one of the partnership owners of the Valentina, as in fact his name appears in the certificate of protection issued by the Bureau of Customs, and the rights he held are evidenced by the articles of partnership; but, the whole ownership in the vessel having been consolidated in behalf of the Chinaman Sy Qui, this latter, in the use of his right as the sole owner of the Valentina, sold this boat to Florentino E. Rivera for P2,500, on 4 January 1915, which facts are set forth in a deed ratified on the same date before a notary. This document was registered in the Bureau of Customs on 17 March 1915. After the sale of the boat to the defendant Rivera, suit having been brought in the justice of the peace court against the Chinaman Sy Qui to enforce payment of a certain sum of money, the latter's creditor Fausto Rubiso. Rubiso later acquired said vessel at a public auction sale and for the sum of P55.45. The certificate of sale and adjudication of the boat in question was issued by the sheriff on behalf of Fausto Rubiso, in the office of the Collector of Customs, on 27 January 1915 and was also entered in the commercial registry on 14 March 1915. On 10 April 1915, the plaintiffs brought suit in the CFI and alleged in the complaint that his clients were the owners of the pilot boat named Valentina, which had been in bad condition since 1914 and was stranded in Tingloy, Bauan, Batangas; and that Florentino E. Rivera took charge or possession of said vessel without the knowledge or consent of the plaintiffs and refused to deliver it to them, under claim that he was the owner thereof. After the hearing of the case and the introduction of documentary evidence, the judgment of 6 September 1915, was rendered, , in which the defendant and appellant was ordered to place at the disposal of the Fausto Rubiso the pilot boat in litigation. No special finding was made for costs. The defendant appealed and moved for a new trial. This motion was denied and appellant excepted. The Supreme Court affirmed the judgment, with the costs against the appellant. 1. Inscription in registry is necessary and indispensable Article 573 of the Code of Commerce provides in its first paragraph that merchant vessels constitute property which may be acquired an transferred by any of the means recognized by law. The acquisition of a vessel must be included in a written instrument, which shall not produce any effect with regard to third persons if not recorded in the commercial registry. With the enactment of Act 1900 on 18 May 1909, said article was amended and appears as Section 2 of that Act; amending solely in charging the Insular Collector of Customs with the fulfillment of the duties of the commercial register concerning the registering of vessels. In both laws, Inscription in the commercial registry is necessary and indispensable in order that the purchaser's rights may be maintained against a claim filed by a third person. The legal rule set down in the Mercantile Code subsists, inasmuch as the amendment solely refers to the official who shall make the entry; but, with respect to the rights of two purchases, whichever of them first registered his acquisition of the vessel in the one entitled to enjoy the protection of the law. 2. Even if public auction is subsequent to private purchase, right of first to register is primary Florentino E. Rivera's rights cannot prevail over those acquired by Fausto Rubiso in the ownership of the pilot boat Valentina, inasmuch as, though the latter's acquisition of the vessel at public auction, on 23 January 1915, was subsequent to its purchase by Rivera, nevertheless the sale at public auction was antecedently record in the office of the Collector of Customs, on January 27, and entered in the commercial registry (an unnecessary proceeding) on March 4; while the private and voluntary purchase made by Rivera on a prior date was not recorded in the office of the Collector of Customs until 17 March 1915. Rubiso, who was careful to record his acquisition, opportunely and on prior date, has, according to the law, a better right than the defendant Rivera who subsequently recorded his purchase. The latter is a third person, who was directly affected by the registration which the plaintiff made of the acquisition. 3. Effects of registration to liabilities of a vessel When the absolute owner of the purchased boat, declaring the latter to be free of all encumbrance and all claims by strangers for, pursuant to article 582 of the Mercantile Code, after the bill of the judicial sale at auction has been executed and recorded in the commercial registry, all the other liabilities of the vessel in favor of the creditors shall be considered canceled. 4. Ships or vessels are of the nature and condition of real property; Article 573 of Code of Commerce vis--vis Article 1473 of the Civil Code Ships or vessels, whether moved by steam or by sail, partake, to a certain extent, of the nature and conditions of real property, on account of their value and importance in the world commerce; and for this reason the provisions of article 573 of the Code of Commerce are nearly identical with article 1473 of the Civil Code. 5. No indemnity for losses and damages Aside from the fact that, subsequent to the date when the judgment appealed from was rendered, the vessel in question emerged unharmed from the place where it was stranded, and was, at the time of the trial, anchored in the port of Maricaban, the record certainly does not furnish any positive evidence of the losses and damages alleged to have been occasioned. On the other hand, it cannot be affirmed that the defendant acted in bad faith specifically because he acquired the vessel on a date prior to that of its acquisition at public auction by the plaintiff Rubiso, who, for the reasons aforestated, is true and sole owner of said pilot boat. Monserrat v. Ceron [G.R. No. 37078. September 27, 1933.] En Banc, Villa-real (J): 4 concur Facts: Enrique Monserrat, was the president and manager of the Manila Yellow Taxicab Company and the owner of 1,200 common shares of stock thereof. On 25 March 1930, in consideration of the interest shown and the financial aid extended him in the organization of the corporation by Carlos G. Ceron, Monserrat assigned to the former the usufruct of half of the said common shares of stock; the corresponding certificate of stock No. 7, having been issued in the name of Ceron to that effect on 24 March 1930. Said assignment or transfer only gave the transferee the right to enjoy, during his lifetime, the profits which might be

derived from the shares assigned him, prohibiting him from selling, mortgaging, encumbering, alienating or otherwise exercising any act implying absolute ownership of all or any of the shares in question, the transferor having reserved for himself and his heirs the right to vote derived from said shares of stock and to recover the ownership thereof at the termination of the usufruct (Exhibit A). The Stock certificate was recorded in the name of Ceron and the deed of transfer was noted on page 22 of the Stock and Transfer Book of the Manila Yellow Taxicab Company. On 26 February 1931, Ceron mortgaged to Eduardo R. Matute, president of Erma Inc., some shares of stock of the Manila Yellow Taxicab, among which were the 600 common shares of stock in question, for the sum of P30,000. Ceron endorsed to Matute the certificate of stock, of which Matute has been in possession ever since. When Ceron mortgaged the shares to Matute, he did not inform Matute of the existence of the document (Exhibit A) and the latter never had any knowledge thereof. Ceron continued as secretary of the Manila Yellow Taxicab until 5 May 1931, the date when the notation of Exhibit A in question appearing in the books of the corporation and the same date when the shares of stock were sold at public auction to satisfy Cerons debt to Matute. Case facts do not provide for details on how the case was instituted in the CFI Manila The CFI Manila rendered judgment in favor of the plaintiff declaring the plaintiff the owner of the 600 shares of stock; and declaring the mortgage constituted on the ownership of the shares of stock null and void and without force and effect, although the mortgage on the usufruct enjoyed by the mortgage debtor Ceron in the said 600 shares of stock is hereby declared valid; with costs against the defendants. Erma Inc. and the Sheriff of Manila, the defendants therein, appealed from the decision. The Supreme Court reversed the judgment appealed from and absolved defendants from the complaint therein; with costs against the appellee. 1. Section 35 of the Corporation Law does not require entry except of transfers of shares of stock in order that such transfers may be valid as against third persons. Section 35 of the Corporation Law provides that the capital stock of stock corporations shall be divided into shares for which certificates signed by the president or the vice-president, countersigned by the secretary or clerk and sealed with the seal of the corporation, shall be issued in accordance with the by-laws. Shares of stock so issued are personal property and may be transferred by delivery of the certificate endorsed by the owner or his attorney in fact or other person legally authorized to make the transfer. No transfer, however, shall be valid, except as between the parties, until the transfer is entered and noted upon the books of the corporation so as to show the names of the parties to the transaction, the date of the transfer, the number of the certificate, and the number of shares transferred; and that no share of stock against which the corporation hold any unpaid claim shall be transferable on the books of the corporation." 2. Words in statute to be taken in their natural, plain and ordinary signification It is a rule of statutory construction that the words of a statute are to be taken in their natural, plain and ordinary signification in accordance with the common and approved usage of the language, giving to words of common use their popularly accepted meaning and to technical terms or words of art, their accepted special signification, unless there is reason to believe from the context of the statute that such words have been used in another sense. In the present case, inasmuch as it does not appear from the text of the Corporation Law that an attempt was made to give a special signification to the word "transfer", we it shall be construed according to its accepted meaning in ordinary parlance. 3. Transfer defined Diccionario de la Academia de la Lengua Castellana: The word "transferencia" (transfer) is defined as "accion y efecto de tarnsferir" (the act and effect of transferring); and the verb "transferir", as "ceder o renunciar en otro el derecho o dominio que se tiene sobre una cosa, hacindole dueo de ella" (to assign or waive the right in, or absolute ownership of, a thing in favor of another, making him the owner thereof). Law Dictionary of "Words and Phrases": "Transfer' means any act by which property of one person is vested in another, and 'transfer of shares', as used in Uniform Stock Transfer Act, implies any means whereby one may be divested of and another acquire ownership of stock. 4. Chattel mortgage defined Section 3 of Act 1508, as amended by Act 2496, defines the phrase "hipoteca mobiliaria" (chattel mortgage) as a conditional sale of personal property as security for the payment of a debt, or the performance of some other obligation specified therein, the condition being that the sale shall be void upon the seller paying to the purchaser a sum of money or doing some other act named. If the condition is performed according to its terms the mortgage and sale immediately become void, and the mortgagee is thereby divested of his title." 5. Transfer in a chattel mortgage a mere security for the payment of the mortgage debt Although a chattel mortgage, accompanied by delivery of the mortgaged thing, transfers the title and ownership thereof to the mortgage creditor, such transfer is not absolute but constitutes a mere security for the payment of the mortgage debt, the transfer in question becoming null and void from the time the mortgage debtor complies with his obligation to pay his debt. 6. Transfer in a chattel mortgage not the transfer contemplated in Section 35 of the Corporation Law, i.e. Absolute and unconditional conveyance of title and ownership of a share of stock "A 'transfer' is the act by which the owner of a thing delivers it to another with the intent of passing the rights which he has in it to the latter, and a chattel mortgage is not within the meaning of such term. (Noble v. Ft. Smith Wholesale Grocery, 127 Pac., 14,17; 34 Okl. 662; 46 LRA [NS] 455). The chattel mortgage is not the transfer referred to in section 35 of Act 1459 (the Corporation Law) which transfer should be entered and noted upon the books of a corporation in order to be valid, and which, as has already been said, means the absolute and unconditional conveyance of the title and ownership of a share of stock. The entry and notation of a chattel mortgage upon the books of the corporation is not a necessary requisite to its validity.

7. Matute (Erma Inc.) a conditional purchaser in good faith Erma, Inc., as a conditional purchaser of the shares of stock in question given as security for the payment of his credit, acquired in good faith Ceron's right and title to the 600 common shares of stock evidenced by certificate No. 7 of the Manila Yellow Taxicab Company., and as such conditional purchaser in good faith, it is entitle to the protection of the law. Evidence would show that Matute found the shares, as recorded in the Stock and Transfer Book of the company, free from all liens and encumbrances and made no reference made to the deed Exhibit A. Chua Guan v. Samahang Magsasaka [G.R. No. 42091. November 2, 1935.] Second Division, Butte (J): 4 concur Facts: On June 18, 1931, Gonzalo H. Co Toco, the owner of 5,894 shares of the capital stock of Samahang Magsasaka Inc. represented by 9 certificates having a par value of P5 per share mortgaged said shares to Chua Chiu to guarantee the payment of a debt of P20,000 due on or before 19 June 1932. The said certificates of stock were delivered with the mortgage to the mortgagee, Chua Chiu. The said mortgage was duly registered in the office of the registered of deeds of Manila on 23 June 1931, and in the office of the said corporation on 30 September 1931. On 28 November 1931, Chua Chiu assigned all his right and interest in said mortgage to the Chua Guan and the assignment in the office of the register of deeds in the City of Manila on 28 December 1931, and in the office of the said corporation on 4 January 1932. Co Toco defaulted in the payment of said debt at maturity and Chua Guan foreclosed said mortgage and delivered the certificates of stock and copies of the mortgage and assignment to the sheriff of the City of Manila in order to sell the said shares at public auction. The sheriff auctioned said shares on 22 December 1932, and the plaintiff having been the highest bidder for the sum of P14,390, the sheriff executed in his favor a certificate of sale of said shares. The plaintiff tendered the certificates of stock standing in the name of Co Toco to the proper officers of the corporation for cancellation and demanded that they issue new certificates in the name of Chua Guan. The officers (the individual defendants) refused and still refuse to issue said new shares in the name of Chua Guan. An action for writ of mandamus was filed with the CFI Nueva Ecija, praying that the defendants transfer the said 5,894 shares of stock to the plaintiff by cancelling the old certificates and issuing new ones in their stead. The parties entered into a stipulation in which the defendants admitted all of the allegations of the complaint and the plaintiff admitted all of the special defenses in the answer of the defendants, and on this stipulation they submitted the case for decision. As special defense, the defendants refused to cancel said certificates (Co Tocos) and to issue new ones in the name of Chua Guan because prior to the date of the latters demand (4 February 1933), 9 attachments had been issued and served and noted on the books of the corporation against Co Tocos shares and Chua Guan objected to having these attachments noted on the new certificates which he demanded. The Supreme Court affirmed the judgment appealed from, holding that the attaching creditors are entitled to priority over the defectively registered mortgage of the appellant; without special pronouncement as to costs. 1. Registration of chattel mortgage in the office of corporation not necessary and had no legal effect The registration of the said chattel mortgage in the office of the corporation was not necessary and had no legal effect. Whether the shares of a corporation could be hypothecated by placing a chattel mortgage on the certificate representing such shares are settled by the case of Monserrat vs. Ceron. In the present case, the registration of such a mortgage or the effect of such registration was not in question. Nothing appears in the record of that case even tending to show that the chattel mortgage there involved was ever registered anywhere except in the office of the corporation, and there was no question involved there as to the right of priority among conflicting claims of creditors of the owner of the shares. 2. Difficulty on the practical application of the Chattel Mortgage law to shares of stock of a corporation The practical application of the Chattel Mortgage Law to shares of stock of a corporation presents considerable difficulty, as an equity in shares of stock is of such an intangible character, and the Court has obtained little aid from the decisions of other jurisdictions because that form of mortgage is ill suited to the hypothecation of shares of stock and has been rarely used elsewhere. In fact, it has been doubted whether shares of stock in a corporation are chattels in the sense in which that word is used in chattel mortgage statutes (see Fua Cun vs. Summers and China Banking Corporation). 3. Ways in executing a valid chattel mortgage effective against third persons Section 4 of Act 1508 provides two ways for executing a valid chattel mortgage which shall be effective against third persons. First, the possession of the property mortgaged must be delivered to and retained by the mortgagee; and, second, without such delivery the mortgage must be recorded in the proper office or offices of the register or registers of deeds. 4. Proper place of registration of a chattel mortgage Section 4 provides that in such a case the mortgage shall be registered in the province in which the mortgagor resides at the time of making the same or, if he is a non-resident, in the province in which the property is situated; and it also provides that if the property is situated in a different province from that in which the mortgagor resides the mortgage shall be recorded both in the province of the mortgagor's residence and in the province where the property is situated. 5. Domicile of corporation decisive for purposes of execution, attachment and garnishment of shares of stock It is a common but not accurate generalization that the situs of shares of stock is at the domicile of the owner. The term situs is not one of fixed or invariable meaning or usage. Nor should one lose sight of the difference between the situs of the shares and the situs of the certificate of shares. The situs of shares of stock for some purposes may be at the domicile of the owner and for others at the domicile of the corporation; and even elsewhere. It is a general rule that for purposes of execution, attachment and garnishment, it is not the domicile of the owner of a certificate but the domicile of the corporation which is decisive.

6. Courts construction of Section 4, Act 1508, as to ownership of shares in a corporation By analogy with the foregoing and considering the ownership of shares in a corporation as property distinct from the certificates which are merely the evidence of such ownership, it seems to be a reasonable construction of section 4 of Act 1508 to hold that the property in the shares may be deemed to be situated in the province in which the corporation has its principal office or place of business. If this province is also the province of the owner's domicile, a single registration is sufficient. If not, the chattel mortgage should be registered both at the owner's domicile and in the province where the corporation has its principal office or place of business. In this sense the property mortgaged is not the certificate but the participation and share of the owner in the assets of the corporation. 7. Method of hypothecating shares of stock by chattel mortgage cumbersome and unusual in the present state of law; Risks of debtor and/or creditor; Remedy is with legislature Apart from the cumbersome and unusual method of hypothecating shares of stock by chattel mortgage, it appears that in the present state of our law, the only safe way to accomplish the hypothecation of share of stock of a Philippine corporation is for the creditor to insist on the assignment and delivery of the certificate and to obtain the transfer of the legal title to him on the books of the corporation by the cancellation of the certificate and the issuance of a new one to him. From the standpoint of the debtor this may be unsatisfactory because it leaves the creditor as the ostensible owner of the shares and the debtor is forced to rely upon the honesty and solvency of the creditor. Of course, the mere possession and retention of the debtor's certificate by the creditor gives some security to the creditor against an attempted voluntary transfer by the debtor, provided by- laws of the corporation expressly enact that transfers may be made only upon the surrender of the certificate. It is to be noted, however, that section 35 of the Corporation Law enacts that shares of stock "may be transferred by delivery of the certificate endorsed by the owner or his attorney in fact or other person legally authorized to make the transfer." The use of the verb "may" does not exclude the possibility that a transfer may be made in a different manner, thus leaving the creditor in an insecure position even though he has the certificate in his possession. Moreover, the shares still standing in the name of the debtor on the books of the corporation will be liable to seizure by attachment or levy on execution at the instance of other creditors. Decisions in the case of Monserrat vs. Ceron and in the present case have done little perhaps to ameliorate the present uncertain and unsatisfactory state of our law applicable to pledges and chattel mortgages of shares of stock of Philippine corporations. The remedy lies with the legislature. 8. Transfer by endorsement and delivery of certificate with intention to pledge sufficient to give legal effect The transfer by endorsement and delivery of a certificate with intention to pledge the shares covered thereby should be sufficient to give legal effect to that intention and to consummate the juristic act without necessity for registration. Republic v. CA [G.R. No. L-43105. August 31, 1984.]; also Bautista v. CA [G.R. No. L-43190] Second Division, Cuevas (J): 4 concur, 1 on leave, 1 took no part. Facts: A lot with an area of 17,311 sq.m. situated in Barrio Pinagbayanan, Pila, Laguna and 20 meters from the shore of Laguna de Bay; was purchased by Benedicto del Rio from Angel Pili on 19 April 1909. The Deed of Sale evidencing said purchase is duly recorded with the Registry of Deeds of Sta. Cruz, Laguna. The land was declared for tax purposes beginning the year 1918, and the realty taxes thereon had been paid since 1948. When Benedicto del Rio died in 1957, his heirs extrajudicially partitioned his estate and the subject parcel passed on to his son, Santos del Rio, as the latter's share in the inheritance. Santos del Rio filed his application for registration of said parcel on 9 May 1966. The application was opposed by the Director of Lands and by private oppositors, petitioners in G.R. No. L-43190. Sometime before 1966, private oppositors obtained permission from Santos del Rio to construct duck houses on the land in question. Although there was no definite commitment as to rentals, some of them had made voluntary payments to private respondent. In violation of the original agreement, private oppositors constructed residential houses on the land which prompted private respondent to file an ejectment suit against the former in 1966. Meanwhile, during the latter part of 1965 and in 1966, private oppositors had simultaneously filed their respective sales applications with the Bureau of Lands, and in 1966, they opposed Santos del Rio's application for registration. The CFI Laguna dismissed the application for registration. Applicant appealed and obtained a favorable judgment from the Court of Appeals, setting aside that of the trial court. The Director of Lands and the private oppositors filed their respective Petitions for Review of said decision. The Supreme Court affirmed the judgment affirmed from, and ordered the registration of the land described in the application in favor of Santos del Rio, applicant private respondent; with costs against private petitioners. 1. Classification of property as either of public dominion or of private ownership; Public lands / public dominion Property, which includes parcels of land found in Philippine territory, is either of public dominion or of private ownership. Public lands, or those of public dominion, have been described as those which, under existing legislation are not the subject of private ownership, and are reserved for public purposes. The New Civil Code enumerates properties of public dominion in Articles 420 and 502 thereof. Article 402 includes those intended for public use, such as roads, canals, rivers, torrents, ports and bridges constructed by the State, banks, shores, roadsteads, and others of similar character; and those which belong to the State without being for public use, and are intended for some public service or for the development of the national wealth" as property belonging to public dominion. Article 502 adds "rivers and their natural beds; continuous or intermittent waters of springs and brooks running in their natural beds and the beds themselves; waters rising continuously or intermittently on lands of public dominion; and lakes and lagoons formed by Nature on public lands and their beds; to the enumeration. 2. Extent of a lake bed The extent of a lake bed is defined in Artcile 74 of the Law of Waters of 1866, as the natural bed or basin of lakes, ponds, or pools, is the ground covered by their waters when at their highest ordinary depth." 3. Highest Ordinary Depth in a lake; Determinant is rainfall and not gravitational pull (tides) The phrase "highest ordinary depth" has been interpreted in the case of Government. vs. Colegio de San Jose to be the highest depth of the waters of Laguna de Bay during the dry season, such depth being the "regular, common, natural, which occurs always

or most of the time during the year; or thus rain "falling directly on or flowing into Laguna de Bay from different sources." While the waters of a lake are also subject to the same gravitational forces that cause the formation of tides in seas and oceans, this phenomenon is not a regular daily occurrence in the case of lakes. The alternation of high tides and low tides, which is an ordinary occurrence, could hardly account for the rise in the water level of the Laguna de Bay as observed 4-5 months a year during the rainy season; rather, it is the rains which bring about the inundation of a portion of the land in question. Since the rise in the water level which causes the submersion of the land occurs during a shorter period than the level of the water at which the land is completely dry, the latter should be considered as the "highest ordinary depth" of Laguna de Bay. The land sought to be registered, therefore, is not part of the bed or basin of Laguna de Bay. 4. Foreshore land defined; Definition does not apply to land adjacent to lake Foreshore land is that part of (the land) which is between high and low water and left dry by the flux and reflux of the tides; or the strip of land that lies between the high and low water marks and that is alternately wet and dry according to the flow of the tide. In the present case, since the inundation of a portion of the land near the lake is not due to flux and reflux of tides, it thus cannot be considered a foreshore land within the meaning cited by the Director of Lands. 5. Purpose of land registration under Torrens System The purpose of land registration under the Torrens System is not the acquisition of lands but only the registration of title which applicant already possesses over the land. Registration under the Torrens Law was never intended as a means of acquiring ownership. Applicant in this case asserts ownership over the parcel of land he seeks to register and traces the roots of his title to a public instrument of sale in favor of his father from whom he inherited said land. 6. Tax declaration strong evidence of ownership acquired by prescription; also Open, continuous, public, peaceful, exclusive and adverse possession of the land Applicant presents tax declarations covering the land since 1918 and also tax receipts dating back to 1948. While it is true that by themselves tax receipts and declarations of ownership for taxation purposes are not incontrovertible evidence of ownership, they become strong evidence of ownership acquired by prescription when accompanied by proof of actual possession of the property. Further, applicant by himself and through his father before him, has been in open, continuous, public, peaceful, exclusive and adverse possession of the disputed land for more than 30 years, counted from 19 April 1909, when the land was acquired from a third person by purchase. Since applicant has possessed the subject parcel in the concept of owner with just title and in good faith, his possession need only last for ten years in order for ordinary acquisitive prescription to set in. Applicant has more than satisfied this legal requirement. 7. Judicial confirmation of imperfect title Even if the land sought to be registered is public land, applicant would be entitled to a judicial confirmation of his imperfect title, since he has also satisfied the requirements of the Public Land Act (CA 141 as amended by RA 1942). Section 48 of the Act enumerates as among the persons entitled to judicial confirmation of imperfect title, such as those who, by themselves or through their predecessors-in-interest, have been in the open, continuous, exclusive, and notorious possession and occupation of agricultural lands of the public domain, under bona fide claim of ownership, for at least thirty years immediately preceding the filing of the application for confirmation of title." 8. Reclamation requires proper permission; reclaimed land does not automatically belong to party reclaiming the same Private persons cannot, by themselves reclaim land from water bodies belonging to the public domain without proper permission from government authorities. And even if such reclamation had been authorized, the reclaimed land does not automatically belong to the party reclaiming the same as they may still be subject to the terms of the authority earlier granted. In the present case, private oppositors-petitioners failed to show proper authority for the alleged reclamation, therefore, their claimed title to the litigated parcel must fall. 9. Tolerance of possession cannot ripen into ownership As the private oppositors-petitioners entered into possession of the land with the permission of, and as tenants of, the applicant del Rio; the fact that some of them at one time or another did not pay rent. Their use of the land and their non-payment of rents thereon were merely tolerated by applicant and these could not have affected the character of the latter's possession which has already ripened into ownership at the time of the filing of this application for registration. Only possession acquired and enjoyed in the concept of owner can serve as the root of a title acquired by prescription. Laurel v. Garcia [G.R. No. 92013 & 92047. July 25, 1990.], Ojeda v. Macaraig [G.R. No. 92047. July 25, 1990.] En Banc, Gutierrez, Jr. (J): 5 concur Facts: The subject Roppongi property is one of the four properties in Japan acquired by the Philippine government under the Reparations Agreement entered into with Japan on 9 May 1956, the other lots being the Nampeidai Property (site of Philippine Embassy Chancery), the Kobe Commercial Property (Commercial lot used as warehouse and parking lot of consulate staff), and the Kobe Residential Property (a vacant residential lot). The properties and the capital goods and services procured from the Japanese government for national development projects are part of the indemnification to the Filipino people for their losses in life and property and their suffering during World War II. The Reparations Agreement provides that reparations valued at $550 million would be payable in 20 years in accordance with annual schedules of procurements to be fixed by the Philippine and Japanese governments (Article 2, Reparations Agreement). RA 1789, the Reparations Law, prescribes the national policy on procurement and utilization of reparations and development loans; those which belong to the government and which may be availed of by private entities. The Roppongi property was acquired from the Japanese government under the Second Year Schedule and listed under the heading "Government Sector", through Reparations Contract 300 dated 27 June 1958. The Roponggi property consists of the land and building "for the Chancery of the Philippine Embassy." As intended, it became the site of the Philippine Embassy until the latter was transferred to Nampeidai on 22 July 1976 when the Roppongi building needed major repairs. Due to the failure of our government to provide necessary funds, the Roppongi property has remained undeveloped since that time.

During the incumbency of President Aquino, a proposal was made by former Philippine Ambassador to Japan, Carlos J. Valdez, to lease the subject property to Kajima Corporation, a Japanese firm, in exchange of the construction of 2 buildings in Roppongi, 1 building in Nampeidai, and the renovation of the Philippine Chancery in Nampeidai. The Government did not act favorably to said proposal, but instead, on 11 August 1986, President Aquino created a committee to study the disposition or utilization of Philippine government properties in Tokyo and Kobe though AO-3, and AO 3-A to 3-D. On 25 July 1987, the President issued EO 296 entitling non-Filipino citizens or entities to avail of reparations' capital goods and services in the event of sale, lease or disposition. The four properties in Japan including the Roppongi were specifically mentioned in the first "Whereas" clause. Amidst opposition by various sectors, the Executive branch of the government has been pushing, with great vigor, its decision to sell the reparations properties starting with the Roppongi lot. Two petitions for prohibition were filed seeking to enjoin respondents, their representatives and agents from proceeding with the bidding for the sale of the 3,179 sq. m. of land at 306 Ropponggi, 5-Chome Minato-ku, Tokyo, Japan scheduled on 21 February 1990; the temporary restaining order of which was granted by the court on 20 February 1990. In G.R. No. 92047, a writ of mandamus was prayed for to compel the respondents to fully disclose to the public the basis of their decision to push through with the sale of the Roppongi property inspite of strong public opposition and to explain the proceedings which effectively prevent the participation of Filipino citizens and entities in the bidding process. After multiple motions for extension to file comment by the respondents, the Supreme Court resolved to decide the 2 cases; thereby granting the petitions and enjoining the respondents from proceeding with the sale of the Roppongi property in Tokyo, Japan. The Court also made permanent the 20 February 1990 temporary restaining order. 1. Roponggi lot is a property of public dominion The nature of the Roppongi lot as property for public service is expressly spelled out. It is dictated by the terms of the Reparations Agreement and the corresponding contract of procurement which bind both the Philippine government and the Japanese government, that these were assigned to the government sector and that the Roppongi property itself was specifically designated under the Reparations Agreement to house the Philippine Embassy. There can be no doubt that it is of public dominion unless it is convincingly shown that the property has become patrimonial; which respondents have failed to show. 2. Property of public dominion outside the commerce of man As property of public dominion, the Roppongi lot is outside the commerce of man. It cannot be alienated. Its ownership is a special collective ownership for general use and enjoyment, an application to the satisfaction of collective needs, and resides in the social group. The purpose is not to serve the State as a juridical person, but the citizens; it is intended for the common and public welfare and cannot be the object of appropriation. 3. Pertinent provisions of the Civil Code Article 419 provides that property is either of public dominion or of private ownership. Article 420 provides that property of public dominion includes (1) those intended for public use, such as roads, canals, rivers, torrents, ports and bridges constructed by the State, banks, shores, roadsteads, and others of similar character; (2) those which belong to the State, without being for public use, and are intended for some public service or for the development of the national wealth. Article 421 provides that all other property of the State, which is not of the character stated in the preceding article, is patrimonial property." In the present case, the Roppongi property is correctly classified under paragraph 2 of Article 420 of the Civil Code as property belonging to the State and intended for some public service. 4. Conversion to patrimonial property happen if property is withdrawn from public use; Abandonment must be certain and positive act based on correct legal premises The fact that the Roppongi site has not been used for a long time for actual Embassy service does not automatically convert it to patrimonial property. Any such conversion happens only if the property is withdrawn from public use (Cebu Oxygen and Acetylene Co. v. Bercilles, 66 SCRA 481 [1975]). A property continues to be part of the public domain, not available for private appropriation or ownership "until there is a formal declaration on the part of the government to withdraw it from being such (Ignacio v. Director of Lands, 108 Phil. 335 [1960]) An abandonment of the intention to use the Roppongi property for public service and to make it patrimonial property under Article 422 of the Civil Code must be definite. Abandonment cannot be inferred from the non-use alone specially if the non-use was attributable not to the government's own deliberate and indubitable will but to a lack of financial support to repair and improve the property (See Heirs of Felino Santiago v. Lazarao, 166 SCRA 368 [1988]). Abandonment must be a certain and positive act based on correct legal premises. In the present case, the recent Administrative Orders authorizing a study of the status and conditions of government properties in Japan were merely directives for investigation but did not in any way signify a clear intention to dispose of the properties. Further EO 296 does not declare that the properties lost their public character, but merely intends to make the properties available to foreigners and not to Filipinos alone in case of a sale, lease or other disposition. 5. RA 6657 does not authorize the disposition of Roppongi property as it is outside the commerce of man; EO 296 amended nationality provision for the sale of procurements for the private sector, not the procurements for the government (the latter which includes Roppongi property) EO 296 is based on the wrong premise or assumption that the Roppongi and the three other properties were earlier converted into alienable real properties. RA 1789 differentiates the procurements for the government sector and the private sector (Sections 2 and 12, RA 1789). Only the private sector properties can be sold to end-users who must be Filipinos or entities owned by Filipinos. It is this nationality provision which was amended by EO 296. Further, Section 63 (c) of RA 6657 (the CARP Law) which provides as one of the sources of funds for its implementation, the proceeds of the disposition of the properties of the Government in foreign countries, did not withdraw the Roppongi property from being classified as one of public dominion when it mentions Philippine properties abroad. Section 63 (c) refers to properties which are alienable and not to those reserved for public use or service. RA 6657, therefore, does not authorize the Executive Department to sell the Roppongi property. It merely enumerates possible sources of future funding to augment (as and when needed) the Agrarian Reform Fund created under EO 299. Obviously any property outside of the commerce of man cannot be tapped as a source of funds.

6. Conflict of law rule does not apply when conflict of law situation does not exist A conflict of law rule cannot apply when no conflict of law situation exists. A conflict of law situation arises only when: (1) There is a dispute over the title or ownership of an immovable, such that the capacity to take and transfer immovables, the formalities of conveyance, the essential validity and effect of the transfer, or the interpretation and effect of a conveyance, are to be determined; and (2) A foreign law on land ownership and its conveyance is asserted to conflict with a domestic law on the same matters. Hence, the need to determine which law should apply. In the present case, none of the above elements exists. 7. Issue on the authority of officials to dispose property belonging to state, and not validity of ownership or title, in question; governed by Philippine law The issues are not concerned with validity of ownership or title. There is no question that the property belongs to the Philippines. The issue is the authority of the respondent officials to validly dispose of property belonging to the State. And the validity of the procedures adopted to effect its sale. This is governed by Philippine Law. The rule of lex situs does not apply. 8. Opinion of Secretary of Justice irrelevant; Issue of whether the property can be sold precedes the issue of who can acquire The assertion that the opinion of the Secretary of Justice sheds light on the relevance of the lex situs rule is misplaced. The opinion does not tackle the alienability of the real properties procured through reparations nor the existence in what body of the authority to sell them. In discussing who are capable of acquiring the lots, the Secretary merely explains that it is the foreign law which should determine who can acquire the properties so that the constitutional limitation on acquisition of lands of the public domain to Filipino citizens and entities wholly owned by Filipinos is inapplicable. There is no need to discuss who can acquire the Roppongi lot when there is no showing that it can be sold. 9. Approval of the President of the recommendation of the committee to sell the Roppongi property premature, and without force and effect of law The subsequent approval on 4 October 1988 by President Aquino of the recommendation by the investigating committee to sell the Roppongi property was premature or, at the very least, conditioned on a valid change in the public character of the Roppongi property. Moreover, the approval does not have the force and effect of law since the President already lost her legislative powers. The Congress had already convened for more than a year. 10. There is no law authorizing the conveyance of the Roppongi property; Conveyance must be authorized by law enacted by Congress and requires executive and legislative concurrence Section 79 (f) of the Revised Administrative Code of 1917 (Conveyances and contracts to which the Government is a party) provides that in cases in which the Government of the Republic of the Philippines is a party to any deed or other instrument conveying the title to real estate or to any other property the value of which is in excess of P100,000, the respective Department Secretary shall prepare the necessary papers which, together with the proper recommendations, shall be submitted to the Congress of the Philippines for approval by the same. Such deed, instrument, or contract shall be executed and signed by the President of the Philippines on behalf of the Government of the Philippines unless the Government of the Philippines unless the authority therefor be expressly vested by law in another officer." The requirement has been retained in Section 48, Book I of the Administrative Code of 1987 (EO 292; Official authorized to convey real property), which provides that Whenever real property of the Government is authorized by law to be conveyed, the deed of conveyance shall be executed in behalf of the government by the following: (1) for property belonging to and titled in the name of the Republic of the Philippines, by the President, unless the authority therefor is expressly vested by law in another officer; (2) for property belonging to the Republic of the Philippines but titled in the name of any political subdivision or of any corporate agency or instrumentality, by the executive head of the agency or instrumentality." Thus, it is not for the President to convey valuable real property of the government on his or her own sole will. Any such conveyance must be authorized and approved by a law enacted by the Congress. It requires executive and legislative concurrence. 11. 1989 case on the Roppongi property: Ojeda v. Bidding Committee; Issue different The resolution of the Supreme Court in Ojeda v. Bidding Committee, et al., did not pass upon the constitutionality of EO 296 nor did it uphold the authority of the President to sell the Roppongi property. The Court stated that the constitutionality of the executive order was not the real issue and that resolving the constitutional question was "neither necessary nor finally determinative of the case." The Court noted that "[W]hat petitioner ultimately questions is the use of the proceeds of the disposition of the Roppongi property." In emphasizing that "the decision of the Executive to dispose of the Roppongi property to finance the CARP cannot be questioned" in view of Section 63 (c) of RA 6657, the Court did not acknowledge the fact that the property became alienable nor did it indicate that the President was authorized to dispose of the Roppongi property. The resolution should be read to mean that in case the Roppongi property is re-classified to be patrimonial and alienable by authority of law, the proceeds of a sale may be used for national economic development projects including the CARP. 12. Constitutional questions raised in the Supreme Court The Court does not ordinarily pass upon constitutional questions unless these questions are properly raised in appropriate cases and their resolution is necessary for the determination of the case (People v. Vera, 65 Phil. 56 [1937]). The Court will not pass upon a constitutional question although properly presented by the record if the case can be disposed of on some other ground such as the application of a statute or general law (Siler v. Louisville and Nashville R. Co., 213 U.S. 175, [1909], Railroad Commission v. Pullman Co., 312 U.S. 496 [1941]). 13. Value of the Roppongi property; besides economic and financial benefits The Roppongi property is valuable not so much because of the inflated prices fetched by real property in Tokyo but more so because of its symbolic value to all Filipinos veterans and civilians alike. The Roppongi property is not just like any piece of property. It was given to the Filipino people in reparation for the lives and blood of Filipinos who died and suffered during the Japanese military occupation, for the suffering of widows and orphans who lost their loved ones and kindred, for the homes and other properties lost by countless Filipinos during the war. The Tokyo properties are a monument to the bravery and sacrifice of the Filipino people in the face of an invader. Roppongi is a reminder that cannot should not be dissipated.

Manila Lodge 761 v. CA [G.R. No. L-41001. September 30, 1976.]; also Tarlac Development Corp. v. CA [G.R. No. L-41012] First Division, Castro (J): 3 concur, 1 concurs in the result. Facts: On 26 June 1905 the Philippine Commission enacted Act 1360 which authorized the City of Manila to reclaim a portion of Manila Bay. The reclaimed area was to form part of the Luneta extension. The Act provided that the reclaimed area "shall be the property of the City of Manila" and that "the City of Manila is hereby authorized to set aside a tract of the reclaimed land formed by the Luneta extension at the north end not to exceed 500 feet by 600 feet in size, for a hotel site, and to lease the same, with the approval of the Governor General, to a responsible person or corporation for a term not to exceed 99 years." Subsequently, the Philippine Commission passed on 18 May 1907 Act 1657, amending Act 1360, so as to authorize the City of Manila either to lease or to sell the portion set aside as a hotel site. The total area reclaimed was a little over 25 hectares. The City of Manila applied for the registration of the reclaimed area, and on 20 January 1911, OCT 1909 was issued in the name of the City of Manila. On 13 July 1911 the City of Manila, affirming a prior sale dated 16 January 1909, conveyed 5,543.07 sq. m. of the reclaimed area to the Manila Lodge No. 761, Benevolent and Protective Order of Elks of the U.S.A. (BPOE) on the basis of which TCT 2195 was issued to the latter over the parcel of land which was part of Luneta Extension in the District of Ermita. For the remainder of the Luneta Extension, i.e. after segregating therefrom the portion sold to the Manila Lodge No. 761, BPOE, a new Certificate of Title 2196 was issued on 17 July 1911 to the City of Manila. Manila Lodge subsequently sold said 5,543.07 sq. m. to the Elks Club, Inc., to which was issued TCT 67488. The registered owner, "The Elks Club, Inc.," was later changed by court order to "Manila Lodge No. 761, Benevolent and Protective Order of Elks, Inc." In January 1963 the BPOE petitioned the CFI Manila, Branch IV, for the cancellation of the right of the City of Manila to repurchase the property. This petition was granted on 15 February 1963. On 19 November 1963 the BPOE sold the land together with all the improvements thereon to the Tarlac Development Corporation which paid P1,700,000 as down payment and mortgaged to the vendor the same realty to secure the payment of the balance to be paid in quarterly installments. At the time of the sale, there was no annotation of any subsisting lien on the title to the property. On 12 December 1963 TCT 73444 was issued to TDC. In June 1964 the City of Manila filed with the CFI Manila a petition for the reannotation of its right to repurchase; the court, after hearing, issued an order, dated November 19, 1964, directing the Register of Deeds of the City of Manila to reannotate in toto the entry regarding the right of the City of Manila to repurchase the property after 50 years. From this order TDC and BPOE appealed to the Supreme Court which on 31 July 1968 affirmed the trial court's order of reannotation, but reserved to TDC the right to bring another action for the clarification of its rights. On 28 April 1971, as a consequence of such reservation, TDC filed a complaint against the City of Manila and Manila Lodge 761, BPOE with the CFI Manila praying that Entry 4608/T-1635 found in TCT 73444 covering the parcel of land with buildings and improvements thereon purchased from BPOE be cancelled; that Manila pays TDS the sum of P100,000 as damages, that TDC reserve the right to recover amounts from BPOE in case that the judgment on the parcel of land declares it a public park. The City of Manila set up a special and affirmative defense claiming that TDC is not a purchaser in good faith. Manila Lodge 761 admitted to have sold the land in good faith, and that it had received quarterly installments from TDC until 15 October 1964 when the latter failed without justifiable cause to pay the subsequent installments. After due trial the court a quo rendered on 14 July 1972 its decision finding the subject land to be part of the "public park or plaza" and, therefore, part of the public domain. The court consequently declared that the sale of the subject land by the City of Manila to Manila Lodge No. 761, BPOE, was null and void; that plaintiff TDC was a purchaser thereof in good faith and for value from BPOE and can enforce its rights against the latter; and that BPOE is entitled to recover from the City of Manila whatever consideration it had paid the latter. From said decision, TDC and Manila Lodge 761, BPOE appealed to the Court of Appeals. In its decision promulgated on 30 June 1975, the Court of Appeals concurred in the findings and conclusions of the lower court upon the ground that they are supported by the evidence and are in accordance with law, and accordingly affirmed the lower court's judgment. Hence, the petitions for review on certiorari. The Supreme Court denied the petitions for lack of merit, and affirmed the decision of the Court of Appeals, at petitioners cost. 1. Statutory Construction; Courts must give effect to the general legislative intent It is a cardinal rule of statutory construction that courts must give effect to the general legislative intent that can be discovered from or is unraveled by the four corners of the statute, and in order to discover said intent, the whole statute, and not only a particular provision thereof, should be considered. In the present case, it is thus necessary to analyze all the provisions of Act 1360, as amended, in order to unravel the legislative intent. 2. Grant of public nature strictly construed against the grantee The grant made by Act 1360 of the reclaimed land to the City of Manila is a grant of a "public" nature, the same having been made to a local political subdivision. Such grants have always been strictly construed against the grantee. One compelling reason given for the strict interpretation of a public grant is that there is in such grant a gratuitous donation of, public money or resources which results in an unfair advantage to the grantee and for that reason, the grant should be narrowly restricted in favor of the public. This reason for strict interpretation obtains relative to the aforesaid grant for although the City of Manila was to pay for the construction of such work and timber bulkheads or sea walls as may be necessary for the making of the Luneta extension, the area to be reclaimed would be filled at the expense of the Insular Government and without cost to the City of Manila, with material dredged from Manila Bay. Hence, the letter of the statute should be narrowed to exclude matters which if included would defeat the policy of the legislation. 3. Reclaimed area of public dominion, intended for public use The reclaimed area, an extension to the Luneta, is declared to be property of the City of Manila; and is of public dominion, intended for public use. It cannot be patrimonial property as Act 1360, as amended, provides by necessary implication, that the City of Manila could not dispose of the reclaimed area without being authorized by the lawmaking body.

4. Ownership defined Article 348 of the Civil Code of Spain provides that "ownership is the right to enjoy and dispose of a thing without further limitations than those established by law." The right to dispose ( jus disponendi) of one's property is an attribute of ownership. 5. Statutory Construction; every word, clause of statute interpreted in a way that no part becomes inoperative or superfluous If the reclaimed area were patrimonial property of the City, the latter could dispose of it without need of the authorization provided by the statute, and the authorization to set aside, lease, or sell given by the statute would indeed be superfluous. To so construe the statute as to render the term "authorize," which is repeatedly used by the statute, superfluous would violate the elementary rule of legal hermeneutics that effect must be given to every word, clause, and sentence of the statute and that a statute should be so interpreted that no part thereof becomes inoperative or superflous. To authorize means to empower, to give a right to act. Act 1360 furthermore qualifies the verb "authorize" with the adverb "hereby," which means "by means of this statute or section." Hence without the authorization expressly given by Act 1360, the City of Manila could not lease or sell even the northern portion; much less could it dispose of the whole reclaimed area. At most, only the northern portion reserved as a hotel site could be said to be patrimonial property, for, by express statutory provision it could be disposed of, and the title thereto would revert to the City should the grantee fail to comply with the terms provided by the statute. 6. Presumption of full knowledge of prior laws and legislation when lawmaking body enacts a statute It is presumed that when the lawmaking body enacted the statute, it had full knowledge of prior and existing laws and legislation on the subject of the statute and acted in accordance or with respect thereto. If by another previous law, the City of Manila could already dispose of the reclaimed area, which it could do if such area were given to it as its patrimonial property, it would be a superfluity for Act 1360 to authorize the City to dispose of the reclaimed land. 7. Extension to Luneta is also a public park or plaza and for public use The reclaimed area, being an "extension to the Luneta in the City of Manila," it is of the same nature or character as the old Luneta. Anent this matter, it has been said that a power to extend (or continue an act or business) cannot authorize a transaction that is totally distinct. It is not disputed that the old Luneta is a public park or plaza and it is so considered by Section 859 of the Revised Ordinances of the City of Manila. Hence the "extension to the Luneta" must be also a public park or plaza and for public use. 8. Extension defined Extension signifies enlargement in any direction in length, breadth, or circumstance. 9. Bays, roadsteads, coast sea, inlets and shores are part of the national domain open for public use A bay is nothing more than an inlet of the sea. Pursuant to Article 1 of the Law of Waters of 1866, bays, roadsteads, coast sea, inlets and shores are parts of the national domain open to public use. These are also property of public ownership devoted to public use, according to Article 339 of the Civil Code of Spain. In the present case, the reclaimed area was formerly a part of the Manila Bay. 10. When shore or part of bay is reclaimed, it does not lose character of being property for public use When the shore or part of the bay is reclaimed, it does not lose its character of being property for public use, according to Government of the Philippine Islands vs. Cabangis. When the tract of land owned by a private individual wears away and later on is submerged in water in ordinary tides (thus becoming part of the shore), until the Government later on undertakes the dredging of the estuary and dumping the sand and silt from estuary on the low lands completely submerged in water forming the reclaimed lots, they belong to the public domain for public use. Hence, a part of the shore, and for that purpose, a part of the bay, did not lose its character of being for public use after it was reclaimed. 11. Expressio unius est exclusio alterius; Southern portion is not the northern portion authorized to be leased or sold Act 1360, as amended, authorized the lease or sale of the northern portion of the reclaimed area as a hotel site. The subject property is not that northern portion authorized to be leased or sold; the subject property is the southern portion. Hence, applying the rule of expresio unius est exclusio alterius, the City of Manila was not authorized to sell the subject property. The application of this principle of statutory construction becomes the more imperative inasmuch as not only must the public grant of the reclaimed area to the City of Manila be strictly construed against the City of Manila, but also because a grant of power to a municipal corporation, as happens in this case where the city is authorized to lease or sell the northern portion of the Luneta extension, is strictly limited to such as are expressly or impliedly authorized or necessarily incidental to the objectives of the corporation. 12. Property of public use; Intention to consider property for public use important, not actual construction or layout Article 344 of the Civil Code of Spain provides that "property of public use, in provinces and in towns, comprises the provincial and town roads, the squares, streets, fountains, and public waters, the promenades, and public works of general service paid for by such towns or provinces." A park or plaza, such as the extension to the Luneta, is undoubtedly comprised in said article. Properties of provinces and towns for public use are governed by the same principles as properties of the same character belonging to the public domain. In order to be property of public domain an intention to devote it to public use is sufficient. It is not necessary, therefore, that a plaza be already construed or laid out as a plaza in order that it be considered property for public use. It is sufficient that it be intended to be such. 13. Conversion of property of public us to patrimonial property requires explicit declaration by the executive and the legislative department As held in Ignacio vs. The Director of Lands, it is only the executive and possibly the legislative department that has the authority and the power to make the declaration that said property is no longer required for public use, and until such declaration is made the property must continue to form part of the public domain. In the present case, there has been no such explicit or unequivocal declaration. The courts are undoubtedly not primarily called upon, and are not in a position, to determine whether any public land is still needed for the purposes specified in Article 4 of the Law of Waters.

14. Circumstantial evidence far removed in time to be considered contemporaneous to the enactment of Act 1360 All items of alleged circumstantial evidence are acts far removed in time from the date of the enactment of Act 1360 such that they cannot be considered contemporaneous with its enactment. Moreover, it is not far-fetched that this mass of circumstantial evidence might have been influenced by the antecedent series of invalid acts, i.e. the City's having obtained over the reclaimed area OCT 1909 (20 January 1911); the sale made by the City of the subject property to Manila Lodge No. 761; and the issuance to the latter of TCT 2195. It cannot be gainsaid that if the subsequent acts constituting the circumstantial evidence have been based on, or at least influenced, by those antecedent invalid acts and Torrens titles, they can hardly be indicative of the intent of the lawmaking body in enacting Act 1360 and its amendatory act. 15. Mention as boundary owner is not a means of acquiring title OCT 7333 (13 November 1935), covering the lot where the American Embassy [Chancery] stands, states that the property is "bounded on the Northwest by properties of Army and Navy Club and Elks Club." Even if said boundaries were delineated by the Philippine Legislature in Act 4269, this does not mean that the Legislature "recognized and conceded the existence of the Elks Club property as a private property and not as a public park or plaza. A contrary argument is non sequitur, plain and simple. Said Original Certificate of Title cannot be considered as an inconvertible declaration that the Elks Club was in truth and in fact the owner of such boundary lot. Such mention as boundary owner is not a means of acquiring title nor can it validate a title that is null and void. 16. Government not estopped by mistake or errors on the part of its agents The Government is never estopped by mistakes or errors on the part of its agents, and estoppel does not apply to a municipal corporation to validate a contract that is prohibited by law or its against public policy. In the present case, the 13 July 1911 sale executed by the city of Manila to Manila Lodge was certainly a contract prohibited by law. Estoppel cannot be urged even if the City of Manila accepted the benefits of such contract of sale and the Manila Lodge No. 761 had performed its part of the agreement, for to apply the doctrine of estoppel against the City of Manila in this case would be tantamount to enabling it to do indirectly what it could not do directly. 17. Sale void and existent; cannot be ratified by lapse of time or by express ratification The sale of the subject property executed by the City of Manila to the Manila Lodge No. 761, BPOE, was void and inexistent for lack of subject matter. It suffered from an incurable defect that could not be ratified either by lapse of time or by express ratification. The Manila Lodge No. 761 therefore acquired no right by virtue of the said sale. Hence to consider now the contract inexistent as it always has been, cannot be, as claimed by the Manila Lodge No. 761, an impairment of the obligations of contracts, for there was in contemplation of law, no contract at all. 18. Good faith of purchaser cannot create title where none exist The inexistence of said sale can be set up against anyone who asserts a right arising from it, not only against the first vendee but also against all its successors, which are not protected by law. The doctrine of bona fide purchaser without notice does not apply where there is a total absence of title in the vendor, and the good faith of the purchaser cannot create title where none exists. Santos v. Moreno [G.R. No. L-15829. December 4, 1967.] En Banc, Bengzon JP (J): 9 concur Facts: The Zobel family of Spain formerly owned a vast track of marshland in Macabebe, Pampanga called Hacienda San Esteban, which was administered and managed by the Ayala y Cia. From 1860 to 1924 Ayala y Cia., devoted the hacienda to the planting and cultivation of nipa palms from which it gathered nipa sap or "tuba". It operated a distillery plant in barrio San Esteban to turn nipa tuba into potable alcohol which was in turn manufactured into liquor. Accessibility through the nipa palms deep into the hacienda posed as a problem; thus Ayala y Cia dug canals leading towards the hacienda's interior where most of them interlinked with each other. The canals facilitated the gathering of tuba and the guarding and patrolling of the hacienda by security guards called "arundines". By the gradual process of erosion these canals acquired the characteristics and dimensions of rivers. In 1924 Ayala y Cia shifted from the business of alcohol production to bangus culture. It converted Hacienda San Esteban from a forest of nipa groves to a web of fishponds. Sometime in 1925 or 1926 Ayala y Cia., sold a portion of Hacienda San Esteban to Roman Santos who also transformed the swamp land into a fishpond. In so doing, he closed and built dikes across Sapang Malauling Maragul, Quiorang Silab, Pepangebunan, Bulacus, Nigui and Nasi. The closing of the man-made canals in Hacienda San Esteban drew complaints from residents of the surrounding communities. Claiming that the closing of the canals caused floods during the rainy season, and that it deprived them of their means of transportation and fishing grounds, said residents demanded re-opening of those canals. Subsequently, Mayor Lazaro Yambao of Macabebe, accompanied by policemen and some residents went to Hacienda San Esteban and opened the closure dikes at Sapang Malauling Maragul, Nigui and Quiorang Silab. Whereupon, Roman Santos filed Civil Case 4488 in the CFI Pampanga which preliminarily enjoined Mayor Yambao and others from demolishing the dikes across the canals. The municipal officials of Macabebe countered by filing a complaint (Civil Case 4527) in the same court. The CFI Pampanga rendered judgment in both cases against Roman Santos who immediately elevated the case to the Supreme Court. In the meantime, the Secretary of Commerce and Communications conducted his own investigation, found and declared on 8 November 1930 that the streams closed by Roman Santos were natural, floatable and navigable and were utilized by the public for transportation since time immemorial. However, on 8 May 1931 the said official revoked his decision and declared the streams in question privately owned because they were artificially constructed. Subsequently, upon authority granted under Act 3982 the Secretary of Commerce and Communications entered into a contract with Roman Santos whereby the former recognized the private ownership of 6 streams and the latter turned over for public use 2 artificial canals and bound himself to maintain them in navigable state. The Provincial Board of Pampanga and the municipal councils of Macabebe and Masantol objected to the contract. However, the Secretary of Justice, in his opinion dated 6 March 1934, upheld its legality. Roman Santos withdrew his appeals in the Supreme Court.

On 25 February 1935 the municipality of Macabebe and the Zobel family executed an agreement whereby they recognized the nature of the streams mentioned in Panopio's report as public or private, depending on the findings in said report. This agreement was approved by the Secretary of Public Works and Communications on 27 February1935 and confirmed the next day by the municipal council of Macabebe under Resolution 36. On 12 June 1935 however, the Secretary of Justice issued an opinion holding that the contract executed by the Zobel family and the municipality of Macabebe has no validity. Still, despite the ruling of the Secretary of Justice, the streams in question remained closed. In 1939 administrative investigations were again conducted by various agencies of the Executive branch of our government culminating in an order of President Manuel Quezon immediately before the national elections in 1941 requiring the opening of Sapang Macanduling Maragul, Macabacle, Balbaro and Cansusu. Said streams were again closed in 1942 allegedly upon order of President Quezon. Roman Santos acquired in 1940 from the Zobel family a larger portion of Hacienda San Esteban wherein are located 25 streams which were closed by Ayala y Cia. 18 years later or in 1950, Congress enacted RA 2056. Thereafter, on 15 August 1958, Senator de la Rosa requested in writing the Secretary of Public Works and Communications to proceed in pursuance of Republic Act No. 2056 against fishpond owners in the province of Pampanga who have closed rivers and appropriated them as fishponds without color of title. On the same day, Benigno Musni and other residents in the vicinity of Hacienda San Esteban petitioned the Secretary of Public Works and Communications to open the following streams: Balbaro, Batasan Matua, Bunga, Cansusu, Macabacle, Macanduling Maragul, Mariablus Malate, Matalabang Maisac, Nigui, Quiorang Silab, Sapang Maragul and Sepung Bato. On 20 October 1958 Musni and his co-petitioners amended their petition to include other streams: Balbaro, Balili, Banawa, Batasan Matua, Bato, Bengco, Bunga, Butabuta, Camastiles, Cansusu, Cela, Don Timpo, Mabalanga, Mabutol, Macabacle, Macabacle qng. Iba, Macanduling Maragul, Malauli, Magasawa, Mariablus Malate, Masamaral, Matalabang Maisac, Mariablus, 3 Nigui, Pita, Quiorang Silab, Sapang Maragul, Sepung Bato, Sinag and Tumbong. On March 2, 4, 10, 30 and 31, and 1 April 1959, the Secretary of Public Works and Communications rendered his decisions ordering the opening and restoration of the channel of all the streams except Sapang Malauling Maragul, Quiorang Silab, Nigui, Pepangebonan, Nasi and Bulacus, within 30 days. On 29 April 1959, after receipt of the Secretary's decision, Roman Santos filed a petition with the CFI Manila for injunction against the Secretary of Public Works and Communications and Julian C. Cargullo. As prayed for, preliminary injunction was granted on 8 May 1959. On April 29 and 12 June 1959, Roman Santos received the decision of the Secretary of Public Works and Communications dated March 10 and March 30, March 31, and 1 April 1959. Consequently, on June 24, 1959 he asked the court to cite in contempt Secretary Florencio Moreno, Undersecretary M. D. Bautista and Julian Cargullo for issuing and serving upon him the said decisions despite the existence of the preliminary injunction. The Cou rt however ruled that Secretary Moreno, Undersecretary Bautista and Cargullo acted in good faith, and hence were merely "admonished to desist from any and further action in this Court, with the stern warning, however, that a repetition of the acts complained of shall be dealt with severely." On 18 July 1959 the trial court declared all the streams under litigation private, and made the writ of preliminary injunction permanent. The Secretary of Public Works and Communications and Julian Cargullo appealed to the Supreme Couurt from the order of 17 July 1959 issued in connection with Roman Santos' motion for contempt and from the decision of the lower court on the merits of the case. The Supreme Court affirmed the decision appealed from, except as to Sapang Cansusu which was declared public and thus as to which the judgment of the lower court was reversed. No costs. 1. Motion for reconsideration not required as a condition precedent to judicial relief in RA 2056; Congress intend decision of Secretary of Public Works and Communication to be final and executory subject to a timely review by the courts RA 2056 does not require the filing of a motion for reconsideration as a condition precedent to judicial relief. From the context of the law, the intention of the legislators to forego a motion for reconsideration manifests itself clearly. RA 2056 underscores the urgency and summary nature of the proceedings authorized thereunder. Thus in Section 2 thereof the Secretary of Public Works and Communications under pain of criminal liability is duty bound to terminate the proceedings and render his decision within a period not exceeding 90 days from the filing of the complaint. Under the same section, the party respondent concerned is given not more than 30 days within which to comply with the decision of the Secretary of Public Works and Communications, otherwise the removal of the dams would be done by the Government at the expense of said party. Congress has precisely provided for a speedy and a most expeditious proceeding for the removal of illegal obstructions to rivers and on the basis of such a provision it would be preposterous to conclude that it had in mind to require a party to file a motion for reconsideration an additional proceeding which would certainly lengthen the time towards the final settlement of existing controversies. The logical conclusion is that Congress intended the decision of the Secretary of Public Works and Communications to be final and executory subject to a timely review by the courts without going through formal and time consuming preliminaries. 2. Question of Constitutionality rightly aired before a competent court; not within competence of Secretary of Public Works and Communications The petitioner assailed the constitutionality of RA 2056 and the jurisdiction of the Secretary of Public Works and Communications to order the demolition of dams across rivers or streams. Those questions are not within the competence of said Secretary to decide upon a motion for reconsideration. They are purely legal questions, not administrative in nature, and should properly be aired before a competent court as was rightly done by the petitioner. 3. Appeal of the decision of the Secretary to the President dispensed with as Secretary is alter-ego of the President As to the failure of Roman Santos to appeal from the decision of the Secretary of Public Works and Communications to the President of the Philippines, suffice it to state that such appeal could be dispensed with because said Secretary is the alter ego of the President. The actions of the former are presumed to have the implied sanction of the latter. 4. Action correctly filed with the CFI Manila; Purpose is to review the decision of the Secretary, even if the resolution of controversy rests in the ownership of the streams; Section 1 of Rule 5, not section 3, is controlling The mere fact that the resolution of the controversy would wholly rest on the ownership of the streams involved would not

necessarily classify it as a real action. The purpose of the suit is to review the decisions of the Secretary of Public Works and Communications, to enjoin him from enforcing them and to prevent him from making and issuing similar decisions concerning the streams in Hacienda San Esteban. The acts of the Secretary of Public Works are Communications are the object of the litigation, hence, the suit ought to be filed in the CFI whose territorial jurisdiction encompasses the place where the Secretary is found or is holding office. For the rule is that outside its territorial limits, the court has no power to enforce its orders. Section 3 of Rule 5 of the Rules of Court does not apply to determine venue of this action, Section of the same rule applies. Section 1 provides that civil actions in CFIs may be commenced and tried where the defendant or any of the defendants resides or may be found, or where the plaintiff or any of the plaintiffs resides, at the election of the plaintiff." Thus, in the present case, the petition for injunction was correctly filed in the CFI Manila as the Secretary of Public Works and Communications and Julian Cargullo are found and hold office in the City of Manila. 5. Review of Secretarys decision by the court; inquiry limited to evidence presented during the administrative proceedings Whether the action instituted in the CFI be for mandamus, injunction or certiorari is not very material. In reviewing the decision of the Secretary of Public Works and Communications, the CFI shall confine its inquiry to the evidence presented during the administrative proceedings. Evidence not presented therein shall not be admitted and considered by the trial court. As held previously by the Court, "the findings of the Secretary can not be enervated by new evidence not laid before him, for that would be tantamount to holding a new investigation, and to substitute for the discretion and judgment of the Secretary the discretion and judgment of the court, to whom the statute had not entrusted the case. It is immaterial that the action should be one for prohibition or injunction and not one for certiorari; in either event the case must be resolved upon the evidence submitted to the Secretary, since a judicial review of executive decisions does not import a trial de novo, but only an ascertainment of whether the executive findings are not in violation of the Constitution or of the laws, and are free from fraud or imposition, and whether they find reasonable support in the evidence. Thus, it was an error for the lower court to conduct a trial de novo. 6. RA 2056 is not unconstitutional The Court has held in Lovina v. Moreno that said law is constitutional. It cannot be held that the law is constitutional but applied unconstitutionally as the petitioners dikes were demolished through an administrative, not judicial, proceeding. Such conclusion amount in effect to declaring the law unconstitutional, stated inversely. Note that the law provides for an expeditious administrative process to determine whether or not a dam or dike should be declared a public nuisance and ordered demolished. And to say that such an administrative process, when put to operation, is unconstitutional is tantamount to saying that the law itself violates the Constitution. 7. RA 2056 applies to 2 types of bodies of water RA 2056 applies to two types of bodies of water, namely, (1) public navigable rivers, streams, coastal waters, waters or waterways and (b) areas declared as communal fishing grounds. 8. Building of dams, dikes or other works on navigable public waters a public nuisance Section 1 of RA 2056 law provides that the construction or building of dams, dikes or any other works which encroaches into any public navigable river, stream, coastal waters and any other navigable public waters or waterways as well as the construction or building of dams, dikes or any other works in areas declared as communal fishing grounds, shall be ordered removed as public nuisances or as prohibited constructions as herein provided. 9. Montano v. Insular Government (marchlands not susceptible to appropriation by occupation) not applicable The doctrine in Montano vs. Insular Government, that a marshland which is inundated by the rise of tides belong to the State and is not susceptible to appropriation by occupation, has no application in the present ccase inasmuch as in said case the land subject matter of the litigation was not yet titled and precisely Isabelo Montano sought title thereon on the strength of 10 years' occupation pursuant to paragraph 6, section 5 of Act 926 of the Philippine Commission. Whereas, Hacienda San Esteban is titled land and private ownership thereof by Ayala y Cia, has been recognized by the King of Spain and later by the Philippine Government when the same was registered under Act 496. 10. Injunction proper if person constructs a dam across a public canal, which is situated within a public land "No private person has a right to usurp possession of a watercourse, branch of a river, or lake of the public domain and use, unless it shall have been proved that he constructed the same within property of his exclusive ownership, and such usurpation constitutes a violation of the legal provisions which explicitly exclude such waterways from the exclusive use or possession of a private party." (Bautista v. Alarcon, 23 Phil 631) Inversely, and as indicated in said case, a private person may take possession of a watercourse if he constructed the same within his property. 11. Public and Private ownership under the Spanish Civil Code of 1889; Spanish Law of Waters of 1866 As to Public Ownership; Articles 339 of the Spanish Civil Code of 1889 provides that property of public ownership includes that devoted to public use, such as roads, canals, rivers, torrents, ports and bridges constructed by the State, riverbanks, shores, roadsteads, and that of a similar character." Article 407 of the same Code provides that (1) Rivers and their natural channels; (2) Continuous or intermittent waters from springs or brooks running in then natural channels and the channels themselves; (3) Waters rising continuously or intermittently on lands of public; (4) Lakes and ponds formed by nature on public lands, and their beds; (5) Rain waters running through ravines or sand beds, the channels of which are of public ownership; (6) Subterranean waters on public lands; (7) Waters found within the zone of operation of public works, even though constructed under contract; Waters which flow continuously or intermittently from lands belonging to private persons, to the State, to provinces, or to towns from the moment they leave such lands; and (9) The waste waters of fountains, sewers, and public institutions are of public ownership. Further, Article 72 of the Spanish Law of Waters (8 August 1866) provides that the water-beds on public land, of creeks through which spring waters run, are a part of the public domain. The natural water-beds or channels of rivers are also part of the public domain." As to Private Ownership; Article 408 of the Spanish Civil Code of 1889 provides that (1) Waters, either continuous or intermittent rising on private estates, while they run through them; (2) Lakes and ponds and their beds when formed by nature on

such estates; (3) Subterranean waters found therein; (4) Rain waters falling thereon as long as they remain within their boundaries; and (5) The channels of flowing streams, continuous or intermittent formed by rain water, and those of brooks crossing estates which are not of public ownership are of private ownership. Further, Article 71 of the Spanish Law of Waters (8 August 1866) provides that the water-beds of all creeks belong to the owners of the estates or lands over which they flow." Further, "the water, bed, banks, and floodgates of a ditch or aqueduct are deemed to be an integral part of the estate or building for which the waters are intended. The owners of estates through or along the boundaries of which the aqueduct passes can assert no ownership over it, nor any right to make use of its beds or banks, unless they base their claim on title deed which specify the right or the ownership claimed." 12. Character of canals based on the estate over which they flow; Canals of private ownership Pursuant to Article 71 of the Spanish Law of Waters of 3 August 1866, and Article 408(5) of the Spanish Civil Code, channels of creeks and brooks belong to the owners of estates over which they flow. The channels, therefore, of the streams in question which may be classified as creeks, belong to the owners of Hacienda San Esteban. Further, the said streams, considered as canals, of which they originally were, are of private ownership in contemplation of Article 339(1) of the Spanish Civil Code. Canals constructed by private persons within private lands and devoted exclusively for private use must be of private ownership. The streams, except for Sapang Cansusu, being artificial and devoted exclusively for the use of the hacienda owner and his personnel, are of private ownership and hence, the dams across them should not be ordered demolished as public nuisances. 13. Mercado v. Municipal President of Macabebe not the same as present case In Mercado v. Municipal President of Macabebe, the Batasan-Limasan creek was originally dug by the estate owner who, subsequently allowed said creek to be used by the public for navigation and fishing purposes for a period of 22 years. The creek could have been of private ownership had not its builder lost it by prescription. Applying the principle therein enunciated to the present case, the conclusion would be inevitably in favor of private ownership, considering that the owners of Hacienda San Esteban held them for their exclusive use and prohibited the public from using them. 14. Mercado case cannot be applied in the present case due to differences in factual premises In his opinion of 12 June 1935, the Secretary of Justice answered in the negative the query of the Secretary of Public Works and Communications whether the latter can declare of private ownership those streams which "were dug up artificially", because it was assumed that the streams were used "by the public as fishing ground and in transporting their commerce in bancas or in small crafts without the objection of the parties who dug" them (applying Mercado v. Municipality of Macabebe). However, the facts, as then found by the Bureau of Public Work, do not support the factual premise that the streams in question were used by the public "without the objection of the parties who dug" them. The Court cannot therefore take as controlling in determining the merits of this case the factual premises and the legal conclusion contained in said opinion. 15. Case different from cases involving dammed natural navigational streams The present case should be differentiated from those cases where the Court held illegal the closing and/or appropriation of rivers or streams by owners of estates through which they flow for purposes of converting them into fishponds or other works. In those cases, the watercourses which were dammed were natural navigable streams and used habitually by the public for a long time as a means of navigation. Consequently, they belong to the public domain either as rivers pursuant to Article 407(1) of the Spanish Civil Code of 1889 or as property devoted to public use under Article 339 of the same Code. Whereas, the streams involved in the present case were artificially made and denoted to the exclusive use of the hacienda owner. 16. Sapang Cansusu is a natural stream, a public stream, which belongs to the public domain Sapang Cansusu, being a natural stream and a continuation of the Cansusu River, admittedly a public stream, belongs to the public domain. Its closure therefore by the predecessors of Roman Santos was illegal. 17. Case involving petition for the opening of Sapang Malauling, and other streams, closed; Parties thereto excluded in present case The petition for the opening of Sapang Malauling Maragul, Quiorang Silab, Nigui, Pepangebunan, Nasi and Bulacus was dismissed by the Secretary of Public Works and Communications and the case considered closed. The said administrative decision has not been questioned in this appeal by either party. Hence, they are deemed excluded herein. 18. Lower court made no finding of contempt to be review by the Supreme Court With respect to the issue of contempt of court on the part of the Secretary of Public Works and Communications and Julian Cargullo for the alleged issuance of administrative decisions ordering demolition of dikes involved in this case after the writ of injunction was granted and served, suffice it to state that the lower court made no finding of contempt of court. Necessarily, there is no conviction for contempt reviewable by this Court and any discussion on the matter would be academic. Mercado v. Municipal President of Macabebe [G.R. No. 37986. March 1, 1934.] Second Division, Diaz (J): 4 concur Facts: Mariano Mercado, the original owner of the hacienda, in order to facilitate the cutting and transportation of firewood and other products, produced on the said hacienda, towards the Nasi River on the east or towards Limasan creek on the west, connected the two recesses or bodies of water in question by means of excavation and, after having so connected them, made other excavations at both ends towards the said directly connecting both bodies of water, and which later became known as the Batasan-Limasan or Pinac Bugalun creek. Said Batasan-Limasan or Pinac Bugalun creek or canal already existed at the time of the institution of the registration proceedings wherein judgment was rendered resulting in the issuance of certificate of title 329 in favor of Romulo Mercado. On the plan of the land, which was presented in the said case, the aforesaid creek appears; and at the time the case was tried as well as when the certificate of title was issued in favor of the Romulo Mercado, none of the herein defendants nor the Insular Government filed opposition or objection thereto.

Once the said Batasan-Limasan or Pinac Bugalun creek or canal was opened from the Nasi River to Limasan creek, not only the residents of the hacienda and those who visited it but also some of the residents of the nearby barrios and municipalities began to use it as a means of communication in attending to their needs, sometimes with the permission of the owners of the hacienda, and at other times without even the latter's knowledge. It was then that Romulo Mercado, the appellant's (Eufemia Mercado) predecessor in interest, decided to convert the said creek into a fish pond and with that object in view, in 1928 he closed the two openings thereof towards the Nasi River on one side and Limasan creek on the other side. The Secretary of Commerce ordered Romulo Mercado to remove the two dikes which he had constructed at both ends of the creek named Batasan-Limasan or Pinac Bugalun, which traverses part of the hacienda described in certificate of title 329 of the registry of deeds of Pampanga, and formerly belonging to Romulo Mercado, but which now belonging to the Eufenua by virtue of a formal donation made to her. Eufemia Mercado appealed the order of the Secretary of Commerce with the CFI Pampanga; which later dismissed said appeal, holding the creek in question as property of the public domain. Hence, the present petition. The Supreme Court affirmed the judgment appealed from, with costs against the appellant. 1. Notice cannot constitute sufficient evidence A mere notice does not constitute sufficient evidence that the creek in question is the property of the public domain. Such evidence cannot be given any weight, as it was not established that anyone ever obtained the privilege of fishing in the BatasanLimasan creek, such privilege being placed at public auction. 2. Public and Private ownership under the Spanish Civil Code of 1889 a. As to Public Ownership; Articles 339 of the Spanish Civil Code of 1889 provides that property of public ownership includes that devoted to public use, such as roads, canals, rivers, torrents, ports and bridges constructed by the State, riverbanks, shores, roadsteads, and that of a similar character." Article 407 of the same Code provides that (1) Rivers and their natural channels; (2) Continuous or intermittent waters from springs or brooks running in then natural channels and the channels themselves; (3) Waters rising continuously or intermittently on lands of public; xxx (8) Waters which flow continuously or intermittently from lands belonging to private persons, to the State, to provinces, or to towns from the moment they leave such lands; xxx are of public ownership. As to Private Ownership; Article 408 of the Spanish Civil Code of 1889 provides that (1) Waters, either continuous or intermittent rising on private estates, while they run through them; (2) Lakes and ponds and their beds when formed by nature on such estates; (3) Subterranean waters found therein; (4) Rain waters falling thereon as long as they remain within their boundaries; and (5) The channels of flowing streams, continuous or intermittent formed by rain water, and those of brooks crossing estates which are not of public ownership are of private ownership. Further, "the water, bed, banks, and floodgates of a ditch or aqueduct are deemed to be an integral part of the estate or building for which the waters are intended. The owners of estates through or along the boundaries of which the aqueduct passes can assert no ownership over it, nor any right to make use of its beds or banks, unless they base their claim on title deed which specify the right or the ownership claimed."

b.

c.

3. Article 408(5) speaks of channel of creeks and not the creek itself Appellant cannot invoke in her favor the Article 408 (5) on the ground that although it is true that the Batasan-Limasan or Pinac Bugalun creek passes through her hacienda, it is none the less true that it is not included in any of the kinds of private property therein enumerated. The appellant and her predecessors in interest, in closing the two openings of the said creek and converting it into a fish pond, not only appropriated for themselves the channel of the said creek but also the creek itself. 4. Creek and brook distinguished A creek is not a brook because the latter is but a short, almost, continuous stream of water (Diccionario de la Real Academia Espaola), while the former is a recess or arm extending from a river, which participates in the ebb and flow of the sea. (15 Enciclopedia Juridica Espaola, 216.) 5. Creeks are property of public domain Article 339 provides that canals, rivers, torrents, and those of a similar character are property of public ownership, and the similarity between rivers, canals, and creeks undoubtedly obvious on the ground that, as has been stated, a creek is no other than arm extending from a river. Furthermore, under article 407, the Batasan-Limasan or Pinac Bugalun creek may be considered as belonging to the class of property enumerated in paragraph 8 thereof. And, in addition to the foregoing, the Contentious Court of Spain (Tribunal Contencioso de Espaa) in a decision dated June 25, 1890, laid down the doctrine that creeks are property of the public domain (15 Enciclopedia Juridica Espaola, 216). 6. Use and enjoyment of a creek may be acquired or lost through prescription Even granting that the Batasan-Limasan creek acquired the proportions which it had, before it was closed, as a result of excavations made by the laborers of the appellant's predecessor in interest, it being a fact that, since the time it was opened as a water route between the Nasi River and Limasan creek, the owners thereof as well as strangers, that is, both the residents of the hacienda and those of other nearby barrios and municipalities, had been using it not only for their bancas to pass through but also for fishing purposes, if the appellant and her predecessors in interest had acquired any right to the creek in question by virtue of excavations which they had made thereon, they had lost such right through prescription inasmuch as they failed to obtain, and in fact they have not obtained, the necessary authorization to devote it to their own use to the exclusion of all others. The use and enjoyment of a creek, as any other property susceptible of appropriation, may be acquired or lost through prescription, and the appellant and her predecessors in interest certainly lost such right through the said cause, and they cannot now claim it exclusively for themselves after the general public had been openly using the same from 1906 to 1928. 7. Title does not confer any right to the creek It is useless for the appellant to allege that she has obtained certificate of title 329 in her favor because the said certificate does not

confer upon her any right to the creek in question, inasmuch as the said creek, being of the public domain, is included among the various exceptions enumerated in section 39 of Act 496 to which the said certificate is subject by express provision of the law, and furthermore, because it so appears in the certificate itself. 8. Government of the Philippine Islands v. Santos does not apply in present case The doctrine laid down in the case of the Government of the Philippine Islands vs. Santos (G.R. No. 27202, promulgated September 2, 1927, not reported) is not applicable in the present case because the subject matter in that case is not of the same nature as the Batasan-Limasan or Pinac Bugalun creek. The thing involved therein was simply a date or, in the words of the trial court, a low depression on the defendant's land where there was a waterway passable by bancas at high tide, but which completely dried up at low tide and during the dry season. The thing involved herein, the Batasan-Limasan creek, is perfectly navigable by bancas throughout the year. Furthermore, in the case of Urbano Santos, the creek in question was closed a few years after excavations had been made in the land under consideration. Binalay v. Manalo [G.R. No. 92161. March 18, 1991.] Third Division, Feliciano (J): 4 concur Facts: The late Judge Taccad originally owned a parcel of land situated in Tumauini, Isabela having an estimated area of 20 hectares. The western portion of this land bordering on the Cagayan River has an elevation lower than that of the eastern portion which borders on the national road. Through the years, the western portion would periodically go under the waters of the Cagayan River as those waters swelled with the coming of the rains. The submerged portion, however, would re-appear during the dry season from January to August. It would remain under water for the rest of the year. On 9 May 1959, Guillermo Manalo acquired 8.65 hectares thereof from Faustina Taccad, daughter of Judge Juan Taccad (N: Francisco Forto, E: National Road, S: Julian Tumolva, W: Cagayan River). In 1964, Manalo purchased another 1.80 hectares from Gregorio Taguba who had earlier acquired the same from Judge Taccad (N: Balug Creek, S: Faustina Taccad [now Manalo], E: Provincial Road, W: Cagayan river). On 21 October 1969, during the rainy season, the two parcels of land were consolidated as one lot during the cadastral survey at Balug, Tamauini, Isabela (Lot 307, 1.8 hectares + 2.9489 hectares of 8.65 hectares purchased; the other portion under water was left unsurveyed). The Cagayan River running from south to north, forks at a certain point to form two (2) branches the western and the eastern branches and then unites at the other end, further north, to form a narrow strip of land. The eastern branch of the river cuts through the land of and is inundated with water only during the rainy season. The bed of the eastern branch is the submerged or the unsurveyed portion of the land belonging to Manalo. For about 8 months of the year when the level of water at the point where the Cagayan River forks is at its ordinary depth, river water does not flow into the eastern branch. While this condition persists, the eastern bed is dry and is susceptible to cultivation. The elongated strip of land formed by the western and the eastern branches of the Cagayan River looked very much like an island. This strip of land was surveyed on 12 December 1969. It was found to have a total area of 22.7209 hectares and was designated as Lot 821 (10.8122 hectares) and Lot 822 (11.9087).. Lot 821 is located directly opposite Lot 307 and is separated from the latter only by the eastern branch of the Cagayan River during the rainy season and, during the dry season, by the exposed, dry river bed, being a portion of the land bought from Faustina Taccad. Manalo claims that Lot 821 also belongs to him by way of accretion to the submerged portion of the property to which it is adjacent. Binalay, et.al., other hand, who are in possession of Lot 821 insist that they own Lot 821. They occupy the outer edges of Lot 821 along the river banks, i.e., the fertile portions on which they plant tobacco and other agricultural products. They also cultivate the western strip of the unsurveyed portion during summer. This situation compelled Manalo to file a case for forcible entry against petitioners on 20 May 1969. The case was dismissed by the Municipal Court of Tumauini, Isabela for failure of both parties to appear. On 15 December 1972, Manalo again filed a case for forcible entry against petitioners. The latter case was similarly dismissed for lack of jurisdiction by the Municipal Court of Tumauini, Isabela. On 24 July 1974, Manalo filed a complaint before the then CFI Isabela, Branch 3 for quieting of title, possession and damages against petitioners. He prayed that judgment be entered ordering petitioners to vacate the western strip of the unsurveyed portion, and prayed that judgment be entered declaring him as owner of Lot 821 on which he had laid his claim during the survey. On 10 November 1982, the trial court rendered a decision declaring Manalo as the lawful owner of Lot 821 and ordering Binalay, et.al. to vacate the premises of Lot 821 and restraining them further from entering said premises; without pronouncement as to costs. Binalay, et.al. appealed to the Court of Appeals which, however, affirmed the decision of the trial court. They filed a motion for reconsideration, without success. In effect, both courts rejected the assertion that the depression on the earth's surface which separates Lot 307 and Lot 821 is, during part of the year, the bed of the eastern branch of the Cagayan River. The Supreme Court set aside the decision of Court of Appeals in CA-GR CV 04892, declared Manalo as the owner of Lot 307, and declared that the regularly submerged portion or the eastern bed of the Cagayan River to be property of public dominion. The Court also declared that the ownership of Lot 821 shall be determined in an appropriate action that may be instituted by the interested parties inter se; without pronouncement as to costs. 1. Finding of facts by lower courts entitled to great respect; Whether the conclusion reached thereafter is correct is a question of law It is a familiar rule that the findings of facts of the trial court are entitled to great respect, and that they carry even more weight when affirmed by the Court of Appeals. This is in recognition of the peculiar advantage on the part of the trial court of being able to observe first-hand the deportment of the witnesses while testifying. Jurisprudence is likewise settled that the Court of Appeals is the final arbiter of questions of fact. But whether a conclusion drawn from such findings of facts is correct, is a question of law cognizable by the Supreme Court. In the present case, the conclusion reached by both courts below apparently collides with their findings that periodically at the onset of and during the rainy season, river water flows through the eastern bed of the Cagayan River.

2. Government of the Philippine Islands v. Colegio de San Jose not applicable to present case Government of the Philippine Islands vs. Colegio de San Jose is not applicable to the present case. That case involved Laguna de Bay; since Laguna de Bay is a lake, the Court applied the legal provisions governing the ownership and use of lakes and their beds and shores, in order to determine the character and ownership of the disputed property. Specifically, the Court applied the definition of the natural bed or basin of lakes found in Article 74 of the Law of Waters of 3 August 1866. Upon the other hand; what is involved in the instant case is the eastern bed of the Cagayan River. 3. Article 70 of the Law of Waters applicable, not Article 74 Article 70 of the Law of Waters of 3 August 1866 is the law applicable to the present case. Article 70 provides that the natural bed or channel of a creek or river is the ground covered by its waters during the highest floods". Article 70 defines the natural bed or channel of a creek or river as the ground covered by its waters during the highest floods. The highest floods in the eastern branch of the Cagayan River occur with the annual coming of the rains as the river waters in their onward course cover the entire depressed portion. The conclusion of the Court that the depressed portion is a river bed rests upon evidence of record. The description of the lot acquired from Taguba and the other from Taccad refer to the dried up bed or the eastern branch of the river as the Cagayan River serving as the western boundary in the Deeds of Sale. Further, Manalo himself, during direct examination, depict the depressed portion separating Lot 821 and Lot 307 as a river bed. The dike-like slope of such depression, or such topographic feature, is compatible with the fact that huge volume of water passes through the eastern bed regularly during the rainy season. Even if there is no record of when the Cagayan River began to carve its eastern channel, the bed already existed even before the sale of the land to Manalo (with the bed being referred to as old bed or even Rio Muerte de Cagayan). 4. Private ownership of the bed of a river cannot be acquired as the land constituted property of public dominion; Article 420 applies to existing beds, Article 462 applies to new beds in relation to Article 457 (Accretion) Pursuant to Article 420 of the Civil Code, Manalo did not acquire private ownership of the bed of the eastern branch of the river even if it was included in the deeds of absolute sale executed by Gregorio Taguba and Faustina Taccad in his favor. These vendors could not have validly sold land that constituted property of public dominion. Article 420 of the Civil Code states that (1) those intended for public use, such as roads, canals, rivers, torrents, ports and bridges constructed by the State, banks, shores, roadsteads, and others of similar character; and (2) those which belong to the State, without being for public use, and are intended for some public service or for the development of the national wealth" are property of public dominion. Although Article 420 speaks only of rivers and banks, "rivers" is a composite term which includes: (1) the running waters, (2) the bed, and (3) the banks. Manresa, in commenting upon Article 339 of the Spanish Civil Code of 1889 from which Article 420 of the Philippine Civil Code was taken, stressed the public ownership of river beds. Still, evven if it were alleged and proved that the Cagayan River first began to encroach on his property after the purchase from Gregorio Taguba and Faustina Taccad; Article 462 of the Civil Code would then apply divesting, by operation of law, Manalo of private ownership over the new river bed. 5. Accretion, requisites Accretion as a mode of acquiring property under Article 457 of the Civil Code requires the concurrence of three (3) requisites: (a) that the deposition of soil or sediment be gradual and imperceptible; (b) that it be the result of the action of the waters of the river (or sea); and (c) that the land where accretion takes place is adjacent to the banks of rivers (or the sea coast). In the present case, the Court notes that the parcels of land bought by Manalo border on the eastern branch of the Cagayan River. Any accretion formed by this eastern branch which Manalo may claim must be deposited on or attached to Lot 307. As it is, the claimed accretion (Lot 821) lies on the bank of the river not adjacent to Lot 307 but directly opposite Lot 307 across the river. 6. Alluvial process is slow and gradual, not sudden and forceful Assuming (arguendo only) that the Cagayan River referred to in the Deeds of Sale transferring ownership of the land to Manalo is the western branch, the decision of the Court of Appeals and of the trial court are bare of factual findings to the effect that the land purchased by Manalo received alluvium from the action of the river in a slow and gradual manner. On the contrary, the flooding, that caused the land to reappear making it susceptible to cultivation, is sudden and forceful action, and is hardly the alluvial process contemplated under Article 457 of the Civil Code. It is the slow and hardly perceptible accumulation of soil deposits that the law grants to the riparian owner. 7. Size of land considered alluvium and topography of land negates conclusion of increment It is important to note that Lot 821 has an area of 11.91 hectares. Lot 821 (11.91 hectares) is the northern portion of the strip of land having a total area of 22.72 hectares. It is difficult to suppose that such a sizable area as Lot 821 resulted from slow accretion to another lot of almost equal size. The total landholding purchased by Manalo is 10.45 hectares, even smaller than Lot 821 which he claims by way of accretion. Further, there are steep vertical dike-like slopes separating the depressed portion or river bed and Lot 821 and Lot 307. This topography of the land, among other things, precludes a reasonable conclusion that Lot 821 is an increment to the depressed portion by reason of the slow and constant action of the waters of either the western or the eastern branches of the Cagayan River. 8. Quieting of title requires equitable title or interest in subject real property Under Article 477 of the Civil Code, the plaintiff in an action for quieting of title must at least have equitable title to or interest in the real property which is the subject matter of the action. The evidence of record on this point is less than satisfactory, such as that both parties claim adverse possession of Lot 821 and both parties presenting tax declarations on the subject land; thus the Court feels compelled to refrain from determining the ownership and possession of Lot 821, adjudging neither petitioners nor respondent Manalo as owner(s) thereof. Hilario v. City of Manila [GR No. L-19570 April 27, 1967] Bengzon JP (J): 8 concur Facts: Dr. Jose Hilario was the registered owner of a large tract of land around 49 hectares in area (Barrio Guinayang, San Mateo, Rizal). Upon his death this property was inherited by his son, Jose Hilario, Jr., to whom a new certificate of title was issued. During the lifetime of plaintiff's father, the Hilario estate was bounded on the western side by the San Mateo River.3 To prevent

its entry into the land, a bamboo and lumber post dike or ditch was constructed on the northwestern side. This was further fortified by a stonewall built on the northern side. For years, these safeguards served their purpose. However, in 1937, a great and extraordinary flood occurred which inundated the entire place including the neighboring barrios and municipalities. The River destroyed the dike on the northwest, left its original bed and meandered into the Hilario estate, segregating from the rest thereof a lenticular piece of land. The disputed area is on the eastern side of this lenticular strip which now stands between the old riverbed site and the new course. In 1945, the US Army opened a sand and gravel plant within the premises, and started scraping, excavating and extracting soil, gravel and sand from the nearby areas along the River. The operations eventually extended northward into the strip of land. Consequently, a claim for damages was filed with the US War Department by Luis Hidalgo, the then administrator of Dr. Hilario's estate. The US Army paid. In 1947, the plant was turned over to herein defendants-appellants and appellee who took over its operations. On 22 October 22, 1949, plaintiff filed his complaint for injunction and damages against the defendants City Engineer of Manila, District Engineer of Rizal, the Director of Public Works, and Engr. Busuego, the Engineer-in-charge of the plant. Subsequently, the Bureau of Mines and Atty. Maximo Calalang were respectively allowed to join the litigation as intervenors; as per issue of fees and penalties for materials (sand and gravel) extracted. On 14 March 1954, defendants filed a petition for injunction against plaintiff and intervenor Calalang in the same case, alleging that the latter have fenced off the disputed area in contravention of an agreement had between the latter and the Director of Public Works wherein the defendants were allowed to continue their operations but subject to the final outcome of the pending suit. On 13 May 1954, plaintiff amended his complaint and impleaded as additional defendants the City of Manila, the Provincial Treasurer of Rizal, and Engr. Eulogio Sese, the new Engineer-incharge of the plant. Plaintiff also converted his claim to one purely for damages directed against the City of Manila and the Director of Public Works, solidarily, in the amount of P1,000,000.00, as the cost of materials taken since 1949, as well as those to be extracted therefrom until defendants stop their operations. On 21 December 1956, the lower court rendered its decision, ordering the City of Manila and Director of Public Works to pay Hilario in solidum the sum of P376,989.60 as cost of gravel and sand extracted from the plaintiffs land, plus costs; and ordering the Provincial Treasurer of Rizal to reimburse intervenor Calalang of P36.80 representing gravel fees illegally collected. None of the parties litigants seemed satisfied with this decision and they all sought a reconsideration of the same. On August 30, 1957, the lower court resolved the motions to reconsider with an order, holding that the 2/5 portion of the area in controversy to Hilario, and dismissing the case against the Bureau of Public Works insofar as money claims are concerned without prejudice to Hilario taking action against proper party in such claim. Hilario and Calalang filed a second motion for reconsideration, which the lower court denied. Hence, the appeal. The Supreme Court set aside the decision and orders appealed from, and entered another judgment to the effect that the City of Manila and the Director of Public Works, and his agent and employees, are absolved of liability from extracting materials from subject property (of public domain); and the portion within the strip of land question declared not part of public domain and confirmed as part of Hilarios private property. No Costs. 1. Old Civil Code and Law of Waters of 1866 controlling law Since the change in the course of the River took place in 1937, long before the present Civil Code took effect, the question should be determined in accordance with the provisions of the old Civil Code and those of the Law of Waters of 3 August 1866. 2. All riverbanks, as part of the riverbeds, are of public ownership Under the old Civil Law and the Law of Waters, all riverbanks are of public ownership, including those formed when a river leaves its old bed and opens a new course through a private estate. Artcile 339 of the old Civil Code is very clear. Without any qualifications, it provides that that devoted to public use, such as roads, canals, rivers, torrents, ports and bridges constructed by the State, riverbanks, shores, roadsteads, and that of a similar character are property of public ownership. Further, the riverbank is part of the riverbed. Article 73 of the Law of Waters which provides that the phrase "banks of a river" is understood those lateral strips of zones of its beds which are washed by the stream only during such high floods as do not cause inundations. The use of the words "of its bed [de sus alveos] " clearly indicates the intent of the law to consider the banks for all legal purposes, as part of the riverbed. Thus, the banks of the River are part of its bed. Since undeniably all beds of river are of public ownership, it follows that the banks, which form part of them, are also of public ownership. 3. Natural bed or channel of a creek or river defined The natural bed or channel of a creek or river is the ground covered by its waters during the highest [ordinary] floods (Article 70 of the Law of the Waters). 4. New bed, when river changes course, is of public ownership; Means to recover Article 372 of the old Civil Code which provides that "whenever a navigable or floatable river changes its course from natural causes and opens a new bed through a private estate, the new bed shall be of public ownership, but the owner of the estate shall recover it in the event that the waters leave it dry again either naturally or as the result of any work legally authorized for this purpose." Banks are not mentioned in the provision, as the nature of banks follows that of the bed and the running water of the river. 5. A river is a compound concept consisting of running waters, bed, and banks A river is a compound concept consisting of three elements; (1) the running waters, (2) the bed and (3) the banks. All these constitute the river. American authorities are in accord with this view, as that "' River' consists of water, bed and banks"; and that "A 'river' consists of water, a bed and banks, these several parts constituting the river, the whole river. It is a compound idea; it cannot exist without all its parts. Evaporate the water, and you have a dry hollow. If you could sink the bed, instead of a river you would have a fathomless gulf. Remove the banks, and you have, a boundless flood" 6. River is of public ownership, elements follow same nature of ownership; Law explicit Since a river is but one compound concept, it should have only one nature, i.e., it should either be totally public or completely private. Since rivers are of public ownership, it is implicit that all the three component elements be of the same nature also. Still,

the law expressly makes all three elements public. Thus, riverbanks and beds are public under Artciles 339 and 407, respectively, of the Code, while the flowing waters are declared so under Articles 33, par. 2 of the Law of Waters of 1866. 7. Natural is not synonymous to original or prior condition "Natural" is not made synonymous to "original" or "prior condition". On the contrary, even if a river should leave its original bed so long as it is due to the force of nature, the new course would still fall within the scope of the definition provided by the Diccionario de La Real Academia Espaola. Hence, the law must have used the word "natural" only because it is in keeping with the ordinary nature and concept of a river always to have a bed and banks. Diccionario De La Real Academia Espaola: "NATURAL-perteneciente a la naturaleza o conforme a la calidad o propriedad de las cosas; -nativa, origivario de un pueblo o imcio'n; hecho con verdad, ni artificio, mezela ni compocision alguna, ingenuo y sin doblez en su modo de proceder; di cese tambien de las cosas que imitar a, la naturaleza con propiedad; regular y que comumnente sucede, y par eso, facilmente creible; que se produce por solas las fuerzas de la naturaleza, canio contraVuesto a sobre natural y milagroso." 8. Article 553 of the old Civil Code does not intend to authorize private acquisition of river banks but recognizes vested rights of riparian owners; History of ownership of River Banks Article 553 was never intended to authorize the private acquisition of river banks, as this would conflict with clear legislative policy enunciated in Article 339 of the Code that all riverbanks were of public ownership. The article merely recognized and preserved the vested rights of riparian owners who, because of prior law or custom, were able to acquire ownership over the banks. This was possible under the Siete Partidas which was promulgated in 1834. Under Law 6, Title 28, Partida 3, the banks of rivers belonged to the riparian owners, following the Roman Law rule. But subsequent legislation radically changed this rule. By the Law of Waters of 3 August 1866, riverbanks became of public ownership, albeit impliedly only because considered part of the bed which was public, by statutory definition. This law, while expressly repealing all prior inconsistent laws, left undisturbed all vested rights then existing. Article 73 of the Law of Waters of 1866 is the reconciliation effected between the private ownership of the banks and the policy of the law to devote all banks to public Use. The easement would preserve the private ownership of the banks and still effectuate the policy of the law. So, the easement in Article 73 only recognized and preserved existing privately owned banks; it did not authorize future private appropriation of riverbanks. Subsequently, the Law of Waters of 13 June 1879 reenacted Article 73 of the Law of Waters of 1866 and affirmed the public ownership of rivers and their beds and the treatment of the banks as part of the bed. But nowhere in the law was there any provision authorizing the private appropriation of the banks. The public nature of riverbanks are obtained only by implication until the promulgation of the Civil Code of 1899, which was explicit in Article 339 that riverbanks were declared public property since they were destined for public use. Since the first paragraph of Article 36 of the Law of Waters if 1879 was reenacted in Article 553 of the Code, this article must also be understood not as authorizing the private acquisition of riverbanks but only as recognizing the vested titles of riparian owners who already owned the banks. In the present case, since the new banks were formed when the river changed its course in 1937, the banks cannot be subjected to the provisions of the Siete Partidas, to claim private ownership of the banks, as such was already superceded by then. 9. Legal definition applies with the legal order, distinction due to physical order cannot prevail The conclusion made by the lower court that only the northern 2/5 of the disputed area remained as plaintiff's private property is predicated from the findings that the portion where rice and corn were found in the ocular inspection of 15 June 1951, was on the northern 2/5 of the disputed area; that this cannot be a part of the bed because of the existence of vegetation which could not have grown underwater, and that this portion is man-made. This is bereft of evidence, as the unexcavated portion of the land is the southwestern . Further, American cases cannot be applied as these do not involve a similar statutory provision, unlike in the Law of Waters, which defined "beds" and "banks" and considered the latter as part of the former. That plants can and do grow on the banks which otherwise could not have grown on the bed which is constantly subjected to the flow of the waters proves the distinction between "beds" and "banks" in the physical order. However, in dealing with the legal order, legal definitions prevail. 10. Limits of banks of rivers Article 73 of the Law of Waters which defines the limits of banks of rivers "By the phrase 'banks of a river' is understood those lateral strips or zones of its bed which are washed by the stream only during such high floods as do not cause inundations. The farthest extremity of the bank on the west side would, therefore, be that lateral line or strip which is reached by the waters during those high floods that do not cause inundations. In other words, the extent reached by the waters when the River is at high tide. 11. Banks of river different in topography There is a difference between the topography of the two sides immediately adjoining the River. The line indicated as "primary bank," which is on the east, is about 3 meters high and has a steep grade right at the edge where it drops almost vertically to the watercourse level. The opposite side, on the other hand, has no such steep acclivity. The bank near the water edge, is about 30 to 50 cms. high only, and gradually slopes up to a height of about 2 to 2-1/2 meters along the line indicated as "secondary bank", which is quite far from the waterline. Considering the peculiar characteristics of the two sides banking the river, the rise in the waterlevel would not have the same effect on the two sides. Thus, on the east, the water would rise vertically, until the top of the "primary bank" is reached, but on the west, there would be a low angled inclined rise, the water covering more ground until the "secondary bank" line is reached. In other words, while the water expansion on the east is vertical, that on the west is more or less lateral, or horizontal. 12. Ordinary and extraordinary flood There are two types of floods in the area during the rainy season. One is the so-called "ordinary" flood, when the river is swollen but the flowing water is kept within the confines of the "primary" and "secondary" banks. This occurs annually, about three to four times during the period. Then there is the "extraordinary" flood, when the waters overflow beyond the said banks, and even inundate the surrounding areas. However, this flood does not happen regularly. From 1947 to 1955, there were only three such floods.

13. Movement of the river, west bank, from 1945-1955 From 1945 to 1949, the west bank of the River extended westward up to the "secondary bank" line; from 1950 to 1952, this bank had moved, with the River, to the east, its lateral borders running along a line just 20 meters west of the camachile tree; and from 1953 to 1955, the extremities of the west bank further receded eastward beyond the camachile tree, until they lay just about 20 meters east of said tree. 14. Floodings not accidental as they are annual; Government v. Colegio de San Jose does not apply Evidence shows that the River floods with annual regularity during the rainy season. These floods can hardly be called "accidental". The Colegio de San Jose case is not exactly in point. What was mainly considered there was Article 74 of the Law of Waters relating to lakes, ponds and pools. In the present case, none of these is involved. 15. Movement of the river not due to excavation and extraction of materials The excavations and extractions of materials, even from the American period, have been made only on the strip of land west of the River. Under the "following-the nature-of-things" argument advanced by plaintiff, the River should have moved westward, where the level of the ground had been lowered. But the movement has been in the opposite direction instead. Therefore, it cannot be attributed to defendants' operations. Moreover, Hilario's own evidence indicates that the movement eastward was all due to natural causes. The movement eastward of the channel by as much as 31 meters, from 1950 to 1953, was due to two typhoons which caused the erosion of the east bank and the depositing of materials on the west side which increased its level from as much as .93 to 2 meters. 16. River of different width; claim of unnatural widening unfounded Reliance is made on the finding by the lower court that in 1943, the river was only 60 meters wide, whereas in 1950, it was already 140 meters wide. Such area sampled shows only the width of the River near the southwestern boundary of the Hilario estate. It does not indicate how wide it was in the other parts, especially up north. 17. Extraction confined on the banks of the river and not beyond limits of the west bank to invade his private estate; Hilario cannot recover damages from defendants From 1947 to the early part of 1949, the defendants conducted their operations only in the New Accretion Area along a narrow longitudinal zone contiguous to the watercourse then. This zone, City Engineer Manila, is about 1 km. long and extends northward up to pt. 50.35. However, no extractions nor excavations were undertaken west of this zone, i.e., above the "temporary bank" line. This line is located east of the "secondary bank" line, the lateral extremity of the west bank then. In the latter part of 1949, plaintiff prohibited the defendants from extracting along the New Accretion Area and constructed a fence across the same. This forced the defendants to go southeast of the "Excavated Area". From 1954 to 1955, defendants' area of operation was still farther east of the New Accretion Area. They were. working within a confined area along the west waterline, the northern and western boundaries of which were 20 meters away east from the camachile tree. It appears sufficiently established, therefore, that defendants have not gone beyond the receding western extremities of the west riverbank. They have confined their extraction of gravel and sand only from which the banks of the River, which constitute part of the public domain wherein they had the right to operate. Plaintiff has not presented sufficient evidence that defendants have gone beyond the limits of the west bank, as previously established, and have invaded his private estate. He cannot, therefore, recover from them. 18. Plaintiff not denied of property without just compensation The Court does not declare that the entire channel, i.e., all that space between the "secondary bank" line and the "primary bank" line, has permanently become part of the riverbed. What is held is that at the time the defendants made their extractions, the excavations were within the confines of the riverbanks then. All that space to the west of said receding line" would still be part of plaintiff's property and also whatever portion adjoining the river is, at present, no longer reached by the non-inundating ordinary floods. Further, it is not correct to say that plaintiff would be deprived of his property without any compensation at all. Under Article 370 of the old Civil Code, the abandoned bed of the old river belongs to the riparian owners either fully or in part with the other riparian owners. And had the change occurred under the Civil Code of the Philippines, plaintiff would even be entitled to all of the old bed in proportion to the area he has lost. 19. Defendants did not unjustly profit at plaintiffs expense as they are not responsible for the shifting of the river Defendants cannot be accused of unjustly profiting at plaintiff's expense. They were not responsible for the shifting of the river. It was due to natural causes for which no one can be blamed. Furher, defendants were extracting from public property then, under proper authorization. The government, through the defendants, may have been enriched by chance, but not unjustly. Republic v. CA [G.R. No. L-61647. October 12, 1984.] First Division, Gutierrez Jr. (J): 5 concur Facts: Benjamin Tancinco, Azucena Tancinco Reyes, Maria Tancinco Imperial and Mario C. Tancinco are registered owners of a parcel of land covered by TCT T-89709 situated at Barrio Ubihan, Meycauayan, Bulacan bordering on the Meycauayan and Bocaue rivers. On 24 June 1973, the Tancincos filed an application for the registration of 3 lots adjacent to their fishpond property (Psu-131892: Lot 1, 33837 sq.m.; Lot 2, 5,453 sq.m.; Lot 3, 1985 sq. m.) On 5 April 1974, Assistant Provincial Fiscal Amando C. Vicente, in representation of the Bureau of Lands filed a written opposition to the application for registration. On 6 March 1975, the Tancincos filed a partial withdrawal of the application for registration with respect to Lot 3 of Plan Psu-131892 in line with the recommendation of the Commissioner appointed by the Court. On 7 March 1975, Lot 3 was ordered withdrawn from the application and trial proceeded only with respect to Lots 1 and 2 covered by Plan Psu-131892. On 26 June 1976, the lower court rendered a decision granting the application on the finding that the lands in question are accretions to the Tancincos' fishponds covered by TCT 89709.

On 30 July 1976, the Republic appealed to the Court of Appeals. On 19 August 1982, the appellate court rendered a decision affirming in toto the decision of the lower cost; without costs. Hence, the petition for certiorari to set aside the decision of the CA. The Supreme Court granted the petition, reversed and set aside the decision appealed from, and ordered the private respondents to move back the dikes of their fishponds to their original location and return the disputed property to the river to which it belongs. 1. Power to review vested in the Court; Binding effect of the findings of facts by lower court not absolute The rule that the findings of fact of the trial court and the Court of Appeals are binding upon this Court admits of certain exceptions. The Court retains the power to review and rectify the findings of fact of said courts when (1) the conclusion is a finding grounded entirely on speculations, surmises and conjectures; (2) when the inference made is manifestly mistaken, absurd, and impossible; (3) where there is grave abuse of discretion; (4) when the judgment is based on a misapprehension of facts; and (5) when the court, in making its findings, went beyond the issues of the case and the same are contrary to the admissions of both appellant and appellee (Carolina Industries Inc. v. CMS Stock Brokerage, Inc., 97 SCRA 734). 2. No accretion to speak of as the transfer of dikes is man-made and artificial; Article 457 NCC There is no accretion to speak of under Article 457 of the New Civil Code because what actually happened is that the private respondents simply transferred their dikes further down the river bed of the Meycauayan Rivers, and thus, if there is any accretion to speak of, it is man-made and artificial and not the result of the gradual and imperceptible sedimentation by the waters of the river. Article 457 of the New Civil Code provides that to the owners of lands adjoining the banks of rivers belong the accretion which they gradually receive from the effects of the current of the waters." 3. Accretion, requisites Article 457 requires the concurrence of three requisites before an accretion covered by this particular provision is said to have taken place. They are (1) that the deposit be gradual and imperceptible; (2) that it be made through the effects of the current of the water; and (3) that the land where accretion takes place is adjacent to the banks of rivers. 4. Alluvion as exclusive work of nature indispensable The requirement that the deposit should be due to the effect of the current of the river is indispensable. This excludes from Art. 457 of the New Civil Code all deposits caused by human intervention. Alluvion must be the exclusive work of nature. In the instant case, there is no evidence whatsoever to prove that the addition to the said property was made gradually through the effects of the current of the Meycauayan and Bocaue rivers; but there is evidence that the alleged alluvial deposits were artificial and man-made and not the exclusive result of the current of the Meycauayan and Bocaue rivers. The alleged alluvial deposits came into being not because of the sole effect of the current of the rivers but as a result of the transfer of the dike towards the river and encroaching upon it. 5. Reason in giving riparian owner the right to any land or alluvion deposited by a river The reason behind the law giving the riparian owner the right to any land or alluvion deposited by a river is to compensate him for the danger of loss that he suffers because of the location of his land. If estates bordering on rivers are exposed to floods and other evils produced by the destructive force of the waters and if by virtue of lawful provisions, said estates are subject to incumbrances and various kinds of easements, it is proper that the risk or danger which may prejudice the owners thereof should be compensated by the right of accretion. (Cortes v. City of Manila, 10 Phil. 567). In the present case, the riparian owner does not acquire the additions to his land caused by special works expressly intended or designed to bring about accretion. When the private respondents transferred their dikes towards the river bed, the dikes were meant for reclamation purposes and not to protect their property from the destructive force of the waters of the river. 6. Adjudication of land in question as private property is null and void The conclusion that can be made from said alleged accretion being declared for taxation purposes only in 1972 is that areas could not have been there in 1939. They existed only after the private respondents transferred their dikes towards the bed of the Meycauayan river in 1951. What private respondents claim as accretion is really an encroachment of a portion of the Meycauayan river by reclamation. Thus, the lower court cannot validly order the registration of Lots 1 & 2 in the names of the private respondents. These lots were portions of the bed of the Meycauayan river and are therefore classified as property of the public domain under Article 420 paragraph 1 and Article 502, paragraph 1 of the Civil Code of the Philippines. They are not open to registration under the Land Registration Act. The adjudication of the lands in question as private property in the names of the private respondents is null and void. Vda. De Nazareno v. CA [G.R. No. 98045. June 26, 1996.] Second Division, Romero (J): 4 concur Facts: A parcel of land situated in Telegrapo, Puntod, Cagayan de Oro City is said to have been formed as a result of sawdust dumped into the dried-up Balacanas Creek and along the banks of the Cagayan river. Sometime in 1979, Jose Salasalan and Leo Rabaya leased the subject lots on which their houses stood from one Antonio Nazareno, petitioners predecessor-in-interest. In the latter part of 1982, Salasalan and Rabaya allegedly stopped paying rentals. As a result, Antonio Nazareno and petitioners filed a case for ejectment with the MTC Cagayan de Oro City, Branch 4. A decision was rendered against Salasalan and Rabaya, which decision was affirmed by the RTC Misamis Oriental, Branch 20. The case was remanded to the Municipal trial court for execution of judgment after the same became final and executory. Private respondents filed a case for annulment of judgment before the RTC Misamis Oriental, Branch 24 and subsequently, a case for certiorari for restraining order and/or writ of preliminary injunction with the RTC of Misamis Oriental, Branch 25; both of which were dismissed. The decision of the lower court was finally enforced with the private respondents being ejected from portions of the subject lots they occupied. Before he died, Antonio Nazareno caused the approval by the Bureau of Lands of the survey plan designated as Plan Csd-10600571 with a view to perfecting his title over the accretion area being claimed by him. Before the approved survey plan could be

released to the applicant, however, it was protested by private respondents before the Bureau of Lands. The report of the Land Investigator, made in compliance with the order of the District Land Officer, recommended the Survey Plan MSI-10-06-000571D (Lot 36302, Cad. 237) in the name of Antonio Nazareno be cancelled and that private respondents be directed to file appropriate public application. Based on the report, the Regional Director of the Bureau of Lands rendered a decision ordering an amendment to the survey plain of Nazareno by segregating therefrom the areas occupied by the private respondents. Antonio Nazareno filed a motion for reconsideration with the Undersecretary of Natural Resources and OIC of the Bureau of Lands; which was denied. The petitioners Desamparada vda. De Nazareno and Leticia Tapia Nazero filed a case before the RTC, branch 22, for the annulment of the verification, report and recommendation, decision and order of the Bureau of Lands regarding the parcel of land. The RTC dismissed the complaint for failure to exhaust administrative remedies, resulting to the finality of the administrative decision of the Bureau of Lands. On appeal, the Court of Appeals affirmed the decision of the RTC dismissing the complaint. Hence, the petition. The Supreme Court dismissed the petition for lack of merit. 1. Accretion belongs to the riparian owners; requisites Article 457 of the Civil Code provides that "to the owners of land adjoining the banks of rivers belong the accretion which they gradually receive from the effects of the current of the waters." In the case of Meneses v. CA, it was held that accretion, as a mode of acquiring property under Article 457 of the Civil Code, requires the concurrence of these requisites: (1) that the deposition of soil or sediment be gradual and imperceptible; (2) that it be the result of the action of the waters of the river (or sea); and (3) that the land where accretion takes place is adjacent to the banks or rivers (or the sea coast). These are called the rules on alluvion which if present in a case, give to the owners of lands adjoining the banks of rivers or streams any accretion gradually received from the effects of the current of waters. 2. Alluvion in present case not accumulated by action of waters or current The application of the rules on alluvion cannot be made in the present case as the first and second requirements of the rules were not met. Thus, the Nazarenos cannot claim the rights of a riparian owner. By their own admission, the accretion was formed by the dumping of boulders, soil and other filling materials on portions of the Balacanas Creek and the Cagayan River bounding their land. It cannot be claimed, therefore, that the accumulation of such boulders, soil and other filling materials was gradual and imperceptible, resulting from the action of the waters or the current of the Balacanas Creek and the Cagayan River. 3. Current defined In Hilario v. City of Manila, it was held that the word "current" indicate the participation of the body of water in the ebb and flow of waters due to high and low tide. 4. Petitioners estopped in the denying the public character of land and jurisdiction of the Bureau of Lands Petitioners are estopped from denying the public character of the subject land, as well as the jurisdiction of the Bureau of Lands when the late Antonio Nazareno filed his Miscellaneous Sales Application MSA (G-6) 571. The mere filing of said Application constituted an admission that the land being applied for was public land, having been the subject of Survey Plan MSI-10-06000571-D which was conducted as a consequence of Antonio Nazareno's Miscellaneous Sales Application wherein said land was described as an orchard. Said description by Antonio Nazareno was controverted by the findings of the ocular inspection that said land actually covers a dry portion of Balacanas Creek and a swampy portion of Cagayan River. 5. Findings of administrative agencies which have acquired expertise in their jurisdiction final; not reviewable by the Courts The Bureau of Lands classified the subject land as an accretion area which was formed by deposits of sawdust in the Balacanas Creek and the Cagayan river, in accordance with the ocular inspection conducted by the Bureau of Lands. It has often enough held that findings of administrative agencies which have acquired expertise because their jurisdiction is confined to specific matters are generally accorded not only respect but even finality. Again, when said factual findings are affirmed by the Court of Appeals, the same are conclusive on the parties and not reviewable by the Supreme Court. 6. Similar cases where alluvion is artificial; Man-made accretion part of public domain In Republic v. CA, it was ruled that the requirement that the deposit should due to the effect of the current of the river is indispensable. This excludes from Article 457 of the Civil Code all deposits caused by human intervention. Putting it differently, alluvion must be the exclusive work of nature. Thus, in Tiongco v. Director of Lands, et al., where the land was not formed solely by the natural effect of the water current of the river bordering said land but is also the consequence of the direct and deliberate intervention of man, it was deemed a man-made accretion and, as such, part of the public domain. In the present case, the subject land was the direct result of the dumping of sawdust by the Sun Valley Lumber Co. consequent to its sawmill operations. As the accretion site was the result of the late Antonio Nazareno's labor consisting in the dumping of boulders, soil and other filling materials into the Balacanas Creek and Cagayan River bounding his land, the same would still be part of the public domain. 7. Subject land being public, Bureau of Lands and Office of the Secretary of Agriculture and Natural Resources have jurisdiction The subject land is public land, a fortiori, the Bureau of Lands, as well as the Office of the Secretary of Agriculture and Natural Resources have jurisdiction over the same in accordance with the Public Land Law. 8. Administrative remedies exhausted; Regional Director of Bureau of Lands decision entered for and by authority of the Director of Lands Administrative remedies have been exhausted in the present case. It would be incongruous to appeal the decision of the Regional Director of the Bureau of Lands acting for and by the authority of the Director of the Bureau of Lands to an Officer-In-Charge of the Bureau of Lands.

9. Undersecretary of Department of Agricultural and Natural Resources has authority to review decisions or orders of the Director of lands with respect to public lands under the administration of the Bureau and the Department In the case of Hamoy v. Secretary of Agriculture and Natural Resources, it was held that the Undersecretary of Agriculture and Natural Resources may modify, adopt, or set aside the orders or decisions of the Director of Lands with respect to question involving public lands under the administration and control of the Bureau of Lands and the Department of Agriculture and Natural Resources. He cannot, therefore, be said to have acted beyond the bounds of his jurisdiction under Sections 3, 4 and 5 of the Public Land Law (CA 141). Section 3 provides that "the Secretary of Agriculture and Natural Resources shall be the exclusive officer charged with carrying out the provisions of this Act through the Director of Lands who shall act under his immediate control, while Section 4 provides that subject to said control, the Director of Lands shall have direct executive control of the survey, classification, lease, sale or any other form of concession or disposition and management of the lands of the public domain, and his decisions as to questions of fact shall be conclusive when approved by the Secretary of Agriculture and Natural Resources." In the present case, when Rolleo Ignacio acted on the motion for reconsideration of the late Antonio Nazareno, he was acting on said motion as an Undersecretary of the Department of Agriculture and Natural Resources; notwithstanding the fact that he was an Officer-in-charge of the Bureau of Lands. 10. Execution order does not award portions of subject land to private respondents but merely segregated the petitioners titled land from the subject land; Does not bar petitioners application It is incorrect for petitioners to assume that respondent Palad awarded portions of the subject land to private respondents Salasalans and Rayabas as they had not yet been issued patents or titles over the subject land. The execution order merely directed the segregation of petitioners titled lot from the subject land which was actually being occupied by private respondents before they were ejected from it. Based on the finding that private respondents were actually in possession or were actually Palad, being the Director of Lands and in the exercise of this administrative discretion, directed petitioners to vacate the subject land on the ground that private respondents have a preferential right, being the occupants thereof. Palad's (Director of Land) execution order merely implements Hilario's order. It should be noted that petitioners own application still has to be given due course. 11. Director of Lands authorized to exercise executive control over any form of concession, disposition and management of lands of the public domain; Error in judgment not annullable by certiorari The Director of Lands is authorized to exercise executive control over any form of concession, disposition and management of the lands of the public domain. He may issue decisions and orders as he may see fit under the circumstances as long as they are based on the findings of fact. In the case of Calibo v. Ballesteros, it was held that where, in the disposition of public lands, the Director of Lands bases his decision on the evidence thus presented, he clearly acts within his jurisdiction, and if he errs in appraising the evidence, the error is one of judgment, but not an act or grave abuse of discretion annullable by certiorari. Government v. Cabangis [G.R. No. 28379. March 27, 1929.] Second Division, Villa-Real (J): 6 concur Facts: Lots 36, 39 and 40, block 3035 of cadastral proceeding 71 of the City of Manila, GLRO. Record 373, were formerly a part of a large parcel of land belonging to the predecessor of Cabangis. From the year 1896 said land began to wear away, due to the action of the waves of Manila Bay, until the year 1901 when the said lots became completely submerged in water in ordinary tides, and remained in such a state until 1912 when the Government undertook the dredging of Vitas Estuary in order to facilitate navigation, depositing all the sand and silt taken from the bed of the estuary on the low lands which were completely covered with water, surrounding that belonging to the Philippine Manufacturing Company, thereby slowly and gradually forming the lots, the subject matter of the proceeding. Up to the month of February 1927 nobody had declared lot 39 for the purposes of taxation, and it was only in the year 1926 that Dr. Pedro Gil, in behalf of Cabangis, declared lot 40 for such purpose. The CFI Manila rendered judgment (in cadastral proceeding 373 of the CFI Manila, GLRO Cadastral Record 373) adjudicated the title and decreed the registration of lots 36, 39 and 40, block 3055 of the cadastral survey of the City of Manila in favor of Consuelo, Consorcia, Elvira and Tomas, all surnamed Cabangis, in equal parts, and dismissed the claims presented by the Government of the Philippine Islands and the City of Manila. The Government of the Philippine Islands appealed said judgment before the Supreme Court. The Supreme Court reversed the judgment appealed from and lots 36, 39 and 40 of cadastral proceeding 373 of the City of Manila are held to be public land belonging to the Government of the United States under the administration and control of the Government of the Philippine Islands. 1. Property of public ownership Article 339, subsection 1, of the Civil Code provides that property of public ownership is that devoted to public use, such as roads, canals, rivers, torrents, ports and bridges constructed by the State, riverbanks, shores, roadsteads, and that of a similar character." Article 1, case 3, of the Law of Waters of 3 August 1866, provides that the shores are part of the national domain open to public use. By the shore is understood that space covered and uncovered by the movement of the tide. Its interior or terrestrial limit is the line reached by the highest equinoctial tides. Where the tides are not appreciable, the shore begins on the land side at the line reached by the sea during ordinary storms or tempests." 2. Abandonment of land to become part of the shore of the sea, roadstead and the like makes the property of public ownership In the case of Aragon vs. Insular Government (19 Phil., 223), with reference to article 339 of the Civil Code, held that that in a case of gradual encroachment or erosion by the ebb and flow of the tide, private property may become 'property of public ownership,' as defined in article 339 of the code, where it appears that the owner has to all intents and purposes abandoned it and permitted it to be totally destroyed, so as to become a part of the 'playa' (shore of the sea), 'rada' (roadstead), or the like. In the Enciclopedia Jurdica Espaola, volume XII, page 558, states that when the sea advances and private properties are permanently invaded by the waves, and in this case they become part of the shore or beach. They then pass to the public domain, but the owner thus dispossessed does not retain any right to the natural products resulting from their new nature; it is a de facto case of eminent

domain, and not subject to indemnity." In the present case, the failure of the Cabangis to protect their land by building a retaining wall with consent of competent authority, and allowing portion of their land to be completely covered by water, constitutes abandonment. Thus, when the land was completely submerged and was reclaimed as a result of a certain work done by the Government in 1912, the portions of the land became a part of the public domain. 3. Ownership of land reclaimed from the sea; When lands are converted to public land, no person could acquire title thereto except in form and manner established by law Article 5 of the Law of Waters of 1866 provides that lands reclaimed from the sea in consequence of works constructed by the State, or by the provinces, pueblos, or private persons, with proper permission, shall become the property of the party constructing such works, unless otherwise provided by the terms of the grant of authority." The fact that from 1912 some fishermen had been drying their fishing nets and depositing their bancas on lots 36, 39 and 40, by permission of Tomas Cabangis, does not confer on the latter or his successors the ownership of said lots, because, as they were converted into public land, no private person could acquire title thereto except in the form and manner established by the law. 4. Case of Buzon v. Insular Government and City of Manila different; Inundation by sea due to acts independent of the will of the owner of land (removal of large quantity of sand) In the case of Buzon vs. Insular Government and City of Manila, the rise of the waters of the sea that covered the lands there in dispute, was due not to the action of the tide but to the fact that a large quantity of sand was taken from the sea at the side of said land in order to fill in Cervantes Street, and this court properly held that because of this act, entirely independent of the will of the owner of said land, the latter could not lose the ownership thereof, and the mere fact that the waters of the sea covered it as a result of said act, is not sufficient to convert it into public land, especially, as the land was high and appropriate for building purposes. 5. Case of Director of Lands v. Aguilar different; Failure of the oppositor to present evidence In the case of the Director of Lands vs. Aguilar, the Insular Government did not present any evidence in support of its contention, thus leaving uncontradicted the evidence adduced by the claimants Aguilar et al., as to the ownership, possession and occupation of said lots. In the present case, the evidence shows that from 1896, the waves of Manila Bay had been gradually and constantly washing away the sand that formed the lots here in question, until 1901, when the sea water completely covered them, and thus they remained until the year 1912. In the latter year they were reclaimed from the sea by filling in with sand and silt extracted from the bed of Vitas Estuary when the Government dredged said estuary in order to facilitate navigation. 6. Land eroded by gradual erosion by ebb and flow of tide and reclaimed by filling done by government are public land The lots in question having disappeared on account of the gradual erosion due to the ebb and flow of the tide, and having remained in such a state until they were reclaimed from the sea by the filling in done by the Government, they are public land. (Aragon vs. Insular Government, 19 Phil., 223; Francisco vs. Government of the Philippine Islands, 28 Phil., 505.) De Buyser v. Director of Lands [G.R. No. L-22763. March 18, 1983.] Second Division, Escolin (J): 5 concur, 1 on leave Facts: De Buyser is the registered owner of Lot 4217 of the Surigao Cadastre, which borders the Surigao Strait. Contiguous to said lot is a parcel of land which was formed by accretion from the sea, the subject-matter of this controversy. Spouses Ignacio and Candida Tandayag have been occupying this foreshore land through a Revocable Permit issued by the Director of Lands. For the use and occupation thereof, said spouses paid the Bureau of Lands the amount of P6.50 annually. They have a house on said lot, which De Buyser alleged had been purchased by the Tandayags from one Francisco Macalinao, a former lessee of the De Buyser. Claiming ownership of the said land, De Buyser filed an action against the Tandayags in the CFI Surigao to recover possession of this land as well as rents in arrears for a period of 6 years. The complaint was subsequently amended to implead the Director of Land as defendant, allegedly for having illegally issued a revocable permit to the Tandayags. After due trial, the court a quo rendered a decision dismissing the complaint; holding said land in question to be formed along the shore by action of the sea and thus part of public domain, and ordering de Buyser to pay P250.00 to the Tandayags as damages; and to pay the costs as well. From the judgment, de Buyser appealed directly to the Supreme Court on a pure question of law; such being perfected before the effectivity of RA 5440. The Supreme Court affirmed the decision appealed from; with costs against the plaintiff-appellant. 1. Alluvial formation along seashore is part of public domain Alluvial formation along the seashore is part of the public domain and, therefore, not open to acquisition by adverse possession by private persons. It is outside the commerce of man, unless otherwise declared by either the executive or legislative branch of the government. 2. Construction of Article 4 of the Spanish Law of Waters (3 August 1866) Article 4 of the Spanish Law of Waters of 3 August 1866 provides that the lands added to the shore by accretion and alluvial deposits caused by the action of the sea, form part of the public domain, when they are no longer washed by the waters of the sea, and are not necessary for purposes of public utility, or for the establishment of special industries, or for the coastguard service, the Government shall declare them to be the property of the owners of the estate adjacent thereto and as an increment thereof." The true construction of the provision is that the State shall grant these lands to the adjoining owners only when they are no longer needed for the purposes mentioned therein. In the present case, de Buyser's evidence failed to prove that the land in question is no longer needed by the government, or that the essential conditions for such grant under Article 4 of the Spanish Law of Waters, exists.

3. Revocable Permit application does not relinquished States ownership over the land In approving the Revocable Permit Application of the Tandayags, the Director of Lands did not declare the land as no longer needed for public use. It merely allowed them to continue their temporary occupation and provisional use of the premises under revocable permit renewable every year in the meantime the land is not actually needed by the government for public improvements (Boulevard and seawall protection purposes). It is clear thus that the State never relinquished ownership over the land. 4. Disposition of property of public domain under Bureau of Lands; Failure to submit to its jurisdiction does not entitle one to protection of the courts on the matter of right to foreshore land The land being property of public dominion, its disposition falls under the exclusive supervision and control of the Bureau of Lands. Under the Public Land Act, an application for the sale or lease of lands enumerated under Section 59 thereof, should be filed with the Bureau of Lands. Like any other private party, one must apply for a permit to use the land. Not having submitted to the jurisdiction of the Bureau of Lands which has administration and control over the area in question, by filing the corresponding application for permit, one has no right whatsoever in the foreshore land as to be entitled to protection in the courts of justice. 5. Occupation or possession of land formed along the shore by action of the sea mere detainer if without authority from the Government to occupy said land The occupation or possession held, without the records disclosing whether the requisite authorization to occupy the parcel of land formed along the shore by the action of the sea from the Spanish Government of the Philippines, is a mere detainer that can merit from the law no protection such as is afforded only to the person legally in possession (Aldecoa v. Insular Government). 6. Grant of revocable permit allowed by the passing of Act 2570, amending Section 5 of Act 1654 The lease of reclaimed lands and of the foreshore was formerly provided by Act 1654; under which. lands could only be leased in the manner and under the conditions provided by the said law in a manner that no revocable permits were allowed. Act 2570, amended Section 5 of Act 1654 so as to authorize the temporary use of the foreshore under a revocable permit. This measure was apparently deemed necessary as well as expedient in order to legalize the habitual use of the coast and shores of these islands by the people, who had erected thereon light material houses and dwellings, temporary structures used in connection with fishing, salt and other maritime industries, as well as to authorize the provisional occupation and use contemplated by the law providing for its formal lease. Such circumstance evidently prompted the legislature to all the temporary use of the foreshore in this manner by means of revocable permit. 7. Bureau of land acting for and in behalf of Secretary of Agricultural and Natural Resources in granting the revocable permit The grant of a Revocable Permit for the temporary use and occupation of the disputed land is valid, having been legally issued by the Bureau of Lands, acting for and in behalf of the Secretary (now Minister) of Agriculture and Natural Resources who is empowered to grant revocable permits under Section 68 of the Public Land Act which provides that the Secretary of Agriculture and Natural Resources may grant to qualified persons temporary permission upon the payment of a reasonable charge, for the use of any portion of the lands covered by this chapter for any lawful private purpose, subject to revocation, at any time when, in his judgment the public interest shall require." Ignacio v. Director of Lands [G.R. No. L-12958. May 30, 1960.] First Division, Montemayor (J): 8 concur Facts: Faustino Ignacio, on 25 January 1950, filed an application for the registration of a parcel of land (mangrove), adjoining a parcel of land that Ignacio has previously acquired from the Government by virtue of a free patent title in 1936, situated in barrio Gasac, Navotas, Rizal, with an area of 37,877 sq.m.. Later, he amended his application by alleging among others that he owned the parcel applied for by right of accretion; the parcel being formed by accretion and alluvial deposits caused by the action of the Manila Bay which borders it on the sourtwest. He also claims that he had occupied the land since 1935, planting it with api-api trees, and that his possession has been continuous, adverse and public for a period of 20 years until said possession was disturbed by Valeriano. To the application, the Director of Lands, Laureano Valeriano and Domingo Gutierrez filed oppositions. Gutierrez later withdrew his opposition. The Director of Lands claimed the parcel applied for as a portion of the public domain, being a foreshore land covered by the ebb and flow of the tide. In his turn, Valeriano alleged that he was holding the land by virtue of a permit granted him by the Bureau of Fisheries, issued on 13 January 1947, and approved by the President. After hearing, the CFI Rizal dismissed Ignacios application for the registration of the parcel of land, holding it to form part of the public domain. The Supreme Court affirmed the appealed decision, with costs. 1. Article 457 NCC does not apply as it covers accretion on banks of rivers Article 457 of the New Civil Code (Article 366, Old Civil Code), which provides that "to the owners of lands adjoining the banks of rivers belong the accretion which they gradually receive from the effects of the current of the waters." This is inapplicable as it refers to accretion or deposits on the banks of rivers, while the accretion in the present case was caused by action of the Manila Bay. 2. Article 1, 4 and 5 of the Law of Waters apply (accretion formed by the sea) as bay is part of the sea Articles 1, 4 and 5 of the Law of Waters are applicable, referring to accretions formed by the sea. Manila Bay is a part of the sea, being a mere indentation of the same. As defined, bay is an opening into the land where the water is shut in on all sides except at the entrance; an inlet of the sea; an arm of the sea, distinct from a river, a bending or curbing of the shore of the sea or of a lake. 3. Application of Law of Waters on lands bordering Manila Bay; cases The Supreme Court has in some cases applied the Law of Waters on Lands bordering Manila Bay; such as the cases of Ker & Co. vs. Cauden, 6 Phil., 732, involving a parcel of land bounded on the sides by Manila Bay, where it was held that such land formed

by the action of the sea is property of the State; Francisco vs. Government of P.I., 28 Phil., 505, involving a land claimed by a private person and subject to the ebb and flow of the tides of the Manila Bay. 4. Interpretation of Article 4 of the Law of Waters of 1866; Declaration that land is not necessary for purposes of public utility, etc., lies with the executive and possibly the legislative departments Article 4 of the Law of Waters of 1866 provides that when a portion of the shore is no longer washed by the waters of the sea and is not necessary for purposes of public utility, or for the establishment of special industries, or for coastguard service, the government shall declare it to be the property of the owners of the estates adjacent thereto and as an increment thereof. We believe that only the executive and possibly the legislative departments have the authority and the power to make the declaration that any land so gained by the sea, is not necessary for purposes of public utility, or for the establishment of special industries, or for coast-guard service. If no such declaration has been made by said departments, the lot in question forms part of the public domain." (Natividad vs. Director of Lands [CA], 37 OG 2905) 5. Court are not in position to determine if land are used as specified in Article 4 of the Law of Waters The courts are neither primarily called upon, nor indeed in a position to determine whether any public land are to be used for the purposes specified in Article 4 of the Law of Waters (Vicente Joven y Monteverde v. Director of Lands (93 Phil. 134). 6. Public domain not subject to ordinary prescription Land of the public domain is not subject to ordinary prescription. "The occupation or material possession of any land formed upon the shore by accretion, without previous permission from the proper authorities, although the occupant may have held the same as owner for seventeen years and constructed a wharf on the land, is illegal and is a mere detainer, inasmuch as such land is outside of the sphere of commerce; it pertains to the national domain; it is intended for public uses and for the benefit of those who live nearby. (Insular Government vs. Aldecoa & Co., 19 Phil., 505) Government v. Colegio de San Jose [G.R. No. 30829. August 28, 1929.] Second Division, Villa-Real (J): 5 concur Facts: During the months of September to November every year, the waters of Laguna de Bay cover a long strip of land along the eastern border of the two parcels of land in question, the width of which strip varies from 50 to 70 meters and up to the eastern border of the pass claimed by the municipality of San Pedro Tunasan. The flooded strip includes the aforementioned pass itself, which is usually completely covered with water, so that the people can fish in said flooded strip. Raised in the CFI Laguna, the claimant Colegio de San Jose contends that the parcels of land are a part of the Hacienda de San Pedro Tunasan belonging it, which has been in possession thereof since time immemorial by means of its tenants or lessees and farmers. On the other hand, the Government contends that the said two parcels of land belong to the public domain, and its evidence tends to prove that they have always been known as the shores of Laguna de Bay. The CFI rendered, in cadastral case 30, GLRO Record 359 of San Pedro, Laguna, a decision in favor of Colegio de San Jose ordering the registration of the 2 parcels of land (lot 1 and 2) in accordance with law; without pronouncement as to costs. It further stated that its ruling should be understood in the manner that the lease of said lands executed by the Colegio de San Jose in favor of Carlos Young y Baldwin is valid and subsists under the terms and conditions set forth in the instruments and providing for the issuance of the proper decree once said decision becomes final. Appeal was made by the Government of the Philippine Islands before the Supreme Court. The Supreme Court affirmed the judgment appealed from, without special pronouncement as to costs. 1. Laguna Lake is a lake not a lagoon The Enciclopedia Juridica Espaola, volume XXI defines "lake" as a body of water formed in depressions of the earth. Ordinarily fresh water, coming from rivers, brooks, or springs, and connected with the sea by them"; and "lagoon" as a small lake, ordinarily of fresh water, and not very deep, fed by floods, the hollow bed of which is bounded by elevations of land." In the present case, Laguna de Bay is a body of water formed in depressions of the earth; it contains fresh water coming from rivers and brooks or springs, and is connected with Manila Bay by the Pasig River; and thus is a lake. 2. Lakes and ponds existing upon public lands belong to public dominion Article 407 of the Civil Code provides that lakes and ponds formed by nature on public lands, and their channels are of public ownership. Article 44 of the Law of Waters of 3 August 1866, provides that natural ponds and lakes existing upon public lands and fed by public waters, belong to the public domain." It is beyond discussion that Laguna de Bay belongs to the public domain, being a natural lake existing upon public lands, and fed by public waters from rivers, brooks and springs. 3. Ordinary and extraordinary depth distinguished The word "ordinary" is defined in the Dictionary of the Spanish Academy as that "not exceeding the average; common, natural, occurring always or most of the time; not going beyond what often happens or takes place," while extraordinary is defined as the uncommon, transcending the general rule, order, or measure; exceeding, surpassing, or going beyond that which is ordinary, commonly met with, current, settled, or admitted by the majority." According to said definitions, the highest depth of the waters of Laguna de Bay during the dry season is the ordinary one, and the highest depth they attain during the rainy season is the extraordinary one; inasmuch as the former is the one which is regular, common, natural, which occurs always or most of the time during the year, while the latter is uncommon, transcends the general rule, order or measure, and goes beyond that which is the ordinary depth. 4. Extent of the bed of Laguna lake Article 74 of the Law of Waters defines the bed of a lake as the natural bed or basin of lakes, ponds, or pools, is the ground covered by their waters when at their highest ordinary depth." Thus, in the present case, the natural bed or basin of the lakes is the ground covered by their waters when at their highest ordinary depth, the natural bed or basin of Laguna de Bay is the ground

covered by its waters when at their highest depth during the dry season, i.e. up to the northeastern boundary of the two parcels of land in question. 5. Land in question do not belong to the public domain Inasmuch as, according to article 407 of the Civil Code, lakes and their beds belong to the public domain, and inasmuch as, according to article 74 of the Law of Waters, the bed of a lake is the ground covered by its waters at their highest ordinary depth; whereas the waters of Laguna de Bay at their highest depth reach no farther than the northeastern boundary of the two parcels of land in question, said parcels are outside said bed and, consequently, do not belong to the public domain. Therefore, they must belong to Colegio de San Jose as part of Hacienda de San Pedro Tunasan, which was owned by it. 6. Article 367 of the Civil Code does not apply; Article 77 of the Law of Waters apply Article 367 provides that the owners of estates bordering on ponds or lagoons, do not acquire the land left dry by the natural decrease of the waters, nor lose those inundated by them in extraordinary floods." The provision refers to ponds and lagoons, and has therefore no application to the present case, which refers to a lake, a lagoon being legally distinct in character from a lake. Considering that the inundation of the land in question is made during the months of September to November, which is extraordinary, the legal provision applicable to the case is that contained in article 77 of the aforesaid Law of Waters. Said article provides that lands accidentally inundated by the waters of lakes, or by creeks, rivers, and other streams, shall continue to be the property of their respective owners." 7. Accretion upon lands contiguous to lakes belong to the owners of such lands Article 84 of the said Law of Waters provides that accretions deposited gradually upon lands contiguous to creeks, streams, rivers, and lakes, by accessions or sediments from the waters thereof, belong to the owners of such lands." Even if, therefore, the two parcels of land in litigation were considered as accretions gradually deposited by accessions or sediments from the waters of Laguna de Bay, they would still belong to the claimant Colegio de San Jose as owner of the lands bordering on said Laguna de Bay. 8. Paragraph 3 of Article 1 of the Law of Waters applies on shores of the sea, not of the lakes Paragraph 3 of article 1 of the Law of Waters provides that shores are part of the national domain open to public use, and that by the shore is understood that space covered and uncovered by the movement of the tide. Its interior or terrestrial limit is the line reached by the highest equinoctial tides. Where the tides are not appreciable, the shore begins on the land side at the line reached by the sea during ordinary storms or tempests." This provision refers to the waters of the sea, being included in Title I, which treats of the ownership and use of said waters of the sea. Lake waters, being terrestrial waters, their ownership and use are governed by Title II of said Law of Waters. In the same manner as the shore of the sea is that space covered and uncovered by the waters during tides, the exterior or terrestrial limit being the line reached by the highest equinoctial tides, so the shore of a lake is that space covered and uncovered by the waters during the tides, its interior or terrestrial limit being the line reached by its highest ordinary depth. 9. Summary of the Courts decision in the present case The Court held (1) that the natural bed or basin of Laguna de Bay is the ground covered by its waters at their highest ordinary depth during the dry season, that is, during the months of December, and January to August; (2) that the highest depth reached by said waters during the rainy season, or during the months of September to November, is extraordinary; (3) that the two parcels of land in litigation form an integral part of the Hacienda de San Pedro Tunasan belonging to the claimant Colegio de San Jose; (4) that said two parcels of land, being accidentally inundated by the waters of Laguna de Bay continue to be the property of the claimant Colegio de San Jose, pursuant to Article 77 of the Law of Waters of 3 August 1866; (5) that even supposing that the said two parcels of land have been formed by accession or deposits of sediment by the waters of said Laguna de Bay, they still belong to the said claimant Colegio de San Jose bordering on said Laguna de Bay, pursuant to Article 84 of the Law of Waters of 3 August 1866; and (6) that the provisions of the Law of Waters regulating the ownership and use of the waters of the sea are not applicable to the ownership and use of lakes, which are governed by special provisions. Republic v. Alagad [G.R. No. 66807. January 26, 1989.] Second Division, Sarmiento (J): 4 concur Facts: On 11 October 1951, Melitona, Carmen (with spouse Espiridion Kolimlim), Justo, Carlos, Librada (with spouse Emerson Abano), Demetrio, and Antonio Alagad filed an application for registration of their title over a parcel of land situated at Linga, Pila, Laguna, with an area of 8.1263 hectares (survey plan Psu-116971), which was amended after the land was divided into two parcels, namely, Lot 1 with an area of 5.2476 hectares and Lot 2 with an area of 2.8421 hectares (survey plan Psu-226971, amendment 2). The Republic opposed the application on the stereo-typed ground that applicants and their predecessors have not been in possession of the land openly, continuously, publicly and adversely under a bona fide claim of ownership since 26 July 1894 and the land has not ceased to be a part of the public domain. It appears that barrio folk also opposed the application. On 16 January 1956, by virtue of a final judgment in said case, supplemented by orders issued on 21 March 1956 and 13 August 1956, the Alagads were declared owners of Lot 1 and the remaining portion, or Lot 2, was declared public land. Decree N-51479 was entered and OCT 0-401, dated 18 October 1956, was issued in the names of the Alagads. In August 1966, the Alagads filed before the Municipal Court of Pila, Laguna (Civil Case 52) an action to evict the barrio folk occupying portions of Lot 1. On 8 August 1968, judgment was rendered in the eviction case ordering the barrio folk therein to return possession of the premises to the Alagads. The barrio folk did not appeal. The Republic filed a petition for annulment of title and reversion, insofar as the 1.42 hectare northwestern portion on end of Lot 1 is concerned, contending that such is foreshore land, and that the Alagads could not have had an imperfect title to it as it was the barrio folk who filled up the land to elevate the land to its present condition. The Court, on 6 October 1970, issued a writ of preliminary injunction enjoining the Provincial Sheriff of Laguna or his deputies from enforcing the writ of execution issued in Civil Case 52, and the Alagads from selling, mortgaging, disposing or otherwise entering into any transaction affecting the area.

The case was set for pre-trial on 6 July 1971, to which the attorney representing the Republic did not appear. On 16 July 1971, the court dismissed the complaint. The Republic filed a motion for reconsideration, was set for hearing, and finally denied by the court. Appeal was made to the Court of Appeals, which sustained the trial court for failure to show in the record on appeal that the appeal was perfected on time. Hence, the appeal. The Supreme Court reversed the decision of the lower courts, and reinstated the Republics complaint and thus remanded the case to the trial court for further proceedings. 1. State cannot be bound by or estopped from the mistakes or negligent acts of its officials or agents The State cannot be bound by, or estopped from, the mistakes or negligent acts of its official or agents, much more, non-suited as a result thereof. This is so because the state as a persona in law is the judicial entity, which is the source of any asserted right to ownership in land under the basic doctrine embodied in the 1935 Constitution as well as the present charter. It is charged moreover with the conservation of such patrimony. There is need therefore of the most rigorous scrutiny before private claims to portions thereof are judicially accorded recognition. Such primordial consideration, not the apparent carelessness, much less the acquiescence of public officials, is the controlling norm. 2. Ramos v. Central Bank, and Nilo v. Romero not applicable to the present case The cases of Ramos v. Central Bank of the Philippines and Nilo v. Romero, are not applicable. In Ramos, the Court applied estoppel upon finding of bad faith on the part of the State (the Central Bank) in deliberately reneging on its promises. In Nilo, the Court denied efforts to impugn the jurisdiction of the court on the ground that the defendant had been "erroneously" represented in the complaint by the City Attorney when it should have been the City Mayor, on a holding that the City Attorney, in any event, could have ably defended the City (Davao City). In both cases, it is seen that the acts that gave rise to estoppel were voluntary and intentional in character, in which cases, it could not be said that the Government had been prejudiced by some negligent act or omission. 3. Res judicata is not an impediment to reversion of property; Republic v. CA, requisites for a prior judgment to become a bar Res judicata is not an impediment to reversion of property. In Republic v. Court of Appeals, the Court stated that a certificate of title may be ordered cancelled (Republic v. Animus, et al.), and the cancellation may be pursued through an ordinary action therefor. This action cannot be barred by the prior judgment of the land registration court, since the said court had no jurisdiction over the subject matter. And if there was no such jurisdiction, then the principle of res judicata does not apply. For it is a wellsettled rule that for a prior judgment to constitute a bar to a subsequent case, the following requisites must concur; (1) it must be a final judgment; (2) it must have been rendered by a court having jurisdiction over the subject matter and over the parties; (3) it must be a judgment on the merits; and (4) there must be, between the first and second actions, identity of parties, identity of subject matter and identity of cause of action (Municipality of Daet vs. C4 93 SCRA 503; Mendoza vs. Arrieta, et al., 91 SCRA 113) 4. Property of public dominion: Property for public use or public service "Property", according to the Civil Code, "is either of public dominion or of private ownership." Property is of public dominion if it is (1) intended for public use, such as roads, canals, rivers, torrents, ports and bridges constructed by the State, banks, shores, roadsteads and others of similar character;or if it (2) belongs to the State, without being for public use, and are intended for some public service or for the development of the national wealth. 5. Patrimonial property and property of public dominion "All other property of the State which is not of the character mentioned in article [420], is patrimonial property," meaning to say, property "open to disposition" by the Government, or otherwise, property pertaining to the national domain, or public lands. Property of the public dominion, on the other hand, refers to things held by the State by regalian right. They are things res publicae in nature and hence, incapable of private appropriation. Thus, under the present Constitution, "[w]ith the exception of agricultural lands, all other natural resources shall not be alienated." 6. Public Dominion, as to waters Article 502 provides that (1) Rivers and their natural beds; (2) Continuous or intermittent waters of springs and brooks running in their natural beds and the beds themselves; (3) Waters rising continuously or intermittently on lands of public dominion; (4) Lakes and lagoons formed by Nature on public lands, and their beds; (5) Rain waters running through ravines or sand beds, which are also of public dominion; (6) Subterranean waters on public lands; (7) Waters found within the zone of operation of public works, even if constructed by a contractor; (8) Waters rising continuously or intermittently on lands belonging to private persons, to the State, to a province, or to a city or municipality from the moment they leave such lands; and (9) The waste waters of fountains, sewers and public establishments are of public dominion. It is also ordained in Article 44 of the Spanish Law of Waters of 3 August 1866 that natural ponds and lakes existing upon public lands and fed by public waters, belong to the public domain. Lakes, ponds, and pools existing upon the lands of private individuals, or the State or provinces, belong to the respective owners of such lands, and those situated upon lands of communal use belong to their respective pueblos." 7. Laguna de Bay is a lake (Colegio de San Jose case); Highest Ordinary Depth Laguna de Bay has long been recognized as a lake. Laguna de Bay is a body of water formed in depressions of the earth; it contains fresh water coming from rivers and brooks or springs, and is connected with Manila Bay by the Pasig River. Inasmuch as Laguna de Bay is a lake, the Court must resort to the legal provisions governing the ownership and use of lakes and their beds and shores, in order to determine the character and ownership of the parcels of land in question. The recourse to legal provisions is necessary, for under Article 74 of the Law of Waters, the natural bed or basin of lakes is the ground covered by their waters when at their highest ordinary depth" and in which case, it forms part of the national dominion. When Laguna de Bay's waters are at their highest ordinary depth has been defined as the highest depth of the waters of Laguna de Bay during the dry season, such depth being the "regular, common, natural, which occurs always or most of the time during the year. Otherwise, where the rise in water level is due to the "extraordinary" action of nature, rainfall for instance, the portions inundated thereby are not considered

part of the bed or basin of the body of water in question. It cannot therefore be said to be foreshore land but land outside of the public dominion, and land capable of registration as private property. 8. Foreshore land A foreshore land has been defined as "that part of (the land) which is between high and low water and left dry by the flux and reflux of the tides," or The strip of land that lies between the high and low water marks and that is alternatively wet and dry according to the flow of the tide." If the submergence, however, of the land is due to precipitation, it does not become foreshore, despite its proximity to the waters. 9. Court not a trier of facts; not enough evidence to arrive a conclusive disposition; Remand The case has to be decided alongside the above principles and regretfully, the Court cannot make a ruling because it is not a trier of facts, and it is in possession of no evidence to assist it in arriving at a conclusive disposition. The Court thus remanded the case to the court a quo to determine whether or not the property subject of controversy is foreshore. Bautista v. Alarcon [G.R. No. 8153. December 24, 1912.] First Division, Torres (J): 4 concur, 2 concur in result. Facts: Toribio Bautista was the owner of a tract of land, used as a fishpond in the barrio of Pangjolo and the sitio called Talinducan, of the pueblo of Obando, Bulacan (N: fishery of Julian Santos; S: Cornelio Enriquez and Benito Enriquez, E: Talinducan River; W: fisheries of Toribio Alarcon and Julian Santos, and a canal or ditch which is between these fisheries). Bautistas fishpond is formed by two parcels of land, one low and the other high, and is divided into two separate fisheries, although together they are supplied with water, the low land from the Talinducan River, and the high land from the said canal or ditch which, in its western part, is connected with and derived from the Obando River, terminating at Bautista's fishpond and serving as a boundary line between Alarcons and Santos fisheries. Said ditch had existed for more than 50 years and had a width of about 6 meters. The bed of the said ditch or canal was not owned by neither Alarcon and Bautista. On October 1907, Alarcon and Santos occupied said ditch, constructed thereon the retaining walls of their respective fisheries, in such manner that they narrowed and reduced the bed of the ditch to an approximate width of 25 centimeters, thus obstructing and almost completely preventing the passage of the water, on which account the plaintiff's fishery on the high land had been almost entirely deprived or water. By written instrument of 10 April 1908, Bautista filed a complaint against Alarcon and Santos with a prayer for a preliminary injunction. After the filing of the complaint, Alarcon and Santos closed the canal completely and rendered Bautistas fishery completely dry. On 25 February 1909, the preliminary injunction was issued by the court. On 21 June 1911, the lower court ordered that the injunction become final and ordered Alarcon and Santos to cease occupying the tracts specified in the judgment (adjacent to the canal or ditch), to remove their respective dikes, and to pay Bautista P100 and the costs of the suit. Alarcon and Teodoro Raymundo, in substitution of Santos, appealed through a bill of exceptions. The Supreme Court affirmed the judgment appealed from, with the costs against the Alarcon and Raymundo. 1. Property of public ownership Article 339, No. 1, of the Civil Code provides that "property of public ownership is (1) that destined to the public use, such as roads, canals, rivers, torrents, ports, and bridges constructed by the State, and banks, shores, roadsteads, and that of a similar character." Article 407 of the same code provides that "(1) Rivers and their natural beds. (2) Continuous or intermittent waters from springs or brooks running in their natural beds and the said beds. (3) Waters using continuously or intermittently in lands of said public ownership. (4) Lakes and ponds formed by nature on public lands and their beds. (5) Rain water running through ravines or sandy beaches, the beds of which shall also be public property" are property of public ownership. 2. Law provisions applicable to the present case The provisions of Articles 339 (1) and 407 substantially agree with those of the Law of Waters of 3 August 1866, made applicable to these Islands by the Spanish Government and published in the Gaceta de Manila of 24 September 1871, as is shown by the text of articles 33, 36, 37, 39, 72, and others relevant to the matter under discussion. 3. No private person has the right to usurp possession of a watercourse belonging to the public domain No private person has a right to usurp possession of a watercourse, branch of a river, or lake of the public domain and use, unless it shall have been proved that he constructed the same within property of his exclusive ownership, and such usurpation constitutes a violation of the legal provisions which explicitly exclude such waterways from the exclusive use or possession of a private party. 4. Canal, together with its water, is a property of public use and not susceptible of private appropriation The canal, together with the water that flows through it, drawn from the Obando River, belongs to that class of property of public use and domain which is not susceptible of private appropriation, and, therefore, private parties could not, under any circumstances, usurp the greater portion of it without committing a notorious and glaring violation of the law that protects the properties of the State and the rights of its citizens. Both parties are entitled to utilize, for the needs and benefit of their fisheries, the water which flows from the Obando River into the said canal, but none of them may utilize and receive such water exclusively and to the detriment of the rest, as did Alarcon and Santos by reducing the bed of the canal, thereby almost depriving Bautista of the quantity of water necessary for the maintenance of his fishery. 5. No proof adduced that Alarcon and Santos are owners of the canal Alarcon and Santos have not adduced any proof whatever that they are the owners of the canal, to allow them to reduce the volume of the flow by constructing, near the middle of the canal, the dikes of their respective fisheries. Neither have they adduced any proof whatever that they are the owners of the right to an easement for conveying water, which was acquired by Bautista through prescription for a period of more than 40 years, as is demonstrated by the record.

6. Alarcon and Santos liable for damages Alarcon and Santos, the latter through Raymundo, are jointly obligated to indemnify Bautista for the losses and damages which they occasioned him by the construction of their fishery dikes and the reduction of the capacity of the said canal, in the amount fixed by the trial court in the judgment appealed from, the findings of which are approved, as they are in accordance with law. Maneclang v. IAC [G.R. No. L-66575. September 30, 1986.] Second Division, Fernan (J): 4 concur Facts: Adriano Maneclang, et.al., petitioners, filed before the then CFI Pangasinan (Branch XI) a complaint for quieting of title over a certain fishpond located within 4 parcels of land belonging to them situated in Barrio Salomague, Bugallon, Pangasinan, and the annulment of Resolutions 38 and 95 of the Municipal Council of Bugallon, Pangasinan. On 15 August 1975, the trial court dismissed the complaint upon a finding that the body of water traversing the titled properties is a creek constituting a tributary of the Agno River (therefore public in nature and not subject to private appropriation); and held that Resolution 38, ordering an ocular inspection of the Cayangan Creek situated between Barrios Salomague Sur and Salomague Norte, and Resolution 95 authorizing public bidding for the lease of all municipal ferries and fisheries were passed by the members of the Municipal Council of Bugallon, Pangasinan in the exercise of their legislative powers. Manaclang appealed said decision to the IAC, which affirmed the same on 29 April 1983. Hence, the petition for review on certiorari. Before the respondents were able to comment on the petition, the petitioners manifested that for lack of interest on the part of respondent Alfredo Maza, the awardee in the public bidding of the fishpond, as the parties desire to amicably settle the case by submitting to the Court a Compromise Agreement praying that judgment be rendered recognizing the ownership of the petitioners over the land the body of water found within their titled properties. The Supreme Court dismissed the petition for lack of merit, and set aside the Compromise Agreement and declare the same null and void for being contrary to law and public policy. 1. Stipulations null and void for being contrary to law and public policy The stipulations contained in the Compromise Agreement partake of the nature of an adjudication of ownership of the fishpond in dispute, which was originally a creek forming a tributary of the Agno River. A creek, defined as a recess or arm extending from a river and participating in the ebb and flow of the sea, is a property belonging to the public domain which is not susceptible to private appropriation and acquisitive prescription (Mercado vs. Municipal President of Macabebe), and as a public water, it cannot be registered under the Torrens System in the name of any individual (Diego v. CA; Mangaldan v. Manaoag) and considering further that neither the mere construction of irrigation dikes by the National Irrigation Administration which prevented the water from flowing in and out of the subject fishpond, nor its conversion into a fishpond, alter or change the nature of the creek as a property of the public domain. The Compromise Agreement, thus, is null and void and of no legal effect, the same being contrary to law and public policy. 2. Municipal council authorized to pass laws dealing with its municipal waters The Municipality of Bugallon, acting thru its duly-constituted municipal council is clothed with authority to pass, as it did the two resolutions dealing with its municipal waters. 3. Publication a constructive notice to the whole world; due process followed Petitioners were not deprived of their right to due process as mere publication of the notice of the public bidding suffices as a constructive notice to the whole world. Mapa v. Insular Government [G.R. No. L-3793. February 19, 1908.] First Division, Willard (J): 2 concur, 1 concur in result Facts: Mapa sought to register a tract of land of about 16 hectares in extent, situated in the barrio San Antonio, Mandurriao, municipality of Iloilo before the Court of Land Registration. Judgment was rendered in favor of Mapa and the Government has appealed. A motion for a new trial was made and denied in the lower court, but no exception was taken to the order denying it. Appeal was made before the Supreme Court. The Supreme Court affirmed the judgment of the court below, with the costs against the appellant. 1. Ruling of the lower court in relation to Section 54, paragraph 6 of Act 926 Section 54, paragraph 6 of Act 926 provides that all persons who by themselves or their predecessors in interest have been in the open, continuous exclusive, and notorious possession and occupation of agricultural public lands, as defined by said act of Congress of 1 July 1902, under a bona fide claim of ownership except as against the Government, for a period of 10 years next preceding the taking effect of this act, except when prevented by war, or force majeure, shall be conclusively presumed to have performed all the conditions essential to a Government grant and to have received the same, and shall be entitled to a certificate of title to such land under the provisions of this chapter. In the present case, the lower court ruled that "from the evidence adduced it appears that the land in question is lowland, and has been uninterruptedly, for more than twenty years, in the possession of the petitioner and his ancestors as owners and the same has been used during the said period, and up to the present, as fish ponds, nipa lands, and salt deposits. The witnesses declare that the land is far from the sea, the town of Molo being between the sea and the said land." 2. Definition of agricultural land as used in Act 926 rather its meaning as to its nature The issue is not what is agricultural land, but what definition has been given to that phrase by the act of Congress. The phrase

"agricultural public lands" as defined by the act of Congress of 1 July 1902 is found not only in section 54 but in other parts of Act 926, and it seems that the same construction must be given to the phrase wherever it occurs in any part of that law. 3. Effect if agricultural land is taken to mean exclusively to be that of its nature, as claim by attorney-general The claim of the Attorney-General seems to be that no lands can be called agricultural lands unless they are such by their nature. If the contention of the Attorney-General is correct, and this land because of its nature is not agricultural land. It could not allow the land to be entered as a homestead, for Chapter I of Act 926 allows the entry of homesteads only upon "agricultural public lands" in the Philippine Islands, as defined by the act of Congress of 1 July 1902. It could not sell it in accordance with the provisions of Chapter II of Act 926, for section 10 only authorizes the sale of "unreserved nonmineral agricultural public land in the Philippine Islands, as defined in the act of Congress of 1 July 1902. It could not lease it in accordance with the provisions of Chapter III of the said act, for section 22 relating to leases limits them to "nonmineral public lands, as defined by section 18 and 20 of the act of Congress of 1 July 1902. The Government could not give a free patent to this land to a native settler, in accordance with the provisions of Chapter IV, for that relates only to "agricultural public land, as defined by act of Congress of 1 July 1902. 4. Section 13 and 15 of the Act provides only semblance of the definition of the phrase Section 13 provides that "the Government of the Philippine Islands, subject to the provisions of this act and except as herein provided, shall classify according to its agricultural character and productiveness, and shall immediately make rules and regulations for the lease, sale, or other disposition of the public lands other than timber or mineral lands, but such rules and regulations shall not go into effect of have the force of law until they have received the approval of the President, and when approved by the President they shall be submitted by him to Congress at the beginning of the next ensuing session thereof and unless disapproved or amended by Congress at said session they shall at the close of such period have the force and effect of law in the Philippine Islands: Provided, That a single homestead entry shall not exceed 16 hectares in extent. On the other hand, Section 15 provides that the Government of the Philippine Islands is hereby authorized and empowered on such terms as it may prescribe, by general legislation, to provide for the granting or sale and conveyance to actual occupants and settlers and other citizens of said Islands such parts and portions of the public domain, other than timber and mineral lands, of the United States in said Islands as it may deem wise, not exceeding sixteen hectares to any one person and for the sale and conveyance of not more than one thousand and twenty-four hectares to any corporation or association of persons: Provided, that the grant or sale of such lands, whether the purchase price be paid at once or in partial payments shall be conditioned upon actual and continued occupancy, improvement, and cultivation of the premises sold for a period of not less than five years, during which time the purchaser or grantee can not alienate or encumber said land or the title thereto; but such restriction shall not apply to transfers of rights and title of inheritance under the laws for the distribution of the estates of decedents." Neither one of these sections gives any express definition of the phrase "agricultural land." 5. Method to deciding the question whether land is agricultural land; Agricultural land are those public lands acquired from Spain which are neither mineral or timber lands There seem to be only 3 possible ways of deciding the question whether the land is agricultural. The first is to say that no definition of the phrase "agricultural land" can be found in the act of Congress; the second, that there is a definition of that phrase in the act and that it means land which in its nature is agricultural; and, third, that there is a definition in the act and that the phrase means all of the public lands acquired from Spain except those which are mineral or timber lands. The court below adopted the latter view, and held that the land, not being timber or mineral land, came within the definition of agricultural land, and that therefore Section 54 paragraph 6, Act 926 was applicable thereto. 6. Construction of the phrase agricultural land cannot be left to remain uncertain The objection to adopting the construction on account of its uncertainty is emphasized when it is consider that whether certain land was or was not agricultural land would be a question that would finally have to be determined by the courts, unless there is some express provision of the law authorizing the administrative officers to determine the question for themselves. After homesteads have been entered, lands, sold, and leases made by the administrative officers (pursuant to Section 2 of Act 926, Section 13, Section 26, Section 34) on the theory that the lands were agricultural lands by their nature, to leave the matter of their true character open for subsequent action by the courts would be to produce an evil that should if possible be avoided. The construction of the phrase would never be entirely free from objection, but the Court believes that the construction adopted is less objectionable than any other one that has been suggested. 7. Power to determine nature of land by Chief of the Bureau of Public Lands pursuant to Act 926 Section 2 of Act 926 relating to homesteads provides that the Chief of The Bureau of Public Lands shall summarily determine whether the land described is prima facie under the law subject to homestead settlement. Section 13, relating to the sale of public lands, provides simply that the Chief of the Bureau of Public Lands shall determine from the certificate of the Chief of the Bureau of Forestry whether the land applied for is more valuable for agricultural than for timber purposes, but it says nothing about his decisions as to whether it is or is not agricultural land in its nature. Section 26 relating to the lease of public lands provides that the Chief of the Bureau of Public Lands shall determine from the certificate of the Chief of the Bureau of Forestry whether the land applied for is more valuable for agricultural than for timber purposes and further summarily determine from available records whether the land is or is not mineral and does not contain deposits of coal or salts. Section 34 relating to free patents to native settlers makes no provision for any determination by the Chief of Bureau of Public Lands in regard to the character of the land applied for. 8. Jones v. Insular Government not conflicting; character of land not raised in that case, but the law pertaining to agricultural lands There is nothing in this case of Jones vs. The Insular Government which at all conflicts with the result here arrived at. The question as to whether the lands there involved were or were not agricultural lands within the meaning of the sections was neither discussed nor decided. In fact, it appears from the decision that those lands, which were in the Province of Benguet, were within the strictest definition of the phrase "agricultural lands." It appears that such lands had been cultivated for more than twelve years. What that case decided was, not that the lands therein involved and other lands referred to in the decision by way of illustration

were not agricultural lands but that the law there in question and the other laws mentioned therein were not rules and regulations within the meaning of section 13. Director of Forestry v. Villareal [G.R. No. L-32266. February 27, 1989.] En Banc, Cruz (J): 13 concur, 1 took no part. Facts: Ruperto Villareal applied for its registration on 25 January 1949, a land consisting of 178,113 sq. m. of mangrove swamps located in the municipality of Sapian, Capiz, alleging that he and his predecessors-in-interest had been in possession of the land for more than 40 years. He was opposed by several persons, including the Director of Foresty on behalf of the Republic of the Philippines. After trial, the application was approved by the CFI Capiz. The decision was affirmed by the Court of Appeals. The Director of Forestry then came to the Supreme Court in a petition for review on certiorari claiming that the land in dispute was forestal in nature and not subject to private appropriation. The Supreme Court set aside the decision of the Court of Appeals and dismissed the application for registration of title of Villareal, with cost against him; the decision being immediately executory. 1. Classification of the lands of the public domain By the Philippine Bill of 1902, one of the earlier American organic acts in the country, lands of the public domain in the Philippine Islands were classified into three grand divisions: agricultural, mineral and timber or forest lands. This classification was maintained in the Constitution of the Commonwealth, promulgated in 1935, until it was superseded by the Constitution of 1973. The 1973 charter expanded the classification of public lands to include industrial or commercial, residential, resettlement, and grazing lands and even permitted the legislature to provide for other categories. This provision has been reproduced, but with substantial modifications, in the present 1987 Constitution. 2. Only public agricultural land allowed to be alienated under Commonwealth Constitution Under the Commonwealth Constitution, which was the charter in force when the case arose, only agricultural lands were allowed to be alienated. Their disposition was provided for under CA 141 (Public Land Act). Mineral and timber or forest lands were not subject to private ownership unless they were first reclassified as agricultural lands and so released for alienation. 3. Decisions reiterating mangrove swamps are agricultural land

a. Montano v. Insular Government; Mangrove swamps or manglares defined In Montano v. Insular Government (1909) mangrove swamps or manglares were defined as mud flats, alternately washed and exposed by the tide, in which grows various kindred plants which will not live except when watered by the sea, extending their roots deep into the mud and casting their seeds, which also germinate there. These constitute the mangrove flats of the tropics, which exist naturally, but which are also, to some extent cultivated by man for the sake of the combustible wood of the mangrove and like trees as well as for the useful nipa palm propagated thereon. Although these flats are literally tidal lands, they cannot be so regarded in the sense in which that term is used in the cases cited or in general American jurisprudence. The waters flowing over them are not available for purpose of navigation, and they may be disposed of without impairment of the public interest in what remains. b. Montano v. Insular Government; Mangrove swamps considered agricultural lands and susceptible to private ownership Mangrove swamps were considered agricultural lands and so susceptible of private ownership, as under the uncertain and somewhat unsatisfactory condition of the law, the custom had grown of converting manglares and nipa lands into fisheries which became a common feature of settlement along the coast and at the same time of the change of sovereignty constituted one of the most productive industries of the Islands, the abrogation of which would destroy vested interests and prove a public disaster. c. Jocson v. Director of Forestry; Manglare with few trees is not timber land Notwithstanding the definition provided by the legislature in the 1917 Administrative Code as to public forests, the Court maintained the doctrine in the Montano case when 2 years later it held in the case of Jocson v. Director of Forestry that the words timber land are always translated in the Spanish translation of that Act (Act of Congress) as 'terrenos forestales.' The Court thought there is an error in this translation and that a better translation would be 'terrenos madereros. Timber land in English means land with trees growing on it. The manglar plant would never be cited a tree in English but a bush, and land which has only bushes, shrubs or aquatic plants growing on it cannot be called 'timber land.' The fact that there are a few trees growing in a manglare or nipa swamps does not change the general character of the land from manglare to timber land." d. Jocson v. Director of Forestry; Definition found in the 1917 Administrative Code cannot affect rights vested prior to its enactment; Act 926 applies to those cases Addressing itself directly to Section 1820, the Court declared that in the case of Mapa vs. Insular Government (10 Phil. Rep., 175), the Court said that the phrase 'agricultural lands' as used in Act 926 means those public lands acquired from Spain which are not timber or mineral lands. Whatever may have been the meaning of the term 'forestry' under the Spanish law, the Act of Congress of July 1st, 1902, classifies the public lands in the Philippine Islands as timber, mineral or agricultural lands, and all public lands that are not timber or mineral lands are necessarily agricultural public lands, whether they are used as nipa swamps, manglares, fisheries or ordinary farm lands. The definition of forestry as including manglares found in the Administrative Code of 1917 cannot affect rights which vested prior to its enactment. These lands being neither timber nor mineral lands, the trial court should have considered them agricultural lands. If they are agricultural lands, then the rights of appellants are fully established by Act 926."

e. Vda. De Centenera v. Obias; Mangrove lands are not forest as used in the Act of Congress In Garchitorena Vda. de Centenera v. Obias, promulgated on 4 March 1933, more than 15 years after the effectivity of the Administrative Code of 1917. The Court declare that the opposition of the Director of Forestry is untenable, inasmuch as it has been definitely decided that mangrove lands are not forest lands in the sense in which this phrase is used in the Act of Congress. No elaboration was made on this conclusion which was merely based on the cases of Montano and Jocson. f. Tongson v. Director of Forestry; Mangrove lands are agricultural in nature In Tongson v. Director of Forestry, the ruling in Vda. De Centenera v. Obias was reiterated, declaring that the mangrove lands in litis were agricultural in nature. The decision even quoted with approval the statement of the trial court that mangrove swamps where only trees of mangrove species grow, where the trees are small and sparse, fit only for firewood purposes and the trees growing are not of commercial value as lumber do not convert the land into public land. Such lands are not forest in character. They do not form part of the public domain." g. Republic v. de Porkan; All public lands not timber or mineral lands are agricultural public lands In Republic v. De Porkan (1988), the Court, citing Krivenko v. Register of Deeds, reiterated the ruling in the Mapa case that "all public lands that are not timber or mineral lands are necessarily agricultural public lands, whether they are used as nipa swamps, manglares, fisheries or ordinary farm lands." 4. Administrative Code of 1917; Mangrove swamps form part of the public forests of the country Subsequently, the Philippine Legislature categorically declared that mangrove swamps form part of the public forests of this country. This it did in the Administrative Code of 1917, which became effective on 1 October 1917, providing in Section 1820 of said code that for the purpose of this chapter 'public forest' includes, except as otherwise specially indicated, all unreserved public land, including nipa and mangrove swamps, and all forest reserves of whatever character." The legislative definition embodied in Section 1820 of the Revised Administrative Code of 1917, remains unamended up to now, provides that mangrove swamps or manglares form part of the public forests of the Philippines. As such, they are not alienable under the Constitution and may not be the subject of private ownership until and unless they are first released as forest land and classified as alienable agricultural land. 5. Decisions declaring mangrove lands form part of public domain

a. Yngson v. Secretary of Agriculture and Natural Resources; Bureau of Fisheries has no jurisdiction over mangrove lands still classified as forest lands In Yngson v. Secretary of Agriculture and Natural Resources (1983), the Court ruled "that the Bureau of Fisheries has no jurisdiction to dispose of swamplands or mangrove lands forming part of the public domain while such lands are still classified as forest lands." b. Heirs of Amunategui v. Director of Forestry; Forests do not lose classification if stripped of its cover In Heirs of Amunategui v. Director of Forestry, the Court held that a forested area classified as forest land of the public domain does not lose such classification simply because loggers or settlers may have stripped it of its forest cover. Parcels of land classified as forest land may actually be covered with grass or planted to crops by kaingin cultivators or other farmers. 'Forested lands' do not have to be on mountains or in out-of-the-way places. Swampy areas covered by mangrove trees, nipa palms, and other trees growing in brackish or sea water may also be classified as forest land. The classification is descriptive of its legal nature or status and does not have to be descriptive of what the land actually looks like. Unless and until the land classified as 'forest' is released in an official proclamation to that effect so that it may form part of the disposable agricultural lands of the public domain, the rules on confirmation of imperfect titles do not apply." c. Vallarta v. IAC; Forest lands or swamp mangrove are not private properties, not registerable In Vallarta v. Intermediate Appellate Court, the Court agreed with the Solicitor General's submission that the land in dispute, which he described as "swamp mangrove or forestal land," were not private properties and so not registerable. This case was decided only 12 days after the De Porkan case. 6. Classification of land is the function initially belonging to the legislature; Role of each branch of government The determination of the question of classification of lands is a function initially belonging to the legislature, which has the authority to implement the constitutional provision classifying the lands of the public domain (and is now even permitted to provide for more categories of public lands). The legislature having made such implementation, the executive officials may then, in the discharge of their own role, administer our public lands pursuant to their constitutional duty "to ensure that the laws be faithfully executed" and in accordance with the policy prescribed. For their part the courts will step into the picture if the rules laid down by the legislature are challenged or, assuming they are valid, it is claimed that they are not being correctly observed by the executive. Thus do the three departments, coordinating with each other, pursue and achieve the objectives of the Constitution in the conservation and utilization of our natural resources. 7. Delegation of powers, President given function of making periodic classification of public lands In CA 141, the National Assembly delegated to the President of the Philippines the function of making periodic classifications of public lands. Section 6 provides that the President, upon the recommendation of the Secretary of Agriculture and Natural Resources, shall from time to time classify the lands of the public domain into: (a) Alienable or disposable, (b) Timber, and (c) Mineral lands, and may at any time and in a like manner transfer such lands from one class to another, for the purposes of their administration and disposition." Section 7 of the same Act provides that for the purposes of the administration and disposition of alienable or disposable lands, the President, upon recommendation by the Secretary of Agriculture and Natural Resources, shall from time to time declare what lands are open to disposition or concession under this Act.

With particular regard to alienable public lands, Section 9 of the same law provides that "for the purpose of their administration and disposition, the lands of the public domain alienable or open to disposition shall be classified, according to the use or purposes to which such lands are destined, as follows: (a) Agricultural; (b) Residential, commercial, industrial, or for similar productive purposes; (c) Educational, charitable, or other similar purposes; and (d) Reservations for townsites and for public and quasi-public uses. The President, upon recommendation by the Secretary of Agriculture and Natural Resources, shall from time to time make the classifications provided for in this section, and may, at any time and in a similar manner, transfer lands from one class to another." As for timber or forest lands, the Revised Administrative Code provides in Section 1826 (Regulation setting apart forest reserves; Revocation of same) that Upon the recommendation of the Director of Forestry, with the approval of the Department Head, the President of the Philippines may set apart forest reserves from the public lands and he shall by proclamation declare the establishment of such reserves and the boundaries thereof, and thereafter such forest reserves shall not be entered, sold, or otherwise disposed of, but shall remain as such for forest uses, and shall be administered in the same manner as public forest. The President of the Philippines may in like manner by proclamation alter or modify the boundaries of any forest reserve from time to time, or revoke any such proclamation, and upon such revocation such forest reserve shall be and become part of the public lands as though such proclamation had never been made. Section 1827 (Assignment of forest land for agricultural purposes) provides that lands in public forest, not including forest reserves, upon the certification of the Director of Forestry that said lands are better adapted and more valuable for agricultural than for forest purposes and not required by the public interests to be kept under forest, shall be declared by the Department Head to be agricultural lands." 8. Mangrove swamps or manglares comprised within the public forests of the Philippines Mangrove swamps or manglares should be understood as comprised within the public forests of the Philippines as defined in the Section 1820 of the Administrative Code of 1917. The legislature having so determined, the Court has no authority to ignore or modify its decision, and in effect veto it, in the exercise of our own discretion. The statutory definition remains unchanged to date and, no less noteworthy, is accepted and invoked by the executive department. More importantly, the said provision has not been challenged as arbitrary or unrealistic or unconstitutional, assuming the requisite conditions, to justify our judicial intervention and scrutiny. The law is thus presumed valid and so must be respected. The Court repeats its statement in the Amunategui case that the classification of mangrove swamps as forest lands is descriptive of its legal nature or status and does not have to be descriptive of what the land actually looks like. That determination having been made and no cogent argument having been raised to annul it, the Court has no duty as judges but to apply it. 9. Holdings as that manglares as agricultural land covers only those lands which ownership had already vested before the effectivity of the 1917 Administrative Code The Courts previous description of the term in question as pertaining to agricultural lands should be understood as covering only those lands over which ownership had already vested before the Administrative Code of 1917 became effective. Such lands could not be retroactively legislated as forest lands because this would be violative of a duly acquired property right protected by the due process clause. 10. Mangrove swamps of Sapian are forest land The land under contention, being admittedly a part of the mangrove swamps of Sapian, and for which a minor forest license had in fact been issued by the Bureau of Forestry from 1920 to 1950, it must be considered forest land. It could therefore not be the subject of the adverse possession and consequent ownership claimed by the private respondent in support of his application for registration. To be so, it had first to be released as forest land and reclassified as agricultural land pursuant to the certification the Director of Forestry may issue under Section 1827 of the Revised Administrative Code. 11. Director of Forestry, not Director of Lands, has jurisdiction over the property The survey plan of the mangrove swamps approved by the Director of Lands, to prove that the land is registerable, cannot be invoked. The mere existence of such a plan would not have the effect of converting the mangrove swamps, as forest land, into agricultural land. Such approval is ineffectual because it is clearly inofficious. The Director of Lands was not authorized to act in the premises. Under the law, it is the Director of Forestry who has the authority to determine whether forest land is more valuable for agricultural rather than forestry uses, as a basis for its declaration as agricultural land and release for private ownership. 12. Right to the registration of subject land not established Applicant has not established his right to the registration of the subject land in his name. The record contains no convincing evidence of the existence of the informacion posesoria allegedly obtained by the original transferor of the property, let alone the fact that the conditions for acquiring title thereunder have been satisfied. Nowhere has it been shown that the informacion posesoria has been inscribed or registered in the registry of property and that the land has been under the actual and adverse possession of the private respondent for twenty years as required by the Spanish Mortgage Law. These matters are not presumed but must be established with definite proof, which is lacking in the present case. 13. Tax declarations are not sufficient to prove possession Tax declarations are not sufficient to prove possession and much less vest ownership in favor of the declarant, as the Court has held in countless cases. Manalo v. IAC [G.R. No. 64753. April 26, 1989.] Third Division, Gutierrez Jr. (J): 3 concur, 1 took no part Facts: The de Ocampos and the Santoses claim to be the co-owners of 2 parcels of land containing an area of 33.6344 hectares, more or less, and presently embraced within TCT T-44205 and T-43298 respectively, both of the Registry of Bataan; that the TCTs were acquired by the de Ocampos and the Santoses by virtue of Sales Patents 5339 and 5387 issued on 17 November 1972 and 3 February 1973, respectively, by the Director of Lands under CA 141(Public Land Law). On 2 October 1971,in view of the representation of Placido and Armando Manalo that they have in actual, peaceful, continuous and open possession of the parcels

of land in Cabcaben, Mariveles, Bataan since 1944 as evidence by their documents duly filed with the Bureau of Forestry and of Lands, although the same were still then part of the U.S. Military Reservation; the Director of Lands issued Free Patents 522897 and 502977 to the Manalos, by virtue of Free Patent Application (III-4) 508 and (III-4) 519 filed with the Bureau of Lands under the provisions of Section 44, Chapter VII of the Public Land Law, and by virtue of which OCTs 296 and 297 were respectively issued in the names of the petitioners covering the disputed parcels of land. On 18 October 1973, the de Ocampos and the Santoses instituted an action for the cancellation of the Manalos titles over certain parcels of land. The former prayed that their titles over the said parcels of land be declared as the true and valid ones. After hearing, the trial court found for the de Ocampos and the Santoses and ordered the cancellation of the Manalos' titles over the lots in dispute. On appeal, the appellate court sustained the trial court's decision. Hence, the Manalos filed a petition for review by certiorari. The Supreme Court denied the petition, and affirmed the decision of the Court of Appeals; with costs against the Manalos. 1. Private respondents de Ocampo and Santos have legal personality to file When the lots in dispute were certified as disposable on 19 May 1971, and free patents were issued covering the same, the said lots ceased to be part of the public domain and, therefore, the Director of Lands lost jurisdiction over them. Since the lots were no longer part of the public domain, holders of the titles based on free patents acquired subsequent to the declaration of alienability and disposability, have the personality to file the case against persons whom they alleged were in possession of void titles (see Heirs of Tanak Pangawaran Patiwayan v. Martinez, 142 SCRA 252, 258-260 [1986]; citing the Sumail case). 2. Land in former Military Reservation made disposable in 1971; Character unchanged during transfer from US to the Philippines in 1965 It is not correct to say that when the U.S. Military Reservation in Bataan, of which the land in question forms part, was turned over to the Philippine government, the same automatically became a disposable land of the public domain. The ownership and control over said reservation was transferred to the Philippine government, but its nature as a military reservation remained unchanged. Said parcels of land became a disposable land of public domain only on 19 May 1971, per certification of the Bureau of Forestry (Project 4-A, C-C. Map 26-40). Its disposition only by sale was duly authorized pursuant to the provisions of RA 274. If the land in question became immediately disposable upon its turn over to the Philippine government in 1965, then there is no need to certify it to be disposable in 1971. The land, thus, continued to be a military reservation land while in the custody of the Philippine government until it was certified alienable in 1971. 3. Unclassified land remains as is until released and rendered open for disposition In Republic v. Intermediate Appellate Court, (155 SCRA 412, 418-419 [1987]), it was held that the classification of public lands is an exclusive prerogative of the Executive Department of the Government and not of the Courts. In the absence of such classification, the land remains as unclassified land until it is released therefrom and rendered open to disposition (Sec. 8, CA 141, as amended: Yngson v. Secretary of Agriculture and Natural Resources, 123 SCRA 441 [193]; Republic v. Court of Appeals, 99 SCRA 742 [1980]). This should be so under time-honored Constitutional precepts. This is also in consonance with the Regalian doctrine that all lands of the public domain belong to the State (Secs. 8 & 10, Art. XIV, 1973 Constitution), and that the State is the source of any asserted right to ownership in land and charged with the conservation of such patrimony (Republic v. Court of Appeals, 89 SCRA 648 [1979]). 4. Finding of facts of CA conclusive absent the recognized exceptions It is not the function of the Supreme Court to evaluate each piece of evidence presented before the lower court. Suffice it to say that we find the conclusions of the lower court and appellate courts amply supported by evidence and the time-honored doctrine is applied; that absent the recognized exceptions, the findings of fact of the Court of Appeals are conclusive on the parties and the Supreme Court; and that this Court decides appeals which only involve questions of law. (See Philippine National Bank v. Court of Appeals, 159 SCRA 433, 445 [1988]). 5. Manalos title null and void as it was obtained in contravention with the requirements provided by law The big tract of land in Mariveles, Bataan to which the parcels of land involved in the case belong was formerly a portion of the U.S. Military Reservation in Mariveles, Bataan which was turned over to the Philippine Government only on 22 December 1965 (Republic v. CA, 89 SCRA 648 [1979]). Under such a situation, the Court seriously doubts whether Placido Manalo and their predecessors in-interest could have been in possession of the land since 1944 as they claimed because "lands covered by reservation are not subject to entry, and no lawful settlement on them can be acquired (Republic v. CA, 76 SCRA 146 [1976])." Thus, the Manalos appeared not to have satisfied the requirement of possession since 4 July 1945. Further, one of the requisites before a free patent could be issued would be an ocular inspection. Nothing was shown that such had been made. Lastly, the Bureau of Lands had no jurisdiction to accept the Manalo application in April 1967 as the lands were not yet surveyed. The survey plans were only approved on 17 June 1971, the annotations that the land became disposable and alienable made only on 19 May 1971. Their titles to the land in question, thus, are null and void, having been obtained in contravention with the requirements provided by law. Director of Lands v. IAC [G.R. No. 73246. March 2, 1993.] Second Division, Nocon (J): 4 concur Facts: Land involved is an island known as Tambac Island in Lingayen Gulf. Situated in the Municipality of Bani, Pangasinan, the area consists of 187,288 sq. m., more or less. The initial application for registration was filed for Pacific Farms, Inc. under the provisions of the Land Registration Act, Act 496, as amended. The Republic of the Philippines, thru the Director of Lands opposed the application alleging that the applicant, Pacific Farms, Inc. does not possess a fee simple title to the land nor did its predecessors possess the land for at least 30 years immediately preceding the filing of application. The opposition likewise specifically alleged that the applicant is a private corporation disqualified under the 1973 Constitution from acquiring alienable lands of the public domain citing Section 11, Article 14. The Director of Forest Development also entered its opposition alleging

that the land is within the unclassified public land and, hence, inalienable. Other private parties also filed their oppositions, but were subsequently withdrawn. In an amended application, Pacific Farms, Inc. filed a manifestation-motion to change the applicant from Pacific Farms, Inc. to J. Antonio Araneta. Despite the supposed amendment, there was no republication. On 4 October 1979, the trial court rendered a decision adjudicating the subject property to J. Antonio Araneta. On appeal to the then Intermediate Appellate Court, the decision of the lower court was affirmed on 12 December 1985. Hence, the petition for review. The Supreme Court granted the petition, and reversed the decisions of the courts a quo. 1. Submission of tracing cloth mandatory in registration; Failure to submit original tracing cloth as evidence fatal In Director of Lands v. The Honorable Intermediate Appellate Court and Lino Anit, it was ruled that the submission of the tracing cloth plan is a mandatory requirement for registration. In Director of Lands v. Reyes, on the other hand, it was held that failure to submit in evidence the original tracing cloth plan is fatal it being a statutory requirement of mandatory character. It is of no import that the Director of Lands failed to object to the presentation of the certified copy of the said plan. What is required is the original tracing cloth plan of the land applied for and objection to such requirement cannot be waived either expressly or impliedly. As held in Director of Lands v. Reyes, if the original tracing cloth plan was indeed with the Land Registration Commission, there is no reason why the applicant cannot easily retrieve the same and submit it in evidence, it being an essential requirement for registration. 2. Amendment o the name of applicant an attempt to evade disqualification The amendment of the application from the name of Pacific Farms, Inc., as applicant, to the name of J. Antonio Araneta, was a mere attempt to evade disqualification. The Constitution, whether the 1973 or 1987, prohibits private corporations or associations from holding alienable lands of the public domain except by lease. Apparently realizing such prohibition, applicant amended its application to conform with the mandates of the law. 3. Requirement of republication due to amendments to the application Amendments to the application may be due to change in parties or substantial change in the boundaries or increase in the area of the land applied for. As to amendments to the application due to change in parties, neither the Land Registration Act, as amended, nor PD 1529 (the Property Registration Decree_, requires republication and registration may be allowed by the court at any stage of the proceeding upon just and reasonable terms. On the other hand, republication is required if the amendment is due to substantial change in the boundaries or increase in the area of the land applied for. 4. PD 1529 applied properly in the present case There is no relevant dispute in the lower court's application of PD 1529, instead of Act 496, in adjudicating the land to the then applicant, assuming that the land involved is registrable. Both laws are existing and can stand together. PD 1529 was enacted to codify the various laws relative to registration of property, in order to facilitate effective implementation of said laws. 5. Classification of lands of the public domain Lands of the public domain are classified under three main categories, namely: Mineral, Forest and Disposable or Alienable Lands. Under the Commonwealth Constitution, only agricultural lands were allowed to be alienated. Their disposition was provided for under CA 141 (Secs. 6-7), which states that it is only the President, upon the recommendation of the proper department head, who has the authority to classify the lands of the public domain into alienable or disposable, timber and mineral lands. Mineral and Timber or forest lands are not subject to private ownership unless they are first reclassified as agricultural lands and so released for alienation. 6. Courts have no authority to classify unclassified land; Regalian Doctrine The courts cannot release the subject property from the unclassified category, which is beyond their competence and jurisdiction. The classification of public lands is an exclusive prerogative of the Executive Department of the Government and not of the Courts. In the absence of such classification, the land remains unclassified until released therefrom and rendered open to disposition. This is in consonance with the Regalian doctrine that all lands of the public domain belong to the State, and that the State is the source of any asserted right to ownership in land and charged with the conservation of such patrimony. Under the Regalian Doctrine, all lands not otherwise appearing to be clearly within private ownership are presumed to belong to the State. Hence, a positive act of the government is needed to declassify a forest land into alienable or disposable land for agricultural or other purposes. 7. Burden of proof in proving the land is alienable or disposable The burden of proof in overcoming the presumption of state ownership of the lands of the public domain is on the person applying for registration that the land subject of the application is alienable or disposable. Unless the applicant succeeds in showing by convincing evidence that the property involved was acquired by him or his ancestors either by composition title from the Spanish Government or by possessory information title, or any other means for the proper acquisition of public lands, the property must be held to be part of the public domain. The applicant must present evidence and persuasive proof to substantiate his claim. 8. Tax declarations and receipts not conclusive evidence of ownership Tax declarations and receipts are not conclusive evidence of ownership or of the right to possess land when not supported by evidence. The fact that the disputed property may have been declared for taxation purposes in the names of the applicants or of their predecessors-in-interest way back in 1921 does not necessarily prove ownership. They are merely indicia of a claim of ownership. 9. State cannot be estopped by omission, mistake or error of its officials In Director of Lands v. CA, it was held that it is a well-settled rule that the State cannot be estopped by the omission, mistake or error of its officials or agents, if omission there was, in fact. Thus, the fact that BF Map LC 673 dated 1 March 1927, in that case, showing subject property to be within unclassified region was not presented in evidence will not operate against the State. Likewise, in the present case, the contention that the BFD, LC Map No. 681, certified on August 8, 1927 which was the basis of

the report and recommendation of the Land Examiner, is too antiquated; that it cannot be conclusively relied upon and was not even presented in evidence, cannot be well taken. 10. Unclassified property; possession cannot ripen into private ownership Since the subject property is still unclassified, whatever possession the applicant may have had and however long, cannot ripen into private ownership. The conversion of subject property does not automatically render the property as alienable and disposable. 11. Government called upon to classify the land The Government should seriously consider the matter of the reclassification of the land in question. The attempt of people to have disposable lands they have been tilling for generations titled in their name should not only be viewed with understanding attitude, but as a matter of policy encouraged. Director of Lands v. CA [G.R. No. 58867. June 22, 1984.] First Division, Melencio-Herrera (J): 4 concur, 1 took no part Facts: In their application for registration filed on 10 May 1976, applicants claimed that they are the co-owners in fee simple of the land applied for (Lot 2347, Cad-302-D, Case 3, Obando Cadastre under Plan Ap-03-000535 situated in Obando Bulacan; approximately 9.3 hectares, adjoining Kailongan River and which has been converted to a fishpond) partly through inheritance in 1918 and partly by purchase on 2 May 1958; that it is not within any forest zone or military reservation; and that the same is assessed for taxation purposes in their names. The Republic of the Philippines, represented by the Director of the Bureau of Forest Development opposed the application on the ground that the land is within the unclassified region of Obando and thus are denominated as forest lands and do not form part of the alienable portion of the public domain. After hearing, the Trial Court ordered registration of the subject land in favor of the Applicants. This was affirmed on appeal by the Appellate Court. The parties stipulated that the land is within an unclassified region of Obando, Bulacan (as per BF Map LC 637, 1 March 1927). No evidence has been submitted that the land has been released or subsequently classified despite an Indorsement (17 November 1976), of the District Forester, to the Director of Forest Development that such land was devoid of any forest growth and forms part of a well-developed and producing fishponds, thus recommending the land to be disposed with the Public Land Law. The Supreme Court reversed the appealed decision, and dismissed the application for registration in Land Registration Case No. N-299-V-76 of the former CFI Bulacan, Branch III; without prejudice to the availment by the applicants of the proper administrative remedy. 1. Classification of public land an exclusive prerogative of the Executive Department; Regalian Doctrine The classification of public lands is an exclusive prerogative of the Executive Department of the Government and not of the Courts. In the absence of such classification, the land remains as unclassified land until it is released therefrom and rendered open to disposition. This should be so under time-honored Constitutional precepts. This is also in consonance with the Regalian doctrine that all lands of the public domain belong to the State, and that the State is the source of any asserted right to ownership in land and charged with the conservation of such patrimony. In the present case, the Courts a quo released the subject property from the unclassified category, which is beyond their competence and jurisdiction. 2. State cannot be estopped by the omission, mistake or error of its officials or agents The recommendation of the District Forester for release of subject property from the unclassified region is not the ultimate word on the matter. And the fact that BF Map LC No. 637 dated 1 March 1927 showing subject property to be within the unclassified region was not presented in evidence will not operate against the State considering the stipulation between the parties and under the well-settled rule that the State cannot be estopped by the omission, mistake or error of its officials or agents. 3. Cadastral survey does not release the land as alienable; Land still within jurisdiction of Bureau of Forest Development While it may be that the Municipality of Obando has been cadastrally surveyed in 1961, it does not follow that all lands comprised therein are automatically released as alienable. A survey made in a cadastral proceeding merely identifies each lot preparatory to a judicial proceeding for adjudication of title to any of the lands upon claim of interested parties. Besides, if land is within the jurisdiction of the Bureau of Forest Development, it would be beyond the jurisdiction of the Cadastral Court to register it under the Torrens System. 4. Unclassified land does not ripen to private ownership Since the subject property is still unclassified, whatever possession applicants may have had, and, however long, cannot ripen into private ownership. The conversion of subject property into a fishpond by applicants, or the alleged titling of properties around it, does not automatically render the property as alienable and disposable. 5. Applicants remedy lies in the release of the property from its present classification Applicants' remedy lies in the release of the property from its present classification. In fairness to applicants, and it appearing that there are titled lands around the subject property, petitioners-officials should give serious consideration to the matter of classification of the land in question. Republic v. IAC [G.R. No. 73085. June 4, 1990.] Second Division, Paras (J): 4 concur Facts: Claiming that they acquired the property by virtue of a document which they alleged to be a Spanish title originally issued in the name of Bernardo Merchan, the Merchans filed a complaint dated 7 August 1974 against Republic of the Philippines for quieting of title over said property located in Sitio de Malapianbato alias Arras, Barrio de Ayuti, Lucban, Quezon, containing an

area of 166 hectares, more or less. The Government moved to dismiss the complaint on the ground that the trial court had no jurisdiction over the subject matter of the case because the land is part of a forest reserve established by Proclamation 42 (14 October 1921), and by Proclamation 716 (26 May 1941) which declared the area as part of the "Mts. Banahaw-San Cristobal National Park." The motion was denied. The Merchans filed a motion to declare the Government in default for failure to file its answer within the reglementary period. The latter motion was granted. The Government filed for a motion for reconsideration. On 18 December 1975, Judge Manolo L. Madella rendered a decision declaring the Merchans as owners of the land subject of the litigation. Meanwhile, the Supreme Court rendered a resolution declaring null and void all judicial acts, decisions, orders and resolutions performed promulgated and issued by then Judge Madella after 2 January 1976. A motion to set aside the decision of 18 December 1975 was filed. On 21 April 1976, Judge Delia P. Medina now presiding in the trial court, issued an order declaring the Governments motion for reconsideration moot and academic in light of the aforementioned resolution; and required the Merchans to file an answer to the motion to set aside the decision of December 1975. Thereafter, and on 23 July 23, the motion to vacate was granted. Motion for reconsideration was filed by the Merchans, but were denied. On 16 September 1976, the Merchans filed a manifestation assailing the jurisdiction of the court to hear the case, which was denied. Thereafter on 27 September 1976, the Merchans filed with the Court of Appeals a petition for certiorari and prohibition with preliminary injunction against Judge Medina. On 29 November 1977, the CA denied the petition for certiorari and lifted the restraining order that it previously issued. Meanwhile, while the case above was pending before the CA and on 29 December 1976, 6 months after the effectivity of PD 892, the Merchans filed an application for the registration of the parcel of land involved in Civil Case 7840 (LRC N-1055). The trial court, this time presided by Judge Benigno M. Puno, issued an order setting the case for pre-trial. For failure of Government's counsel to attend the scheduled hearing, the trial court issued an order declaring the said failure as a waiver to present evidence and to cross-examine the Merchans' witnesses and declared the case submitted for decision. On 3 March 1980, the trial court rendered its decision in favor of the Merchans. The Government appealed to the then IAC, which affirmed the judgment of the trial court. Hence, the petition. The Supreme Court reversed the decision of the appellate court, and dismissed Civil Case 7840 and Registration Case N-1055. 1. Document not a Spanish title The document itself dated 29 July 1870, allegedly a Spanish title, is a mere instrument executed by Bernardo Merchan, private respondents' predecessor-in-interest, claiming possession over the land described therein which he sought to be recognized by the government during the Spanish regime. The document does not say it is a title, nor does it state that Bernardo Merchan has acquired ownership over the land. The document does not contain the specific area of the land which is claimed to be owned by the Merchans. 2. Land in question proclaimed as part of a forest reserved by Proclamation 42 and 716; Forest lands or forest reserves not capable of private appropriation; Possession does not convert them to private property The land in question was proclaimed part of a forest reserve by virtue of Proclamation 42 (14 October 1921). This proclamation was superseded by Proclamation 716 (26 May 1941) establishing the Mts. Banahaw-San Cristobal National Park. It is already a settled rule that forest lands or forest reserves are not capable of private appropriation, and possession thereof, however long, can not convert them into private property (Vano v. Government of the Philippine Islands, 41 Phil. 161 [1920]; Adorable v. Director of Forestry, 107 Phil. 401 [1960]; Director of Forestry v. Muoz, 132 Phil. 637 [1968]; Republic v. De la Cruz, 67 SCRA 221 [1975]; Director of Lands v. Reyes and Alinsunurin v. Director of Lands, 68 SCRA 177 [1975]; Republic v. Court of Appeals, 89 SCRA 648 [1979]; Republic v. Animas, 56 SCRA 499 [1974]; Director of Lands v. Court of Appeals, 133 SCRA 701 [1984]; Republic v. Court of Appeals, 135 SCRA 156 [1985]; Director of Lands v. Rivas, 141 SCRA 329 [1986]) unless such lands are reclassified and considered disposable and alienable by the Director of Forestry (Republic v. Court of Appeals, 154 SCRA 476 [1987]). In the present case, there is no proof of reclassification by the Director of Forestry that the land in question is disposable or alienable. 3. Lands not under Torrens System considered unregistered; PD 892 (16 February 1976) With the passage of PD 892, effective 16 February 1976, Spanish Titles can no longer be used as evidence of land ownership. Under the same decree, lands not under the Torrens System shall be considered as unregistered. Dacanay v. Asistio [G.R. No. 93654. May 6, 1992.] En Banc, Grino-Aquino (J): 13 concur, 1 took no part Facts: On 5 January 1979, MMC Ordinance 79-02 was enacted by the Metropolitan Manila Commission, designating certain city and municipal streets, roads and open spaces as sites for flea markets. Pursuant thereto, the Caloocan City mayor opened up 7 flea markets in that city. One of those streets was the "Heroes del '96" where the Francisco Dacanay lives. Upon application of vendors Rodolfo Teope, Mila Pastrana, Carmen Barbosa, Merle Castillo, Bienvenido Menes, Nancy Bugarin, Jose Manuel, Crisaldo Paguirigan, Alejandro Castron, Ruben Araneta, Juanita and Rafael Malibaran, and others, the city mayor and city engineer, issued them licenses to conduct vending activities on said street. In 1987, Antonio Martinez, as OIC city mayor of Caloocan City, caused the demolition of the market stalls on Heroes del '96, V. Gozon and Gonzales streets. To stop Mayor Martinez' efforts to clear the city streets, Teope, Pastrana and other stallowners filed an action for prohibition against the City of Caloocan, the OIC City Mayor and the City Engineer and/or their deputies before the RTC Caloocan City (Branch 122, Civil Case C-12921), praying the court to issue a writ of preliminary injunction ordering these city officials to discontinue the demolition of their stalls during the pendency of the action. The court issued the writ prayed for. However, on 20 December 1987, it dismissed the petition and lifted the writ of preliminary injunction which it had earlier issued. However, shortly after the decision came out, the city administration in Caloocan City changed hands. City Mayor Macario Asistio, Jr. did

not pursue the latter's policy of clearing and cleaning up the city streets. Invoking the trial courts decision, Francisco Dacanay wrote a letter to Mayor Asistio calling his attention to the illegally-constructed stalls on Heroes del 96 street and asked for demolition on 7 March 1988, wrote a follow-up letter to the mayor and the city engineer on 7 April 1988, and without receiving any response, sought the intervention of President Aquino through a letter. These letter was referred to the city mayor for appropriate action.

On 3 April 1989, Dacanay filed a complaint against Mayor Asistio and Engineer Sarne (OMB-0-89-0146) in the Office of the Ombudsman. After conducting a preliminary investigation, the Ombudsman rendered a final evaluation and report on 28 August 1989, finding that the Mayors and the City Engineer's inaction is purely motivated by their perceived moral and social responsibility toward their constituents, but "the fact remains that there is an omission of an act which ought to be performed, in clear violation of Sections 3(e) and (f) of RA 3019." The Ombudsman recommended the filing of the corresponding information in court. As the stallholders continued to occupy Heroes del '96 Street, through the tolerance of the city officials, and in clear violation of the decision in Civil Case C-12921, Dacanay filed a petition for mandamus on 19 June 1990, praying that the city officials be ordered to enforce the final decision in Civil Case C-12921 which upheld the city mayor's authority to order the demolition of market stalls on V. Gozon, Gonzales and Heroes del '96 Streets and to enforce PD 772 and other pertinent laws. The Supreme Court established that Dacanay and the general public have a legal right to the relief demanded and that the city officials have the corresponding duty, arising from public office, to clear the city streets and restore them to their specific public purpose (Enriquez vs. Bidin, 47 SCRA 183; City of Manila vs. Garcia et al., 19 SCRA 413 citing Unson vs. Lacson, 100 Phil. 695), and thus ordered the City Mayor and City Engineer of Caloocan City or their successors in office to immediately enforce and implement the decision in Civil Case C-1292 declaring that Heroes del '96, V. Gozon, and Gonzales Streets are public streets for public use, and they are ordered to remove or demolish, or cause to be removed or demolished, the market stalls occupying said city streets with utmost dispatch within 30 days from notice of the decision; the decision being immediately executory. 1. Jurisprudence applicable to property of public dominion The streets, being of public dominion must be outside of the commerce of man. Considering the nature of the subject premises, the following jurisprudence co/principles are applicable on the matter: (1) They cannot be alienated or leased or otherwise be the subject matter of contracts. (Municipality of Cavite vs. Rojas, 30 Phil. 602); (2) They cannot be acquired by prescription against the state (Insular Government vs. Aldecoa, 19 Phil. 505). Even municipalities can not acquire them for use as communal lands against the state (City of Manila vs. Insular Government, 10 Phil. 327); (3) They are not subject to attachment and execution (Tan Toco vs. Municipal Council of Iloilo, 49 Phil. 52); (4) They cannot be burdened by any voluntary easement (2-II Colin & Captain 520; Tolentino, Civil Code of the Phil. Vol. II, 1983 Ed. pp. 29-30). 2. Context of the ordinance of the Metropolitan Manila Commission as to the establishment of flea markets on municipal streets, roads and open spaces Ordinance 2, s. 1979 of the Metropolitan Manila Commission is an ordinance authorizing and regulating the use of certain city and/or municipal streets, roads and open spaces within Metropolitan Manila as sites for flea market and/or vending areas, under certain terms and conditions, subject to the approval of the Metropolitan Manila Commission, and for other purposes. Section 2 of said ordinance provides that the streets, roads and open spaces to be used as sites for flea markets (tiangge) or vending areas; the design, measurement or specification of the structures, equipment and apparatuses to be used or put up: the allowable distances: the days and time allowed for the conduct of the businesses and/or activities herein authorized; the rates or fees or charges to be imposed, levied and collected; the kinds of merchandise, goods and commodities sold and services rendered: and other matters and activities related to the establishment, maintenance and management and operation of flea markets and vending areas, shall be determined and prescribed by the mayors of the cities and municipalities in the Metropolitan Manila where the same are located, subject to the approval of the Metropolitan Manila Commission and consistent with the guidelines hereby prescribed. Section 6(m) of said ordinance provides that in the establishment operation, maintenance and management of flea markets and vending areas, the following guidelines, among others, shall be observed: xxx (m) that the permittee shall remove the equipment, facilities and other appurtenances used by him in the conduct of his business after the close or termination of business hours. 3. Related case, Municipality of Cavite; Return of rent In the case of Municipality of Cavite vs. Rojas, it was held that properties for public use may not be leased to private individuals. Such a lease is null and void for the reason that a municipal council cannot withdraw part of the plaza from public use. If possession has already been given, the lessee must restore possession by vacating it and the municipality must thereupon restore to him any sums it may have collected as rent. 4. Relate case, City of Manila v. Garcia; Ordinance legalizing the occupancy of squatters of public land is null and void In the case of City of Manila vs. Gerardo Garcia, 19 SCRA 413, it was held that tThe property being a public one, the Manila Mayors did not have the authority to give permits, written or oral, to the squatters, and that the permits granted are therefore considered null and void. As reiterated in the case of Baguio Citizens Action Inc. vs. The City Council. 121 SCRA 368, an ordinance legalizing the occupancy by squatters of public land is null and void. 5. Occupation of private individuals of public places devoted for public use a nuisance The occupation and use of private individuals of sidewalks and other public places devoted for public use constitute both public and private nuisances and nuisance per se, and this applies to even case involving the use or lease of public places under permits and licenses issued by competent authority, upon the theory that such holders could not take advantage of their unlawful permits and license and claim that the land in question is a part of a public street or a public place devoted to public use, hence, beyond the commerce of man. (Padilla. Civil Code Annotated, Vol. II, p. 59, 6th Ed., citing Umali vs. Aquino, IC. A. Rep. 339.).

6. Authority of the city mayor and the city engineer to order the demolition of illegal structures "The authority of the Municipality to demolish the shanties is mandated by PD 772, and Section 1 of Letter of Instruction 19 ordering certain public officials, one of whom is the Municipal Mayor to remove all illegal constructions including buildings on and along esteros and river banks, those along railroad tracks and those built without permits on public or private property (Zansibarian Residents Association vs. Mun. of Makati, 135 SCRA 235). The City Engineer is also among those required to comply with said Letter of Instruction. Further, Section 185, paragraph 4 of BP 337(Local Government Code) provides that the City Engineer shall (c) Prevent the encroachment of private buildings and fences on the streets and public places, (j) nspect and supervise the construction, repair, removal and safety of private buildings, (k) With the previous approval of the City Mayor in each case, order the removal of materials employed in the construction or repair of any building or structures made in violation of law or ordinance, and cause buildings and structures dangerous to the public to made secure or torn down, among others. Likewise, the Charter of the City of Caloocan, RA 5502, Article VII, Section 27, paragraph g, 1 and m, grants the City Engineer similar powers. 7. Public street cannot be made subject to a lease A public street is property for public use hence outside the commerce of man (Arts. 420, 424. Civil Code). Being outside the commerce of man, it may not be the subject of lease or other contract (Villanueva et al. vs. Castaeda and Macalino, 15 SCRA 142, citing the Municipality of Cavite vs. Rojas, 30 SCRA 602; Espiritu vs. Municipal Council of Pozorrubio, 102 Phil. 869, and Muyot vs. De la Fuente, 48 O.G. 4860). The disputed areas from which the market stalls are sought to be evicted are public streets, as found by the trial court in Civil Case C-12921. 8. Lease or licenses null and void for being contrary to law As the stallholders pay fees to the City Government for the right to occupy portions of the public street, the City Government, contrary to law, has been leasing portions of the streets to them. Such leases or licenses are null and void for being contrary to law. The right of the public to use the city streets may not be bargained away through contract. The interests of a few should not prevail over the good of the greater number in the community whose health, peace. safety, good order and general welfare, the respondent city officials are under legal obligation to protect. The Executive Order issued by the Acting Mayor authorizing the use of Heroes del '96 Street as a vending area for stallholders who were granted licenses by the city government contravenes the general law that reserves city streets and roads for public use. The Executive Order may not infringe upon the vested right of the public to use city streets for the purpose they were intended to serve: i.e., as arteries of travel for vehicles and pedestrians. Vda. De Tantoco v. Muncipal Council of Iloilo [G.R. No. 24950. March 25, 1926.] En Banc, Villamor (J): 7 concur Facts: The widow of Tan Toco sued the municipal council of Iloilo for the amount of P42,966.40, being the purchase price of two strips of land, one on Calle J. M. Basa consisting of 592 sq. m., and the other on Calle Aldiguer consisting of 59 sq. m., which the municipality of Iloilo had appropriated for widening said street. The CFI Iloilo sentenced the said municipality to pay the Tantoco the amount so claimed, plus the interest. Said judgment was appealed, and was affirmed by the Supreme Court. On account of lack of funds the municipality of Iloilo was unable to pay the said judgment, wherefore plaintiff had a writ of execution issue against the property of the said municipality, by virtue of which the sheriff attached two auto trucks used for street sprinkling, one police patrol automobile, the police stations on Mabini street, and in Molo and Mandurriao and the concrete structures, with the corresponding lots, used as markets by Iloilo, Molo, and Mandurriao. After notice of the sale of said property had been made, and a few days before the sale, the provincial fiscal of Iloilo filed a motion with the CFI praying that the attachment on the said property be dissolved, that the said attachment be declared null and void as being illegal and violative of the rights of the municipality. By order of 12 August 1925, the Court declared the attachment levied upon the aforementioned property of the municipality null and void, thereby dissolving the said attachment. From this order Tantoco has appealed by bill of exceptions. The Supreme Court affirmed the judgment appealed from with costs against Tantoco. 1. Municipalities capacity to sue and to be sued; Capacity to acquire property The municipal law, section 2165 of the Administrative Code, provides that "Municipalities are political bodies corporate, and as such are endowed with the faculties of municipal corporations, to be exercised by and through their respective municipal government in conformity with law. It shall be competent for them, in their proper corporate name, to sue and be sued, to contract and be contracted with, to acquire and hold real and personal property for municipal purposes, and generally to exercise the powers hereinafter specified or otherwise conferred upon them by law." The Administrative Code does not specify the kind of property that a municipality may acquire. 2. Property of provinces and municipalities Article 343 of the Civil Code divides the property of provinces and (municipalities) into property for public use and patrimonial property. According to article 344 of the Code, provincial roads and foot-path, squares, streets, fountains, and public waters, drives and public improvements of general benefit built at the expense of the said towns or provinces, are property for public use. All other property possessed by the said towns and provinces is patrimonial and shall be subject to the provision of the Civil Code except as provided by special laws. 3. Distinction according to Manresa; of little practical importance in view of different principles under American Rule Commenting upon article 344, Manresa says that "In accordance with administrative legislation" (Spanish) we must distinguish, as to the patrimonial property of the towns, "between that of common benefit and that which is private property of the town. The first differs from property for public use in that generally its enjoyment is less, as it is limited to neighbors or to a group or class thereof; and furthermore, such use, more or less general, is not intrinsic with this kind of property, for by its very nature it may be enjoyed as though it were private property. The third group, that is, private property, is used in the name of the town or province by the entities representing it and, like any private property, giving a source of revenue." Such distinction, however, is of little

practical importance in this jurisdiction in view of the different principles underlying the functions of a municipality under the American rule. 4. Property of public domain applies to municipal property for public use; both not within the commerce of man The principle governing property of the public domain of the State is applicable to property for public use of the municipalities as said municipal property is similar in character. The principle is that the property for public use of the State is not within the commerce of man and, consequently, is unalienable and not subject to prescription. Likewise, property for public use of the municipality is not within the commerce of man so long as it is used by the public and, consequently, said property is also inalienable. 5. US Law and Jurisprudence: Municipal properties for public use, but not properties for quasi-private purposes, exempted from attachment and execution As expounded by McQuillin in Municipal Corporations (Vol. 3, par.1160): State statutes often provide that court houses, jails other buildings owned by municipalities and the lots on which they stand shall be exempt from attachment and execution. But independent of express statutory exemption, as a general proposition, property, real and personal, held by municipal corporations, in trust for the benefit of their inhabitants, and used for public purposes, is exempt. Public buildings, school houses, streets, squares, parks, wharves, engines and engine houses, and the like, are not subject to execution. So city waterworks, and a stock of liquors carried in a town dispensary, are exempt. The reason for the exemption is obvious. Municipal corporations are created for public purposes and for the good of the citizens in their aggregate or public capacity. That they may properly discharge such public functions corporate property and revenues are essential, and to deny them these means the very purpose of their creation would be materially impeded, and in some instances practically destroy it. There is something very repugnant to the moral sense in the idea that a municipal corporation should contract debts, and that having no resources but the taxes which are due to it these should not be subjected by legal process to the satisfaction of its creditors. This consideration, deduced from the principles of moral equity has only given way to the more enlarged contemplation of the great and paramount interests of public order and the principles of government. Thus, tt is generally held that property owned by a municipality, where not used for a public purpose but for quasi private purposes, is subject to execution on a judgment against the municipality, and may be sold. This rule applies to shares of stock owned by a municipal corporation and the like. But the mere fact that corporate property held for public uses is being temporarily used for private purposes does not make it subject to execution. As stated in Corpus Juris (Vol. 23, p. 355), the rule is that property held for public uses, such as public buildings, streets, squares, parks, promenades, wharves landing places, fire engines, hose and hose carriages. engine houses, public markets, hospitals, cemeteries, and generally everything held for governmental purposes, is not subject to levy and sale under execution against such corporation. The rule also applies to funds in the hands of a public officer. Likewise it has been held that taxes due to a municipal corporation or county cannot be seized under execution by a creditor of such corporation. But where a municipal corporation or county owns in its proprietary, as distinguished from its public or governmental capacity, property not useful or used for a public purpose but for quasi private purposes, the general rule is that such property may be seized and sold under execution against the corporation, precisely as similar property of individuals is seized and sold. But property held for public purposes is not subject to execution merely because it is temporarily used for private purposes, although if the public use is wholly abandoned it becomes subject to execution. Whether or not property held as public property is necessary for the public use is a political, rather than a judicial question. Where property of a municipal or other public corporation is sought to be subjected to execution to satisfy judgments recovered against such corporation, the question as to whether such property is leviable or not is to be determined by the usage and purposes for which it is held." 6. US Law and Jurisprudence: Insurance money derived from destroyed municipal property exempt from execution likewise exempted If municipal property exempt from execution is destroyed, the insurance money stands in lieu thereof and is also exempt (McQuillin). 7. US Law and Jurisprudence: Members or inhabitants not personally liable for debts of the municipality The members or inhabitants of a municipal corporation proper are not personally liable for the debts of the municipality, except that in the New England States the individual liability of the inhabitant is generally maintained (McQuillin). 8. US Jurisprudence: Wharf a property for public use and not subject to attachment In City of New Orleans vs. Louisiana Construction Co., Ltd. (140 U. S., 654; 35 Law. ed., 556), it was held that a wharf for unloading sugar and molasses, open to the public, was property for the public use of the City of New Orleans and was not subject to attachment for the payment of the debts of the said city. In Klein vs. City of New Orleans (98 U S., 149; 25 Law. ed., 430), it was held that a public wharf on the banks of the Mississippi River was public property and not subject to execution for the payment of a debt of the City of New Orleans where said wharf was located. It was held that land was public property as necessary as a public street and was not subject to execution on account of the debts of the city. It was further held that the fees collected were also exempt from execution because they were a part of the income of the city. 9. Tufexis v. Olaguera; Special concession of the right to usufruct in a public market cannot be attached like any ordinary right It was held in Tufexis v. Olaguera, where the public market had been levied upon by virtue of the execution arising from the debt of the municipality of Guinobatan, that even though a creditor is unquestionably entitled to recover out of his debtor's property, yet when among such property there is included the special right granted by the Government of usufruct in a building intended for a public service, and when this privilege is closely related to a service of a public character, such right of the creditor to the collection of a debt owed him by the debtor who enjoys the said special privilege of usufruct in a public market is not absolute and may be exercised only through the action of a court of justice with respect to the profits or revenue obtained under the special right of usufruct enjoyed by debtor. The special concession of the right to usufruct in a public market cannot be attached like any ordinary right, because that would be to permit a person who has contracted with the state or with the administrative officials thereof to conduct and manage a service of a public character, to be substituted, without the knowledge and consent of the administrative authorities, by one who took no part in the contract, thus giving rise to the possibility of the regular course of a public service being disturbed by the more or less legal action of a grantee, to the prejudice of the state and the public interests.

The privilege or franchise granted to a private person to enjoy the usufruct of a public market cannot lawfully be attached and sold, and a creditor of such person can recover his debt only out of the income or revenue obtained by the debtor from the enjoyment or usufruct of the said privilege, in the same manner that the rights of the creditors of a railroad company can be exercised and their creditors collected only out of the gross receipts remaining after deduction has been made therefrom of the operating expenses of the road. 10. Character of property for public use basis why property of a municipality necessary for governmental purposes may not be attached The movable and immovable property of a municipality, necessary for governmental purposes, may not be attached and sold for the payment of a judgment against the municipality. The supreme reason for this rule is the character of the public use to which such kind of property is devoted. The necessity for government service justifies that the property of public use of the municipality be exempt from execution just as it is necessary to exempt certain property of private individuals in accordance with section 452 of the Code of Civil Procedure. 11. Municipal income exempt from levy and execution Even the municipal income is exempt from levy and execution. In Municipal Corporations by Dillon (Vol. 1, p. 467), it was stated that "municipal corporations are instituted by the supreme authority of a state for the public good. They exercise, by delegation from the legislature, a portion of the sovereign power. The main object of their creation is to act as administrative agencies for the state, and to provide for the police and local government of certain designated civil divisions of its territory. To this end they are invested with certain governmental powers and charged with civil, political, and municipal duties. To enable them beneficially to exercise these powers and discharge these duties, they are clothed with the authority to raise revenues, chiefly by taxation, and subordinately by other modes, as by licenses, fines, and penalties. The revenue of the public corporation is the essential means by which it is enabled to perform its appointed work. Deprived of its regular and adequated supply of revenue, such a corporation is practically destroyed, and the ends of its erection thwarted. Based upon considerations of this character, it is the settled doctrine of the law that not only the public-property but also the taxes and public revenues of such corporations cannot be seized under execution against them, either in the treasury or when in transit to it. Judgments rendered for taxes, and the proceeds of such judgments in the hands of officers of the law, are not subject to execution unless so declared by statute. The doctrine of the inviolability of the public revenues by the creditor is maintained, although the corporation is in debt, and has no means of payment but the taxes which it is authorized to collect." Jacinto v. Director of Lands [G.R. No. 26374. December 31, 1926.] En Banc, Ostrand (J): 7 concur Facts: During the period from 1911 to 1913, sales certificates were issued by the Bureau of Lands to Frank W. Carpenter for more than 100 lots of the Tala and Piedad Friar Lands states located in Novaliches, Caloocan, Rizal including the lots 670, 690, 691, 695, 696, 697 698, 699, 700, 701, 950, 951, 952, 953, 954, 955, 956, 957, and 1050. The total area of the land covered by the sales certificates being over 1,490 hectares and the purchase price amounting to about P56,600, of which amount Carpenter up to the year 1923, had paid in installments the sum of P16,272. Under a judgment rendered against Carpenter in the CFI of Manila (Civil Case 24607), execution was levied upon all of his right, title and interest in the lots purchased together with the improvements thereon, and on 16 November 1923, the sheriff of Rizal sold the property to Nicanor Jacinto. The sheriff's sale was registered in the Bureau of Lands, assignments of the Bureau of Lands' sales certificates were duly recorded, and certificates of assignment were issued and delivered to Nicanor Jacinto in September 1924. On 31 March 1925, the Metropolitan Water District instituted proceedings in the CFI Rizal for the condemnation of certain parcels of land situated in the municipality of Caloocan for the construction of an earth dam and a first-class highway 3 kilometers long, in connection with the so-called Angat Water Works Project, and on the same date the CFI Rizal issued an order authorizing the Metropolitan Water District to take possession of said parcels of land upon deposit with the provincial treasurer of the sum of P3,000 as the provisional value, fixed by the court, of the parcels so to be condemned. By virtue of this order, the Metropolitan Water District entered into occupation of the land and began the construction of permanent improvements thereon. Copies of the complaint as well as of the order of 31 March 1925, were filed with the register of deeds of the Province of Rizal on 11 February 1926, to be recorded as notices of lis pendens. The lots enumerated above were included in the land sought to be expropriated and Nicanor Jacinto was made a party defendant in the proceedings. He admitted the existence of the right of condemnation and the necessity for the expropriation, but demanded the sum of P64,839.33 as indemnity for the expropriation. As the actual purchase price to be paid by the purchaser from the Government only amounts to P13,725, including interest, the Metropolitan Water District considered Jacinto's demand excessive and declined to pay the claim. In the month of July 1926, the applicant tendered payment to the Director of Lands of the sum of P4,650 to cover the remaining balance of the sales price of the lots in question and demanded a corresponding deed of conveyance for said lots. The Director of Lands, upon the advice of the Attorney-General, rejected the tender and refused to execute and deliver the instrument of conveyance demanded from him. Applicant filed a petition for a writ of mandamus to compel the Director of Lands to execute a deed of conveyance in favor of the applicant for the lots enumerated belonging to the Tala Friar Lands Estate in Novaliches. The Supreme Court ordered the Director of Lands to receive the balance of the purchase money for any or all of the lots in question if and when payment thereof is tendered by Jacinto, and denied the petition as to the execution of deeds of conveyance; without costs. 1. Proprietary rights, except that of occupation, not affected by condemnation proceedings The proprietary rights, except the right of occupation, are not affected by the condemnation proceedings until the title has passed to the plaintiff and that does not occur until the award of compensation or damages has been satisfied.

2. Petition for a writ of mandamus not proper remedy to compel a conveyance Mandamus is not the proper remedy to enforce purely contract rights, such as that in the present case sought to be enforced. (18 R. C. L., 121; Quiogue vs. Romualdez, 46 Phil., 337.) 3. Land is patrimonial property of the Government; Duty to execute deeds of conveyance devolved upon the GovernorGeneral The writ cannot issue in the present case unless it appears that the Director of Lands "unlawfully neglects the performance of an act which the law specially enjoins as a duty resulting from an office, trust, or station." (Section 222, Code of Civil Procedure.) The land in question is private or patrimonial property of the Philippine Government and we can find no law specially enjoining upon the Director of Lands the duty to execute deeds of conveyance to purchasers of such lands; on the contrary, that duty, under section 567 of the Administrative Code, appears to devolve upon the Governor-General. 4. Director of Land has duty to receive purchase money payable under Act 1120 By section 14 of Act No. 1120 the Director of Lands is charged with the duty of receiving the purchase money payable under that Act and may therefore be compelled by mandamus to receive, as a purely ministerial act, such purchase money when tendered. Municipality of Oas v. Roa [G.R. No. L-2017. November 24, 1906.] First Division, Willard (J): 3 concur Facts: The Municipality brought the action for the recovery of a tract of land in the pueblo of Oas, claiming that it was a part of the public square of said town, while the Roa alleged that he was the owner of the property. The municipality claims ownership in view of a document (minutes of the 27 February 1892 meeting of the principalia of the town) stating that the land was bought in 1832 by the towns parish priest; that erection of houses in said land is prohibited by order of the corregidor of Nueva Caceres; and that the repair of the building was prohibited upon the owner thereof, Jose Castillo. Said document was signed by Roa himself. When the building on the land was destroyed by typhoon on 13-14 May 1893, authorities of the town ordered the demolition thereof and declared that the owner of the building, Jose Castillo, had no right to reconstruct said building as the land did not belong to him. This resolution was also signed by Roa. Roa, on the other hand, claims that Juana Ricarte and Juana Riquiza sold the land to Juan Roco in 1876, and that on 17 December 1894, Jose Castillo sold it to Roa. No deed of conveyance from Juan Roco to Jose Castillo was presented by evidence, but with Castillo testifying that he bought the property by verbal agreement with Roco, his father-in-law. Roa, after his purchase in 1894, procured a possessory of information which was allowed by an order of the justice of the peace of Oas on the 19 January 1895, and recorded in the Registry of Property on the 28 March of the same year. As early as 1852, the Municipality has constructed thereon buildings for the storage of property of the State, quarters for the cuadrilleros, and others of a like character. Roa was also able to construct a substantial building on the said after he acquired the property from Castillo. No case facts providing details of how the case was filed by lower court, nor the actual judgment of said court The Supreme Court modified the judgment appealed from and declared the Municipality is the owner of the land and that it has the option of buying the building thereon, which is the property of the defendant, or of selling to him the land on which it stands. The Court also declared Municipality is entitled to recover the costs of both instances, and ordered the judgment entered after the expiration of 20 days and the records of the case remanded to the lower court for proper action. 1. Proof should be manifestly against decision for the factual finding to be reversed In this state of the evidence, we can not say that the proof is plainly and manifestly against the decision of the lower court. Unless it is so, the finding of fact made by that court can not be reversed. (De la Rama vs. De la Rama, 201 U. S., 303.) 2. Statements signed by Roa competent, but not conclusive, evidence against him; Roa not estopped The two statements signed by Roa, one in 1892 and the other in 1893, are competent evidence against him. They are admissions by him to the effect that at that time the pueblo was the owner of the property in question. They are, of course, not conclusive against him. He was entitled to, and did present evidence to overcome the effect of these admissions. The evidence does not make out a case of estoppel against him. (sec. 333, par. 1, Code of Civil Procedure.) The admissibility of these statements made by Roa do not rest upon section 278 of the Code of Civil Procedure, which relates to declarations or admissions made by persons not a party to the suit, but it rests upon the principle that when the defendant in a suit has himself made an admission of any fact pertinent to issue involved, it can be received against him. 3. Purchaser in bad faith; Prescription requirement of 30 years not met The present action was commenced on the 17 December 1902. There is no evidence of any adverse occupation of this land for 30 years, consequently the extraordinary period of prescription does not apply. The defendant can not rely upon the ordinary period of prescription of 10 years because he was not a holder in good faith. He knew at that time of his purchase in 1894, and had so stated in writing, that the pueblo was the owner of the property. So that, even if the statute of limitations ran against a municipality in reference to a public square, it could not avail the defendant in the present case. 4. Property is a patrimonial property of the State As early as 1852, the land had been used by the municipality constructed thereon buildings for the storage of property of the State, quarters for the cuadrilleros, and others of a like character. It therefore had ceased to be property used by the public and had become a part of the bienes patrimoniales of the pueblo. (Civil Code, arts. 341, 344.) 5. Both Roa and the Municipality in bad faith; therefore, rights determined as if both are in good faith Roa constructed the building in bad faith for he had knowledge of the fact that his grantor was not the owner thereof. There was a

bad faith also on the part of the Municpality in accordance with the express provisions of article 364 since it allowed Roa to construct the building without any opposition on its part and to so occupy it for 8 years. The rights of the parties must, therefore, be determined as if they both had acted in good faith. Article 364 of the Civil Code provides that where there has been bad faith, not only on the part of the person who built, sowed, or planted on another's land, but also on the part of the owner of the latter, the rights of both shall be the same as if they had acted in good faith. Bad faith on the part of the owner is understood whenever the act has been executed in his presence with his knowledge and tolerance and without objection. Article 361 of the Civil Code provides that "the owner of the land on which the building, sowing, or planting is done in good faith shall have a right to appropriate as his own the work, sowing, or planting after the indemnity mentioned in articles 453 and 454, or, to oblige the person who has built or planted, to pay him the value of the land and to force the person who sowed to pay the proper rent." Municipality of Hinunangan v. Director of Lands [G.R. No. 7054. January 20, 1913.] First Division, Moreland (J): 5 concur Facts: Land in question is situated in Hinunangan, Leyte, and contains an area of 10, 328.8 sq. m. it is bounded on the northwest by the maritime zone; on the southeast by North America Street; on the southwest by Manalili Street, and on the northwest by San Isidro Labrador Street. Upon this lot is built a stone fort which has stood there from time immemorial and was in times past used as a defense against the invasion of the Moros. Petitioners applied for the registration of the title to the lands so described. The Court of Land Registration ordered said registration in favor of the petitioners. The Insular Government appealed as to the registration of the title of one of the parcels of land only. The Supreme Court reversed the judgment in relation to the parcel of land described, and dismissing the petition as to that parcel of land; but affirming the judgment in all other respects. 1. Defense of national territory rests upon the state and not upon towns and villages The defense of the national territory against invasion by foreign enemies rested upon the state and not upon the towns and villages and for this reason all of the defenses were constructed by the National Government. 2. Fortresses and its land property of the State; Legal anchors In volume 2, book 3, title 7, law 1 of the Laws of the Indies, it was stated that we command that all the ground roundabout the castles and fortresses be clear and unoccupied, and if any building is erected within 300 paces of the wall or other building so strong that ever at a greater distance it would prejudice the defenses, it shall be torn down, and the owner of the same shall be paid from the Royal Treasury for the damages caused him." Book 4, title 7, law 12, reads as We order that, for the security and defense of the cities as is now assured by the castles and fortresses, no building shall be erected within 300 paces of the walls or stockades of the new cities." Article 399 of the Civil Code, in part, provides that that which belongs privately to the state, which is not for public use and which is destined for the public good or to increase the national riches, such as walls, fortresses and other constructions for the defense of the country, and the mines as long as no concession in regard to them is made" is public property. Article 341 of the Civil Code provides that public property, when it ceases to be used for the public use and which is destined for the necessities of the defense of the country, becomes a part of the property of the state." It is clear thus that the fortress in question was erected for the national defense and was a part of the property of the state destined and used for that purpose; and as a result, the land which it stands on is also dedicated to that purpose. 3. Fortress not being in use does not deprive the state of its ownership The fact that said fortress may not have been used for many years for the purposes for which it was originally built does not of necessity deprive the state of its ownership therein. The Civil Code provides that, when the fortress ceases to be used for the purpose for which it was constructed, it becomes the property of the state in what may be called the private sense. 4. Presumption of grant by the state to municipality may be invoked only is property is used distinctly for public purposes The Court has ruled where the municipality has occupied lands distinctly for public purposes (such as for the municipal court house, the public school, the public market, or other necessary municipal building) and in the absence of proof to the contrary, presume a grant from the state in favor of the municipality. Still, the rule may be invoked only as to property which is used distinctly for public purposes; and cannot be applied against the state when occupied for any other purpose. In the present case, the evidence does not disclose that the municipality has used the land for purposes distinctly public, but has that it has exercised acts of ownership over the land by permitting it to be occupied and consenting to the erection of private houses thereon. Cebu Oxygen & Acetylene v. Bercilles [G.R. No. L-40474. August 29, 1975.] Second Division, Concepcion Jr. (J): 4 concur Facts: The parcel of land sought to be registered was originally a portion of M. Borces Street, Mabolo, Cebu City. On 23 September 1968, the City Council of Cebu, through Resolution 2193 (3 October 1968), declared the terminal portion of M. Borces Street, Mabolo, Cebu City, as an abandoned road, the same not being included in the City Development Plan. Subsequently, on 19 December 1968, the City Council of Cebu passed Resolution 2755, authorizing the Acting City Mayor to sell the land through a public bidding. Pursuant thereto, the lot was awarded to the herein petitioner being the highest bidder and on 3 March 1969, the City of Cebu, through the Acting City Mayor, executed a deed of absolute sale to the petitioner for a total consideration of P10,800.00. By virtue of the aforesaid deed of absolute sale, the petitioner filed an application with the CFI Cebu to have its title to the land registered (LRC N-948; LRC Record N-44531). On 26 June 1974, the Assistant Provincial Fiscal of Cebu filed a motion to dismiss the application on the ground that the property sought to be registered being a public road intended for public use is considered part of the public domain and therefore outside the commerce of man. After hearing the parties, on 11 October 1914 the trial court issued an order dismissing the petitioner's application for registration of title. Hence, the instant petition for review.

The Supreme Court set aside the order of the lower court, and the ordered said court to proceed with the hearing of the petitioner's application for registration of title. 1. City is empowered to close city road or street and withdraw the same from public use Section 31 of the Revised Charter of Cebu City (Legislative Powers) provides that any provision of law and executive order to the contrary notwithstanding, the City Council shall have the following legislative powers xxx to close any city road, street or alley, boulevard, avenue, park or square. Property thus withdrawn from public servitude may be used or conveyed for any purpose for which other real property belonging to the City may be lawfully used or conveyed." It is undoubtedly clear that the City of Cebu is empowered to close a city road or street. 2. Discretion of the city council cannot ordinarily be interfered with by the court The city council is the authority competent to determine whether or not a certain property is still necessary for public use. The power to vacate a street or alley is discretionary, and the discretion will not ordinarily be controlled or interfered with by the courts, absent a plain case of abuse or fraud or collusion. Faithfulness to the public trust will he presumed. So the fact that some private interests may be served incidentally will not invalidate the vacation ordinance (Favis v. City of Baguio). 3. Street withdrawn from public use becomes patrimonial property; Subsequent sale valid When a portion of the city street was withdrawn from public use, such withdrawn portion becomes patrimonial property which can be the object of an ordinary contract. As expressly provided by Article 422 of the Civil Code, "property of public dominion, when no longer intended for public use or for public service, shall form part of the patrimonial property of the State." Further, the Revised Charter of the City of Cebu, in very clear and unequivocal terms, states that "property thus withdrawn from public servitude may be used or conveyed for any purpose for which other real property belonging to the City may be lawfully used or conveyed." Thus, the withdrawal of the property in question from public use and its subsequent sale to the petitioner is valid. Harty v. Victoria, Tarlac [G.R. No. 5013. March 11, 1909.] En Banc, Torres (J): 5 concur Facts: On 17 January 1908, the representative of Monsignor Jeremiah J. Harty, archbishop of the Roman Catholic Church, as the legal administrator of the properties and rights of the Catholic Church within the archbishopric of Manila, filed a written complaint in the CFI Tarlac against the municipality of Victoria, alleging that the parish of the said town had been and was then the owner of a parcel of land within the said municipality, known as the plaza of the church of Victoria; that it had acquired said parcel of land more than 60 years previously, and had continued to possess the same ever since up to 1901, in which year the municipality unlawfully and forcibly seized the said property, claiming to be entitled thereto and retaining it to the present day. On 15 June 1908, the trial court rendered judgment, holding that the parish of Victoria of the Roman Catholic Apostolic Church, had a better right to the possession of the land described in the complaint, and sentenced the Municipality to vacate the same and to pay the costs. To said judgment the representative of the Municipality excepted and moved for a new trial on the ground that it was contrary to the weight of the evidence, and he notified the court that, if his motion were overruled, he would appeal to the Supreme Court. The motion for a new trial was overruled; the Municipality excepted, and presented the corresponding bill of exceptions which, after receipt of a copy had been acknowledged by the adverse party, was approved. On 1 September, the Municipality was ordered to furnish bond in the sum of P1,000 to insure the fulfillment of the judgment in the event that it should be totally or partially affirmed. To said order the Municipality excepted, but furnished the bond as directed by the court. The Supreme Court reversed the judgment appealed from, and held that the whole of the land not occupied by the church of the town of Victoria and its parish house, is a public plaza of the said town, of public use, and that in consequence thereof, the Municipality is absolved of the complaint without any special ruling as to the costs of both instances. 1. Property of public ownership Article 339 of the Civil Code provides that "property of public ownership is (1) that destined to the public use, such as roads, canals, rivers, torrents, ports, and bridges constructed by the State, and banks, shores, roadsteads, and that of a similar character." Further, Article 344 of said code provides that "property for public use in provinces and in towns comprises the provincial and town roads, the squares, streets, fountains, and public waters, the promenades, and public works of general service supported by the said towns or provinces." 2. History of the municipality of Victoria, Tarlac; Customs in creation of new town under the old Laws of the Indies The town of Victoria was formerly only a barrio of the town of Tarlac and known as Canarum. It was converted into a town in 1855, and named Victoria. To this end they must have laid out the streets and the plaza of the town, in the center of which were situated the church and parish house from the commencement, and at the expiration of about 12 years the parish of said town was constituted and the priest who was to perform the office of curate was appointed; that from the very beginning, the large tract of land that surrounds the church and the parish house was known as a public plaza, destined to the use of all the residents of the recently founded town; public performances and religious processions were held thereon without hindrance either on the part of the local authorities or of the curate of said town. Further, it was a custom observed by all the towns established administratively in these Islands under the old Laws of the Indies, that on their creation, a certain amount of land was always reserved for plazas, commons, and special and communal property, and as it is unquestionable that the said large space of land was left vacant in the center of the town of Victoria when it was constituted as a civil town. 3. The late Tanedo donated the land occupied by the Church to the church and not to the parish curate There are good grounds to suppose that the late Vicente Tanedo donated the land now occupied by the church and parish house in said municipality for religious purposes, or to the church, but not to the parish curate, because at the time there was no curate at the new town of Victoria.

4. Proof lacking if the land Tanedo donated include the whole large tract constituting the town plaza; Waiver of rights thereon in favor of the public presumed It may be true that the father of the witness Casimiro Taedo, who owned the space of land where the church and parish house were erected, had voluntarily donated it to the Catholic Church but proper proof is lacking that the donation affirmed by the said Tanedo comprehended the whole of the large tract which at the present time constitutes the plaza of the town. Even though all the remaining space of land which now forms the great plaza of the town of Victoria had been owned by the said Tanedo, it must be presumed that he waived his right thereto for the benefit of the townspeople, since all the residents have enjoyed the free use of said plaza. It has not been satisfactorily shown that the municipality or the principales of the town of Victoria had donated the whole of said land to the curate of Victoria or to the Catholic Church, nor could it have been so donated, it being a public plaza destined to public use and was not private ownership, or patrimony of the town of Victoria, or of the Province of Tarlac. Certain it is that the Curate has not proven that the Catholic Church or the parish of Victoria was the owner or proprietor of the said extensive piece of land which now forms the public plaza of said town, nor that it was in possession thereof under the form and conditions required by law, inasmuch as it has been fully proven that said plaza has been used without let or hindrance by the public and the residents of the town of Victoria ever since its creation. 5. Plazas destined for public use not subject to prescription Pursuant to Article 1936 of the Civil Code, plazas, among other things, destined to the public use are not subject to prescription. 6. Procured trees set out in the plaza does not constitute an act of private ownership That both the curates and the gobernadorcillos of the said town procured fruit trees and plants to be set out in the plaza, does not constitute an act of private ownership, but evidences the public use thereof, or perhaps the intention to improve the and embellish the said plaza for the benefit of the townspeople. Antipolo v. Zapanta [G.R. No. 65334. December 26, 1984.] First Division, Melencio-Herrera (J): 4 concur, 1 took no part. Facts: The Municipality of ANTIPOLO, for more than 50 years now, has considered the disputed property, described below, to be public land subject to ANTIPOLO's use and permission to use within the prerogatives and purposes of a municipal corporation. There is indication to the effect that it had been the site of the public market as far back as 1908, or at the latest, since 1920 "up to today." Gradually, additional public structures were built thereon, like the Puericulture and Family Planning Center, the Integrated National Police Building, the Office of the Municipal Treasurer, and the public abattoir. Those public structures occupy almost the entire area of the land. On 8 August 1977, a single application for the registration of two distinct parcels of land was filed by two distinct applicants before the then CFI Rizal, Branch XV, Makati (the Registration Court). One of the two applicants was Conrado Eniceo. He had applied for registration under the Torrens system of a parcel of land containing 258 sq. m. The other applicant was "Heirs of Joaquin Avendao", and the land they were applying for registration was a parcel containing 9,826 sq. m. (the disputed property) surveyed in the name of the Municipality of Antipolo. Both parcels were situated in the Municipality of Antipolo. The application were approved by the Registration Court on 26 February 1980. ANTIPOLO took steps to interpose an appeal but because it failed to amend the Record on Appeal, its appeal was disallowed. On 22 May 1981, ANTIPOLO filed a complaint (Civil Case 41353) of the CFI Rizal, Branch XIII, Pasig against named "Heirs of Joaquin Avendao", and their assignees praying for nullification of the judgment rendered by the Registration Court. The defendants, in their Answer, pleaded a special defense of res judicata. After a preliminary hearing on the mentioned special defense, the case was dismissed. ANTIPOLO perfected an appeal to the then Court of Appeals. A notice to file Brief was issued by the Appellate Court, which ANTIPOLO claimed it had not received. Upon motion of the Avendano heirs to dismiss on the ground the ANTIPOLO had not filed its Brief within the reglementary period, the appeal was dismissed on 23 August 1983 despite the fact that before the dismissal, ANTIPOLO had submitted its Appellant's Brief. ANTIPOLO filed a motion for reconsideration, which the Appellate Court denied on 27 September 1983 for lack of legal and factual basis. Hence, the petition for review on certiorari. The Supreme Court (1) set aside the resolutions of the appellate court (now IAC) dated 23 August 1983 and 27 September 1983; (2) set aside the judgment of the CFI Rizal in Civil Case 41353 and rendered the judgment and decree of the CFI Rizal in LRC N9995, LRC Record N-52176 null and void in respect of the Heirs of Joaquin Avendano; (3) ordered the Register of Deed of Rizal to cancel all certificates issued by virtue of decree issued in LRC N-9995, LRC Rec. N-52176 in respect of the Heirs of Isabela Avendano; and (4) declared the certificate of title issuied in the name of Conrado Eniceo, and transfers therefrom as decreed in LRC N-995, LRC Rec. N-52176 to continue to be valid; without pronouncements as to costs. 1. Technicality yields to broader interests of substantial justice; When jurisdiction is questioned Although failure to file Brief within the time provided by the Rules is, indeed, a ground for dismissal of an appeal, this Court had held that rules of technicality must yield to the broader interests of substantial justice specially where, as in this case, the important issue of lack of jurisdiction over the subject matter of the Land Registration Court has been raised. 2. Remand is ordinarily the appropriate relief; Court resolves merits due to 3 motions for early decision filed A remand to the lower Court, for the entertainment of the appeal on the merits, would ordinarily be the appropriate relief. However, considering the three Motions for Early Decision filed by private respondents, we shall resolve the substantive merits of the appeal to the appellate tribunal from the judgment rendered in the case. 3. Disputed property devoted to public use and public service; Possession does not presuppose ownership; Lands presumed to be public lands unless contrary is proven; Tax declaration mere indicia of a claim to ownership At the time the application for registration was filed on 8 August 1977, the disputed property was already devoted to public use

and public service. Therefore, it was outside the commerce of man and could no longer be subject to private registration. The claim of the Avendano heirs that they merely tolerated occupancy by ANTIPOLO which had borrowed the disputed property from them, since they had been in possession, since as far back as 1916, erroneously presupposes ownership thereof since that time. They forget that all lands are presumed to be public lands until the contrary is established. The fact that the disputed property may have been declared for taxation purposes in their names or of their predecessors-in-interest as early as 1918 does not necessarily prove ownership. They are merely indicia of a claim of ownership. ANTIPOLO had also declared the disputed property as its own in Tax Declarations 909, 993 and 454. 4. Res judicata does not apply since Land Registration Court had no jurisdiction to entertain the registration of public property Since the Land Registration Court had no jurisdiction to entertain the application for registration of public property of ANTIPOLO, its Decision adjudicating the disputed property as of private ownership is null and void. It never attained finality, and can be attacked at any time. It was not a bar to the action brought by ANTIPOLO for its annulment by reason of res judicata. 5. Effects of decision which is null and void by virtue of lack of jurisdiction The want of jurisdiction by a court over the subject-matter renders the judgment void and a mere nullity, and considering that a void judgment is in legal effect no judgment, by which no rights are divested, from which no rights can be obtained, which neither binds nor bars any one, and under which all acts performed and all claims flowing out of are void, and considering, further, that the decision, for want of jurisdiction of the court, is not a decision in contemplation of law, and hence, can never become executory, it follows that such a void judgment cannot constitute a bar to another case by reason of res judicata. 6. Titles null and void; Cancellation pursued through ordinary action When the titles issued by a Court, or by virtue of the decision or decree of the court, with no jurisdiction over the subject matter; such are to be held to be null and void and perforce, they must be ordered cancelled. It follows that 'if a person obtains a title under the Public Land Act which includes, by oversight, lands which cannot be registered under the Torrens System, or when the Director of Lands did not have jurisdiction over the same because it is a public forest, the grantee does not, by virtue of the said certificate of title alone, become the owner of the land illegally included' (Republic vs. Animas, 56 SCRA 499, 503; Ledesma vs. Municipality of Iloilo, 49 Phi. 769). Under these circumstances, the certificate of title may be ordered cancelled and the cancellation may be pursued through an ordinary action therefor. This action cannot be barred by the prior judgment of the land registration court, since the said court had no jurisdiction over the subject matter. And if there was no such jurisdiction, then the principle of res judicata does not apply. Macasiano v. Diokno [G.R. No. 97764. August 10, 1992.] En Banc, Medialdea (J): 12 concur Facts: On 13 June 1990, the Municipality of Paranaque passed Ordinance 86, s. 1990 which authorized the closure of J. Gabrielle, G.G. Cruz, Bayanihan, Lt. Garcia Extension and Opena Streets located at Baclaran, Paraaque, Metro Manila and the establishment of a flea market thereon. The said ordinance was approved by the municipal council pursuant to MCC Ordinance 2, s. 1979, authorizing and regulating the use of certain city and/or municipal streets, roads and open spaces within Metropolitan Manila as sites for flea market and/or vending areas, under certain terms and conditions. On 20 July 1990, the Metropolitan Manila Authority approved Ordinance 86, s. 1990 of the municipal council subject to conditions. On 20 June 1990, the municipal council issued a resolution authorizing the Paraaque Mayor to enter into contract with any service cooperative for the establishment, operation, maintenance and management of flea markets and/or vending areas. On 8 August 1990, the municipality and Palanyag, a service cooperative, entered into an agreement whereby the latter shall operate, maintain and manage the flea market with the obligation to remit dues to the treasury of the municipal government of Paraaque. Consequently, market stalls were put up by Palanyag on the said streets. On 13 September 1990 Brig. Gen. Macasiano, PNP Superintendent of the Metropolitan Traffic Command, ordered the destruction and confiscation of stalls along G.G. Cruz and J. Gabrielle St. in Baclaran. These stalls were later returned to Palanyag. On 16 October 1990, Macasiano wrote a letter to Palanyag giving the latter 10 days to discontinue the flea market; otherwise, the market stalls shall be dismantled. On 23 October 1990, the municipality and Palanyag filed with the trial court a joint petition for prohibition and mandamus with damages and prayer for preliminary injunction. On 17 December 1990, the trial court issued an order upholding the validity of Ordinance 86 s. 1990 of the Municipality of Paraaque and enjoining Macasiano from enforcing his letter-order against Palanyag. Hence, a petition for certiorari under Rule 65 was filed by Macasiano thru the OSG. The Supreme Court granted the petition, and reversed and set aside the 17 December 1990 decision of the RTC which granted the writ of preliminary injunction enjoining the PNP Superintendent, Metropolitan Traffic Command from enforcing the demolition of market stalls along J. Gabrielle, G.G. Cruz, Bayanihan, Lt. Garcia Extension and Opena streets. 1. Property of provinces, cities and municipalities; Property for public use The property of provinces, cities and municipalities is divided into property for public use and patrimonial property (Art. 423, Civil Code). As to property for public use, Article 424 of Civil Code provides that "property for public use, in the provinces, cities and municipalities, consists of the provincial roads, city streets, the squares, fountains, public waters, promenades, and public works for public service paid for by said provinces, cities or municipalities. All other property possessed by any of them is patrimonial and shall be governed by this Code, without prejudice to the provisions of special laws." In the present case, thus, J. Gabrielle G.G. Cruz, Bayanihan, Lt. Gacia Extension and Opena streets are local roads used for public service and are therefore considered public properties of the municipality. 2. Properties for public service deemed public and under absolute control of Congress Properties of the local government which are devoted to public service are deemed public and are under the absolute control of Congress (Province of Zamboanga del Norte v. City of Zamboanga, 22 SCRA 1334 [1968]).

3. Local governments have no authority to regulate use of public properties unless authority is vested upon by Congress; e.g. Closure of roads Local governments have no authority whatsoever to control or regulate the use of public properties unless specific authority is vested upon them by Congress. One such example of this authority given by Congress to the local governments is the power to close roads as provided in Section 10, Chapter II of the Local Government Code (BP 337), which states A local government unit may likewise, through its head acting pursuant to a resolution of its sangguniang and in accordance with existing law and the provisions of this Code, close any barangay, municipal, city or provincial road, street, alley, park or square. No such way or place or any part thereof shall be closed without indemnifying any person prejudiced thereby. A property thus withdrawn from public use may be used or conveyed for any purpose for which other real property belonging to the local unit concerned might be lawfully used or conveyed." 4. Legal provision should be read and interpreted in accordance with basic principles already established by law; LGU has no power to lease a road available to public and ordinarily used for vehicular traffic The legal provision (Chapter II, Section 10 of the LGC) which gives authority to local government units to close roads and other similar public places should be read and interpreted in accordance with basic principles already established by law. These basic principles have the effect of limiting such authority of the province, city or municipality to close a public street or thoroughfare. Article 424 NCC lays down the basic principle that properties of public dominion devoted to public use and made available to the public in general are outside the commerce of man and cannot be disposed of or leased by the local government unit to private persons. Aside from the requirement of due process which should be complied with before closing a road, street or park, the closure should be for the sole purpose of withdrawing the road or other public property from public use when circumstances show that such property is no longer intended or necessary for public use or public service. When it is already withdrawn from public use, the property then becomes patrimonial property of the local government unit (LGU) (Article 422 NCC; Cebu Oxygen v. Bercilles, 66 SCRA 481 [1975]). It is only then that the LGU can "use or convey them for any purpose for which other real property belonging to the local unit concerned might be lawfully used or conveyed." However, those roads and streets which are available to the public in general and ordinarily used for vehicular traffic are still considered public property devoted to public use. In such case, the LGU has no power to use it for another purpose or to dispose of or lease it to private persons. 5. Related case, Cebu Oxygen v. Bercilles In Cebu Oxygen v. Bercilles, the City Council of Cebu, through a resolution, declared the terminal road of M. Borces Street, Mabolo, Cebu City as an abandoned road, the same not being included in the City Development Plan. Thereafter, the City Council passed another resolution authorizing the sale of the said abandoned road through public bidding. The Court held that the City of Cebu is empowered to close a city street and to vacate or withdraw the same from public use. Such withdrawn portion becomes patrimonial property which can be the object of an ordinary contract 6. Related case, Dacanay v. Asistio In Dacanay v. Asistio, the disputed areas from which the market stalls are sought to be evicted are public streets. A public street is property for public use hence outside the commerce of man (Arts. 420, 424, Civil Code). Being outside the commerce of man, it may not be the subject of lease or other contract (Villanueva, et al. v. Castaeda and Macalino, 15 SCRA 142 citing the Municipality of Cavite v. Rojas, 30 SCRA 602; Espiritu v. Municipal Council of Pozorrubio, 102 Phil. 869; and Muyot v. De la Fuente, 48 O.G. 4860). The right of the public to use the city streets may not be bargained away through contract. The interests of a few should not prevail over the good of the greater number in the community whose health, peace, safety, good order and general welfare, the respondent city officials are under legal obligation to protect. The leases or licenses granted by the City Government to stallholders are null and void for being contrary to law. The Executive Order issued by the acting Mayor authorizing the use of Heroes del '96 Street as a vending area for stallholders contravenes the general law that reserves city streets and roads for public use. The Executive Order may not infringe upon the vested right of the public to use city streets for the purpose they were intended to serve: i.e., as arteries of travel for vehicles and pedestrians. 7. In gratia argumenti, ordinance cannot be validly implemented as municipality has not complied with conditions imposed by the MMA for the approval of the ordinance Even assuming, in gratia argumenti, that the municipality has the authority to pass the disputed ordinance, the same cannot be validly implemented because it cannot be considered approved by the Metropolitan Manila Authority due to non-compliance by the municipality of the conditions imposed by the former for the approval of the ordinance. The allegations of the municipality that the closed streets were not used for vehicular traffic and that the majority of the residents do not oppose the establishment of a flea market on said streets are unsupported by any evidence that will show that the first condition has been met. Likewise, the designation by the Municipality of a time schedule during which the flea market shall operate is absent (fourth condition). 8. Baclaran area congested; establishment of flea market on municipality streets does not help solve problem of congestion It is of public notice that the streets along Baclaran area are congested with people, houses and traffic brought about by the proliferation of vendors occupying the streets. To license and allow the establishment of a flea market along J. Gabrielle, G.G. Cruz, Bayanihan, Lt. Garcia Extension and Opena streets in Baclaran would not help in solving the problem of congestion but rather leads to inconvenience to children as the normal transportation flow is disrupted, to pollution and deterioration of health of residents due to the garbage left by the vendors on the streets. Further, ambulances and fire engines are not able to use the roads for a more direct access to the fire area and thus lose valuable time that should have been spent in saving properties and lives. And further, the ambulances and people rushing patients to St. Rita Hospital located along GG Cruz Street are delayed as they are unable to pass through said street due to the stalls and vendors. 9. Powers of local government unit not absolute The powers of a local government unit are not absolute. They are subject to limitations laid down by the Constitution and the laws such as our Civil Code. Moreover, the exercise of such powers should be subservient to paramount considerations of health and well-being of the members of the community. Every local government unit has the sworn obligation to enact measures that will enhance the public health, safety and convenience, maintain peace and order, and promote the general prosperity of the inhabitants of the local units. Based on this objective, the local government should refrain from acting towards that which might prejudice or adversely affect the general welfare.

10. General public has legal right to demand the restoration of city streets to their specific public purpose As in the Dacanay case, the general public have a legal right to demand the demolition of the illegally constructed stalls in public roads and streets and the officials of municipality have the corresponding duty arising from public office to clear the city streets and restore them to their specific public purpose. 11. Applicability of the Dacanay case; Contracts by Local Government governed by the original terms and conditions, and the law in force at time the rights were vested As in the Dacanay case, both cases involve an ordinance which is void and illegal for lack of basis and authority in laws applicable during its time. However, BP 337 (Local Government Code), has already been repealed by RA7160 (Local Government Code of 1991) which took effect on 1 January 1992. Section 5(d) of the new Code provides that rights and obligations existing on the date of effectivity of the new Code and arising out of contracts or any other source of prestation involving a local government unit shall be governed by the original terms and conditions of the said contracts or the law in force at the time such rights were vested. Espiritu v. Municipal Council of Pozorrubio, Pangasinan [G.R. No. L-11014. January 21, 1958.] En Banc, Montemayor (J): 10 concur Facts: During the last world war, the market building of the town of Pozorrubio was destroyed, and after Liberation, the market vendors began constructing temporary and make- shift stalls, even small residences, on a portion of the town plaza. The Municipal Treasurer collected from these stall owners fees at the rate of P.25 per square meter a month. In time, the whole municipal market was rehabilitated, but the owners of the structures on the plaza failed and refused to transfer to said market place. In 1951, the Municipal Council of Pozorrubio passed Resolution 209, ordering the occupants and owners of the structures on the plaza to remove their buildings within 60 days from receipt of the resolution. In answer to this resolution, 8 of the market stall building owners filed a petition for prohibition in the CFI Pangasinan against the Municipal Council, the Municipal Mayor, and the Chief of Police of Pozorrubio. The trial court, on 28 April 1956, dismissed the petition for prohibition filed by appellants and lifting the preliminary injunction it initially issued, and ordering the removal of the appellants stalls from the public plaza within 10 days from notice. Appeal was filed by the appellants to the Supreme Court. On 25 January 1957, appellants voluntarily vacated the public plaza of Pozorrubio. Concrete fences were constructed in the premises, and absent any complaints from the appellants nor their counsels, a motion for the dismissal of the case was filed as the case has become moot and academic. The appellants failed to comment on the petition for dismissal. The Supreme Court decided the case in a formal resolution, instead of a resolution for the satisfaction of the parties and guidance of the town officials and residents. It affirmed the decision appealed from; with costs against appellants. 1. No contract or agreement between appellants and municipality; fee not rent but market stall fees The fee of P.25 per square meter collected by the Municipal Treasurer, was not for the rent of the portion of the public plaza occupied by the market stalls, but rather the market stall fees charges on all market vendors in a public market. There was absolutely no contract or agreement between the appellants and the municipality, about renting of the Plaza to the former. The occupation of the plaza and the construction of temporary buildings thereon mostly for market, even residence purposes, was merely tolerated by the municipality, because of the destruction of the public market during the war. 2. Town plazas are properties of public dominion to be devoted to public use and made available to public in general; illegal private construction a nuisance subject to abatement according to law Town plaza cannot be used for the construction of market stalls, specially of residences, and that such structures constitute a nuisance subject to abatement according to law. Town plazas are properties of public dominion, to be devoted to public use and to be made available to the public in general. They are outside the commerce of man and cannot be disposed of or even leased by the municipality to private parties. While in case of war or during an emergency, town plazas may be occupied temporarily by private individuals, when the emergency has ceased, said temporary occupation or use must also cease, and the town officials should see to it that the town plazas should ever be kept open to the public and free from encumbrances or illegal private constructions. Villanueva v. Castaneda [G.R. No. L-61311. September 21, 1987.] First Division, Cruz (J): 3 concur, 1 on leave Facts: On 7 November 1961, the municipal council of San Fernando adopted Resolution 218 authorizing some 24 members of the Fernandino United Merchants and Traders Association to construct permanent stalls and sell in the vicinity of the public market of San Fernando, Pampanga, along Mercado Street, a strip of land measuring 12 by 77 meters on which stands a conglomeration of vendors stalls together forming what is commonly known as a talipapa. The action was protested on 10 November 1961, where the CFI Pampanga (Branch 2, Civil Case 2040) issued a writ of preliminary injunction that prevented Villanueva, et.al. from constructing the said stalls until final resolution of the controversy. On 18 January 1964, while the case was pending, the municipal council of San Fernando adopted Resolution 29, which declared the subject area as "the parking place and as the public plaza of the municipality," thereby impliedly revoking Resolution 218, s. 1961. On 2 November 1968, the lower court decided the case and held that the land occupied by Villanueva et.al., being public in nature, was beyond the commerce of man and therefore could not be the subject of private occupancy. The writ of preliminary injunction was made permanent. The decision was apparently not enforced, as Villanueva, et.al. were not evicted from the place and that by 1971, they and the 128 other persons were assigned specific areas or space allotments therein for which they paid daily fees to the municipal government. On 12 January 1982, however, the Association of Concerned Citizens and Consumers of San Fernando filed a petition for the

immediate implementation of Resolution 29, to restore the subject property "to its original and customary use as a public plaza. On 14 June 1982, after investigation by the municipal attorney, Macalino as OIC of the office of the mayor of San Fernando issued a resolution requiring the municipal treasurer and engineer to demolish the stalls. Villanueva, et.al. filed a petition for prohibition with the CFI Pampanga (Civil Case 6470), on 26 June 1982. The court denied the petition on 19 July 1982, and the motion for reconsideration on 5 August 1982, prompting Villanueva, et.al. to come to the Supreme Court on certiorari. The Supreme Court dismissed the petition, affirmed the 19 July 1982 decision and the 5 August 1982 order of the lower court, and lifted the temporary restraining order issued on 9 August 1982; the decision being immediately executory; with costs against Villanueva, et.al. 1. Land reserved for a public plaza According to former San Fernando Mayor Rodolfo Hizon, who later became governor of Pampanga, the National Planning Commission had reserved the area for a public plaza as early as 1951. This intention was reiterated in 1964 through the adoption of Resolution 29. The place occupied by Villanueva et.al from which they are sought to be evicted is a public plaza. There is no reason to disturb the findings in Civil Case 2040 in the current case Civil Case 6740. 2. Public plaza beyond the commerce of man and cannot be subject of lease or any contractual undertaking; Municipality of Cavite v. Rojas A public plaza is beyond the commerce of man and so cannot be the subject of lease or any other contractual undertaking. This rule was settled as early as in Municipality of Cavite v. Rojas (1915), where the Court declared as null and void the lease of a public plaza of the said municipality in favor of a private person. This is pursuant to Article 344 of the Civil Code which provides that property for public use in provinces and in towns comprises the provincial and town roads, the squares, streets, fountains, and public waters, the promenades, and public works of general service supported by said towns or provinces; and Article 1271 which prescribes that everything which is not outside the commerce of man may be the object of a contract. Thus, plazas and streets, like common lands, rivers, fountains, etc., which are for public use are communal things that cannot be sold because they are by their very nature outside of commerce of man. In leasing a portion of the plaza or public place, the Municipality exceeded its authority in the exercise of its powers by executing a contract over a thing of which it could not dispose, nor is it empowered so to do. That being the case, the contract of lease is null and void and of no force or effect because it is contrary to the law and the thing leased cannot be the object of a contract. 3. Portion of sidewalk also beyond commerce of man; Muyot v. del a Fuente In Muyot v. de la Fuente, it was held that the City of Manila could not lease a portion of a public sidewalk on Plaza Sta. Cruz, being likewise beyond the commerce of man. The sidewalk, forming part of the public plaza of Sta. Cruz, could not be a proper subject matter of the contract, as it was not within the commerce of man (Article 1347, new Civil Code, and article 1271, old Civil Code). Any contract entered into by the City of Manila in connection with the sidewalk, is ipso facto null and ultra vires. (Municipality of Cavite vs. Roxas, et al., 30 Phil. 603.) The sidewalk was intended for and was used by the public, in going from one place to another (The streets and public places of the city shall be kept free and clear for the use of the public, and the sidewalks and crossings for the pedestrians, and the same shall only be used or occupied for other purposes as provided by ordinance or regulation [Sec. 1119, Revised Ordinances of the City of Manila.] The booths served as fruit stands for their owners and often, if not always, blocked the free passage of pedestrians who had to take the plaza itself which used to be clogged with vehicular traffic. 4. Town plazas are properties of public dominion, and devoted to public use; Espiritu v. Pozorrubio The town plaza cannot be used for the construction of market stalls, specially of residences, and that such structures constitute a nuisance subject to abatement according to law. Town plazas are properties of public dominion, to be devoted to public use and to be made available to the public in general. They are outside the commerce of man and cannot be disposed of or even leased by the municipality to private parties (Espiritu v. Municipal Council of Pozorrubio). 5. OIC of the office of the mayor had authority to issuing order to demolish the stalls as declared by judicial and legislative authorities; Investigation merely a deference to the requirements of due process The officer-in-charge of the office of the mayor had the duty to clear the area and restore it to its intended use as a parking place and public plaza of the municipality of San Fernando, conformably to the orders from the court (Civil case 2040) and the council (Resolution 29, s. 1964). It is not correct to say that he had acted without authority or taken the law into his hands in issuing his order. Even without the investigation and recommendation of the municipal attorney, the mayor was justified in ordering the area cleared on the strength alone of its status as a public plaza as declared by the judicial and legislative authorities. In calling first for the investigation (which Villanueva, et.al. saw fit to boycott), he was just scrupulously paying deference to the requirements of due process, to remove all taint of arbitrariness in the action he was called upon to take. 6. General Welfare Clause; Police power delegated to the municipality The problems caused by the usurpation of the place are covered by the police power as delegated to the municipality under the general welfare clause. This authorizes the municipal council "to enact such ordinances and make such regulations, not repugnant to law, as may be necessary to carry into effect and discharge the powers and duties conferred upon it by law and such as shall seem necessary and proper to provide for the health and safety, promote the prosperity, improve the morals, peace, good order, comfort, and convenience of the municipality and the inhabitants thereof, and for the protection of property therein." This authority was validly exercised in this case through the adoption of Resolution 29, s. 1964, by the municipal council of San Fernando. 7. Police power cannot be bargained away through contract Police power cannot be surrendered or bargained away through the medium of a contract. Every contract affecting the public interest suffers a congenital infirmity in that it contains an implied reservation of the police power as a postulate of the existing legal order. This power can be activated at any time to change the provisions of the contract, or even abrogate it entirely, for the promotion or protection of the general welfare. Such an act will not militate against the impairment clause, which is subject to and limited by the paramount police power.

Municipality of Cavite v. Rojas [G.R. No. 9069. March 31, 1915.] En Banc, Torres (J): 3 concur, 1concur in result Facts: The Municipality (constituted through Act 82), and as the successor to the rights said entity had under the late Spanish government, and by virtue of Act 1039, had exclusive right, control and administration over the streets, lanes, plazas, and public places of the municipality of Cavite. Rojas, et.al., by virtue of a lease secured from the Municipality (Resolution 10, dated 3 July 1907), occupied a parcel of land 93 sq. m. in area that forms part of the public plaza known under the name of Soledad, belonging to the municipality of Cavite. Rojas constructed thereon a house, paying the Municipality a rental of P5.58 a quarter in advance for occupation thereof (schedule fixed in Ordinance 43, s. 1903), with the condition that Rojas are obligated to vacate the leased land within 60 days subsequent to the Municipality's demand to that effect. Rojas has been required by the municipality to vacate and deliver possession of the said land, wherein the 60days within which it was ought to vacated elapsed without Rojas doing so. Thus, by an instrument dated 5 December 1911, afterwards amended on 14 March 1912, the provincial fiscal of Cavite, representing the municipality, filed a complaint in the CFI Cavite against Rojas alleging that the lease secured from the municipality of Cavite is ultra vires and therefore ipso facto null and void and of no force or effect, for the said land is an integral portion of a public plaza of public domain, and thus prayed that judgment be rendered declaring that possession of the said land lies with the Municipality and ordering Rojas to vacate the land and deliver possession thereof to the Municipality. After hearing and on 27 March 1913, the court rendered the judgment dismissing the complaint with cost against the Municipality. The counsel for the municipality excepted and in writing asked for a reopening of the case and the holding of a new trial. This motion was denied, with exception on the part of the Municipality, and the corresponding bill of exceptions was filed, approved and forwarded to the clerk of the Supreme Court. The Supreme Court reversed the judgment appealed from and declared the land occupied public, as it formed part of the public plaza called Soledad, and the lease of said parcel of land as null and void. The Court ordered Rojas to vacate it and release the land within 30 days, leaving it and as it was before her occupation. There is no ground for the indemnity sought in the nature of damages, but the municipality must in its turn restore to Rojas the rentals collected; without special finding as to the costs. 1. Plaza Soledad; Municipality or objectors not entitled for inscription of land for public use and reserved for the common benefit By section 3 of the said Act No. 1039, passed January 12, 1904, the Philippine Commission granted to the municipality of Cavite all the land included in the tract called Plaza Soledad. In the case of Nicolas vs. Jose (6 Phil 589), wherein the municipality of Cavite, represented by its president Catalino Nicolas, sought inscription in its name of the land comprised in the said Plaza Soledad, with objection on the part of Maria Jose et al., who occupied some parts thereof with their houses and who also sought that inscription be decreed in their name of the parcels of land in this plaza occupied by them, this court decided that neither the municipality nor the objectors were entitled to inscription, for with respect to the objectors said plaza belonged to the municipality of Cavite and with respect to the latter the said Plaza Soledad was not transferable property of that municipality to be inscribed in its name, because the intention of Act No. 1039 was that the said plaza and other places therein enumerated should be kept open for public transit; wherefore there can be no doubt that the defendant has no right to continue to occupy the land of the municipality leased by her, for it is an integral portion of Plaza Soledad, which is for public use and is reserved for the common benefit. 2. Property for public use in provinces and in towns Article 344 of the Civil Code provides that "property for public use in provinces and in towns comprises the provincial and town roads, the squares, streets, fountains, and public waters, the promenades, and public works of general service supported by said towns or provinces." Plaza Soledad being a promenade for public use, the municipal council of Cavite could not in 1907 withdraw or exclude from public use a portion thereof in order to lease it for the sole benefit of Rojas. In leasing a portion of said plaza or public place for private use, municipality exceeded its authority in the exercise of its powers by executing a contract over a thing of which it could not dispose, nor is it empowered so to do. 3. Communal things cannot be sold as they are outside the commerce of man Article 1271 of the Civil Code prescribes that everything which is not outside the commerce of man may be the object of a contract. As plazas and streets are outside of this commerce, the 12 February 1895 decision of the Spanish Supreme Court stated that "communal things that cannot be sold because they are by their very nature outside of commerce are those for public use, such as the plazas, streets, common lands, rivers, fountains, etc." The lease contract, whereby the municipality of Cavite leased to Rojas a portion of the Plaza Soledad, is null and void and of no force or effect, in accordance with the provision of Article 1303 of the Civil Code, because it is contrary to the law and the thing leased cannot be the object of a contract. Thus, Rojas must restore and deliver possession of the land described in the complaint to the municipality of Cavite, which in its turn must restore to Rojas all the sums it may have received from her in the nature of rentals just as soon as she restores the land improperly leased. For the same reasons as have been set forth, consequently Rojas is not entitled to claim that the municipality indemnify her for the damages she may suffer by the removal of her house from the said land. Province of Zamboanga del Norte v. City of Zamboanga [G.R. No. L-24440. March 28, 1968.] En Banc, Bengzon (J): 8 concur, 1 on leave Facts: Prior to its incorporation as a chartered city, the Municipality of Zamboanga used to be the provincial capital of the then Zamboanga Province. On 12 October 1936, Commonwealth Act 39 was approved converting the Municipality of Zamboanga into Zamboanga City. Section 50 of the Act also provided that "buildings and properties which the province shall abandon upon the transfer of the capital to another place will be acquired and paid for by the City of Zamboanga at a price to be fixed by the Auditor General." The properties and buildings referred to consisted of 50 lots and some buildings constructed thereon, located in the City of Zamboanga and covered individually by Torrens certificates of title in the name of Zamboanga Province. The lots are utilized as the Capitol Site (1 lot), School site (3 lots), Hospital site (3 lots), Leprosarium (3 lots), Curuan school (1 lot), Trade school (1 lot), Burleigh school (2 lots), burleigh (9 lots), high school playground (2 lots), hydro-electric site (1 lot), san roque (?1 lot), and

another 23 vacant lots. In 1945, the capital of Zamboanga Province was transferred to Dipolog and on 16 June 1948, RA 286 created the municipality of Molave and making it the capital of Zamboanga Province. On 26 May 1949, the Appraisal Committee formed by the Auditor General, pursuant to CA 39, fixed the value of the properties and buildings in question left by Zamboanga Province in Zamboanga City at P1,294,244.00. However, on 14 July 1951, a Cabinet Resolution was passed, conveying all the said 50 lots and buildings thereon to Zamboanga City for P1.00, effective as of 1945, when the provincial capital of the Zamboanga Province was transferred to Dipolog. On 6 June 1952, RA 711 was approved dividing the province of Zamboanga into Zamboanga del Norte and Zamboanga del Sur. As to how the assets and obligations of the old province were to be divided between the two new ones, Section 6 of the law provided that upon the approval of the Act, the funds, assets and other properties and the obligations of the province of Zamboanga shall be divided equitably between the Province of Zamboanga del Norte and the Province of Zamboanga del Sur by the President of the Philippines, upon the recommendation of the Auditor General." On 11 January 1955, the Auditor General apportioned the assets and obligations of the defunct Province of Zamboanga, apportioning 54.39% for Zamboanga del Norte and 45.61% for Zamboanga del Sur. On 17 March 1959, the Executive Secretary, by order of the President, issued a ruling holding that Zamboanga del Norte had a vested right as owner (should be co-owner pro-indiviso) of the properties mentioned in Section 50 of CA 39, and is entitled to the price thereof, payable by Zamboanga City. This effectively revoked the Cabinet Resolution of 14 July 1951. The Secretary of Finance then authorized the Commissioner of Internal Revenue to deduct an amount equal to 25% of the regular internal revenue allotment for the City of Zamboanga for the quarter ending 31 March 1960, then for the quarter ending 30 June 1960, and again for the first quarter of the fiscal year 1960-1961. The deductions, all aggregating P57,373.46 was credited to the province of Zamboanga del Norte, in partial payment of the P704,220,05 due it. However, on 17 June 1961, RA 3039 was approved amending Section 50 of CA 39 by providing that "all buildings, properties and assets belonging to the former province of Zamboanga and located within the City of Zamboanga are hereby transferred, free of charge, in favor of the said City of Zamboanga." On 12 July 1961, the Secretary of Finance ordered the Commissioner of Internal Revenue to stop from effecting further payments to Zamboanga del Norte and to return to Zamboanga City the sum of P57,373.46 taken from it out of the internal revenue allotment of Zamboanga del Norte. Zamboanga City admits that since the enactment of RA 3039, P43,030.11 of the P57,373.46 has already been returned to it. This constrained Zamboanga del Norte to file on 5 March 1962, a complaint entitled "Declaratory Relief with Preliminary Mandatory Injunction" in the CFI Zamboanga del Norte against Zamboanga City, the Secretary of Finance and the Commissioner of Internal Revenue. On 4 June 1962, the lower court ordered the issuance of preliminary injunction as prayed for. After trial and on 12 August 1963, judgment was rendered declaring RA 3039 unconstitutional as it deprives the province of its private properties, ordered the city to pay the province the sum of P704,200.05 and in relation to this ordered the finance secretary to direct the Commissioner of Internal revenue to deduct from its regular quarterly internal revenue allotment equivalent to 25%, 25% from the regular quarterly internal revenue allotment for the City and to remit the same to the province until the sum has been fully paid; ordered the province to execute the corresponding public instrument deeding to the city the 50 parcels of land and the improvements thereon under the certificates of title upon full payment; dismissed the counterclaim of the city; and declared permanent the preliminary mandatory injunction issued on 8 June 1967. The province filed a motion to reconsider praying that the City be ordered instead to pay the P704,220.05 in lump sum with 6% interest per annum. Over the citys opposition, the lower court granted the provinces motion. Hence, the appeal to the Supreme Court. The Supreme Court set aside the decision appealed from and entered another ordering the city to return to the province in lump sum the amount of P43,030,11 which the city took back from the province out of the sum of P57,373.46 previously paid to the latter, and ordering the city to effect payments in favor of the province of whatever balance remains of the provinces's 54.39% share in the 26 patrimonial properties, after deducting therefrom the sum of P57,373.46, on the basis of Resolution 7 dated 26 March 1949 of the Appraisal Committee formed by the Auditor General, by way of quarterly payments from the allotments of the City, in the manner originally adopted by the Secretary of Finance and the Commissioner of Internal Revenue; without costs. 1. Procedural aspect; Rules authorize conversion of proceedings to an ordinary action Brushing aside the procedural point concerning the propriety of declaratory relief filed in the lower court on the assertion that the law had already been violated and that the province of Zamboanga del Norte sought to give it coercive effect, since assuming the same to be true, the Rules anyway authorize the conversion of the proceedings to an ordinary action, the Court proceeded to the more important and principal question of the validity of RA 3039. 2. Property owned by municipality in its governmental capacity, property is public; Congress has absolute control over it If the property is owned by the municipality (meaning municipal corporation) in its public and governmental capacity, the property is public and Congress has absolute control over it. But if the property is owned in its private or proprietary capacity, then it is patrimonial and Congress has no absolute control. The municipality cannot be deprived of it without due process and payment of just compensation. 3. Properties of provinces, cities and municipalities Article 423 of the Civil Code provides that the property of provinces, cities and municipalities, is divided into property for public use and patrimonial properly." Article 424 of the same code provides that property for public use, in the provinces, cities, and municipalities, consists of the provincial roads, city streets, municipal streets, the squares, fountains, public waters, promenades, and public works for public service paid for by said provinces, cities, or municipalities. All other property possessed by any of them is patrimonial and shall be governed by this Code, without prejudice to the provisions of special laws." 4. Application of Article 424 NCC

a. Properties patrimonial except for playground Applying the norm in the Civil Code, all the properties in question, except the two (2) lots used as High School

playgrounds, could be considered as patrimonial properties of the former Zamboanga province. Even the capitol site, the hospital and leprosarium sites, and the school sites will be considered patrimonial for they are not for public use. b. Ejusdem Generis, properties not included in public works for public service They would not fall under the phrase "public works for public service" for it has been held that under the ejusdem generis rule, such public works must be for free and indiscriminate use by anyone, just like the preceding enumerated properties in the first paragraph of Article. 424. The playgrounds, however, would fit into this category. This was the norm applied by the lower court. c. Jurisprudence And it cannot be said that its actuation was without jurisprudential precedent for in Municipality of Catbalogan v. Director of Lands, and in Municipality of Tacloban v. Director of Lands, 9 it was held that the capitol site and the school sites in municipalities constitute their patrimonial properties. This result is understandable because, unlike in the classification regarding State properties, properties for public service in the municipalities are not classified as public. 5. Application of the Law of Municipal Corporations

a. Properties devoted to public service are deemed public Applying the norm obtaining under the principles constituting the law of Municipal Corporations, all those of the 50 properties in question which are devoted to public service are deemed public; the rest remain patrimonial. Under this norm, to be considered public, it is enough that the property be held and devoted for governmental purposes like local administration, public education, public health, etc. b. Jurisprudence Supporting jurisprudence are found in the following cases: (1) Hinunangan v. Director of Lands, where it was stated that "where the municipality has occupied lands distinctly for public purposes, such as for the municipal court house, the public school, the public market, or other necessary municipal building, the Court will, in the absence of proof to the contrary, presume a grant from the State in favor of the municipality; but, as indicated by the wording, that rule may be invoked only as to property which is used distinctly for public purposes." (2) Viuda de Tantoco v. Municipal Council of Iloilo held that municipal properties necessary for governmental purposes are public in nature. Thus, the auto trucks used by the municipality for street sprinkling, the police patrol automobile, police stations and concrete structures with the corresponding lots used as markets were declared exempt from execution and attachment since they were not patrimonial properties. (3) Municipality of Batangas v. Cantos, held squarely that a municipal lot which had always been devoted to school purposes is one dedicated to public use and is not patrimonial property of a municipality. 6. RA 3039 valid insofar as it affects 24 lots previously used by the province in its governmental capacity RA 3039 is valid insofar as it affects the lots used as capitol site, school sites and its grounds, hospital and leprosarium sites and the high school playground sites, a total of 24 lots, since these were held by the former Zamboanga province in its governmental capacity and therefore are subject to the absolute control of Congress. 7. Burleigh lots constitute appurtenant grounds of the Burleigh schools and partake of the nature of the same The eight Burleigh lots are adjoining each other and in turn are between the two lots wherein the Burleigh schools are built. Said eight lots constitute the appurtenant grounds of the Burleigh schools and partake of the nature of the same. 8. Buildings on 24 lots are public; Assumption based on history, and Character of buildings as accessory to the land The records do not disclose whether they were constructed at the expense of the former Province of Zamboanga. Considering however the fact that said buildings must have been erected even before 1936 when CA 39 was enacted and the further fact that provinces then had no power to authorize construction of buildings such as those in the present case at their own expense, it can be assumed that said buildings were erected by the National Government, using national funds. Hence, Congress could very well dispose of said buildings in the same manner that it did with the lots in question. But even assuming that provincial funds were used, still the buildings constitute mere accessories to the lands, which are public in nature, and so, they follow the nature of said lands, i.e., public. Moreover, said buildings, those located in the city, will not be for the exclusive use and benefit of city residents for they could be availed of also by the provincial residents. The province then, and its successors-in-interest, are not really deprived of the benefits thereof. 9. Province cannot be deprived of its share in the value of the 26 other lots RA 3039 cannot be applied to deprive Zamboanga del Norte of its share in the value of the rest of the 26 remaining lots which are patrimonial properties since they are not being utilized for distinctly governmental purposes. The fact that these 26 lots are registered strengthens the proposition that they are truly private in nature. 10. Registration cannot convert public property to private The fact that the 24 lots used for governmental purposes are registered is of no significance since registration cannot convert public property to private. 11. Law of Municipal Corporations prevails over that of Civil Code; Dire consequence of holding property for public service similar to ordinary private property The Court is more inclined to uphold the view that the controversy is more along the domains of the Law of Municipal Corporations (State v. Province) than along that of Civil Law. The Court is not inclined to hold that municipal property held and devoted to public service is in the same category as ordinary private property. The consequences are dire. As ordinary private properties, they can be levied upon and attached. They can even be acquired thru adverse possession, all these to the detriment of the local community.

12. Law of Municipal Corporation, for the purpose of Article 424 NCC, considered special law The classification of properties other than those for public use in the municipalities as patrimonial under Article 424 of the Civil Code is "without prejudice to the provisions of special laws." For purposes of the article, the principles obtaining under the Law of Municipal Corporations can be considered as "special laws". Hence, the classification of municipal property devoted for governmental purposes as public should prevail over the Civil Code classification in this particular case. 13. Province not guilty of laches Under CA 39, Section 50, the cause of action in favor of the defunct Zamboanga Province arose only in 1949 after the Auditor General fixed the value of the properties in question. While in 1951, the Cabinet resolved to transfer said properties practically for free to Zamboanga City, a reconsideration thereof was seasonably sought. In 1952, the old province was dissolved. As successorin-interest to more than half of the properties involved, Zamboanga del Norte was able to get a reconsideration of the Cabinet Resolution in 1959. In fact, partial payments were effected subsequently and it was only after the passage of RA 3039 in 1961 that the present controversy arose. Plaintiff brought suit in 1962. All the foregoing, thus, are negative laches. 14. Zamboanga del Norte entitled to 54.39% share in the amounts collected in the 26 patrimonial properties; Payment cannot be paid in lump sum Zamboanga del Norte is still entitled to collect from the City of Zamboanga the former's 54.39% share in the 26 properties which are patrimonial in nature, said share to be computed on the basis of the valuation of said 26 properties as contained in Resolution 7, dated 26 March 1949, of the Appraisal Committee formed by the Auditor General. The share, however, cannot be paid in lump sum, except as to the P43,030.11 already returned to the City, as the return of said amount to the city was without legal basis. RA 3039 took effect only on 17 June 1961 after a partial payment of P57,373.46 had already been made. Since the law did not provide for retroactivity, it could not have validly affected a completed act. Hence, the amount of P43,030.11 should be immediately returned by the City to the province. The remaining balance, if any, in the amount of plaintiff's 54.39% share in the 26 lots should then be paid by the City in the same manner originally adopted by the Secretary of Finance and the Commissioner of Internal Revenue, and not in lump sum. Article 1169 of the Civil Code on reciprocal obligations invoked by plaintiff to justify lump sum payment is inapplicable since there has been so far in legal contemplation no complete delivery of the lots in question. The titles to the registered lots are not yet in the name of Zamboanga City. Bachrach v. Seifert [G.R. No. L-2659. October 12, 1950.] En Banc, Ozaeta (J): 8 concur Facts: The deceased Emil Maurice Bachrach left no forced heir except his widow Mary McDonald Bachrach. In his last will and testament made varius legacies in cash and willed all the fruits and usufruct the remainder of his estate (after payment of legacies, bequests and gifts) to his wifes enjoyment. The will further provided that upon the death of Mary McDonald Bachrach, one-half of all his estate shall be divided share and share alike by and betweenhis legal heirs, to the exclusion of his brothers. The estate of E. M. Bachrach, as owner of 108,000 shares of stock of the Atok-Big Wedge Mining Co., Inc., received from the latter 54,000 shares representing 50% stock dividend on the said 108,000 shares. On 10 June 1948, Mary McDonald Bachrach, as usufructuary or life tenant of the estate, petitioned the lower court to authorize the Peoples Bank and Trust Company, as administrator of the estate of E. M. Bachrach, to transfer to her the said 54,000 shares of stock dividend by indorsing and delivering to her the corresponding certificate of stock, claiming that said dividend, although paid out in the form of stock, is fruit or income and therefore belonged to her as usufructuary or life tenant. Sophie Siefert and Elisa Elianoff, legal heirs of the deceased, opposed said petition on the ground that the stock dividend in question was not income but formed part of the capital and therefore belonged not to the usufructuary but to the remainderman. The lower court granted the petition and overruled their objection. Siefer and Elianoff appealed. The Supreme Court affirmed the order appealed from, being in accordance with the above-quoted provisions of the Civil Code; with costs against the appellants. 1. Massachusetts rule, declares stock dividend is not an income but merely represents an addition to the investment capital The so-called Massachusetts rule, which prevails in certain jurisdictions in the United States, regards cash dividends, however large, as income, and stock dividends, however made, as capital. (Minot vs. Paine, 99 Mass., 101; 96 Am. Dec., 705.) It holds that a stock dividend is not in any true sense any dividend at all since it involves no division or severance from the corporate assets of the subject of the dividend; that it does not distribute property but simply dilutes the shares as they existed before; and that it takes nothing from the property of the corporation, and adds nothing to the interests of the shareholders. Thus, the rule supports the appellants contention that a stock dividends is not an income (unlike a cash dividend), but merely represents an addition to the invested capital. 2. Pennsylvania rule; all earnings of the corporation made prior to the testator stockholder belong to his estate The so-called Pennsylvania rule, which prevails in various other jurisdictions in the United States, declares that all earnings of the corporation made prior to the death of the testator stockholder belong to the corpus of the estate, and that all earnings, when declared as dividends in whatever form, made during the lifetime of the usufructuary or life tenant are income and belong to the usufructuary or life tenant. (Earp's Appeal, 28 Pa., 368.) The rule supports appellee's contention. 3. Pennsylvania Rule; Remaindermen in trust of the corpus of the estate, while dividends go to life tenants The testator intended the remaindermen should have only the corpus of the estate he left in trust, and that all dividends should go to the life tenants. It is true that profits realized are not dividends until declared by the proper officials of the corporation, but distribution of profits, however made, is dividends, and the form of the distribution is immaterial." (In re Thompson's Estate, 262 Pa., 278; 105 Atl. 273, 274.)

4. Dividend, even if declared as stock, which is based upon the earnings of the company is an income of the capital invested in it In Hite vs. Hite (93 Ky., 257; 20 S. W., 778, 780), the Court of Appeals of Kentucky, held that "where a dividend, although declared in stock, is based upon the earnings of the company, it is in reality, whether called by one name or another, the income of the capital invested in it. It is but a mode of distributing the profit. A stock dividend proper is the issue of new shares paid for by the transfer of a sum equal to their par value from the profit and loss account to that representing capital stock; and really a corporation has no right to declare a dividend, either in cash or stock, except from its earnings. In the present case; the 108,000 shares of stock are part of the property in usufruct. The 54,000 shares of stock dividend are civil fruits of the original investment. They represent profits, and the delivery of the certificate of stock covering said dividend is equivalent to the payment of said profits. Said shares may be sold independently of the original shares, just as the offspring of a domestic animal may be sold independently of its mother. 5. Testators intent; substance not form The law regards substance, and not form, and such a rule might result not only in a violation of the testator's intention, but it would give the power to the corporation to beggar the life tenants, who, in this case, are the wife and children of the testator, for the benefit of the ramaindermen, who may perhaps be unknown to the testator, being unborn when the will was executed. The Court is unwilling to adopt a rule which to us seems so arbitrary, and devoid of reason and justice. If the dividend be in fact a profit, although declared in stock, it should be held to be income. 6. Pennsylvania rule more in accord with local statutes The Pennsylvania rule is more in accord with Philippine statutory laws than the Massachusetts rule. Under section 16 of the Corporation Law, no corporation may make or declare any dividend except from the surplus profits arising from its business. Any dividend, therefore, whether cash or stock, represents surplus profits. 7. Usufructuary entitled to fruits of the property in usufruct Article 471 of the Civil Code provides that the usufructuary shall be entitled to receive all the natural, industrial, and civil fruits of the property in usufruct. Further, Articles 474 provides that " Civil fruits are deemed to accrue day by day, and belong to the usufructuary in proportion to the time the usufruct may last. Article 475, on the other hand, provides that When a usufruct is created on the right to receive an income or periodical revenue, either in money or fruits, or the interest on bonds or securities payable to bearer, each matured payment shall be considered as the proceeds or fruits of such right. When it consists of the enjoyment of the benefits arising from an interest in an industrial or commercial enterprise, the profits of which are not distributed at fixed periods, such profits shall have the same consideration. In either case they shall be distributed as civil fruits, and shall be applied in accordance with the rules prescribed by the next preceding article." Bachrach Motors v. Talisay-Silay Milling [G.R. No. 35223. September 17, 1931.] En Banc, Romualdez (J): 7 concur Facts: On 22 December 1923, the Talisay-Silay Milling Co., Inc., was indebted to the PNB. To secure the payment of its debt, it succeeded in inducing its planters, among whom was Mariano Lacson Ledesma, to mortgage their land to the bank. And in order to compensate those planters for the risk they were running with their property under that mortgage, the aforesaid central, by a resolution passed on the same date, and amended on 23 March 1928, undertook to credit the owners of the plantation thus mortgaged every year with a sum equal to 2% of the debt secured according to the yearly balance, the payment of the bonus being made at once, or in part from time to time, as soon as the central became free of its obligations to the bank, and of those contracted by virtue of the contract of supervision, and had funds which might be so used, or as soon as it obtained from said bank authority to make such payment. It seems Mariano Lacson Ledesma is indebted from Bachrach Motor; the circumstance of which is not found in the case facts. Bachrach Motor Co., Inc. filed a complaint against the Talisay-Silay Milling Co., Inc., for the delivery of the amount of P13,850 or promissory notes or other instruments of credit for that sum payable on 30 June 1930, as bonus in favor of Mariano Lacson Ledesma. The complaint further prays that the sugar central be ordered to render an accounting of the amounts it owes Mariano Lacson Ledesma by way of bonus, dividends, or otherwise, and to pay Bachrach Motors a sum sufficient to satisfy the judgment mentioned in the complaint, and that the sale made by said Mariano Lacson Ledesma be declared null and void. The PNB filed a third party claim alleging a preferential right to receive any amount which Mariano Lacson Ledesma might be entitled from Talisay-Silay Milling as bonus. Talisay-Silay answered the complaint that Mariano Lacson Ledesmas credit (P7,500) belonged to Cesar Ledesma because he had purchase it. Cesar Ledesma claimed to be an owner by purchase in good faith. At the trial all the parties agreed to recognize and respect the sale made in favor of Cesar Ledesma of the P7,500 part of the credit in question, for which reason the trial court dismissed the complaint and cross-complaint against Cesar Ledesma authorizing the central to deliver to him the sum of P7,500. And upon conclusion of the hearing, the court held that the Bachrach Motor Co., Inc., had a preferred right to receive the amount of P11,076.02 which was Mariano Lacson Ledesma's bonus, and it ordered the central to deliver said sum to Bachrach Motors. PNB appealed. The Supreme Court affirmed the judgment appealed from, as it found no merit in the appeal;, without express finding as to costs. 1. Civil Fruits under Article 355 of the Civil Code Article 355 of the Civil Code considers three things as civil fruits: First, the rents of buildings; second, the proceeds from leases of lands; and, third, the income from perpetual or life annuities, or other similar sources of revenue. According to the context of the law, the phrase "u otras analogas" refers only to rents or income, for the adjectives "otras" and "analogas" agree with the noun "rentas," as do also the other adjectives "perpetuas" and "vitalicias." The "civil fruits" the Civil Code understands one of three and only three things, to wit: the rent of a building, the rent of land, and certain kinds of income.

2. Bonus not a civil fruit; not an income of the land The amount of the bonus, according to the resolution of the central granting it, is not based upon the value, importance or any other circumstance of the mortgaged property, but upon the total value of the debt thereby secured, according to the annual balance, which is something quite distinct from and independent of the property referred to. As the bonus is not obtained from the land, it is not civil fruits of that land. It is neither rent of buildings, proceeds from lease of lands, or income under Article 355 of the Civil Code. Pacific Farms v. Esguerra [G.R. No. L-21783. November 29, 1969.] En Banc, Castro (J): 8 concur, 1 concurs in result Facts: On several occasions from 1 October 1956 to 2 March 1957 the Carried Lumber Company sold and delivered lumber and construction materials to the Insular Farms, Inc. which the latter used in the construction of 6 buildings at its compound in Bolinao, Pangasinan, of the total procurement price of P15,000, the sum of P4,710.18 has not been paid by Insular Farms, Inc. Consequently, on 17 October 1958 the Company instituted Civil Case D-775 with the CFI Pangasinan to recover the said unpaid balance from the Insular Farms. On August 23, 1961 the trial court rendered judgment sustaining the Company's claim. The judgment-debtor did not appeal; so on 19 December 1961 the corresponding writ of execution was issued. On 16 January 1962 the sheriff levied upon the 6 buildings. On 30 January 1962, the Pacific Farms filed a third-party claim asserting ownership over the levied buildings which it had acquired from the Insular Farms by virtue of a deed of absolute sale executed on 21 March 1958, about 7 months before the Company filed the present action (Civil case D-775). Shielded by an indemnity bond of P7,120 put up by the Company and the Cosmopolitan Insurance Company, the sheriff proceeded with the announced public auction on 12 February 1962 and sold the levied buildings to the Company for P6,110.78. Asserting absolute and exclusive ownership of the buildings in question, the Pacific Farms filed a complaint on 14 May 1962 against the Company and the sheriff with the CFI Pangasinan, praying that judgment be rendered, (a) declaring null and void the levy and judicial sale of the 6 buildings, and (b) adjudging the defendants jointly and severally liable to the plaintiff in the sum of P2,000 by way of actual damages and for such amount as the court may deem proper and just to impose by way of exemplary damages and for costs of the suit. After due trial and on 30 May 1963, the court rendered judgment annulling the levy of 16 January 1962 and the certificate of sale of 12 February 1962. The court, however, denied the plaintiff's claim for actual and exemplary damages on the ground that it was not "prepared to find that there was gross negligence or bad faith on the part of any of the defendants." Hence, the appeal. The Supreme Court reversed the judgment, and dismissed the complaint. The Court granted, however in view of the equities attendant in the case, Pacific Farms a period of 30 days from the date of the finality of the judgment, within which it may exercise the option of redeeming the 6 buildings, by paying Carried Lumber Company the sum of P4,710.18 with legal interest from 23 September 1961 until the said amount shall have been fully paid; without pronouncement as to costs. 1. De Barretto v. Villanueva not applicable The case De Barretto, et al. vs. Villanueva, et al., L-14938 (6 SCRA 928) is inapplicable in the present case because it concerned not one but two or more preferred creditors who, pursuant to articles 2242 and 2249 of the Civil Code, must necessarily be convened and the nature and extent of their respective claims ascertained. As held, before there can be a pro rata payment of credits entitled to preference as to the same specific real property, there must first be some proceeding where the claims of all the preferred creditors may be bindingly adjudicated, such as insolvency, the settlement of a decedent's estate under Rule 87 of the Rules of Court, or liquidation proceedings of similar import. The present case, however, does not involve a question of preference of credits, and is not one where two or more creditors have separate and distinct claims against the same debtor who has insufficient property. 2. Carried Lumber Company is an unpaid furnisher of materials The appellant is an unpaid furnisher of materials. It is undenied and undeniable that the appellant furnished lumber and construction materials to the Insular Farms which the latter used in the construction of the six buildings. Likewise unchallenged is the lower court's factual finding that out of the total procurement price of P15,000, the amount of P4,710.18 remains outstanding and unpaid by the Insular Farms, Inc. 3. Application by analogy of the rules of accession suffice, no need to decide on the existence of a materialmans lien; Application of Article 447 by analogy The application by analogy of the rules of accession would suffice for a just adjudication. Article 447 of the Civil Code provides that the owner of the land who makes thereon personally or through another, plantings, constructions or works with the materials of another, shall pay their value; and, if he acted in bad faith, he shall also be obliged to the reparation of damages. The owner of the materials shall have the right to remove them only in case he can do so without injury to the work constructed, or without the plantings, constructions or works being destroyed. However, if the landowner acted in bad faith, the owner of the materials may remove them in any event with a right to be indemnified for damages." The legal provision contemplates a principal and an accessory, the land being considered the principal, and the plantings, constructions or works, the accessory. The owner of the land who in good faith, whether personally or through another, makes constructions or works thereon, using materials belonging to somebody else, becomes the owner of the said materials with the obligation however of praying for their value. The owner of the materials, on the other hand, is entitled to remove them, provided no substantial injury is caused to the landowner. Otherwise, he has the right to reimbursement for the value of his materials. Applying article 447 by analogy, the buildings are considered as the principal and the lumber and construction materials that went into their construction as the accessory. Thus the appellee, if it does own the six buildings, must bear the obligation to pay for the value of the said materials; the appellant which apparently has no desire to remove the materials, and even if it were minded to do so, cannot remove them without necessarily damaging the buildings has the corresponding right to recover the value of the unpaid lumber and construction materials. 4. Compensation should be borne by the person who benefited by the accession Well-established in jurisprudence is the role that compensation should be borne by the person who has been benefited by the

accession. No doubt, the appellee benefited from the accession, i.e., from the lumber and materials that went into the construction of the six buildings. It should therefore shoulder the compensation due to the appellant as unpaid furnisher of materials. 5. Pacific Farms not a buyer in good faith so as to exonerate it from making compensation The appellee's stance that it is an innocent purchaser for value and in good faith is open to grave doubt because of certain facts of substantial import (evident from the records) that cannot escape notice; and thus cannot possibly exonerate it from making compensation. First, J. Antonio Araneta is not only the president of the Insular Farms but also a director and counsel of Pacific Farms. Second, Atty. Amadeo Santiago Jr. of J. Antonio Aranetas law firm, represented Insular Farms in Civil Case D-775 is the same lawyer for the present action. Third, Insular Farms and Pacific Farms are housed in adjacent rooms at the Insular Life Building as early as 21 March 1958. It may be reasonable that Araneta as president of Insular Farm knew about the unpaid balance of the purchase price of the lumber and construction materials supplied or furnished by the appellant to the Insular Farms. Curiously, Insular Farms never moved to implead Pacific Farms as a necessary party-defendant in Civil Case D-775. 6. Carried Lumber not as advantageously situated as Pacific Farms; Current action proper Carried Lumber not as advantageously situated as Pacific Farms. There being no separate registry of property for buildings and no procedure provided by law for registering or annotating the claim of an unpaid furnisher of materials, it was helpless to prevent the sale of the property built from lumber and construction materials it furnished. But certainly, because it has a right, pursuant to article 447, to reimbursement for the value of its unpaid materials, the appellant could pursue any remedy available to it under the law in order to enforce the said right. Thus, the appellant acted correctly in bringing an action (D-775) against the Insular Farms, Inc. and enforcing its right of reimbursement through the execution of the final judgment it obtained in the said case against the six buildings in the possession of the appellee who now stands to benefit therefrom. It follows, as a necessary corollary, that the sale at public auction conducted by the defendant sheriff of the six buildings described in the certificate of sale dated 12 February 1962, was valid and effective. Manotok Realty v. Tecson [G.R. No. L-47475. August 19, 1988.] Third Division, Gutierrez Jr. (J): 4 concur Facts: Manotok Realty filed a complaint against Nilo Madlangawa for recovery of possession and damages with the then CFI Manila. Said court rendered judgment, declaring Madlangawa as a builder or possessor in good faith; ordering the company to recognize the right of Madlangawa to remain in Lot 345, Block 1, of the Clara Tambunting Subdivision until after he shall have been reimbursed by the company the sum of P7,500.00, without pronouncement as to costs. Not satisfied with the trial court's decision, the company appealed to the Court of Appeals and upon affirmance by the latter of the decision below, the company elevated its case to the Supreme Court. On 13 July 1977, the Supreme Court issued a resolution dated 11 July 1977 denying the company's petition for lack of merit. On 5 August 1977, the company filed with the trial court (Judge Jose H. Tecson), a motion for the approval of the company's exercise of option and for satisfaction of judgment, praying that the court issue an order: a) approving the exercise of the company's option to appropriate the improvements introduced by Madlangawa on the property; b) thereafter, Madlangawa be ordered to deliver possession of the property in question to the company. On 7 October 1977, the judge held that in view of peculiar circumstances which supervened the institution of the case, e.g. the introduction of certain repairs of and other substantial improvements on the controverted property, the motion for approval was denied. After a denial of its motion for reconsideration, the company filed the present petition for mandamus alleging that the judge committed grave abuse of discretion in denying his motion to exercise option and for execution of judgment on the grounds that under Articles 448 and 546 of the Civil Code, the exercise of option belongs to the owner of the property and that upon finality of judgment, the prevailing party is entitled, as a matter of right, to its execution which is only a ministerial act on the part of the judge. On 28 December 1980, PD 1669 was issued providing for the expropriation of the Tambunting Estate. However, this decree was challenged before the Supreme Court in GR 55166 (Elisa R. Manotok, et al. v. National Housing Authority, et al.). On 21 May 1987, the Court rendered a decision in the Elisa Manotok case ruling that PD 1669 is unconstitutional for being violative of the due process clause. Thus, the present petition has not been rendered moot and academic by the decision in Manotok v. NHA. The Supreme Court granted the petition and ordered Judge Tecson to immediately issue a writ of execution ordering the Madlangawa to vacate the disputed premises and deliver possession of the same to the company, Manotok Realty. 1. When decision becomes final and executory, judge incumbent to issue necessary writ of execution When the decision of the trial court became final and executory, it became incumbent upon the trial court judge to issue the necessary writ for the execution of the same. There is no basis for the judge to deny the petitioner's motion to avail of its option to appropriate the improvements made on its property. 2. When decision becomes final, no addition can be made thereto In Duenas v. Mandi (151 SCRA 530, 545), it was held that after a judgment has become final, no additions can be made thereto, and nothing can be done therewith except its execution, otherwise there would be no end to legal processes. (Fabular v. Court of Appeals, 119 SCRA 329)" The judge cannot deny the issuance of a writ of execution because the Madlangawa was adjudged a builder in good faith or on the ground of "peculiar circumstances which supervened after the institution of this case, like, for instance, the introduction of certain major repairs of and other substantial improvements" because the option given by law either to retain the premises and pay for the improvements thereon or to sell the said premises to the builder in good faith belongs to the owner of the property. 3. Options available to the parties In Queme v. Olaes (1 SCRA 1159, 1163), it was held that under Article 448, the right to appropriate the works or improvements or 'to oblige the one who built or planted to pay the price of the land' belongs to the owner of the land. The only right given to the

builder in good faith is the right to reimbursement for the improvements; the builder, cannot compel the owner of the land to sell such land to the former." 4. Builder in good faith In Paz Mercado, et al. v. Hon. Court of Appeals, et al., (GR L-44001, 10 June 1988), it was held that "to be deemed a builder in good faith, it is essential that a person assert title to the land on which he builds; i.e., that he be a possessor in concept of owner, (Art. 525, Civil Code; Lopez, Inc. v. Phil. Eastern Trading Co., Inc., 98 Phil. 348) and that he be unaware 'that there exists in his title or mode of acquisition any flaw which invalidates it.' (Art. 526, Civil Code; Granados v. Monton, 86 Phil. 42; Arriola v. Gomez de la Serna, 14 Phil. 627; See also Manotok Realty, Inc. v. C.A., 134 SCRA 329, citing Caram v. Laureta, 103 SCRA 7) It is such a builder in good faith who is given the right to retain the thing, even as against the real owner, until he has been reimbursed in full not only for the necessary expenses but also for useful expenses. (Art. 546, Civil Code; Policarpio v. CA., 129 SCRA 51; Sarmiento v. Agana, 129 SCRA 122; cf, Queto v. C.A. ,122 SCRA 206)" 5. Good faith cease after filing of the complaint In Mindanao Academy, Inc. v. Yap (13 SCRA 190, 196), it was held that "although the bad faith of one party neutralizes that of the other and hence as between themselves their rights would be as if both of them had acted in good faith at the time of the transaction, this legal fiction of (Yap)'s good faith ceased when the complaint against him was filed, and consequently the court's declaration of liability for the rents thereafter is correct and proper. A possessor in good faith is entitled to the fruits only so long as his possession is not legally interrupted, and such interruption takes place upon service of judicial summons (Arts. 544 and 1123, Civil Code)." 6. Repairs and improvements introduced after the filing of the complaint in bad faith The repairs and improvements introduced by the builder after the complaint was filed cannot be considered to have been built in good faith, much less, justify the denial of the landowner's exercise of option. 7. Improvements gutted by fire, builders right to retain extinguished Since the improvements have been gutted by fire, and therefore, the basis for the builder's right to retain the premises has already been extinguished without the fault of the landowner, there is no other recourse for the builder but to vacate the premises and deliver the same to the landowner. Mercado v. CA [G.R. No. L-44001. June 10, 1988.] First Division, Narvasa (J): 4 concur Facts: Lolita C. Bulaong, Florentino Agulto, Severino Salaysay, Susana Bernardino (the Bulaong Group), from 1956 to 1972, have been individual lessees of stalls in the public market of Baliuag, Bulacan. The market was destroyed by fire on 17 February 1956; the members of the Bulaong Group constructed new stalls therein at their expense; and they thereafter paid rentals thereon to the Municipality of Baliuag. In 1972, the members of the group sub-leased their individual stalls to other persons: Paz Mercado, Carolina S. Chico, Luciana Cabrera, Joaquin Ignacio, Elmer Flores, and Avelina C. Nucom (the Mercado Group). After the Mercado Group had been in possession of the market stalls for some months, the municipal officials of Baliuag cancelled the long standing leases of the Bulaong Group and declared the persons comprising the Mercado Group as the rightful lessees of the stalls in question, in substitution of the former. The municipal authorities justified the cancellation of the leases of the Bulaong Group by invoking the provisions of Municipal Ordinance 14, dated 14 December 1964, which prohibited the sub-leasing stalls by the lessees thereof, as well as a 29 May 1973 directive of the Office of the President requiring enforcement of said Ordinance 14. Recognition of the Mercado Group's rights over the stalls was subsequently manifested in Municipal Ordinance 49, approved on 5 July 1973. The members of the Bulaong Group sued. They filed several individual complaints with the CFI seeking recovery of their stalls from the Mercado Group as well as damages. Their theory was anchored on their claimed ownership of the stalls constructed by them at their own expense, and their resulting right, as such owners, to sub-lease the stalls, and necessary, to recover them from any person withholding possession thereof from them. The Mercado Group thereafter filed motions for summary judgment, asserting that in light of the admissions made at the pre-trial and in the pleadings, no issue remained under genuine controversion, to which the Bulaong Group objected. Assuming the indifference of the Court and the other group to its opposition, the former presented affidavits and depositions to prove the value of the improvements, for which they were seeking reimbursement, to which the Mercado Group did not respond. On 24 October 1975, Judge Benigno Puno rendered a summary judgment in all the cases. It rejected the claim of the Municipality of Baliuag that it had automatically acquired ownership of the new stalls constructed after the old stalls had been razed by fire, declaring the members of the Bulaong Group to be builders in good faith, entitled to retain possession of the stalls respectively put up by them until and unless indemnified for the value thereof The decision also declared that the Bulaong and Mercado Groups had executed the sub-letting agreements with full awareness that they were thereby violating Ordinance 14; they were thus in pari delicto, and hence had no cause of action one against the other and no right to recover whatever had been given or demand performance of anything undertaken. The Mercado Group and the Municipality filed on 14 November 1975, motions for reconsideration of the summary judgment, notice of which had been served on them on 3 November 1975. These were denied, and notice of the order of denial was received by them on 18 December 1975. On 7 January 1976, the Mercado Group filed a notice of appeal, an appeal bond and a motion for extension of time to file their record on appeal. But by Order dated 9 January 1976, the Trial Court directed inter alia the execution of the judgment, adjudging that its decision had become final because the appeal documents had "not been seasonably filed." The writ was issued, and the Mercado Group's motion to quash the same and to re-open the case was denied. The Group went to the Court of Appeals, instituting in that court a special civil action of certiorari and prohibition "to annul that portion of the summary judgment awarding damages to the Bulaong Group and to restrain the Judge and the Provincial Sheriff of Bulacan from enforcing the same." The appellate Court rendered judgment on 14 May 1976 affirming the judgment of the lower court, holding that the judgment had become final and certiorari or prohibition could not be availed of as a substitute for the

group's lost appeal. Once again, the Mercado Group moved for reconsideration of an adverse judgment, and once again were rebuffed. An appeal by way of certiorari was filed before the Supreme Court. The Supreme Court dismissed the petition and affirmed the appealed judgment, with costs against the Mercado Group. 1. Court bound by factual holding of the CA, unless there are reasons to depart Upon the factual findings of the Court of Appeals, by which this court is bound, and taking account of well established precedent from which there is no perceivable reason in the premises to depart, there is no question that the the Mercado Group had failed to perfect an appeal from the summary judgment within the reglementary period fixed by the Rules of Court. The record discloses that they received copy of the summary judgment on 3 November 1975; that on 14 November 1975, or after the lapse of 11 days from receipt of said decision, they filed their motion for reconsideration of said decision; that on 18 December 1975, they received copy of the order denying their motion for reconsideration; and that they did not file their notice of appeal, appeal bond and motion for extension of time until 7 January 1976, or 20 days after receipt of the order denying their motion for reconsideration. The notice of appeal, appeal bond and motion for extension were, therefore, presented 1 day after the expiration of the 30-day period to perfect an appeal. 2. Computation of 30-day reglementary period In accordance with Section 3, Rule 41 of the Rules of Court, the 30-day reglementary period of appeal shall be deducted the "time during which a motion to set aside the judgment or order or for a new trial has been pending." 3. Jurisdiction of the lower court; Parties recognized jurisdiction when they filed pleadings Under the law, the Trial Court validly acquired jurisdiction not only over the persons of the parties but also over the subject matter of the actions at bar. The parties composing the Mercado Group cannot dispute this; they recognized the Court's competence when they filed their answers to the complaints without questioning the Court's jurisdiction of the subject-matter; indeed neither at that time nor at any other time thereafter did any one of them ever raise the question. 4. Jurisdiction not lost by any error in the exercise thereof committed by the court; Erroneous judgment does not render decision void; Error of judgment reviewable by appeal, not by special action of certiorari or prohibition Jurisdiction, once acquired, is not lost by any error in the exercise thereof that might subsequently be committed by the court. Where there is jurisdiction over the person and the subject matter, the decision of all other questions arising in the case is but an exercise of that jurisdiction. And when a court exercises its jurisdiction, an error committed while engaged in that exercise does not deprive it of the jurisdiction being exercised when the error is committed. If it did, every error committed by a court would deprive it of jurisdiction and every erroneous judgment would be a void judgment. This, of course, cannot be allowed. The administration of justice would not survive such a rule. Moreover, any error that the Court may commit in the exercise of its jurisdiction, being merely an error of judgment, is reviewable only by appeal, not by the special civil action of certiorari or prohibition. 5. Peculiar scenario; Summary judgment requested by the Mercado Group The petitioners (Mercado Group) were the parties who, as defendants, had moved for summary judgment. They knew or were supposed to know that, as stated by the Rules, their motion would be granted if "the pleadings, depositions, and admissions on file, together with the affidavits, show that, except as to the amount of damages, there is no genuine issue as to any material fact and that they are entitled to a judgment as a matter of law." They knew that the Bulaong Group, had in fact opposed their motion and had pointed out precisely the need for a hearing on the controverted matter of damages. That they did not join in the move to have a hearing on the issue of damages is an indication that they considered it unnecessary. When the Bulaong Group, apparently in view of the Court's and the Mercado Group's indifference to the notion of having a hearing on the matter of damages implicitly indicating the belief of the superfluity of a hearing, presented affidavits and depositions to prove the value of the improvements, for which they were seeking reimbursement, the Mercado Group did not ask that the matter be ventilated at a hearing, or submit counter-affidavits, as was their right. They made no response whatever. They were evidently quite confident of obtaining a favorable judgment, and that such an eventuality would preclude the claimed reimbursement or recovery of damages. As it turned out, they were wrong in their prognostication. 6. Rule in summary judgment It is only the ascertainment of the character of the issues raised in the pleadings as genuine, or sham or fictitious which can be done by depositions, admissions, or affidavits; the resolution of such issues as are found to be genuine should be made upon proof proferred at a formal hearing. The peculiar circumstances of the present case operate to exclude it from the scope of the rule. It is an exception that should however be taken, as affirming and not eroding the rule. 7. Lessees cannot be considered builders in good faith The members of the Bulaong group were admittedly lessees of space in the public market; they therefore could not, and in truth never did make the claim, that they were owners of any part of the land occupied by the market so that in respect of any new structure put up by them thereon, they could be deemed builders in good faith (in accordance with Article 526 of the Civil Code). To be deemed a builder in good faith, it is essential that a person assert title to the land on which he builds; i.e., that he be a possessor in concept of owner, and that he be unaware "that there exists in his title or mode of acquisition any flaw which invalidates it. It is such a builder in good faith who is given the right to retain the thing, even as against the real owner, until he has been reimbursed in full not only for the necessary expenses but also for useful expenses. On the other hand, unlike the builder in good faith, a lessee who "makes in good faith useful improvements which are suitable to the use for which the lease is intended, without altering the form or substance of the property leased," can only claim payment of "one-half of the value of the improvements" or, "should the lessor refuse to reimburse said amount, remove the improvements, even though the principal thing may suffer damage thereby." 8. Summary judgment a final judgment, not an interlocutory disposition or order The summary judgment rendered on 24 October 1975 was not an interlocutory disposition or order but a final judgment within the

meaning of Section 2, Rule 41 of the Rules of Court. By that summary judgment the Court finally disposed of the pending action, leaving nothing more to be done by it with respect to the merits, thus putting an end to the litigation as its level. 9. Remedy to a final judgment is an appeal The remedy available against a final judgment was an appeal in accordance with Rule 41 of the Rules of Court. The availability of the right of appeal obviously precluded recourse to the special civil action of certiorari. This is axiomatic. It is a proposition made plain by Section 1 of Rule 65 which lays down as a condition for the filing of a certiorari petition that there be 'no appeal, nor any plain, speedy and adequate remedy in the ordinary course of law. The remedy to obtain a reversal or modification of the judgment on the merits is appeal. This is true even if the error, or one of the errors, ascribed to the Court rendering the judgment is its lack of jurisdiction of the subject matter, or the exercise of power in excess thereof, or grave abuse of discretion in the findings of fact or of law set out in its decision. The existence and availability of the right of appeal proscribes a resort to certiorari, one of the requisites for availment of the latter remedy being precisely that "there should be no appeal." There may to be sure, be instances when certiorari may exceptionally be permitted in lieu of appeal, as when their appeal would be inadequate, slow, insufficient, and will not promptly relieve a party from the injurious effect of the judgment complained of, or to avoid future litigations, none of which situations obtains in the present case. And certain it is that the special civil action of certiorari cannot be a substitute for appeal, specially where the right to appeal has been lost through a party's fault or inexcusable negligence. The judgment became final and has thus gone beyond the reach of any court to modify in any substantive aspect. 10. Judgment contrary to express provisions which becomes final and executory, binding and effective as any valid judgment That the judgment of the Trial Court applied the wrong provision of the law in the resolution of the controversy has ceased to be of any consequence, i.e. instead of the legal provision governing lessees' rights over improvements on leased realty, the judgment invoked that relative to the rights of builders in good faith. A judgment contrary to the express provisions of a statute is of course erroneous, but it is not void; and if it becomes final and executory, it becomes as binding and effective as any valid judgment; and though erroneous, will henceforth be treated as valid, and will be enforced in accordance with its terms and dispositions. Tan Queto v. CA [G.R. No. L-35648. February 27, 1987.] Resolution En Banc, Paras (J): 10 concur, 1 votes to deny reconsideration, 1 dissents, 2 took no part Facts: Restituta Tagalinar Guangco de Pombuena received the questioned lot (Lot 304-B of the Cadastre Survey of the Municipality of Centro, Misamis Occidental) either as a purported donation or by way of purchase on 11 February 1927 for P50.00 as the alleged consideration thereof. The transaction took place during her mother's lifetime (her father having predeceased the mother) and consummated while Restituta was already married to her husband Juan Pombuena. On 22 January 1935, Juan filed an application of Torrens title over the land for himself and his supposed co-owner Restituta. On 22 November 1938, a decision was promulgated (GLRC 1638, Cadastral Case 12) pronouncing Juan (married to Restituto) as the owner of the land. On 22 September 1949 a contract of lease over the lot was entered into between Pershing Tan Queto and Restituta (with the consent of her husband) for a period of 10 years. Meanwhile, On 27 December 1960 Restituta sued Tan Queto for unlawful detainer (the lease contract having expired) before the Municipal Court of Ozamis City. On 22 April 1962, as a consequence of the cadastral case, an OCT was issued in Juans name. On 10 October 1962, Tan Queto and Juan entered into a barter agreement whereby Tan Queto became the owner of the disputed lot, and the spouses in turn became the owners of a parcel of land with the house constructed thereon previously owned (that is, before the barter) by Tan Queto. Thereafter, Tan Queto constructed on the disputed land a concrete building, without any objection on the part of Restituta. The Municipal court ruled in favor of the spouses in the unlawful detainer case; but on appeal in the CFI, the entire case was dismissed because of an understanding (barter) entered into by Juan and Tan Queto. Restituta sued both Juan and Tan Queto for reconveyance of the title over the registered but disputed lot, for annulment of the barter, and for recovery of the land with damages. The CFI and the Court of Appeals found the disputed lot as paraphernal and that Tan Queto was a builder in bad faith. These findings were regarded by the Supreme Court as findings of facts and thus ordinarily conclusive upon the Court. Tan Queto filed for a motion for reconsideration of the Supreme Court decision dated 16 May 1983. The Supreme Court set aside its decision promulgated on 16 May 1983, and rendered a new one declaring the questioned lot together with the building thereon, as Tan Queto's exclusive property; without costs. 1. Findings of the lower courts ordinary conclusive upon the Court; exception, if erroneous The findings of the Court of First Instance and the Court of Appeals were regarded by the Supreme Court as findings of facts and thus ordinarily conclusive upon the Court. Assuming they are factual findings, still if they are erroneous inferences from certain facts, they cannot bind the Court. 2. Land not transferred to Restituta by donation, for it to be paraphernal The oral donation of the lot cannot be a valid donation inter-vivos because it was not executed in a public instrument (Art. 749, Civil Code), nor as a valid donation mortis causa for the formalities of a will were not complied with. The allegation that the transfer was a conveyance to Restituta of her hereditary share in the estate of her mother (or parents) cannot be sustained for the contractual transmission of future inheritance is generally prohibited. 3. Land is conjugal, not paraphernal; Ownership by tradition The land is conjugal, not paraphernal. Ownership was acquired by the spouses by tradition (delivery) as a consequence of the

contract of sale (See Art. 712, Civil Code) with P50.00 (then a considerable amount) as the cause or consideration of the transaction. The lot is therefore conjugal, having been acquired by the spouses thru onerous title (the money used being presumably conjugal, there being no proof that Restituta had paraphernal funds of her own). 4. Sale not fictitious nor simulated; Allegation of simulation cannot prejudice a stranger The sale cannot be said to be fictitious or simulated (and therefore void) as there was a valid consideration therefor. Assuming that there had indeed been a simulation, the parties thereto cannot use said simulation to prejudice a stranger to said strategem (like petitioner herein). 5. Tan Queto recognized Restituta as an owner, not the owner Tan Queto admitted Restituta was "an owner" (not the owner) of the lot in his Answer, and this is true, for she was a co-owner (with Juan, and therefore "an owner.") There is no admission of Restituta's exclusive ownership. 6. Assuming Tan Queto recognized Restituta as the owner; bad faith of one neutralizes the bad faith of the other Even assuming that despite registration of the lot as conjugal, Tan Queto nursed the belief that the lot was actually Restituta's (making him in bad faith), still Restitutas failure to prohibit him from building despite her knowledge that construction was actually being done, makes her also in bad faith. The net resultant of mutual bad faith would entitle Tan Qyeto to the rights of a builder in good faith (Art. 448, Civil Code), ergo, reimbursement should be given him if Restituta decides to appropriate the building for herself (Art. 448, Civil Code). 7. Tan Queto an owner-possessor Tan Queto having bartered his own lot and small house with the questioned lot with Juan (who has been adverted to by a court decision and by the OCT a conjugal owner) may be said to be the owner-possessor of the lot. Certainly he is not merely a possessor or builder in good faith (this phrase presupposes ownership in another); much less is he a builder in bad faith. He is a builder-possessor (jus possidendi) because he is the owner himself. 8. Jus possessionis, jus possidendi; good faith and bad faith The Chapter on Possession (jus possessionis, not jus possidendi) in the Civil Code refers to a possessor other than the owner. The difference between a builder (or possessor) in good faith and one in bad faith is that the former is not aware of the defect or flaw in his title or mode of acquisition while the latter is aware of such defect or flaw (Art. 526, Civil Code). But in either case there is a flaw or defect. In the present case, there is no such flaw or defect because it is Tan Queto himself (not somebody else) who is the owner of the property. Pleasantville Development v. CA [G.R. No. 79688. February 1, 1996.] Third Division, Panganiban (J): 4 concur, 1 took no part Facts: Edith Robillo purchased from Pleasantville Development Corporation a parcel of land designated as Lot 9, Phase II and located at Taculing Road, Pleasantville Subdivision, Bacolod City. Eldred Jardinico bought the rights to the lot from Robillo. At that time, Lot 9 was vacant. On the other hand, on 26 March 1974, Kee bought on installment Lot 8 of the same subdivision from C.T. Torres Enterprises, Inc. (CTTEI), the exclusive real estate agent of Pleasantville Development. Under the Contract to Sell on Installment, Kee could possess the lot even before the completion of all installment payments. On 20 January 1975, Kee paid CTTEI the relocation fee of P50.00 and another P50.00 on 27 January 1975, for the preparation of the lot plan. These amounts were paid prior to Kee's taking actual possession of Lot 8. After the preparation of the lot plan and a copy thereof to Kee, CTTEI through its employee, Zenaida Octaviano, accompanied Kee's wife, Donabelle Kee, to inspect Lot 8. Unfortunately, the parcel of land pointed by Octaviano was Lot 9. Thereafter, Kee proceeded to construct his residence, a store, an auto repair shop and other improvements on the lot. Upon completing all payments, Jardinico secured on 19 December 1978 from the Register of Deeds of Bacolod City TCT 106367 in his name. It was then that he discovered that improvements had been introduced on Lot 9 by Wilson Kee, who had taken possession thereof. After the discovery, Jardinico confronted Kee. The parties tried to reach an amicable settlement, but failed. On 30 January 1981, Jardinico's lawyer wrote Kee, demanding that the latter remove all improvements and vacate Lot 9. When Kee refused to vacate, Jardinico filed with the MTCC a complaint for ejectment with damages against Kee. Kee, in turn, filed a third-party complaint against Pleasantville and CTTEI. The MTCC held that the erroneous delivery of Lot 9 to Kee was attributable to CTTEI. The MTCC also found that Pleasantville had already rescinded its contract with Kee over Lot 8 for the latter's failure to pay the installments due, and that Kee had not contested the rescission. The rescission was effected in 1979, before the complaint was instituted. The MTCC concluded that Kee no longer had any right over the lot subject of the contract between him and Pleasantville. Consequently, Kee must pay reasonable rentals for the use of Lot 9, and, furthermore, he cannot claim reimbursement for the improvements he introduced on said lot. The MTCC thus ordered Kee to vacate Lot 9, to remove all structures and improvements he introduced thereon, and to pay the Jardinico rentals of P15.00 a day computed from the time the suit was filed on 12 March 1981 until he vacateds the premises; such amount bearing an interest of 12% per annum. The MTCC also ordered CTTI and Pleasantville to pay Jardinico in solidum for the amount of P3,000 as attorneys fees and P700 as cost and litigation expenses. On appeal, the RTC Bacolod City (Branch 48) ruled that Pleasantville and CTTEI were not at fault or were not negligent and found Kee a builder in bad faith. Thus, the appellate court affirmed the decision with respect to the order to vacate the premises of Lot 9, the removal of the structure and improvements introduced thereon at Kees expense, and to pay a rental of P15.00 a day until he vacates the premises, with an interest of 12% per annum. The Court further rendered judgment against Kee to pay Jardinico the sum of P3,000.00 as attorney's fees, plus costs of litigation; dismissed the third-party complaint against Pleasantville CTTEI, and reversed the order Pleasantville and CTTEI to pay Jardinico attorney's fees and costs of litigation. Following the

denial of his motion for reconsideration on 20 October 1986, Kee appealed directly to the Supreme Court, which referred the matter to the Court of Appeals. Pending resolution of the case before the Court of Appeals, Jardinico and Kee on 24 July 1987 entered into a deed of sale, wherein the former sold Lot 9 to Kee. Jardinico and Kee did not inform the Court of Appeals of such deal. The appellate court ruled that Kee was a builder in good faith (entitled to rights under Articles 448, 546 and 548 of the Civil Code), as he was unaware of the "mix-up" when he began construction of the improvements on Lot 8. It further ruled that the erroneous delivery was due to the negligence of CTTEI, and that such wrong delivery was likewise imputable to its principal, Pleasantville; and thus ordered the CTTEI and Pleasantville to be solidarily liable for the demolition expenses and value of improvements destroyed or rendered useless in case Jardinico decides to appropriate the improvements and thereafter remove the structures; or for the amount representing the value of Lot 9 that Kee should pay to Jardinico if Jardinico chose to sell the land to Kee. The appellate court ordered CTTEI and Pleasantville to pay in solidum the amount of P3,000.00 to Jardinico as attorney's fees, as well as litigation expenses; and ruled that the award of rentals was without basis. Further, the appellate court remanded the case to the court of origin for the determination of the actual value of the improvements and the property (Lot 9). Pleasantville filed the petition for review on certiorari. The Supreme Court partially granted the petition, and modified the decision of the Court of Appeals by declaring Wilson Kee a builder in good faith; and that Pleasantville Development and C.T. Torres Enterprises solidarily liable for damages due to negligence (however, since the amount and/or extent of such damages was proven during the trial, the same cannot now be quantified and awarded). The Court also ordered Pleasantville Development and C.T. Torres Enterprises to pay in solidum the amount of P3,000.00 to Jardinico as attorney's fees, as well as litigation expenses. The Court dispensed with the award of rentals to Jardinico. 1. Kee a builder in good faith; Prudent acts to ascertain land to build on Under the Torrens system of land registration, Kee is presumed to have knowledge of the metes and bounds of the property with which he is dealing. But as Kee is a layman not versed in the technical description of his property, he had to find a way to ascertain that what was described in TCT 69561 matched Lot 8. Thus, he went to the subdivision developer's agent and applied and paid for the relocation of the lot, as well as for the production of a lot plan by CTTEI's geodetic engineer. Upon Kee's receipt of the map, his wife went to the subdivision site accompanied by CTTEI's employee, Octaviano, who authoritatively declared that the land she was pointing to was indeed Lot 8. Having full faith and confidence in the reputation of CTTEI, and because of the company's positive identification of the property, Kee saw no reason to suspect that there had been a misdelivery. The steps Kee had taken to protect his interests were reasonable. There was no need for him to have acted ex-abundantia cautela. such as being present during the geodetic engineer's relocation survey or hiring an independent geodetic engineer to countercheck for errors, for the final delivery of subdivision lots to their owners is part of the regular course of everyday business of CTTEI. Because of CTTEI's blunder, what Kee had hoped to forestall did in fact transpire. Kee had acted in the manner of a prudent man in ascertaining the identity of his property. 2. Scenario of bad faith improbable; Good faith presumed It is thus highly improbable that a purchaser of a lot would knowingly and willingly build his residence on a lot owned by another, deliberately exposing himself and his family to the risk of being ejected from the land and losing all improvements thereon, not to mention the social humiliation that would follow. Good faith consists in the belief of the builder that the land he is building on is his and his ignorance of any defect or flaw in his title. And as good faith is presumed, the one alleging bad faith has the burden of proving bad faith. 3. Contractual breach cannot be the basis to negate the presumption of builder in good faith Violations of paragraphs 22 and 26 of the Contract of Sale on Installment have no bearing whatsoever on whether Kee was a builder in good faith, i.e. on his state of mind at the time he built the improvements on Lot 9. These alleged violations may give rise to Pleasantville's cause of action against Kee under the said contract (contractual breach), but may not be bases to negate the presumption that Kee was a builder in good faith. 4. Rescission does not negate the negligence of CTTEI The Contract of Sale on Installment covering Lot 8 between Pleasantville and Kee, which was rescinded long before the present action was instituted, has no relevance on the liability of Pleasantville, as such fact does not negate the negligence of its agent in pointing out the wrong lot to Kee. Such circumstance is relevant only as it gives Jardinico a cause of action for unlawful detainer against Kee. 5. Recovery of damages not waived Kee did not contracted away his right to recover damages resulting from Pleasantville's negligence. Such waiver would be contrary to public policy and cannot be allowed. "Rights may be waived, unless the waiver is contrary to law, public order, public policy, morals, or good customs, or prejudicial to a third person with a right recognized by law." 6. Principal responsible for acts of agent if damaged caused to third persons; Agent is personally liable for damages if he exceeds his authority The principal is responsible for the acts of the agent, done within the scope of his authority, and should bear the damage caused to third persons. On the other hand, the agent who exceeds his authority is personally liable for the damages. In the present case, CTTEI was acting within its authority as the sole real estate representative of Pleasantville when it made the delivery to Kee. In acting within its scope of authority, it was, however, negligent. It is this negligence that is the basis of Pleasantville's liability, as principal of CTTEI, per Articles 1909 and 1910 of the Civil Code. 7. Deed of Sale between Kee and Jardinico merely regulates the reciprocal rights of the parties and has no effect on the liability of Pleasantville

The deed of sale regulates the reciprocal rights of Kee and Jardinico; it stressed that they had reached an agreement independent of the outcome of the case. The "terms and conditions in the said deed of sale are strictly for the parties thereto" and that "there is no waiver made by either of the parties in said deed of whatever favorable judgment or award the Court of Appeals may make in Kees and Jardinicos favor against Pleasantville and CTTEI. The deed of sale can have no effect on the liability of Pleasantville. Pleasantville's liability is grounded on the negligence of its agent. 8. Pleasantville supposedly liable for damages due to agents negligence; Due to lack of evidence, no damages due Pleasantvilles liability lies in the negligence of its agent CTTEI. For such negligence, Pleasantvilles should be held liable for damages. The extent and/or amount of damages to be awarded is a factual issue which should be determined after evidence is adduced. However, there is no showing that such evidence was actually presented in the trial court; hence no damages could be awarded. 9. Appellate court erred in modification of the application of the law on ground of equity The rights of Kee and Jardinico vis-a-vis each other, as builder in good faith and owner in good faith, respectively, are regulated by law (i.e., Arts. 448, 546 and 548 of the Civil Code). It was error for the Court of Appeals to make a "slight modification" in the application of such law, on the ground of "equity". At any rate, Kee and Jardinico have amicably settled through their deed of sale their rights and obligations with regards to Lot 9. Thus, the Court deleted the dispositive portion of the Court of Appeals' Decision holding Pleasantville and CTTEI solidarily liable for demolition expenses or the amount pertaining to the value of the lot, whichever is applicable in the exercise of the landowners options. 10. Award of attorneys fees lies with the discretion of the court depending on the cases circumstances The award of attorney's fees lies within the discretion of the court and depends upon the circumstances of each case. The Supreme Court shall not interfere with the discretion of the Court of Appeals. Jardinico was compelled to litigate for the protection of his interests and for the recovery of damages sustained as a result of the negligence of Pleasantvile's agent. 11. No need to remand the case for the determination of the value and the land In view of the deed of sale entered into by Kee and Jardinico, which deed governs the rights of Jardinico and Kee as to each other, there is also no further need to remand the case to the court of origin "for determination of the actual value of the improvements and the property (Lot 9), as well as for further proceedings in conformity with Article 448 of the New Civil Code." Baltazar v. Caridad [G.R. No. L-23509. June 23, 1966.] En Banc, Reyes JBL (J): 8 concur Facts: In the cadastral proceeding (Cadastral Case 54, GLRO Cadastral Case 1222), the trial court rendered decision, dated 23 January 1941, awarding Lot 8864 of the Laoag (Ilocos Norte) cadastre to the spouses Julio Baltazar and Constancia Valencia as their conjugal partnership property. Said decision having become final, the corresponding decree was issued on 12 July 1941, and pursuant thereto, said lot was registered in the names of applicant spouses under OCT O-1445, which was later transcribed, on 5 November 1959, in the office of the Register of Deeds of Ilocos Norte. In the meanwhile, Julio Baltazar, the registered owner of Lot 8864, died. On 6 December 1961, his surviving wife and children filed a motion, in the cadastral case praying for writ of possession against Silvina Caridad and her daughter, Eduarda Caridad, who had been in possession of the southern portion of said Lot 8864 since 1939, while the cadastral case involving said lot was pending before the trial court, and before the decision was rendered and the corresponding decree issued in 1941. No writ having theretofore been issued in Baltazar's favor, the trial court issued an order, on 11 December 1961, granting Baltazar's motion, and overruled Caridads opposition but directed the sheriff not to remove or destroy the permanent improvements on the lot without an express command. On 2 January 1962, the order having become final, the sheriff enforced the writ and placed Baltazar in possession of the southern portion of the lot. On 23 January 1962, Baltazar presented a motion to compel Eduarda and Silvina Caridad to remove their respective houses which they built in 1958 and 1959, respectively, in the southern portion of the disputed lot, and, in the event of their failure to do so, to order the sheriff to demolish the same. On 20 March 1962, the trial court, after due hearing, granted Baltazars motion, ordering the Caridads to remove their respective houses from the southern portion of said lot 8864 within 30 days from receipt of said order. Not satisfied, the Caridads appealed (CA-GR 31289-R). The appellate court, however, certified the appeal to the Supreme Court for raising only questions of law. The Supreme Court affirmed the appealed order; with costs against the Caridads. 1. Order compelling Caridads to remove their respective houses from the disputed lot; Lack of opposition in previous proceedings The order, dated 20 March 1962, of the cadastral court, granting Baltazar's motion to compel the Caridads to remove their respective houses from the disputed lot, is valid and enforceable against the latter. This may be concluded based on the circumstances that (1) that the Caridads do not dispute that during the pendency of the cadastral proceeding (to which judgment was rendered awarding said lot 8864, and consequent issuance of the final decree of registration of the same, in favor of Julio Baltazar), the late Andres Caridad, his surviving spouse Silvina Caridad, and their children, one of whom is Eduarda Caridad, were in possession of the southern portion of the disputed lot ; (2) that Eduarda Caridad claims right and title thereto as a mere heir and successor-in-interest of said Andres Caridad; and (3) that the Caridads do not dispute the propriety and validity of the order of the cadastral court, granting the writ of possession in favor of Baltazar as well as its enforcement. 2. Jurisdiction of the CFI, sitting as a land registration court, to issue writ of possession and order demolition of improvements In Marcelo vs. Mencias, etc., et al. (L-15609, 29 April 1960, 58 OG 3349), the Court had already upheld the jurisdiction or

authority of the CFI, sitting as a land registration court, to order, as a consequence of the writ of possession issued by it, the demolition of improvements introduced by the successor-in-interest of a defeated oppositor in the land registration case. 3. Section 13, Rule 39 of the Rules of Court Section 13, Rule 39 of the Rules of Court (How execution for the delivery or restitution of property enforced), provides that the officer must enforce an execution for the delivery or restitution of property by placing the plaintiff in possession of such property, and by levying as hereinafter provided upon so much of the property of the judgment debtor as will satisfy the amount of costs, damages, rents, and profits included in the execution. However, the officer shall not destroy, demolish or remove the improvements made by the defendant or his agent on the property, except by special order of the court, which order may only issue upon petition of the plaintiff after due hearing and upon the defendant's failure to remove the improvements within a reasonable time to be fixed by the court. 4. Rule applies to land registration cases in a suppletory character and not only to ordinary actions involving the delivery or restitution of property The provision of the Rules of Court applies not only to ordinary actions involving the delivery or restitution of property, but also to proceedings under the land registration law. The provisions of the Rules of Court are applicable to land registration cases in a suppletory character (Rule 132). Thus, if the writ of possession issued in a land registration proceeding implies the delivery of possession of the land to the successful litigant therein (Demorar vs. Ibaez, 51 OG 2872, Pasay Estate Company vs. Del Rosario, et al., 11 Phil. 391; Manlapas vs. Llorente, 48 Phil. 298), a writ of demolition must, likewise, issue, especially considering that the latter writ is but a complement of the former which without said writ of demolition would be ineffective. To require a successful litigant in a land registration case to institute another action for the purpose of obtaining possession of the land adjudged to him, or for the purpose of securing fruits of his victory, would be a cumbersome process. It would foster unnecessary and expensive litigations and result in multiplicity of suits, which the judicial system abhors. 5. Power of a judge to issue all auxiliary writs to carry into effect the jurisdiction conferred upon the court by law Section 6, Rule 124, of the Rules of Court provides that when by law jurisdiction is conferred on a court or judicial officer, all auxiliary writs, processes and other means necessary to carry it into effect may be employed by such court or officer; and if the procedure to be followed in the exercise of such jurisdiction is not specifically pointed out by these rules, any suitable process or mode of proceeding may be adopted which appears most conformable to the spirit of said rules. In the present case, the Judge has the power to issue all auxiliary writs, including the writ of demolition, processes and other means necessary to carry into effect the jurisdiction conferred upon it by law in land registration cases to issue a writ of possession to the successful litigant. 6. Every court has inherent power to do necessary thing s for the administration of justice In Shioji vs. Harvey, 43 Phil 333, it was pointed out that independent of any statutory provision, every court has inherent power to do all things reasonably necessary for the administration of justice within the scope of its jurisdiction. In line with this doctrine, the Judge has the inherent power to issue the writ of demolition. Its issuance is reasonably necessary to do justice to petitioner who is being deprived of the possession of the lots in question, by reason of the continued refusal of the respondent to remove his house thereon and restore possession of the premises to petitioner. 7. Builders in bad faith; Rights of registered owner cannot be defeated by an unsuccessful opponent through the subterfuge of replacing his old house with a new one from time to time The Caridads cannot be regarded as builders in good faith because they are bound by the 1941 decree of registration that obligated their parents and predecessors-in-interest. Good faith must rest on a colorable right in the builder, beyond a mere stubborn belief in one's title despite judicial adjudication. The fact that in 1959 the Caridads demolished and replaced their old house with new and bigger ones cannot enervate the rights of the registered owners. Otherwise, the rights of the latter to enjoy full possession of their registered property could be indefinitely defeated by an unsuccessful opponent through the simple subterfuge of replacing his old house with a new one from time to time. Balucanag v. Francisco [G.R. No. L-33422. May 30, 1983.] Second Division, Escolin (J): 4 concur, 1 took no part Facts: Cecilia dela Cruz Charvet was the owner of a 177.50 sq.m. lot located in Zamora Street, Pandacan, Manila (TCT 25664). On 31 August 1952, Mrs. Charvet leased said lot to Richard Stohner for a period of 5 years at the monthly rental of P40.00, payable in advance within the first 10 days of each month. The lease contract provided, among others, that the lessee may erect such buildings upon and make such improvements to the leased land as he shall see fit. All such buildings and improvements shall remain the property of the lessee and he may remove them at any time, it being agreed, however, that should he not remove the said buildings and improvements within a period of two months after the expiration of this Agreement, the Lessor may remove the said buildings and improvements or cause them to be removed at the expense of the Lessee." During the existence of the lease, Stohner made fillings on the land and constructed a house thereon, said improvements being allegedly valued at P35,000.00. On 8 March 1966, Mrs. Charvet sold the said lot to Rosendo Balucanag. For Stohner's failure to pay the rents, Balucanag, thru counsel, wrote Stohner a letter demanding that he vacate the premises. In reply thereto, Stohner, also thru counsel, claimed that he was a builder in good faith of the residential house erected in the land. He offered proposals for a possible compromise, either for Stohner would purchase the lot at 12% per annum on the value or that Balucanag reimburse Stohner the amount P35,000 for the improvements and constructions he has made on the lot. As no agreement was reached, Balucanag instituted in the City Court of Manila an ejectment suit against Stohner and after due trial, the court rendered a decision, ordering Stohner to pay Baculanag the sum of P360.00 as back rentals from December 1965 to August 1966, to vacate the premises and the sum of P100 as attorneys fees. On appeal, the CFI Manila, Branch IX, after conducting a trial de novo, rendered a decision, setting aside the judgment of the city court and dismissing Baculanag's complaint. The CFI held that Stohner was a builder in good faith because he had constructed the residential house with the consent of the original lessor, Mrs. Charvet, and also because the latter, after the expiration of the lease contract on 31 August 1957, had neither sought Stohner's ejectment from the premises, nor the removal of his house therefrom;

and thus, invoking Articles 448 and 546 of the Civil Code, Stohner cannot be ejected until he is reimbursed of the value of the improvements. Baculanag filed a motion for reconsideration, which the CFI denied. Baculanag filed the petition for review with the Supreme Court. The Supreme Court set aside the decision in Civil Case 67503, with costs against Stohner. The latter was ordered to vacate the premises in question and to pay Balucanag the rentals due from March 1969 up to the time he surrenders the premises, at the rate of P40.00 a month. 1. Lessee cannot be considered a builder in good faith Article 448 of the Civil Code applies only to a case where one builds on land in the belief that he is the owner thereof and it does not apply where one's only interest in the land is that of a lessee under a rental contract. In the present case, there is no dispute that the relation between Balucanag and Stohner is that of lessor and lessee, the former being the successor in interest of the original owner of the lot. 2. Possessor in good faith; Lopez v. Philippine and Eastern Trading In Lopez, Inc. vs. Phil. and Eastern Trading Co., Inc., it was held that the principle of possessor in good faith refers only to a party who occupies or possess property in the belief that he is the owner thereof and said good faith ends only when he discovers a flaw in his title so as to reasonably advise or inform him that after all he may not be the legal owner of said property. It cannot apply to a lessee because as such lessee he knows that he is not the owner of he leased premises. Neither can he deny the ownership or title of his lessor. A lessee who introduces improvements in the leased premises, does so at his own risk in the sense that he cannot recover their value from the lessor, much less retain the premises until such reimbursement. " 3. Article 1678 is law applicable; Options of the lessor and the lessee The law applicable to the present case is Article 1678 of the Civil Code, which provides that if the lessee makes, in good faith, useful improvements which are suitable to the use for which the lease is intended, without altering the form or substance of the property leased, the lessor upon the termination of the lease shall pay the lessee of the value of the improvements at the time. Should the lessor refuse to reimburse said amount, the lessee may remove the improvements, even though the principal thing may suffer damage thereby. He shall not, however, cause any more impairment upon the property leased than is necessary. " The article gives the lessor the option to appropriate the useful improvements by paying of their value, and the lessee cannot compel the lessor to appropriate the improvements and make reimbursement, for the lessee's right under the law is to remove the improvements even if the leased premises may suffer damage thereby. But he shall not cause any more damage upon the property than is necessary. 4. Implied new lease (tacit recondition) created when lessee continued in possession of the premises with the acquiescence of the lessor; Period fixed according to Article 1687 While the lease contract entered into expired on 31 August 1957, the lessee nevertheless continued in possession of the premises with the acquiescence of the lessor. An implied new lease or tacita recondition was thus created between the parties, the period of which is established by Article 1687 of the Civil Code (If the period for the lease has not been fixed, it is understood to be from year to year, if the rent agreed upon is annual; from month to month, if it is monthly; from week to week, if the rent is weekly; and from day to day, if the rent is to be paid daily"). Thus, the duration of the new lease must be deemed from month to month, the agreed rental in the instant case being payable on a monthly basis. The lessor may thus terminate the lease after each month with due notice upon the lessee. After such notice, the lessee's right to continue in possession ceases and his possession becomes one of detainer. Furthermore, the lessee's failure to pay the stipulated rentals entitles lessor to recover possession of the premises. Geminiano v. CA [G.R. No. 120303. July 24, 1996.] Third Division, Davide Jr (J): 4 concur Facts: Lot 3765-B-1 (314 sq. m.) was originally owned by Paulina Amado vda. de Geminiano, the mother of Federico, Maria, Ernesto, Asuncion, Larry and Marlyn Geminiano. On a 12-sq. m. portion of that lot stood the Geminianos' unfinished bungalow, which the Geminianos sold in November 1978 to Dominador and Mary Nicolas for the sum of P6,000.00, with an alleged promise to sell to the latter that portion of the lot occupied by the house. Subsequently, Paulina Amado-Geminiano executed a contract of lease over a 126 sq. m. portion of the lot, including that portion on which the house stood, in favor of the Nicolas spouse for P40 per month for a period of 7 years commencing on 15 November 1978. The Nicolas spouses then introduced additional improvements and registered the house in their names. After the expiration of the lease contract in November 1985, however, the Paulina refused to accept the monthly rentals. It turned out that the lot in question was the subject of a suit, which resulted in its acquisition by one Maria Lee in 1972. In 1982, Lee sold the lot to Lily Salcedo, who in turn sold it in 1984 to the spouses Agustin and Ester Dionisio. On 14 February 1992, the Dionisio spouses executed a Deed of Quitclaim over the said property in favor of the Geminianos. As such, the lot was registered in the latter's names. On 9 February 1993, the Geminianos sent, via registered mail, a letter addressed to Mary Nicolas demanding that she vacate the premises and pay the rentals in arrears within 20 days from notice. Upon failure of the Nicolas spouses to heed the demand, the Geminianos filed with the MTCC of Dagupan City a complaint for unlawful detainer and damages. The trial court held that there was no lease to speak of to be renewed as the lot was acquired by Maria Lee in 1972, and that if indeed there is a legal lease existing, its renewal can only be made on a month-to-month pursuant to Article 1687 of the Civil Code; that the lessees were not builders in good faith and the reimbursement as such are governed by Article 1678; and that the value of the house and improvements was P180,000 as there was controverting evidence presented. The Court thus ordered the Nicolas spouses to vacate the premises, to pay the Geminianos P40 a month as reasonable compensation for their stay thereon from the filing of the complaint on 14 April 1993 until they vacated, and to pay the sum of P1,000 as attorney's fees, plus costs.

On appeal by the Nicolas spouses, the RTC Dagupan City reversed the trial court's decision and rendered a new judgment: (1) ordering the Geminianos to reimburse the Nicolas spouses for the value of the house and improvements in the amount of P180,000.00 and to pay the latter P10,000.00 as attorney's fees and P2,000.00 as litigation expenses; and (2) allowing the Nicolas spouses to remain in possession of the premises until they were fully reimbursed for the value of the house. It ruled that since the Nicolas spouses were assured by the Geminianos that the lot they leased would eventually be sold to them, they could be considered builders in good faith, and as such, were entitled to reimbursement of the value of the house and improvements with the right of retention until reimbursement had been made. On appeal, this time by Geminianos, the Court of Appeals affirmed the decision of the RTC and denied the Geminianos' motion for reconsideration. Hence, the petition for review on certiorari. The Supreme Court granted the petition; reversing and setting aside the decision of the Court of Appeals of 27 January 1995 in CA-GR SP 34337; and reinstating the decision of Branch 3 of the Municipal Trial Court in Cities of Dagupan City in Civil Case 9214; with costs against the Nicolas spouses. 1. Non-owner of the premises may lease property While the right to lease property is an incident of title and possession, a person may be a lessor and occupy the position of a landlord to the tenant although he is not the owner of the premises leased. After all, ownership of the property is not being transferred, only the temporary use and enjoyment thereof. 2. Nicolas spouses estopped; Estoppel applies even if lessor has no title, may be asserted not only by original lessor but also those who succeed to his title The Nicolas spouses came into possession of a 126 sq. m. portion of the said lot by virtue of a contract of lease executed by the Geminianos' mother in their favor. The juridical relation between the Geminianos' mother as lessor, and the Nicolas spouses as lessees, is therefore well-established, and carries with it a recognition of the lessor's title. The lessees who had undisturbed possession for the entire term under the lease, are then estopped to deny their landlord's title, or to assert a better title not only in themselves, but also in some third person while they remain in possession of the leased premises and until they surrender possession to the landlord. This estoppel applies even though the lessor had no title at the time the relation of lessor and lessee was created, and may be asserted not only by the original lessor, but also by those who succeed to his title. 3. Lessees not possessors not builders in good faith Being mere lessees, the Nicolas spouses knew that their occupation of the premises would continue only for the life of the lease. Plainly, they cannot be considered as possessors nor builders in good faith. 4. Article 448 in relation to Article 546 applies only to a possessor in good faith; does not apply to lessee Article 448 of the Civil Code, in relation to Article 546 of the same Code, which allows full reimbursement of useful improvements and retention of the premises until reimbursement is made, applies only to a possessor in good faith, i.e., one who builds on land with the belief that he is the owner thereof. It does not apply where one's only interest is that of a lessee under a rental contract; otherwise, it would always be in the power of the tenant to "improve" his landlord out of his property. 5. Alleged option to buy not supported by evidence; Promise unenforceable unless option is in writing Neither the deed of sale over the house nor the contract of lease contained an option in favor of the Nicolas spouses to purchase the said lot. The first thing that the spouses should have done was to reduce the alleged promise into writing, because under Article 1403 of the Civil Code, an agreement for the sale of real property or an interest therein is unenforceable, unless some note or memorandum thereof be produced. Not having taken any steps in order that the alleged promise to sell may be enforced, the private respondents cannot bank on that promise and profess any claim nor color of title over the lot in question. 6. Option does not render the Nicolas spouses builders in good faith Even if the Germinianos indeed promised to sell, it would not make the spouses possessors or builders in good faith so as to be covered by the provisions of Article 448 of the Civil Code. The latter cannot raise the mere expectancy of ownership of the lot because the alleged promise to sell was not fulfilled nor its existence even proven. 7. Pecson v. CA does not apply; No forced co-ownership There is no need to apply by analogy the provisions of Article 448 on indemnity as was done in Pecson vs. Court of Appeals, because the situation sought to be avoided and which would justify the application of that provision, is not present in the present case. "A state of forced co-ownership" would not be created between the Germinianos and the Nicolas spouses. 8. Lessees governed by Article 1678 The rights of the lessees are governed by Article 1678 of the Civil Code which allows reimbursement to the extent of one-half of the value of the useful improvements. The right to indemnity under Article 1678 of the Civil Code, however, arises only if the lessor opts to appropriate the improvements. Since the Germinianos refused to exercise that option, the Nicolas spouses cannot compel them to reimburse the one-half value of the house and improvements. Neither can they retain the premises until reimbursement is made. The spouses sole right then is to remove the improvements without causing any more impairment upon the property leased than is necessary. Del Campo v. Fernandez-Abesia [G.R. No. L-49219. April 15, 1988.] First Division, Gancayco (J): 4 concur Facts: Spouses Estanislao and Concepcion del Campo and Bernarda Abesia are co-owners pro indiviso of a lot (Lot 1161 of the Cadastral Survey of Cebu, 45 sq.m. at the corner of F. Flores and Cavan Streets, Cebu City; TCT 61850) in the proportion of 2/3 and 1/3 share each, respectively. The spouses filed an action for partition with the CFI Cebu. The trial court appointed a

commissioner in accordance with the agreement of the parties. The said commissioner conducted a survey, prepared a sketch plan and submitted a report to the trial court on 29 May 1976, recommending that the property be divided into two lots: Lot 1161-A (30 sq.m.) and Lot No. 1161-B (15 sq. m.). The houses of the spouses and Abesia were surveyed and shown on the sketch plan. Abesias house occupied the portion with an area of 5 sq.m. of the spouses Lot 1161-A. The parties manifested their conformity to the report and asked the trial court to finally settle and adjudicate who among the parties should take possession of the 5 sq. m. of the land in question. The lower court held that Article 448 of the New Civil Code does not apply in the case, and ordered Abesia to demolish at her own expense part of her house which has encroached Lot 1161-A and deliver said area to the spouses, with 60 days upon notice. The parties were ordered to pay the commissioners fee (P400 in proportional share), and the cost of suit (also in proportional share). Abesia appealed to the Court of Appeals, which certified the case to the Supreme Court on account of the question of law involved, the sole issue is the applicability of the provisions of Article 448 of the Civil Code relating to a builder in good faith when the property involved is owned in common. The Supreme Court modified the decision appealed from by ordering the spouses to indemnify Abesia for the value of the said portion of the Abesias house in accordance with Article 546 of the Civil Code, if the spouses elect to appropriate the same. Otherwise, Abesia shall pay the value of the 5 sq. m. of land occupied by their house at such price as may be agreed upon with the spouses and if its value exceeds the portion of the house that Abesia built thereon, Abesia may choose not to buy the land but Abesia must pay a reasonable rental for the use of the portion of the spouses land as may be agreed upon between the parties. In case of disagreement, the rate of rental shall be determined by the trial court. Otherwise, defendants may remove or demolish at their own expense the said portion of their house; Without costs. 1. Article 448 of the Civil Code does not apply when co-owner built, sown, or planted in good faith, as he is not a third party; Situation governed by co-ownership Article 448 of the Civil Code provides that the owner of the land on which anything has been built, sown, or planted in good faith, shall have the right to appropriate as his own the works, sowing or planting, after payment of the indemnity provided for in articles 546 and 548, or to oblige the one who built or planted to pay the price of the land, and the one who sowed, the proper rent. However, the builder or planter cannot be obliged to buy the land if its value is considerably more than that of the building or trees. In such case, he shall pay reasonable rent, if the owner of the land does not choose to appropriate the building or trees after proper indemnity. The parties shall agree upon the terms of the lease and in case of disagreement, the court shall fix the terms thereof." Article 448 of the Civil Code cannot apply where a co-owner builds, plants or sows on the land owned in common for then he did not build, plant or sow upon land that exclusively belongs to another but of which he is a co-owner. The co-owner is not a third person under the circumstances, and the situation is governed by the rules of co-ownership. 2. Article 448 of the Civil Code applies if co-ownership is terminated by the partition The provisions of Article 448 of the new Civil Code should apply when the co-ownership is terminated by the partition, as in the present case, and when it appears that the house of Abesia overlaps or occupies a portion of 5 sq. m. of the land pertaining to the spouses which Abesia obviously built in good faith. Manresa and Navarro Amandi agree that the said provision of the Civil Code may apply even when there was co-ownership if good faith has been established. 3. Options available to the parties Applying Article 448 of the Civil Code, the spouses have the right to appropriate said portion of the house of Abesia upon payment of indemnity to the latter as provided for in Article 546 of the Civil Code. Otherwise, the spouses may oblige Abesia to pay the price of the land occupied by their house. However, if the price asked for is considerably much more than the value of the portion of the Abesias house built thereon, then the latter cannot be obliged to buy the land. Abesia shall then pay the reasonable rent to the spouses upon such terms and conditions that they may agree. In case of disagreement, the trial court shall fix the terms thereof. Of course, Abesia may demolish or remove the said portion of their house, at their own expense, if they so decide. Ignao v. IAC [G.R. No. 72876. January 18, 1991.] Third Division, Fernan (J): 3 concur Facts: A lot situated in Barrio Tabon, Municipality of Kawit, Cavite, with an area of 534 sq. m. was originally owned by Baltazar Ignao who married twice. In his first marriage, he had 4 children, namely Justo (the father of Florencio), Leon, Juan and Isidro. In his second marriage, Baltazar had also 4 children but the latter waived their rights over the controverted land in favor of Justo. Thus, Justo owned 4/8 of the land which was waived by his half-brothers and sisters plus his 1/8 share or a total of 5/8. Thereafter, Justo acquired the 1/8 share of Leon for P500.00 which he later sold to his son Florencio for the same amount. When Justo died, Florencio inherited the 5/8 share of his father Justo plus his 1/8 share of the land which he bought or a total of 6/8 (representing 400.5 sq. m.) Juan and Isidro, on the other hand, had 1/8 share (66.75 sq. m.) each of the land or a total of 133.5 sq. m. Thus, Florencio and his uncles Juan and Isidro were co-owners of a parcel of land. Pursuant to an action for partition filed by Florencio Ignao (Civil Case N-1681), the then CFI Cavite in a decision dated 6 February 1975 directed the partition of the aforesaid land, alloting 133.5 sq. m. or 2/8 thereof to Juan and Isidro, and giving the remaining portion with a total area of 266.5 sq. m. to petitioner Florencio. However, no actual partition was ever effected. On 17 July 1978, Florencio instituted a complaint for recovery of possession of real property against Juan and Isidro before the CFI Cavite (Civil Case 2662). In his complaint, Florencio alleged that the area occupied by the 2 houses built by Juan and Isidro exceeded the 133.5 sq. m. previously alloted to them by the trial court in Civil Case N-1681. An ocular inspection was conducted by the lower court; which found that the houses of Juan and Isidro actually encroached upon a portion of the land belonging to Florencio. Upon agreement of the parties, the trial court ordered a licensed geodetic engineer to conduct a survey to determine the exact area occupied by the houses of Juan and Isidro. The survey subsequently disclosed that the house of Juan occupied 42 sq. m. while that of Isidro occupied 59 sq. m. of Florencio's land or a total of 101 sq. m. In its decision, the trial court (thru Judge Luis L. Victor) ruled that although Juan and Isidro occupied a portion of Florencio's property, they should be considered builders in good faith. The trial court observed that based on the facts of the case, it would be useless and unsuitable for Florencio to exercise the first option (of appropriating part of the house standing on his lot) since this would render the entire houses of Juan and Isidro worthless. The trial court then applied the ruling in the similar case of Grana vs. Court of Appeals, where the Supreme Court had

advanced a more "workable solution". Thus, it ordered Florencio to sell to Juan and Isidro those portions of his land respectively occupied by the latter at P40.00 per sq. m., and to execute the necessary deed of conveyance to the Juan and Isidro; without pronouncement as to costs. Florencio appealed to the IAC. On 27 August 1985, the Appellate Court (Second Civil Cases Division), promulgated a decision, affirming the decision of the trial court. Hence the petition for review by certiorari. The Supreme Court modified the decision appealed from. The Court directed Florencio to exercise his option to either appropriate as his own the portions of the houses of Juan and Isidro Ignao occupying his land upon payment of indemnity in accordance with Articles 546 and 548 of the Civil Code, or sell to private respondents the 101 sq. m. occupied by them at such price as may be agreed upon; within 30 days from entry of judgment. Should the value of the land exceed the value of the portions of the houses that Juan and Isidro have erected thereon, the latter may choose not to buy the land but they must pay reasonable rent for the use of the portion of Florencio's land as may be agreed upon by the parties. In case of disagreement, the rate of rental and other terms of the lease shall be determined by the trial court. Otherwise, Juan and Isidro may remove or demolish at their own expense the said portions of their houses encroaching upon Florencio's land; Without costs. 1. Co-owners hold property in common dominion, with each an owner of share (abstract and undetermined until partition is effected Prior to partition, all the co-owners hold the property in common dominion but at the same time each is an owner of a share which is abstract and undetermined until partition is effected. As cited in Eusebio vs. Intermediate Appellate Court, "an undivided estate is co-ownership by the heirs." As co-owners, the parties may have unequal shares in the common property, quantitatively speaking. But in a qualitative sense, each co-owner has the same right as any one of the other co-owners. Every co-owner is therefore the owner of the whole, and over the whole he exercises the right of dominion, but he is at the same time the owner of a portion which is truly abstract, because until division is effected such portion is not concretely determined. 2. Article 448 Article 448 provides that the owner of the land on which anything has been built, sown or planted in good faith, shall have the right to appropriate as his own the works, sowing or planting, after payment of the indemnity provided for in articles 546 and 548, or to oblige the one who built or planted to pay the price of the land, and the one who sowed, the proper rent. However, the builder or planter cannot be obliged to buy the land if its value is considerably more than that of the building or trees. In such case, he shall pay reasonable rent, if the owner of the land does not choose to appropriate the building or trees after proper indemnity. The parties shall agree upon the terms of the lease and in case of disagreement, the court shall fix the terms thereof." 3. Article 448 does not apply to a co-owner; except co-ownership is terminated In the case of Spouses del Campo vs. Abesia, it was held that Article 448 of the Civil Code cannot apply where a co-owner builds, plants or sows on the land owned in common for then he did not build, plant or sow upon land that exclusively belongs to another but of which he is a co-owner. The co-owner is not a third person under the circumstances, and the situation is governed by the rules of co-ownership. However, when the co-ownership is terminated by a partition and it appears that the house of an erstwhile co-owner has encroached upon a portion pertaining to another co-owner which was however made in good faith, then the provisions of Article 448 should apply to determine the respective rights of the parties. 4. Right to appropriate works or to oblige builder to pay the price of the land belongs to the landowner As held in Quemuel vs. Olaes, it was categorically ruled that the right to appropriate the works or improvements or to oblige the builder to pay the price of the land belongs to the landowner. Both the trial court and the Appellate Court erred when they peremptorily adopted the "workable solution" in the case of Grana vs. Court of Appeals, and ordered the owner of the land, Florencio, to sell to Juan and Isidro, the part of the land they intruded upon, thereby depriving Florencio of his right to choose. Such ruling contravened the explicit provisions of Article 448 to the effect that "the owner of the land shall have the right to appropriate or to oblige the one who built to pay the price of the land." The law is clear and unambiguous when it confers the right of choice upon the landowner and not upon the builder and the courts. 5. Determination of price to be paid premature The question on the price to be paid on the land need not be discussed as this would be premature inasmuch as Florencio has yet to exercise his option as the owner of the land. Sarmiento v. Agana [G.R. No. 57288. April 30, 1984.] Second Division, Melencio-Herrera (J): 5 concur Facts: While Ernesto Valentino was still courting his wife (Rebecca Lorenzo), the latter's mother had told him the couple could build a residential house on a lot of 145 sq. ms. (Lot D of a subdivision in Paraaque). In 1967, Ernesto did construct a residential house on the land at a cost of P8,000.00 to P10,000.00. It was probably assumed that the wife's mother was the owner of the land and that, eventually, it would somehow be transferred to the spouses. It subsequently turned out that the land had been titled in the name of Mr. & Mrs. Jose C. Santos, Jr. who, on 7 September 1974, sold the same to Leonila Sarmiento. The following 6 January 1975, Sarmiento asked Ernesto and wife to vacate and, on 21 April 1975, filed an Ejectment suit against them. In the evidentiary hearings before the Municipal Court, Sarmiento submitted the deed of sale of the land in her favor, which showed the price to be P15,000.00. On the other hand, Ernesto testified that the then cost of the residential house would be from P30,000.00 to P40,000.00. The figures were not questioned by Sarmiento. On 17 October 1977, the Municipal Court found that the Valentinos had built the residential house in good faith, and, disregarding the testimony of Ernesto, that it had a value of P20,000.00. It then ordered Ernesto and wife to vacate the land after Sarmiento has paid them the mentioned sum of P20,000.00. The Ejectment suit was elevated to the CFI Pasay where, after the submission of memoranda (pursuant to the provisions of RA 6031) and on 29 March 1979, said Court rendered a modifying decision under Article 448 of the Civil Code. Sarmiento was required, within 60 days, to exercise the option to reimburse Ernesto and wife the sum of P40,000.00 as the value of the residential house, or the option to allow them to purchase the land for P25,000.00. Sarmiento did not exercise any of the two

options within the indicated period, and Ernesto was then allowed to deposit the sum of P25,000.00 with the Court as the purchase price for the land. Sarmiento then instituted the instant Certiorari proceedings. The Supreme Court dismissed the petition for certiorari, without pronouncement as to costs. 1. Valentinos in good faith Ernesto Valentino and his wife were builders in good faith in view of the peculiar circumstances under which they had constructed the residential house. As far as they knew, the land was owned by Ernestos mother-in-law who, having stated they could build on the property, could reasonably be expected to later on give them the land. 2. Article 448 of the Civil Code In regards to builders in good faith, Article 448 of the Code provides that The owner of the land on which anything has been built, sown or planted in good faith, shall have the right to appropriate as his own the works, sowing or planting, after payment of the indemnity provided for in articles 546 and 548, or to oblige the one who built or planted to pay the price of the land, and the one who sowed, the proper rent. However, the builder or planter cannot be obliged to buy the land if its value is considerably more than that of the building or trees. In such case, he shall pay reasonable rent, if the owner of the land does not choose to appropriate the building or trees after proper indemnity. The parties shall agree upon the terms of the lease and in case of disagreement, the court shall fix the terms thereof." 3. Options available for owner of the land The owner of the building erected in good faith on a land owned by another, is entitled to retain the possession of the land until he is paid the value of his building, under article 453 (now Article 546). The owner of the land, upon the other hand, has the option, under article 361 (now Article 448), either to pay for the building or to sell his land to the owner of the building. But he cannot as respondents here did, refuse both to pay for the building and to sell the land and compel the owner of the building to remove it from the land where it is erected. He is entitled to such remotion only when, after having chosen to sell his land, the other party fails to pay for the same. In the present case, the provision for the exercise by Sarmiento of either the option to indemnify the Valentinos in the amount of P40,000.00, or the option to allow the Valentinos to purchase the land at P25,000.00, in the Courts opinion, was a correct decision. The order of the trial court compelling the Valentinos to remove the building on Sarmientos land because the latter chose neither to pay for such buildings nor to sell the land, is null and void, for it amends substantially the judgment sought to be executed and is, furthermore, offensive to articles 361 (now Article 448) and 453 (now Article 546) of the Civil Code." (Ignacio vs. Hilario, 76 Phil. 605, 608 [1946]). Pecson v. CA [G.R. No. 115814. May 26, 1995.] First Division, Davide Jr (J): 3 concur, 1 on leave Facts: Pedro P. Pecson was the owner of a commercial lot located in Kamias Street, Quezon City, on which he built a 4-door 2storey apartment building. For his failure to pay realty taxes amounting to P12,000.00, the lot was sold at public auction by the City Treasurer of Quezon City to Mamerto Nepomuceno who in turn sold it on 12 October 1983 to the spouses Juan Nuguid and Erlinda Tan-Nuguid, for P103,000.00. Pecson challenged the validity of the auction sale in Civil Case Q-41470 before the RTC Quezon City. In its decision of 8 February 1989, the RTC dismissed the complaint, but as to the Nuguid's claim that the sale included the apartment building, it held that the issue concerning it was "not a subject of the litigation." In resolving Nuguids motion to reconsider this issue, the trial court held that there was no legal basis for the contention that the apartment building was included in the sale.Both parties then appealed the decision to the Court of Appeals (CA-GR CV 293)1. In its decision of 30 April 1992, the Court of Appeals affirmed in toto the assailed decision. It also agreed with the trial court that the apartment building was not included in the auction sale of the commercial lot. The petition to review the said decision was subsequently denied by the Supreme Court. Entry of judgment was made on 23 June 1993. On November 1993, Nuguid filed with the trial court a motion for delivery of possession of the lot and the apartment building, citing Article 546 of the Civil Code. Acting thereon, the trial court issued on 15 November 1993 the challenged order, ordering Nuguid to reimburse Pecson the construction cost of P53,000 and upon such payment, Nuguid is entitled to immediate issuance of a writ of possession over the lot and improvements thereon. The Court also held that as Nuguid has been declared the uncontested owner of the lot in question as per Entry of Judgment of the Supreme Court dated 23 June 1993, Pecson should pay to Nuguid rent of no less than P21,000.00 per month from said date (as this is the very same amount paid monthly by the tenants occupying the lot), such rent being offset against the amount pertaining to the construction cost. Pecson moved for the reconsideration of the order but it was not acted upon by the trial court. Instead, on 18 November 1993, the trial court issued a writ of possession directing the deputy sheriff "to place Juan Nuguid in possession of subject property located at 79 Kamias Road, Quezon City, with all the improvements thereon and to eject therefrom all occupants therein, their agents, assignees, heirs and representatives." Pecson then filed with the Court of Appeals a special civil action for certiorari and prohibition assailing the order of 15 November 1993 (CA-GR SP 32679). In its decision of 7 June 1994, the Court of Appeals affirmed in part the order of the trial court citing Articles 448 of the Civil Code. The Court of Appeals ruled that Pecsons quest to be restored in possession of the premises is rendered moot and academic as the deputy sheriff has enforced the writ of possession and the premises had been turned over to Nuguids possession; and ordered Pecson to account for any and all fruits of the improvements received by him starting 23 June 1993 with the amount of P53,000 to be offset therefrom. Aggrieved by the Court of Appeals' decision, Pecson filed the petition for review on certiorari. The Supreme Court set aside the decision of the Court of Appeals in CA-GR SP 32679 and the Order of 15 November 1993 of the RTC, Branch 101, Quezon City in Civil Case Q-41470. The Court remanded the case to the trial court for it to determine the current market value of the apartment building on the lot. For this purpose, the parties shall be allowed to adduce evidence on the current market value of the apartment building. The value so determined shall be forthwith paid by Nuguid to Pecson, otherwise Pecson shall be restored to the possession of the apartment building until payment of the required indemnity.

1. Pertinent provisions: Article 448 and Article 546 of the Civil Code Article 448 provides that the owner of the land on which anything has been built, sown or planted in good faith, shall have the right to appropriate as his own the works, sowing or planting, after payment of the indemnity provided for in articles 546 and 548, or to oblige the one who built or planted to pay the price of the land, and the one who sowed, the proper rent. However, the builder or planter cannot be obliged to buy the land if its value is considerably more than that of the building or trees. In such case, he shall pay reasonable rent, if the owner of the land does not choose to appropriate the building or trees after proper indemnity. The parties shall agree upon the terms of the lease and in case of disagreement, the court shall fix the terms thereof. (361a) while Article 546 provides that Necessary expenses shall be refunded to every possessor; but only the possessor in good faith may retain the thing until he has been reimbursed therefor. Useful expenses shall be refunded only to the possessor in good faith with the same right of retention, the person who has defeated him in the possession having the option of refunding the amount of the expenses or of paying the increase in value which the thing may have acquired by reason thereof. (453a) 2. Applicability of Articles 448 and 526 Article 448 refers to a land whose ownership is claimed by two or more parties, one of whom has built some works, or sown or planted something. The building, sowing or planting may have been made in good faith or in bad faith. The rule on good faith laid down in Article 526 of the Civil Code shall be applied in determining whether a builder, sower or planter had acted in good faith. 3. Strict application of Article 448: does not apply to owner who subsequently loses ownership of land by sale or donation; Coleongco v. Regalado Article 448 does not apply to a case where the owner of the land is the builder, sower, or planter who then later loses ownership of the land by sale or donation. As held in Coleongco vs. Regalado, Article 361 of the old Civil Code is not applicable in the case, for Regalado constructed the house on his own land before he sold said land to Coleongco. Article 361 applies only in cases where a person constructs a building on the land of another in good or in bad faith, as the case may be. It does not apply to a case where a person constructs a building on his own land, for then there can be no questions as to good or bad faith on the part of the builder. Elsewise stated, where the true owner himself is the builder of works on his own land, the issue of good faith or bad faith is entirely irrelevant. 4. Application of Article 448 by analogy in preventing a state of forced co-ownership The provision on indemnity in Article 448 may be applied by analogy considering that the primary intent of Article 448 is to avoid a state of forced co-ownership and that the parties, including the two courts below, agree that Articles 448 and 546 of the Civil Code are applicable and indemnity for the improvements may be paid although they differ as to the basis of the indemnity. 5. Article 546 does not state the manner to determine value of useful improvements; Value to be based on market or present value, not the cost during time of construction Article 546 does not specifically state how the value of the useful improvements should be determined. In Javier vs. Concepcion, Jr., the Court pegged the value of the useful improvements consisting of various fruits, bamboos, a house and camarin made of strong material based on the market value of the said improvements. In Sarmiento vs. Agana, despite the finding that the useful improvement, a residential house, was built in 1967 at a cost of between P8,000.00 to P10,000.00, the landowner was ordered to reimburse the builder in the amount of P40,000.00, the value of the house at the time of the trial. In De Guzman vs. De la Fuente, the landowner was required to pay the "present value" of the house, a useful improvement. In the present case, the lower court erred in holding the cost of construction of the apartment building in 1965, and not its current market value, is sufficient reimbursement for necessary and useful improvements made by Pecson. 6. Objective of Article 546 is to administer justice between parties involved; Current value of the property, to prevent unjust enrichment to one of the parties; Parties right to adduce evidence on present market value of improvement The objective of Article 546 of the Civil Code is to administer justice between the parties involved. In this regard, as stated in Rivera vs. Roman Catholic Archbishop of Manila, the said provision was formulated in trying to adjust the rights of the owner and possessor in good faith of a piece of land, to administer complete justice to both of them is such a way as neither one nor the other may enrich himself of that which does not belong to him. Guided by this precept, it is therefore the current market value of the improvements which should be made the basis of reimbursement. A contrary ruling would unjustly enrich the private respondents who would otherwise be allowed to acquire a highly valued income-yielding 4-unit apartment building for a measly amount. Consequently, the parties should therefore be allowed to adduce evidence on the present market value of the apartment building upon which the trial court should base its finding as to the amount of reimbursement to be paid by the landowner. 7. Builder in good faiths right of retention until indemnification for improvements; no rents due The trial court erred in ordering Pecson to pay monthly rentals equal to the aggregate rentals paid by the lessees of the apartment building. Since Nuguid has opted to appropriate the apartment building, Pecson is thus entitled to the possession and enjoyment of the apartment building, until he is paid the proper indemnity, as well as of the portion of the lot where the building has been constructed. This is so because the right to retain the improvements while the corresponding indemnity is not paid implies the tenancy or possession in fact of the land in which it is built, planted or sown. As Pescon has not been paid, he was entitled to retain ownership of the building and, necessarily, the income therefrom. Depra v. Dumlao [G.R. No. L-57348. May 16, 1985.] First Division, Melencio-Herrera (J): 5 concur, 1 took no part Facts: Francisco Depra is the owner of a parcel of land registered under TCT T-3087, known as Lot 685, situated in the municipality of Dumangas, Iloilo, with an area of approximately 8,870 sq. m. Agustin Dumlao owns an adjoining lot, designated as Lot 683, with an approximate area of 231 sq. ms. In 1972, when Dumlao constructed his house on his lot, the kitchen thereof had encroached on an area of 34 sq. m. of Depra's property. After the encroachment was discovered in a relocation survey of Depra's lot made on 2 November 1972, his mother, Beatriz Derla, after writing a demand letter asking Dumlao to move back from his encroachment, filed an action for Unlawful Detainer on 6 February 1973 against Dumlao in the Municipal Court of Dumangas. Said complaint was later amended to include Depra as a party plaintiff. After trial the Municipal Court found that Dumlao was a builder in good faith, and applying Article 448 of the Civil Code, rendered judgment on 29 September 1973,

ordering that a forced lease is created between the parties with Depra, as lessor, and the Dumlao as lessee, over the disputed portion with an area of 34 sq. m., the rent to be paid is P5.00 a month, payable by the lessee to the lessors within the first 5 days of the month the rent is due; and the lease shall commence on that day that this decision shall have become final. From the foregoing judgment, neither party appealed so that, if it were a valid judgment, it would have ordinarily lapsed into finality. Still, Depra did not accept payment of rentals so that Dumlao deposited such rentals with the Municipal Court. On 15 July 1974, Depra filed a Complaint for Quieting of Title against Dumlao before the then CFI Iloilo (Branch IV), involving the same 34 sq. m., which was the bone of contention in the Municipal Court. Dumlao, in his Answer, admitted the encroachment but alleged, in the main, that the present suit is barred by res judicata by virtue of the Decision of the Municipal Court, which had become final and executory. Premised on the joint motion for judgment based on the stipulation of facts by the parties, the Trial Court on 31 October 1974, issued the assailed Order, decreeing that the 34 sq.m. is part and parcel of Lot 685 of the Cadastral Survey of Dumangas of which Depra is owner as evidenced by TCT 3087 and such plaintiff is entitled to possess the same; without pronouncement as to cost. Thus, an appeal was filed with the then Court of Appeals, which the latter certified to the Supreme Court as involving pure questions of law. The Supreme Court set aside the judgment of the trial Court and ordered the case remanded to the RTC Iloilo for further proceedings consistent with Articles 448 and 546 of the Civil Code. It ordered (1) the trial court to determine the present fair price of Depras 34 square meter-area of land, the amount of the expenses spent by Dumlao for the building of the kitchen, the increase in value ("plus value") which the said area of 34 square meters may have acquired by reason thereof, and whether the value of said area of land is considerably more than that of the kitchen built thereon; and after said amount have been determined by competent evidence, (2) the RTC shall render judgment, (a) granting Depra a period of 15 days within which to exercise his option under the law (Article 448, Civil Code), whether to appropriate the kitchen a his own by paying to Dumlao either the amount of the expenses spent by Dumlao for the building of the kitchen, or the increase in value ("plus value") which the said area of 34 square meters may have acquired by reason thereof, or to oblige Dumlao to pay the price of said area. The amounts shall be paid by the obligor within 15 days from notice of the option by tendering the amount to the Court in favor of the party entitled to receive it, (b) that if Depra exercises the option to oblige Dumlao to pay the price of the land but the latter rejects such purchase because the value of the land is considerably more than that of the kitchen, Dumlao shall give written notice of such rejection to Depra and to the Court within 15 days from notice of Depra's option to sell the land. In that event, the parties shall be given a period of 15 days from such notice of rejection within which to agree upon the terms of the lease, and give the Court formal written notice of such agreement and its provisos. If no agreement is reached by the parties, the trial Court, within 15 days from and after the termination of the said period fixed for negotiation, shall then fix the terms of the lease, provided that the monthly rental to be fixed by the Court shall not be less than P10 per month, payable within the first 5 days of each calendar month. The period for the forced lease shall not be more than 2 years, counted from the finality of the judgment, considering the long period of time since 1952 that Dumlao has occupied the subject area. The rental thus fixed shall be increased by 10% for the second year of the forced lease. Dumlao shall not make any further constructions or improvements on the kitchen. Upon expiration of the 2-year period, or upon default by Dumlao in the payment of rentals for 2 consecutive months, Depra shall be entitled to terminate the forced lease, to recover his land, and to have the kitchen removed by Dumlao or at the latter's expense. The rentals herein provided shall be tendered by Dumlao to the Court for payment to Depra, and such tender shall constitute evidence of whether or not compliance was made within the period fixed by the Court, (c) ordering Dumlao to pay Depra an amount computed at P10 per month as reasonable compensation for the occupancy of Depra's land for the period counted from 1952, the year Dumlao occupied the subject area, up to the commencement date of the forced lease, and (d) that the periods to be fixed by the trial Court in its Decision shall be inextendible, and upon failure of the party obliged to tender to the trial Court the amount due to the obligee, the party entitled to such payment shall be entitled to an order of execution for the enforcement of payment of the amount due and for compliance with such other acts as may be required by the prestation due the obligee; Without costs. 1. Decision of the Municipal Court null and void, cannot operate as res judicata The Decision of the Municipal Court is null and void. The judgment in a detainer case is effective in respect of possession only (Sec. 7, Rule 70, Rules of Court). The Municipal Court overstepped its bounds when it imposed upon the parties a situation of "forced lease", which like "forced co-ownership" is not favored in law. Furthermore, a lease is an interest in real property, jurisdiction over which belongs to CFI (now RTC) (Sec. 44(b), Judiciary Act of 1948; 2 Sec. 19 (2) BP 129). Since the Municipal Court, acted without jurisdiction, its Decision was null and void and cannot operate as res judicata to the subject complaint for Queting of Title. 2. Difference in causes in detainer and action to quiet title; Judgment in detainer case not a bar to action respecting title to land Even if the Decision of the Municipal Court were valid, the rule on res judicata would not apply due to difference in cause of action. In the Municipal Court, the cause of action was the deprivation of possession, while in the action to quiet title, the cause of action was based on ownership. Furthermore, Sec. 7, Rule 70 of the Rules of Court explicitly provides that judgment in a detainer case "shall not bar an action between the same parties respecting title to the land." 3. Court system a dispute resolving mechanism; Legal effect of agreement of parties within context of mutual concession and stipulation Consistent with the principles that the Court system must be a dispute resolving mechanism, the Court accords legal effect to the agreement of the parties, within the context of their mutual concession and stipulation. They have, thereby, chosen a legal formula to resolve their dispute (Stipulation of Facts) to apply to Dumlao the rights of a "builder in good faith" and to Depra those of a "landowner in good faith" as prescribed in Article 448. The Court thus refrained from further examining whether the factual situations of Dumlao and Depra conform to the juridical positions respectively defined law, for a "builder in good faith" under Article 448, a "possessor in good faith" under Article 526 and a "landowner in good faith" under Article 448.

4. Builder in good faith; Article 448 Article 448 of the Civil Code provides that the owner of the land on which anything has been built sown or planted in good faith shall have the right to appropriate as his own the works, sowing or planting, after payment of the indemnity provided for in articles 546 and 548, or to oblige the one who built or planted to pay the price of the land, and the one who sowed, the proper rent. However, the builder or planter cannot be obliged to buy the land if its value is considerably more than that of the building or trees. In such case, he shall pay reasonable rent, if the owner of the land does not choose to appropriate the building or trees after proper indemnity. The parties shall agree upon the terms of the lease and in case of disagreement, the court shall fix the terms thereof." Pursuant to the foregoing provision, Depra has the option either to pay for the encroaching part of Dumlao's kitchen, or to sell the encroached 34 square meters of his lot to Dumlao. He cannot refuse to pay for the encroaching part of the building, and to sell the encroached part of his land. 5. Right of remotion The owner of the building erected in good faith on a land owned by another, is entitled to retain the possession of the land until he is paid the value of his building, under article 453 (now 546). The owner of the land, upon the other hand, has the option, under article 361 (now 448), either to pay for the building or to sell his land to the owner of the building. But he cannot refuse both to pay for the building and to sell the land and compel the owner of the building to remove it from the land where it erected. He is entitled to such remotion only when, after having chosen to sell his land, the other party fails to pay for the same. In the present case, Dumlao had expressed his willingness to pay for the land, but Depra refused to sell. 6. Ignacio v. Hilario An order of the lower compelling the builder to remove their buildings from the land belonging to the landowner only because the latter chose neither to pay for such buildings nor to sell the land, is null and void, for it amends substantially the judgment sought to be executed and is, furthermore, offensive to articles 361 (now Article 448) and 453 (now Article 546) of the Civil Code. (Ignacio vs. Hilario, 76 Phil. 605, 608 [1946])." 7. Article 361 of the Spanish Civil Code vis--vis Article 448 of the New Civil Code; Benefits extended to builder but landowner retained his option The original provision found in Article 361 of the Spanish Civil Code provides that the owner of land on which anything has been built, sown or planted in good faith, shall have the right to appropriate as his own the work, sowing or planting, after the payment of the indemnity stated in Articles 453 and 454, or to oblige the one who built or planted to pay the price of the land, and the one who sowed, the proper rent." The Code Commission must have taken account of the objections (of some commentators) to Article 361 of the Spanish Civil Code. Hence, the Commission provided a modification thereof, and Article 448 of our Code has been made to provide that the owner of the land on which has been built, sown or planted in good faith, shall have the right to appropriate as his own the works, sowing or planting, after payment of the indemnity provided for in articles 546 and 548, or to oblige the one who built or planted to pay the price of the land, and the one who sowed, the proper rent. However, the builder or planter cannot be obliged to buy the land if its value is considerably more than that of the building or trees. In such case, he shall pay reasonable rent, if the owner of the land does not choose to appropriate the building or trees after proper indemnity. The parties shall agree upon the terms of the lease and in case of disagreement, the court shall fix the terms thereof." Additional benefits were extended to the builder but the landowner retained his options. 8. The fairness of the rules in Article 448 Where the builder, planter or sower has acted in good faith, a conflict of rights arises between the owners, and it becomes necessary to protect the owner of the improvements without causing injustice to the owner of the land. In view of the impracticability of creating a state of forced co-ownership, the law has provided a just solution by giving the owner of the land the option to acquire the improvements after payment of the proper indemnity, or to oblige the builder or planter to pay for the land and the sower to pay for the proper rent. It is the owner of the land who is authorized to exercise the option, because his right is older, and because, by the principle of accession, he is entitled to the ownership of the accessory thing. (3 Manresa 213; Bernardo vs. Bataclan, 37 Off. Gaz. 1382; Co Tao vs. Chan Chico, G.R. No. 49167, April 30, 1949; Article applied: see Cabral, et al vs. Ibaez [S.C.] 52 Off. Gaz. 217; Marfori vs. Velasco, [C.A.] 52 Off. Gaz. 2050). Quemuel v. Olaes [G.R. No. L-11084. April 29, 1961.] En Banc, Paredes (J): 9 concur Facts: Angel S. Olaes and his wife, Juliana Prudente, registered owners of lot 1095 of the San Francisco de Malabon Estate, located in Rosario, Cavite, sought the recovery of the possession of the said lot and rentals therefore in Civil Case 5442 CFI Cavite, from Alejandro Quemuel and his wife Ruperta Solis, who claimed to be in possession under the tolerance of the former. On 16 March 1954, the trial court ordered Quemuels to return the possession of lot 1095 to the Olaes spouses and to pay the latter P20 a month from January 1954, until they shall have vacated the premises. The Quemuels did not appeal from said decision which became final on 22 April 1954. Thereafter, the Olaes spouses sought the execution of the decision. To forestall ejectment, the Quemuels filed on 1 July 1954 the complaint (Civil Case 5518, CFI Cavite). The Quemuels seek to reduce the monthly rental of P20 fixed in Civil Case 5442, and to compel the Olaes spouses to sell to them the portion of the lot 1095 where their house is erected. The Olaes spouses filed a motion to dismiss on 9 July 1954, alleging lack of cause of action, res adjudicata; prescription, and the cause of action, if any, is barred by the Quemuels' failure to set it up as a counter-claim in Civil Case 5442. On 17 September 1954, the trial court dismissed the complaint, without pronouncement as to costs. An appeal was taken by the Quemuels to the Court of Appeals (CA-GR 14837-R) which, by the agreement of the parties, certified the case to the Court. The ex parte petition filed by the Quemuels in the Supreme Court on 9 August 1956, asking that a writ of preliminary injunction be issued to the Provincial Sheriff of Cavite and the Olaes, enjoining them from demolishing the house of the Quemuels until there is a final decision in said Case 14837, was denied on 14 August 1956. The Supreme Court affirmed the order appealed from, with costs against the Quemuels.

1. The Complaint states no cause of action; If the rent imposed by court are found excessive, the Quemuels are free to vacate the property A cause of action presupposes a right of the plaintiff and a violation of such right by the defendant. According to the complaint itself, the rental of P20 monthly and the order to vacate, were provided in a prior judgment (Civil Case 5442) which is final and its validity is not assailed. There being no law that fixes the rental of the same land at 7 1/2% of its alleged market value, the plaintiffs have no right thereto, or a right which could be violated. The defendants are not compelling the plaintiffs to rent the property but wanted them to vacate the premises (Civil Case 5442). If the rental determined by the trial court were excessive, the plaintiffs are free to vacate the property. For plaintiffs to insist on possessing the property and fixing the rentals themselves, would have no legal sanction at all. 2. Builder in good faith: Right to appropriate works or improvements belong to owner of the land, builders right to reimbursement; Builder cannot compel owner of the land to sell land Under Article 448, the right to appropriate the works or improvements or "to oblige the one who built or planted to pay the price of the land" belongs to the owner of the land. The only right given to the builder in good faith is the right to reimbursement for the improvements; the builder, cannot compel the owner of the land to sell such land to the former. This is assuming that the plaintiffs are builders in good faith. 3. Quemuels not builders in good faith; Builder builds under claim of title The Quemuels are not builders in good faith. Article 448 of the new Civil Code, (equivalent to Article 361 of the old Civil Code) is intended to apply only to a case where one builds, or sows, or plants on land in which he believes himself to have a claim of title and not to lands wherein one's only interest is that of tenant, under a rental contract, which is the present case (Alburo vs. Villanueva, 7 Phil., 277). The tenant cannot be said to be a builder in good faith as he has no pretension to be owner (Rivera vs. Thailand, 48 Phil., 396; see also 3 Manresa 4th Ed. pp. 215-216). 4. Quemuels are lessees From the pleadings and the documentary evidence submitted, it is indisputable that the land in question originally belonged to the government as part of the Friar Lands Estate and the title thereto was in the name of the government until it was purchased by Agapita Solis who applied, thru the Bureau of Lands, to purchase the land by installments. The corresponding Sale Certificate 531, effective 1 July 1909 was executed. In Olaes spouses' complaint, they alleged that they are the owners of lot 1095 and that the Quemuels have been occupying southeastern half portion thereof, without any right thereto, except the tolerance of the Olaes spouses. The Quemuels were not unaware of the flaw in their title, if any, and that their true relation with the Olaes spouses was that of tenant and landlord, and, that their rights are governed by Article 1573 in relation to article 487 of the old Civil Code. Article 1573 provides that a lessee shall have with respect to useful and voluntary improvements, the same right which are granted the usufructuaries." Article 487 provides that the usufructuary may make on the property held in usufruct any improvements, useful or recreative, which he may deem proper, provided he does not change its form or substance, but he shall have no right to be indemnified therefor. He may, however, remove such improvements, should it be possible to do so without injury to the property". 5. Lessee cannot compel lessor to pay for the improvements or to sell the land; Right to remove improvements if it can be done without damage to the land From Articles 487 and 1573, it can clearly be inferred that the Quemuels cannot even compel the Olaes spouses to pay for the improvements the former made in the property or to sell the latter's land. The Quemuels' only right, is to remove the improvements, if it is possible to do so without damage to the land. 6. Identity of case 5442 and 5518; Assuming otherwise, covered by Res judicata In Case 5442, the matter of the rental was in issue, and the same was considered and decided by the trial court, which ordered the Quemuels therein "to pay a reasonable compensation of P20 a month beginning with January, 1954, until they shall have left the premises". In the present case (5518), the parties are the identical parties in Civil Case 5442, the same lot 1095 is the subject matter of both cases; the same issue, namely, the amount of the rental is involved. Even assuming that Quemuels have a cause of action, the doctrine of res judicata already operates against them. 7. Lot purchased by Agapita Solis; Assuming otherwise, covered by Prescription Lot 1095 was purchased by Agapita Solis from the Government on 1 July 1909. After full payment of the purchase price, TCT 10771 covering said lot was issued to said Agapita Solis on 8 June 1933. Assuming that the Quemuels or their alleged predecessor-in-interest, had a cause of action for claiming the ownership of portions of said lot, such cause of action accrued at the latest on 8 June 1933. The statute of limitations provide that the Quemuels or their predecessor had 10 years from said date, within which to file the corresponding action; which cannot be had as the Quemuels filed the complaint after more than 21 years or on 1 July 1954. 8. Failure to set up a counterclaim in Civil Case 5442 Whether the cause of action is for recovery of ownership or for an alleged right to purchase the property, or for reimbursement for some improvements, the Quemuels should have set it up as a counterclaim in Civil Case 5442, because same was necessarily connected with, or arose out of the transactions involved in Civil Case 5442 (Section 6, Rule 10, Rules of Court). 9. De Jesus v. Belarmino does not apply In De Jesus, et al. vs. Belarmino, et al. (GR L-6665, 30 June 1954; OG July 1954, p. 3064), it was held that "where the complaint was dismissed not because of any evidence presented by the parties, or as a result of a trial on the merits, but merely on a motion to dismiss filed by the defendants, the sufficiency of the motion should be tested on the strength of the allegation of facts contained in the complaint and no other", which has been interpreted to apply to cases where the motion to dismiss is based solely on the ground of lack of cause of action. Considering the fact that (1) In the present case, documentary evidence and the records of the Civil Case 5442 were presented and considered by the trial court; and (2) in the De Jesus case, the only ground for dismissal was the lack of cause of action, while in the present case, aside from said ground, plaintiffs alleged other grounds, the said ruling finds no application.

San Diego v. Montesa [G.R. No. L-17985. September 29, 1962.] En Banc, Reyes JBL (J): 8 concur Facts: After trial in Civil Case 770 of the CFI Bulacan, on complaint of Jose, Maria, and Urbano, all surnamed "de la Cruz", to recover a parcel of land and damages from Gil San Diego and Rufina San Diego, the Court rendered a decision declaring the deed of sale null and void, ordering the San Diegos to vacate the land upon payment of the sum P3,500 by the de la Cruzes within 30 days after the decision becomes final, dismissing the counterclaim of the de la Cruzes, without pronouncement as to costs. The court found that the disputed portion of a parcel of land belonged to the de la Cruzes through hereditary succession; that the San Diegos built a house on the land in good faith, having acquired the land from Catalina Anastacio, mother of the de la Cruzes, by purchase for P1,000.00. During the proceedings, the San Diegos filed a third-party complaint against said vendor. The vendor (mother of the de la Cruzes) subsequently died; hence, the de la Cruzes, who were the plaintiffs, became at the same time thirdparty defendants in substitution of their deceased mother. The court voided the sale on the ground that the vendor had no right to the land, but upheld the defense of the San Diegos as builders in good faith. On appeal by the de la Cruzes, the Court of Appeals affirmed in toto the lower court's decision, and the same, thereafter, became final and executory. Over 2 years later, the San Diegos, who were in possession of the parcel of land in litigation, moved to execute paragraph (b) of the dispositive portion of the decision in order to collect the sum of P3,500.00 and thereafter to vacate the premises. The motion was denied by the court, and a motion for reconsideration was likewise of no avail. Hence, the instant petition for mandamus was filed to compel the judge to issue the writ applied for. The Supreme Court granted the writ prayed for, and ordered the CFI Bulacan to issue the writ of execution in favor of San Diegos. Costs against de la Cruz 1. Judgment based on right of retention due to possessors in good faith; No rental required during period of retention The judgment affirmed by the Court of Appeals, and now final, explicitly ordains the payment by the de la Cruzes of the amount of P3,500.00 "within 30 days after this decision becomes final" to the San Diegos. If it also orders the San Diegos to vacate only upon such payment, it did so in recognition of the right of retention granted to possessors in good faith by Article 546 of the Civil Code of the Philippines. This provision is expressly made applicable to builders in good faith (Article 448). The right of retention thus granted is merely a security for the enforcement of the possessor's right to indemnity for the improvements made by him. As a result, the possessor in good faith, in retaining the land and its improvements pending reimbursement of his useful expenditure, is not bound to pay any rental during the period of retention; otherwise, the value of his security would be impaired (cf. Tufexis vs. Chunaco (C.A.) 36 O.G. 2455). 2. Options of the landowner; Decision limited to appropriation of the improvement which the landowner did not object to; Decision final and cannot be altered Normally the landowner has the option to either appropriate the improvement or to sell the land to the possessor. This option is no longer open to the landowners herein because the decision in the former suit limits them to the first alternative by requiring the San Diegos to vacate the land (and surrender the improvements) upon payment of P3,500.00. Evidently, the CFI and the CA opined that the de la Cruzes' suit to recover the property was an exercise of their right to choose to appropriate the improvements and pay the indemnity fixed by law. The de La Cruzes acquiesced in this view, since they did not ask for the modification of the judgment and allowed it to become final. Consequently, they can no longer insist on selecting another alternative; nor can they be heard now to urge that the value of the indemnity, set at P3,500.00, is exorbitant, for the same reason that the judgment fixing that amount is no longer subject to alteration. 3. Courts duty to execute a final judgment; Writ of Mandamus The judgment ordering payment to the San Diegos of P3,500.00, by way of indemnity, having become final, and the 30 days for its payment having elapsed, the CFI has the ministerial duty to order its execution (Zulueta vs. Paredes, 62 Phil. 5; Buenaventura vs. Garcia, 78 Phil. 759; Amor vs. Jugo, 17 Phil. 703; Viquiera vs. Baraa, 78 Phil. 456). That duty is compellable by mandamus; and the execution is leviable on any property of the de la Cruzes, including the land now in question and its improvements. De Guzman v. dela Fuente [G.R. No. 32433. December 29, 1930.] First Division, Romualdez (J): 8 concur Facts: Francisco de Guzman and Crisanto De la Fuente, being close friends and compadres, agreed verbally that the latter should occupy, in addition to a house he intended to build, the land here in question, belonging to the former. In 1912 De la Fuente built his house upon said land, which house was repaired in 1928 (house was appraised by the court at P7,504). De la Fuente was required to pay rent at P45 per month from 1 October 1926 to 31 December 1927 and at P120 a month from 1 January 1928. De la Fuente on the other hand receives rent at P40 per month (from Payumo and later, Taw Pe Chan). Raised in the trial court (actual controversy not available in the case facts), the de Guzmans contend that de la Fuente occupies their land by mere tolerance. De la fuente however claims that the land belongs to him. The trial court ordered de la Fuente to vacate the premises and restoring such to the de Guzmans. For lack of evidence, no judgment was rendered against de la Fuente and Payumo to pay damages at P697.50 from 1 October 1926 to 15 January 1928, or against de la Fuente and Taw Pe Chan to pay in solidum damages of P120 a month from 15 January 1928. Both plaintiffs and defendants appealed from this judgment. The Supreme Court modified the judgment appealed from, and held that the land in question belongs not to the defendant De la Fuente, but to the plaintiffs, who are entitled to acquire said defendant's house built thereon, by paying its owner the sum of P7,504, or to compel him to pay them the price of the land agreed upon by the interested parties; and in default thereof, the price as fixed by the competent court; and should the plaintiff choose to acquire the house, the defendant shall have the right to retain the same until the above- mentioned amount is satisfied.affirmed the the remainder of the judgment appealed from insofar as it is not incompatible with this decision. Without express pronouncement of costs.

1. Land owned by the de Guzmans The record shows that the owners of the land were Francisco de Guzman and his deceased wife, and it now belongs to said Francisco de Guzman and his children. 2. De la Fuentes possession of the land in good faith De la Fuente's possession of the land commenced and continues in good faith, inasmuch as, on the one hand, the extrajudicial notice given by the defendant about November, 1927 did not by itself destroy said good faith, and will continue to exist as long as there is no final judgment to the contrary, which to date has not been rendered; and, on the other hand, the necessity for the repairs of the house has been sufficiently proved. 3. One party owner of the land, one party owner of the building This is a case where one party is the owner of the land, and the other is the owner, in good faith, of the building thereon, provided for in article 361 of the Civil Code. Miranda v. Fadullon [G.R. No. L-8220. October 29, 1955.] En Banc, Montemayor (J): 8 concur Facts: In 1939 Lucio Tio was the owner of a parcel of land, lot 1589-J of the Banilad Estate, Cebu, under TCT 10548. On 9 December 1939, a power of attorney in favor of one Esteban Fadullon executed by Lucio Tio was registered in the land records of Cebu City and annotated at the back of the TCT. On the same date the deed of mortgage in favor of the Cebu Mutual Building and Loan Association was also annotated on the same certificate of title. In 1946, on the strength of the said power of attorney Fadullon sold the property to the spouses Dionisio Segarra and Clemencia N. de Segarra with right to repurchase within the short period of 30 days. Upon failure of Fadullon to make the repurchase within the period, the Segarras within 10 days after the expiration of the period filed a sworn petition for the consolidation of their ownership and registered said petition in the office of the Register of Deeds on 15 May 1946. Apprised of the sale of his property, Lucio Tio on 4 June 1946, filed a complaint in the CFI Cebu, Civil Case 181 to annul the sale. After hearing the trial court rendered judgment annulling the sale;canceling the new Certificate of Torrens Title 392 on the property in the name of the Segarras, and ordered a new one to be issued in the name of Lucio Tio and his wife Salvacion Miranda. The Segarras appealed to the Court of Appeals under CA-GR. 6550-R and the said Tribunal affirmed the appealed decision and further required the Segarras to pay Tio the reasonable rentals on the property from the filing of the action until said property shall have been returned to the latter. Upon the decision becoming final the corresponding writ of execution was issued directing the Sheriff to put Tio in possession of the lot. It turned out however that during the possession of the property by the Segarras they had introduced improvements thereon consisting of a building of 3 rooms and a storage room, and one artesian well, with tower and water tank and a cement flooring covering about 1/3 of the lot which according to the Segarras cost them P5,300. They filed a motion with the trial court claiming that they were possessors in good faith of the lot in question, and that they had introduced the improvements aforementioned in good faith and asked the court to order Tio to pay for the said improvements valued at P5,300 or to allow them to buy the land should Tio decide not to pay for the improvements. On 28 August 1952, the trial court ordered Tio to pay the Segarras the sum of P5,300 or otherwise allow the latter to purchase the lot. Tio filed a motion for reconsideration claiming that the Segarras are possessors and builders in bad faith, and explained his lack of opposition for the reimbursement with the understanding that the trial court was sufficiently informed and impressed with the bad faith with which defendants bought the land and introduced improvements thereon. Upon the denial of his motion for reconsideration, he took the appeal. The Court of Appeals certified the case to the Supreme Court on the opinion that the case involved only questions of law. (Lucio Tio later on succeeded by Salvacion Miranda, his wife, after his demise; the case facts did not provide when) The Supreme Court set aside the appealed order of 28 August 1952 and the order of 15 October 1952, denying plaintiff's motion for reconsideration are set aside; with costs against Fadullon and the spouses Segarra. 1. Good faith wanting in the manner the lot was sold; Collusion or conspiracy vis--vis short period of redemption While the evidence did not disclose a collusion or conspiracy between Fadullon and the Segarras, yet, considering the short period of one month within which to redeem and the surrounding circumstances, the possibility of such collusion lingers. Obviously there was in this transaction a prevailing intention of railroading the property into a new ownership as may be proven by the fact that said purchasers filed a sworn petition for consolidating their ownership barely 10 days after the expiration of 30 days, i.e. on 13 April 13, 1946, and registered with the office of Register of Deeds for Cebu 12 days thereafter, i.e. on 15 May 1946. 2. Good faith wanting in the manner the lot was sold; Inquiry on the agents power of attorney The alleged power of attorney executed by the late Lucio Tio in favor of appellant Fadullon was registered in the land record of the Register of Deeds of Cebu City and annotated at the back of TCT 10548 on 29 December 1939. On the same date, the deed of mortgage in favor of the Cebu Mutual Building and Loan Association was annotated in the said Torrens title. This encumbrance alone should have been sufficient to put the Segarra spouses upon an inquiry as to the authority of Fadullon to sell to them the same property 6 years later, but they did not. Further, the Segarras did not require Fadullon to produce his power of attorney. While it is true that said power of attorney is annotated at the back of the Torrens title of Tio, it was still incumbent upon the Segarras to ascertain the scope and authority of Fadullon under said power of attorney. Fadullon executed the sale with the right to repurchase within the extraordinary short period of 30 days. This circumstance, again, should have placed the Segarras on their guards, knowing, as they did, that they were dealing with an agent under a power of attorney executed before the war. 3. Order to pay rent an indicatum that the lower courts perceive defendants as possessors in bad faith From the decisions of the lower courts, one can logically infer that the conclusion of the two courts, to say it more mildly, that the

defendants were not possessors in good faith. The fact that the Court of Appeals sentenced the defendants to pay rentals is an indication, even proof that defendants were considered possessors and builders in bad faith, or at least that they were not possessors and builders in good faith. A builder in good faith may not be required to pay rentals. He has a right to retain the land on which he has built in good faith until he is reimbursed the expenses incurred by him. Possibly he might be required to pay rental only when the owner of the land chooses not to appropriate the improvement and requires the builder in good faith to pay for the land, but that the builder is unwilling or unable to buy the land, and then they decide to leave things as they are and assume the relation of lessor and lessee, and should they disagree as to the amount of the rental then they can go to the court to fix that amount. Furthermore, Miranda (Tio) in her brief says without denial or refutation on the part of Fadullon and the Segarra spouses that the latter applied for a building permit to construct the improvements in question on 4 December 1946 and the permit was granted on 11 January 1947, about seven months after they received the summons on 10 June 1946. Chua v. CA [G.R. No. 109840. January 21, 1999.] Second Division, Mendoza (J): 4 concur Facts: Jose L. Chua and Co Sio Eng were lessees of a commercial unit at 3086 Redemptorist Street in Baclaran, Paraaque, Metro Manila. The lease was for a period of 5 years, from 1 January 1985 to 31 December 1989. The contract expressly provided for the renewal of the lease at the option of the lessees "in accordance with the terms of agreement and conditions set by the lessor." Prior to the expiration of the lease, the parties discussed the possibility of renewing it. They exchanged proposal and counterproposal, but they failed to reach agreement. The dispute was referred to the barangay captain for conciliation but still no settlement was reached by the parties. On 24 July 1990, Ramon Ibarra filed a complaint for unlawful detainer against petitioners in the MTC Paraaque, Metro Manila, which on 4 February 1992 rendered a decision, giving a period of 2 years extension of occupancy to Chua and Co Sio Eng starting 24 July 1990, ordering them to pay Ibarra the sum of P188,806.00 representing back rentals as of 1991 and a monthly rental of P10,000.00 thereafter until the expiration of the aforesaid extension of their occupancy or until the subject premises is actually vacated, and ordering them to pay Ibarra the amount of P15,000 as attorneys fees; and the cost of suit. On appeal by both parties, the RTC Makati (Branch 59) ruled that the lease was for a fixed period of 5 years and that, upon its expiration on 1 January 1990, Chuas and Co Sio Eng's continued stay in the premises became illegal. The court ordered Chua and Co Sio Eng to vacate the premises and to turn over possession thereof to Ibarra; to pay Ibarra the amount of P42,306 representing accrued or bank rentals from 1 January 1987 to 31 December 1989, a monthly rental of P7,320.50 for the use or occupancy of the premises starting 1 January 1990 until 24 July 1990 and at P10,000 from 24 July 1990 until Chua and Co Sio Eng shall have vacated the same, the amount of P10,000 representing reasonable attorneys fees. The court also ordered the dismissal of Chuas and Co Sio Engs counterclaim for lack of merit; with costs against them. Chua and Co Sio Eng appealed to the Court of Appeals, which affirmed the decision. In its decision, dated 8 October 1992, the Court of Appeals affirmed the decision of the lower court except for the modification that the monthly rental that petitioners should pay private respondent from 24 July 1990 until the latter finally vacate the premises in question is reduced to P7,320.00; with costs against Chua and Co Sio Eng. Chua and Co Sio Eng filed motion for reconsideration, which was likewise denied. Hence, the petition for review on certiorari. The Supreme Court affirmed the decision of the Court of Appeals, dated 8 October 1992; costs against Chua and Co Sio Eng. 1. Contract is the law between parties; Absolving rentals leads to unjust enrichment Chuas and Co Sio Engs rental arrearages from 1986 to 1989 was an issue raised at the pre-trial and on which issue Ibarra presented evidence without any objection from the former. Considering that Chua and Co Sio Eng incurred said rental arrearages because they did not pay Ibarra the automatic 10% increase in their monthly rental every year for the years 1986 to 1989 as agreed upon and stipulated in their lease contract which contract is the law between the parties, justice and good faith demand that Chua and Co Sio Eng should pay said rental arrearages. To absolve the defendants from paying rentals in arrears while they continue occupying and enjoying the premises would be allowing the defendants to enrich themselves at Ibrarras expense. 2. Objection waived if objection to admission of evidence not made at time the evidence is offered; Amendments to pleading allowed to conform to the evidence in the record Any objection to the admissibility of evidence should be made at the time such evidence is offered or as soon thereafter as the objection to its admissibility becomes apparent, otherwise the objection will be considered waived and such evidence will form part of the records of the case as competent and admissible evidence. Rule 10, 5 8 of the Rules of Civil Procedure allows the amendment of the pleadings in order to make them conform to the evidence in the record. 3. No lease to be extended when the lease contract expired After the lease terminated on 1 January 1990 and without the parties thereafter reaching any agreement for its renewal, Chua and Co Sio Eng became deforciants subject to ejectment from the premises. They are not entitled to a reasonable extension of time to occupy the premises on account of the fact that the lease contract between the parties has already expired, as there was no longer any lease to speak of which could be extended. 4. Authority of the court to fix a longer term of lease applies to cases where no period is fixed by parties The MTC was in effect making a contract for the parties which it obviously did not have the power to do. The potestative authority of the courts to fix a longer term for a lease under Article 1687 of the Civil Code applies only to cases where there is no period fixed by the parties. In the present case, the contract of lease provided for a fixed period of 5 years from 1 January 1985 to 31 December 1989. As held in Bacolod-Murcia Milling Co., Inc. v. Banco Nacional Filipino, it is not the province of the court to alter a contract by construction or to make a new contract for the parties; its duty is confined to the interpretation of the one which

they have made for themselves, without regard to its wisdom or folly, as the court cannot supply material stipulations or read into contract words which it does not contain. 5. Article 1675 excludes cases falling under Artilce 1673 Article 1675 of the Civil Code excludes cases falling under Article 1673 (which provides among others, that the lessor may judicially eject the lessee when the period agreed upon or that which is fixed has expired) from the cases wherein, pursuant to Article 1687, courts may fix a longer period of lease. 6. Article 448 applies to possessor in good faith and not to lessee There is no provision of law which grants the lessee a right of retention over the leased premises on the ground that the lessee made repairs and improvements on the premises. Article 448 of the Civil Code, in relation to Article 546, which provides for full reimbursement of useful improvements and retention of the premises until reimbursement is made, applies only to a possessor in good faith, i.e. one who builds on a land in the belief that he is the owner thereof. 7. Rationale on why Article 448 does not apply to lessees; Article 1678 as relief In a number of cases, the Court has held that this right does not apply to a mere lessee otherwise, it would always be in his power to "improve" his landlord out of the latter's property. Article 1678 merely grants to such a lessee making in good faith useful improvements the right to be reimbursed one-half of the value of the improvements upon the termination of the lease, or, in the alternative, to remove the improvements if the lessor refuses to make reimbursement. 8. Award of attorneys fees reasonable Chua and Co Sio Eng were correctly ordered to pay attorney's fees considering that Ibarra had to go to court to protect his interest. The award of P10,000 is reasonable in view of the time it has taken this rather simple case for ejectment to be decided. Bataclan v. CFI Cavite [G.R. No. 43456. May 6, 1935.] En Banc, Vickers (J): 4 concur Facts: The CFI Cavite issued on 24 April 1934 an order for the sale at public auction the land subject of civil case 2428 between Vicente Santo Domingo Bernardo (plaintiff therein) and Catalino Bataclan (defendant therein). The lower court held that Bernardo has the right to the possession of the land and that Bataclan has the right to demand from the former the sum of P1,642 for the expenses done in good faith, such as the wall and the existing coco and abaca, and also with the right to retain the possession of the land until this amount is paid to him. Bernardo can choose, within 30 days, from the date he is notified, to pay that sum to Bataclan the cost of the existing wall and all plantations in the land, or to compel Bataclan to pay him the price the land, at P300 per hectare. In case Bernardo chose to compel Bataclan to pay him the price of the land, Bataclan shall carry out the payment in the terms suitable and fixed by the Court; Without costs. On appeal to this court the value of the land containing 90 hectares was reduced from P300 to P200 a hectare, and the value of the improvements made by Bataclan thereon was increased from P1,642 to P2,212. The decision of the lower court was affirmed in all other respects (GR 37319). Bernardo elected to compel Bataclan to pay him the value of the land, P18,000. Bernardo informed the CFI Cavite that he elected to compel Bataclan to pay him the value of the land, and that Bataclan appeared in court and stated that he did not have any money; that the court on 24 January 1934 then granted Bernardo 30 days in which to pay bataclan the sum of P2,212, and ordered that if said payment was not made the land should be sold at public auction for the payment of said sum to Bataclan, the balance after deducting the expenses of the sale to be delivered to Bernardo. On 16 March 1934 the lower court (Judge Rovira) modified his order and ordered that from the proceeds of the sale Bernardo should be paid for the land at the rate of P200 a hectare, and that the balance, if any, should be delivered to Bataclan. On 24 April 1934, the lower court (Judge Sixto de la Costa) issued an order for the sale of the land at public auction in order that the Bernardo might be paid from the proceeds the sum of P18,000 and the legal expenses of the sale, and that from the balance, if any, P2,212 should be paid to Bataclan. Hence, the petition for a writ of certiorari to annul the order of the CFI. The Supreme Court denied the petition, with the costs against Bataclan. 1. Article 361 of the Civil Code Article 361 of the civil code, the basis of the decision of the lower court, provides that any owner of land on which anything has been built, sown, or planted, in good faith, shall be entitled to appropriate the thing so built, sown, or planted, upon paying the compensation mentioned in articles 453 and 454, or to compel the person who has built or planted to pay him the value of the land or, to require the person who sowed thereon to pay the proper rent therefor." 2. Court may fix the period of payment It is true that in the decision in question it was provided that in case the plaintiff elected to compel the defendant to pay him the value of the land, the payment should be made within the period agreed upon by the parties or that it would be fixed by the court. However, Bataclan asked for a period of 15 years in which to pay to owner of the land the value thereof; and when he appeared in court he informed the court that he had no money with which to pay for the land. Under those circumstances, it would have been futile for the court to grant Bataclan a reasonable period of 30 or 60 days in which to pay Bernardo the sum of P18,000. If there was any irregularity in the court's ordering the sale of the property at public auction under the conditions stated in the orders of 16 March 1934 and 24 April 1934, it was not prejudicial but favorable to the petitioner, because his only right was to purchase the land for the sum of P18,000. 3. Appeal could have been the proper remedy Bataclan could have appealed from the order in question, and his right to appeal was an adequate remedy.

Ignacio v. Hilario [G.R. No. L-175. April 30, 1946.] En Banc, Moran (J): 10 concur Facts: Elias Hilario and his wife Dionisia Dres filed a complaint Damian, Francisco and Luis Ignacio concerning the ownership of a parcel of land, partly rice-land and partly residential. After the trial of the case, the lower court (Judge Alfonso Felix), rendered judgment holding Hilario and Dres as the legal owners of the whole property but conceding to the Ignacios the ownership of the houses and granaries built by them on the residential portion with the rights of a possessor in good faith, in accordance with article 361 of the Civil Code; without pronouncement is made as to damages and costs. Subsequently, in a motion filed in the same CFI (Judge Hon. Felipe Natividad), Hilario and Dres prayed for an order of execution alleging that since they chose neither to pay the Ignacios for the buildings nor to sell to them the residential lot, the Ignacios should be ordered to remove the structure at their own expense and to restore Hilario and Dres in the possession of said lot. After hearing, the motion was granted by Judge Natividad. Hence, the petition for certiorari was filed by the Ignacios praying for (a) a restraint and annulment of the order of execution issued by Judge Natividad; (b) an order to compel Hilario and Dres to pay them the sum of P2,000 for the buildings, or sell to them the residential lot for P45; or (c) a rehearing of the case for a determination of the rights of the parties upon failure of extra-judicial settlement. The Supreme Court set aside the writ of execution issued by Judge Natividad and ordered the lower court to hold a hearing in the principal case wherein it must determine the prices of the buildings and of the residential lot where they are erected, as well as the period of time within which Hilario and Dres may exercise their option either to pay for the buildings or to sell their land, and, in the last instance, the period of time within which the Ignacios may pay for the land, all these periods to be counted from the date the judgment becomes executory or unappealable. After such hearing, the court shall render a final judgment according to the evidence presented by the parties; with costs against Hilarion and Dres. 1. Right of retention of builder in good faith The owner of the building erected in good faith on a land owned by another, is entitled to retain the possession of the land until he is paid the value of his building, under article 453. Article 453 provides that Necessary expenses shall be refunded to every possessor; but only the possessor in good faith may retain the thing until such expenses are made good to him. Useful expenses shall be refunded to the possessor in good faith with the same right of retention, the person who has defeated him in the possession having the option of refunding the amount of the expenses or paying the increase in value which the thing may have acquired in consequence thereof." 2. Option of the landowner to pay for the building or sell his land to the owner of the building; Right of remotion only available if he chose the latter and the owner of the building cannot pay The owner of the land, upon the other hand, has the option, under article 361, either to pay for the building or to sell his land to the owner of the building. Article 361 provides that The owner of land on which anything has been built, sown or planted in good faith, shall have the right to appropriate as his own the work, sowing or planting, after the payment of the indemnity stated in articles 453 and 454, or to oblige the one who built or planted to pay the price of the land, and the one who sowed, the proper rent. He cannot however refuse both to pay for the building and to sell the land and compel the owner of the building to remove it from the land where it is erected. He is entitled to such remotion only when, after having chosen to sell his land, the other party fails to pay for the same. 3. Order amends judgment substantially and thus null and void The order of Judge Natividad compelling the Ignacios to remove their buildings from the land belonging to Hilario and Dres only because the latter chose neither to pay for such buildings nor to sell the land, is null and void, for it amends substantially the judgment sought to be executed and is, furthermore, offensive to articles 361 and 453 of the Civil Code. 4. Original decision did not become final as it failed to determine the value of the buildings and of the lot; and the time to which the option may be exercised In the decision of Judge Felix, the rights of both parties were well defined under articles 361 and 453 of the Civil Code, but it failed to determine the value of the buildings and of the lot where they are erected as well as the periods of time within which the option may be exercised and payment should be made, these particulars having been left for determination apparently after the judgment has become final. The procedure is erroneous, for after the judgment has become final, no additions can be made thereto and nothing can be done therewith except its execution. And execution cannot be had, the sheriff being ignorant as to how, for how much, and within what time may the option be exercised, and certainty no authority is vested in him to settle these matters which involve exercise of judicial discretion. Thus, the judgment rendered by Judge Felix has never become final, it having left matters to be settled for its completion in a subsequent proceeding, matters which remained unsettled up to the time the petition is filed in the present case. Grana v. CA [G.R. No. L-12486. August 31, 1960.] En Banc, Gutierrez-David (J): 8 concur Facts: In 1909 a cadastral survey of Butuan, Agusan, was made by the Bureau of Lands. In that survey, the parcel of land in question was included as part of the lot belonging to Gregorio Bongato and Clara Botcon for which OCT RO-72 (138) was issued in their favor on 12 February 1923. On 25 November 1933, this lot was purchased by the spouses Marcos Bongato and Eusebia More, and upon their death, the land was inherited by Aurora Bongato and Jardenio Sanchez, the former being the daughter of Marcos Bongato by his first marriage while the latter is the son of Eusebia More also by her first marriage. On 13 April 1951, Aurora Bongato and Jardenio Sanchez sued Leonor Grana and Julieta Torralba before the CFI Agusan, for the recovery of 87 sq. m. of residential land. After trial, the court rendered judgment declaring Bongato and Sanchez owners of the

land in controversy and ordering Grana and Torralba to vacate and deliver it to the former and to pay a monthly rental of P10.00 from the filing of the complaint until they actually vacate the same, plus attorney's fees and costs. The decision, on appeal, having been affirmed by the Court of Appeals with the only modification of disallowing the award for attorney's fees. Grana and Torralba brought the case to the Supreme Court through a petition for review. The Supreme Court modified the appealed decision in the sense that Bongato and Sanchez were directed to exercise within 30 days from the decision their option to either buy the portion of the house of Grana and Torralba on their land or sell to the latter the portion of their land on which it stands. If Bongato and Sanchez choose to sell the land and Grana and Torralba are unwilling or unable to buy, then they must vacate the same and must pay reasonable rent of P10.00 monthly from the time Bongato and Sanchez made their choice up to the time they actually vacate the premises. But if the value of the land is considerably more than the value of the improvement, then Grana and Torralbe may elect to rent the land, in which case the parties shall agree upon the terms of a lease. Should they disagree, the court of origin is hereby instructed to intervene and fix the terms thereof. Grana and Torralba shall pay reasonable rent of P10.00 monthly from the moment Bongato and Sanchez exercised their option up to the time the parties agree on the terms of the lease or until the court fixes such terms; without pronouncement as to costs. 1. No proof that first survey is erroneous No proof was presented to show that the first survey was erroneous or that it included part of the contiguous land of Granas and Torralbas predecessor in interest (Isidaria Trillo) as part of the lot now covered by OCT RO-72 (138). The alleged sketch plan of the resurvey (TS-65 Butuan Cadastre) was not presented in evidence. 2. Torrens certificate conclusive and indefeasible after lapse of period to which it may be impugned A Torrens certificate of title becomes conclusive and indefeasible after the lapse of the period within which it may be impugned (Reyes, et al. vs. Borbon, et al., 50 Phil., 791; Yumul vs. Rivera, et al., 64 Phil., 13). In the present case, the land in question is part of the lot covered by the Torrens title issued way back in 1923 in the name of Bongatos and Sanchez' predecessors in interest. Said title has not been contested up to the present, and, therefore, has become incontrovertible evidence of the ownership of the land covered by it. 3. No new evidence to grant motion for new trial The Court of Appeals did not err in denying the motion for new trial on the ground of newly discovered evidence as the new evidence sought to be introduced was the sketch plan of the second survey, which, with the employment of reasonable diligence would have easily been discovered and produced at the trial. 4. Resurvey plan cannot alter or modify a title If a subsequent certificate of title cannot be permitted to prevail over a previous Torrens title (Reyes, et al, vs. Borbon, et al., supra) with more reason should a resurvey plan not be allowed to alter or modify such title so as to make the area of the land therein described agree with that given in the plan. (See Government of the Philippines vs. Arias, 36 Phil., 195). 5. Builders in good faith Although without any legal and valid claim over the land in question, Grana and Torralba were found to have constructed a portion of their house thereon in good faith. Under Article 361 of the old Civil Code (Article 448 of the new), the owner of the land on which anything has been built in good faith shall have the right to appropriate as his own the building, after payment to the builder of necessary and useful expenses, and in the proper case, expenses for pure luxury or mere pleasure, or to oblige the builder to pay the price of the land. 6. Options of the owners of the land; Practicality of the options considered The owners of the land have the choice of either appropriating the portion of the house which is on their land upon payment of the proper indemnity to the builders, or selling to builders that part of their land on which stands the improvement. However, it would be impractical for respondents to choose to exercise the first alternative, i.e., buy that portion of the house standing on their land, for in that event the whole building might be rendered useless. The more workable solution, it would seem, is for the owners to sell to the builders that part of their land on which was constructed a portion of the latter's house. If the builders are unwilling or unable to buy, then they must vacate the land and must pay rentals until they do so. Of course, the owners cannot oblige the builders to buy the land if its value is considerably more than that of the portion of the house. If such is the case, then the builders must pay reasonable rent. The parties must come to an agreement as to the conditions of the lease, and should they fail to do so, then the court shall fix the same. (Article 361, old Civil Code; Article 448 of the new). 7. Right of retention; Builder in good faith not required to pay rentals until he is reimbursed expenses incurred by him The appellate court erred in ordering Grana and Torralba to pay monthly rentals of P10.00 from the date of filing of the complaint until they actually vacate said land. A builder in good faith may not be required to pay rentals. He has a right to retain the land on which he has built in good faith until he is reimbursed the expenses incurred by him. (Miranda vs. Fadullon, et al., 97 Phil., 801; 51 Off. Gaz., 6226, see also Martinez vs. Baganus, 28 Phil., 500; De Guzman vs. De la Fuente, 55 Phil., 501; Kasilag vs. Rodriguez, Off. Gaz., Supp., August 16, 1941, p. 247). 8. Petitioners estopped in claiming mother was not included as an indispensable party for the complaint to be dismissed Grana and Torralba clearly asserted ownership over the land in dispute as well as over Lot 310 in their answer to the complaint. They are consequently estopped from alleging that the complaint should have been dismissed for nonjoinder of an indispensable party, it being alleged that their mother Maria Cupin, who owns the land in question as part of her Lot 310, has not been made a party defendant in the case. Santos v. Mojica [G.R. No. L-25450. January 31, 1969.] En Banc, Capistrano (J): 10 concur, 1 took no part

Facts: On 19 March 1959, Teodorico, Carmen, Antero, Vidal, Catalina, Melanio, Manuel, Felicidad, Aurelio, Pacita and Eleuteria, all surnamed Allanigue (being brothers and sisters), brought an action (Civil Case 217-R) before the CFI Rizal against their sister, Lorenza Allanigue, her husband, Simeon Santos, Maria San Agustin and Felicidad San Agustin, for partition of a 360sq. m. lot situated at San Dionisio, Paraaque, Rizal, and for the annulment of certain conveyances involving the same. Defendants having been declared in default, the trial court, after hearing the plaintiffs' evidence, rendered judgment ordering the partition of the lot among the 11 plaintiffs and the defendant Lorenza Allanigue. In a subsequent order the court set off Lorenza Allanigue's share against the amount that she had failed to pay as rents to the plaintiffs as directed in the decision. A writ of execution was issued on the judgment ordering the defendants to vacate the lot and deliver its possession to the plaintiffs. Leonardo Santos, not a party defendant but a son of defendants Simeon Santos and Lorenza Allanigue, owned a house standing on the lot. He filed with the sheriff a third-party claim, and with the court, a motion to recall the writ of execution insofar as his house was concerned. The motion was denied. On 15 March 1962, the defendants and movant Leonardo Santos having failed to remove their houses from the lot within the period given them, the court ordered the sheriff to demolish said houses. On 2 April 1962, Leonardo Santos and the defendants in the case, as petitioners, filed in the Supreme Court a petition for certiorari and prohibition (GR L-19618), against Judge Angel H. Mojica, the Provincial Sheriff of Rizal and the plaintiffs in the case, as respondents. In its decision of 28 February 1964, the Supreme Court denied the petition after finding that Leonardo Santos did not follow the procedure sanctioned by law in vindicating his alleged ownership, i.e., he should have filed an ordinary civil action to vindicate his alleged ownership of the house and the portion of land on which it was built. After the said decision of the Supreme Court had become final, the lower court (Judge Angel H. Mojica), on motion of the plaintiffs in the same Civil Case 217-R, ordered the demolition of the defendants' houses. The defendants having voluntarily removed their houses, the only house that remained standing on the lot was that belonging to Leonardo Santos. Subsequently, the Judge, on motion of the plaintiffs, issued an order dated 9 December 1965, directing the sheriff to demolish the house of Leonardo Santos. Hence, the present petition for certiorari and prohibition in the Supreme Court. The Supreme Court denied the petition, with costs against the petitioner Leonardo Santos. 1. Successor-in-interest bound by the judgment in Civil Case 217-R Leonardo Santos is bound by the judgment in Civil Case No. 217-R because he is a successor-in-interest of his parents, Simeon Santos and Lorenza Allanigue, defendants in Civil Case 217- R, and his right, if any, is claimed under them. The fact that the sale to Leonardo Santos from his parents was registered, is of no moment because, he is bound by the judgment against them. 2. Improvement of the house made after predecessor-in-interest were summoned; Santos a builder in bad faith, no right of indemnity Leonardo Santos' house having been built and reconstructed (after March 1962) into a bigger one after his predecessors-ininterest, his parents, had been summoned in 1959 in Civil Case 217-R, he must be deemed a builder in bad faith. As builder in bad faith he lost the improvement made by him consisting of the reconstructed house to the owners of the land without right to indemnity, pursuant to Article 449 of the Civil Code (He who builds, plants or sows in bad faith on the land of another, loses what is built, planted or sown without right to indemnity.") 3. Options of the landowner in good faith The owners of the land became owners of the improvement consisting of the house built in bad faith if they chose to appropriate the accession. (Article 445 and 449, Civil Code.) However, said owners could choose instead the demolition of the improvement or building at the expense of the builder, pursuant to Article 450 of the Civil Code, which, in part, provides The owner of the land on which anything has been built, planted or sown in bad faith may demand the demolition of the work, or that the planting or sowing be removed, in order to replace things in their former condition at the expense of the person who built, planted or sowed." In the present case, the Allanigue brothers and sisters chose to have the house or improvement built by Leonardo Santos demolished pursuant to their motion for demolition. 4. Res Judicata; Identity of the case The present petition is barred by the prior judgment of the Court in GR L-19618. Petitioner Leonardo Santos, was one of the petitioners in that case against the same official and private respondents in the instant petition; having an identity of subject matter (the portion of the lot and the house standing on said portion alleged by petitioner to belong to him), and identity of cause of action (the order of the Judge for the removal or demolition of the houses standing on the lot). The prior judgment, based on merits, had become final. The judgment thus in GR L-19618 is res judicata in the instant case on the question of the validity of the order of demolition of 9 December 1965. MWSS v. CA [G.R. No. L-54526. August 25, 1986.] Second Division, Feria (J): 4 concur, 1 took no part Facts: The City of Dagupan filed a complaint against the former National Waterworks and Sewerage Authority (NAWASA), now the Metropolitan Waterworks and Sewerage System (MWSS), for recovery of the ownership and possession of the Dagupan Waterworks System. NAWASA interposed as one of its special defenses RA 1383 which vested upon it the ownership, possession and control of all waterworks systems throughout the Philippines and as one of its counterclaims the reimbursement of the expenses it had incurred for necessary and useful improvements amounting to P255,000.00. Judgment was rendered by the trial court in favor of the city on the basis of a stipulation of facts. The trial court found NAWASA to be a possessor in bad faith and hence not entitled to the reimbursement claimed by it. NAWASA appealed to the then Court of Appeals and argued in its lone assignment of error that the city should have been held liable for the amortization of the balance of the loan secured by NAWASA for the improvement of the Dagupan Waterworks System. The appellate court affirmed the judgment of the trial court. MWSS, successor-in-interest of the NAWASA, appealed (petition for review on certiorari) to the Supreme Court raising the sole issue of whether or not it has the right to remove all the useful improvements introduced by NAWASA to the Dagupan Waterworks System, notwithstanding the fact that NAWASA was found to be a possessor in bad faith.

The Supreme Court affirmed the decision of the appellate court, with costs against MWSS. 1. Practice; Common error in joining the court as party respondent in an appeal under Rule 45 of the Rules of Court It is a common error of joining the court (be it a Regional Trial Court, the Intermediate Appellate Court, or the Sandiganbayan) as a party respondent in an appeal by certiorari to this Court under Rule 45 of the Rules of Court. The only parties in an appeal by certiorari are the appellant as petitioner and the appellee as respondent. (Cf. Elks Club vs. Rovira, 80 Phil. 272) The court which rendered the judgment appealed from is not a party in said appeal. It is in the special civil action of certiorari under Section 5 of Rule 65 of the Rules of Court where the court or judge is required to be joined as party defendant or respondent. The joinder of the Intermediate Appellate Court or the Sandiganbayan as party respondent in an appeal by certiorari is necessary in cases where the petitioner-appellant claims that said court acted without or in excess of its jurisdiction or with grave abuse of discretion. An example of this is a case where the petitioner-appellant claims that the Intermediate Appellate Court or the Sandiganbayan acted with grave abuse of discretion in making its findings of fact, thus justifying the review by this court of said findings of fact. (See the exceptions to the rule of conclusiveness of the findings of fact of the Intermediate Appellate Court or the Sandiganbayan in the case of Sacay vs. Sandiganbayan, G.R. Nos. 66497-98, July 10, 1986.) In such a case, the petition for review on certiorari under Rule 45 of the Rules of Court is at the same time a petition for certiorari under Rule 65, and the joinder of the Intermediate Appellate Court or the Sandiganbayan becomes necessary. (Cf. Lianga Lumber Company vs. Lianga Timber Co., Inc., March 31, 1977, 76 SCRA 197). 2. Removability of useful improvement should have in the counterclaim; Pleadings could not be deemed amended to conform to the evidence The procedural objection of the City is technically correct, as that assuming that MWSS (NAWASA) has the right to remove the useful improvements, such improvements were not actually identified, and hence a rehearing would be required which is improper at the current stage of the proceedings; and finally, that such improvements, even if they could be identified, could not be separated without causing substantial injury or damage to the Dagupan Waterworks System; NAWASA should have alleged its additional counterclaim in the alternative - for the reimbursement of the expenses it had incurred for necessary and useful improvements or for the removal of all the useful improvements it had introduced. In the present case, no evidence whatsoever had been introduced by MWSS on the issue of removability of the improvements and the case was decided on a stipulation of facts. Consequently, the pleadings could not be deemed amended to conform to the evidence. 3. Possessor in bad faith does not have the right to remove useful improvements Article 449 of the Civil Code of the Philippines provides that "he who builds, plants or sows in bad faith on the land of another, loses what is built, planted or sown without right to indemnity." As a builder in bad faith, NAWASA lost whatever useful improvements it had made without right to indemnity (Santos vs. Mojica, Jan. 31, 1969, 26 SCRA 703). 4. Only possessor in good faith has right to be refunded for useful expenses with right of retention until reimbursed; or removal of useful improvements without damage to the principal thing Under Article 546 of said code, only a possessor in good faith shall be refunded for useful expenses with the right of retention until reimbursed; and under Article 547 thereof, only a possessor in good faith may remove useful improvements if the can be done without damage to the principal thing and if the person who recovers the possession does not exercise the option of reimbursing the useful expenses. 5. Possessor in bad faith has right to remove improvements for pure luxury or mere pleasure, provided such suffers no injury thereby The right given a possessor in bad faith is to remove improvements applies only to improvements for pure luxury or mere pleasure, provided the thing suffers no injury thereby and the lawful possessor does not prefer to retain them by paying the value they have at the time he enters into possession (Article 549, Id.). 6. Mindanao Academy v. Yap In Mindanao Academy, Inc. vs. Yap (13 SCRA 190), it was held that "if the defendant constructed a new building, as he alleges, he cannot recover its value because the construction was done after the filing of the action for annulment, thus rendering him a builder in bad faith who is denied by law any right of reimbursement." What this Court allowed appellant Yap to remove were the equipment, books, furniture and fixtures brought in by him, because they were outside of the scope of the judgment and may be retained by him. 7. Carbonell v. CA cannot invoke to modify the provisions of the Code; Case is not a precedent The decision in the case of Carbonell vs. Court of Appeals (69 SCRA 99) cannot be invoked to modify the clear provisions of the Civil Code of the Philippines that a possessor in bad faith is not entitled to reimbursement of useful expenses or to removal of useful improvements. In said case, the lower courts found that respondents Infantes were possessors in good faith. On appeal, the First Division of this Court reversed the decision of the Court of Appeals and declared petitioner Carbonell to have the superior right to the land in question. On the question of whether or not respondents Infantes were possessors in good faith, four Members ruled that they were not, but as a matter of equity allowed them to remove the useful improvements they had introduced on the land. Justice Teehankee (now Chief Justice) concurred on the same premise as the dissenting opinion of Justice Muoz Palma that both the conflicting buyers of the real property in question, namely petitioner Carbonell as the first buyer and respondents Infantes as the second buyer, may be deemed purchasers in good faith at the respective dates of their purchase. Justice Muoz Palma dissented on the ground that since both purchasers were undoubtedly in good faith, respondents Infantes' prior registration of the sale in good faith entitled them to the ownership of the land. Inasmuch as only four Members concurred in ruling that respondents Infantes were possessors in bad faith and two Members ruled that they were possessors in good faith, said decision does not establish a precedent. Assistant Secretary for Legal Affairs v. CA [G.R. No. 76761. January 9, 1989.] Second Division, Melencio-Herrera (J): 4 concur

Facts: On 15 April 1948, Jesus M. Larrabaster applied with the National Land Settlement Administration (NLSA) for a home lot at the Marbel Settlement District, Cotabato. On 10 July 1950 Larrabaster's application was granted. Home Lot 336 (later known as Lot 355, 1,500 sq.m.) was allocated to him on the basis of a report of the supervisor of the Settlement District that the lot was vacant and free from any claim or conflict. Larrabaster leased the lot to Basilio Mendoza and tolerated Jorge Geller to squat on the portion thereof. On 25 November 1952 the Land Settlement and Development Corporation (LSDC) took over the functions of the NLSA. On 29 June 1956 Larrabaster and his wife assigned their rights and interests over Lot 336 to Jose B. Pena. Notwithstanding the transfer, Pena allowed Mendoza and Geller to stay on the lot. On 8 September 1956 a Supplementary Deed of Sale was executed by the same parties defining the boundaries of Lot 355 (formerly 336: N by Bulok creek and a street; S by Bulok creek and the National Highway; E by a street beside the public plaza; and W by Bulok creek). On 18 June 1954, RA 1160 transferred the custody and administration of the Marbel Townsite to the National Resettlement and Rehabilitation Administration (NARRA). On 20 August 1956 Pena requested NARRA to approve the transfer of rights but the latter did not act thereon in view of Proclamation 336, s. 1956, returning to the Bureau of Lands the disposition of the lots which remained unallocated by the LSDC at the time of its abolition. The Bureau of Lands did not act on Pena's request either, prompting him to bring up the matter to the Board of Liquidators (BOL), which was created to wind up the affairs of LSDC. Although LSDC had initially denied the request, it subsequently confirmed the sale to Pena in its Resolution 139, s. 1964. Pena must have realized that the property contained an area bigger than 1,500 sq. ms., hence, his request to BOL that the area be adjusted from 1,500 to 3,616.93 sq. ms. to conform to its actual area. In its Resolution 139, s. 1964, the BOL denied the request. Pena moved for reconsideration stressing that the award should be for 3,616.93 sq. ms., but the BOL again denied the same under its Resolution 439, s. 1967. Pena appealed to the Office of the President. The BOL conducted an investigation and reported that Lot 355, as awarded to Larrabaster, contained only 1,500 sq. ms. but due to accretion, since the lot was almost surrounded by a creek, the area increased to 3,616.93 sq. ms.; and since home lots had an average area of 1,500 sq. ms. only, the Bureau of Lands subdivided the property into 3 parts, namely: Lot 107 (1,455 sq. ms., allocated to Basilio Mendoza); Lot 108, (1,500 sq. ms., allocated to Pena); and Lot 109 (661 sq. ms., allocated to Arturo Roxas). The BOL then recommended that Pena be awarded Lot 108 instead of the whole of former Lot 355. Excepting to this, Pena alleged that the lot transferred to him by Larrabaster contains 3,616.93 and not 1,500 sq. ms., this being the area embraced within the boundaries described in the Supplementary Deed of Sale executed between him and Larrabaster on 8 September 1956. On 10 February 1969 the Office of the President ordered that the area of Pena's lot (Lot 108) be maintained at 1,500 sq. ms. on the premise that accretion belonged to the Government. Upon Pena's motion for reconsideration, the same Office, on 13 May 1969, modified its Decision and held that the award to Pena of the original Lot 355 is maintained; reasoning that the benefits of accretion accrue to the owner, Pena, and not to the Government (Article 457). On 14 May 1969 the BOL approved Resolution 236, s. 1969, directing its LSDC unit to advise Pena accordingly. And on 3 September 1969 the BOL recommended to the Director of Lands the issuance of a patent in Pena's favor. On 1 August 1969 Mendoza addressed a letter-protest to the BOL, to which the latter in its Resolution 488, dated 6 August 1969, responded by advising Mendoza to direct its protest to the Office of the President. Mendoza did so and on 28 September 1971 said Office rendered its letter-decision affirming its 13 May 1969 decision. In the meantime, on 27 January 1970, and while his protest with the Office of the President was still pending, Mendoza resorted to Civil Case 98 for Certiorari before the then CFI Cotabato against the public officials and Pena. On 23 June 1978, Mendoza followed up with a Supplemental Petition to annul the administrative Decision of 20 September 1971 denying his protest. On 10 May 1985 the Trial Court rendered its Decision in Civil Case 98 dismissing Mendoza's Petition for Certiorari. On appeal, the Court of Appeals reversed the Trial Court in its 28 November 1986 Decision, without prejudice to the reopening of the administrative case in said Office as to accord all parties concerned, including Mendoza, their constitutional rights to due process of law. The Supreme Court set aside the Decision of 28 November 1986 of the Court of Appeals, and reinstated the Decision of 10 May 1985 of the RTC (Branch 24, Koronadal, South Cotabato, Civil Case 98). 1. Due process; Decision of the Office of the President not vitiated After the Office of the President had rendered its Decision dated 13 May 1969, Mendoza filed a letter-protest on 1 August 1969 with the BOL. The latter office directed him to file his protest with the Office of the President, which he did. On 28 September 1971, Mendoza's request for reconsideration was denied by said Office. These observations do not justify the conclusion that the decisions of the Office of the President were vitiated; allegedly by failure to accord due process of law to Mendoza, i.e. that he was not made a party to the administrative case, not served with a copy of the 10 February 1969 Decision; and not notified of proceedings before the 13 May 1969 decision nor served a copy thereof. 2. Assuming absence of notice and opportunity to be present in proceedings: Defect cured Even assuming that there was absence of notice and opportunity to be present in the administrative proceedings prior to the rendition of the 10 February 1969 and 13 May 1969 Decisions by the Office of the President, such procedural defect was cured when Mendoza elevated his letter protest to the Office of the President, which subjected the controversy to appellate review but eventually denied reconsideration. Having thus been given a chance to be heard with respect to his protest there is sufficient compliance with the requirements of due process. (See also Dormitorio v. Fernandez, L-25897, August 21, 1976, 72 SCRA 388, 394-395; Montemayor vs. Araneta Univ. Foundation, L-44251, May 31, 1977, 77 SCRA 321 [1977]; also Sumpang v. Inciong, L50992, June 19, 1985, 137 SCRA 56 [1985]). 3. Due process; Pending administrative request for reconsideration abandoned in favor of judicial proceedings Mendoza had abandoned his pending administrative request for reconsideration in favor of judicial proceedings, when he filed his petition for certiorari before the CFI Cotabato, which was presented while his request for reconsideration with the Office of the President, involving the same Decision of 13 May 1969, was pending. Mendoza cannot justifiably claim that he was denied due process.

4. Accretion; Article 457 also applies to creeks Upon re-study, the Office of the President modified its conclusions in its Decision of 13 May 1969by taking into account Article 457 of the Civil Code, which provides that "to the owners of lands adjoining the banks of rivers belong the accretion which they gradually receive from the effects of the current of the waters." It also opined that creeks are included within the meaning of this Civil Code provision." 5. Accretion belongs to riparian owner, even if it occurred prior to completion of installment payment While it may be conceded that Lot No. 355 technically belongs to the government because it was bought from the latter under an installment plan, it cannot be rightfully concluded that the benefits of accretion must still be retained by the said seller. In Director of Lands, et al. vs. Ricardo Rizal, et al. (87 Phil. 806, at 810, 814 [1950]), it was held that "When the lot bordering on a public stream is sold on installment basis by the government, said stream is made the boundary. The stream may advance or recede but it will always constitute the boundary or boundaries of the lot, and the purchaser has the right to insist that the original boundaries be preserved, and all the area inside said boundaries be considered as included in the sale; and that "In the sale of a friar land, lot or parcel bordering on rivers under Act. No. 1120 pending payment in full of the purchase price, although the government reserves title thereto, merely for its protection, the beneficial and equitable title is in the purchaser, and that any accretion by the lot even before payment of the last installment belongs to the purchaser thereof." 6. Authority of Bureau of Lands limited to the disposal of lots in unallocated areas Under Proclamation 336, s. 1956, the authority of the Bureau of Lands to dispose of lots was limited to "unallocated areas." Since the property no longer belonged to the Government the subdivision thereof by the Bureau of Lands into 3 lots, as well as the allocation of said lots to two other individuals, was beyond the scope of its authority. Larrabaster had already acquired the beneficial and equitable title over the Lot 355, albeit the Government still retained the naked title thereto. Consequently, to Larrabaster and now to his assignee (Pea) belong the accretions to said lot which may no longer be allocated to others by the Government. He is entitled to all the benefits which may accrue to the land as well as suffer the losses that may befall it. 7. Mendozas filing of Miscellaneous Sales Application inappropriate and without legal force and effect Mendoza's filing of a Miscellaneous Sales Application over the Disputed Property with the Bureau of Lands on 6 November 1962 is inappropriate and without any legal force and effect since the same was no longer public land subject to disposition by the Government. 8. No irregularity that can be imputed to the administrative decisions No irregularity may be imputed to the administrative decisions by reason of the fact that allegedly a copy of the investigation report of the BOL was not among those elevated to the Trial Court or among those marked in evidence. The Office of the President could not have relied upon said report if the same had not been before it when it rendered the questioned Decisions. 9. Finding of facts of Executive Branch given respect as long as supported by substantial evidence In reviewing administrative decisions of the Executive Branch of the government, the findings of fact made therein must be respected, as long as they are supported by substantial evidence, even if not overwhelming or preponderant (Ang Tibay vs. Court of Industrial Relations, 69 Phil. 635 [1940]); that it is not for the reviewing court to weigh the conflicting evidence, determine the credibility of the witnesses, or otherwise substitute its own judgment for that of the administrative agency on the sufficiency of the evidence (Lao Tang Bun, et al. vs. Fabre, 81 Phil. 682 [1948]); that the administrative decision in matters within the executive jurisdiction can only be set aside on proof of gross abuse of discretion, fraud, or error of law (Lovina vs. Moreno, L-17821, November 29, 1963, 9 SCRA 557; Timbancaya vs. Vicente, L-19100, December 27, 1963, 9 SCRA 852), which were absent in the present case. 10. Reopening the case will lead to protracted litigation To reopen the case as ordered by the Court of Appeals would open wide the doors to a protracted litigation of a controversy that has been pending for approximately 19 years. Ferrer vs. Bautista [G.R. No. 46963. March 14, 1994.] Third Division, Vitug (J): 4 concur Facts: On 25 November 1956, Gloria A. Ferrer filed with the CFI La Union, Branch III, a complaint for reivindicacion (Civil Case A-86) against Mariano Balanag and Magdalena Domondon. Judge Antonio G. Bautista dismissed, on 10 February 1976, the complaint, without prejudice, on the ground that the court had no authority to cancel or annul the decree and the title issued by the Director of Lands on the basis of a mere collateral attack. Gloria A. Ferrer claims ownership a strip of land south of Lot 1980 of the Cadastral survey of Aringay, La Union by virtue of accretion, she being the owner of Lot 1980 covered by TCT T-3280, which is immediately north of the land in question. On the other hand, Mariano Balanag and Magdalena Domondon equally assert ownership over the property on account of long occupation and by virtue of Certificate of Title P-168, in the name of Magdalena Domondon, pursuant to Free Patent 309504 issued on 24 January 1966. On 23 March 1976, Ferrer filed a complaint with Branch III of the then CFI La Union to "Quiet Title to Real Property" against Balanag and Domondon (Civil Case A-514). On 07 December 1976, Judge Bautista issued an order dismissing Ferrer's complaint; because it constitutes a collateral or indirect attack on the Free Patent and Original Certificate of Title. Ferrer filed a motion for reconsideration but was denied on 3 May 1977. Pursuant to the Supreme Court's Resolution, dated 19 August 1977, Ferrer was allowed to file the petition for review on certiorari under RA 5440 considering that only questions of law had been raised. On 03 August 1978, the Court dismissed the petition for lack of interest due to the failure of Ferrer's counsel to submit the requisite memorandum in support of the petition. In a Resolution dated 28 September 1978, however, the Court resolved to reconsider the dismissal and to reinstate the petition.

The Supreme Court reversed and set aside the questioned order of dismissal of the trial court (Civil Case 514-A), and rendered judgment declaring Ferrer to be the owner of the disputed parcel of land and ordering Balanag and Domondon to reconvey the same to Ferrer; without costs. 1. Ferrer the lawful owner of accretion Article 457 of the Civil Code provides that to the owners of lands adjoining the banks of rivers belong the accretion which they gradually receive from the effects of the current of the waters." Undoubtedly, Ferrer is the lawful owner of the accretion, she being the registered owner of Lot 1980 which adjoins the alluvial property. Parenthetically, the same finding has also been made by the trial court in Civil Case A-86. 2. Alluvion belongs to riparian owner; Rationale for the rule Alluvion gives to the owners of lands adjoining the banks of rivers or streams any accretion which is gradually received from the effects of the current of waters (Art. 457, Civil Code; Tuason vs. Court of Appeals, 147 SCRA 37; Cureg vs. IAC, 177 SCRA 313). The rationale for the rule is to provide some kind of compensation to owners of land continually exposed to the destructive force of water and subjected to various easements (Agustin vs. IAC, 187 SCRA 218; Binalay vs. Manalo, 195 SCRA 374). 3. Director of lands has no authority to grant a free patent over land which ceased to be public land; Title issued null and void The Director of Lands has no authority to grant a free patent over land that has passed to private ownership and which has thereby ceased to be public land. Any title thus issued or conveyed by him would be null and void (Tuason vs. Court of Appeals, 147 SCRA 37). The nullity arises, not from fraud or deceit, but from the fact that the land is no longer under the jurisdiction of the Bureau of Lands, the latter's authority being limited only to lands of public dominion and not those that are privately owned (Agne vs. Director of Lands, 181 SCRA 793). In the present case, Balanag and Domondon acquired no right or title over the disputed land by virtue of the free patent since at the time it was issued in 1966, it was already private property and not a part of the disposable land of the public domain. 4. Title incontrovertible 1 year after it is issued pursuant to a public grant; Does not apply if issuance is null and void; Action to declare title void does not prescribe Although, ordinarily, a title becomes incontrovertible one year after it is issued pursuant to a public grant, the rule does not apply when such issuance is null and void. An action to declare the nullity of that void title does not prescribe (Agne vs. Director of Lands, supra); in fact, it is susceptible to direct, as well as to collateral, attack (Estoesta, Sr. vs. Court of Appeals, 179 SCRA 203). 5. 10 year prescription period applies to action for reconveyance if it is based on an implied or constructive trust; In the present case, the judicial relationship is a patent nullity The ten-year prescriptive period is applicable to an action for reconveyance if, indeed, it is based on an implied or constructive trust. Article 1456 of the Civil Code, upon which a constructive trust can be predicated, cannot be invoked, however, since the public grant and the title correspondingly issued to Balanag and Domondon that can create that juridical relationship is a patent nullity. 6. Assuming, Prescriptive period interrupted when action is filed in court or if action is pending Even assuming, nonetheless, that a constructive trust did arise, the running of the prescriptive period is to be deemed interrupted when an action is filed in court (Art. 1155, Civil Code) or, obviously, when one is already there pending. In the present case, the Free Patent was issued on 24 January 1966 and OCT P-168 was transcribed in the Registration Book of La Union on 08 February 1966. At that time, Civil Case A-86 for reivindicacion between the parties was still pending in court. After Civil Case A-86 was dismissed, without prejudice, on 10 February 1976, Ferrer, on 22 March 1976, promptly filed Civil Case A-514. 7. Balanag and Domondon cannot claim ownership by acquisitive prescription Balanag and Domondon claim ownership of the disputed property by acquisitive prescription. Ownership and other real rights over immovable property are acquired by ordinary prescription through possession of 10 years if the adverse possession is with a just title and the possession is in good faith. Ownership and other real rights over immovables also prescribe through uninterrupted adverse possession thereof for thirty years, this time without need of title or of good faith. (See Art. 1134, Civil Code.) Given the settings in the present case, the applicable period of acquisitive prescription, if at all, would be thirty years. That possession, for purposes of acquisitive prescription, was deemed interrupted upon their receipt of summons (Art. 1123, Civil Code) in Civil Case A-86 pending since 1965, as well as Civil Case A-514 filed in 1976 following the dismissal the month previous of Civil Case A-86. 8. Expeditious administration of justice Where the determinative facts are before the Supreme Court, and it is in a position to finally resolve the dispute, the expeditious administration of justice will be subserved by the resolution of the case and thereby obviate the needless protracted proceedings consequent to the remand of the case to the trial court (Heirs of Crisanta Almoradie, et al. vs. Court of Appeals, et al., G.R. No. 91385, January 4, 1994; Lianga Bay Logging Co., et al. vs. Court of Appeals, 157 SCRA 357; Escudero vs. Dulay, 158, SCRA 69). Where the owner of the land is determined, the court then, in the exercise of its equity jurisdiction may, instead of remanding the case to the trial court, direct the owner to reconvey the disputed parcel to its lawful owner (Limaza vs. IAC, 182 SCRA 855; Agne vs. Director of Lands, supra). Considering, moreover, the length of time that the case has been pending between the parties, an order from the Supreme Court requiring such reconveyance can certainly be just and warranted. Agustin v. IAC [G.R. Nos. 66075-76. July 5, 1990.] First Division, Grino-Aquino (J): 4 concur

Facts: The Cagayan River separates the towns of Solana on the west and Tuguegarao on the east in the province of Cagayan. In 1919 the lands east of the river were covered by the Tuguegarao Cadastre. In 1925, OCT 5472 was issued for land east of the Cagayan River owned by Eulogio Agustin. As the years went by, the Cagayan River moved gradually eastward, depositing silt on the western bank. The shifting of the river and the siltation continued until 1968. In 1950, all lands west of the river were included in the Solana Cadastre. Among these occupying lands covered by the Solana Cadastre were Pablo Binayug and Maria Melad. Binayug was in possession since 1947 of Lots 3349, 7875 to 7879, 7881 to 7885, 7891 and 7892. It is has an area of 8 hectares planted to tobacco and corn and another 12 hectares overgrown with talahib. Binayug's Homestead Application W-79055 over this land was approved in 1959 and his possession recognized in the decision in Civil Case 101. On the other hand, as a result of Civil Case 343-T, Macario Melad, the predecessor-in-interest of Maria Melad and Timoteo Melad, was issued OCT P-5026 for Lot 3351 of Cad. 293 on 1 June 1956. Through the years, the Cagayan River eroded lands of the Tuguegarao Cadastre on its eastern bank among which was Agustin's Lot 8457, depositing the alluvium as accretion on the land possessed by Binayug on the western bank. However, in 1968, after a big flood, the Cagayan River changed its course, returned to its 1919 bed, and, in the process, cut across the lands of Maria Melad, Timoteo Melad, and the spouses Pablo Binayug and Geronima Ubina whose lands were transferred on the eastern, or Tuguegarao, side of the river. To cultivate those lots they had to cross the river. In April 1969, while the Melads, Binayug, Urbina and their tenants were planting corn on their lots located on the eastern side of the Cagayan River, Agustin, the Heirs of Baldomero Langcay, Juan Langcay, and Arturo Balisi, accompanied by the mayor and some policemen of Tuguegarao, claimed the same lands as their own and drove away the Melads, Binayug and Urbina from the premises. On 21 April 1970, Maria and Timoteo Melad filed a complaint (Civil Case 343-T) to recover Lot 3351 with an area of 5 hectares and its 6.6-hectare accretion. On 24 April 1970, Pablo Binayug filed a separate complaint (Civil Case 344-T) to recover his lots and their accretions. On 16 June 1975, the trial court rendered a decision in Civil Case 343-T, ordering Eulogio Agustin, Gregorio Tuliao, Jacinto Buquel and Octavio Bancud, their representatives or agents to vacate Lot 3351 of Solana Cadastre together with its accretion consisting of portions of Lots 9463, 9462 and 9461 of Tuguegarao Cadastre and to restore ownership in favor of Maria Melad and Timoteo Melad who are the only interested heirs of Macario Melad. The trial court likewise ordered, in Civil Case 344-T, Justo Adduru, Andres Pastor, Teofilo Tagacay, Vicente Camilan, Nicanor Mora, Baldomero Cagurangan, Domingo Quilang, Cesar Cabalza, Elias Macababbad, Titong Macababbad, Arturo Balisi, Jose Allabun, Eulogio Agustin, Banong Aquino, Junior Cambri and Juan Langoay, their representatives or agents to vacate Lots 3349, 7875 to 7879, 7881 to 7885, 7891 and 7892, together with its accretion and to restore possession to Pablo Binayug and Geronimo Urbina. Without pronouncement as to damages which were not properly proven and to costs. Eulogio Agustin appealed the decision in Civil Case 343-T, while Eulogio Agustin, Baldomero Cagurangan (substituted by his heir), Arturo Balisi and Juan Langcay appealed the decision in Civil Case 344-T. But upon motion of the Melads, Binayug and Urbina, the trial court ordered on 15 August 1975 the execution pending appeal of the judgment in Civil Case 344-T against Cagurangan, Balisi and Langcay on the ground that their appeal was dilatory as they had not presented evidence at the trial. On 29 November 1983, the Intermediate Appellate Court rendered a decision affirming in toto the judgment of the trial court, with costs against the Agustin, Cagurangan, Balisi and Langcay. Hence, the petition for review. The Supreme Court denied the petition for lack of merit, and affirmed the decision of the IAC, now CA; with costs against Agustin, et.al. 1. Findings of fact of the Court of Appeal conclusive with the Supreme Court The finding of the Court of Appeals that there had been accretions to the lots of the Melads, Binauyg and Urbina who did not lose the ownership of such accretions even after they were separated from the principal lots by the sudden change of course of the river, is a finding of fact which is conclusive on this Court. That finding is supported by Art. 457 of the New Civil Code which provides that "to the owners of lands adjoining the banks of rivers belong the accretion which they gradually receive from the effects of the current of the waters. (366)" 2. Conditions for accretion to benefit a riparian owner Accretion benefits a riparian owner when the following requisites are present: (1) that the deposit be gradual and imperceptible; (2) that it resulted from the effects of the current of the water; and (3) that the land where accretion takes place is adjacent to the bank of a river (Republic vs. CA, 132 SCRA 514). In the present case, the accretion on the western bank of the Cagayan River had been going on from 1919 up to 1968 or for a period of 49 years. It was gradual and imperceptible. Only when Lot 3351, with an original area of 5 hectares described in the free patent that was issued to Macario Melad in June 1956, was resurveyed in 1968 did it become known that 6.6 hectares had been added to it. Lot 3351, covered by a homestead patent issued in June 1950 to Pablo Binayug, grew from its original area of 18 hectares, by an additional 50 hectares through alluvium as the Cagayan River gradually moved to the east. These accretions belong to riparian owners upon whose lands the alluvial deposits were made (Roxas vs. Tuason, 9 Phil. 408; Director of Lands vs. Rizal, 87 Phil. 806). 3. Reason for the principle of accretion benefiting a riparian owner The reason for the principle is because, if lands bordering on streams are exposed to floods and other damage due to the destructive force of the waters, and if by virtue of law they are subject to encumbrances and various kinds of easements, it is only just that such risks or dangers as may prejudice the owners thereof should in some way be compensated by the right of accretion (Cortes vs. City of Manila, 10 Phil. 567). 4. Ownership of accretion not lost upon sudden and abrupt change of the river The' ownership of the accretion to the lands was not lost upon the sudden and abrupt change of the course of the river (Cagayan River in 1968 or 1969 when it reverted to its old 1919 bed), and separated or transferred said accretions to the other side (or eastern bank) of the river. Articles 459 and 463 of the New Civil Code apply to this situation. Article 459 provides that whenever the current of a river, creek or torrent segregates from an estate on its bank a known portion of land and transfers it to another estate, the owner of the land to which the segregated portion belonged retains the ownership of it, provided that he removes the same within two years." Article 463 provides that whenever the current of a river divides itself into branches, leaving a piece of

land or part thereof isolated, the owner of the land retains his ownership. He also retains it if a portion of land is separated from the estate by the current. Reynante v. CA [G.R. No. 95907. April 8, 1992.] Second Division, Paras (J): 5 concur Facts: More than 50 years ago, Jose Reynante was taken as tenant by the late Don Cosme Carlos, over a fishpond located at Barrio Liputan, Meycauayan, Bulacan with an area of 188.711 sq. m. (TCT 25618, Land Registry of Bulacan). During the tenancy, Reynante constructed a nipa hut where he and his family lived and took care of the nipa palms (sasahan) he had planted on lots 1 and 2 covering an area of 5,096 sq. m. and 6,011 sq. m. respectively. These lots are located between the fishpond covered by TCT 25618 and the Liputan (formerly Meycauayan) River. Reynante harvested and sold said nipa palms without interference and prohibition from anybody. Neither did the late Don Cosme Carlos question his right to plant the nipa palms near the fishpond or to harvest and appropriate them as his own. After the death of Don Cosme Carlos, his heirs entered into a written agreement denominated as Sinumpaang Salaysay ng Pagsasauli ng Karapatan dated 29 November 1984 with Reynante whereby the latter for and in consideration of the sum of P200,000 turned over the fishpond he was tenanting to the heirs of Don Cosme Carlos and surrendered all his rights therein as caretaker or "bantay-kasama at tagapamahala. Pursuant to the said written agreement, Reynante surrendered the fishpond and the 2 huts located therein to the heirs of Don Cosme Carlos. The heirs of Leoncio and Dolores Carlos, and the heirs of Gorgonio and Concepcion Carlos thereafter leased the said fishpond to one Carlos de la Cruz. Reynante continued to live in the nipa hut constructed by him on lots 1 and 2 and to take care of the nipa palms he had planted therein. On 17 February 1988, the heirs formally demanded that Reynante vacate said portion since the latter had already been indemnified for the surrender of his rights as a tenant. Despite receipt thereof, Reynante refused and failed to relinquish possession of lots 1 and 2. On 22 April 1988, the heirs filed a complaint for forcible entry with preliminary mandatory injunction against Reynante with the MTC Meycauayan Bulacan (Branch 1, 3rd Judicial Region, Civil Case 1526) alleging that the latter by means of strategy and stealth, took over the physical, actual and material possession of lots 1 and 2 by residing in one of the kubos or huts bordering the Liputan River and cutting off and/or disposing of the sasa or nipa palms adjacent thereto. On 10 January 1989, the trial court rendered its decision dismissing the complaint and finding that Reynante had been in prior possession of lots 1 and 2. The heirs appealed to the RTC Malolos Bulacan (Branch 8, 3rd Judicial Region) and on 8 August 1989 it rendered its decision in favor of the heirs, and reversed the decision of the lower court. The Court ordered Reynante to restore possession of the piece of land, together with the sasa or nipa palms planted theron; without pronouncement as to attorney's fees, and each party bearing their respective costs of the suit. From said decision, Reynante filed with the Court of Appeals a petition for review. On 28 February 1990, the Court of Appeals rendered its decision (CA-GR 19171), affirming the decision of the lower court in toto, and thus denied the petition seeking to issue a restraining order. On 5 November 1990, the Court of Appeals denied the motion for reconsideration filed by Reynante. Hence, the petition for review on certiorari. The Supreme Court reversed and set aside the decision of the Court of Appeals dated 28 February 1990 and reinstated the decision of the MTC Meycauayan, Bulacan (Branch I). 1. Action for forcible entry An action for forcible entry is merely a quieting process and actual title of the property is never determined. A party who can prove prior possession can recover such possession even against the owner himself. Whatever may be the character of his prior possession, if he has in his favor priority in time, he has the security that entitles him to remain on the property until he is lawfully ejected by a person having a better right by accion publiciana or accion reinvindicatoria (German Management & Services, Inc. v. Court of Appeals, G.R. No. 76216, September 14, 1989, 177 SCRA 495, 498, 499). On the other hand, if a plaintiff cannot prove prior physical possession, he has no right of action for forcible entry and detainer even if he should be the owner of the property (Lizo v. Carandang, 73 Phil. 469 [1942]). In the present case, the Court of Appeals could not legally restore the heirs' possession over lots 1 and 2 simply because Reynante has clearly proven that he had prior possession over lots 1 and 2. 2. Reynante in prior possession Reynante was in possession of the questioned lots for more than 50 years. He was the caretaker of the fishpond owned by the late Don Cosme Carlos for more than 50 years and that he constructed a nipa hut adjacent to the fishpond and planted nipa palms therein. This fact is bolstered by the "Sinumpaang Salaysay " executed by Epifanio Lucero, Apolonio D. Morte, and Carling Dumalay, all of whom are disinterested parties with no motive to falsify that can be attributed to them, except their desire to tell the truth. Moreover, an occular inspection was conducted by the trial court dated 2 December 1988 which was attended by the parties and their respective counsels. The court observed that the controversial premises is beyond the titled property of the plaintiffs but situated along the Liputan, Meycauayan River it being a part of the public domain. On the other hand, the heirs based their claim of possession over lots 1 and 2 simply on the written agreement signed by petitioner whereby the latter surrendered his rights over the fishpond. There is nothing, however, on the document that the tenant was giving other matters not mentioned in the document. Neither was there any mention of the hut and nipa palms for such to be included in the subsequent least to de la Cruz, a a circumstance that gives the impression that said hut and palms do not belong to the heirs. 3. Disputed lands not included in TCT 25618 The disputed lots involved in the present case are not included in TCT 25618 as per verification made by the Forest Management Bureau, Department of Environment and Natural Resources. That tract of land situated at Barrio Liputan, Meycauayan, Bulacan containing an area of 1.1107 hectares as described in the plan prepared and surveyed by Geodetic Engineer Restituto Buan for Reynante falls within Alienable and Disposable Land (for fishpond development) under Project 15 per B.F.L.C. Map 3122 dated 8 May 1987.

4. Requisites of accretion Accretion benefits a riparian owner when the following requisites are present: (1) that the deposit be gradual and imperceptible; (2) that it resulted from the effects of the current of the water; and (c) that the land where accretion takes place is adjacent to the bank of a river (Republic v. Court of Appeals, G.R. No. L-61647, October 12, 1984, 132 SCRA 514, cited in Agustin v. Intermediate Appellate Court, G.R. Nos. 66075-76, July 5, 1990, 187 SCRA 218). 5. Accretion does not automatically become registered land Granting without conceding that lots 1 and 2 were created by alluvial formation and while it is true that accretions which the banks of rivers may gradually receive from the effect of the current become the property of the owner of the banks (Article 457), such accretion to registered land does not preclude acquisition of the additional area by another person through prescription. In Ignacio Grande, et al. v. Hon. Court of Appeals, et al. (GR L-17652, 30 June 1962, 115 Phil. 521) it was held that "an accretion does not automatically become registered land just because the lot which receives such accretion is covered by a Torrens Title. Ownership of a piece of land is one thing; registration under the Torrens system of that ownership is another. Ownership over the accretion received by the land adjoining a river is governed by the Civil Code. Imprescriptibility of registered land is provided in the registration law. Registration under the Land Registration and Cadastral Act does not vest or give title to the land, but merely confirms and, thereafter, protects the title already possessed by the owner, making it imprescriptible by occupation of third parties. But to obtain this protection, the land must be placed under the operation of the registration laws, wherein certain judicial procedures have been provided." In the present case, assuming that the heirs had acquired the alluvial deposit (the lot in question), by accretion, still their failure to register said accretion for a period of 50 years subjected said accretion to acquisition through prescription by third persons. Roxas v. Tuason [G.R. No. L-3788. December 21, 1907.] First Division, Torres (J): 4 concur Facts: On 19 February 1906, attorneys Rosado, Sanz & Opisso, on behalf of Pedro P. Roxas, applied for the registration of the estate owned by Roxas (Hacienda de San Pedro Macati) in accordance with the provisions of the Land Registration Act. Said hacienda was acquired by Roxas by inheritance under the will of his late father, Jose Bonifacio Roxas, y Ubaldo. The property consists of 4 parcels of land, irregular shape, designated on the accompanying plan under the letters "A" to "D", containing a total area of 1,761 hectares 51 ares and 5 centares, equivalent to 17,615,105 sq.m., and assessed at P415,221.34, of which P59,904 corresponded to the portion of said hacienda included within the limits of the city of Manila and P256,769 corresponded to that portion situated in the Province of Rizal. The building constructed of strong materials, called the "Casa-Quinta" or "Casa de Ingenieros," belonging also to Roxas, is erected within parcel "C," occupying, together with its appurtenances, an area of 8,430 sq.m., and was assessed at P98,557.34. The hacienda was not mortgaged nor that any person has any right to or any interest therein; and it is almost wholly occupied, under lease, by about 429 tenants whose names, residences, and postal addresses, as well as the residence of the owner of the property and of his attorney in fact, are stated in the application. On 24 April, Roxas requested the summoning of the persons therein named, and stated in addition that the total area of the hacienda is 17,613,595.91 sq.ms., as specified in the corrections made to the technical description. On 24 July 1906, Roxas amended his application and gave the postal address and names of several occupants of the property; and by other amendments to his original petition dated 30 August and 25 September 1906, rectifications are made in the boundaries of the hacienda, the last of which represents a decrease of 1,446.70 sq.ms., or 14 ares and 46.70 centares which must be deducted from the original description. The owners of the adjoining properties having been summoned and notified by means of subpoenas and notices published in the daily papers, one of them, Julia Tuason, appeared and by a document dated 10 September 1906, set forth her opposition to the registration and authentication of the title of Roxas, as regards the parcel marked "C," for the reason that two old monuments which had separated their respective properties had been pulled down and new ones erected without her consent, and in her opinion the latter included a considerable portion of the land owned by her. The municipality of San Pedro Macati also filed opposition to the requested registration, alleging that the land occupied by the municipal building and the public school had been in the possession of the town from time immemorial, and that all the land occupied by roads, highways, lanes, and public landing places belonged to the public domain and should be excluded from registration in favor of Roxas. On 18 September 1906, the attorney for Alejandro Aguirre and Consolacion Aguirre also filed opposition to said application for registration alleging that the 2 parcels of land owned by them had been improperly included within the bounds of said hacienda in the parcel marked "C," the second said parcels, which is the only subject of the respective bill of exceptions and appeal interposed by them, consists of a building lot situated in Calle San Pedro, opposite the first parcel of land, which was the subject of another bill of exceptions and appeal by the Roxas; said second parcels measures 10 meters and 87 centimeters on its front and rear, and 9 meters and 20 centimeters along each of its sides, its boundaries being stated. On 17 October 1906, the court rendered decision, overruling the opposition made by Julia Tuason, by the municipality of San Pedro Macati, and by Alejandro and Consolacion Aguirre as to the second parcel, and ordered the registration of the Hacienda of San Pedro Macati in favor of Pedro Roxas excluding the parcel of land with a frontage of 23 Spanish yards and a depth of 24 Spanish yards occupied by the municipal building, which the government has the right to use without the payment of rent therefor, so long as the same is occupied by the said building or by another in substitution thereof and used for the public good and for official purposes. Julia Tuason, the municipality of San Pedro Macati, and Alejandro and Consolacion Aguirre, excepted to said judgment and moved for a new trial on the ground that the same was contrary to law and to the weight of the evidence; said motion was overruled, Tuason, Aguirre and the municipality of San Pedro Macati again excepting. The respective bills of exceptions having been presented, the same were forwarded in the ordinary manner to the Supreme Court. The Supreme Court affirmed the judgment appealed from as regards Julia Tuason and the municipality of San Pedro Macati, declared the appeal of Alejandro and Consolacion Aguirre to be abandoned, and ordered Tuason, the Aguirres and the municipality of San Pedro to pay their respective share of the costs. 1. No evidence showing Julia Tuason owns the strip of land on the bank of the creek; Relief is application of survey The record does not show that the boundary of the land of Julia Tuason was inclosed by monuments belonging to her or that the

creek which divides the sitio or Island of Suavoy from the land of the said hacienda is included within the Tuason's land, since in the bill of sale executed by the procurador general of the Augustinian friars on 28 March 1893, to Julia Tuason, no mention is made of monuments erected thereon nor of any creek existing in the large tract of land purchased by her, except that the land is situated in the barrio of Suavoy and that it is bounded on two sides by the Hacienda of San Pedro Macati. Nor does the record show that there was more land on the side of the hacienda, forming part of the barrio or sitio of Suavoy, not included in the tract acquired by Tuason from the Augustinian Fathers, and that said creek traversed said barrio from one end to the other, or the Tuason's land, in order to affirm on good grounds that her land extended to the opposite bank of the aforesaid creek. Thus, from the fact that the land of Julia Tuason was bounded on two sides by the Hacienda of San Pedro Macati it does not follow that the strip of a few meters in width on the bank of the creek belonged to her, there being no evidence in support thereof, and if her statement were true, she would have applied for a survey and demarcation of her property in accordance with the area of the same stated in her title deed; and if she did not do so it must be because she renounces its verification in this manner or for some other reason. 2. Strip of land occupied by tenants of the hacienda The strip of land, irregular in shape, running parallel to the creek and forming a portion of its bank, has always been occupied by tenants of the hacienda as being an integral portion thereof, even at the time when the land owned by Julia Tuason belonged to the Augustinian Fathers, the original owners thereof. The haciendas tenants were never molested or interfered with by the Augustinian Fathers or their tenants, nor later by Julia Tuason or her tenants when cultivating the strip of land. Said tenants when cultivating the land did not cross the creek, it being recognized as the boundary line between both properties. 3. Creek wider in 1871 In 1871 the said creek was wider, having then a width of about 4 Spanish yards, small bancas plying on it around the Island of Suavoy,. and some of the monuments of the hacienda were 4 meters distant from the bank, others 2 meters, and some 1 meter. The proven fact that said creek was wider in 1871 is the best explanation as to why some of the monuments of the Hacienda of San Pedro Macati are at some distance from the bank of the same. 4. No legal reason for Tuason to own strip of land No legal reason whatever exists why the slow increase which has taken place on the hacienda's side should be considered as belonging to Tuason, inasmuch as the latter does not own the bed of the creek and because it may be assumed that the slow decrease in the width thereof benefited both properties equally since Tuason has not been able to show or prove that her land has been thereby reduced. 5. Accretions belong to riparian owners Article 366 of the Civil Code in dealing with the right of accession to real property provides that "rhe accretions which banks of rivers may gradually receive from the effects of the currents belong to the owners of the estates bordering thereon." The provision is perfectly applicable to the strip of land, which, on account of the accretion, has come to be undeniable increase in the land of the hacienda inasmuch as it has increased all along the bank of the creek, the gradual effect of the currents; and even though the law does not require an express act of possession of the accretion which has enlarged the estate, it is certain that the owner of the hacienda has possessed it for more than 30 years through his tenants, who have been cultivating their respective parcels of land together with the corresponding portion of the said strip down to the bank of said creek. 6. Old monuments of no importance in this case The situation of the old monuments and the placing of new ones in the intervening space is of no importance, inasmuch as it has already been shown that Tuason has no title to the accretion which by spontaneous increase formed the strip of land between the creek and the monuments, and no proof is offered in the record that the land of Julia Tuason reached the other side of the creek toward the Hacienda of San Pedro Macati. 7. Municipality of San Pedro Macati merely enjoys usufruct of the plots occupied by the municipal building and town cemetery Roxas is the owner of the building lots and portions of land to which the opposition of the municipality of San Pedro Macati refers; the latter has only the usufruct of the plot occupied by the municipal building as long as the same or any other building of a public and official nature is erected thereon; the municipality can not dispose of it as a property of its own because, according to the documents offered in evidence by Roxas, the Spanish Government had recognized the dominion of Roxas' predecessor over the land occupied by said municipal building and by the town cemetery, and the grant made by the owner was ever understood to be only of the usufruct thereof so long as used for public purposes, the same being returnable to him upon ceasing to be used for such purpose. 8. Prescription unavailing to plot occupied by public school In connection with the land occupied by the public school of said town, no opposition based on ordinary or on extraordinary prescription may be made by the municipality because the plot was granted only for the purpose of erecting thereon a public school, and the possession thereof, on the part of the municipality, was simply usufructuary, the government of the Province of Manila having recognized the title thereto which pertained to Roxas, owner of the hacienda, whereof the said plot forms a part. The possession thereof by the municipality has been but for a few years only. When the school building having been destroyed, the land was abandoned. Prescription thus can not be invoked because the possession thereof was interrupted and ceased many years since. 9. Aguirres appeal abandoned The attorney for Alejandro and Consolacion Aguirre excepted to the decision of 17 October 1906, whereby their claim to the second parcel of land was dismissed. Their bill of exceptions, entered in the general register under 3788, was duly forwarded. Notwithstanding the fact that the time prescribed has been exceeded, the Aguirres have not filed their brief nor notified Roxas regarding the same. Roxas requested in a petition on 26 June 1907 that the Aguirres appeal be considered as having been abandoned. Said request is held to be well based and in accordance with the law.

Cureg v. IAC [G.R. No. 73465. September 7, 1989.] First Division, Medialdea (J): 3 concur Facts: On 5 November 1982, Domingo Apostol, Soledad Gerardo, Rosa Gerardo, Nieves Gerardo, Flordeliza Gerardo and Lilia Maquinad filed a complaint for quieting of title and damages with preliminary injunction against Leonida, Romeo, Pepito, Hernando, Manuel, Antonio and Elpidio Carniyan with the RTC Isabela (Civil Case Br. 111-373). A temporary restraining order was issued by the trial court on 12 November 1982. The complaint alleged that the Gerardos and Maquinad are the legal and/or the forced heirs of the late Domingo Gerardo, who died in February 1944, the latter being the only issue of the late Francisco Gerardo, who died before the outbreak of WWII; that since time immemorial and/or before 26 July 1894, the late Francisco Gerardo, together with his predecessors-in-interest have been in actual, open, peaceful and continuous possession, under a bona fide claim of ownership and adverse to all other claimants, of a parcel of land, situated in Casibarag-Cajel, Cabagan, Isabela, containing an area of 2.5 hectares [N: Cagayan River; E: Domingo Guingab (formerly Rosa Cureg); S: Antonio Carniyan; and W: Sabina Mola]. Said land was declared for taxation purposes under TD 08-3023 in the name of Francisco Gerardo, which cancelled TD C-9669, in the name of Francisco; that upon the death of Francisco Gerardo, the ownership and possession of the land was succeeded by his only issue, Domingo Gerardo who, together with 3 legal or forced heirs, namely Soledad Gerardo, Primo Gerardo(+) and Salud Gerardo(+) have also been in actual, open, peaceful and continuous possession of the same. Primo Gerardo was survived by Rosa, Nieves and Flordeliza Gerardo; while Salud Gerardo was survived by Lilia Maquinad. In 1979, Soledad, Rosa, Nieves, and Flordeliza Gerardo along with Lilia Maquinad verbally sold the land to Domingo Apostol. On 10 September 1982, the verbal sale and conveyance was reduced into writing by the vendors who executed an "Extra-Judicial Partition with Voluntary Reconveyance. About the time of the execution of the Extra-Judicial Partition, the land already manifested signs of accretion of about 3 hectares on the north caused by the northward movement of the Cagayan River; that Domingo Apostol declared the land and its accretion for tax purposes under TD 08-13281 on 15 September 1982. Sometime about the last week of September and or the first week of October 1982, when the Gerardos, Maquinad and Apostol were about to cultivate their land together with its accretion, they were prevented and threatened by the Carniyans (Leonida Cureg and Romeo, Pepito, Hernando, Manuel, Antonio and Elpidio: surviving spouse and children of Antonio Carniyan) from continuing to do so. The late Antonio Carniyan was the owner of a piece of land (acquired from his father-in-law Marcos Cureg on 5 October 1956 as evidenced by an Absolute Deed of Sale) situated in Casibarag-Cajel, Cabagan, Isabela which contained an area of 2,790 sq. m.(N: Domingo Gerardo; E: Domingo Guingab; S: Pelagio Camayo; and W: Marcos Cureg), and which was declared for taxation purposes under TD 13131, with an assessed value of P70.00. Carniyan revised on 28 November 1968 his TD 13131 dated 24 July 1961 to conform with the correct area and boundaries of his OCT P-19093 issued on 25 November 1968 pursuant to Free Patent 399431 dated 21 May 1968; that the area under the new TD 15663 was increased from 2,790 sq.ms. to 4,584 sq.ms. and the boundary on the north became Cagayan River, purposely eliminating completely the original boundary on the north which is Domingo Gerardo. The heirs of Antonio Carniyan (Cureg, et.al.) alleged in their answer that the land claimed by the Gerardos and Apostol is non-existent; that Antonio Carniyan was the owner of a piece of land bounded on the north by Cagayan River and not by the land of Francisco Gerardo; that the "subject land" is an accretion to their registered land and that they have been in possession and cultivation of the "accretion" for many years. The application for the issuance of a writ of preliminary injunction was denied on 28 July 1983 on the ground that the Carniyans (Cureg) were in actual possession of the land in litigation prior to September 1982. In a decision rendered on 6 July 1984, the trial court rendered judgment declaring Domingo Apostol the absolute owner of the parcel of land containing an area of 5.5000 hectares (N: Cagayan River; E: Domingo Guingab; S: Antonio Carniyan; and W: by Sabina Mola) and with an assessed value of P3,520; ordering the issuance of a writ of preliminary injunction against Cureg, et.al.; ordering that the writ be made permanent; and ordering Cureg, et.al. to pay Apostol, et.al. a reasonable attorney's fee of P5,000.00, litigation expenses of P1,500.00 and costs. On 17 July 1984, Cureg appealed to the then IAC Court which affirmed the decision of the trial court on 15 October 1985 (CAGR CV 03852). Cureg's Motion for Reconsideration was denied on 8 January 1986. Hence, the petition for review under Rule 45 of the Rule of Court. The Supreme Court granted the petition, reversed and set aside the decision appealed from, and rendered judgment dismissing Civil Case Br. III-373 for quieting of title and damages; with costs against Apostol, et.al. 1. Tax Declaration not sufficient evidence to prove ownership; OCT indicates true and legal ownership Gerardos' and Maquinads (therefore Apostols) claim of ownership of their alleged 2 & 1/2 hectare land is anchored mainly on 4 tax declarations. The declaration of ownership for purposes of assessment on the payment of the tax is not sufficient evidence to prove ownership. (Evangelista v. Tabayuyong, 7 Phil. 607; Elumbaring v. Elumbaring, 12 Phil. 384; cited in Camo v. Riosa Bayco, 29 Phil. 437, 444). On the other hand, Cureg et.al. relied on the indefeasibility and incontrovertibility of their OCT P19093. In the case of Ferrer-Lopez v. Court of Appeals (GR 50420, 29 May 1987, 150 SCRA 393, 401-402), it was ruled that as against an array of proofs consisting of tax declarations and/or tax receipts which are not conclusive evidence of ownership nor proof of the area covered therein, an original certificate of title indicates true and legal ownership by the registered owners over the disputed premises. Cureg's OCT P-19093 should be accorded greater weight as against the tax declarations offered by Apostol, et.al. in support of their claim, which declarations are all in the name of the latters predecessor-in-interest, Francisco Gerardo, and appear to have been subscribed by him after the last war, when it was established during the trial that Francisco Gerardo died long before the outbreak of the last war. 2. Decree of registration bars all claims and rights arising or existing prior to decree A decree of registration bars all claims and rights which arose or may have existed prior to the decree of registration (FerrerLopez v. CA, supra., p. 404). By the issuance of the decree, the land is bound and title thereto quieted, subject only to exceptions stated in Section 39, Act 496 (now Section 44 of PD 1529). Since Cureg's original certificate of title clearly stated that subject land is bounded on the north by the Cagayan River, Apostol's claim over the land allegedly existing between Cureg's land and the Cagayan River, is deemed barred and nullified with the issuance of the original certificate of title.

3. Tax Declaration of earlier date cannot defeat OCT of later date; Cureg not estopped as Tax Declarations subsequent to issuance of OCT states northern boundary is Cagayan river A tax declaration, being of an earlier date cannot defeat an original certificate of title which is of a later date. The appellate court erred in considering Tax Declaration 13131, in the name of Antonio Carniyan, as an admission by him that his land is bounded on the north by the land of Domingo Gerardo and thus is estopped from claiming otherwise. The tax declarations of the late Antonio Carniyan subsequent to the issuance of OCT P-19093 already states that its northern boundary is Cagayan River. In effect, he has repudiated any previous acknowledgment by him, granting that he caused the accomplishment of the tax declarations in his name before the issuance of OCT P-19093, of the existence of Francisco Gerardo's land. 4. Cureg, et.al. in actual possession; Evidence Evidence on record proves that Cureg, et.al. are in actual possession of the land. First, the trial court in its Decision stated the reason for denying private respondents' petition for the issuance of a preliminary injunction, is that Cureg, et.al. were in actual possession of the land in litigation prior to September 1982. Second, witness for Apostol, et.al., Esteban Guingab, boundary owner on the east of the land in question and whose own land is bounded on the north of Cagayan River, on cross-examination, revealed that when his property was only more than 1 hectare in 1958, (now more than 4 hectares) his boundary on the west is the land of Antonio Carniyan. Third, witness Rogelio C. Albano, a geodetic engineer, on direct examination stated that in 1974, the late Antonio Carniyan requested him to survey the land covered by his title and the accretion attached to it, but he did not pursue the same because he learned from the Office of the Director of the Bureau of Lands that the same accretion is the subject of an application for homestead patent of one Democrata Aguila, contrary to the statement of the trial court and the appellate court that Albano "made three attempts to survey the land but he did not continue to survey because persons other than defendants were in possession of the land," which statement appears only to be a conclusion. Fourth, an order by the Director of Lands dated 14 August 1980 in connection with the Homestead Application of Democrata Aguila of an accretion situated in Catabayungan, Cabagan, Isabela, such application was disapproved because in an investigation conducted by the Bureau of Lands of the area applied for which is an accretion, the same was found to be occupied and cultivated by, among others, Antonio Carniyan, who claimed it as an accretion to his land. Apostol, et.al. nor their predecessors-in-interest appeared as one of those found occupying and cultivating said accretion. 5. Accretion belongs to riparian owners The land in question is an alluvial deposit left by the northward movement of the Cagayan River and pursuant to Article 457 of the New Civil Code, it is said that "to the owners of land adjoining the banks of river belong the accretion which they gradually receive from the effects of the current of the waters." 6. Accretion does not automatically become registered land The area covered by OCT P-19093 is only 4,584 sq. ms. The accretion attached to said land is approximately 5.5 hectares. The increase in the area of Cureg's land, being an accretion left by the change of course or the northward movement of the Cagayan River does not automatically become registered land just because the lot which receives such accretion is covered by a Torrens title. (See Grande v. Court of Appeals, L-17652, June 30, 1962). As such, it must also be placed under the operation of the Torrens System." Grande v. CA [G.R. No. L-17652. June 30, 1962.] En Banc, Barrera (J): 7 concur, 3 took no part Facts: Eulogia, Alfonso, Eulalia, and Sofia Grande are the owners of a parcel of land, with an area of 3.5032 hectares, located at barrio Ragan, municipality of Magsaysay (formerly Tumauini), province of Isabela, by inheritance from their deceased mother Patricia Angui (who inherited it from her parents Isidro Angui and Ana Lopez, in whose name said land appears registered, as shown by OCT 2982, issued on 9 June 1934). Said property is identified as Lot 1, Plan PSU-83342. When it was surveyed for purposes of registration sometime in 1930, its northeastern boundary was the Cagayan River (the same boundary stated in the title). Since then, and for many years thereafter, a gradual accretion on the northeastern side took place, by action of the current of the Cagayan River, so much so, that by 1958, the bank thereof had receded to a distance of about 105 meters from its original site, and an alluvial deposit of 19,964 square meters (1.9964 hectares), more or less, had been added to the registered area. On 25 January 1958, the Grandes instituted an action in the CFI Isabela against Domingo and Esteban Calalung to quiet title to said portion (19,964 square meters) formed by accretion, alleging in their complaint (Civil Case 1171) that they and their predecessors-in-interest, were formerly in peaceful and continuous possession thereof, until September, 1948, when the Calalungs entered upon the land under claim of ownership. The Grandes also asked for damages corresponding to the value of the fruits of the land as well as attorney's fees and costs. In their answer, dated 18 February 1958, the Calalungs claim ownership in themselves, asserting that they have been in continuous, open, and undisturbed possession of said portion, since prior to the year 1933 to the present. After trial, the CFI Isabela, on 4 May 1959, rendered a decision adjudging the ownership of the portion in question to the Grandes, and ordering the Calalungs to vacate the premises and deliver possession thereof to the Grandes, and to pay to the latter P250.00 as damages and costs. Unsatisfied, the Calalungs appealed to the Court of Appeals, which rendered, on 14 September 1960, a decision (CA-GR 25169R) reversing that of the CFI Isabela, and dismissing the Grandes' action against the Calalungs, to quiet title to and recover possession of a parcel of land allegedly occupied by the latter without the Grandes' consent. Thus, the appeal by the Grandes to the Supreme Court. The Supreme Court affirmed the decision of the Court of Appeals, with costs against the Grandes. 1. Alluvium belongs to riparian owner By law, unless some superior title has supervened, alluvium should properly belong to the riparian owners, specifically in accordance with the rule of natural accession in Article 366 of the old Civil Code (now Article 457), which provides that to the owner of lands adjoining the banks of rivers, belongs the accretion which they gradually receive from the effects of the current of

the water. The area in controversy has been formed through a gradual process of alluvion, which started in the early thirties, is a fact conclusively established by the evidence for both parties. There can be no dispute that both under Article 457 of the new Civil Code and Article 366 of the old, the Grandes are the lawful owners of said alluvial property, as they are the registered owners of the land to which it adjoins. 2. Accretion to registered land does not ipso jure becomes entitled to the protection of the rule of imprescriptability of title under Land Registration Act An accretion to registered land, while declared by specific provision of the Civil Code to belong to the owner of the land as a natural accession thereof, does not ipso jure become entitled to the protection of the rule of imprescriptibility of title established by the Land Registration Act. Such protection does not extend beyond the area given and described in the certificate. To hold otherwise, would be productive of confusion. It would virtually deprive the title, and the technical description of the land given therein, of their character of conclusiveness as to the identity and area of the land that is registered. Just as the Supreme Court, albeit in a negative manner, has stated that registration does not protect the riparian owner against the erosion of the area of his land through gradual changes in the course of the adjoining stream (Payatas Estate Development Co. vs. Tuason, 53 Phil. 55), so registration does not entitle him to all the rights conferred by the Land Registration Act, in so far as the area added by accretion is concerned. What rights he has, are declared not by said Act, but by the provisions of the Civil Code on accession; and these provisions do not preclude acquisition of the additional area by another person through prescription. (See Galindez, et al. vs. Baguisa, et al., CA-GR 19249-R, 17 July 1959). 3. Ownership of a piece of land and registration under Torrens system are different; Imprescriptibility of registered land under land registration law; Accretion not registered subject to acquisition through prescription by third persons Accretion does not automatically become registered land just because the lot which receives it is covered by a Torrens title thereby making the alluvial property imprescriptible; just as an unregistered land purchased by the registered owner of the adjoining land does not, by extension, become ipso facto registered land. Ownership of a piece of land is one thing, and registration under the Torrens system of that ownership is quite another. Ownership over the accretion received by the land adjoining a river is governed by the Civil Code. Imprescriptibility of registered land is provided in the registration law. Registration under the Land Registration and Cadastral Acts does not vest or give title to the land, but merely confirms and thereafter protects the title already possessed by the owner, making it imprescriptible by occupation of third parties. But to obtain this protection, the land must be placed under the operation of the registration laws wherein certain judicial procedures have been provided. The fact remains that the Grandes never sought registration of said alluvial property (which was formed sometime after the Grandes' property covered by OCT 2982 was registered on 9 June 1934) up to the time they instituted the action in the CFI Isabela in 1958. The increment, therefore, never became registered property, and hence is not entitled or subject to the protection of imprescriptibility enjoyed by registered property under the Torrens system. Consequently, it was subject to acquisition through prescription by third persons. 4. Calalungs in possession of land since 1934, not 1948 Domingo Calalung testified that he occupied the land in question for the first time in 1934, not in 1948 as claimed by the Grandes. The area under occupancy gradually increased as the years went by. In 1946, he declared the land for purposes of taxation, the tax declaration of which was superseded in 1948 by another, after the name of the municipality wherein it is located was changed from Tumauini to Magsaysay. Calalung's testimony is corroborated by two witnesses, both owners of properties nearby. Pedro Laman, 72 years of age, who was Municipal president of Tumauini for three terms (recollection of the Calalungs peaceful possession since 1940 or 1941), and Vicente C. Bacani (Calalungs' possession started sometime in 1933 or 1934. The area thereof was then less than one hectare). The testimony of the said witnesses entitled to much greater weight and credence than that of Pedro Grande and his lone witness, Laureana Rodriguez. 5. Grandes relinquished possession of lot occupied by the Calalungs The Grandes did not file an action until 1958, because it was only then that they were able to obtain the certificate of title from the surveyor, Domingo Parlan; and that they never declared the land in question for taxation purposes or paid the taxes thereon. The excuse they gave for not immediately taken steps to recover possession of the lot was that they did not receive their copy of the certificate of title to their property until 1958 for lack of funds to pay the fees of the surveyor Domingo Parlan. When the Calalungs had their land surveyed in April 1958, Pedro Grande tried to stop it, not because he claimed the accretion for himself and for the other Grandes, but because the survey included a portion of the property covered by their title. Thus, the Grandes relinquished their possession to the part thus included, containing an area of some 458 sq. ms. 6. Finding of Court of Appeals conclusive; Prescription supervened in favor of Calalungs The Court of Appeals, upon consideration of the evidence, was convinced that the Calalungs were really in possession openly, continuously and adversely, under a claim of ownership since 1933 or 1934, immediately after the process of alluvion started up to the filing of the action in 1958; and that the Grandes woke up to their rights only when they received their copy of the title in 1958. By then, however, prescription had already supervened in favor of the Calalungs. This finding of the existence of these facts, arrived at by the Court of Appeals after an examination of the evidence presented by the parties, is conclusive as to them and can not be reviewed by the Supreme Court. 7. Applicable law on prescription: Act 190 not Civil Code, since possession started in 1933 or 1934 The law on prescription applicable to the case is that provided in Act 190 and not the provisions of the Civil Code, since the possession started in 1933 or 1934 when the pertinent articles of the Old Civil Code were not in force and before the effectivity of the New Civil Code in 1950. The conclusion of the Court of Appeals that the Calalungs acquired the alluvial lot in question by acquisitive prescription is in accordance with law. Viajar v. CA [G.R. No. 77294. December 12, 1988.] First Division, Medialdea (J): 3 concur, 1 on leave

Facts: The spouses Ricardo and Leonor Ladrido were the owners of Lot 7511 of the Cadastral Survey of Pototan situated in barangay Cawayan, Pototan, Iloilo (154,267 sq. ms., TCT T-21940 of the Register of Deeds of Iloilo). Spouses Rosendo and Ana Te were also the registered owners of a parcel of land described in their title as Lot 7340 of the Cadastral Survey of Pototan. On 6 September 1973, Rosendo Te, with the conformity of his wife, sold this lot to Angelica F. Viajar and Celso F. Viajar for P5,000. A Torrens title was later issued in the latters names. Later, Angelica Viajar had Lot 7340 relocated and found out that the property was in the possession of Ricardo Y. Ladrido. Consequently, she demanded its return but Ladrido refused. The piece of real property which used to be Lot 7340 of the Cadastral Survey of Pototan was located in barangay Guibuanogan, Pototan, Iloilo; that it consisted of 20,089 sq.ms.; that at the time of the cadastral survey in 1926, Lot 7511 and Lot 7340 were separated by the Suague River; that the area of 11,819 sq.ms of what was Lot 7340 has been in the possession of Ladrido; that the area of 14,036 sq.ms., which was formerly the river bed of the Suague River per cadastral survey of 1926, has also been in the possession of Ladrido; and that the Viajars have never been in actual physical possession of Lot 7340. On 15 February 1974, Angelica and Celso Viajar instituted a civil action for recovery of possession and damages against Ricardo Y. Ladrido (Civil Case 9660) with the CFI Iloilo. Summoned to plead, Ladrido filed his answer with a counterclaim. The Viajars filed their reply to the answer. Subsequently, the complaint was amended to implead Rosendo Te as another defendant. The Viajars sought the annulment of the deed of sale and the restitution of the purchase price with interest in the event the possession of defendant Ladrido is sustained. Te filed his answer to the amended complaint and he counterclaimed for damages. The Viajars answered the counterclaim. During the pendency of the case, Celso Viajar sold his rights over Lot 7340 to his mother and coplaintiff, Angelica F. Viajar. For this reason, Angelica F. Viajar appears to be the sole registered owner of the lot. On 25 May 1978, Ricardo Ladrido died. He was substituted in the civil action by his wife, Leonor P. Ladrido, and children, namely: Lourdes Ladrido-Ignacio, Eugenio P. Ladrido and Manuel P. Ladrido, as parties defendants. After trial on the merits, a second amended complaint which included damages was admitted. On 10 December 1981, the trial court rendered its decision in favor of Ladrido, dismissing the complaint of Angelica and Celso Viajar with costs against them, declaring Leonor P. Ladrido, Lourdes LadridoIgnacio, Eugenio P. Ladrido and Manuel P. Ladrido as owner of the parcel of land indicated as Lots A and B in the sketch plan situated in barangays Cawayan and Guibuanogan, Pototan, Iloilo, and containing an area of 25,855 sq.ms., and pronouncing that as owners of the land, the Ladridos are entitled to the possession thereof. Not satisfied with the decision, the Viajars appealed to the Court of Appeals. The Court of Appeals affirmed the decision of the court a quo on 29 December 1986 (CA-GR CV 69942). The Viajars filed a petition for review on certiorari. The Supreme Court dismissed the petition for lack of merit without pronouncement as to costs. 1. Accretion belongs to riparian owners Article 457 of the New Civil Code provides that to the owners of lands adjoining the banks of rivers belong the accretion which they gradually receive from the effects of the current of the waters." 2. Change in the course of the river gradual The presumption is that the change in the course of the river was gradual and caused by accretion and erosion (Martinez Caas vs. Tuason, 5 Phil. 668; Payatas Estate Improvement Co. vs. Tuason, 53 Phil. 55; C.H. Hodges vs. Garcia, 109 Phil. 133). In the present case, the lower court correctly found that the evidence introduced by the Viajars to show that the change in the course of the Suague River was sudden or that it occurred through avulsion is not clear and convincing. The Ladridos have sufficiently established that for many years after 1926 a gradual accretion on the eastern side of Lot 7511 took place by action of the current of the Suague River so that in 1979 an alluvial deposit of 29,912 sq.ms. (2.9912 hectares), more or less, had been added to Lot 7511 (Lot A: 14,036 sq.ms., Lot B, 11,819 sq.ms. and Lot C, which is not in litigation, 4,057 sq.ms). The established facts indicate that the eastern boundary of Lot 7511 was the Suague River based on the cadastral plan. For a period of more than 40 years (before 1940 to 1980) the Suague River overflowed its banks yearly and the property of the defendant gradually received deposits of soil from the effects of the current of the river. The consequent increase in the area of Lot 7511 due to alluvion or accretion was possessed by the defendants whose tenants plowed and planted the same with corn and tobacco. The quondam river bed had been filled by accretion through the years. The land is already plain and there is no indication on the ground of any abandoned river bed. Under the law, accretion which the banks or rivers may gradually receive from the effects of the current of the waters becomes the property of the owners of the lands adjoining the banks. (Art. 366, Old Civil Code; Art. 457, New Civil Code which took effect on 30 August 1950 [Lara v. Del Rosario, 94 Phil. 778]. Therefore, the accretion to Lot 7511 which consists of Lots A and B belong to the Ladridos. 3. Finding of facts by lower court conclusive upon Supreme Court The trial court found that the change in the course of the Suague River was gradual and this finding was affirmed by the Court of Appeals. The Supreme Court found no valid reason to disturb the finding of fact. 4. Section 45 and 46 of Act 496, Land Registration Act Section 45 of Act 496 provides that the obtaining of a decree of registration and the entry of a certificate of title shall be regarded as an agreement running with the land, and binding upon the applicant and all successors in title that the land shall be and always remain registered land, and subject to the provisions of this Act and all Acts amendatory thereof." Section 46 of the same act provides that No title to registered land in derogation to that of the registered owner shall be acquired by prescription or adverse possession." 5. Registration does not protect riparian owner against diminution of the area of registered land; Interpretation of Article 366 of the Civil Code with the Land Registration Act / Torrens System The rule that registration under the Torrens System does not protect the riparian owner against the diminution of the area of his registered land through gradual changes in the course of an adjoining stream is well settled. In Payatas Estate Improvement Co. vs. Tuason (53 Phil. 55), it was ruled that Article 366 of the Civil Code provides that any accretions which the banks of rivers may gradually receive from the effects of the current belong to the owners of the estates bordering thereon. Accretions of that character are natural incidents to land bordering on running streams and are not affected by the registration laws. It follows that registration does not protect the riparian owner against diminution of the area of his land through gradual changes in the course of

the adjoining stream. Similarly in C.N. Hodges vs. Garcia (109 Phil. 133), it was ruled that if the land in question has become part of ones estate as a result of accretion, it follows that said land now belongs to him. The fact that the accretion to his land used to pertain to another's estate, which is covered by a Torrens Certificate of Title, cannot preclude the former from being the owner thereof. Registration does not protect the riparian owner against the diminution of the area of his land through gradual changes in the course of the adjoining stream. Accretions which the banks of rivers may gradually receive from the effect of the current become the property of the owners of the banks (Art. 366 of the Old Civil Code; Art. 457 of the New). Such accretions are natural incidents to land bordering on running streams and the provisions of the Civil Code in that respect are not affected by the Registration Act. 6. No damages recoverable from Ladridos As the Ladridos are the owners of the premises in question, no damages are recoverable from them. Hodges v. Garcia [G.R. No. L-12730. August 22, 1960.] En Banc, Gutierrez-David (J): 8 concur Facts: A lot formerly a part of Lot 908 of the Cadastral Survey of Jaro, Iloilo was acquired by C. N. Hodges from Salustiano Mirasol in January 1950, and subsequently registered in his name as evidenced by TCT T-2504 issued by the Register of Deeds of Iloilo. This property was bounded on the north by the Salog River. Adjoining that river on the other side is Lot 2290, which was purchased by Amador D. Garcia from Dr. Manuel Hechanova on 15 April 1950. On 12 July 1950, Garcia had the land he bought resurveyed. The survey plan disclosed that the land, which was originally surveyed in 1912 and was then bounded on the SE and SW by the Salog river, had increased in area by the river bank, and that the added area, which bounds the land on the SE and SW, is in turn bounded on the SE and SW by the Salog river. In due time, Garcia applied for the registration of the additional area under the Land Registration Act, and on 22 March 1952, the cadastral court rendered a decision holding that the land sought to be registered is an accretion to Lot 2290 and decreeing that the land be registered in his name. On 30 June 1952, the corresponding OCT O-229 was issued in favor of Garcia. CN Hodges filed an action with the CFI Iloilo for the possession of a portion of land designated as Lot 908-Q with an area of 5,931 sq. m. , which is alleged to have been separated from plaintiff's land by the "natural change" in the course of a river. The court ruled in favor of Garcia. Hodges appealed to the Court of Appeals. The appellate court certified the case to the Supreme Court that it was decided upon a stipulation of facts and for that reason questions of fact can no longer be raised on appeal. The Supreme Court affirmed the decision appealed from, with costs against CN Hodges. 1. Presumption in the absence of evidence that the change in the course of the river was sudden (avulsion) In the absence of evidence to show that the change in the course of the river was sudden or that it occurred through avulsion, the presumption is that the change was gradual and caused by accretion and erosion. (Payatas Estate Improvement Co. vs. Tuason, 53 Phil., 55.) 2. Fact on alluvion or accretion not contradicted In any event, it was agreed upon by the parties in open court that from the year 1917 until the construction of the dike (in 1939) along the river the course of the Salog river, starting from the edge of lot 2290, gradually ate up the bank towards the side of the poblacion of Jaro and at the same time gradually deposited sediments towards the side of Lot 2290; that when Garcia bought lot 2290 from Dr. Manuel Hechanova in 1950, he found out that there was an accretion along one side of said lot, to which Garcia applied for its registration, to which culminated in the issuance of an OCT in his favor; and that because of the gradual deposit of sediments of the Salog River along his land, lot 2290, Garcia has been in possession of said land since 1950 until now, while Hodges and his predecessors in interest since the gradual loss of lot 908-Q, covered by water, has never been in actual possession of the said lot. The foregoing facts have never been denied or contradicted by plaintiff, and they clearly show that the increase in area of Lot 2290 by the river bank was due to alluvion or accretions which it gradually received (from 1917 to 1939, or for a period of 22 years) from the effects of the current of the river. Neither did Hodges file any opposition despite due publication of the notice of the application and hearing. Further, the land now being claimed by Hodges had been litigated in three civil cases, while Garcia was recognized as the owner of the land and held legally entitled to its possession. 3. Registration does not protect the riparian owner against diminution of the area of his land The land in question has become part of Garcia's estate as a result of accretion, it follows that said land now belongs to him. The fact that the accretion to his land used to pertain to Hodges' estate, which is covered by a Torrens certificate of title, cannot preclude Garcia from being the owner thereof. Registration does not protect the riparian owner against the diminution of the area of his land through gradual changes in the course of the adjoining stream. Accretions which the banks of rivers may gradually receive from the effect of the current become the property of the owners of the banks. (Art. 366 of the old Civil Code; Art. 457 of the new.) Such accretions are natural incidents to land bordering on running streams and the provisions of the Civil Code in that respect are not affected by the Land Registration Act. (Payatas Estate Improvement Co. vs. Tuason) Baes v. CA [G.R. No. 108065. July 6, 1993.] First Division, Cruz (J): 3 concur Facts: In 1962, the government dug a canal on a private parcel of land (Lot 2958, with area of 33,902 sq.m.) to streamline the Tripa de Gallina creek. This lot was later acquired by Felix Baes, who registered it in his name under TCT 10990 and then had it subdivided into 3 lots (Lot 2958-A, 28,889 sq.m, TCT 11041.; Lot 2958-B, 3,588 sq.m, TCT 11042.; and Lot 2958-C, 452 sq.m., TCT 11043). In exchange for Lot 2958-B, which was totally occupied by the canal, the government gave Baes a lot with exactly the same area as Lot 2958-B through a Deed of Exchange of Real Property dated 20 June 1970. The property, which was near but not contiguous to Lot 2958-C, was denominated as Lot 3271-A and later registered in the name of Felix Baes under TCT 24300.

The soil displaced by the canal was used to fill up the old bed of the creek. Meanwhile, Baes had Lot 2958-C and a portion of Lot 2958-A designated as Lot 1, Blk. 4, resurveyed and subdivided. On 12 January 1968, he submitted a petition for the approval of his resurvey and subdivision plans, claiming that after the said lots were plotted by a competent surveyor, it was found that there were errors in respect of their bearings and distances. The resurvey-subdivision plan was approved by the CFI Pasay City in an order dated 15 January 1968. As a result, the old TCTs covering the said lots were canceled and new ones were issued (Blk. 4: Lot 1-A, 672 sq.m., TCT T-14404; Lot 1-B representing the increase in size after the resurvey, 826 sq.m., TCT T-14405; Lot 2958-C-1, 452 sq.m., TCT T-14406; and Lot 2958-C-2 representing the increase after resurvey, 2,770 sq.m., TCT T-14407). Lots 2958-C-1 and 2958-C-2 were later consolidated and this time further subdivided into 4 lots (Lot 1, 147 sq.m., TCT 29592.; Lot 2, 950 sq.m., TCT 29593; Lot 3, 257 sq.m., TCT 29594; and Lot 4, 1,868 sq.m., TCT 29595). In 1978, the Republic of the Philippines discovered that Lot 1-B (TCT 14405) on which the Baes had erected an apartment building, covered Lot 3611 of the Pasay Cadastre, which is a filled-up portion of the Tripa de Gallina creek. Moreover, Lot 2958-C (TCT 29592 to 29595), with an increased area of 2,770 sq.m. after resurvey and subdivision, had been unlawfully enlarged. On 17 November 1982, the Republic filed a petition for cancellation of TCT 14405 and 29592 to 29595 with the trial court (Civil Case 0460-P). Baes did not object in his answer to the cancellation of TCT 29592, 29594 and 29595 and was not able to prove during the trial that the government utilized a portion of Lot 2 under TCT 29593. The trial court therefore decreed that the original Lot 2958-C (452 sq.m.) be reverted to its status before the resurvey-subdivision of Lot 2958-C. Baes appealed to the Court of Appeals, which affirmed in toto the ruling of the trial court, declaring TCTs 14405, 29592, 29593, 29594, 29595, and TCT 29593's derivative titles TCTs 124725, 124726, 124727 and 124729, ordering the Register of Deeds for Pasay City to cancel them and issue new ones in their stead in the name of the Baes after segregating from TCT 29593 452 sq. m., the actual area of Lot 2958-C (covered by cancelled TCT 11043) belonging to Felix Baes, and dismissing the counterclaim. Baes appealed to the Supreme Court by way of certiorari. The Supreme Court denied the petition, with costs against Baes. 1. Article 461 of the Civil Code applies to a natural change in the course of the stream; If change of course is due to works, belongs to concessioners if not to owners of land covered by the waters Article 461 of the Civil Code provides that River beds which are abandoned through the natural change in the course of the waters ipso facto belong to the owners whose lands are occupied by the new course in proportion to the area lost. However, the owners of the lands adjoining the old bed shall have the right to acquire the same by paying the value thereof, which value shall not exceed the value of the area occupied by the new bed. Article 461 refers to a natural change in the course of a stream. If the change of the course is due to works constructed by concessioners authorized by the government, the concession may grant the abandoned river bed to the concessioners. If there is no such grant, then, by analogy, the abandoned river bed will belong to the owners of the land covered by the waters, as provided in this article, without prejudice to a superior right of third persons with sufficient title. 2. Riparian owner entitled to compensation if change in the course of river by artificial means If the riparian owner is entitled to compensation for the damage to or loss of his property due to natural causes, there is all the more reason to compensate him when the change in the course of the river is effected through artificial means. Baes loss of the land covered by the canal was the result of a deliberate act on the part of the government when it sought to improve the flow of the Tripa de Gallina creek. It was therefore obligated to compensate the Baeses for their loss. 3. Ownership of dried up portion of creek in addition to fair exchange of similar lots by parties a double compensation Baes has already been compensated as there has been a fair exchange of Lot 3271-A belonging to the Government and Lot 2958B belonging to Baes, which were similar in area and value, through a Deed of Exchange of Real Property which the parties freely entered into. Baes cannot claim additional compensation because allowing Baes to acquire ownership of the dried-up portion of the creek would be a clear case of double compensation and unjust enrichment at the expense of the state. The exchange of lots between the petitioners and the Republic was the result of voluntary negotiations. If these had failed, the government could still have taken Lot 2958-B under the power of eminent domain, upon payment of just compensation, as the land was needed for a public purpose. Ronquillo v. CA [G.R. No. 43346. March 20, 1991.] Second Division, Regalado (J): 4 concur Facts: Rosendo del Rosario was a registered owner of a parcel of land known as Lot 34, Block 9, Sulucan Subdivision, situated at Sampaloc, Manila and covered by TCT 34797 of the Registry of Deeds of Manila. Florencia and Amparo del Rosario were daughters of said Rosendo del Rosario. Adjoining said lot is a dried-up portion of the old Estero Calubcub occupied by Mario C. Ronquillo since 1945. Both del Rosario and Ronquillo have filed with the Bureau of Lands miscellaneous sales application for the purchase of the abandoned river bed known as Estero Calubcub and their sales applications, dated 5 August 1958 and 13 October 1959, respectively, are still pending action before the Bureau of Lands. Del Rosario claims that long before 1930, when TCT 34797 over Lot 34 was issued in the name of Rosendo del Rosario, the latter had been in possession of said lot including the adjoining dried-up portion of the old Estero Calubcub, having bought the same from Arsenio Arzaga. Sometime in 1935, said titled lot was occupied by Isabel Roldan with the tolerance and consent of del Rosario on condition that the former will make improvements on the adjoining dried-up portion of the Estero Calubcub. In the early part of 1945 defendant occupied the eastern portion of said titled lot as well as the dried-up portion of the old Estero Calubcub which abuts del Rosario's titled lot. After a relocation survey of the land in question sometime in 1960, del Rosario learned that Ronquillo was occupying a portion of their land and thus demanded Ronquillo to vacate said land when the latter refused to pay the reasonable rent for its occupancy. However, despite said demand Ronquillo refused to vacate. On the other hand, Ronquillo claims that sometime before 1945 he was living with his sister who was then residing or renting Del Rosario's titled lot. In 1945 he built his house on the disputed dried-up portion of the Estero Calubcub with a small portion thereof on the titled lot of del Rosario. Later in 1961, said house was destroyed by a fire which prompted him to rebuild the same but, this time it was built only on the dried-up portion of the old

Estero Calubcub without touching any part of del Rosarios titled land. He further claims that said dried-up portion is a land of public domain. Rosendo, Amparo and Florencia del Rosario lodged a complaint with the CFI Manila praying, among others, that they be declared the rightful owners of the dried-up portion of Estero Calubcub. Ronquillo filed a motion to dismiss the complaint on the ground that the trial court had no jurisdiction over the case since the dried-up portion of Estero Calubcub is public land and, thus, subject to the disposition of the Director of Lands. The Del Rosarios opposed the motion arguing that since they are claiming title to the dried-up portion of Estero Calubcub as riparian owners, the trial court has jurisdiction. The resolution of the motion to dismiss was deferred until after trial on the merits. On 26 December 1962, the trial court rendered judgment ordering Ronquillo to deliver to del Rosario the portion of the land covered by TCT 34797 which is occupied by him and to pay for the use and occupation of said portion of land at the rate of P5 a month from the date of the filing of the complaint until such time as he surrenders the same to del Rosario and declaring Del Rosario to be the owners of the dried-up portion of estero Calubcub which is abutting del Rosario' property; with costs against Ronquillo. On appeal (CA-GR 32479-R), the Court of Appeals affirmed the decision of the trial court on 25 September 1975 and declared that since Estero Calubcub had already dried-up way back in 1930 due to the natural change in the course of the waters, under Article 370 of the old Civil Code which it considers applicable to the present case, the abandoned river bed belongs to the Del Rosarios as riparian owners. Consequently, respondent court opines, the dried-up river bed is private land and does not form part of the land of the public domain. It stated further that even assuming for the sake of argument that said estero did not change its course but merely dried up or disappeared, said dried-up estero would still belong to the riparian owner, citing its ruling in the case of Pinzon vs. Rama. Upon motion of Ronquillo, respondent court modified its decision on 28 January 1976 by setting aside the first portion of the trial court's decision ordering Ronquillo to surrender to the Del Rosarios that portion of land covered by TCT 34797 occupied by the former, based on the former's representation that he had already vacated the same prior to the commencement of this case. However, the appellate court upheld its declaration that the Del Rosarios are the rightful owners of the dried-up river bed. Hence, the petition for review. On 17 May 1976, the Supreme Court issued a resolution requiring the Solicitor General to comment on the petition in behalf of the Director of Lands as an indispensable party in representation of the Republic of the Philippines, and who, not having been impleaded, was subsequently considered impleaded as such in the Courts resolution of 10 September 1976. In his Motion to Admit Comment, the Solicitor General manifested that pursuant to a request made by this office with the Bureau of Lands to conduct an investigation, the Chief of the Legal Division of the Bureau sent a communication informing him that the records of his office do not show that Mario Ronquillo, Rosendo del Rosario, Amparo del Rosario or Florencia del Rosario has filed any public land application covering parcels of land situated at Estero Calubcub, Manila as verified by its Records Division. The position taken by the Director of Lands (in his comment on 3 September 1978, in the 4 May 1989 reply, and 17 August 1989 comment) explicates that Article 370 of the old Code does not apply as the abandoned riverbed as such was abandoned not by the natural change in the course of the river but by the drying up of the bed caused by human activity. The Director of Lands also added that the del Rosario and Ronquillo have claimed pending sales applications over the dried portion of the estero (admitting thus that it is public land under the authority of the Bureau of Lands), which were rejected as the Manila City Engineers Office needed the dried portion of the estero for drainage purposes. On 29 June 1979, Florencia del Rosario manifested to this Court that Rosendo, Amparo and Casiano del Rosario have all died, and that she is the only one still alive among the private respondents in the case. In a resolution dated 20 January 1988, the Court required Ronquillo to implead one Benjamin Diaz pursuant to the former's manifestation that the land adjacent to the dried-up river bed has already been sold to the latter, and the Solicitor General was also required to inquire into the status of the investigation being conducted by the Bureau of Lands. In compliance therewith, the Solicitor General presented a letter from the Director of Lands to the effect that neither of the parties involved in the present case has filed any public land application. On 3 April 1989, Ronquillo filed an Amended Petition for Certiorari, this time impleading the Development Bank of the Philippines (DBP) which subsequently bought the property adjacent to the dried-up river bed from Benjamin Diaz. In its resolution dated 10 January 1990, the Court ordered that DBP be impleaded as a party respondent. On 13 September 1990, DBP filed a Manifestation/ Compliance stating that DBP's interest over TCT 139215 issued in its name (formerly TCT 34797 of the Del Rosarios and TCT 135170 of Benjamin Diaz) has been transferred to Spouses Victoriano and Pacita A. Tolentino pursuant to a Deed of Sale dated 11 September 1990. The Supreme Court reversed and set aside the remaining effective portion of the appealed decision which declares Del Rosario as riparian owner of the dried-up portion of Estero Calubcub. 1. Findings of appellate court conclusive to the Supreme Court; Exceptions The jurisdiction of the Supreme Court in cases brought to it from the Court of Appeals in a petition for certiorari under Rule 45 of the Rules of Court is limited to the review of errors of law, and that said appellate court's finding of fact is conclusive upon this Court. However, there are certain exceptions, such as (1) when the conclusion is a finding grounded entirely on speculation, surmises or conjectures; (2) when the inference made is manifestly absurd, mistaken or impossible; (3) when there is grave abuse of discretion in the appreciation of facts; (4) when the judgment is premised on a misapprehension of facts; (5) when the findings of fact are conflicting; and (6) when the Court of Appeals in making its findings went beyond the issues of the case and the same is contrary to the admissions of both appellant and appellee. 2. Evidence reveal change in the course of river not caused by natural forces A careful perusal of the evidence presented by both parties in the case at bar will reveal that the change in the course of Estero Calubcub was caused, not by natural forces, but due to the dumping of garbage therein by the people of the surrounding neighborhood. There is nothing in the testimony of lone witness Florencia del Rosario nor in said relocation plan which would indicate that the change in the course of the estero was due to the ebb and flow of the waters. On the contrary, the testimony of the witness belies such fact, while the relocation plan is absolutely silent on the matter. The inescapable conclusion is that the driedup portion of Estero Calubcub was occasioned, not by a natural change in the course of the waters, but through the active intervention of man.

3. Article 370 of the old Civil Code applies only to natural change in the course of the waters; Law clear, no room for interpretation Article 370 of the old Civil Code which provides that "the beds of rivers, which are abandoned because of a natural change in the course of the waters, belong to the owners of the riparian lands throughout the respective length of each. If the abandoned bed divided tenements belonging to different owners the new dividing line shall be equidistant from one and the other." The law is clear and unambiguous; and leaves no room for interpretation. Article 370 applies only if there is a natural change in the course of the waters. The rules on alluvion do not apply to man-made or artificial accretions 23 nor to accretions to lands that adjoin canals or esteros or artificial drainage systems. Considering the finding that the dried-up portion of Estero Calubcub was actually caused by the active intervention of man, it follows that Article 370 does not apply to the present case and, hence, the Del Rosarios cannot be entitled thereto supposedly as riparian owners. 4. Dried up portion of Estero Calubcub belongs to public domain; Land used for drainage purposes cannot be subject of a miscellaneous sales application The dried-up portion of Estero Calubcub should thus be considered as forming part of the land of the public domain which cannot be subject to acquisition by private ownership. This is made more evident in the letter, dated 28 April 1989, of the Chief of the Legal Division of the Bureau of Lands, stating that the alleged application filed by Ronquillo no longer exists in its records as it must have already been disposed of as a rejected application for the reason that other applications covering Estero Calubcub, Sampaloc, Manila for areas other than that contested in the presented case, were all rejected by the office because of the objection interposed by the City Engineer's office that they need the same land for drainage purposes. Since the land is to be used for drainage purposes the same cannot be the subject of a miscellaneous sales application. 5. Del Rosario and Ronquillo estopped from claiming land is not public land The fact that Ronquillo and del Rosario filed their sales applications with the Bureau of Lands covering the subject dried-up portion of Estero Calubcub cannot but be deemed as outright admissions by them that the same is public land. They are now estopped from claiming otherwise. Panlilio v. Mercado [G.R. No. 18771. March 26, 1923.] First Division, Ostrand (J): 7 concur Facts: Nicolas Panlilio, Eutiquiano Cuyugan, and Sixto Timbol (plaintiffs) are the owners of various parcels of land in the municipality of Mexico, Pampanga. From 1911 until August, 1919, the parcels of land belonging to them were divided by a small river known as the Estero Abacan. The defendants Atilano Mercado and Ciriaco Pimping are the owners of various parcels of land which, previously to the month of August 1919, were situated to the east of the land of the plaintiffs and were not touched by the Abacan River. During the said month, a very heavy flood occurred in the Abacan River and when the flood subsided, the river no longer flowed in the channel through the lands of the plaintiffs but had opened a new course for itself through the lands of the defendants where it still continues to flow. This new course was the course of the river previous to the year 1911. It may be noted that in the years 1916 and 1917 a cadastral survey was made of the district where the lands of both the plaintiffs and the defendants are situated and that upon the plans of that survey the then course of the river is excluded from the cadaster and set apart as a public stream. After the termination of the 1919 rainy season and early in the year 1920, a complaint was made to the provincial board of Pampanga by various land owners, including the defendants Atilano Mercado and Ciriaco Pimping, setting forth that the new course of the river was destroying their land and rendering it useless and asking that the river be returned to its former channel. The complaint was endorsed to the district engineer and on 10 June 1920, the defendants Atilano Mercado and Ciriaco Pimping, accompanied by the defendant district engineer (Manuel Reyes), proceeded to the point where the river had first begun to change its course, and after locating this point upon the cadastral plan, proceeded with laborers of the defendants Atilano Mercado and Ciriaco Pimping to excavate the old bed of the river for the purpose of causing the river to return to this bed. On 25 June 1920, a petition for a writ of injunction was filed by the plaintiffs to restrain the defendants from entering upon the lands, and from disturbing the plaintiffs in their peaceful possession of the same. They also pray for damages for trespass on the land. Defendant Mercado and Pimping set up a counterclaim for P40,000, alleging that the plaintiffs, by placing bamboo stakes in the River Abacan, caused it to change its course, thus invading said defendants' lands and causing damages in the sum mentioned. The court absolved the defendants (landowners Mercado and Pimping, district engineer Manuel Reyes, and Pampanga Constabulary Commander Telesforo Martinez) from the complaint and the plaintiffs from the counterclaim, without cost. From this judgment, all of the parties appeal. The Supreme Court affirmed the judgment appealed from, without cost. 1. Action of river not the only and final determining factor in the abandonment of a river bed While the abandonment of the bed may be the consequence of the riving changing its course, it is not necessarily the action of the river itself which is the only and final determining factor in such abandonment. Article 370 of the Civil Code cannot be solely relied on to have the conclusion that the old bed of the river Abacan became ipso facto absolutely abandoned upon the river varying its course in 1919. 2. Estero Abacan is a public stream; Action of the government to return the river to its old course The river is a public stream; its bed is of public ownership and was definitely located and determined in the cadastral survey. In the case of a public stream, the bed is of public ownership and the public cannot be considered absolutely divested of this ownership until there is some indication of an intention of the part of the Government to acquiesce in the change in the course of the stream. The Government is not compelled to stand idly by and let nature take its course is clearly indicated by article 372 of the Civil Code. As soon as practicable after the river changed its course, steps were taken under the direction of the Government functionaries to bring it back into its old course and work was continued until interrupted by the present action. This certainly does not indicate abandonment on the part of the Government.

3. Claim for damages; Evidence falls short As to the claim for damages, the evidence undoubtedly shows that the plaintiffs placed bamboo stakes across the river and that the stakes may have caused an accumulation of sand or sediment which in turn may have contributed to the change in the course of the river, such evidenced however falls short of showing that this was the primary cause of the change and of the damage to the defendants' property. Agne v. Director of Lands [G.R. No. 40399. February 6, 1990.]; also Agne v. IAC [G.R. No. 72255. February 6, 1990.] Second Division, Regalado (J): 4 concur Facts: The land subject matter of the case was originally covered by Free Patent 23263 issued on 17 April 1937 in the name of Herminigildo Agpoon. On 21 May 1937, pursuant to the said patent, the Register of Deeds of Pangasinan issued to said Herminigildo Agpoon OCT 2370. Presentacion Agpoon Gascon inherited the said parcel of land upon the death of her father, Herminigildo, and was issued TCT 32209 on 6 April 1960. Presentacion declared the said land for taxation purposes in her name under TD 11506 and taxes were paid thereon in her name. On 13 April 1971, spouses Joaquin and Presentacion Gascon filed Civil Case U-2286 in the then CFI Pangasinan for recovery of possession and damages against Marcelino C. Agne, Felix Oriane, Agaton Taganas (represented by Florentino C. Taganas), Hilario Escorpizo, Isabelo Mauricio, Heirs of Roman Damaso (Jorge Damaso and Alejandro Damaso), Heirs of Francisco Ramos (Encarnacion R. Leano and Dominga R. Medrano), Heirs of Sabina Gelacio Agapito (Serapio Agapito and Nicolasa Agapito), Feliza Diccion Agne, Estanislao Gorospe (represented by Elizabeth G. Badua), Librado Badua, Nicolas Villanieva, Heirs of Carlos Palado (Fortunata Palado and Isabelita Palado), Primitivo Taganas, Panfilo Soingco, Bernardo Palattao, Marcelino S. Santos and Paulino D. Agne Jr. (minor, represented by Feliza Diccion Agne). Their complaint states that they are the registered owners under TCT 32209 of the parcel of land situated in Barrio Bantog, Asingan, Pangasinan which is now in the possession of Agne, et.al.; that during the Japanese occupation, the latter, taking advantage of the abnormal conditions then obtaining, took possession of said land by means of fraud, stealth, strategy and intimidation; that Gascon repeatedly demanded the surrender of the physical possession of said property but the latter refused. Agne, et.al. alleged that the land in question was formerly a part of the river bed of the Agno-Chico River; that in the year 1920, a big flood occurred which caused the said river to change its course and abandon its original bed; that by virtue of the provisions of Article 370 of the Spanish Civil Code which was then the law in force, Agne, et.al., by operation of law, became the owners by accession or accretion of the respective aliquot parts of said river bed bordering their properties; that since 1920, they and their predecessors in interest occupied and exercised dominion openly and adversely over said portion of the abandoned river bed in question abutting their respective riparian lands continuously up to the present to the exclusion of all other persons, particularly Herminigildo Agpoon; that they have introduced improvements thereon by constructing irrigation canals and planting trees and agricultural crops thereon and converted the land into a productive area.

On 6 March 1974, while the above case was still pending, Agne, et.al. filed a complaint against Director of Lands and spouses Agpoon with the former CFI Pangasinan for annulment of title, reconveyance of and/or action to clear title to a parcel of land, which action was docketed as Civil Case U-2649. Agne, et. al. alleged in their said complaint that the land in question, which was formerly a portion of the bed of Agno-Chico river which was abandoned as a result of the big flood in 1920, belongs to them pursuant to the provision of Article 370 of the old Civil Code; that it was only on 13 April 1971, when spouses filed a complaint against them, that they found out that the said land was granted by the Government to Herminigildo Agpoon under Free Patent 23263, pursuant to which OCT 2370 was issued in the latter's name; and that the said patent and subsequent titles issued pursuant thereto are null and void since the said land, an abandoned river bed, is of private ownership and, therefore, cannot be the subject of a public land grant. On 21 June 1974, the trial court rendered a decision in Civil Case U-2286, ordering Agne, et.al. to surrender physical possession of land in question, to pay in soludum the produce of the land (P5,000 per year) from the date of the filing of the action at the rate of 6% interest per annum until fully paid; to pay in solidum the amount of P800 representing attorneys fees; and to pay the costs. Agne, et.al. appealed to the appellate court. On 30 January 1985 the former IAC affirmed in toto in AC-GR CV 60388-R the decision of the trial court, and with the denial of Agne et.al.'s motion for reconsideration, Agne, et.al. filed a petition for review on certiorari with the Supreme Court (GR 72255).

On 24 June 1974, the CFI Pangasinan, acting on the motion to dismiss filed by the Director of Lands and spouses Agpoon, issued an order dismissing Civil Case U-2649 for annulment of title by merely citing the statement in the case of Antonio, et al. vs. Barroga, et al. that an action to annul a free patent many years after it had become final and indefeasible states no cause of action. Agne, et.al's motion for the reconsideration of the order was denied on 11 September 1974, hence the petition for review on certiorari (GR L-40399) The Supreme Court reversed and set aside the assailed decision of IAC in AC-GR CV 60388-R and the questioned order of dismissal of the trial court in its Civil Case 2649, and rendered judgment ordering the Gascon to reconvey the parcel of land to Agne, et.al. 1. Case of Antonio v. Barroga not controlling The lower court erred in ordering the dismissal of Civil Case U-2649, as the case of Antonio relied upon in the dismissal order is not controlling. In that case, the complaint was dismissed for failure to state a cause of action, not only because of the delay in the filing of the complaint but specifically since the ground relied upon by the plaintiff therein, i.e. that the land was previously covered by a titulo real, even if true, would not warrant the annulment of the free patent and the subsequent original certificate of title issued to defendant.

2. Cause of action; Facts alleged in complaint hypothetically admitted upon the filing of the motion to dismiss The facts alleged in the complaint, which are deemed hypothetically admitted upon the filing of the motion to dismiss, constitute a sufficient cause of action against private respondents. 3. Agne owns property; Riparian owners ipso facto owners o abandoned river bed (Old Civil Code) The old Civil Code, the law then in force, provides that the beds of rivers which remain abandoned because the course of the water has naturally changed belong to the owners of the riparian lands throughout their respective lengths. If the abandoned bed divided estates belonging to different owners, the new dividing line shall run at equal distance therefrom." Thus, once the river bed has been abandoned, the riparian owners become the owners of the abandoned bed to the extent provided by this article. The acquisition of ownership is automatic. There need be no act on the part of the riparian owners to subject the accession to their ownership, as it is subject thereto ipso jure from the moment the mode of acquisition becomes evident, without the need of any formal act of acquisition. Such abandoned river bed had fallen to the private ownership of the owner of the riparian land even without any formal act of his will and any unauthorized occupant thereof will be considered as a trespasser. The right in re to the principal is likewise a right in re to the accessory, as it is a mode of acquisition provided by law, as the result of the right of accretion. Since the accessory follows the nature of the principal, there need not be any tendency to the thing or manifestation of the purpose to subject it to our ownership, as it is subject thereto ipso jure from the moment the mode of acquisition becomes evident. The right of the owner of land to additions thereto by accretion has been said to rest in the law of nature, and to be analogous to the right of the owner of a tree to its fruits, and the owner of flocks and herds to their natural increase. In the present case, Agne, et.al. became owners of aliquot portions of said abandoned river bed as early as 1920, when the Agno River changed its course, without the necessity of any action or exercise of possession on their part, it being an admitted fact that the land in dispute, prior to its registration, was an abandoned bed of the Agno River and that Agne, et. al. are the riparian owners of the lands adjoining the said bed. 4. Ownership of accession governed by Civil Code, imprescriptibility of registered land governed by Land Registration Act; Imprescriptibility does not apply as title based on public grant The failure of Agne et.al. to register the accretion in their names and declare it for purposes of taxation did not divest it of its character as a private property. Although an accretion to registered land is not automatically registered and therefore not entitled or subject to the protection of imprescriptibility enjoyed by registered property under the Torrens system. The said rule is not applicable to the case since the title claimed by spouses Gascon is not based on acquisitive prescription but is anchored on a public grant from the Government, which presupposes that it was inceptively a public land. Ownership over the accession is governed by the Civil Code. Imprescriptibility of registered land is a concern of the Land Registration Act. 5. Land of private ownership; Director of Lands does not have authority to grant free patent for said land Under the provisions of Act 2874 pursuant to which the title of Gascon's predecessor in interest was issued, the President of the Philippines or his alter ego, the Director of Lands, has no authority to grant a free patent for land that has ceased to be a public land and has passed to private ownership, and a title so issued is null and void. The nullity arises, not from the fraud or deceit, but from the fact that the land is not under the jurisdiction of the Bureau of Lands. The jurisdiction of the Director of Lands is limited only to public lands and does not cover lands privately owned. The purpose of the Legislature in adopting the former Public Land Act, Act 2874, was and is to limit its application to lands of the public domain, and lands held in private ownership are not included therein and are not affected in any manner whatsoever thereby. Land held in freehold or fee title, or of private ownership, constitute no part of the public domain and cannot possibly come within the purview of said Act 2874, inasmuch as the "subject" of such freehold or private land is not embraced in any manner in the title of the Act and the same are excluded from the provisions or text thereof. In the present case, the land in dispute since early 1920 was already under the private ownership of Agne et.al. and no longer a part of the lands of the public domain, the same could not have been the subject matter of a free patent. Free Patent 23263 issued to Herminigildo Agpoon is null and void and the subsequent titles issued pursuant thereto cannot become final and indefeasible. 6. No title acquired by applicant patentees if land already a private property of another As ruled in Director of Lands vs. Sisican, et al. that if at the time the free patents were issued in 1953 the land covered therein were already private property of another and, therefore, not part of the disposable land of the public domain, then applicants patentees acquired no right or title to the land. 7. Void patents produce no legal effects A certificate of title fraudulently secured is null and void ab initio if the fraud consisted in misrepresenting that the land is part of the public domain, although it is not. The nullity arises, not from the fraud or deceit but, from the fact that the land is not under the jurisdiction of the Bureau of Lands. Being null and void, the free patent granted and the subsequent titles produce no legal effects whatsoever. Quod nullum est, nullum producit effectum. 8. Rule on incontrovertibility of certificate of title upon expiration of 1 year, does not apply in the present case The rule on the incontrovertibility of a certificate of title upon the expiration of one year, after the entry of the decree, pursuant to the provisions of the Land Registration Act, does not apply where an action for the cancellation of a patent and a certificate of title issued pursuant thereto is instituted on the ground that they are null and void because the Bureau of Lands had no jurisdiction to issue them at all, the land in question having been withdrawn from the public domain prior to the subsequent award of the patent and the grant of a certificate of title to another person. Such an action is different from a review of the decree of title on the ground of fraud. 9. Action to annul void title issued pursuant to a public grant does not prescribe Although a period of one year has already expired from the time a certificate of title was issued pursuant to a public grant, said title does not become incontrovertible but is null and void if the property covered thereby is originally of private ownership, and an action to annul the same does not prescribe. 10. Action to quite title imprescriptible since petitioners are in possession of land Since Agne, et.al. are in possession of the land in dispute, an action to quiet title is imprescriptible. Their action for reconveyance

which, in effect, seeks to quiet title to property in one's possession is imprescriptible. Their undisturbed possession for a number of years gave them a continuing right to seek the aid of a court of equity to determine the nature of the adverse claims of a third party and the effect on her title. As held in Caragay-Layno vs. Court of Appeals, et al., an adverse claimant of a registered land, undisturbed in his possession thereof for a period of more than 50 years and not knowing that the land he actually occupied had been registered in the name of another, is not precluded from filing an action for reconveyance which, in effect, seeks to quiet title to property as against the registered owner who was relying upon a Torrens title which could have been fraudulently acquired. To such adverse claimant, the remedy of an action to quiet title is imprescriptible. In actions for reconveyance of property predicated on the fact that the conveyance complained of was void ab initio, a claim of prescription of the action would be unavailing. 11. Land Registration Act and Cadastral Act does not give anybody better title than what he really or lawfully has A free patent which purports to convey land to which the Government did not have any title at the time of its issuance does not vest any title in the patentee as against the true owner. As stated in Gustillo v. Maravill (48 Phil 838), the Land Registration Act as well as the Cadastral Act protects only the holders of a title in good faith and does not permit its provisions to be used as a shield for the commission of fraud, or that one should enrich himself at the expense of another. The Acts do not give anybody, who resorts to the provisions thereof, a better title than he really and lawfully has. If he happened to obtain it by mistake or to secure, to the prejudice of his neighbor, more land than he really owns, with or without bad faith on his part, the certificate of title, which may have been issued to him under the circumstances, may and should be cancelled or corrected (Legarda and Prieto vs. Saleeby, 31 Phil., 590). 12. Agnes title superior over Gascons; Possession The title of Agne, et.al. over the land in dispute is superior to the title of the registered owner, Gascon, which is a total nullity. The long and continued possession of the former under a valid claim of title cannot be defeated by the claim of a registered owner whose title is defective from the beginning. 13. Conclusiveness of a Torrens title not available for use to perpetual fraud and chicanery; Land Registration Act; Registration is not a mode of acquiring property The quality of conclusiveness of a Torrens title is not available for use to perpetrate fraud and chicanery. To paraphrase from Angeles vs. Samia, the Land Registration Act does not create or vest title. It only confirms and records title already existing and vested. It does not protect a usurper from the true owner. It cannot be a shield for the commission of fraud. It does not permit one to enrich himself at the expense of another. The Torrens system was not established as a means for the acquisition of title to private land. It is intended merely to confirm and register the title which one may already have on the land. Where the applicant possesses no title or ownership over the parcel of land, he cannot acquire one under the Torrens system of registration. Resort to the provisions of the Land Registration Act does not give one a better title than he really and lawfully has. Registration does not vest title. It is not a mode of acquiring property. It is merely evidence of such title over a particular property. It does not give the holder any better title than what he actually has, especially if the registration was done in bad faith. The effect is that it is as if no registration was made at all. 14. Laches The failure of the spouses Gascon to assert their claim over the disputed property for almost 30 constitute laches and bars an action to recover the same. The registered owners' right to recover possession of the property and title thereto from Agne, et. al. has, by long inaction or inexcusable neglect, been converted into a stale demand. 15. Authority of the Court to order the reconveyance of property Where the evidence show that the plaintiff is the true owner of the land subject of the free patent and title granted to another and that the defendant and his predecessor in interest were never in possession thereof, the Court, in the exercise of its equity jurisdiction and without ordering the cancellation of said title issued upon the patent, may direct the defendant registered owner to reconvey the property to the plaintiff. Further, if the determinative facts are before the Court and it is in a position to finally resolve the dispute, the expeditious administration of justice will be subserved by such a resolution and thereby obviate the needless protracted proceedings consequent to the remand of the case of the trial court. On these considerations, as well as the fact that these cases have been pending for a long period of time, the Court, in the present case, see no need for remanding Civil Case 2649 for further proceedings, and hold that the facts and the ends of justice in this case require the reconveyance by the Gascons to Agne, et.al. of the disputed lot. Jagualing v. CA [G.R. No. 94283. March 4, 1991.] First Division, Gancayco (J): 4 concur Facts: A certain parcel of land is located in Sta. Cruz, Tagoloan, Misamis Oriental with an area of 16,452 sq. m., forming part of an island in a non-navigable river, bounded by the Tagoloan river on the north, south, and east and by the portion belonging to Vicente Neri on the west. Janita Eduave claims that she inherited the land from her father, Felomino Factura, together with his coheirs, Reneiro Factura and Aldenora Factura, and acquired sole ownership of the property by virtue of a Deed of Extra Judicial Partition with sale. The land is declared for tax purposes under Tax Declaration 26137 with an area of 16,452 sq. m. Since the death of her father on 5 May 1949, Eduave had been in possession of the property although the tax declaration remains in the name of the deceased father. The entire land had an area of 16,452 sq. m. appearing in the deed of extrajudicial partition, while in tax declaration the area is only 4,937 sq. m., and she reasoned out that she included the land that was under water. The land was eroded sometime in November 1964 due to typhoon Ineng, destroying the bigger portion and the improvements leaving only a coconut tree. In 1966 due to the movement of the river deposits on the land that was not eroded increased the area to almost half a hectare and in 1970 Eduave started to plant banana trees. In 1973, Maximo and Anuncita Jagualing asked her permission to plant corn and bananas provided that they prevent squatters to come to the area. Eduave engaged the services of a surveyor who conducted a survey and placed concrete monuments over the land. Eduave also paid taxes on the land in litigation, and mortgaged the land to the Luzon Surety and Co., for a consideration of P6,000.00. The land was the subject of a reconveyance case, in the CFI Misamis Oriental (Branch V, Cagayan de Oro City, Civil Case 5892), between Janita Eduave vs. Heirs of Antonio Factura, which was the subject of judgment by compromise in view of the amicable settlement of the parties, dated 31 May 1979. The

heirs of Antonio Factura had ceded a portion of the land with an area of 1,289 sq. m., to Janita Eduave in a notarial document of conveyance, pursuant to the decision of the CFI, after a subdivision of the lot 62 Pls-799, and containing 1,289 sq. m. was designated as Lot 62-A, and the subdivision plan was approved as Pls-799-Psd-10-001782. Eduave also applied for concession with the Bureau of Mines to extract 200 m3 of grave, and after an ocular inspection the permit was granted. Eduave, after permit was granted, entered into an agreement with Tagoloan Aggregates to extract sand and gravel, which agreement was registered in the office of the Register of Deeds. Maximo and Anuncita Jagualing assert that they are the real owners of the land in litigation containing an area of 18,000 sq. m. During the typhoon Ineng in 1964 the river control was washed away causing the formation of an island. Jagualing started occupying the land in 1969, paid land taxes as evidenced by tax declaration 26380 and tax receipts, and tax clearances. Actual occupation of the land by Jagualing included improvements and the house. Rudygondo and Janita Eduave filed with the RTC Misamis Oriental an action to quiet title and/or remove a cloud over the property in question against Jagualing. On 17 July 1987 the trial court dismissed the complaint for failure of Eduave to establish by preponderance of evidence their claim of ownership over the land in litigation. The court found that the island is a delta forming part of the river bed which the government may use to reroute, redirect or control the course of the Tagoloan River. Accordingly, it held that it was outside the commerce of man and part of the public domain, citing Article 420 of the Civil Code. As such it cannot be registered under the land registration law or be acquired by prescription. The trial court, however, recognized the validity of Jagualing's possession and gave them preferential rights to use and enjoy the property. The trial court added that should the State allow the island to be the subject of private ownership, the Jagualings have rights better than that of Eduave. On appeal to the Court of Appeals, the court found that the island was formed by the branching off of the Tagoloan River and subsequent thereto the accumulation of alluvial deposits. Basing its ruling on Articles 463 and 465 of the Civil Code, the Court of Appeals reversed the decision of the trial court, declared Eduave as the lawful and true owners of the land subject of the case and ordered Jagualing to vacate the premises and deliver possession of the land to Eduave. Hence, the present petition. The Supreme Court found no error committed by the appellate court, denied the petition for lack of sufficient merit, and affirmed the decision of the Court of Appeals; without pronouncement as to costs. 1. Evidence not properly appreciated by trial court; CA properly applied Article 463 The appellate court reversed the decision of the trial court because it did not take into account the other pieces of evidence in favor of the private respondents. The complaint was dismissed by the trial court because it did not accept Eduaves explanation regarding the initial discrepancy as to the area they claimed (4937sq.m. v. 16452 sq.m.); because it favored the theory that Eduave became interested in the land only in 1979 not for agricultural purposes but in order to extract gravel and sand, which is belied by other circumstances tantamount to acts of ownership exercised by Eduave over the property prior to said year (e.g. the payment of land taxes thereon, the monuments placed by the surveyor whose services were engaged by Eduave, the agreement entered into by Eduave and Tagoloan Aggregates to extract gravel and sand, which agreement was duly registered with the Register of Deeds); because it disregarded the testimony of 2 disinterested witnesses (Gergorio Neri, as to metes and bounds of the property and the effect of the typhoon; and Candida Ehem, as to the caretaker agreement between her and Eduave) without explaining why it doubted their credibility. From the evidence thus submitted, the appellate court had sufficient basis for the finding that the property of Eduave actually existed and was identified prior to the branching off or division of the river. The Court of Appeals, therefore, properly applied Article 463 of the Civil Code which allows the ownership over a portion of land separated or isolated by river movement to be retained by the owner thereof prior to such separation or isolation. 2. Island formed in a non-navigable and non-floatable river; Article 465 The parcel of land is part of an island that formed in a non-navigable and non-flotable river; from a small mass of eroded or segregated outcrop of land, it increased to its present size due to the gradual and successive accumulation of alluvial deposits. The Court of Appeals did not err in applying Article 465 of the Civil Code. Under this provision, the island belongs to the owner of the land along the nearer margin as sole owner thereof; or more accurately, because the island is longer than the property of Eduave, they are deemed ipso jure to be the owners of that portion which corresponds to the length of their property along the margin of the river. 3. Land formed by accretion belongs to riparian owner, even without a specific act of possession over it; Land however may yield to adverse possession of third party if riparian owner fails to assert claim Lands formed by accretion belong to the riparian owner. This preferential right is, under Article 465, also granted the owners of the land located in the margin nearest the formed island for the reason that they are in the best position to cultivate and attend to the exploitation of the same. In fact, no specific act of possession over the accretion is required. If, however, the riparian owner fails to assert his claim thereof, the same may yield to the adverse possession of third parties, as indeed even accretion to land titled under the torrens system must itself still be registered. 4. Doctrine of acquisitive prescription The property may be acquired by adverse possession for the required number of years under the doctrine of acquisitive prescription. Jagualings possession cannot be considered in good faith, however, because they are presumed to have notice of the status of Eduave as riparian owners who have the preferential right to the island as recognized and accorded by law; they may claim ignorance of the law, specifically Article 465 of the Civil Code, but such is not, under Articles 3 and 526 of the same code, an adequate and valid defense to support their claim of good faith. Hence, not qualifying as possessors in good faith, they may acquire ownership over the island only through uninterrupted adverse possession for a period of thirty years. By their own admission, Jagualing have been in possession of the property for only about 15 years, and thus, the island cannot be adjudicated in their favor. 5. Origin of island not tackled as case is not between opposing riparian owners but between a riparian owner and one in possession of the land There is no need to make a final determination regarding the origins of the island, i.e., whether the island was initially formed by the branching off or division of the river and covered by Article 463 of the Civil Code, in which case there is strictly no accession because the original owner retains ownership, or whether it was due to the action of the river under Article 465, or whether it was

caused by the abrupt segregation and washing away of the stockpile of the river control, which makes it a case of avulsion under Article 459, as the case is not between parties as opposing riparian owners contesting ownership over an accession but rather between a riparian owner and the one in possession of the island. 6. Quasi in Rem; Judgment conclusive upon the parties and does not bind the State and other riparian owners The Court is not prepared to concede that the island is a delta which should be outside the commerce of man and that it belongs to the State as property of the public domain in the absence of any showing that the legal requirements to establish such a status have been satisfied, which duty properly pertains to the State. Since the petition is an upshot of the action to quiet title brought by Eduave against Jagualing, it is thus not technically an action in rem or an action in personam, but characterized as quasi in rem, which is an action in personam concerning real property. Thus, the judgment in proceedings of this nature is conclusive only between the parties and does not bind the State or the other riparian owners who may have an interest over the island involved herein. German Management & Services v. CA (Gernale, Villeza) [G.R. No. 76216 and 76217. September 14, 1989.] Third Division, Fernan (J): 2 concur, 1 concur in result, 1 on leave Facts: Spouses Cynthia Cuyegkeng Jose and Manuel Rene Jose, residents of Pennsylvania, Philadelphia, USA are the owners of a parcel of land situated in Sitio Inarawan, San Isidro, Antipolo, Rizal, with an area of 232,942 sq. m. (TCT 50023 of the Register of Deeds Rizal issued 11 September 1980 cancelling TCT 56762/ T-560). The land was originally registered on 5 August 1948 in the Office of the Register of Deeds Rizal as OCT 19, pursuant to a Homestead Patent granted by the President of the Philippines on 27 July 1948, under Act 141. On 26 February 1982, the spouses Jose executed a special power of attorney authorizing German Management Services to develop their property into a residential subdivision. Consequently, on 9 February 1983 the German Management obtained Development Permit 00424 from the Human Settlements Regulatory Commission for said development. Finding that part of the property was occupied by Gernale and Villeza and 20 other persons, German Management advised the occupants to vacate the premises but the latter refused. Nevertheless, German Management proceeded with the development of the subject property which included the portions occupied and cultivated by Gernale, et.al. Gernale, et.al. filed an action for forcible entry against German Management before the MTC Antipolo, Rizal, alleging that they are mountainside farmers of Sitio Inarawan who have occupied and tilled their farmholdings some 12 to 15 years prior to the promulgation of PD27, and that they were deprived of their property without due process of law when German Management forcibly removed and destroyed the barbed wire fence enclosing their farmholdings without notice and bulldozing the rice, corn, fruit bearing trees and other crops that they planted by means of force, violence and intimidation.. On 7 January 1985, the MTC dismissed Gernale et.al.'s complaint for forcible entry. On appeal, the RTC Antipolo, Rizal, Branch LXXI sustained the dismissal by the MTC. Gernale then filed a petition for review with the Court of Appeals. On 24 July 1986, said court gave due course to their petition and reversed the decisions of the MTC and the RTC. The Appellate Court held that since Gernale, et.al. were in actual possession of the property at the time they were forcibly ejected by German Management, they have a right to commence an action for forcible entry regardless of the legality or illegality of possession. German Management moved to reconsider but the same was denied by the Appellate Court in its resolution dated 26 September 1986. Hence the present recourse. The Supreme Court denied the petition, and affirmed the CA decision; with costs against German Management. 1. No violation of due process by appellate court The Court of Appeals need not require petitioner to file an answer for due process to exist. The comment filed by petitioner on 26 February 1986 has sufficiently addressed the issues presented in the petition for review filed by private respondents before the Court of Appeals. Having heard both parties, the Appellate Court need not await or require any other additional pleading. Moreover, the fact that petitioner was heard by the Court of Appeals on its motion for reconsideration negates any violation of due process. 2. Actual possessors can commence forcible entry; Ownership not an issue Notwithstanding the claim that German Management was duly authorized by the owners to develop the subject property, the actual possessors can commence a forcible entry case against the former because ownership is not in issue. Forcible entry is merely a quieting process and never determines the actual title to an estate. Title is not involved. Gernale et.al were already in peaceable possession of the property at the time German Management entered the property, manifested by the fact that they even planted rice, corn and fruit bearing trees 12 to 15 years prior to German Management's act of destroying their crops. 3. Prior possessor has security to remain in property until lawfully ejected by person having better right by accion publiciana or accion reivindicatoria A party may validly claim ownership based on the muniments of title it may present, such evidence does not responsively address the issue of prior actual possession raised in a forcible entry case. It must be stated that regardless of the actual condition of the title to the property, the party in peaceable quiet possession shall not be turned out by a strong hand, violence or terror. Thus, a party who can prove prior possession can recover such possession even against the owner himself. Whatever may be the character of his prior possession, if he has in his favor priority in time, he has the security that entitles him to remain on the property until he is lawfully ejected by a person having a better right by accion publiciana or accion reivindicatoria. 4. Doctrine of self-help can only be exercised only at the time of actual or threatened dispossession of property The justification that the drastic action of bulldozing and destroying the crops of the prior possessor on the basis of the doctrine of self help (enunciated in Article 429 NCC) is unavailing because the such doctrine can only be exercised at the time of actual or threatened dispossession, which is absent in the present case. When possession has already been lost, the owner must resort to judicial process for the recovery of property. This is clear from Article 536 NCC that , "in no case may possession be acquired through force or intimidation as long as there is a possessor who objects thereto. He who believes that he has an action or right to deprive another of the holding of a thing, must invoke the aid of the competent court, if the holder should refuse to deliver the thing."

TUMALAD V. VICENCIO 41 SCRA 143 FACTS: Vicencio and Simeon executed a chattel mortgage in favor of plaintiffs Tumalad over their house, which was being rented by Madrigal and company. This was executed to guarantee a loan, payable in one year with a 12% per annum interest. The mortgage was extrajudicially foreclosed upon failure to pay the loan. The house was sold at a public auction and the plaintiffs were the highest bidder. A corresponding certificate of sale was issued. Thereafter, the plaintiffs filed an action for ejectment against the defendants, praying that the latter vacate the house as they were the proper owners. ISSUE: WHETHER OR NOT THE SUBJECT MATTER OF THE MORTGAGE, A HOUSE OF STRONG MATERIALS, BE THE OBKECT OF A CHATTEL MORTGAGE? HELD: Certain deviations have been allowed from the general doctrine that buildings are immovable property such as when through stipulation, parties may agree to treat as personal property those by their nature would be real property. This is partly based on the principle of estoppel wherein the principle is predicated on statements by the owner declaring his house as chattel, a conduct that may conceivably stop him from subsequently claiming otherwise. In the case at bar, though there be no specific statement referring to the subject house as personal property, yet by ceding, selling or transferring a property through chattel mortgage could only have meant that defendant conveys the house as chattel, or at least, intended to treat the same as such, so that they should not now be allowed to make an inconsistent stand by claiming otherwise. PRUDENTIAL BANK V. PANIS

153 SCRA 390 FACTS: Spouses Magcale secured a loan from Prudential Bank. To secure payment, they executed a real estate mortgage over a residential building. The mortgage included also the right to occupy the lot and the information about the sales patent applied for by the spouses for the lot to which the building stood. After securing the first loan, the spouses secured another from the same bank. To secure payment, another real estate mortgage was executed over the same properties. The Secretary of Agriculture then issued a Miscellaneous Sales Patent over the land which was later on mortgaged to the bank. The spouses then failed to pay for the loan and the REM was extrajudicially foreclosed and sold in public auction despite opposition from the spouses. The respondent court held that the REM was null and void. ISSUES: 1. WHETHER OR NOT THE DEEDS OF REAL ESTATE MORTGAGE ARE VALID; AND 2. WHETHER OR NOT THE SUPERVENING ISSUANCE IN FAVOR OF PRIVATE RESPONDENTS OF MISCELLANEOUS SALES PATENT NO. 4776 ON APRIL 24, 1972 UNDER ACT NO. 730 AND THE COVERING ORIGINAL CERTIFICATE OF TITLE NO. P-2554 ON MAY 15,1972 HAVE THE EFFECT OF INVALIDATING THE DEEDS OF REAL ESTATE MORTGAGE. HELD: A real estate mortgage can be constituted on the building erected on the land belonging to another. The inclusion of building distinct and separate from the land in the Civil Code can only mean that the building itself is an immovable

property. While it is true that a mortgage of land necessarily includes in the absence of stipulation of the improvements thereon, buildings, still a building in itself may be mortgaged by itself apart from the land on which it is built. Such a mortgage would still be considered as a REM for the building would still be considered as immovable property even if dealt with separately and apart from the land. The original mortgage on the building and right to occupancy of the land was executed before the issuance of the sales patent and before the government was divested of title to the land. Under the foregoing, it is evident that the mortgage executed by private respondent on his own building was a valid mortgage. As to the second mortgage, it was done after the sales patent was issued and thus prohibits pertinent provisions of the Public Land Act. SERGS PRODUCTS AND GOQUIOLAY V. PCI LEASING AND FINANCE 338 SCRA 499 FACTS: PCI filed a case for collection of a sum of money as well as a writ of replevin for the seizure of machineries, subject of a chattel mortgage executed by petitioner in favor of PCI. Machineries of petitioner were seized and petitioner filed a motion for special protective order. It asserts that the machineries were real property and could not be subject of a chattel mortgage. The Issue

A. Whether or not the machineries purchased and imported by SERGS became real property by virtue of immobilization. HELD: The machineries in question have become immobilized by destination because they are essential and principal elements in the industry, and thus have become immovable in nature. Nonetheless, they are still proper subjects for a chattel mortgage. Contracting parties may validly stipulate that a real property be considered as personal. After agreement, they are consequently estopped from claiming otherwise.

TSAI V. COURT OF APPEALS 336 SCRA 324 FACTS: EVERTEX secured a loan from PBC, guaranteed by a real estate and chattel mortgage over a parcel of land where the factory stands, and the chattels located therein, as included in a schedule attached to the mortgage contract. Another loan was obtained secured by a chattel mortgage over properties with similar descriptions listed in the first schedule. During the date of execution of the second mortgage, EVERTEX purchased machineries and equipment. Due to business reverses, EVERTEX filed for insolvency proceedings. It failed to pay its obligation and thus, PBC initiated extrajudicial foreclosure of the mortgages. PBC was the highest bidder in the public auctions, making it the owner of the properties. It then leased the factory premises

to Tsai. Afterwards, EVERTEX sought the annulment of the sale and conveyance of the properties to PBC as it was allegedly a violation of the INSOLVENCY LAW. The RTC held that the lease and sale were irregular as it involved properties not included in the schedule of the mortgage contract. HELD: While it is true that the controverted properties appear to be immobile, a perusal of the contract of REM and CM executed by the parties gives a contrary indication. In the case at bar, both the trial and appellate courts show that the intention was to treat the machineries as movables or personal property. Assuming that the properties were considered immovables, nothing detracts the parties from treating it as chattels to secure an obligation under the principle of estoppel. Sec. of DENR vs. yap g.r. no. 167707 Facts: Respondents-claimants contended that there is no need for a proclamation reclassifying Boracay into agricultural land. Being classified as neither mineral nor timber land, the island is deemed agricultural pursuant to the Philippine Bill of 1902 and Act No. 926, known as the first Public Land Act.[32] Thus, their possession in the concept of owner for the required period entitled them to judicial confirmation of imperfect title. Opposing the petition, the OSG argued that petitioners-claimants do not have a vested right over their occupied portions in the island. Boracay is an unclassified public forest land pursuant to Section 3(a) of PD No. 705. Being public forest, the claimed portions of the island are

inalienable and cannot be the subject of judicial confirmation of imperfect title. It is only the executive department, not the courts, which has authority to reclassify lands of the public domain into alienable and disposable lands. There is a need for a positive government act in order to release the lots for disposition. Issue: Is PGMA's presidential proclaimation no. 1064 classifying boracay island into 400 hectares of reserved and forest land and 628 hectares of agricultural land (alienable and disposable) valid and constitutional? Ruling: In issuing Proclamation No. 1064, President Gloria MacapagalArroyo merely exercised the authority granted to her to classify lands of the public domain, presumably subject to existing vested rights. Classification of public lands is the exclusive prerogative of the Executive Department, through the Office of the President. Proclamation No. 1064 classifies Boracay into 400 hectares of reserved forest land and 628.96 hectares of agricultural land. The Proclamation likewise provides for a 15-meter buffer zone on each side of the center line of roads and trails, which are reserved for right of way and which shall form part of the area reserved for forest land protection purposes. Contrary to private claimants argument, there was nothing invalid or irregular, much less unconstitutional, about the classification of Boracay Island made by the President through Proclamation No. 1064. It was within her authority to make such classification, subject to existing vested rights. HEIRS OF MARIO MALABANAN vs. REPUBLIC OF THE PHILIPPINES FACTS: On 20 February 1998, Mario Malabanan filed an application for land registration before the RTC of Cavite-Tagaytay, covering a parcel of land situated in Silang Cavite, consisting of 71,324 square meters.

Malabanan claimed that he had purchased the property from Eduardo Velazco, and that he and his predecessors-in-interest had been in open, notorious, and continuous adverse and peaceful possession of the land for more than thirty (30) years. Velazco testified that the property was originally belonged to a twenty-two hectare property owned by his great-grandfather, Lino Velazco. Lino had four sons Benedicto, Gregorio, Eduardo and Estebanthe fourth being Aristedess grandfather. Upon Linos death, his four sons inherited the property and divided it among themselves. But by 1966, Estebans wife, Magdalena, had become the administrator of all the properties inherited by the Velazco sons from their father, Lino. After the death of Esteban and Magdalena, their son Virgilio succeeded them in administering the properties, including Lot 9864-A, which originally belonged to his uncle, Eduardo Velazco. It was this property that was sold by Eduardo Velazco to Malabanan. Among the evidence presented by Malabanan during trial was a Certification dated 11 June 2001, issued by the Community Environment & Natural Resources Office, Department of Environment and Natural Resources (CENRO-DENR), which stated that the subject property was verified to be within the Alienable or Disposable land per Land Classification Map No. 3013 established under Project No. 20-A and approved as such under FAO 4-1656 on March 15, 1982. On 3 December 2002, the RTC approved the application for registration. The Republic interposed an appeal to the Court of Appeals, arguing that Malabanan had failed to prove that the property belonged to the alienable and disposable land of the public domain, and that the RTC had erred in finding that he had been in possession of the property in the manner and for the length of time required by law for confirmation of imperfect title. On 23 February 2007, the Court of Appeals reversed the RTC ruling and dismissed the application of Malabanan.

G. R. NO. 158449

October 22, 2004

LUNINGNING P. DEL ROSARIO-IGTIBEN, JOSE REYES IGTIBEN, JOSE DEL ROSARIO IGTIBEN, JR. and THERESA TOPACIO MEDINA, petitioners, vs. REPUBLIC OF THE PHILIPPINES and THE COURT OF APPEALS, respondents. petitioners filed with the trial court an application for registration of land under Presidential Decree (PD) No. 1529, covering a parcel of land. Petitioners alleged that they acquired the Subject Property by purchase, and that they, by themselves and through their predecessors-in-interest, had been in actual, continuous, uninterrupted, open, public, and adverse possession of the Subject Property in the concept of owner for more that 30 years.3 the Tonido family sold the Subject Property to petitioners, as evidenced by a Deed of Absolute Sale.7 The history of possession of the Subject Property, was supported by tax declarations in the name of petitioners and their predecessors-ininterest from 1958 to 1998.8 , the trial court rendered a decision approving petitioners application for registration of the Subject Property. The Republic of the Philippines, represented by the Office of the Solicitor General, appealed the decision of the trial court to the Court of Appeals. In its appeal, the Republic alleged that the trial court erred in approving the application for registration despite petitioners failure to prove open, continuous, exclusive and notorious possession and occupation of the Subject Property since 12 June 1945, or earlier, as required by Section 48(b) of Commonwealth Act No. 141, otherwise known as the Public Land Act, as amended by PD No. 1073. Moreover, petitioners

also failed to produce monuments of title to tack their possession to those of their predecessors-in-interest in compliance with the prescriptive period required by law.9 On 20 December 2002, the Court of Appeals rendered a decision finding the appeal meritorious, setting aside the decision of the trial court, and dismissing the application for registration of petitioners.10 The Court of Appeals denied petitioners Motion for Reconsideration in its resolution dated 22 May 2003.11Petitioners filed this petition for review on certiorari Issue: 1) Whether petitioners have complied with the period of possession and occupation required by the Public Land Act? Held: (1) Section 44 of the Public Land Act, as amended by RA No. 6940, which provides for a prescriptive period of thirty (30) years possession, applies only to applications for free patents; (2) The case at bar is a judicial application for confirmation of an imperfect or incomplete title over the Subject Property covered by Section 48(b) of the Public Land Act; and (3) Section 48(b) of the Public Land Act requires for judicial confirmation of an imperfect or incomplete title the continuous possession of the land since 12 June 1945, or earlier, which petitioners herein failed to comply with.

G.R. No. 134209

January 24, 2006

REPUBLIC OF THE PHILIPPINES, Petitioner, vs. CELESTINA NAGUIAT, Respondent. This is an application for registration of title to four (4) parcels of land located in Panan, Botolan, Zambales, filed by Celestina Naguiat on 29 December 1989 with the Regional Trial Court of Zambales, Branch 69. Applicant [herein respondent] alleges, inter alia, that she is the owner of the said parcels of land having acquired them by purchase from the LID Corporation which likewise acquired the same from Demetria Calderon, Josefina Moraga and Fausto Monje and their predecessors-ininterest who have been in possession thereof for more than thirty (30) years; and that to the best of her knowledge, said lots suffer no mortgage or encumbrance of whatever kind nor is there any person having any interest, legal or equitable, or in possession thereof. On 29 June 1990, the Republic of the Philippines [herein petitioner]. . . filed an opposition to the application on the ground that neither the applicant nor her predecessors-in interest have been in open, continuous, exclusive and notorious possession and occupation of the lands in question since 12 June 1945 or prior thereto; that the muniments of title and tax payment receipts of applicant do not constitute competent and sufficient evidence of a bona-fide acquisition of the lands applied for or of his open, continuous, exclusive and notorious possession and occupation thereof in the concept of (an) owner; that the applicants claim of ownership in fee simple on the basis of Spanish title or grant can no longer be availed of . . .; and that the parcels of land applied for are part of the public domain belonging to the Republic of the Philippines not subject to private appropriation.

the trial court rendered judgment for herein respondent Celestina Naguiat, adjudicating unto her the parcels of land in question and decreeing the registration thereof in her name, With its motion for reconsideration having been denied by the trial court, petitioner Republic went on appeal to the CA in CA-G.R. CV No. 37001. As stated at the outset hereof, the CA, in the herein assailed decision of May 29, 1998, affirmed that of the trial court, Hence, the Republics present recourse on its basic submission that the CAs decision "is not in accordance with law, jurisprudence and the evidence, since respondent has not established with the required evidence her title in fee simple or imperfect title in respect of the subject lots which would warrant their registration under (P.D. 1529 or Public Land Act (C.A.) 141." In particular, petitioner Republic faults the appellate court on its finding respecting the length of respondents occupation of the property subject of her application for registration and for not considering the fact that she has not established that the lands in question have been declassified from forest or timber zone to alienable and disposable property. Issue: whether or not the areas in question have ceased to have the status of forest or other inalienable lands of the public domain. public lands not shown to have been reclassified or released as alienable agricultural land or alienated to a private person by the State remain part of the inalienable public domain.13 Under Section 6 of the Public Land Act, the prerogative of classifying or reclassifying lands of the public domain, i.e., from forest or mineral to agricultural and vice versa, belongs to the Executive Branch of the government and not the court.14 Needless to stress, the onus to overturn, by incontrovertible

evidence, the presumption that the land subject of an application for registration is alienable or disposable rests with the applicant.15 respondent never presented the required certification from the proper government agency or official proclamation reclassifying the land applied for as alienable and disposable. declassification of forest and mineral lands, as the case may be, and their conversion into alienable and disposable lands need an express and positive act from the government.21 The foregoing considered, the issue of whether or not respondent and her predecessor-in-interest have been in open, exclusive and continuous possession of the parcels of land in question is now of little moment. For, unclassified land, as here, cannot be acquired by adverse occupation or possession; occupation thereof in the concept of owner, however long, cannot ripen into private ownership and be registered as title.22

Director of Forestry v. Villareal [G.R. No. L-32266. February 27, 1989.] En Banc, Cruz (J): 13 concur, 1 took no part. Facts: Ruperto Villareal applied for its registration on 25 January 1949, a land consisting of 178,113 sq. m. of mangrove swamps located in the municipality of Sapian, Capiz, alleging that he and his predecessors-ininterest had been in possession of the land for more than 40 years. He was opposed by several persons, including the Director of Foresty on behalf of the Republic of the Philippines. After trial, the application was approved by the CFI Capiz. The decision was affirmed by the Court of Appeals. The Director of Forestry then came to the Supreme Court in a petition for review on certiorari.

ISSUE: Whether or not the land in dispute was forestal in nature and not subject to private appropriation? HELD: The Supreme Court set aside the decision of the Court of Appeals and dismissed the application for registration of title of Villareal, 4. Administrative Code of 1917; Mangrove swamps form part of the public forests of the country Subsequently, the Philippine Legislature categorically declared that mangrove swamps form part of the public forests of this country. This it did in the Administrative Code of 1917, which became effective on 1 October 1917, providing in Section 1820 of said code that for the purpose of this chapter 'public forest' includes, except as otherwise specially indicated, all unreserved public land, including nipa and mangrove swamps, and all forest reserves of whatever character." The legislative definition embodied in Section 1820 of the Revised Administrative Code of 1917, remains unamended up to now, provides that mangrove swamps or manglares form part of the public forests of the Philippines. As such, they are not alienable under the Constitution and may not be the subject of private ownership until and unless they are first released as forest land and classified as alienable agricultural land. EDUBIGIS GORDULA, CELSO V. FERNANDEZ, JR., CELSO A. FERNANDEZ, NORA ELLEN ESTRELLADO, DEVELOPMENT BANK OF THE PHILIPPINES, J.F. FESTEJO AND CO., INC. AND REGISTER OF DEEDS OF LAGUNA, Petitioners, vs. THE HONORABLE COURT OF APPEALS and REPUBLIC OF THE PHILIPPINES (represented by the National Power Corporation), Respondents. DECISION PUNO, J.: virtualawlibrary

former President Ferdinand E. Marcos issued Proclamation No. 573[3] withdrawing from sale and settlement and setting aside as permanent forest reserves, subject to private rights, certain parcels of the public domain which included Parcel No. 9 - Caliraya-Lumot River Forest Reserve. They were primarily for use as watershed area. the parcel of land subject of the case at bar is, by petitioners' explicit admission,[4] within Parcel No. 9, the Caliraya-Lumot River Forest Reserve. virtualawlibrary petitioner Edubigis Gordula filed with the Bureau of Lands, an Application[5] for a Free Patent over the land. Manuel Fernandez and several others also filed free patent applications covering other parcels of land in the area. Mr. Antonio Aquino, Jr., the Civil Security Officer of the Cavinti reservoir complex, sent a Memorandum to the President of the Napocor informing him of the fences and roads being constructed in the saddle area, more particularly, in the lots sold by petitioner Fernandez to petitioner Estrellado. virtualawlibrary Respondent Republic, through the Napocor, filed against petitioners a Complaint for Annulment of Free Patent and Cancellation of Titles and Reversion with Writ of Preliminary Injunction in the RTC of Sta. Cruz, Laguna.[8] the trial court rendered judgment in favor of petitioners. Respondent Republic, through the Napocor, elevated the case to the respondent Court of Appeals. virtualawlibrary On June 20, 1996, the respondent Court of Appeals ruled against petitioners. Hence, this petition.

ISSUE: Whether or not the subject parcel of lands are non-disposable and inalienable public land? HELD: the two (2) parcels of land were public disposable and alienable lands before the issuance, by the former President, of Proclamation No. 573, on June 26, 196[9]. x x x The property was, however, later reserved, under Proclamation No. 573, as a permanent forest, on June 26, 196[9]. Since then, the property became non-disposable and inalienable public land. By their very nature[13] or by executive or statutory fiat, they are outside the commerce of man, unsusceptible of private appropriation in any form,[14] and inconvertible into any character less than of inalienable public domain, regardless of their actual state, for as long as the reservation subsists and is not revoked by a subsequent valid declassification.[15] virtualawlibrary Petitioners do not contest the nature of the land in the case at bar. It is admitted that it lies in the heart of the Caliraya-Lumot River Forest Reserve, which Proclamation No. 573 classified as inalienable and indisposable. No public land can be acquired by private persons without any grant, express or implied from the government; it is indispensable that there be a showing of a title from the state.[17] The facts show that petitioner Gordula, did not acquire title to the subject land prior to its reservation under Proclamation No. 573. He filed his application for free patent only in January, 1973, more than three (3) years after the issuance of Proclamation No. 573 in June, 1969. At that time, the land, as part of the Caliraya-Lumot River Forest Reserve, was no longer open to private ownership as it has been classified as public forest reserve for the public good. virtualawlibrary

brary

Spouses GEMINIANO and AMPARO DE OCAMPO and Spouses PEDRO and CRISANTA SANTOS, petitioners, vs. FEDERICO ARLOS, MARY ARLOS, TEOFILO OJERIO and BELLA OJERIO, respondents. DECISION PANGANIBAN, J.: Under the Public Land Act as amended, only titles to alienable and disposable lands of the public domain may be judicially confirmed. Unless a public land is reclassified and declared as such, occupation thereof in the concept of owner, no matter how long ago, cannot confer ownership or possessory rights. A suit for the reversion of such property to the State may be instituted only by the Office of the Solicitor General (OSG).
The Facts

Federico S. Arlos and Teofilo D. Ojerio filed an application for registration, docketed as Land Registration Case No. N-340, wherein they seek judicial confirmation of their titles [to] three parcels of land, all located at Cabcaben, Mariveles, Bataan,. Spouses Geminiano de Ocampo and Amparo De Ocampo and spouses Pedro Santos and Crisanta Santos opposed the application for registration, alleging that they are the co-owners of Lots 1 and 2 of Plan SGS 3062, situated at Cabcaben, Mariveles, Bataan, and their ownership is evidenced by Transfer Certificate of Title Nos. T-43298 and T-44205, and that they became owners of said lots by purchase from the government through sales patents. The Republic of the Philippines also opposed the application, contending that neither the applicants nor their predecessors-in-interests have been in open, continuous, exclusive and notorious possession and occupation of the lands in question for at least 30 years immediately

preceding the filing of the application; and that the parcels of land applied for are portions of the public domain belonging to the Republic of the Philippines not subject to private appropriation. the CA ruled that petitioners had failed to comply with the Public Land Act, which required sales patent applicants to be the actual occupants and cultivators of the land. It held that the testimonies of petitioners, which were incongruous with reality, bolstered the finding that [they had] never occupied, cultivated or made improvements on the property.
The Issues

ISSUE: Whther or not (1) the registration of respondents title under the Public Land Act is proper?
HELD:

Respondents application for registration of title to the three parcels of land that were once part of the public domain is governed by the Public Land Act, a title may be judicially confirmed under Section 48 of the Public Land Act only if it pertains to alienable lands of the public domain. Unless such assets are reclassified and considered disposable and alienable, occupation thereof in the concept of owner, no matter how long cannot ripen into ownership and be registered as a title. Verily, Presidential Decree No. 1073 clarified Section 48 (b) of the Public Land Act by specifically declaring that the latter applied only to alienable and disposable lands of the public domain. In the present case, the disputed land which was formerly a part of a US military reservation that had been turned over to the Philippine government in 1965, was declared disposable and alienable only in 1971. Second, respondents and their predecessors-in-interest could not have occupied the subject property from 1947 until 1971 when the land was

declared alienable and disposable, because it was a military reservation at the time. Hence, it was not subject to occupation, entry or settlement. We reiterate that the land was declared alienable only in 1971; hence, respondents have not satisfied the thirty-year requirement under the Public Land Act. Moreover, they could not have occupied the property for thirty years, because it formed part of a military reservation. Clearly then, their application for the registration of their titles was erroneously granted by the appellate and the trial courts.

G.R. No. L-10510 LEONCIO ZARATE, applicant-appellant, vs. THE DIRECTOR OF LANDS, ET AL., objectors-appellees. Aurelio Cecilio for appellant. Attorney-General Avancea in behalf of Director of Lands. Moreland, J.: FACTS:This is a proceeding to register the title to lands described in the petition. The Government of the Philippine Islands interposed an objection to the registration of title on the ground "that said parcel of land was part of the public domain and is occupied by Apolonio Gamido and Bibiana Olivite by virtue of applications made by them for homesteads Nos. 2061 and 5626, respectively. Registration of title to that portion of the land found to be occupied by the persons and highway named was denied; and from that judgment the applicant appealed. the lands in question belong to the applicant who has shown by a strong preponderance of the evidence that he is the owner thereof. The land in question not being public and, the Government of the Philippine Islands had no authority to declare it open for homesteads; and as a necessary

consequence, whatever concessions the Government has made with respect to such land are without force and effect, except as to the homestead of Apolonio Gamido who, prior to the commencement of this proceeding, appears to have received his homestead patent from the Government. Under Act No. 926 a patent issued under the Homestead Law has all the force and effect of a Torrens title acquired under Act No. 496; and that being the case, and no question having been raised here or in the court below as to the validity of that Act in connection with the proceedings for homesteads mentioned in this case, we must respect the title so secured, provided it be a fact that a patent has been secured in any of said homestead proceedings. IT clearly appears that the applicant and his predecessors in interest were the owners of and had a good title thereto. In our judgment the evidence falls far short of showing abandonment, the record discloses no acts of the owners on which abandonment can be cased. Nor is there any claim of title by adverse possession. ISSUE: WHETHER OR NOT THE APPLICANT HAS THE RIGHT TO REGISTER THE TITLE DESCRIBED IN THE APPLICATION? HELD: it is declared that the applicant has the right to register title to all of the lands described in the application, with the exception of that portion claimed as a homestead by Apolonio Gamido, which homestead shall be excluded from registration by the applicant provided the Court of Land Registration shall find that said Apolonio Gamido has obtained a patent for said land; but if the Court of Land Registration finds that said Gamido has not yet obtained a patent therefor, then the court shall register title in favor of the applicant to all the lands described in the application. G.R. No. 163766 June 22, 2006

REPUBLIC OF THE PHILIPPINES, Petitioner, vs.

CANDY MAKER, INC., as represented by its President, ONG YEE SEE,* Respondent DECISION Candy Maker, Inc. decided to purchase a parcel of land located below the reglementary lake elevation of 12.50 meters, about 900 meters away from the Laguna de Bay, The Community Environment and Natural Resources Officer (CENRO) of Antipolo City filed on August 18, 1999 his Report8 declaring that "[t]he land falls within the Alienable and Disposable Zone, On the other hand, the LRA, in its September 21, 1999 Report,9 recommended the exclusion of Lot No. 3138-B on the ground that it is a legal easement and intended for public use, hence, inalienable and indisposable. On September 30, 1999, the Laguna Lake Development Authority (LLDA) approved Resolution No. 113, Series of 1993, providing that untitled shoreland areas may be leased subject to conditions enumerated therein. The applicant filed its Amended Application10 on December 15, 1999 for the confirmation of its alleged title on Lot No. 3138, On July 20, 2001, the Republic of the Philippines, the LLDA filed its Opposition17 to the Amended Application in which it alleged that the lot subject of the application for registration may not be alienated and disposed since it is considered part of the Laguna Lake bed, a public land within its jurisdiction pursuant to Republic Act (R.A.) No. 4850, as amended. the municipal court hereby rendered judgment confirming title of the applicants over the real property On appeal to the CA, the petitioner contended that the MTC did not acquire jurisdiction over the application for registration since the actual copies of the Official Gazette (O.G.) where the notice of hearing was published were not adduced in evidence; the applicant likewise failed to

establish exclusive ownership over the subject property in the manner prescribed by law. The applicant averred in its Appellees Brief54 that Sec. 14, par. 1 of P.D. 1529 is inapplicable since it speaks of possession and occupation of alienable and disposable lands of the public domain. Instead, par. 4 of the same section55 should govern because the subject parcels of land are lands of private ownership, having being acquired through purchase from its predecessors-in-interest, who, in turn, inherited the same from their parents. the appellate court rendered judgment which dismissed the appeal and affirmed in toto the Decision of the MTC,56: The issues in this case are the following: (2) whether the property subject of the amended application is alienable and disposable property of the State, and, if so, (3) whether respondent adduced the requisite quantum of evidence to prove its ownership over the property under Section 14 of P.D. 1529. The petition is meritorious. On the second and third issues, we find and so rule that the property subject of this application was alienable and disposable public agricultural land until July 18, 1966. However, respondent failed to prove that it possesses registerable title over the property. the petition is GRANTED. The decision of the Court of Appeals in CAG.R. CV No. 73278 is SET ASIDE. The Municipal Trial Court of Taytay, Rizal is DIRECTED to dismiss the application for registration of respondent Candymaker, Inc. in Land Registration Case No. 99-0031. Usero v Court of Appeals, G.R. No. 152115, 26 January 2005 Corona (J.) Facts of the Case:

This is a consolidated petition assailing the decision of the Court of Appeals (CA). Petitioners and the private respondent are registered owners of neighboring parcels of land wherein between the lots is a lowlevel strip of land with stagnant body of water. Whenever there is a storm or heavy rain, the water therein would flood thereby causing damage to houses of the Polinars prompting them to build a concrete wall on the bank of the strip of land about 3meters from their house and riprapped the soil in that portion. The Useros claimed ownership of the strip, demanded the halt of the construction but the Polinars never heeded believing that the strip is part of a creek. However, the Polinars offered to pay for the land. As the parties still failed to settle, both filed separate complaints for forcible entry. The Municipal Trial Court ruled in favor of the petitioner, while the regional trial court reversed and ordered the dismissal of the complaint and confirmed the existence of the creek between the lots. Issue: Whether or not the disputed strip of land is part of the creek hence part of public domain Ruling: Art. 420 of the Philippine New Civil Code provides for properties which are part of public domain. A creek is included in the phrase "and others of similar character". A creek, which refers to a recess or arm of a river is a property belonging to the public domain, therefore not susceptible of private ownership. Being a public water, it cannot be registered under the Torrens system under the name of any individual. JOSE MENCHAVEZ, JUAN MENCHAVEZ JR., SIMEON MENCHAVEZ, RODOLFO MENCHAVEZ, CESAR MENCHAVEZ, REYNALDO, MENCHAVEZ, ALMA MENCHAVEZ, ELMA MENCHAVEZ, CHARITO M. MAGA, FE M. POTOT, THELMA M. REROMA, MYRNA M. YBAEZ, and SARAH M. VILLABER, petitioners, vs. FLORENTINO TEVES JR., respondent.

DECISION PANGANIBAN, J.: A void contract is deemed legally nonexistent. It produces no legal effect. As a general rule, courts leave parties to such a contract as they are, because they are in pari delicto or equally at fault. Neither party is entitled to legal protection. The Case Before us is a Petition for Review[1] under Rule 45 of the Rules of Court, assailing the February 28, 2001 Decision[2] and the April 16, 2002 Resolution[3] of the Court of Appeals (CA) in CA-GR CV No. 51144. The challenged Decision disposed as follows: WHEREFORE, the assailed MODIFIED, as follows: decision is hereby

1. Ordering [petitioners] to jointly and severally pay the [respondent] the amount of P128,074.40 as actual damages, and P50,000.00 as liquidated damages; 2. Dismissing the third party complaint against the third party defendants; 3. Upholding the counterclaims of the third party defendants against the [petitioners. Petitioners] are hereby required to pay [the] third party defendants the sum of P30,000.00 as moral damages for the clearly unfounded suit;

4. Requiring the [petitioners] to reimburse the third party defendants the sum of P10,000.00 in the concept of attorneys fees and appearance fees of P300.00 per appearance; 5. Requiring the [petitioners] to reimburse the third party defendants the sum of P10,000.00 as exemplary damages pro bono publico and litigation expenses including costs, in the sum of P5,000.00.[4] The assailed Resolution denied petitioners Motion for Reconsideration. The Facts On February 28, 1986, a Contract of Lease was executed by Jose S. Menchavez, Juan S. Menchavez Sr., Juan S. Menchavez Jr., Rodolfo Menchavez, Simeon Menchavez, Reynaldo Menchavez, Cesar Menchavez, Charito M. Maga, Fe M. Potot, Thelma R. Reroma, Myrna Ybaez, Sonia S. Menchavez, Sarah Villaver, Alma S. Menchavez, and Elma S. Menchavez, as lessors; and Florentino Teves Jr. as lessee Of an area covered by FISHPOND APPLICATION No. VI-1076 of Juan Menchavez, Sr., covering an area of 10.0 hectares more or less located at Tabuelan, Cebu; On June 2, 1988, Cebu RTC Sheriffs Gumersindo Gimenez and Arturo Cabigon demolished the fishpond dikes constructed by respondent and delivered possession of the subject property to other parties.[6] As a result, he filed a Complaint for damages with application for preliminary attachment against

petitioners. In his Complaint, he alleged that the lessors had violated their Contract of Lease, As a consequence of these provisions, and the declared public policy of the State under the Regalian Doctrine, the lease contract between Florentino Teves, Jr. and Juan Menchavez Sr. and his family is a patent nullity. Being a patent nullity, [petitioners] could not give any rights to Florentino Teves, Jr. under the principle: NEMO DAT QUOD NON HABET - meaning ONE CANNOT GIVE WHAT HE DOES NOT HAVE, considering that this property in litigation belongs to the State and not to [petitioners]. Therefore, the first issue is resolved in the negative, as the court declares the contract of lease as invalid and void ab-initio. On the issue of whether [respondent] and [petitioners] are guilty of mutual fraud, the court rules that the [respondent] and [petitioners] are in pari-delicto. As a consequence of this, the court must leave them where they are found. x x x. xxx xxx xxx

x x x. Why? Because the defendants ought to have known that they cannot lease what does not belong to them for as a matter of fact, they themselves are still applying for a lease of the same property under litigation from the government. On the other hand, Florentino Teves, being fully aware that [petitioners were] not yet the owner[s], had assumed the risks and under the principle of VOLENTI NON FIT INJURIA NEQUES DOLUS - He who voluntarily

assumes a risk, does not suffer damage[s] thereby. As a consequence, when Teves leased the fishpond area from [petitioners]- who were mere holders or possessors thereof, he took the risk that it may turn out later that his application for lease may not be approved. After the court has ruled that the contract of lease is null and void ab-initio, there is no right of the [respondent] to protect and therefore[,] there is no basis for questioning the Sheriffs authority to demolish the dikes in order to restore the prevailing party, under the principle VIDETUR NEMO QUISQUAM ID CAPERE QUOD EI NECESSE EST ALII RESTITUERE - He will not be considered as using force who exercise his rights and proceeds by the force of law. The court hereby renders judgment as follows: 1. Dismissing the x x x complaint by the [respondent] against the [petitioners]; 2. Dismissing the third party complaint against the third party defendants; Respondent elevated the case to the Court of Appeals, where it was docketed as CA-GR CV No. 51144. Ruling of the Court of Appeals The CA disagreed with the RTCs finding that petitioners and respondent were in pari delicto. It contended that while there was negligence on the part of respondent for failing to verify the ownership of the subject property, there was no evidence that he had knowledge of petitioners lack of ownership.[11]

Hence, this Petition.[15] The Issues Were the Parties in Pari Delicto? The Court shall discuss the two issues simultaneously. In Pari Delicto Rule on Void Contracts The parties do not dispute the finding of the trial and the appellate courts that the Contract of Lease was void.[17] Indeed, the RTC correctly held that it was the State, not petitioners, that owned the fishpond. The 1987 Constitution specifically declares that all lands of the public domain, waters, fisheries and other natural resources belong to the State.[18] Included here are fishponds, which may not be alienated but only leased.[19] Possession thereof, no matter how long, cannot ripen into ownership.[20] Being merely applicants for the lease of the fishponds, petitioners had no transferable right over them. And even if the State were to grant their application, the law expressly disallowed sublease of the fishponds to respondent.[21] Void are all contracts in which the cause, object or purpose is contrary to law, public order or public policy.[22] Finding of In Pari Delicto: A Question of Fact Unquestionably, petitioners leased out a property that did not belong to them, one that they had no authority to sublease. The trial court correctly observed that petitioners still had a pending lease application with the

State at the time they entered into the Contract with respondent.[33] Respondent, on the other hand, claims that petitioners misled him into executing the Contract.[34] He insists that he relied on their assertions regarding their ownership of the property. His own evidence, however, rebuts his contention that he did not know that they lacked ownership. At the very least, he had notice of their doubtful ownership of the fishpond. Respondent himself admitted that he was aware that the petitioners lease application for the fishpond had not yet been approved.[35] Thus, he knowingly entered into the Contract with the risk that the application might be disapproved. Noteworthy is the fact that the existence of a fishpond lease application necessarily contradicts a claim of ownership. That respondent did not know of petitioners lack of ownership is therefore incredible. A cursory examination of the Contract suggests that it was drafted to favor the lessee. It can readily be presumed that it was he or his counsel who prepared it -a matter supported by petitioners evidence.[39] The ambiguity should therefore be resolved against him, being the one who primarily caused it.[40] The CA erred in finding that petitioners had failed to prove actual knowledge of respondent of the ownership status of the property that had been leased to him. it reveals his fault in entering into a void Contract. As both parties are equally at fault, neither may recover against the other.[42]

Since there is no contract, the injured party may only recover through other sources of obligations such as a law or a quasi-contract.[47] A party recovering through these other sources of obligations may not claim liquidated damages, which is an obligation arising from a contract. WHEREFORE, the Petition is GRANTED and the assailed Decision and Resolution SET ASIDE. The Decision of the trial court is hereby REINSTATED. No pronouncement as to costs. SO ORDERED. CHAVEZ V. PUBLIC ESTATES AUTHORITY 384 SCRA 152

FACTS: President Marcos through a presidential decree created PEA, which was tasked with the development, improvement, and acquisition, lease, and sale of all kinds of lands. The then president also transferred to PEA the foreshore and offshore lands of Manila Bay under the Manila-Cavite Coastal Road and Reclamation Project. Thereafter, PEA was granted patent to the reclaimed areas of land and then, years later, PEA entered into a JVA with AMARI for the development of the Freedom Islands. These two entered into a joint venture in the absence of any public bidding.

Later, a privilege speech was given by Senator President Maceda denouncing the JVA as the grandmother of all scams. An investigation was conducted and it was concluded that the lands that PEA was conveying to AMARI were lands of the public domain; the certificates of title over the Freedom Islands were void; and the JVA itself was illegal. This prompted Ramos to form an investigatory committee on the legality of the JVA. Petitioner now comes and contends that the government stands to lose billions by the conveyance or sale of the reclaimed areas to AMARI. He also asked for the full disclosure of the renegotiations happening between the parties.

ISSUE: W/N stipulations in the amended JVA for the transfer to AMARI of the lands, reclaimed or to be reclaimed, violate the Constitution.

HELD: The ownership of lands reclaimed from foreshore and submerged areas is rooted in the Regalian doctrine, which holds that the State owns all lands and waters of the public domain. The 1987 Constitution recognizes the Regalian doctrine. It declares that all natural resources are owned by the State and except for alienable agricultural lands of the public domain, natural resources cannot be alienated. The Amended JVA covers a reclamation area of 750 hectares. Only 157.84 hectares of the 750 hectare reclamation project have been

reclaimed, and the rest of the area are stillsubmerged areas forming part of Manila Bay. Further, it is provided that AMARI will reimburse the actual costs in reclaiming the areas of land and it will shoulder the other reclamation costs to be incurred. The foreshore and submerged areas of Manila Bay are part of the lands of the public domain, waters and other natural resources and consequently owned by the State. As such, foreshore and submerged areas shall not be alienable unless they are classified as agricultural lands of the public domain. The mere reclamation of these areas by the PEA doesnt convert these inalienable natural resources of the State into alienable and disposable lands of the public domain. There must be a law or presidential proclamation officially classifying these reclaimed lands as alienable and disposable if the law has reserved them for some public or quasi-public use. FERNANDA ARBIAS, PETITIONER, VS. THE REPUBLIC OF THE PHILIPPINES, RESPONDENT.

The factual antecedents of the case are as follows: Jardeleza) executed a Deed of Absolute Sale[5] selling to petitioner, married to Jimmy Arbias (Jimmy), a parcel of unregistered land for the sum of P33,000.00. According to the Deed, the subject property was residential and consisted of 600 square meters, more or less. petitioner filed with the RTC a verified Application for Registration of Title[6] over the subject property, the respondent Republic of the Philippines, through the Office of the Solicitor General (OSG), filed its Notice of Appearance and deputized the City Prosecutor of Iloilo City to appear on its behalf before the RTC in LRC Case No. N-1025. Thereafter, the respondent filed an Opposition

to petitioner's application for registration of the subject property.[12] the RTC ruled on petitioner's application for registration the Court of Appeals rendered the assailed Decision in which it REVERSED and SET ASIDE. Accordingly, the application for original registration of title is hereby DISMISSED.[21]

It was on the issue of possession, however, that the Court of Appeals digressed from the ruling of the RTC. The appellate court found that other than petitioner's own general statements and tax declarations, no other evidence was presented to prove her possession of the subject property for the period required by law. Likewise, petitioner failed to establish the classification of the subject property as an alienable and disposable land of the public domain. ISSUES: Petitioner ascribes error on the part of the Court of Appeals for failing to conclude that she and her predecessor-in-interest possessed the subject property in the concept of an owner for more than 30 years and that the said property had already been classified as an alienable and disposable land of the public domain. Petitioner contends that her documentary and testimonial evidence were sufficient to substantiate the said allegations, as correctly and conclusively pronounced by the RTC. Petitioner likewise points out that no third party appeared before the RTC to oppose her application and possession other than respondent. Respondent, then represented by the City Prosecutor, did not even adduce any evidence before the RTC to rebut petitioner's claims; thus, respondent, presently represented by the OSG, is now estopped from assailing the RTC Decision. Petitioner finally maintains that assuming her possession was indeed not proven under the circumstances, the Court of Appeals should have remanded the case to the trial court for further

proceedings, instead of dismissing it outright. This Court finds the petition plainly without merit. HELD: the burden of proof in overcoming the presumption of State ownership of lands of the public domain is on the person applying for registration. The applicant must show that the land subject of the application is alienable or disposable.[26] In the case at bar, petitioner miserably failed to discharge the burden of proof imposed on her by the law. First, the documentary evidence that petitioner presented before the RTC did not in any way prove the length and character of her possession and those of her predecessor-in-interest relative to the subject property. Second, neither does the evidence on record establish to our satisfaction that the subject property has been classified as alienable and disposable. petitioner failed to prove that she had an imperfect title to the subject property, which could be confirmed by registrationSO ORDERED. REPUBLIC OF THE PHILIPPINES, Petitioner, vs. CAYETANO L. SERRANO,1cralaw and HEIRS OF CATALINO M. ALAAN, represented by PAULITA P. ALAAN, Respondents. DECISION CARPIO MORALES, J.: Respondent Cayetano L. Serrano (Cayetano) filed on September 21, 1988 before the Regional Trial Court (RTC) of Butuan City an application for registration,2cralaw Cayetano claimed to have acquired the lot by inheritance from his deceased parents, Simeon Serrano (Simeon) and Agustina Luz; by virtue of a Deed of Exchange3cralaw dated February 10, 1961; and by a private deed of partition and extrajudicial settlement forged by him and his co-heirs.

Cayetano also claimed to have been in open, continuous, exclusive and notorious possession of the lot under a claim of ownership before 1917 by himself and through his deceased parentspredecessors-in-interest or for more than 70 years. The Heirs of Catalino Alaan, represented by Paulita Alaan (Paulita),5cralaw intervened and filed an application for registration,6cralaw their predecessor-in-interest Catalino Alaan (Catalino) having purchased7cralaw a 217.45-square meter undivided portion of the lot from Cayetano on February 27, 1989 during the pendency of Cayetano's application for registration. The intervenor-heirs of Catalino, also invoking the provisions of the Property Registration Decree or, alternatively, of Chapter VIII, Section 48(b) of Commonwealth Act No. 141, prayed that their application for confirmation of title be considered jointly with that of Cayetano's, and that, thereafter, original certificates of title be issued in both their names. Cayetano raised no objection or opposition to the intervenor-Heirs of Catalino's application for registration.8cralaw By Decision of November 3, 2003,27cralaw the RTC granted respondents applications, The Office of the Solicitor General, on behalf of herein petitioner, appealed the RTC decision before the Court of Appeals on the grounds that respondents failed to present evidence that the property was alienable or that they possessed the same in the manner and duration required by the provisions of the Property Registration Decree.28cralaw By Decision of May 13, 2008,29cralaw the appellate court affirmed the decision of the RTC ISSUE: that respondents failed to present evidence that the property was alienable or that they possessed the same in the manner and duration

required by the provisions of the Property Registration Decree.28cralaw raised by petitioner before the appellate court. HELD: The requisites for the filing of an application for registration of title under Section 14(1) of the Property Registration Decree are: that the property is alienable and disposable land of the public domain; that the applicants by themselves or through their predecessors-in-interest have been in open, continuous, exclusive and notorious possession and occupation thereof; and that such possession is under a bona fide claim of ownership since June 12, 1945 or earlier.31cralaw While Cayetano failed to submit any certification which would formally attest to the alienable and disposable character of the land applied for, the Certification by DENR Regional Technical Director Celso V. Loriega, Jr., as annotated on the subdivision plan submitted in evidence by Paulita, constitutes substantial compliance with the legal requirement. It clearly indicates that Lot 249 had been verified as belonging to the alienable and disposable area as early as July 18, 1925. The DENR certification enjoys the presumption of regularity absent any evidence to the contrary. It bears noting that no opposition was filed or registered by the Land Registration Authority or the DENR to contest respondents applications on the ground that their respective shares of the lot are inalienable. There being no substantive rights which stand to be prejudiced, the benefit of the Certification may thus be equitably extended in favor of respondents. Leonardo clearly established the character of the possession of Cayetano and his predecessors-in-interest over the lot. The totality of the evidence thus points to the unbroken chain of acts exercised by Cayetano to demonstrate his occupation and possession of the land in the concept of owner, to the exclusion of all others.

SO ORDERED. REPUBLIC OF THE PHILIPPINES, PETITIONER, VS. HEIRS OF JUAN FABIO, NAMELY: DOMINGA C. FABIO, SOCORRO D. FABIO, LYDIA D. FABIO, ROLANDO D. FABIO, NORMA D. FABIO, NORMA L. FABIO, ANGELITA FABIO, ROSALIE FABIO, DANILO FABIO, RENATO FABIO, LEVITA FABIO, IRENE FABIO, TERESITA MOLERA, ROSEMARIE C. PAKAY, LIGAYA C. MASANGKAY, ALFREDO F. CASTILLO, MELINDA F. CASTILLO, MERCEDITA F. CASTILLO, ESTELA DE JESUS AQUINO, FELECITO FABIO, AND ALEXANDER FABIO, REPRESENTED HEREIN BY ANGELITA F. ESTEIBAR AS THEIR ATTORNEY-IN-FACT, RESPONDENTS. DECISION CARPIO, J.: The Facts respondents, who are the heirs of Juan Fabio, , filed with the Regional Trial Court of Naic, Cavite, an application for registration of title situated in Barangay Sapang, Ternate, Cavite. The respondents sought the registration of title under the provisions of Act No. 496 or the Land Registration Act, as amended by Presidential Decree No. 1529 (PD 1529).[4] In the application, respondents alleged that they are the owners of the Lot, including all the improvements, having acquired the same through a bona fide claim of ownership. They declared that they and their predecessors-in-interest were in open, continuous, exclusive and notorious possession of the Lot in the concept of an owner for more than 100 years.[5]

On 29 September 1997, the trial court rendered a Decision ordering the registration of the Lot in the name of Juan Fabio. The Republic of the Philippines (petitioner), through the Office of the Solicitor General, filed an appeal with the Court of Appeals. Petitioner claimed that the trial court erred in ruling that respondents have acquired a vested right over the Lot which falls within the Calumpang Point Naval Reservation. the Court of Appeals affirmed the ruling of the trial court Hence, the instant petition. The Issues whether the respondents have acquired a right over the Lot. The Court's Ruling Second Issue: Validity of Respondents' Title Petitioner asserts that both the trial and appellate courts failed to recognize the import of the notation in the survey plan stating that the Lot falls within the Calumpang Point Naval Reservation. At the time the application for registration of title was filed, the Lot was no longer open to private ownership as it had been classified as a military reservation for public service. Thus, respondents are not entitled to have the Lot registered under the Torrens system. persons claiming the protection of "private rights" in order to exclude their lands from military reservations must show by clear and convincing evidence that the properties in question have been acquired by a legal method of acquiring public lands. Here, respondents failed to do so, and are thus not entitled to have the Lot registered in their names. Clearly, both the trial and appellate courts gravely erred in granting respondents' application for registration of title.

DIAZ VS REPUBLIC RESOLUTION CORONA, J .: This is a letter-motion praying for reconsideration (for the third time) of the June 16, 2008 resolution of this Court denying the petition for review filed by petitioner Florencia G. Diaz. Petitioner's late mother, Flora Garcia (Garcia), filed an application for registration of a vast tract of land1cralaw located in Laur, Nueva Ecija and Palayan City in the then Court of First Instance (CFI), Branch 1, Nueva Ecija on August 12, 1976.2cralaw She alleged that she possessed the land as owner and worked, developed and harvested the agricultural products and benefits of the same continuously, publicly and adversely for more or less 26 years. The Republic of the Philippines, represented by the Office of the Solicitor General (OSG), opposed the application because the land in question was within the Fort Magsaysay Military Reservation (FMMR), established by virtue of Proclamation No. 237 (Proclamation 237)3cralaw in 1955. Thus, it was inalienable as it formed part of the public domain. Significantly, on November 28, 1975, this Court already ruled in Director of Lands v. Reyes 4cralaw that the property subject of Garcia's application was inalienable as it formed part of a military reservation. Moreover, the existence of Possessory Information Title No. 216 (allegedly registered in the name of a certain Melecio Padilla on March 5, 1895), on which therein respondent Paraaque Investment and Development Corporation anchored its claim on the land, was not proven. Accordingly, the decree of registration issued in its favor was declared null and void.

Reyes notwithstanding, the CFI ruled in Garcia's favor in a decision5cralaw dated July 1, 1981. The Republic eventually appealed the decision of the CFI to the Court of Appeals (CA). In its decision6cralaw dated February 26, 1992, penned by Justice Vicente V. Mendoza (Mendoza decision),7cralaw the appellate court reversed and set aside the decision of the CFI. The CA found that Reyes was applicable to petitioner's case as it involved the same property. The CA observed that Garcia also traced her ownership of the land in question to Possessory Information Title No. 216. As Garcia's right to the property was largely dependent on the existence and validity of the possessory information title the probative value of which had already been passed upon by this Court in Reyes, and inasmuch as the land was situated inside a military reservation, the CA concluded that she did not validly acquire title thereto. During the pendency of the case in the CA, Garcia passed away and was substituted by her heirs, one of whom was petitioner Florencia G. Diaz.8cralaw Petitioner filed a motion for reconsideration of the Mendoza decision. While the motion was pending in the CA, petitioner also filed a motion for recall of the records from the former CFI. Without acting on the motion for reconsideration, the appellate court, with Justice Mendoza as ponente , issued a resolution9cralaw upholding petitioner's right to recall the records of the case. Subsequently, however, the CA encouraged the parties to reach an amicable settlement on the matter and even gave the parties sufficient time to draft and finalize the same. The parties ultimately entered into a compromise agreement with the Republic withdrawing its claim on the more or less 4,689 hectares supposedly outside the FMMR. For her part, petitioner withdrew her

application for the portion of the property inside the military reservation. They filed a motion for approval of the amicable settlement in the CA.10cralaw On June 30, 1999, the appellate court approved the compromise agreement.11cralaw On January 12, 2000, it directed the Land Registration Administration to issue the corresponding decree of registration in petitioner's favor.12cralaw However, acting on a letter written by a certain Atty. Restituto S. Lazaro, the OSG filed a motion for reconsideration of the CA resolution ordering the issuance of the decree of registration. The OSG informed the appellate court that the tract of land subject of the amicable settlement was still within the military reservation. On April 16, 2007, the CA issued an amended resolution (amended resolution)13cralaw annulling the compromise agreement entered into between the parties. The relevant part of the dispositive portion of the resolution read: ACCORDINGLY,the Court resolves to: (1) x x x x x x (2) x x x x x x (3) x x x x x x (4) x x x x x x (5) x x x x x x (6) REVERSEthe Resolution dated June 30, 1999 of this Court approving the Amicable Settlement dated May 18, 1999 executed between the Office of the Solicitor General and Florencia Garcia Diaz[;]

(7) ANNUL and SET ASIDEthe Amicable Settlement dated May 18, 1999 executed between the Office of the Solicitor General and Florencia Garcia Diaz; the said Amicable Settlement is hereby DECLAREDto be without force and effect; (8) GRANTthe Motion for Reconsideration filed by the Office of the Solicitor General and, consequently, SET ASIDEthe Resolution dated January 12, 2000 which ordered, among other matters, that a certificate of title be issued in the name of plaintiffappellee Florencia Garcia Diaz over the portion of the subject property in consonance with the Amicable Settlement dated May 18, 1999 approved by the Court in its Resolution dated June 30, 1999; (9) SET ASIDEthe Resolution dated June 30, 1999 approving the May 18, 1999 Amicable Settlement and the Resolution dated September 20, 1999 amending the aforesaid June 30, 1999 Resolution; and (10) REINSTATE the Decision dated February 26, 1992 dismissing applicant-appellee Diaz registration herein. SO ORDERED. (Emphasis supplied) Petitioner moved for reconsideration. For the first time, she assailed the validity of the Mendoza decision the February 26, 1992 decision adverted to in the CA's amended resolution. She alleged that Justice Mendoza was the assistant solicitor general during the initial stages of the land registration proceedings in the trial court and therefore should have inhibited himself when the case reached the CA. His failure to do so, she laments, worked an injustice against her constitutional right to due process. Thus, the Mendoza decision should be declared null and void. The motion was denied.14cralaw

Thereafter, petitioner filed a petition for review on certiorari 15cralaw in this Court. It was denied for raising factual issues.16cralaw She moved for reconsideration.17cralaw This motion was denied with finality on the ground that there was no substantial argument warranting a modification of the Court's resolution. The Court then ordered that no further pleadings would be entertained. Accordingly, we ordered entry of judgment to be made in due course.18cralaw Petitioner, however, insisted on filing a motion to lift entry of judgment and motion for leave to file a second motion for reconsideration and to refer the case to the Supreme Court en banc .19cralaw The Court denied20cralaw it considering that a second motion for reconsideration is a prohibited pleading.21cralaw Furthermore, the motion to refer the case to the banc was likewise denied as the banc is not an appellate court to which decisions or resolutions of the divisions may be appealed.22cralaw We reiterated our directive that no further pleadings would be entertained and that entry of judgment be made in due course. Not one to be easily deterred, petitioner wrote identical letters, first addressed to Justice Leonardo A. Quisumbing (then Acting Chief Justice) and then to Chief Justice Reynato S. Puno himself.23cralaw The body of the letter, undoubtedly in the nature of a third motion for reconsideration, is hereby reproduced in its entirety: This is in response to your call for "Moral Forces" in order to "redirect the destiny of our country which is suffering from moral decadence," that to your mind, is the problem which confronts us. (Inquirer, January 15, 2009, page 1)[.] I recently lost my case with the Supreme Court, G.R. N[o] . 181502, and my lawyer has done all that is humanly possible to convince the court to take a second look at the miscarriage of justice that will result from the implementation of the DISMISSAL in a MINUTE RESOLUTION of our Petition for Review.

Pending before your Division (First Division) is a last plea for justice so that the case may be elevated to the Supreme Court en banc . I hope the Court exercises utmost prudence in resolving the last plea. For ready reference, a copy of the Motion is hereto attached as Annex "A". The issue that was brought before the Honorable Supreme Court involves the Decision of then Justice Vicente Mendoza of the Court of Appeals, which is NULL and VOID, ab initio . It is null and void because destiny placed Hon. Justice Vicente Mendoza in a position in which it became possible for him to discharge the minimum requirement of due process, [ i.e .] the ability of the court to render "impartial justice," because Mr. Justice Mendoza became the ponente of the Court of Appeals Decision, reversing the findings of the trial court, notwithstanding the fact that he, as Assistant Solicitor General, was the very person who appeared on behalf of the Republic, as the oppositor in the very same land registration proceedings in which he lost. In other words, he discharged the duties of prosecutor and judge in the very same case. In the case of the "Alabang Boys[,] " the public was outraged by the actions of Atty. Verano who admitted having prepared a simple resolution to be signed by the Secretary of Justice. In my case, the act complained of is the worst kind of violation of my constitutional right. It is simply immoral, illegal and unconstitutional, for the prosecutor to eventually act as the judge, and reverse the very decision in which he had lost. If leaked to the tri-media[,] my case will certainly evoke even greater spite from the public, and put the Supreme Court in bad light. I must confess that I was tempted to pursue such course of action. I however

believe that such an action will do more harm than good, and even destroy the good name of Hon. Justice Mendoza. I fully support your call for "moral force" that will slowly and eventually lead our country to redirect its destiny and escape from this moral decadence, in which we all find ourselves. I am content with the fact that at least, the Chief Justice continues to fight the dark forces that surround us everyday. I only ask that the Supreme Court endeavor to ensure that cases such as mine do not happen again, so that the next person who seeks justice will not experience the pain and frustration that I suffered under our judicial system. Thank you, and more power to you, SIR. (Emphasis in the original). The language of petitioner's letter/motion is unmistakable. It is a thinly veiled threat precisely worded and calculated to intimidate this Court into giving in to her demands to honor an otherwise legally infirm compromise agreement, at the risk of being vilified in the media and by the public. This Court will not be cowed into submission. We deny petitioner's letter/third motion for reconsideration. APPLICABILITY OF REYES The Court agrees with the Republic's position that Reyes is applicable to this case. To constitute res judicata , the following elements must concur: (1) the former judgment or order must be final; (2) the judgment or order must be on the merits;

(3) it must have been rendered by a court having jurisdiction over the subject matter and parties; and (4) there must be between the first and second actions, identity of parties, of subject matter, and of causes of action. 24cralaw The first three requisites have undoubtedly been complied with. However, petitioner takes exception to the fourth requisite, particularly on the issue of identity of parties. In her petition for review filed in this Court, she contends that since the applicants in the two cases are different, the merits of the two cases should, accordingly, be determined independently of each other.25cralaw This contention is erroneous. The facts obtaining in this case closely resemble those in Aquino v. Director of Lands .26cralaw In that case, Quintin Taedo endeavored to secure title to a considerable tract of land by virtue of his possession thereof under CA 141. When the case eventually reached this Court, we affirmed the trial court's decision to dismiss the proceedings as the property in question was part of the public domain. Quintin's successorin-interest, Florencia Taedo, who despite knowledge of the proceedings did not participate therein, thereafter sold the same property to Benigno S. Aquino. The latter sought to have it registered in his name. The question in that case, as well as in this one, was whether our decision in the case in which another person was the applicant constituted res judicata as against his successors-in-interest. We ruled there, and we so rule now, that in registration cases filed under the provisions of the Public Land Act for the judicial confirmation of an incomplete and imperfect title, an order dismissing an application for registration and declaring the land as part of the public domain constitutes res judicata , not only against the adverse claimant, but also against allpersons.27cralaw We also declared in Aquino that:

From another point of view, the decision in the first action has become the "law of the case" or at least falls within the rule of stare decisis . That adjudication should be followed unless manifestly erroneous. It was taken and should be taken as the authoritative view of the highest tribunal in the Philippines. It is indispensable to the due administration of justice especially by a court of last resort that a question once deliberately examined and decided should be considered as settled and closed to further argument. x x x28cralaw Be that as it may, the fact is that, even before the CFI came out with its decision in favor of petitioner on July 1, 1981, this Court, in Reyes , already made an earlier ruling on November 28, 1975 that the disputed realty was inalienable as it formed part of a military reservation. Thus, petitioner's argument that the findings of fact of the trial court on her registrable title are binding on us on the principle that findings of fact of lower courts are accorded great respect and bind even this Court is untenable. Rather, it was incumbent upon the court a quo to respect this Court's ruling in Reyes , and not the other way around. However, despite having been apprised of the Court's findings in Reyes (which should have been a matter of judicial notice in the first place), the trial court still insisted on its divergent finding and disregarded the Court's decision in Reyes, declaring the subject land as forming part of a military reservation, and thus outside the commerce of man. By not applying our ruling in Reyes , the trial judge virtually nullified the decision of this Court and therefore acted with grave abuse of discretion.29cralaw Notably, a judgment rendered with grave abuse of discretion is void and does not exist in legal contemplation.30cralaw All lower courts, especially the trial court concerned in this case, ought to be reminded that it is their duty to obey the decisions of the Supreme Court. A conduct becoming of inferior courts demands a conscious awareness of the position they occupy in the interrelation and operation of our judicial system. As eloquently declared by Justice J.B. L. Reyes,

"There is only one Supreme Court from whose decision all other courts should take their bearings."31cralaw ACQUISITION OF PRIVATE RIGHTS Petitioner, however, argues that Proclamation 237 itself recognizes that its effectivity is "subject to private rights, if any there be." By way of a background, we recognized in Reyes that the property where the military reservation is situated is forest land. Thus: Before the military reservation was established, the evidence is inconclusive as to possession, for it is shown by the evidence that the land involved is largely mountainous and forested. As a matter of fact, at the time of the hearing, it was conceded thatapproximately 13,957 hectares of said land consist of public forest.x x x (Emphasis supplied)32cralaw Concomitantly, we stated therein, and we remind petitioner now, that forest lands are not registrable under CA 141. [E] ven more important, Section 48[b] of CA No. 141, as amended, applies exclusively to public agricultural land. Forest lands or area covered with forest are excluded. It is well-settled that forest land is incapable of registration; and its inclusion in a title, whether such title be one issued using the Spanish sovereignty or under the present Torrens system of registration, nullifies the title. (Emphasis supplied).33cralaw However, it is true that forest lands may be registered when they have been reclassified as alienable by the President in a clear and categorical manner (upon the recommendation of the proper department head who has the authority to classify the lands of the public domain into alienable or disposable, timber and mineral lands)34cralaw coupled with possession by the claimant as well as that of her predecessors-in-interest. Unfortunately for petitioner, she was not able to produce such evidence.

Accordingly, her occupation thereof, and that of her predecessors-ininterest, could not have ripened into ownership of the subject land. This is because prior to the conversion of forest land as alienable land, any occupation or possession thereof cannot be counted in reckoning compliance with the thirty-year possession requirement under Commonwealth Act 141 (CA 141) or the Public Land Act.35cralaw This was our ruling in Almeda v. CA .36cralaw The rules on the confirmation of imperfect titles do not apply unless and until the land classified as forest land is released through an official proclamation to that effect. Then and only then will it form part of the disposable agricultural lands of the public domain.37cralaw Coming now to petitioner's contention that her "private rights" to the property, meaning her and her predecessors possession thereof prior to the establishment of the FMMR, must be respected, the same is untenable. As earlier stated, we had already recognized the same land to be public forest even before the FMMR was established . To reiterate: Before the military reservation was established, the evidence is inconclusive as to possession, for it is shown by the evidence that the land involved is largely mountainous and forested. As a matter of fact, at the time of the hearing, it was conceded that approximately 13,957 hectares of said land consist of public forest. x x x Therefore, even if possession was for more than 30 years, it could never ripen to ownership. But even assuming that the land in question was alienable land before it was established as a military reservation, there was nevertheless still a dearth of evidence with respect to its occupation by petitioner and her predecessors-in-interest for more than 30 years. In Reyes , we noted: Evidently, Melecio Padilla, having died on February 9, 1900, barely five (5) years after the inscription of the informacion possessoria, could not have converted the same into a record of ownership twenty (20) years

after such inscription, pursuant to Article 393 of the Spanish Mortgage Law. xxx During the lifetime of Melecio Padilla, only a small portion thereof was cleared and cultivated under the kaingin system, while some portions were used as grazing land. After his death, his daughter, Maria Padilla, caused the planting of vegetables and had about forty (40) tenants for the purpose. During the Japanese occupation, Maria Padilla died. x x x xxx A mere casual cultivation of portions of the land by the claimant, and the raising thereon of cattle, do not constitute possession under claim of ownership. In that sense, possession is not exclusive and notorious as to give rise to a presumptive grant from the State. While grazing livestock over land is of course to be considered with other acts of dominion to show possession, the mere occupancy of land by grazing livestock upon it, without substantial inclosures, or other permanent improvements, is not sufficient to support a claim of title thru acquisitive prescription. The possession of public land, however long the period may have extended, never confers title thereto upon the possessor because the statute of limitations with regard to public land does not operate against the State unless the occupant can prove possession and occupation of the same under claim of ownership for the required number of years to constitute a grant from the State.38cralaw xxx Furthermore, the fact that the possessory information title on which petitioner also bases her claim of ownership was found to be inexistent in Reyes ,39cralaw thus rendering its probative value suspect, further militates against granting her application for registration. NULLITY OF COMPROMISE AGREEMENT

On the compromise agreement between the parties, we agree with the CA that the same was null and void. An amicable settlement or a compromise agreement is in the nature of a contract and must necessarily comply with the provisions of Article 1318 of the New Civil Code which provides: Art. 1318. There is no contract unless the following requisites concur: (1) Consent of the contracting parties; (2) Object certain which is the subject matter of the contract; (3) Cause of the obligation which is established. Petitioner was not able to provide any proof that the consent of the Republic, through the appropriate government agencies, i.e. the Department of Environment and Natural Resources, Land Management Bureau, Land Registration Authority, and the Office of the President, was secured by the OSG when it executed the agreement with her.40cralaw The lack of authority on the part of the OSG rendered the compromise agreement between the parties null and void because although it is the duty of the OSG to represent the State in cases involving land registration proceedings, it must do so only within the scope of the authority granted to it by its principal, the Republic of the Philippines.41cralaw In this case, although the OSG was authorized to appear as counsel for respondent, it was never given the specific or special authority to enter into a compromise agreement with petitioner. This is in violation of the provisions of Rule 138 Section 23, of the Rules of Court which requires "special authority" for attorneys to bind their clients. Section 23. Authority of attorneys to bind clients. Attorneys have authority to bind their clients in any case by any agreement in relation thereto made in writing, and in taking appeals, and in all matters of ordinary judicial procedure. But they cannot, without special

authority, compromise their client's litigation,or receive anything in discharge of a client's claim but the full amount in cash. (Emphasis supplied). Moreover, the land in question could not have been a valid subject matter of a contract because, being forest land, it was inalienable. Article 1347 of the Civil Code provides: Art. 1347. All things which are not outside the commerce of men, including future things, may be the object of a contract.All rights which are not intransmissible may also be the object of contracts. No contract may be entered into upon future inheritance except in cases expressly authorized by law. All services which are not contrary to law, morals, good customs, public order or public policy may likewise be the object of a contract. (Emphasis supplied) Finally, the Court finds the cause or consideration of the obligation contrary to law and against public policy. The agreement provided that, in consideration of petitioner's withdrawal of her application for registration of title from that portion of the property located within the military reservation, respondent was withdrawing its claim on that part of the land situated outside said reservation. The Republic could not validly enter into such undertaking as the subject matter of the agreement was outside the commerce of man. PETITIONERS CONTEMPT OF COURT This Court, being the very institution that dispenses justice, cannot reasonably be expected to just sit by and do nothing when it comes under attack. That petitioner's letter-motion constitutes an attack against the integrity of this Court cannot be denied. Petitioner started her letter innocently enough by stating:

This is in response to your call for "Moral Forces" in order to "redirect the destiny of our country which is suffering from moral decadence," that to your mind, is the problem which confronts us. (Inquirer, January 15, 2009, page 1)[.] It, however, quickly progressed into a barely concealed resentment for what she perceived as this Court's failure to exercise "utmost prudence" in rendering "impartial justice" in deciding her case. Petitioner recounted: I recently lost my case with the Supreme Court, G.R. N[o] . 181502, and my lawyer has done all that is humanly possible to convince the court to take a second look at the miscarriage of justice that will result from the implementation of the DISMISSAL in a MINUTE RESOLUTION of our Petitionfor Review. Pending before your Division (First Division) is a last plea for justice so that the case may be elevated to the Supreme Court en banc . I hope the Court exercises utmost prudence in resolving the last plea. For ready reference, a copy of the Motion is hereto attached as Annex "A". The issue that was brought before the Honorable Supreme Court involves the Decision of then Justice Vicente Mendoza of the Court of Appeals, which is NULL and VOID, ab initio . It is null and void because destiny placed Hon. Justice Vicente Mendoza in a position in which it became possible for him to discharge the minimum requirement of due process, [ i.e .] the ability of the court to render "impartial justice," because Mr. Justice Mendoza became the ponente of the Court of Appeals Decision, reversing the findings of the trial court, notwithstanding the fact that he, as Assistant Solicitor General, was the very person who appeared on behalf of the Republic, as the oppositor in the very same land registration proceedings in which he lost. (Emphasis supplied).

Petitioner then indirectly hints that, when push comes to shove, she has no choice but to expose the irregularity concerning the Mendoza decision to the media. This is evident in her arrogant declaration that: If leaked to the tri-media[,] my case will certainly evoke even greater spite from the public, and put the Supreme Court in bad light. But she hastens to add in the same breath that: I must confess that I was tempted to pursue such course of action. I however believe that such an action will do more harm than good, and even destroy the good name of Hon. Justice Mendoza. Petitioner ends her letter by taking this Court to task: . . . endeavor to ensure that cases such as mine do not happen again, so that the next person who seeks justice will not experience the pain and frustration that I suffered under our judicial system. When required to show cause why she should not be cited for contempt for her baseless charges and veiled threats, petitioner answered: xxx The Letter of January 26, 2009 is not a "veiled threat[.] It was written in response to the call of the Chief Justice for a moral revolution. Juxtaposed against the factual backdrop of the "Alabang Boys" case and the Meralco [c] ase, involving Mr. Justice Jose L. Sabio which also enjoyed wide publicity over the tri-media, petitioner felt that the facts of the said cases pale in comparison to the facts of her case where the lawyer of her opponent eventually became justice of the appellate court and ended up reversing the very decision in which he lost, in clear violation of her [c] onstitutional [r] ight to fundamental fair play for no contestant in any litigation can ever serve as a judge without transgression of the due process clause. This is basic.

Petitioner confesses that she may have been emotional in the delivery of her piece, because correctly or incorrectly[,] she believes they are irrefutable. If in the course of that emotional delivery, she has offended your honors sensibilities, she is ready for the punishment, and only prays that his Court temper its strike with compassion as her letter to the Chief Justice was never written with a view of threatening the Court. xxx Petitioner wrote the Chief Justice in order to obtain redress and correction of the inequity bestowed upon her by destiny. It was never meant as a threat. The Court now puts an end to petitioner's irresponsible insinuations and threats of "going public" with this case. We are not blind to petitioner's clever and foxy interplay of threats alternating with false concern for the reputation of this Court. It is well to remind petitioner that the Court has consistently rendered justice with neither fear nor favor. The disposition in this case was arrived at after a careful and thorough deliberation of the facts of this case and all the matters pertaining thereto. The records of the case, in fact, show that all the pertinent issues raised by petitioner were passed upon and sufficiently addressed by the appellate court and this Court in their respective resolutions. As to petitioner's complaint regarding this Court's denial of her petition through a mere minute resolution (which allegedly deprived her of due process as the Court did not issue a full-blown decision stating the facts and applicable jurisprudence), suffice it to say that the Court is not dutybound to issue decisions or resolutions signed by the justices all the time. It has ample discretion to formulate ponencias , extended resolutions or even minute resolutions issued by or upon its authority, depending on its evaluation of a case, as long as a legal basis exists. When a minute resolution (signed by the Clerk of Court upon orders of the Court) denies or dismisses a petition or motion for reconsideration

for lack of merit, it is understood that the assailed decision or order, together with all its findings of fact and legal conclusions, are deemed sustained.42cralaw Furthermore, petitioner has doggedly pursued her case in this Court by filing three successive motions for reconsideration, including the lettermotion subject of this resolution. This, despite our repeated warnings that "no further pleadings shall be entertained in this case." Her unreasonable persistence constitutes utter defiance of this Court's orders and an abuse of the rules of procedure. This, alongside her thinly veiled threats to leak her case to the media to gain public sympathy although the tone of petitioner's compliance with our show-cause resolution was decidedly subdued compared to her earlier letters constitutes contempt of court. In Republic v. Unimex ,43cralaw we held: A statement of this Court that no further pleadings would be entertained is a declaration that the Court has already considered all issues presented by the parties and that it has adjudicated the case with finality. It is a directive to the parties to desist from filing any further pleadings or motions. Like all orders of this Court, it must be strictly observed by the parties. It should not be circumvented by filing motions ill-disguised as requests for clarification. A FEW OBSERVATIONS If petitioner was, as she adamantly insists, only guarding her constitutional right to due process, then why did she question the validity of the Mendoza decision late in the proceedings, that is, only after her motion for reconsideration in the CA (for its subsequent annulment of the compromise agreement) was denied? It is obvious that it was only when her case became hopeless that her present counsel frantically searched for some ground, any ground to resuscitate his client's lost cause, subsequently raising the issue. This is evident from a statement in her petition to this Court that:

It is this fresh discovery by the undersigned counsel of the nullity of the proceedings of the Court of Appealsthat places in doubt the entire proceedings it previously conducted, which led to the rendition of the February 26, 1992 Decision, a fact that escaped the scrutiny of applicant for registration Flora L. Garcia, as well as her lawyer, Atty. Cayetano Dante Diaz,who died in 1993, and the late Justice Fernando A. Santiago, who stood as counsel for Flora L. Garcia's successor-in-interest, herein petitioner, Florencia G. Garcia.44cralaw (Emphasis supplied). The above cited statement does not help petitioner's cause at all. If anything, it only proves how desperate the case has become for petitioner and her counsel. WHEREFORE,the letter-motion dated January 26, 2009 of petitioner is NOTED andis hereby treated as a third motion for reconsideration. The motion is DENIED considering that a third motion for reconsideration is a prohibited pleading and the plea utterly lacks merit. Petitioner is found GUILTYof contempt of court. Accordingly, a FINEof Five Thousand Pesos is hereby imposed on her, payable within ten days from receipt of this resolution. She is hereby WARNEDthat any repetition hereof shall be dealt with more severely. Treble costs against petitioner. SO ORDERED. G.R. No. 163072 : April 2, 2009 MANILA INTERNATIONAL AIRPORT AUTHORITY, Petitioner, vs. CITY OF PASAY, SANGGUNIANG PANGLUNGSOD NG PASAY, CITY MAYOR OF PASAY, CITY TREASURER OF PASAY, and CITY ASSESSOR OF PASAY, Respondents. DISSENTING OPINION

TINGA, J.: DECISION.: DISSENTING OPINION, YNARES-SANTIAGO, J.: SEPARATE OPINION, NACHURA, J.: I maintain my dissent expressed in the 2006 ruling in MIAA v. City of Paraaque1 (the "Paraaque case.") The majority relies on two main points drawn from the 2006 Paraaque case in this instance as it rules once again that the MIAA is exempt from realty taxes assessed by the City of Pasay. First, because MIAA is a government instrumentality, it somehow finds itself exempt from the said taxes, supposedly by operation of the Local Government Code. Second, the subject properties are allegedly owned by the Republic of the Philippines, notwithstanding that legal title thereto is in the name of the MIAA, which is a distinct and independent juridical personality from the Republic. I. Once again, attempts are drawn to classify MIAA as a government instrumentality, and not as a government owned or controlled corporation. Such characterization was apparently insisted upon in order to tailor-fit the MIAA to Section 133 of the Local Government Code, which reads: Sec. 133. Common Limitations on the Taxing Powers of Local Government Units. - Unless otherwise provided herein, the exercise of the taxing powers of provinces, cities, municipalities, and barangays shall not extend to the levy of the following: xxx 15. Taxes, fees or charges of any kind on the National Government, its agencies and instrumentalities and local government units. (emphasis and underscoring supplied).

How was the Paraaque case able to define the MIAA as a instrumentality of the National Government? The case propounded that MIAA was not a GOCC: There is no dispute that a government-owned or controlled corporation is not exempt from real estate tax. However, MIAA is not a governmentowned or controlled corporation. Section 2(13) of the Introductory Provisions of the Administrative Code of 1987 defines a governmentowned or controlled corporation as follows: SEC. 2. General Terms Defined. - ... (13) Government-owned or controlled corporation refers to any agency organized as a stock or non-stock corporation, vested with functions relating to public needs whether governmental or proprietary in nature, and owned by the Government directly or through its instrumentalities either wholly, or, where applicable as in the case of stock corporations, to the extent of at least fifty-one (51) percent of its capital stock: .... (Emphasis supplied) A government-owned or controlled corporation must be "organized as a stock or non-stock corporation." MIAA is not organized as a stock or non-stock corporation. MIAA is not a stock corporation because it has no capital stock divided into shares. MIAA has no stockholders or voting shares. xxx Clearly, under its Charter, MIAA does not have capital stock that is divided into shares. Section 3 of the Corporation Code 10 defines a stock corporation as one whose "capital stock is divided into shares and . . . authorized to distribute to the holders of such shares dividends ...." MIAA has capital but it is not divided into shares of stock. MIAA has no stockholders or voting shares. Hence, MIAA is not a stock corporation.

MIAA is also not a non-stock corporation because it has no members. Section 87 of the Corporation Code defines a non-stock corporation as "one where no part of its income is distributable as dividends to its members, trustees or officers." A non-stock corporation must have members. Even if we assume that the Government is considered as the sole member of MIAA, this will not make MIAA a non-stock corporation. Non-stock corporations cannot distribute any part of their income to their members. Section 11 of the MIAA Charter mandates MIAA to remit 20% of its annual gross operating income to the National Treasury. 11 This prevents MIAA from qualifying as a non-stock corporation. Section 88 of the Corporation Code provides that non-stock corporations are "organized for charitable, religious, educational, professional, cultural, recreational, fraternal, literary, scientific, social, civil service, or similar purposes, like trade, industry, agriculture and like chambers." MIAA is not organized for any of these purposes. MIAA, a public utility, is organized to operate an international and domestic airport for public use.2 This "black or white" categorization of "stock" and "non-stock" corporations utterly disregards the fact that nothing in the Constitution prevents Congress from creating government owned or controlled corporations in whatever structure it deems necessary. Note that this definitions of "stock" and "non-stock" corporations are taken from the Administrative Code, and not the Constitution. The Administrative Code is a statute, and is thus not superior in hierarchy to any other subsequent statute created by Congress, including the charters for GOCCs. Since MIAA was presumed not to be a stock or non-stock corporation, the majority in the Paraaque case then strived to fit it into a category. Since MIAA is neither a stock nor a non-stock corporation, MIAA does not qualify as a government-owned or controlled corporation. What then is the legal status of MIAA within the National Government?

MIAA is a government instrumentality vested with corporate powers to perform efficiently its governmental functions. MIAA is like any other government instrumentality, the only difference is that MIAA is vested with corporate powers. Section 2(10) of the Introductory Provisions of the Administrative Code defines a government "instrumentality" as follows: SEC. 2. General Terms Defined. - ... (10) Instrumentality refers to any agency of the National Government, not integrated within the department framework, vested with special functions or jurisdiction by law, endowed with some if not all corporate powers, administering special funds, and enjoying operational autonomy, usually through a charter.... (Emphasis supplied) When the law vests in a government instrumentality corporate powers, the instrumentality does not become a corporation. Unless the government instrumentality is organized as a stock or non-stock corporation, it remains a government instrumentality exercising not only governmental but also corporate powers. Thus, MIAA exercises the governmental powers of eminent domain, police authority and the levying of fees and charges. At the same time, MIAA exercises "all the powers of a corporation under the Corporation Law, insofar as these powers are not inconsistent with the provisions of this Executive Order."3 Unfortunately, this cited statutory definition of an "instrumentality" is incomplete. Worse, the omitted portion from Section 2(10) completely contradicts the premise of the ponente that an instrumentality is mutually exclusive from a GOCC. For the provision reads in full, with the omitted portion highlighted, thus: (10)Instrumentality refers to any agency of the National Government not integrated within the department framework, vested with special functions or jurisdiction by law, endowed with some if not all corporate powers, administering special funds, and enjoying operational

autonomy, usually through a charter. This term includes regulatory agencies, chartered institutions and government-owned or controlled corporations. This previous omission had not escaped the attention of the outside world. For example, lawyer Gregorio Batiller, Jr., has written a paper on the Paraaque case entitled "A Tale of Two Airports," which is published on the Internet.4 He notes therein: Also of interest was the dissenting opinion of Justice Dante Tinga to the effect that the majority opinion failed to quote in full the definition of "government instrumentality:" The Majority gives the impression that a government instrumentality is a distinct concept from a government corporation. Most tellingly, the majority selectively cites a portion of Section 2(10) of the Administrative Code of 1987, as follows: Instrumentality refers to any agency of the National Government not integrated within the department framework, vested with special functions or jurisdiction by law, endowed with some if not all corporate powers, administering special funds, and enjoying operational autonomy, usually through a charter. xxx (emphasis omitted)" However, Section 2(10) of the Administrative Code, when read in full, makes an important clarification which the majority does not show. The portions omitted by the majority are highlighted below: xxx "(10)Instrumentality refers to any agency of the National Government not integrated within the department framework, vested with special functions or jurisdiction by, law endowed with some if not all corporate powers, administering special funds, and enjoying operational autonomy, usually through a charter. This term includes regulatory agencies, chartered institutions and government - owned or controlled corporations.

So the majority opinion effectively begged the question in finding that the MIAA was not a GOCC but a mere government instrumentality, which is other than a GOCC.5 The Office of the President itself was alarmed by the redefinition made by the MIAA case of instrumentalities, causing it on 29 December 2006 to issue Executive Order No. 596 creating the unwieldy category of "Government Instrumentality Vested with Corporate Powers or Government Corporate Entities" just so that it was clear that these newly defined "instrumentalities" or "government corporate entities" still fell within the jurisdiction of the Office of the Government Corporate Counsel. The E.O. reads in part: EXECUTIVE ORDER NO. 596 DEFINING AND INCLUDING "GOVERNMENT INSTRUMENTRALITY VESTED WITH CORPORATE POWERS" OR "GOVERNMENT CORPORATE ENTITIES" UNDER THE JURISDICTION OF THE OFFICE OF THE GOVERNMENT CORPORATE COUNSEL (OGCC) AS PRINCIPAL LAW OFFICE OF GOVERNMENT-OWNED OR CONTROLLED CORPORATIONS (GOCCs) AND FOR OTHER PURPOSES. WHEREAS, the Office of the Government Corporate Counsel (OGCC), as the principal law office of all Government-Owned or Controlled Corporations (GOCCs), including their subsidiaries, other corporate offsprings and government acquired assets corporations, plays a very significant role in safeguarding the legal interests and providing the legal requirements of all GOCCs; WHEREAS, there is an imperative need to integrate, strengthen and rationalize the powers and jurisdiction of the OGCC in the light of the Decision of the Supreme Court dated July 20, 2006, in the case of "Manila International Airport Authority vs. Court of Appeals, City of Paraaque, et al" (G.R. No. 155650), where the High Court differentiated "government corporate entities" and government

instrumentalities with corporate powers" from GOCCs for purposes of the provisions of the Local Government Code on real estate taxes, and other fees and charges imposed by local government units; WHEREAS, in the interest of an effective administration of justice, the application and definition of the term "GOCCs" need to be further clarified and rationalized to have consistency in referring to the term and to avoid unintended conflicts and/or confusion' NOW, THEREFORE, I, GLORIA MACAPAGAL-ARROYO, President of the Republic of the Philippines, by virtue of the powers vested in my by law, do hereby order: SECTION 1. The Office of the Government Corporate Counsel (OGCC) shall be the principal law office of all GOCCs, except as may otherwise be provided by their respective charter or authorized by the President, their subsidiaries, corporate offsprings, and government acquired asset corporations. The OGCC shall likewise be the principal law of the "government instrumentality vested with corporate powers" or "government corporate entity," as defined by the Supreme Court in the case of "MIAA v. Court of Appeals, City of Paraaque, et al.," supra, notable examples of which are: Manila International Airport Authority (MIAA), Mactan International Airport Authority, the Philippine Ports Authority (PPA), Philippine Deposit Insurance Corporation (PDIC), Metropolitan Water and Sewerage Services (MWSS), Philippine Rice Research Institute (PRRI), Laguna Lake Development Authority (LLDA), Fisheries Development Authority (FDA), Bases Conversion Development Authority (BCDA), Cebu Port Authority (CPA), Cagayan de Oro Port Authority, and San Fernando Port Authority. SECTION 2. As provided under PD 2029, series of 1986, the term GOCCs is defined as a stock or non-stock corporation, whether performing governmental or proprietary functions, which is directly chartered by a special law or if organized under the general corporation law, is owned or controlled by the government directly, or indirectly, through a parent corporation or subsidiary corporation, to the extent of at

least majority of its outstanding capital stock or of its outstanding voting capital stock. Under Section 2(10) of the Introductory Provisions of the Administrative Code of 1987, a government "instrumentality" refers to any agency of the National Government, not integrated within the department framework, vested with special functions or jurisdiction by law, endowed with some, if not all corporate powers, administering special funds, and enjoying operational autonomy, usually through a charter. SECTION 3. The following corporations are considered GOCCs under the conditions and/or circumstances indicated: a) A corporation organized under the general corporation law under private ownership at least a majority of the shares of stock of which were conveyed to a government financial institution, whether by foreclosure or otherwise, or a subsidiary corporation of a government corporation organized exclusively to own and manage, or lease, or operate specific assets acquired by a government financial institution in satisfaction of debts incurred therewith and which in any case by enunciated policy of the government is required to be disposed of to private ownership within a specified period of time, shall not be considered a GOCC before such disposition and even if the ownership or control thereof is subsequently transferred to another GOCC; b) A corporation created by special law which is explicitly intended under that law for ultimate transfer to private ownership under certain specified conditions shall be considered a GOCC, until it is transferred to private ownership; c) A corporation that is authorized to be established by special law, but which is still required under that law to register with the Securities and Exchange Commission in order to acquire a juridical personality, shall not, on the basis of the special law alone, be considered a GOCC.

xxx Reading this Executive Order, one cannot help but get the impression that the Republic of the Philippines, ostensibly the victorious party in the Paraaque case, felt that the 2006 ponencia redefining "instrumentalities" was wrong. Ostensibly, the Office of the Government Corporate Counsel, the winning counsel in the MIAA case, cooperated in the drafting of this E.O. and probably also felt that the redefinition of "instrumentalities" was wrong. I had pointed out in my Dissent to the MIAA case that under the framework propounded in that case, GOCCs such as the Philippine Ports Authority, the Bases Conversion Development Authority, the Philippine Economic Zone Authority, the Light Rail Transit Authority, the Bangko Sentral ng Pilipinas, the National Power Corporation, the Lung Center of the Philippines, and even the Philippine Institute of Traditional and Alternative Health Care have been reclassified as instrumentalities instead of GOCCs. Notably, GOCCs are mandated by Republic Act No. 7656 to remit 50% of their annual net earnings as cash, stock or property dividends to the National Government. By denying categorization of those abovementioned corporations as GOCCs, the Court in MIAA effectively gave its imprimatur to those entities to withhold remitting 50% of their annual net earnings to the National Government. Hence, the necessity of E.O. No. 596 to undo the destructive effects of the Paraaque case on the national coffers. In a welcome development, the majority now acknowledges the existence of that second clause in Section 2(10) of the Introductory Provisions of the Administrative Code, the clause which made explicit that government instrumentalities include GOCCs. In truth, I had never quite understood this hesitation in plainly saying that GOCCs are instrumentalities. That fact is really of little consequence in determining whether or not the MIAA or other government instrumentalities or GOCCs are exempt from real property taxes.

As I had consistently explained, the liability of such entities is mandated by Section 232, in relation with Section 234 of the Local Government Code. Section 232 lays down the general rule that provinces, cities or municipalities within Metro Manila may levy an ad valorem tax on real property "not hereinafter specifically exempted." Such specific exemptions are enumerated in Section 234, and the only exemption tied to government properties extends to "real property owned by the Republic of the Philippines or any of its political subdivisions except when the beneficial use thereof has been granted.to a taxable person."6 Moreover, the final paragraph of Section 234 explains that "[e]xcept as provided herein [in Section 234], any exemption from payment of real property tax previously granted to, or presently enjoyed by all persons, whether natural or juridical, including all government-owned or controlled corporations are hereby withdrawn upon the effectivity of this Code." What are the implications of Section 232 in relation to Section 234 as to the liability for real property taxes of government instrumentalities such as MIAA? 1) All persons, whether natural or juridical, including GOCCs are liable for real property taxes. 2) The only exempt properties are those owned by the Republic or any of its political subdivisions. 3) So-called "government corporate entities," so long as they have juridical personality distinct from the Republic of the Philippines or any of its political subdivisions, are liable for real property taxes. 4) After the enactment of the Local Government Code in 1991, Congress remained free to reenact tax exemptions from real property taxes to government instrumentalities, as it did with the Government Service Insurance System in 1997.

It is that simple. The most honest intellectual argument favoring the exemption of the MIAA from real property taxes corresponds with the issue of whether its properties may be deemed as "owned by the Republic or any of its political subdivisions". The matter of whether MIAA is a GOCC or an instrumentality or a "government corporate entity" should in fact be irrelevant. However, the framework established by the ponente beginning with the Paraaque case has inexplicably and unnecessarily included the question of what is a GOCC? That issue, utterly irrelevant to settling the question of MIAA's tax liability, has caused nothing but distraction and confusion. It should be remembered that prior to the Paraaque case, the prevailing rule on taxation of GOCCs was as enunciated in Mactan Cebu International Airport v. Hon. Marcos.7 That rule was a highly sensible rule that gave due respect to national government prerogatives and the devolution of taxing powers to local governments. Neither did Mactan Cebu prevent Congress from enacting legislation exempting selected GOCCs to be exempt from real property taxes. A significant portion of my Dissenting Opinion in the Paraaque case was devoted to explaining Mactan Cebu, and criticizing the ponencia for implicitly rejecting that doctrine without categorically saying so. In the years since, significant confusion has arisen on whether Mactan Cebu and the framework it established in real property taxation of GOCCs and instrumentalities, remains extant. Batiller makes the same point in his paper, expressly asking why "the Supreme Court did not explicitly declare that the Mactan Cebu International Airport case was deemed repealed." He added: Inevitably, the refusal of the Supreme Court to clarify whether its Decision in the Mactan Cebu International Airport case is deemed repealed would leave us with an ambiguous situation where two (2) of our major international airports are treated differently tax wise: one in Cebu which is deemed to be a GOCC subject to real estate taxes and the other in Manila which is not a GOCC and exempt from real estate taxes.

Where lies the substantial difference between the two (2) airports? Your guess is as good as mine.8 There are no good reasons why the Court should not reassert the Mactan Cebu doctrine. Under that ruling, real properties owned by the Republic of the Philippines or any of its political subdivisions are exempted from the payment of real property taxes, while instrumentalities or GOCCs are generally exempted from local government taxes, save for real property taxes. At the same time, Congress is free should it so desire to exempt particular GOCCs or instrumentalities from real property taxes by enacting legislation for that purpose. This paradigm is eminently more sober than that created by the Paraaque case, which attempted to amend the Constitution by elevating as a constitutional principle, the real property tax exemption of all government instrumentalities, most of which also happen to be GOCCs. Considering that the Constitution itself is supremely deferential to the notion of local government rule and the power of local governments to generate revenue through local taxes, the idea that not even the local government code could subject such "instrumentalities" to local taxes is plainly absurd. II. I do recognize that the present majority opinion has chosen to lay equal, if not greater emphasis on the premise that the MIAA properties are supposedly of public dominion, and as such are exempt from realty taxes under Section 234(a) of the Local Government Code. Again, I respectfully disagree. It is Article 420 of the Civil Code which defines what are properties of public dominion. I do not doubt that Article 420 can be interpreted in such a way that airport properties, such as its runways, hangars and the like, can be considered akin to ports or roads, both of which are among those properties considered as part of the public dominion under Article 420(1). It may likewise be possible that those properties considered as "property of public dominion" under Article 420 of the Civil Code are

also "property owned by the Republic," which under Section 234 of the Local Government Code, are exempt from real property taxes. The necessary question to ask is whether properties which are similar in character to those enumerated under Article 420(1) may be considered still part of the public dominion if, by virtue of statute, ownership thereof is vested in a GOCC which has independent juridical personality from the Republic of the Philippines. The question becomes even more complex if, as in the case of MIAA, the law itself authorizes such GOCC to sell the properties in question. One of the most recognizable characteristics of public dominion properties is that they are placed outside the commerce of man and cannot be alienated or leased or otherwise be the subject matter of contracts.9 The fact is that the MIAA may, by law, alienate, lease or place the airport properties as the subject matter of contracts. The following provisions of the MIAA charter make that clear: SECTION 5. Functions, Powers, and Duties. - The Authority shall have the following functions, powers and duties: xxx (i) To acquire, purchase, own, administer, lease, mortgage, sell or otherwise dispose of any land, building, airport facility, or property of whatever kind and nature, whether movable or immovable, or any interest therein; xxx SECTION 16. Borrowing Power. - The Authority may, after consultation with the Minister of Finance and with the approval of the President of the Philippines, as recommended by the Minister of Transportation and Communications, raise funds, either from local or international sources, by way of loans, credits or securities, and other

borrowing instruments, with the power to create pledges, mortgages and other voluntary liens or encumbrances on any of its assets or properties. There is thus that contradiction where property which ostensibly is classified as part of the public dominion under Article 420 of the Civil Code is nonetheless classified to lie within the commerce of man by virtue of a subsequent law such as the MIAA charter. In order for the Court to classify the MIAA properties as part of public dominion, it will be necessary to invalidate the provisions of the MIAA charter allowing the Authority to lease, sell, create pledges, mortgages and other voluntary liens or encumbrances on any of the airport properties. The provisions of the MIAA charter could not very well be invalidated with the Civil Code as basis, since the MIAA charter and the Civil Code are both statutes, and thus of equal rank in the hierarchy of laws, and more significantly the Civil Code was enacted earlier and therefore could not be the repealing law. If there is a provision in the Constitution that adopted the definition of and limitations on public dominion properties as found in the Civil Code, then the aforequoted provisions from the MIAA charter allowing the Authority to place its properties within the commerce of man may be invalidated. The Constitution however does not do so, confining itself instead to a general statement that "all lands of the public domain, waters, minerals, coal, petroleum, and other mineral oils, all forces of potential energy, fisheries, forests or timber, wildlife, flora and fauna, and other natural resources are owned by the State." Note though that under Article 420, public dominion properties are not necessarily owned by the State, the two subsections thereto referring to (a) properties intended for public use; and (b) those which belong to the State and are intended for some public service or for the development of the national wealth.10 In Laurel v. Garcia,11 the Court notably acknowledged that "property of public dominion is not owned by the State but pertains to the State." Thus,

there is no equivalence between the concept of public dominion under the Civil Code, and of public domain under the Constitution. Accordingly, the framework of public dominion properties is one that is statutory, rather than constitutional in design. That being the case, Congress is able by law to segregate properties which ostensibly are, by their nature, part of the public dominion under Article 420(1) of the Civil Code, and place them within the commerce of man by vesting title thereto in an independent juridical personality such as the MIAA, and authorizing their sale, lease, mortgage and other similar encumbrances. When Congress accomplishes that by law, the properties could no longer be considered as part of the public dominion. This point has been recognized by previous jurisprudence which I had cited in my dissent in the Paraaque case. For example, in Philippine Ports Authority v. City of Iloilo, the Court stated that "properties of public dominion are owned by the general public and cannot be declared to be owned by a public corporation, such as [the Philippine Ports Authority]."12 I had likewise previously explained: The second Public Ports Authority case, penned by Justice Callejo, likewise lays down useful doctrines in this regard. The Court refuted the claim that the properties of the PPA were owned by the Republic of the Philippines, noting that PPA's charter expressly transferred ownership over these properties to the PPA, a situation which similarly obtains with MIAA. The Court even went as far as saying that the fact that the PPA "had not been issued any torrens title over the port and port facilities and appurtenances is of no legal consequence. A torrens title does not, by itself, vest ownership; it is merely an evidence of title over properties.... It has never been recognized as a mode of acquiring ownership over real properties." The Court further added: . . . The bare fact that the port and its facilities and appurtenances are accessible to the general public does not exempt it from the payment of

real property taxes. It must be stressed that the said port facilities and appurtenances are the petitioner's corporate patrimonial properties, not for public use, and that the operation of the port and its facilities and the administration of its buildings are in the nature of ordinary business. The petitioner is clothed, under P.D. No. 857, with corporate status and corporate powers in the furtherance of its proprietary interests . . . The petitioner is even empowered to invest its funds in such government securities approved by the Board of Directors, and derives its income from rates, charges or fees for the use by vessels of the port premises, appliances or equipment.... Clearly then, the petitioner is a profit-earning corporation; hence, its patrimonial properties are subject to tax. There is no doubt that the properties of the MIAA, as with the PPA, are in a sense, for public use. A similar argument was propounded by the Light Rail Transit Authority in Light Rail Transit Authority v. Central Board of Assessment, 118 which was cited in Philippine Ports Authority and deserves renewed emphasis. The Light Rail Transit Authority (LRTA), a body corporate, "provides valuable transportation facilities to the paying public." 119 It claimed that its carriage-ways and terminal stations are immovably attached to government-owned national roads, and to impose real property taxes thereupon would be to impose taxes on public roads. This view did not persuade the Court, whose decision was penned by Justice (now Chief Justice) Panganiban. It was noted: Though the creation of the LRTA was impelled by public service - to provide mass transportation to alleviate the traffic and transportation situation in Metro Manila - its operation undeniably partakes of ordinary business. Petitioner is clothed with corporate status and corporate powers in the furtherance of its proprietary objectives. Indeed, it operates much like any private corporation engaged in the mass transport industry. Given that it is engaged in a service-oriented commercial endeavor, its carriageways and terminal stations are

patrimonial property subject to tax, notwithstanding its claim of being a government-owned or controlled corporation. xxx Petitioner argues that it merely operates and maintains the LRT system, and that the actual users of the carriageways and terminal stations are the commuting public. It adds that the public use character of the LRT is not negated by the fact that revenue is obtained from the latter's operations. We do not agree. Unlike public roads which are open for use by everyone, the LRT is accessible only to those who pay the required fare. It is thus apparent that petitioner does not exist solely for public service, and that the LRT carriageways and terminal stations are not exclusively for public use. Although petitioner is a public utility, it is nonetheless profit-earning. It actually uses those carriageways and terminal stations in its public utility business and earns money therefrom. xxx Even granting that the national government indeed owns the carriageways and terminal stations, the exemption would not apply because their beneficial use has been granted to petitioner, a taxable entity. There is no substantial distinction between the properties held by the PPA, the LRTA, and the MIAA. These three entities are in the business of operating facilities that promote public transportation. The majority further asserts that MIAA's properties, being part of the public dominion, are outside the commerce of man. But if this is so, then why does Section 3 of MIAA's charter authorize the President of the Philippines to approve the sale of any of these properties? In fact, why does MIAA's charter in the first place authorize the transfer of these airport properties, assuming that indeed these are beyond the commerce of man?13

III. In the present case, the City of Pasay had issued notices of levy and warrants of levy for the NAIA Pasay properties, leading MIAA to file with the Court of Appeals a petition for prohibition and injunction, seeking to enjoin the City of Pasay from imposing real property taxes, levying against and auctioning for public sale the NAIA Pasay properties. In the Paraaque case, I had expressed that while MIAA was liable for the realty taxes, its properties could not be foreclosed upon by the local government unit seeking the taxes. I explained then: Despite the fact that the City of Paraaque ineluctably has the power to impose real property taxes over the MIAA, there is an equally relevant statutory limitation on this power that must be fully upheld. Section 3 of the MIAA charter states that "[a]ny portion [of the [lands transferred, conveyed and assigned to the ownership and administration of the MIAA] shall not be disposed through sale or through any other mode unless specifically approved by the President of the Philippines." Nothing in the Local Government Code, even with its wide grant of powers to LGUs, can be deemed as repealing this prohibition under Section 3, even if it effectively forecloses one possible remedy of the LGU in the collection of delinquent real property taxes. While the Local Government Code withdrew all previous local tax exemptions of the MIAA and other natural and juridical persons, it did not similarly withdraw any previously enacted prohibitions on properties owned by GOCCs, agencies or instrumentalities. Moreover, the resulting legal effect, subjecting on one hand the MIAA to local taxes but on the other hand shielding its properties from any form of sale or disposition, is not contradictory or paradoxical, onerous as its effect may be on the LGU. It simply means that the LGU has to find another way to collect the taxes due from MIAA, thus paving the way for a mutually acceptable negotiated solution.

Accordingly, I believe that MIAA is entitled to a writ of prohibition and injunctive relief enjoining the City of Pasay from auctioning for public sale the NAIA Pasay properties. Thus, the Court of Appeals erred when it denied those reliefs to the MIAA. I VOTE to PARTIALLY GRANT the petition and to issue the Writ of Prohibition insofar as it would enjoin the City of Pasay from auctioning for public sale the NAIA Pasay properties. In all other respects, I respectfully dissent. DANTE O. TINGA Associate Justice MIAA v. Court of Appeals G.R. No. 155650, July 20, 2006 Facts: The Manila International Airport Authority (MIAA) operates the Ninoy Aquino International Airport (NAIA) Complex in Paraaque City under Executive Order No. 903 (MIAA Charter), as amended. As such operator, it administers the land, improvements and equipment within the NAIA Complex. In March 1997, the Office of the Government Corporate Counsel (OGCC) issued Opinion No. 061 to the effect that the Local Government Code of 1991 (LGC) withdrew the exemption from real estate tax granted to MIAA under Section 21 of its Charter. Thus, MIAA paid some of the real estate tax already due. In June 2001, it received Final Notices of Real Estate Tax Delinquency from the City of Paraaque for the taxable years 1992 to 2001. The City Treasurer subsequently issued notices of levy and warrants of levy on the airport lands and buildings. At the instance of MIAA, the OGCC issued Opinion No. 147 clarifying Opinion No. 061, pointing out that Sec. 206 of the LGC requires persons

exempt from real estate tax to show proof of exemption. According to the OGCC, Sec. 21 of the MIAA Charter is the proof that MIAA is exempt from real estate tax. MIAA, thus, filed a petition with the Court of Appeals seeking to restrain the City of Paraaque from imposing real estate tax on, levying against, and auctioning for public sale the airport lands and buildings, but this was dismissed for having been filed out of time. Hence, MIAA filed this petition for review, pointing out that it is exempt from real estate tax under Sec. 21 of its charter and Sec. 234 of the LGC. It invokes the principle that the government cannot tax itself as a justification for exemption, since the airport lands and buildings, being devoted to public use and public service, are owned by the Republic of the Philippines. On the other hand, the City of Paraaque invokes Sec. 193 of the LGC, which expressly withdrew the tax exemption privileges of government-owned and controlled corporations (GOCC) upon the effectivity of the LGC. It asserts that an international airport is not among the exceptions mentioned in the said law. Meanwhile, the City of Paraaque posted and published notices announcing the public auction sale of the airport lands and buildings. In the afternoon before the scheduled public auction, MIAA applied with the Court for the issuance of a TRO to restrain the auction sale. The Court issued a TRO on the day of the auction sale, however, the same was received only by the City of Paraaque three hours after the sale. Issue: Whether or not the airport lands and buildings of MIAA are exempt from real estate tax? Held: The Petition is GRANTED.

The airport lands and buildings of MIAA are exempt from real estate tax imposed by local governments. Sec. 243(a) of the LGC exempts from real estate tax any real property owned by the Republic of the Philippines. This exemption should be read in relation with Sec. 133(o) of the LGC, which provides that the exercise of the taxing powers of local governments shall not extend to the levy of taxes, fees or charges of any kind on the National Government, its agencies and instrumentalities. These provisions recognize the basic principle that local governments cannot tax the national government, which historically merely delegated to local governments the power to tax. The rule is that a tax is never presumed and there must be clear language in the law imposing the tax. This rule applies with greater force when local governments seek to tax national government instrumentalities. Moreover, a tax exemption is construed liberally in favor of national government instrumentalities. MIAA is not a GOCC, but an instrumentality of the government. The Republic remains the beneficial owner of the properties. MIAA itself is owned solely by the Republic. At any time, the President can transfer back to the Republic title to the airport lands and buildings without the Republic paying MIAA any consideration. As long as the airport lands and buildings are reserved for public use, their ownership remains with the State. Unless the President issues a proclamation withdrawing these properties from public use, they remain properties of public dominion. As such, they are inalienable, hence, they are not subject to levy on execution or foreclosure sale, and they are exempt from real estate tax. However, portions of the airport lands and buildings that MIAA leases

to private entities are not exempt from real estate tax. In such a case, MIAA has granted the beneficial use of such portions for a consideration to a taxable person. G.R. No. 150301 October 2, 2007

PHILIPPINE FISHERIES DEVELOPMENT AUTHORITY, petitioner, vs. THE HONORABLE COURT OF APPEALS, THE HONORABLE REGIONAL TRIAL COURT, BRANCH 169, MALABON, METRO MANILA, THE MUNICIPALITY OF NAVOTAS, METRO MANILA, HON. FLORANTE M. BARREDO, in his official capacity as Municipal Treasurer of Navotas, Metro Manila, and HON. NORBERTO E. AZARCON, in his capacity as Chairman of the Public Auction Sale Committee of Navotas, Metro Manila, respondent. DECISION AZCUNA, J.: This is a petition for review1 of the decision and resolution of the Court of Appeals (CA), dated July 19, 2001 and September 19, 2001, respectively, in CA-G.R. CV No. 42472, entitled "Philippine Fisheries Development Authority v. The Municipality of Navotas, Metro Manila, et al." The facts appear as follows: The controversy arose when respondent Municipality of Navotas assessed the real estate taxes allegedly due from petitioner Philippine Fisheries Development Authority (PFDA) for the period 1981-1990 on properties under its jurisdiction, management and operation located inside the Navotas Fishing Port Complex (NFPC).

The assessed taxes had remained unpaid despite the demands made by the municipality which prompted it, through Municipal Treasurer Florante M. Barredo, to give notice to petitioner on October 29, 1990 that the NFPC will be sold at public auction on November 30, 1990 in order that the municipality will be able to collect on petitioners delinquent realty taxes which, as of June 30, 1990, amounted to P23,128,304.51, inclusive of penalties. Petitioner sought the deferment of the auction sale claiming that the NFPC is owned by the Republic of the Philippines, and pursuant to Presidential Decree (P.D.) No. 977, it (PFDA) is not a taxable entity. In view of the refusal of PFDA to pay the assessed realty taxes, the matter was referred to the Department of Finance (DOF). On July 14, 1990 the DOF stated that: This Department takes cognizance of the allegations of [the Office of the Mayor of Navotas] that PFDA has leased its properties to beneficial users, such as "businessmen, private persons and entities who are taxable persons." For this reason, it is imperative that the Municipality should conduct an ocular inspection on the real properties (land and building owned by PFDA) in order to identify the properties actually leased and the taxable persons enjoying the beneficial use thereof. The ocular inspection is necessary for reason that the real properties, the use of which has been granted to taxable persons, for consideration or otherwise, are subject to the payment of real property taxes which must be paid by the grantees pursuant to the provisions of the Real Property Tax Code, as amended. ... Therefore, it is imperative to determine who the actual users of the properties concerned [are]. If used by a non-taxable person other than PFDA itself, it remains to be non-taxable. Otherwise, if said properties are being used by taxable persons, same becomes taxable properties. For this purpose, it is also incumbent upon PFDA to furnish the Municipality copies of the deed of lease or

other relevant documents showing the leased properties and their beneficial users for proper assessment.2 Notwithstanding the DOFs instruction, respondent Municipality proceeded to publish the notice of sale of NFPC in the November 2, 1990 issue of Balita, a local newspaper. On November 19, 1990, petitioner instituted Civil Case No. 1524 in the Regional Trial Court (RTC) of Malabon, Metro Manila against respondent Municipality, its Municipal Treasurer and the Chairman of the Public Auction Sale Committee. Petitioner asked the RTC to enjoin the auction of the NFPC on the ground that the properties comprising the NFPC are owned by the Republic of the Philippines and are, thus, exempt from taxation. According to petitioner, only a small portion of NFPC which had been leased to private parties may be subjected to real property tax which should be paid by the latter. Respondent Municipality, on the other hand, insisted that: 1) the real properties within NFPC are owned entirely by petitioner which, despite the opportunity given, had failed to submit proof to the Municipal Assessor that the properties are indeed owned by the Republic of the Philippines; 2) if the properties in question really belong to the government, then the complaint should have been instituted in the name of the Republic of the Philippines, represented by the Office of the Solicitor General; and 3) the complaint is fatally defective because of non-compliance with a condition precedent, which is, payment of the disputed tax assessment under protest. On December 8, 1990, the RTC issued a writ of preliminary injunction enjoining respondent Municipality from proceeding with the public auction. On February 19, 1993, however, the RTC dismissed the case and dissolved the writ of preliminary injunction, thus:

[T]he plaintiff [petitioner] failed to present convincing evidence to support its claim of realty tax exemption and ownership of the property by the Republic of the Philippines as mandated by Sec. 9 of P.D. 464. Notwithstanding receipt of the notices of tax assessments from the defendants [public respondent], the plaintiff did not avail of the remedies under the law by raising on appeal the said tax assessments to the Local Board of Assessment Appeals, then to the Central Board of Assessment Appeals and ultimately, to the Court of Tax Appeals. Instead, the plaintiff continuously ignored the notices of tax assessments on the pretext that the properties inside the NFPC are exempt from payment of real estate taxes as they are owned by the Republic of the Philippines. Assailing the validity of the tax assessments of the NFPC properties is not the proper recourse for the plaintiff but to pay first the tax assessments under protest and then raise the same on appeal to the Local Board of Assessment Appeals, then to the Central Board of Assessment Appeals, then ultimately, to the Court of Tax Appeals pursuant to the Real Property Tax Code. The plaintiff failed in this regard, hence the Municipality, exercising its power to assess and collect taxes on real properties within its jurisdiction, did the right thing, that is, to schedule the NFPC properties for public auction. Furthermore, while the plaintiff is insisting that the NFPC properties are owned by the Republic of the Philippines, and is therefore exempt from payment of real estate taxes, yet it admitted that there are those lessees who leased portion[s] of the complex, and [it was] even willing to submit [a] list of these lessees for proper tax assessments. ... WHEREFORE, premises considered, judgment is hereby rendered in favor of the defendant [public respondent Municipality of Navotas] and against the plaintiff, ordering: 1. The DISMISSAL of this case;

2. The preliminary injunction previously issued in this case DISSOLVED; and 3. The plaintiff to pay the defendant [public respondent] Municipality the sum of P13,767.00 as actual damages. SO ORDERED.3 The CA affirmed the ruling of the RTC in a Decision dated July 19, 2001, the pertinent portions of which read: The thrust of appellant PFDAs arguments has shoved to the fore the fact that the 67-hectare land on which the NFPC Navotas Fishing Port Complex stands was reclaimed from the sea which explains why it was bounded on the North by the Manila Bay, on the East by Roxas Boulevard, on the South by the Manila Bay and on the West, by the breakwater. Even the Municipalitys counsel, Atty. Victorino Landas; Assessor, Arturo Coronel; and Treasurer, Florante Barredo have admitted that much, as pointed out by PFDA.4 Such being the origin of the land, its ownership by the State as property of public dominion5 can hardly be disputed. The "reclaimed land; breakwaters; piers; wharves and quaywalls; and, fish market building forming part of the Navotas Fish Port" were furthermore certified by the Undersecretary of Public Works and Highways6 as belonging to the national government since they were built using the proceeds of the loan agreement entered into by and between the Republic of the Philippines and the Asian Development Bank on December 12, 1971.7 On August 11, 1976, the Philippine Fish Marketing Authority (PFMA) was created as a body corporate by P.D. No. 977 to carry out ... the policy of the Government to promote the development of the fishing industry and improve efficiency in the handling,

preserving, marketing and distribution of fish and fishery/aquatic products through the establishment and operation of fish markets and the efficient operation of fishing ports harbors and other marketing facilities.8 ... The PFMA was furthermore extended exemption from the payment of income tax in this tenor: The authority shall be exempted from the payment of income tax. The foregoing exemption may, however, be entirely or partly lifted by the President of the Philippines, upon recommendation of the Secretary of Finance, not earlier than five years from the approval of this Decree, if the President shall find the authority to be selfsustaining and financially capable to pay such tax after providing for debt service requirements of the authority and its projected capital and operating expenditures.9 Meanwhile, harbor operations at the Navotas Fishing Port Complex (NFPC) commenced on January 15, 1997 while the market operation started on April 3, 1977. On February 8, 1982, P.D. No. 977 was amended by Executive Order No. 772. Insofar as material to the case at bar, the salient features of the amendments introduced by the E.O. are: (a) The creation of the Philippine Fisheries Development Authority (PFDA) to replace the Philippine Fish Marketing Authority (PFMA). ... (b) The capitalization of the PFDA has included the Navotas Fishing Port Complex (NFPC).

... (c) The NFPC has been transferred to the exclusive jurisdiction, control, administration, and supervision of the PFDA. ... There can, therefore, [be] no escaping the conclusion that the appellant PFDA became the owner of the Navotas Fishing Port Complex as of February 8, 1982. It cannot be any sooner because under P.D. No. 977, the NFPC was not made part of the capital of the Philippine Fish Marketing Authority (PFMA), PFDAs predecessor, as only the Navotas Fish Landing was made part of such capital while the Navotas Fishing Port and Fish Market were transferred merely to the "exclusive jurisdiction, control, administration, and supervision" of the PFMA. It was not then altogether clear if the Navotas Fishing Port Complex (NFPC) was conveyed to the PFMA. ... Indeed, it is quite true that a property continues to be part of the public domain, and not available for alienation, private appropriation or ownership, until it is withdrawn from being such by the Government through the Executive Department or the Legislative,10 and that it is not for the President to convey valuable real property of the Government on his own sole will as any such conveyance requires executive and legislative concurrence.11 But the stark reality is that at the time E.O. No, 772 was issued on February 8, 1982, President Marcos was exercising both executive and legislative powers.12 Hence, his conveyance of the NFPC to form part of the capital of PFDA cannot but be valid. The fact that the PFDA has up to now no certificate of title to the NFPC nor has the PFDA declared it for tax purposes is of no

consequence. Such a certificate is merely an evidence of ownership and not the title itself,13 while a tax declaration does not prove nor disprove ownership. What is significant is that the PFDA has openly declared and represented that it "owns, maintains and operates" the NFPC when it leased a portion thereof to the Frabelle Fishing Corporation on March 13, 1989. All told, the PFDA being the owner of the NFPC beginning February 8, 1982 is liable for the realty taxes due thereon, its tax exemption being only from the payment of income tax.14 WHEREFORE, the appealed decision is AFFIRMED, without pronouncement as to costs. SO ORDERED.15 Petitioner filed a motion for reconsideration but the same was denied by the CA. Petitioner now raises the following arguments: One, the CA acknowledged that the property in question is a reclaimed land. As such, it is a property of public dominion (Art. 420, Civil Code) and is owned by the State. Notwithstanding this, the CA erroneously ruled that the government had validly transferred ownership of the land to PFDA in 1982 when P.D. No. 977 was amended by E.O. No. 772 by virtue of which the property became part of the assets of PFDA (Sec. 5 of E.O. No. 772); Two, as a reclaimed land, the port complex should be considered a reserved land. In NDC v. Cebu City,16 the Supreme Court held that a reserved land is a public land that has been withheld or kept back from sale or disposition. The land remains an absolute property of the government. As its title remains with the State, the reserved land is tax exempt;

Three, in Government v. Cabangis17 and Lampria v. Director of Lands,18 this Court declared that the land reclaimed from the sea, as a result of the construction by the government of a breakwater fronting the place where it is situated, belongs to the State in accordance with Article 5 of the Law of Waters of 1866; Four, petitioner merely operates the area or the NFPC complex in favor of the Republic of the Philippines. Section 4.A of P.D. No. 977, as amended by E.O. No. 772, provides that PFDA shall: [M]anage, administer, operate, improve and modernize, coordinate and otherwise govern the activities, operation and facilities in the fishing ports, markets and landings that may hereinafter be placed under, or transferred to the Authority, and such other fish markets, fishing ports/harbors and infrastructure facilities as may be established under this Decree; to investigate, prepare, adopt, implement and execute a comprehensive plan for the overall development of fishing port and market complexes and update such plan as may be necessary from time to time; to construct or authorize the construction in the land area under its jurisdiction, of infrastructure facilities, factory buildings, warehouses, cold storage and ice plants, and other structures related to the fishing industry or necessary and useful in the conduct of its business or in the attainment of the purpose and objectives of this Decree; to acquire, hold and dispose real and personal property in the exercise of its functions and powers. Lastly, the NFPC property is intended for public use and public service. As such, it is owned by the State, hence, exempt from real property tax. The issue is whether petitioner is liable to pay real property tax. Local government units, pursuant to the fiscal autonomy granted by the provisions of Republic Act No. 7160 or the 1991 Local Government Code, can impose realty taxes on juridical persons19 subject to the limitations enumerated in Section 133 of the Code:

SEC. 133. Common Limitations on the Taxing Power of Local Government Units. Unless otherwise provided herein, the exercise of the taxing powers of provinces, cities, municipalities, and barangays shall not extend to the levy of the following: ... (o) taxes, fees, charges of any kind on the national government, its agencies and instrumentalities, and local government units. Nonetheless, the above exemption does not apply when the beneficial use of the government property has been granted to a taxable person. Section 234 (a) of the Code states that real property owned by the Republic of the Philippines or any of its political subdivisions is exempted from payment of the real property tax "except when the beneficial use thereof has been granted, for consideration or otherwise, to a taxable person." Thus, as a rule, petitioner PFDA, being an instrumentality20 of the national government, is exempt from real property tax but the exemption does not extend to the portions of the NFPC that were leased to taxable or private persons and entities for their beneficial use. This is in consonance with the ruling in Philippine Fisheries Development Authority v. Court of Appeals21 where this Court held that: On the basis of the parameters set in the MIAA [Manila International Airport Authority v. Court of Appeals]22 case, the Authority should be classified as an instrumentality of the national government. As such, it is generally exempt from payment of real property tax, except those portions which have been leased to private entities. In the MIAA case, petitioner Philippine Fisheries Development Authority was cited as among the instrumentalities of the national government 23

Indeed, the Authority is not a GOCC24 but an instrumentality of the government. The Authority has a capital stock but it is not divided into shares of stocks.25 Also, it has no stockholders or voting shares. Hence, it is not a stock corporation. Neither it is a nonstock corporation because it has no members. ... The real property tax assessments issued by the City of Iloilo should be upheld only with respect to the portions leased to private persons. In case the Authority fails to pay the real property taxes due thereon, said portions cannot be sold at public auction to satisfy the tax delinquency. ... The port built by the State in the Iloilo fishing complex is a property of public dominion and cannot therefore be sold at public auction. Article 420 of the Civil Code provides: ARTICLE 420. The following things are property of public dominion: (1) Those intended for public use, such as roads, canals, rivers, torrents, ports and bridges constructed by the State, banks, shores, roadsteads, and others of similar character; (2) Those which belong to the State, without being for public use, and are intended for some public service or for the development of national wealth. The Iloilo [F]ishing [P]ort [Complex/IFPC] which was constructed by the State for public use and/or public service falls within the term "port" in the aforecited provision. Being a property of public dominion the same cannot be subject to execution or foreclosure sale.26 Whether there are improvements in the fishing port complex that should not be construed to be embraced within the

term port involves evidentiary matters that cannot be addressed in the present case. As for now, considering that the Authority is a national government instrumentality, any doubt on whether the entire IFPC may be levied upon to satisfy the tax delinquency should be resolved against the City of Iloilo. Similarly, for the same reason, the NFPC cannot be sold at public auction in satisfaction of the tax delinquency assessments made by the Municipality of Navotas on the entire complex. Additionally, the land on which the NFPC property sits is a reclaimed land, which belongs to the State. In Chavez v. Public Estates Authority,27 the Court declared that reclaimed lands are lands of the public domain and cannot, without Congressional fiat, be subject of a sale, public or private. 28 In light of the above, petitioner is only liable to pay the amount of P62,841,947.79 representing the total taxes due as of December 31, 2001 from PFDA-owned properties that were leased, as shown in the Summary of Realty Taxes Due Properties Owned and/or Managed by PFDA as per Realty Tax Order of Payment dated September 16, 2002.29 WHEREFORE, the petition is GRANTED. The Decision and Resolution of the Court of Appeals, dated July 19, 2001 and September 19, 2001, respectively, in CA-G.R. CV No. 42472 are SET ASIDE. The Realty Tax Order of Payment issued by respondent Municipality of Navotas on September 16, 2002 is declared VOID EXCEPT as to the amount of P62,841,947.79 representing the total taxes due as of December 31, 2001 on the properties leased by petitioner to private parties. Respondent Municipality of Navotas is DIRECTED to refrain from levying on the Navotas Fishing Port Complex (NFPC) to satisfy the payment of the real property tax delinquency. No costs. SO ORDERED.

Puno, C.J., Chairperson, Sandoval-Gutierrez, Corona, Garcia, JJ., concur.

Macasiano v. Diokno [G.R. No. 97764. August 10, 1992.] En Banc, Medialdea (J): 12 concur Facts: On 13 June 1990, the Municipality of Paranaque passed Ordinance 86, s. 1990 which authorized the closure of J. Gabrielle, G.G. Cruz, Bayanihan, Lt. Garcia Extension and Opena Streets located at Baclaran, Paraaque, Metro Manila and the establishment of a flea market thereon. The said ordinance was approved by the municipal council pursuant to MCC Ordinance 2, s. 1979, authorizing and regulating the use of certain city and/or municipal streets, roads and open spaces within Metropolitan Manila as sites for flea market and/or vending areas, under certain terms and conditions. On 20 July 1990, the Metropolitan Manila Authority approved Ordinance 86, s. 1990 of the municipal council subject to conditions. On 20 June 1990, the municipal council issued a resolution authorizing the Paraaque Mayor to enter into contract with any service cooperative for the establishment, operation, maintenance and management of flea markets and/or vending areas. On 8 August 1990, the municipality and Palanyag, a service cooperative, entered into an agreement whereby the latter shall operate, maintain and manage the flea market with the obligation to remit dues to the treasury of the municipal government of Paraaque. Consequently, market stalls were put up by Palanyag on the said streets. On 13 September 1990 Brig. Gen. Macasiano, PNP Superintendent of the Metropolitan Traffic Command, ordered the destruction and confiscation of stalls along G.G. Cruz and J. Gabrielle St. in Baclaran. These stalls were later returned to Palanyag. On 16 October 1990, Macasiano wrote a letter to Palanyag giving the latter 10 days to discontinue the flea market; otherwise, the market stalls shall be dismantled.

On 23 October 1990, the municipality and Palanyag filed with the trial court a joint petition for prohibition and mandamus with damages and prayer for preliminary injunction. On 17 December 1990, the trial court issued an order upholding the validity of Ordinance 86 s. 1990 of the Municipality of Paraaque and enjoining Macasiano from enforcing his letter-order against Palanyag. Hence, a petition for certiorari under Rule 65 was filed by Macasiano thru the OSG. The Supreme Court granted the petition, and reversed and set aside the 17 December 1990 decision of the RTC which granted the writ of preliminary injunction enjoining the PNP Superintendent, Metropolitan Traffic Command from enforcing the demolition of market stalls along J. Gabrielle, G.G. Cruz, Bayanihan, Lt. Garcia Extension and Opena streets. 1. Property of provinces, cities and municipalities; Property for public use The property of provinces, cities and municipalities is divided into property for public use and patrimonial property (Art. 423, Civil Code). As to property for public use, Article 424 of Civil Code provides that "property for public use, in the provinces, cities and municipalities, consists of the provincial roads, city streets, the squares, fountains, public waters, promenades, and public works for public service paid for by said provinces, cities or municipalities. All other property possessed by any of them is patrimonial and shall be governed by this Code, without prejudice to the provisions of special laws." In the present case, thus, J. Gabrielle G.G. Cruz, Bayanihan, Lt. Gacia Extension and Opena streets are local roads used for public service and are therefore considered public properties of the municipality. 2. Properties for public service deemed public and under absolute control of Congress Properties of the local government which are devoted to public service are deemed public and are under the absolute control of Congress

(Province of Zamboanga del Norte v. City of Zamboanga, 22 SCRA 1334 [1968]). 3. Local governments have no authority to regulate use of public properties unless authority is vested upon by Congress; e.g. Closure of roads Local governments have no authority whatsoever to control or regulate the use of public properties unless specific authority is vested upon them by Congress. One such example of this authority given by Congress to the local governments is the power to close roads as provided in Section 10, Chapter II of the Local Government Code (BP 337), which states A local government unit may likewise, through its head acting pursuant to a resolution of its sangguniang and in accordance with existing law and the provisions of this Code, close any barangay, municipal, city or provincial road, street, alley, park or square. No such way or place or any part thereof shall be closed without indemnifying any person prejudiced thereby. A property thus withdrawn from public use may be used or conveyed for any purpose for which other real property belonging to the local unit concerned might be lawfully used or conveyed." 4. Legal provision should be read and interpreted in accordance with basic principles already established by law; LGU has no power to lease a road available to public and ordinarily used for vehicular traffic The legal provision (Chapter II, Section 10 of the LGC) which gives authority to local government units to close roads and other similar public places should be read and interpreted in accordance with basic principles already established by law. These basic principles have the effect of limiting such authority of the province, city or municipality to close a public street or thoroughfare. Article 424 NCC lays down the basic principle that properties of public dominion devoted to public use and made available to the public in general are outside the commerce of man and cannot be disposed of or leased by the local government unit to private persons. Aside from the requirement of due process which

should be complied with before closing a road, street or park, the closure should be for the sole purpose of withdrawing the road or other public property from public use when circumstances show that such property is no longer intended or necessary for public use or public service. When it is already withdrawn from public use, the property then becomes patrimonial property of the local government unit (LGU) (Article 422 NCC; Cebu Oxygen v. Bercilles, 66 SCRA 481 [1975]). It is only then that the LGU can "use or convey them for any purpose for which other real property belonging to the local unit concerned might be lawfully used or conveyed." However, those roads and streets which are available to the public in general and ordinarily used for vehicular traffic are still considered public property devoted to public use. In such case, the LGU has no power to use it for another purpose or to dispose of or lease it to private persons. 5. Related case, Cebu Oxygen v. Bercilles In Cebu Oxygen v. Bercilles, the City Council of Cebu, through a resolution, declared the terminal road of M. Borces Street, Mabolo, Cebu City as an abandoned road, the same not being included in the City Development Plan. Thereafter, the City Council passed another resolution authorizing the sale of the said abandoned road through public bidding. The Court held that the City of Cebu is empowered to close a city street and to vacate or withdraw the same from public use. Such withdrawn portion becomes patrimonial property which can be the object of an ordinary contract 6. Related case, Dacanay v. Asistio In Dacanay v. Asistio, the disputed areas from which the market stalls are sought to be evicted are public streets. A public street is property for public use hence outside the commerce of man (Arts. 420, 424, Civil Code). Being outside the commerce of man, it may not be the subject of lease or other contract (Villanueva, et al. v. Castaeda and Macalino, 15 SCRA 142 citing the Municipality of Cavite v. Rojas, 30 SCRA 602; Espiritu v. Municipal Council of Pozorrubio, 102 Phil. 869; and Muyot v. De la Fuente, 48 O.G. 4860). The right of the public to use the city

streets may not be bargained away through contract. The interests of a few should not prevail over the good of the greater number in the community whose health, peace, safety, good order and general welfare, the respondent city officials are under legal obligation to protect. The leases or licenses granted by the City Government to stallholders are null and void for being contrary to law. The Executive Order issued by the acting Mayor authorizing the use of Heroes del '96 Street as a vending area for stallholders contravenes the general law that reserves city streets and roads for public use. The Executive Order may not infringe upon the vested right of the public to use city streets for the purpose they were intended to serve: i.e., as arteries of travel for vehicles and pedestrians. 7. In gratia argumenti, ordinance cannot be validly implemented as municipality has not complied with conditions imposed by the MMA for the approval of the ordinance Even assuming, in gratia argumenti, that the municipality has the authority to pass the disputed ordinance, the same cannot be validly implemented because it cannot be considered approved by the Metropolitan Manila Authority due to non-compliance by the municipality of the conditions imposed by the former for the approval of the ordinance. The allegations of the municipality that the closed streets were not used for vehicular traffic and that the majority of the residents do not oppose the establishment of a flea market on said streets are unsupported by any evidence that will show that the first condition has been met. Likewise, the designation by the Municipality of a time schedule during which the flea market shall operate is absent (fourth condition). 8. Baclaran area congested; establishment of flea market on municipality streets does not help solve problem of congestion It is of public notice that the streets along Baclaran area are congested with people, houses and traffic brought about by the proliferation of vendors occupying the streets. To license and allow the establishment of a flea market along J. Gabrielle, G.G. Cruz, Bayanihan, Lt. Garcia Extension and Opena streets in Baclaran would not help in solving the

problem of congestion but rather leads to inconvenience to children as the normal transportation flow is disrupted, to pollution and deterioration of health of residents due to the garbage left by the vendors on the streets. Further, ambulances and fire engines are not able to use the roads for a more direct access to the fire area and thus lose valuable time that should have been spent in saving properties and lives. And further, the ambulances and people rushing patients to St. Rita Hospital located along GG Cruz Street are delayed as they are unable to pass through said street due to the stalls and vendors. 9. Powers of local government unit not absolute The powers of a local government unit are not absolute. They are subject to limitations laid down by the Constitution and the laws such as our Civil Code. Moreover, the exercise of such powers should be subservient to paramount considerations of health and well-being of the members of the community. Every local government unit has the sworn obligation to enact measures that will enhance the public health, safety and convenience, maintain peace and order, and promote the general prosperity of the inhabitants of the local units. Based on this objective, the local government should refrain from acting towards that which might prejudice or adversely affect the general welfare. 10. General public has legal right to demand the restoration of city streets to their specific public purpose As in the Dacanay case, the general public have a legal right to demand the demolition of the illegally constructed stalls in public roads and streets and the officials of municipality have the corresponding duty arising from public office to clear the city streets and restore them to their specific public purpose. 11. Applicability of the Dacanay case; Contracts by Local Government governed by the original terms and conditions, and the law in force at time the rights were vested As in the Dacanay case, both cases involve an ordinance which is void and illegal for lack of basis and authority in laws applicable during its

time. However, BP 337 (Local Government Code), has already been repealed by RA7160 (Local Government Code of 1991) which took effect on 1 January 1992. Section 5(d) of the new Code provides that rights and obligations existing on the date of effectivity of the new Code and arising out of contracts or any other source of prestation involving a local government unit shall be governed by the original terms and conditions of the said contracts or the law in force at the time such rights were vested. Republic of the Philippines SUPREME COURT Manila SECOND DIVISION G.R. No. 172102 July 2, 2010

REPUBLIC OF THE PHILIPPINES, Petitioner, vs. HANOVER WORLWIDE TRADING CORPORATION, Respondent. DECISION PERALTA, J.: Before the Court is a petition for review on certiorari under Rule 45 of the Rules of Court, seeking the reversal and setting aside of the Decision1 dated May 6, 2005 of the Court of Appeals (CA) in CA-G.R. CV No. 70077, which affirmed the August 7, 1997 Decision of the Regional Trial Court (RTC) of Mandaue City, Branch 56, in LAND REG. CASE NO. N-281. Petitioner also assails the CA Resolution2 dated March 30, 2006, denying its Motion for Reconsideration. The facts of the case are as follows:

On October 15, 1993, Hanover Worldwide Trading Corporation filed an application for Registration of Title over Lot No. 4488 of Consolacion Cad-545-D (New) under Vs-072219-000396, situated in Barrio Sacsac, Consolacion, Cebu, containing an area of One Hundred Three Thousand Three Hundred Fifty (103,350) square meters, more or less, pursuant to Presidential Decree (P.D.) No. 1529, otherwise known as the Property Registration Decree. The application stated that Hanover is the owner in fee simple of Lot No. 4488, its title thereto having been obtained through purchase evidenced by a Deed of Absolute Sale. Attached to the petition are: 1) a Verification Survey Plan; 2) a copy of the approved Technical Description of Lot 4488; 3) a copy of the Deed of Sale in favor of Hanovers President and General Manager; 4) a copy of a Waiver executed by the President and General Manager of Hanover in favor of the latter; 5) a Geodetic Engineer's Certificate attesting that the property was surveyed; 6) a Tax Declaration; 7) a tax clearance; 8) a Municipal Assessor's Certification stating, among others, the assessed value and market value of the property; and 9) a CENRO Certification on the alienability and disposability of the property. Except for the Republic, there were no other oppositors to the application. The Republic contended, among others, that neither Hanover nor its predecessors-in-interest are in open, continuous, exclusive and notorious possession and occupation of the land in question since June 12, 1945 or prior thereto; the muniments of title, tax declarations and receipts of tax payments attached to or alleged in the application do not constitute competent and sufficient evidence of a bona fide acquisition of the lands applied for; Hanover is a private corporation disqualified under the Constitution to hold alienable lands of the public domain; the parcels of land applied for are portions of the public domain belonging to the Republic and are not subject to private appropriation. The case was then called for trial and respondent proceeded with the presentation of its evidence. The Republic was represented in the

proceedings by officers from the Office of the Solicitor General (OSG) and the Department of Environment and Natural Resources (DENR). On August 7, 1997, the RTC rendered its Decision3 approving Hanovers application for registration of the subject lot. It held that from the documentary and oral evidence presented by Hanover, the trial court was convinced that Hanover and its predecessors-in-interest had been in open, public, continuous, notorious and peaceful possession, in the concept of an owner, of the land applied for registration of title, and that it had registrable title thereto in accordance with Section 14 of P.D. 1529. On appeal by the State, the judgment of the RTC was affirmed by the CA via the presently assailed Decision and Resolution. Hence, the instant petition based on the following grounds: I THE DEFECTIVE AND/OR WANT OF NOTICE BY PUBLICATION OF THE INITIAL HEARING OF THE CASE A QUO DID NOT VEST THE TRIAL COURT WITH JURISDICTION TO TAKE COGNIZANCE THEREOF. II DEEDS OF SALE AND TAX DECLARATIONS/CLEARANCES DID NOT CONSTITUTE THE "WELL-NIGH INCONTROVERTIBLE" EVIDENCE NECESSARY TO ACQUIRE TITLE THROUGH ADVERSE OCCUPATION.4 Petitioner claims that the RTC failed to acquire jurisdiction over the case. It avers that the RTC set the initial hearing of the case on September 25, 1995 in an Order dated June 13, 1995. Petitioner contends, however, that, pursuant to Section 23 of P.D. 1529, the initial hearing of the case must be not earlier than forty-five (45) days and not later than ninety (90) days from the date of the Order setting the date and

hour of the initial hearing. Since the RTC Order was issued on June 13, 1995, the initial hearing should have been set not earlier than July 28, 1995 (45 days from June 13, 1995) and not later than September 11, 1995 (90 days from June 13, 1995). Unfortunately, the initial hearing was scheduled and actually held on September 25, 1998, some fourteen (14) days later than the prescribed period. Petitioner also argues that respondent failed to present incontrovertible evidence in the form of specific facts indicating the nature and duration of the occupation of its predecessor-in-interest to prove that the latter has been in possession of the subject lot under a bona fide claim of acquisition of ownership since June 12, 1945 or earlier. The petition is meritorious. As to the first assigned error, however, the Court is not persuaded by petitioners contention that the RTC did not acquire jurisdiction over the case. It is true that in land registration cases, the applicant must strictly comply with the jurisdictional requirements. In the instant case, though, there is no dispute that respondent complied with the requirements of the law for the court to acquire jurisdiction over the case. With respect to the setting of the initial hearing outside the 90-day period set forth under Section 23 of P.D. 1529, the Court agrees with the CA in ruling that the setting of the initial hearing is the duty of the land registration court and not the applicant. Citing Republic v. Manna Properties, Inc.,5 this Court held in Republic v. San Lorenzo Development Corporation6 that: The duty and the power to set the hearing date lie with the land registration court. After an applicant has filed his application, the law requires the issuance of a court order setting the initial hearing date. The notice of initial hearing is a court document. The notice of initial hearing is signed by the judge and copy of the notice is mailed by the clerk of court to the LRA [Land Registration Authority]. This involves a process to which the party-applicant absolutely has no participation. x x x

xxxx x x x a party to an action has no control over the Administrator or the Clerk of Court acting as a land court; he has no right to meddle unduly with the business of such official in the performance of his duties. A party cannot intervene in matters within the exclusive power of the trial court. No fault is attributable to such party if the trial court errs on matters within its sole power. It is unfair to punish an applicant for an act or omission over which the applicant has neither responsibility nor control, especially if the applicant has complied with all the requirements of the law. Moreover, it is evident in Manna Properties, Inc. that what is more important than the date on which the initial hearing is set is the giving of sufficient notice of the registration proceedings via publication. x x x In the instant case, there is no dispute that sufficient notice of the registration proceedings via publication was duly made.1avvphi1 Moreover, petitioner concedes (a) that respondent should not be entirely faulted if the initial hearing that was conducted on September 25, 1995 was outside the 90-day period set forth under Section 23 of Presidential Decree No. 1529, and (b) that respondent substantially complied with the requirement relating to the registration of the subject land. Hence, on the issue of jurisdiction, the Court finds that the RTC did not commit any error in giving due course to respondents application for registration. The foregoing notwithstanding, the Court agrees with petitioner on the more important issue that respondent failed to present sufficient evidence to prove that it or its predecessors-in-interest possessed and occupied the subject property for the period required by law. Section 14 (1) of P.D. 1529, as amended, provides:

SEC. 14. Who may apply. The following persons may file in the proper Court of First Instance an application for registration of title to land, whether personally or through their duly authorized representatives: (1) Those who by themselves or through their predecessors-in-interest have been in open, continuous, exclusive and notorious possession and occupation of alienable and disposable lands of the public domain under a bona fide claim of ownership since June 12, 1945, or earlier.7 Likewise, Section 48 (b) of Commonwealth Act 141, as amended by Section 4 of P.D. 1073, states: Section 48. The following described citizens of the Philippines, occupying lands of the public domain or claiming to own any such lands or an interest therein, but whose titles have not been perfected or completed, may apply to the Court of First Instance [now Regional Trial Court] of the province where the land is located for confirmation of their claims and the issuance of a certificate of title therefor, under the Land Registration Act, to wit: xxxx (b) Those who by themselves or through their predecessors-in-interest have been in open, continuous, exclusive and notorious possession and occupation of agricultural lands of the public domain, under a bona fide claim of acquisition of ownership, since June 12, 1945, or earlier, immediately preceding the filing of the application for confirmation of title except when prevented by war or force majeure. These shall be conclusively presumed to have performed all the conditions essential to a Government grant and shall be entitled to a certificate of title under the provisions of this chapter.8 As the law now stands, a mere showing of possession and occupation for 30 years or more is not sufficient. Therefore, since the effectivity of P.D. 1073 on January 25, 1977, it must now be shown that possession and occupation of the piece of land by the applicant, by himself or through

his predecessors-in-interest, started on June 12, 1945 or earlier. This provision is in total conformity with Section 14 (1) of P.D. 1529.9 Thus, pursuant to the aforequoted provisions of law, applicants for registration of title must prove: (1) that the subject land forms part of the disposable and alienable lands of the public domain, and (2) that they have been in open, continuous, exclusive and notorious possession and occupation of the same under a bona fide claim of ownership since June 12, 1945, or earlier. It is true, as respondent argues, that an examination of these requisites involve delving into questions of fact which are not proper in a petition for review on certiorari. Factual findings of the court a quo are generally binding on this Court, except for certain recognized exceptions,10 to wit: (1) When the conclusion is a finding grounded entirely on speculation, surmises and conjectures; (2) When the inference made is manifestly mistaken, absurd or impossible; (3) Where there is a grave abuse of discretion; (4) When the judgment is based on a misapprehension of facts; (5) When the findings of fact are conflicting; (6) When the Court of Appeals, in making its findings, went beyond the issues of the case and the same is contrary to the admissions of both appellant and appellee; (7) When the findings are contrary to those of the trial Court; (8) When the findings of fact are conclusions without citation of specific evidence on which they are based;

(9) When the facts set forth in the petition as well as in the petitioners main and reply briefs are not disputed by the respondents; and (10) When the findings of fact of the Court of Appeals are premised on the supposed absence of evidence and contradicted by the evidence on record.11 The Court finds that the instant case falls under the third and ninth exceptions. A careful reading of the Decisions of the RTC and the CA will show that there is neither finding nor discussion by both the trial and appellate courts which would support their conclusion that respondents predecessors-in-interest had open, continuous, exclusive and notorious possession and occupation of the disputed parcel of land since June 12, 1945 or earlier. No testimonial evidence was presented to prove that respondent or its predecessors-in-interest had been possessing and occupying the subject property since June 12, 1945 or earlier. Hanovers President and General Manager testified only with respect to his claim that he was the former owner of the subject property and that he acquired the same from the heirs of a certain Damiano Bontoyan; that he caused the payment of realty taxes due on the property; that a tax declaration was issued in favor of Hanover; that Hanover caused a survey of the subject lot, duly approved by the Bureau of Lands; and that his and Hanovers possession of the property started in 1990.12 The pieces of documentary evidence submitted by respondent neither show that its predecessors possession and occupation of the subject land is for the period or duration required by law. The earliest date of the Tax Declarations presented in evidence by respondent is 1965, the others being 1973, 1980, 1992 and 1993. Respondent failed to present any credible explanation why the realty taxes due on the subject property were only paid starting in 1965. While tax declarations are not

conclusive evidence of ownership, they constitute proof of claim of ownership.13 In the present case, the payment of realty taxes starting 1965 gives rise to the presumption that respondents predecessors-ininterest claimed ownership or possession of the subject lot only in that year. Settled is the rule that the burden of proof in land registration cases rests on the applicant who must show by clear, positive and convincing evidence that his alleged possession and occupation of the land is of the nature and duration required by law.14 Unfortunately, as petitioner contends, the pieces of evidence presented by respondent do not constitute the "well-nigh incontrovertible" proof necessary in cases of this nature. Lastly, the Court notes that respondent failed to prove that the subject lot had been declared alienable and disposable by the DENR Secretary. The well-entrenched rule is that all lands not appearing to be clearly of private dominion presumably belong to the State.15 The onus to overturn, by incontrovertible evidence, the presumption that the land subject of an application for registration is alienable and disposable rests with the applicant.16 In the present case, to prove the alienability and disposability of the subject property, Hanover submitted a Certification issued by the Community Environment and Natural Resources Offices (CENRO) attesting that "lot 4488, CAD-545-D, containing an area of ONE HUNDRED THREE THOUSAND THREE HUNDRED FIFTY (103,350) square meters, more or less, situated at Sacsac, Consolacion, Cebu" was found to be within "Alienable and Disposable Block-1, land classification project no. 28, per map 2545 of Consolacion, Cebu." However, this certification is not sufficient. In Republic v. T.A.N. Properties, Inc.17 this Court held that it is not enough for the Provincial Environment and Natural Resources Offices

(PENRO) or CENRO to certify that a land is alienable and disposable, thus: x x x The applicant for land registration must prove that the DENR Secretary had approved the land classification and released the land of the public domain as alienable and disposable, and that the land subject of the application for registration falls within the approved area per verification through survey by the PENRO or CENRO. In addition, the applicant for land registration must present a copy of the original classification approved by the DENR Secretary and certified as a true copy by the legal custodian of the official records. These facts must be established to prove that the land is alienable and disposable x x x.18 In the instant case, even the veracity of the facts stated in the CENRO Certification was not confirmed as only the President and General Manager of respondent corporation identified said Certification submitted by the latter. It is settled that a document or writing admitted as part of the testimony of a witness does not constitute proof of the facts stated therein.19 In the present case, Hanovers President and General Manager, who identified the CENRO Certification, is a private individual. He was not the one who prepared the Certification. The government official who issued the Certification was not presented before the RTC so that he could have testified regarding its contents. Hence, the RTC should not have accepted the contents of the Certification as proof of the facts stated therein. The contents of the Certification are hearsay, because Hanovers President and General Manager was incompetent to testify on the truth of the contents of such Certification. Even if the subject Certification is presumed duly issued and admissible in evidence, it has no probative value in establishing that the land is alienable and disposable.20 Moreover, the CENRO is not the official repository or legal custodian of the issuances of the DENR Secretary declaring the alienability and disposability of public lands.21 Thus, the CENRO Certification should

have been accompanied by an official publication of the DENR Secretarys issuance declaring the land alienable and disposable. Respondent, however, failed to comply with the foregoing requirements. WHEREFORE, the petition is GRANTED. The May 6, 2005 Decision and March 30, 2006 Resolution of the Court of Appeals in CA-G.R. CV No. 70077 and the August 7, 1997 Decision of the Regional Trial Court of Mandaue City, Branch 56 in Land Registration Case No. N-281 are SET ASIDE. Respondent Hanover Worldwide Trading Corporations application for registration of Lot No. 4488 of Consolacion Cad-545-D (New), under Vs-072219-000396, Barrio Sacsac, Consolacion, Cebu, is DENIED. SO ORDERED. DIOSDADO M. PERALTA Associate Justice WE CONCUR:

Ladera v. Hodges G.R. No. 8027-R, September 23, 1952, Vol. 48, No. 12, Official Gazette 5374 Reyes, J.B.L., J. FACTS: Paz G. Ladera entered into a contract with C.N. Hodges. Hodges promised to sell a lot with an area of 278 square meters to Ladera, subject to certain terms and conditions. The agreement called for a down payment of P 800.00 and monthly installments of P 5.00 each with interest of 1% per month, until P 2,085 is paid in full. In case of failure of the purchaser to make any monthly payment within 60 days after it fell due, the contract may be considered as rescinded or annulled. Ladera built a house on the lot. Later on, she defaulted in the payment of the agreed monthly installment. Hodges filed an action for the ejectment of Ladera. The court issued an alias writ of execution and pursuant thereto, the city sheriff levied upon all rights, interests, and participation over the house of Ladera. At the auction sale, Laderas house was sold to Avelino A. Magno. Manuel P. Villa, later on, purchased the house from Magno. Ladera filed an action against Hodges and the judgment sale purchasers. Judgment was rendered in favor of Ladera, setting aside the sale for non-compliance with Rule 39, Rules of Court regarding judicial sales of real property. On appeal, Hodges contends that the house, being built on a lot owned by another, should be regarded as movable or personal property. ISSUE: Whether or not Laderas house is an immovable property. HELD: YES. The old Civil Code numerates among the things declared by it as immovable property the following: lands, buildings, roads and constructions of all kind adhered to the soil. The law does not make any distinction whether or not the owner of the lot is the one who built. Also, since the principles of accession regard buildings and constructions as mere accessories to the land on which it is built, it is logical that said accessories should partake the nature of the principal thing.

Mindanao Bus Company v. The City Assessor and Treasurer G.R. No. L-17870, September 29, 1962, 6 SCRA 197 Labrador, J. FACTS: Petitioner Mindanao Bus Company is a public utility solely engaged in transporting passengers and cargoes by motor trucks, over its authorized lines in the Island of Mindanao, collecting rates approved by the Public Service Commission. Respondent sought to assess the following real properties of the petitioner; (a) Hobart Electric Welder Machine, (b) Storm Boring Machine; (c) Lathe machine with motor; (d) Black and Decker Grinder; (e) PEMCO Hydraulic Press; (f) Battery charger (Tungar 1

charge machine) and (g) D-Engine Waukesha-M-Fuel. It was alleged that these machineries are sitting on cement or wooden platforms, and that petitioner is the owner of the land where it maintains and operates a garage for its TPU motor trucks, a repair shop, blacksmith and carpentry shops, and with these machineries, which are placed therein. Respondent City Assessor of Cagayan de Oro City assessed at P4, 400 petitioner's above-mentioned equipment. Petitioner appealed the assessment to the respondent Board of Tax Appeals on the ground that the same are not realty. Respondents contend that said equipments, though movable, are immobilized by destination, in accordance with paragraph 5 of Article 415 of the New Civil Code. ISSUE: Whether the equipments in question are immovable or movable properties. HELD: The equipments in question are movable. So that movable equipments to be immobilized in contemplation of the law, it must first be "essential and principal elements" of an industry or works without which such industry or works would be "unable to function or carry on the industrial purpose for which it was established." Thus, the Court distinguished those movable which become immobilized by destination because they are essential and principal elements in the industry from those which may not be so considered immobilized because they are merely incidental, not essential and principal. The tools and equipments in question in this instant case are, by their nature, not essential and principle municipal elements of petitioner's business of transporting passengers and cargoes by motor trucks. They are merely incidentalsacquired as movables and used only for expediency to facilitate and/or improve its service. Even without such tools and equipments, its business may be carried on, as petitioner has carried on, without such equipments, before the war. The transportation business could be carried on without the repair or service shop if its rolling equipment is repaired or serviced in another shop belonging to another.

Makati Leasing and Finance Corporation v. Wearever Textile Mills, Inc. G.R. No. L-58469, May 16, 1983, 122 SCRA 29 De Castro, J. FACTS: To obtain financial accommodations from the Makati Leasing and Finance Corporation, the Wearever Textile discounted and assigned several receivables with them under a receivable purchase agreement. To secure the collection of receivables assigned, Wearever Textile executed a chattel mortgage over certain raw materials inventory, as well as machinery described as an aero dryer stentering range. Upon default of Wearever Textile, the Makati Leasing petitioned for extrajudicial foreclosure of the properties mortgaged to it. When the sheriff failed to enter Wearever Textiles premises to seize the machinery, Makati Leasing applied for a replevin. Wearever Textile contended that it cannot be a subject of replevin or a chattel mortgage because

it is a real property as it is attached to the ground by means of bolts and that the only way to remove it is to destroy the concrete floor. ISSUE: Whether or not the machinery is real or personal property. HELD: The machinery is a personal property. The Supreme Court explained that if a house of strong materials may be considered as personal property for purposes of executing a chattel mortgage, there is absolutely no reason why a machinery, which is movable in its nature and becomes immobilized only by destination or purpose, may not be likewise treated as such.

Santos Evangelista v. Alto Surety and Insurance Co., Inc. G.R. No. L-11139, April 23, 1958, 103 Phil. 401 Concepcion, J. FACTS: On June 4, 1949, Santos Evangelista instituted a civil case for a sum of money. On the same date, he obtained a writ of attachment, which was levied upon a house, built by Rivera on a land situated in Manila and leased to him. In due course, judgment was rendered in favor of Evangelista, who bought the house at public auction held in compliance with the writ of execution issued in said case. When Evangelista sought to take possession of the house, Rivera refused to surrender it, upon the ground that he had leased the property from the Alto Surety & Insurance Co., Inc. and that the latter is now the true owner of said property. It appears that on May 10, 1952, a definite deed of sale of the same house had been issued to Alto Surety, as the highest bidder at an auction sale held. Hence, Evangelista instituted an action against Alto Surety and Ricardo Rivera, for the purpose of establishing his title over said house, and securing possession thereof, apart from recovering damages. After due trial, the CFI Manila rendered judgment for Evangelista, sentencing Rivera and Alto Surety to deliver the house in question to Evangelista and to pay him, jointly and severally, P40.00 a month from October, 1952, until said delivery, plus costs. ISSUE: Whether or not a house constructed by the lessee of the land on which it is built, should be dealt with, for purposes of attachment, as immovable property or as personal property. HELD: The house is not personal property, much less a debt, credit or other personal property not capable of manual delivery, but immovable property. As explicitly held, in Ladera vs. Hodges (48 OG 5374), "a true building (not merely superimposed on the soil) is immovable or real property, whether it is erected by the owner of the land or by a usufructuary or lessee. The opinion that the house of Rivera should have been attached in accordance with subsection (c) of said section 7, as "personal property capable of manual delivery, by taking and safely keeping in his custody", for it declared that "Evangelista could not have validly purchased Ricardo Rivera's house from the sheriff

as the latter was not in possession thereof at the time he sold it at a public auction is untenable.

Tsai v. Court of Appeals G.R. No. 120098, October 2, 2001, 366 SCRA 324 Quisumbing, J. FACTS: On November 26, 1975, respondent Ever Textile Mills, Inc. (EVERTEX) obtained a three million peso (P3,000,000.00) loan from petitioner Philippine Bank of Communications (PBCom). As security for the loan, EVERTEX executed in favor of PBCom, a deed of Real and Chattel Mortgage over the lot where its factory stands, and the chattels located therein. On April 23, 1979, PBCom granted a second loan to EVERTEX. The loan was secured by a chattel mortgage over personal properties enumerated in a list attached thereto. After April 23, 1979, the date of the execution of the second mortgage mentioned above, EVERTEX purchased various machines and equipments. Upon EVERTEX's failure to meet its obligation to PBCom, the latter commenced extrajudicial foreclosure proceedings against EVERTEX. On December 15, 1982, the first public auction was held where petitioner PBCom emerged as the highest bidder and a Certificate of Sale was issued in its favor on the same date. On March 7, 1984, PBCom consolidated its ownership over the lot and all the properties in it. In November 1986, it leased the entire factory premises to petitioner Ruby L. Tsai. On May 3, 1988, PBCom sold the factory, lock, stock, and barrel to Tsai, including the contested machineries. On March 16, 1989, EVERTEX filed a complaint for annulment of sale, reconveyance, and damages with the Regional Trial Court against PBCom. EVERTEX claimed that no rights having been transmitted to PBCom over the assets of insolvent EVERTEX, therefore Tsai acquired no rights over such assets sold to her, and should reconvey the assets. ISSUE: Whether or not the inclusion of the questioned properties in the foreclosed properties is proper. HELD: Yes. While it is true that the questioned properties appear to be immobile, a perusal of the contract of Real and Chattel Mortgage executed by the parties gives a contrary indication. In the case at bar, the true intention of PBCOM and the owner, EVERTEX, is to treat machinery and equipment as chattels. Assuming that the properties in question are immovable by nature, nothing detracts the parties from treating it as chattels to secure an obligation under the principle of estoppel. It has been held that an immovable may be considered a personal property if there is a stipulation as when it is used as security in the payment of an obligation where a chattel mortgage is executed over it, as in the case at bar. 4

Sergs Products, Inc. v. PCI Leasing and Finance, Inc. G.R. No. 137705, August 22, 2000, 338 SCRA 499 Panganiban, J. FACTS: Respondent PCI Leasing and Finance Inc. filed with the RTC of Quezon City a complaint for sum of money, with an application for a writ of replevin. A writ of replevin was issued, directing the sheriff to seize and deliver the machineries and equipment to PCI Leasing after five days and upon payment of the necessary expenses. The sheriff proceeded to petitioner's factory and seized one machinery. Petitioner filed a motion for special protective order invoking the power of the court to control the conduct of its officers and amend and control its processes, praying for a directive for the sheriff to defer enforcement of the writ of replevin. The motion was opposed by PCI on the ground that the properties were personal and therefore still subject to seizure and writ of replevin. In their reply, petitioners asserted that the properties were immovable as defined in Article 415 of the Civil Code, the parties' agreement to the contrary notwithstanding. Petitioners went to the Court of Appeals via an original action for certiorari. The Court of Appeals ruled that the subject machines were personal property as provided by the agreement of the parties. ISSUE: Whether or not the subject machines were personal, not real, property, which may be a proper subject of a writ of replevin. HELD: The contracting parties may validly stipulate that a real property be considered as personal. After agreeing to such stipulation, they are consequently estopped from claiming otherwise. Under the principle of estoppel, a party to a contract is ordinarily precluded from denying the truth of any material fact found therein. In the present case, the lease agreement clearly provides that the machines in question are to be considered as personal properties. Clearly then, petitioners were estopped from denying the characterization of the subject machines as personal property. Under the circumstances, they are proper subject of the writ of seizure. Accordingly, the petition was denied and the assailed decision of the Court of Appeals was affirmed.

Burgos v. Chief of Staff, AFP G.R. No. 64261, December 26, 1984, 133 SCRA 800 Escolin, J. FACTS: On December 7, 1982, two search warrants where issued and the premises at 19, Road 3, Project 6, Quezon City, and 784 Units C & D, RMS Building, Quezon Avenue, Quezon City, business addresses of the "Metropolitan Mail" and "We Forum" newspapers were searched. Office and printing machines, equipment, paraphernalia, motor vehicles and other articles used in the printing, publication and distribution of the 5

said newspapers, as well as numerous papers, documents, books and other written literature alleged to be in the possession and control of Jose Burgos, Jr. publisher-editor of the "We Forum" newspaper, were seized. ISSUE: Whether or not real properties were seized under the disputed warrants. HELD: No. Under Article 415 (5) of the Civil Code, "machinery, receptacles, instruments or implements intended by the owner of the tenement for an industry or works which may be carried on in a building or on a piece of land and which tend directly to meet the needs of the said industry or works" are considered immovable property. In Davao Sawmill Co. v. Castillo, it was said that machinery which is movable by nature becomes immobilized when placed by the owner of the tenement, property or plant, but not so when placed by a tenant, usufructuary, or any other person having only a temporary right, unless such person acted as the agent of the owner. In the present case, petitioners do not claim to be the owners of the land and/or building on which the machineries were placed. The machineries, while in fact bolted to the ground, remain movable property susceptible to seizure under a search warrant.

Lopez v. Orosa, Jr., and Plaza Theatre, Inc. G.R. No. L-10817-18, February 28, 1958, 103 Phil. 98 Felix, J. FACTS: Lopez was engaged in business under the name Lopez-Castelo Sawmill. Orosa approached Lopez and invited the latter to make an investment in the theatre business he was forming, the Plaza Theatre. Lopez expressed his unwillingness to invest. Nonetheless, Lopez agreed to supply the lumber for the construction of the theatre. Lopez further agreed that that the payment therefore would be on demand and not cash on delivery basis. Lopex delivered the lumber which was used for the construction of the Plaza Theatre. However, of the total cost of materials amounting to P62, 255.85, Lopez was paid only P 20, 848.50, thus leaving a balance of P 41, 771.35. Due to Lopez demands, Orosa issued a deed of assignment over his shares of stock of the Plaza Theatre, Inc. As there was still an unpaid balance, Lopez filed a case against Orosa and Plaza Theatre. He asked that Orosa and Plaza theatre be held liable solidarily for the unpaid balance, and in case defendants failed to pay, the land and building should be sold in public auction with the proceeds to be applied to the balance, or that the shares of stock be sold in public auction. ISSUE: Whether or not the lien for the value of the materials used in the construction of the building attaches to said structure alone and does not extend to the land on which the building is adhered to. HELD: No. While it is true that generally, real estate connotes the land and the building constructed thereon, it is obvious that the inclusion of the building, separate and distinct 6

from the land, in the enumeration of what may constitute real properties could only mean one thingthat a building is by itself an immovable property. In view of the absence of any specific provision to the contrary, a building is an immovable property irrespective of whether or not said structure and the land on which it is adhered to belong to the same owner. The lien so created attaches merely to the immovable property for the construction or repair of which the obligation was incurred. Therefore, the lien in favor of appellant for the unpaid value of the lumber used in the construction of the building attaches only to said structure and to no other property of the obligors.

Yap v. Taada G.R. No. L-32917, July 18, 1988, 163 SCRA 464 Narvasa, J. FACTS: Goulds Pumps International (Phil.), Inc. filed a complaint against Yap and his wife seeking recovery of P1,459.30 representing the balance of the price and installation cost of a water pump in the latter's premises. Goulds presented evidence ex parte and judgment by default was rendered by Judge Taada requiring Yap to pay to Goulds the unpaid balance of the pump purchased by him and interest of 12% per annum. Thereafter, the water pump in question was levied by the sheriff and by notice dated November 4, 1969, scheduled the execution sale thereof. But in view of the pendency of Yap's motion for reconsideration, suspension of the sale was directed. It appears however that a copy of the order suspending the sale was not transmitted to the sheriff Hence, the Deputy Provincial Sheriff went ahead with the scheduled auction sale and sold the property levied on to Goulds as the highest bidder. Yap argues that "the sale was made without the notice required by Sec. 18, Rule 39, of the New Rules of Court," i.e., notice by publication in case of execution sale of real property, the pump and its accessories being immovable because attached to the ground with character of permanency (Art. 415, Civil Code). ISSUE: Whether or not the water pump in question is an immovable property. HELD: No. Yap's argument is untenable. The Civil Code considers as immovable property, among others, anything "attached to an immovable in a fixed manner, in such a way that it cannot be separated therefrom without breaking the material or deterioration of the object." The pump does not fit this description. It could be, and was in fact separated from Yap's premises without being broken or suffering deterioration. Obviously, the separation or removal of the pump involved nothing more complicated than the loosening of bolts or dismantling of other fasteners.

Machinery and Engineering Supplies, Inc. v. Court of Appeals G.R. No. L-7057, October 29, 1954, 96 Phil. 70 Concepcion, J. FACTS: Petitioner Machinery and Engineering Supplies filed a complaint for replevin for the recovery of the machinery and equipment sold and delivered to Ipo Limestone Co. An order was issued to seize and take immediate possession of the properties specified in the order. Upon carrying out the courts order, Roco, the companys President, along with a crew of technical men and labourers, proceeded to the factory. The manager of Ipo Limestone Co. and Torres protested against the seizure of the properties on the ground that they are not personal properties. However, since the sheriff contended that his duty is purely ministerial, they all went to the factory and dismantled the equipment despite the fact that the equipment could not be dismantled without causing damage or injuries to the wooden frames attached to them. Consequently, they had to cut some of the supports of the equipment which rendered its use impracticable. ISSUE: Whether or not the machinery and equipment in question could be the subject of replevin. HELD: No. Replevin is applicable only to personal property. The machinery and equipment in question appeared to be attached to the land, particularly to the concrete foundation of said premises, in a fixed manner, in such a way that the former could not be separated from the latter without breaking the material or deterioration of the object. Hence, in order to remove the said outfit, it became necessary not only to unbolt the same, but also to cut some of its wooden supports. Moreover, said machinery and equipment were intended by the owner of the tenement for an industry carried on said immovable. For these reasons, they were already immovable pursuant to paragraphs 3 and 5 of Article 415 of the Civil Code.

FELS Energy, Inc. v. The Province of Batangas G.R. No. 168557, February 16, 2007 Callejo, Sr., J. FACTS: On January 18, 1993, NPC entered into a lease contract with Polar Energy, Inc. over diesel engine power barges moored at Balayan Bay in Calaca, Batangas. The contract staes that NPC shall be responsible for the payment of all taxes other levies imposed government to which POLAR may be or become subject to in respect of the Power Barges. Subsequently, Polar Energy, Inc. assigned its rights under the agreement to FELS Energy Inc. On August 7, 1995, FELS received an assessment of real property taxes on the power barges from Provincial Assessor of Batangas City. The assessed tax amounted to P56,184,088.40 per annum. FELS referred the matter to NPC, reminding it of its obligation under the agreement to pay all real estate taxes. NPC sought reconsideration 8

of the Provincial Assessors decision to assess real property taxes on the power barges, alleging that barges are non-taxable items. In its answer, the Provincial Assessor averred that the barges were real property for purposes of taxation under Section 199(c) of Republic Act (R.A.) No. 7160. ISSUE: Whether power barges, which are floating and movable, are personal properties and therefore, not subject to real property tax. HELD: NO. The power barges are real property and are thus subject to real property tax. Tax assessments by tax examiners are presumed correct and made in good faith, with the taxpayer having the burden of proving otherwise. Besides, factual findings of administrative bodies, which have acquired expertise in their field, are generally binding and conclusive upon the Court.

Laurel v. Garcia G.R. No. 92013, July 25, 1990, 187 SCRA 797 Gutierrez, J. FACTS: In view of the Reparations Agreement between the Philippines and Japan, four properties located in Japan were given to the Philippines. One of these properties is the Roppongi property. The said property was formerly the location of the Chancery of the Philippine Embassy until it was transferred to Nampeidai on July 22, 1976. The Roppongi property has remained abandoned from the time of the transfer due to lack of funds to develop the said property. Consequently, Administrative orders were issued by the President authorizing the study of the condition of the properties of the Philippines in Japan. Subsequently, Executive Order 296 was issued by President Aquino allowing non-Filipinos to buy or lease some of the properties of the Philippines located in Japan, including Roppongi. Petitioners now contend that the Roppongi property cannot be alienated as it is classified as public dominion and not of private ownership because it is a property intended for public service under paragraph 2, article 420 of the Civil Code. On the other hand, respondents aver that it has already become part of the patrimonial property of the State which can be alienated because it has not been used for public service for over 13 years. They further contend that EO 296 converted the subject property to patrimonial property. ISSUE: Whether or not the Roppongi property still forms part of the public dominion hence cannot be disposed nor alienated. HELD: Yes. The respondents failed to convincingly show that the property has already become patrimonial. The fact that the Roppongi site has not been used for a long time for actual Embassy service does not automatically convert it to patrimonial property. Under Art. 422 of the Civil Code, there must be a definite and a formal declaration on the part of the government to withdraw it from being public. Abandonment must be a certain and a positive act based on correct legal premises. The mere transfer of the embassy to Nampeidai is not a relinquishment of the propertys original purpose. The Administrative orders authorizing the study of the conditions of government properties in Japan were merely directives for investigation but did not in any way signify a clear intention to dispose of the properties. Likewise, EO 296 did not declare that the properties lost their public character; it merely made them available to foreigners in case of sale, lease or other disposition. Thus, since there is no law authorizing its conveyance, the Roppongi property still remains part of the inalienable properties of the State.

Rabuco v. Villegas G.R. No. L-24916, February 28, 1974, 55 SCRA 658 10

Teehankee, J. FACTS: The issue in this case involves the constitutionality of Republic Act No. 3120 whereby the Congress converted the lots in question together with another lot in San Andres, Malate that are reserved as communal property into disposable or alienable lands of the State. Such lands are to be placed under the administration and disposal of the Land Tenure Administration for subdivision into small lots not exceeding 120 square meters per lot for sale on instalment basis to the tenants or bona fide occupants thereof and expressly prohibited ejectment and demolition of petitioners' homes under Section 2 of the Act. Respondent contends that the Act is invalid and unconstitutional for it constitutes deprivation of property without due process of law and without just compensation. ISSUE: Whether or not Republic Act No. 3120 is constitutional. HELD: Yes. The lots in question are manifestly owned by the city in its public and governmental capacity and are therefore public property over which Congress had absolute control as distinguished from patrimonial property owned by it in its private or proprietary capacity of which it could not be deprived without due process and without just compensation. It is established doctrine that the act of classifying State property calls for the exercise of wide discretionary legislative power, which will not be interfered with by the courts. The Acts in question were intended to implement the social justice policy of the Constitution and the government program of land for the landless and that they were not intended to expropriate the property involved but merely to confirm its character as communal land of the State and to make it available for disposition by the National Government. The subdivision of the land and conveyance of the resulting subdivision lots to the occupants by Congressional authorization does not operate as an exercise of the power of eminent domain without just compensation in violation of Section 1, subsection (2), Article III of the Constitution, but simply as a manifestation of its right and power to deal with state property.

Macasiano v. Diokno G.R. No. 97764, August 10, 1992, 212 SCRA 464 Medialdea, J. FACTS: The Municipality of Paranque passed an ordinance that authorized the closure of J. Gabriel, G.G. Cruz, Bayanihan, Lt. Garcia Extension and Opena Streets located at Baclaran, Paranaque Metro Manila and the establishment of a flea market thereon. Thereafter, the municipal council of Paranaque issued a resolution authorizing Paranaque Mayor Walfrido N. Ferrer to enter into a contract with any service cooperative for the establishment, operation, maintenance and management of flea markets and/or vending areas. By virtue of this, respondent municipality and respondent Palanyag, a service cooperative, entered into an agreement whereby the latter shall operate, maintain and manage the flea market in the aforementioned streets with the 11

obligation to remit dues to the treasury of the municipal government of Paranaque. Consequently, market stalls were put up by Palanyag on the said streets. Petitioner Macasiano, PNP Superintendent of the Metropolitan Traffic Command, then ordered the destruction and confiscation of the stalls along the abovementioned streets. Hence, respondents filed with the trial court a joint petition for prohibition and mandamus with damages and prayer for preliminary injunction, to which the petitioner filed his opposition to the issuance of the writ of preliminary injunction. The trial court upheld the validity of the ordinance in question. ISSUE: Whether or not an ordinance or resolution which authorizes the lease and use of public streets or thoroughfares as sites for flea markets is valid. HELD: No. The aforementioned streets are local roads used for public service and are therefore considered public properties of respondent municipality. Article 424 of the Civil Code provides that properties of public dominion devoted for public use and made available to the public in general are outside the commerce of man and cannot be disposed of or leased by the local government unit to private persons. Properties of the local government which are devoted to public service are deemed public and are under the absolute control of Congress. Hence, LGUs have no authority whatsoever to control or regulate the use of public properties unless specific authority is vested upon them by Congress.

Republic of the Philippines v. Court of Appeals G.R. No. 100709, November 14, 1997, 281 SCRA 639 Panganiban, J. FACTS: Morato filed for a patent on a parcel of land located in Calauag, Quezon, which was approved, provided that the land shall not be encumbered or alienated within a period of five years from the date of the issuance of the patent. Later on, the land was established to be a portion of Calauag Bay, which was five to six feet deep during high tides and three feet deep on low tides. The water level rose because of the ebb and flow of tides from the bay and the storms that frequently passed through the area. Furthermore, it was observed by the Director of Lands from his investigation, that the land of Morato was leased to Advincula for P100 per month and it was also mortgaged to Co for P10,000. The Director of Lands filed a suit with the contention that Morato violated the 5-year prohibitory period and thus the patent should be cancelled and the land should revert back to the State. ISSUE: Whether or not there is a violation of the prohibition of the patent, and thus, the subject land should revert back to the ownership of the State. HELD: Yes. The lease was an encumbrance included in the prohibitions of the patent because it impairs the use of the land by Morato herself. As for the mortgage, it is a 12

legal limit on the title and if there will be foreclosure because Morato was not able to pay her debts, the property will be auctioned. It is also a limitation on Morato's right to enjoy and possess the land for herself. Encumbrance, as defined, is an impairment on the use or transfer of property, or a claim or lien on the property where there is a burden on the title. Thus, Morato clearly violated the terms of the patent on these points. Moreover, the property became a foreshore land because it turned into a portion of land which was covered most of the time with water, whether it was low or high tide. Foreshore is defined as land between high and low waters which is dry depending on the reflux or ebb of the tides. In accordance with this land reclassification, the land can no longer be subject to a pending patent application and must be returned to the State.

Province of Zamboanga del Norte v. City of Zamboanga G.R. No. L-24440, March 28, 1968, 22 SCRA 1334 Bengzon, J.P., J. FACTS: On June 6, 1952, Republic Act 711 was approved dividing the province of Zamboanga into two (2): Zamboanga del Norte and Zamboanga del Sur. Republic Act 3039 was approved providing that all buildings, properties and assets belonging to the former province of Zamboanga and located within the City of Zamboanga are hereby transferred, free of charge, in favor of the said City of Zamboanga. Plaintiff-appellee Zamboanga del Norte filed a complaint in the Court of First Instance of Zamboanga del Norte against defendants-appellants Zamboanga City, the Secretary of Finance and the Commissioner of Internal Revenue. It was prayed that Republic Act 3039 be declared unconstitutional for depriving plaintiff province of property without due process and just compensation. Included in the properties were the capital site and capitol building, certain school sites, hospital and leprosarium sites, and high school playground. ISSUE: Whether or not the properties mentioned are properties for public use or patrimonial. HELD: The subject properties are properties for public use. The validity of the law ultimately depends on the nature of the lots and buildings in question. The principle itself is simple: If the property is owned by the municipality (meaning municipal corporation) in its public and governmental capacity, the property is public and Congress has absolute control over it. But if the property is owned in its private or proprietary capacity, then it is patrimonial and Congress has no absolute control. The municipality cannot be deprived of it without due process and payment of just compensation. Applying the norm obtaining under the principles constituting the law of Municipal Corporations, all those of the 50 properties in question which are devoted to public service are deemed public; the rest remain patrimonial. Under this norm, to be 13

considered public, it is enough that the property be held and, devoted for governmental purposes like local administration, public education, public health, etc. Regarding the several buildings existing on the lots above-mentioned, the records do not disclose whether they were constructed at the expense of the former Province of Zamboanga. Considering however the fact that said buildings must have been erected even before 1936 when Commonwealth Act 39 was enacted and the further fact that provinces then had no power to authorize construction of buildings such as those in the case at bar at their own expense, it can be assumed that said buildings were erected by the National Government, using national funds. Hence, Congress could very well dispose of said buildings in the same manner that it did with the lots in question.

Chavez v. Public Estates Authority G.R. No. 133250, July 9, 2002 Carpio, J. FACTS: In 1973, the Government through the Commissioner of Public Highways and the Construction and Development Corporation of the Philippines (CDCP) signed a contract to reclaim certain foreshore and offshore areas of Manila Bay. PD 1084 was issued, creating Public Estates Authority (PEA), and PD 1085, transferring the reclaimed lands under the MCCRRP to PEA. In 1995, PEA entered into a Joint Venture Agreement (JVA) with AMARI, a private corporation to develop the Freedom Islands, and the JVA was approved by President Ramos. However, PEA and AMARI entered into the JVA through negotiation without public bidding. A Legal Task Force was created to look into the issue. The said task force upheld the legality of the JVA. In 1998, Frank I. Chavez, as a taxpayer, filed a petition to compel PEA to disclose all facts on its negotiations with AMARI, invoking the constitutional right of the people to information on matters of public concern. He assails the sale to AMARI of lands of the public domain as a blatant violation of the constitutional prohibiting in the sale of alienable lands of the public domain to private corporations. Despite the ongoing court petitions, PEA and AMARI signed an Amended Joint Venture Agreement (Amended JVA) in 1999, and such was approved by President Estrada. The Amended JVA seeks to convey to AMARI the ownership of 77.34 hectares of the Freedom Islands. ISSUE: Whether AMARI has the capacity to acquire the lands held by PEA. HELD: No. Under the 1987 Constitution, private corporations such as AMARI cannot acquire alienable land of the public domain. Reclaimed lands comprising the Freedom Islands, which are covered by certificates of title in the name of PEA, are alienable 14

lands of the public domain. PEA may lease these lands to private corporations but may not sell or transfer ownership of these lands to private corporations. PEA may only sell these lands to Philippine citizens, subject to the ownership limitations in the 1987 Constitution and existing laws. Thus, the Amended Joint Venture Agreement between AMARI and PEA was null and void.

Chavez v. National Housing Authority G.R. No. 164527, August 15, 2007 Velasco, Jr., J. FACTS: President Corazon Aquino issued Memorandum Order No. 161 approving and directing the implementation of the Comprehensive and Integrated Metropolitan Manila Waste Management Plan. Respondent National Housing Authority was ordered to conduct feasibility studies and develop lowcost housing projects at the dumpsite and absorb scavengers in NHA resettlement/lowcost housing projects, particularly in the Smokey Mountain. It produced the Smokey Mountain Development Plan and Reclamation of the Area Across R-10 or the Smoke Mountain Development and Reclamation Project. The Project aimed to covert Smokey mountain dumpsite into a habitable housing project, inclusive of the reclamation of the area. President Aquino approved the said Project through MO 415. After President Aquinos term, President Fidel Ramos, through Proclamation No. 39, authorized the NHA to enter into a Joint Venture Agreement with R-II Builders, Inc. (RBI) for the implementation of the project. Afterwards, President Ramos issued Proclamation No. 465 increasing the proposed area for reclamation across R-10 from 40 hectares to 79 hectares. The petitioner Francisco Chavez contended that the respondent NHA or respondent RBI has no authority to reclaim foreshore and submerged land. ISSUE: Whether or not respondent NHA has the authority to reclaim foreshore and submerged land. HELD: Yes. The National Housing Authority (NHA) is a government agency not tasked to dispose of public lands under its charter it is an end-user agency authorized by law to administer and dispose of reclaimed lands. The moment titles over reclaimed lands based on the special patents are transferred to the National Housing Authority (NHA) by the Register of Deeds, they are automatically converted to patrimonial properties of the State which can be sold to Filipino citizens and private corporations, 60% of which are owned by Filipinos. The combined and collective effect of Proclamations Nos. 39 and 465 with Special Patents Nos. 3592 and 3598 is tantamount to and can be considered to be an official declaration that the reclaimed lots are alienable or disposable lands of the public domain. Even if it is conceded that there was no explicit declaration that the lands are no longer needed for public use or public service, there was however an implicit executive declaration that the reclaimed areas are not necessary anymore for public use or public service when President Aquino through MO 415 conveyed the same to the National Housing Authority (NHA) partly for 15

housing project and related commercial/industrial development intended for disposition to and enjoyment of certain beneficiaries and not the public in general and partly as enabling component to finance the project.

Manila International Airport Authority v. Court of Appeals G.R. No. 155650, July 20, 2006 Carpio, J. FACTS: MIAA received Final Notices of Real Estate Tax Delinquency from the City of Paraaque for the taxable years 1992 to 2001. MIAAs real estate tax delinquency was estimated at P624 million. Thus, the City of Paraaque, through its City Treasurer, issued notices of levy and warrants of levy on the Airport Lands and Buildings. The Mayor of the City of Paraaque threatened to sell at public auction the Airport Lands and Buildings should MIAA fail to pay the real estate tax delinquency. City of Paraaque contends that Section 193 of the Local Government Code expressly withdrew the tax exemption privileges of government-owned and-controlled corporations upon the effectivity of the Local Government Code. However, MIAA avers that airport lands and buildings are owned by the State, and thus, exempt from tax. ISSUE: Whether or not airport lands and buildings of MIAA are exempt from real estate tax. HELD: Yes. MIAA is a government instrumentality vested with corporate powers to perform efficiently its governmental functions. MIAA is like any other government instrumentality, the only difference is that MIAA is vested with corporate powers. Unless the government instrumentality is organized as a stock or non-stock corporation, it remains a government instrumentality exercising not only governmental but also corporate powers. Thus, MIAA exercises the governmental powers of eminent domain, police authority and the levying of fees and charges. The airport lands and buildings of MIAA are property of public dominion and therefore owned by the State or the Republic of the Philippines. Hence, the subject properties are not subject to tax.

16

Javier v. Veridiano II G.R. No. L-48050, October 10, 1994, 237 SCRA 565 Bellosillo, J. FACTS: Javier filed a Miscellaneous Sales Application for lot 1641. She later instituted a complaint for forcible entry against Babol, alleging that she was forcibly dispossessed of a portion of said land. The case for forcibly entry was however dismissed as it was found by the court that the occupied portion was outside Lot 1641. The same was dismissed on appeal. Javier was eventually granted a Miscellaneous Sales Patent and issued an OCT for lot 1641. Babol, however had sold the property he was occupying, including a portion of 200 square meters to Rosete. Javier demanded the surrender of the same area from Rosete who repeatedly refused to comply. After 4 years, Javier instituted a complaint for quieting of title and recovery of possession with damages against Babol and Rosete. Rosete moved to dismiss the complaint on the ground of res judicata. The CFI sustained the argument of Rosete and granted his motion to dismiss. Javier contends that res judicata cannot apply in the instant case since there is no identity of parties and causes of action between her complaint for forcible entry, which had long become final and executory, and her subsequent petition for quieting of title. Javier maintains that there is no identity of causes of action since the first case was for forcible entry, which is merely concerned with the possession of the property, whereas the subsequent case was for quieting of title, which looks into the ownership of the disputed land. ISSUE: Whether or not there are really different causes of action between the forcible entry case and the later quieting of title case. HELD: Yes. For res judicata to bar the institution of a subsequent action the following requisites must concur: (1) There must be a final judgment or order; (2) The court rendering the judgment must have jurisdiction over the subject matter; (3) The former judgment is a judgment on the merits; and, (4) There is between the first and second actions identity of (4a) parties, (4b) of subject matter and (4c) of causes of action. Javier's argument that there is no identity of parties between the two actions is without merit. We have repeatedly ruled that for res judicata to apply, what is required is not absolute but only substantial identity of parties. But, there is merit in Javier's argument that there is no identity of causes of action. "The only issue in an action for forcible entry is the physical or material possession of real property, that is, possession de facto and not possession de jure. The philosophy underlying this remedy is that irrespective of the actual condition of the title to the property, the party in peaceable quiet possession shall not be turned out by strong hand, violence or terror." A judgment rendered in a case for recovery of possession is conclusive only on the question of possession and not on the ownership. It does not in any way bind the title or affects the ownership of the land or building. On the other hand, Civil Case No. 2203-0 is in reality an action to recover a parcel of land or an accion reivindicatoria under Art. 434 of the Civil Code, and should be 17

distinguished from Civil Case No. 926, which is an accion interdictal. Accion interdictal, which is the summary action for forcible entry ( detentacion) where the defendant's possession of the property is illegal ab initio, or the summary action for unlawful detainer (desahuico) where the defendant's possession was originally lawful but ceased to be so by the expiration of his right to possess, both of which must be brought within one year from the date of actual entry on the land, in case of forcible entry, and from the date of last demand, in case of unlawful detainer, in the proper municipal trial court or metropolitan trial court; accion publiciana which is a plenary action for recovery of the right to possess and which should be brought in the proper regional trial court when the dispossession has lasted for more than one year; and, accion reivindicatoria or accion de reivindicacion which seeks the recovery of ownership and includes the jus utendi and the jus fruendi brought in the proper regional trial court. Accion reivindicatoria or accion de reivindicacion is thus an action whereby plaintiff alleges ownership over a parcel of land and seeks recovery of its full possession. It is different from accion interdictal or accion publiciana where plaintiff merely alleges proof of a better right to possess without claim of title. In Civil Case No. 926 Javier merely claimed a better right or prior possession over the disputed area without asserting title thereto. It should be distinguished from Civil Case No. 2203-0 where she expressly alleged ownership.

Bustos v. Court of Appeals G.R. No. 120784-85, January 24, 2001, 350 SCRA 155 Pardo, J. FACTS: Paulino Fajardo died intestate on April 2, 1957. He had four (4) children, namely: Manuela, Trinidad, Beatriz and Marcial, all surnamed Fajardo. On September 30, 1964, the heirs executed an extra-judicial partition of the estate of Paulino Fajardo. On the same date, Manuela sold her share to Moses G. Mendoza, husband of Beatriz by deed of absolute sale. At the time of the sale, there was no cadastral survey in Masantol, Pampanga. Later, the cadastre was conducted and the property involved in the partition case was specified as Lots 280, 283, 284, 1000-A and 1000-B. The share of Manuela, which was sold to Moses, includes Lot 284 of the Masantol Cadastre and Lot 284 was subdivided into Lots 284-A and 284-B. Trinidad was in physical possession of the land. She refused to surrender the land to her brother-in-law Moses G. Mendoza, despite several demands. On September 3, 1971, Moses filed with the Court of First Instance, Pampanga a complaint for partition claiming the one fourth (1/4) share of Manuela which was sold to him. During the pendency of the case for partition, Trinidad Fajardo died. On December 15, 1984, the heirs executed an extra-judicial partition of the estate of Trinidad Fajardo. On February 16, 1987, Lucio Fajardo Ignacio, son of Trinidad sold Lot 284-B to spouses Venancio Viray and Cecilia Nunga-Viray. 18

On February 8, 1989, the Regional Trial Court, Pampanga, Macabebe, Branch 55 rendered a decision in favor of Moses G. Mendoza.In the meantime, on November 6, 1989, spouses Venancio Viray and Cecilia Nunga-Viray, buyers of Lucio Ignacio's share of the property, filed with the Municipal Circuit Trial Court, Macabebe-Masantol, Pampanga an action for unlawful detainer against spouses Bustos, the buyers of Moses G. Mendoza, who were in actual possession as lessees of the husband of Trinidad, Francisco Ignacio, of the subject land. The municipal circuit trial court decided the case in favor of spouses Viray. Subsequently, the trial court issued writs of execution and demolition, but stayed when spouses Bustos filed with the regional Trial Court, Pampanga, Macabebe, Branch 55, a petition for certiorari, prohibition and injunction. On December 18, 1992, the regional trial court rendered a decision dismissing the case. On September 9, 1994, petitioners filed a motion for reconsideration; however, on June 21, 1995, the Court of Appeals denied the motion. ISSUE: Whether or not petitioners could be ejected from what is now their own land. HELD: In this case, the issue of possession is intertwined with the issue of ownership. In the unlawful detainer case, the Court of Appeals affirmed the decision of the trial court as to possession on the ground that the decision has become final and executory. This means that the petitioners may be evicted. In the accion reinvindicatoria, the Court of Appeals affirmed the ownership of petitioners over the subject land. Hence, the court declared petitioners as the lawful owners of the land. In the present case, the stay of execution is warranted by the fact that petitioners are now legal owners of the land in question and are occupants thereof. To execute the judgment by ejecting petitioners from the land that they owned would certainly result in grave injustice. Besides, the issue of possession was rendered moot when the court adjudicated ownership to the spouses Bustos by virtue of a valid deed of sale. Placing petitioners in possession of the land in question is the necessary and logical consequence of the decision declaring them as the rightful owners is possession. It follows that as owners of the subject property, petitioners are entitled to possession of the same. "An owner who cannot exercise the seven (7) "juses" or attributes of ownership-the right to possess, to use and enjoy, to abuse or consume, to accessories, to dispose or alienate, to recover or vindicate and to the fruits is a crippled owner.

Heirs of Roman Soriano v. Court of Appeals G.R. No. 128177, August 15, 2001, 363 SCRA 87 Ynares Santiago, J. FACTS: The land in dispute in this case is originally owned by Adriano Soriano who died sometime in 1947. Adriano Soriano has 7 heirs whom leased the subject parcel of land to David de Vera and Consuelo Villasista for a term of 15 years starting July 1, 1967. The lease contract states that Roman Soriano will serve as the caretaker of the said property during the period of lease. During the effectivity of the lease contract, the 19

heirs of Adriano Soriano entered into extrajudicial settlement of his estate. As a result of the settlement, the property was divided into two property, Lot No. 60052 which was assigned to Lourdes and Candido, heirs of Adriano and the heirs of Dionisia another heir of Adriano. The other property, Lot No. 8459 was assigned to Francisco, Librada, Elcociado and Roman all heirs of Adriano. The owners of Lot No. 60052 sold the lot to spouses Braulio and Aquiliana Abalos, and the owners of Lot No. 8459, except Roman also sold their shares to spouses Briones. On March 14, 1968, the de Vera spouses ousted Roman as caretaker and appointed Isidro Versoza and Vidal Versoza as his substitute. Roman filed a case for reinstatement and reliquidation against the de Vera spouses in CAR Case No. 1724-P68. On September 30, 1969, the Agrarian Court rendered a decision authorizing the ejectment of Roman. On appeal, the decision was reversed by the Court of Appeals. The deicion became final and executor. However, before it was executed, the parties entered into a post-decisional agreement wherein the de Vera spouses allowed Roman Soriano to sub-lease the property until the termination of the original lease on June 30, 1982. This agreement was approved by the CAR court in an order dated December 22, 1972. On August 16, 1976, the Abalos spouses applied for the registration of the disputed parcel of land. Roman Soriano and the Director of Lands acted as oppositors. On June 27, 1983, the Land Registration Court granted the application for registration. On April 13, 1983, after the expiration of the original lease and sub-lease in favor of Roman Soriano, the Abalos spouses filed a case for unlawful detainer against Roman Soriano, later, this case was dismissed on motion of the Abalos spouses. On July 14, 1983, Elcociado, Librada, Roman, Francisco, Lourdes, Candido and the heirs of Dionisia filed a complaint to annul the deeds of sale they executed in favor of the Abalos spouses or should the deeds be not annulled, to allow Roman, Elcociado and Librada to redeem their shares in the disputed land and to uphold Roman Sorianos possession of the fishpond portion of the property as a tenant-caretaker. After the dismissal of the case for unlawful detainer, the Abalos spouses filed on August 22, 1984, a motion for execution of the post-decisional order embodying the agreement of Roman Soriano and the de Vera spouses allowing the former to sublease the property. On October 25, 1984, Roman filed a motion to suspend hearing on the rental demanded by the Abalos spouses until after the other issues raised in his opposition to the motion for execution are resolved. The motion to suspend hearing on the issue of the rentals was denied and the trial court authorized the substitution of the de Vera spouses by the Abalos spouses. Roman Soriano's motion for reconsideration was denied on March 16, 1985. Roman filed petition for certiorari and prohibition in the Court of Appeals but the latter denied the petition, pending the denial of this petition, Roman Soriano died. Not satisfied with the decision of the Court of Appeals, the heirs of Roman Soriano brought this case in the Supreme Court.

20

ISSUE: Whether or not a winning party (ABALOS) in a land registration case can effectively eject the possessor (SORIANO) thereof, whose security of tenure rights is still pending determination before the DARAB. HELD: No. The Court held that a judgment in a land registration case cannot effectively used to oust the possessor of the land, whose security of tenure rights are still pending determination before the DARAB. There is no dispute that Abalos spouses' title over the land under litigation has been confirmed with finality. However, the declaration pertains only to ownership and does not automatically include possession, especially soin the instant case where there is a third party occupying the said parcel of land, allegedly in the concept of an agricultural tenant. Agricultural lessees are entitled to security of tenure and they have the right to work on their respective landholdings once the leasehold relationship is established. Security of tenure is a legal concession to agricultural lessees which they value as life itself ad deprivation of their landholdings is tantamount to deprivation of their only means of livelihood. The exercise of the right of ownership, then, yields to the exercise of the rights of an agricultural tenant. The Supreme Court decided to refrain from ruling whether petitioners may be dispossessed of the subject property while petitioner's status as tenant has not yet been declared by the DARAB.

Garcia v. Court of Appeals G.R. No. 133140, August 10, 1999, 312 SCRA 180 Puno, J. FACTS: Petitioner Atty. Pedro Garcia, with the consent of his wife Remedios Garcia, sold a parcel of land situated at Bel Air II Village, Makati to his daughter Maria Luisa Magpayo and her husband Luisito Magpayo. The Magpayos mortgaged the land to the Philippine Bank of Communications (PBCom) to secure a loan. The Magpayos failed to pay their loan upon its maturity, hence, the mortgage was extrajudicially foreclosed and at the public auction sale in which PBCom bought the land. The redemption period of the foreclosed mortgage expired without the Magpayos redeeming the same, hence, title over the land was consolidated in favor of PBCom. PBCom subsequently filed a petition for the issuance of a writ of possession over the land with the Regional Trial Court (RTC) of Makati. The RTC granted the petition. Upon service of the writ of possession, Maria Luisa Magpayos brother, Jose Ma. T. Garcia, who was in possession of the land, refused to honor it. Jose Garcia thereupon filed against PBCom, the Magpayos, and the RTC Sheriff the instant suit for recovery of realty and damages wherein he contended, inter alia, that at the time of the alleged sale to the Magpayo spouses, he was in possession of the property; that, when his mother Remedios Tablan Garcia died, sometime in October, 1980, he became, by operation of law, a co-owner of the property; and that, Atty. Pedro V. Garcia, at the time of the execution of the instrument in favor of the Magpayo spouses was not in possession of the subject property. 21

ISSUE: Whether or not Jose Magpayo was a co-owner of the parcel of the land in dispute. HELD: No. Possession and ownership are distinct legal concepts. Ownership exists when a thing pertaining to one person is completely subjected to his will in a manner not prohibited by law and consistent with the rights of others. Ownership confers certain rights to the owner, one of which is the right to dispose of the thing by way of sale. Atty. Pedro Garcia and his wife Remedios exercised their right to dispose of what they owned when they sold the subject property to the Magpayo spouses. On the other hand, possession is defined as the holding of a thing or the enjoyment of a right. Literally, to possess means to actually and physically occupy a thing with or without right. Possession may be had in one of two ways: possession in the concept of an owner and possession of a holder. A possessor in the concept of an owner may be the owner himself or one who claims to be so. On the other hand, one who possesses as a mere holder acknowledges in another a superior right which he believes to be ownership, whether his belief be right or wrong. The records show that petitioner Jose Garcia occupied the property not in the concept of an owner for his stay was merely tolerated by his parents. An owners act of allowing another to occupy his house, rent-free does not create a permanent and indefeasible right of possession in the latters favor. Consequently, it is of no moment that petitioner was in possession of the property at the time of the sale to the Magpayo spouses. It was not a hindrance to a valid transfer of ownership. All said, the Magpayo spouses were already the owners when they mortgaged the property to PBCom.

Rodil Enterprises, Inc. v. Court of Appeals G.R. No. 129609, November 29, 2001, 371 SCRA 79 Bellosillo, J. FACTS: Rodil Enterprises Inc. (RODIL) is the lessee of the Ides O'Racca Building (O'RACCA) since 1959 which is a property owned by the Republic of the Philippines. In 1980, Rodil entered into a sublease contract with respondents Carmen Bondoc, Teresita Bondoc-Esto, Divisoria Footwear and Chua Huay Soon, members of the Ides ORacca Building Tenants Association, Inc. On 12 September 1982 BP 233 was enacted. It authorized the sale of "former alien properties" classified as commercial and industrial, and the O'RACCA building was classified as commercial property. RODIL and Ides ORacca Building Tenants Association, Inc., offered to purchase the subject property. Pending action on the offer of RODIL to purchase the property, Director Factora of the Building Services and Real Property Management Office granted RODIL's request for another renewal of the lease contract on 23 September 1987 for another five (5) years from 1 September 1987. The renewal contract was forwarded to then Secretary Jose de Jesus of Department of General Services and Real Estate Property Management (DGSREPM) for approval. Upon recommendation of DGSREPM Rufino Banas, De Jesus disapproved the renewal contract in favour of Rodil and 22

recalled all papers signed by him regarding the subject. Secretary De Jesus likewise directed RODIL to pay its realty tax delinquency and ordered the issuance of a temporary occupancy permit to the ASSOCIATION. On 6 October 1987 RODIL filed an action for specific performance, damages and injunction with prayer for temporary restraining order before the Regional Trial Court of Manila against the REPUBLIC, De Jesus, Banas, Factora and the ASSOCIATION. De Jesus, Banas and Factora were later substituted by Secretary Fulgencio Factoran of the Department of Environment and Natural Resources (DENR) in the action for specific performance. On 31 May 1988 Factora issued Order No. 1 designating the Land Management Bureau represented by Director Abelardo Palad, Jr. as custodian of all "former alien properties" owned by the REPUBLIC. Pending the action for specific performance, RODIL signed a renewal contract with Director Palad which was approved by Secretary Factora. The renewal contract would extend the lease for ten (10) years from 1 September 1987. A supplement to the renewal contract was subsequently entered into on 25 May 1992 where rentals on the previous lease contract were increased. As a result, the action was dismissed in favour of Rodil. Rodil then filed an action for unlawful detainer against Divisoria Footwear, Bondoc, Bondoc-Esto and Chua Huay Soon. Upon appeal, the Court of Appeals declared the contracts null and void and dismissed the actions for unlawful detainer. ISSUE: Whether or not Rodil has the right to occupy the building by virtue of its lease contract with the Republic. HELD: Yes. The owner has the right to enjoy and dispose of a thing, without other limitations than those established by law. Every owner has the freedom of disposition over his property. It is an attribute of ownership, and this rule has no exception. The REPUBLIC being the owner of the disputed property enjoys the prerogative to enter into a lease contract with RODIL in the exercise of its jus disponendi. Hence, as lessor, the REPUBLIC has the right to eject usurpers of the leased property where the factual elements required for relief in an action for unlawful detainer are present. Private respondents claim that the agreements of 23 September 1987, 18 May 1992 and 25 May 1992 did not give rise to valid contracts.This is true only of the Contract of Lease entered into on 23 September 1987 which the REPUBLIC did not approve. RODIL neither alleged nor proved that such approval was made known to it. The socalled approval of the lease contract was merely stated in an internal memorandum of Secretary De Jesus addressed to Director Factora. This is evident from the fact that Secretary De Jesus, in his letter, asked Factora to duly execute a lease contract and forward it to his office for approval. The consequences of this fact are clear. The Civil Code provides that no contract shall arise unless acceptance of the contract is communicated to the offeror. Until that moment, there is no real meeting of the minds, no concurrence of offer and acceptance, hence, no contract. However, the same is not true of the contracts of 18 May 1992 and 25 May 1992. As argued by RODIL, these contracts are not proscribed by law; neither is there a law 23

prohibiting the execution of a contract with provisions that are retroactive. Where there is nothing in a contract that is contrary to law, morals, good customs, public policy or public order, the validity of the contract must be sustained. The Court of Appeals invalidated the contracts because they were supposedly executed in violation of a temporary restraining order issued by the Regional Trial Court. The appellate court however failed to note that the order restrains the REPUBLIC from awarding the lease contract only as regards respondent ASSOCIATION but not petitioner RODIL. While a temporary restraining order was indeed issued against RODIL, it was issued only on 25 May 1992 or after the assailed contracts were entered into. As correctly stated by petitioner, one cannot enjoin an act already fait accompli. Private respondents argue that the "renewal contract" cannot "renew" a void contract. However, they could cite no legal basis for this assertion. It would seem that respondents consider the renewal contract to be a novation of the earlier lease contract of 23 September 1987. However, novation is never presumed. Also, the title of a contract does not determine its nature. On the contrary, it is the specific provisions of the contract which dictate its nature. Furthermore, where a contract is susceptible of two (2) interpretations, one that would make it valid and another that would make it invalid, the latter interpretation is to be adopted. The assailed agreement of 18 May 1992, "Renewal of Contract of Lease," merely states that the term of the contract would be for ten (10) years starting 1 September 1987. This is hardly conclusive of the existence of an intention by the parties to novate the contract of 23 September 1987. Nor can it be argued that there is an implied novation for the requisite incompatibility between the original contract and the subsequent one is not present. Based on this factual milieu, the presumption of validity of contract cannot be said to have been overturned. Respondent ASSOCIATION claims that the Decision of the Office of the President declaring null and void the lease contracts of 18 May 1992 and 25 May 1992 should be counted in its favor. We do not agree. The contention does not hold water. It is wellsettled that a court's judgment in a case shall not adversely affect persons who were not parties thereto.

Isaguirre v. De Lara G.R. No. 138053, May 31, 2000, 332 SCRA 803 Gonzaga Reyes, J. FACTS: Alejandro de Lara was the original applicant-claimant for a Miscellaneous Sales Application over a parcel of land with an area of 2,342 square meters. Upon his death, Alejandro de Lara was succeeded by his wife-respondent Felicitas de Lara as claimant. On this lot stands a two-story residential-commercial apartment declared for taxation purposes in the name of respondents sons, Apolonio and Rodolfo de Lara. When Felicitas encountered financial difficulties, she approached petitioner Cornelio M. Isaguirre. On February 10, 1960, a document denominated as Deed of Sale and Special Cession of Rights and Interests was executed by Felicitas and Isaguirre, 24

whereby the former sold a 250 square meter portion of the subject lot, together with the two-story commercial and residential structure standing thereon. Sometime in May 1969, Apolonio and Rodolfo de Lara filed a complaint against petitioner for recovery of ownership and possession of the two-story building. However, petitioner filed a sales application over the subject property and was issued an OCT. Due to overlapping of title, petitioner filed an action for quieting of title. Judgment was rendered in favor of the respondents. When respondent filed a motion for execution, petitioner opposed, and alleged that he had a right of retention over the property until payment of the value of the improvements he had introduced on the property. ISSUE: Whether or not petitioner can be considered a builder in good faith with respect to the improvements he made on the property. HELD: No. The petitioner is a possessor in bad faith. Based on the factual findings from this case, it is evident that petitioner knew from the very beginning that there was really no sale and that he held respondents property as mere security for the payment of the loan obligation. Therefore, petitioner may claim reimbursement only for necessary expenses; however, he is not entitled to reimbursement for any useful expenses which he may have incurred.

25

German Management & Services, Inc. v. Court of Appeals G.R. No. 76216 and 76217, September 14, 1989, 177 SCRA 495 Fernan, J. FACTS: Spouses Cynthia Cuyegkeng Jose and Manuel Rene Jose, residents of Pennsylvania, Philadelphia, USA are the owners of a parcel of land situated in Sitio Inarawan, San Isidro, Antipolo, Rizal, with an area of 232,942 sq. M. The land was originally registered on 5 August 1948 in the Office of the Register of Deeds Rizal as OCT 19, pursuant to a Homestead Patent granted by the President of the Philippines on 27 July 1948. On 26 February 1982, the spouses Jose executed a special power of attorney authorizing German Management Services to develop their property into a residential subdivision. Consequently, the German Management obtained Development Permit 00424 from the Human Settlements Regulatory Commission for said development. Finding that part of the property was occupied by Gernale and Villeza and 20 other persons, German Management advised the occupants to vacate the premises but the latter refused. Nevertheless, German Management proceeded with the development of the subject property which included the portions occupied and cultivated by Gernale, et.al. Gernale, et.al. filed an action for forcible entry against German Management before the MTC Antipolo, Rizal, alleging that they are mountainside farmers of Sitio Inarawan who have occupied and tilled their farmholdings some 12 to 15 years prior to the promulgation of PD 27, and that they were deprived of their property without due process of law when German Management forcibly removed and destroyed the barbed wire fence enclosing their farmholdings without notice and bulldozing the rice, corn, fruit bearing trees and other crops that they planted by means of force, violence and intimidation The MTC dismissed Gernale et.al.'s complaint for forcible entry. On appeal, the RTC sustained the dismissal by the MTC. Gernale then filed a petition for review with the Court of Appeals. Said court gave due course to their petition and reversed the decisions of the MTC and the RTC. The Appellate Court held that since Gernale, et.al. were in actual possession of the property at the time they were forcibly ejected by German Management, they have a right to commence an action for forcible entry regardless of the legality or illegality of possession. German Management moved to reconsider but the same was denied by the Appellate Court. Hence, here is the present recourse. ISSUE: Whether the doctrine of self-help may be availed of when respondents refused to vacate the premises. HELD: No. The justification that the drastic action of bulldozing and destroying the crops of the prior possessor on the basis of the doctrine of self help (enunciated in Article 429 NCC) is unavailing because the such doctrine can only be exercised at the time of actual or threatened dispossession, which is absent in the present case. When possession has already been lost, the owner must resort to judicial process for the recovery of property. This is clear from Article 536 New Civil Code which provides that "in no case may possession be acquired through force or intimidation as long as there is a possessor who objects thereto. He, who believes that he has an action or right to

26

deprive another of the holding of a thing, must invoke the aid of the competent court, if the holder should refuse to deliver the thing."

Caisip v. People of the Philippines G.R. No. L-28716, November 18, 1970, 36 SCRA 17 Concepcion, C. J. FACTS: Spouses Marcelino Guevarra and Gloria Cabalag cultivated a parcel of land known as Lot 105-A of Hacienda Palico situated in Nasugbu, Batangas, the same land used to be tenanted by Cabalags father when he was still alive. Hacienda Palico is owned by Roxas y Cia, administered by Antonio Chuidian, and supervised by the overseer, Felix Caisip. Prior to the incident involved, Guevarra sought recognition as a lawful tenant of Royas y Cia from the Court of Agrarian Relations but his action was dismissed. Thereafter, Roxas y Cia filed an action against Guevarra for forcible entry with prayer that Guevarra be ejected from the premises of Lot 105-A. The Justice of the Peace of Court of Nasugbu decided in favor of Roxas y Cia and on June 6, 1959, a trouble between Cabalag and Caisip occurred regarding the cutting of sugarcane. A day later, Cabalag entered again the premises of Lot 105-A and refused to be driven out by Caisip. Due to Cabalags tenacious attitude, Caisip sought the help of the Chief of Police of Nasugbu. The Deputy Sheriff, however, informed Caisip that his request to eject Cabalag cannot be acted upon without a proper court order. Nevertheless, the Chief of Police assigned Sergeant Ignacio Rojales and Corporal Frederico Villadelrey to Haciendo Palico. On June 17, 1959, Cabalag was seen weeding a portion of Lot 105-A which was a ricefield. Caisip approached her and bade her to leave, but she refused to do so. So, Caisip went to Sgt. Rojales and Cpl. Villadelrey and brought them to Cabalag. Rojales told Cabalag to stop weeding but she insisted on her right to stay in the said lot. While in squatting position, Cabalag was grabbed by Rojales who twisted her right arm and wrested the trowel she was holding. Villadelrey held her left hand and together Rojales forcibly dragged her towards a banana plantation while Caisip stood nearby, with a drawn gun. Cabalag shouted, Ina ko po! Ina ko po! and was heard by some neighbors. Zoilo Rivera, head of the tenant organization to which Cabalag was affiliated, went with them on their way to the municipal building. Upon arrival, Cabalag was turned over by Rojales and Villadelrey to the policemen on duty, who interrogated her. But upon representations made by Rivera, she was released and allowed to go home. Cabagan then filed a complaint charging Caisip, Rojales and Villadelrey of the crime of grave coercion. The Court of First Instance of Batangas found them guilty as charged. On appeal, The Court of Appeals affirmed the trial courts decision. ISSUE: Whether or not the force employed by Caisip and others, in the exercise of his right granted by Article 429, is reasonably necessary to repel or prevent an actual or threatened unlawful physical invasion or usurpation of his property. 27

HELD: No. Caisip was not even entitled to the right granted by Article 429. This is totally inapplicable to the case, for, having been given 20 days from June 6th within which to vacate the lot, Cabalag did not, on June 17th and within said period, invades or usurps the said lot. She had merely remained in possession thereof, even though the hacienda owner may have become its co-possessor by reason of the prior order of the Justice of Peace Court of Nasugbu. Caisip and others did not repel or prevent an actual or threatened physical invasion or usurpation. They expelled Cabalag from a property which she and her husband were in possession, despite the fact that the Sheriff had explicitly authorized Guevarra and Cabalag to stay in said property up to June 26th, and had expressed the view that he could not oust them without a judicial order. It is clear, therefore, that Caisip, Rojales and Villadelrey, by means of violence, and without legal authority, had prevented the complainant from doing something not prohibited by law (weeding and being in Lot 105-A), and compelled her to do something against her will (stopping the weeding and leaving said lot), whether it be right or wrong, thereby taking the law into their hands, in violation of Article 286 of the Revised Penal Code.

People of the Philippines v. Pletcha G.R. No. 19029-CR, June 27, 1977, 22 CA Rep. 807 Sison, J. FACTS: Tito Pletcha, Jr., farmer, invoking self-help in defense of the land he inherited from his father 19 years ago against the workers of Radeco Corporation, who without court order, were constructing a fence in a hacienda allegedly leased by the corporation from a certain Lopinco. Claiming actual possession and ownership and believing that the land sought to be fenced was an integral part of the land he inherited, Pletcha asked the group to desist from fenicing pending a resurvey he proposed, but he was totally ignored, thus he fought off and prevented the workers. As a result of such resistance he was prosecuted and convicted of grave coercion by the Municipal Trial Court. Pletcha appealed the decision of the MTC with the Court of Appeals. ISSUE: Whether the appellants action is a legitimate exercise of a private citizens selfhelp. HELD: Yes. In the instant case,the usurpers possession has not yet become complete and the complainants were in the act of building a fence. Such an act constitutes force in contemplation of the law. This act of trespass justified the appellant to drive them away, even by means of bolo because they refused to listen to his appeal which is reasonable. The appellant need not rush to the court to seek redress before reasonably resisting the invasion of his property. The situation required immediate action and Art. 429 gave him the self executory mechanics of self-defense and self-reliance. The provision in Art 429 of the New Civil Code confirms the right of the appellant, an owner 28

and lawful possessor, to use reasonable force to repel an invasion or usurpation, actual, threatened or physical of his property. The principle of self-defense and the protective measures related thereto, covers not only his life, but also his liberty and property. The principle of self-help authorizes the lawful possessor to use force, not only to prevent a threatened unlawful invasion or usurpation thereof; it is a sort of self-defense. It is lawful to repel force by force. He who merely uses force to defend his possession does not possess by force. The use of such necessary force to protect propriety or possessory rights constitutes a justifying circumstance under the Penal Code.

29

Andamo v. Intermediate Appellate Court G.R. No. 74761, November 6, 1990, 191 SCRA 195 Fernan, C.J. FACTS: Petitioner spouses Emmanuel and Natividad Andamo are the owners of a parcel of land situated in Biga (Biluso) Silang, Cavite which is adjacent to that of private respondent, Missionaries of Our Lady of La Salette, Inc., a religious corporation. Within the land of respondent corporation, waterpaths and contrivances, including an artificial lake, were constructed, which allegedly inundated and eroded petitioners' land, caused a young man to drown, damaged petitioners' crops and plants, washed away costly fences, endangered the lives of petitioners and their laborers during rainy and stormy seasons, and exposed plants and other improvements to destruction. Petitioners instituted a criminal action against the officers and directors of respondent corporation, for destruction by means of inundation under Article 324 of the Revised Penal Code. Subsequently, petitioners filed a civil action against respondent corporation for damages. The trial court dismissed the civil case for lack of jurisdiction, as the criminal case which was instituted ahead of the civil case was still unresolved. The appellate court affirmed the order of the trial court. The motion for reconsideration was also denied. ISSUE: Whether a corporation, which has built through its agents, waterpaths, water conductors and contrivances within its land, thereby causing inundation and damage to an adjacent land, can be held civilly liable for damages. HELD: Yes. Petitioners' complaint sufficiently alleges that petitioners have sustained and will continue to sustain damage due to the waterpaths and contrivances built by respondent corporation. It must be stressed that the use of one's property is not without limitations. Article 431 of the Civil Code provides that "the owner of a thing cannot make use thereof in such a manner as to injure the rights of a third person." SIC UTERE TUO UT ALIENUM NON LAEDAS. Moreover, adjoining landowners have mutual and reciprocal duties which require that each must use his own land in a reasonable manner so as not to infringe upon the rights and interests of others. Although we recognize the right of an owner to build structures on his land, such structures must be so constructed and maintained using all reasonable care so that they cannot be dangerous to adjoining landowners and can withstand the usual and expected forces of nature. If the structures cause injury or damage to an adjoining landowner or a third person, the latter can claim indemnification for the injury or damage suffered. Article 2176 of the Civil Code imposes a civil liability on a person for damage caused by his act or omission constituting fault or negligence. However, responsibility for fault or negligence under the said article is entirely separate and distinct from the civil liability arising from negligence under the Penal Code. The plaintiff cannot recover damages twice for the same act or omission of the defendant. The decision is reversed and set aside.

30

Republic of the Philippines v. Court of Appeals G.R. No. L-43938, April 15, 1988, 160 SCRA 228 Cruz, J. FACTS: An application for registration of a parcel of land was filed on February 11, 1965, by Jose de la Rosa on his own behalf and on behalf of his three children. The land, situated in Tuding, Itogon, Benguet Province, was divided into 9 lots. According to the application, Lots 1-5 were sold to Jose de la Rosa and Lots 6-9 to his children by Mamaya Balbalio and Jaime Alberto, respectively, in 1964. In support of the application, both Balbalio and Alberto testified that they had acquired the subject land by virtue of prescription Balbalio claimed to have received Lots 1-5 from her father shortly after the Liberation. Alberto said he received Lots 6-9 in 1961 from his mother, Bella Alberto. She was corroborated by Felix Marcos, who recalled the earlier possession of the land by Alberto's father. Benguet opposed on the ground that the June Bug mineral claim covering Lots 1-5 was sold to it on September 22, 1934, by the successors-in-interest of James Kelly, who located the claim in September 1909 and recorded it on October 14, 1909. From the date of its purchase, Benguet had been in actual, continuous and exclusive possession of the land in concept of owner. Atok alleged that a portion of Lots 1-5 and all of Lots 6-9 were covered by the Emma and Fredia mineral claims located by Harrison and Reynolds on December 25, 1930, and recorded on January 2, 1931, in the office of the mining recorder of Baguio. These claims were purchased from these locators on November 2, 1931, by Atok, which has since then been in open, continuous and exclusive possession of the said lots. The Bureau of Forestry Development also interposed its objection, arguing that the land sought to be registered was covered by the Central Cordillera Forest Reserve under Proclamation No. 217 dated February 16, 1929. Moreover, by reason of its nature, it was not subject to alienation under the Constitutions of 1935 and 1973. The trial court denied the application, holding that the applicants had failed to prove their claim of possession and ownership of the land sought to be registered. The applicants appealed to the respondent court, which reversed the trial court and affirmed the surface rights of the de la Rosas over the land while at the same time reserving the sub-surface rights of Benguet and Atok by virtue of their mining claims. Both Benguet and Atok appealed to the Supreme Court, invoking their superior right of ownership. The Republic filed its own petition for review and reiterated its argument that neither the private respondents nor the two mining companies had any valid claim to the land because it was not alienable and registerable. ISSUE: Whether or not Benguet and Atok have a better right over the property in question. HELD: Yes. It is true that the subject property was considered forest land and included in the Central Cordillera Forest Reserve, but this did not impair the rights already vested in Benguet and Atok at that time. The perfection of the mining claim converted the property to mineral land and under the laws then in force removed it from the public domain. By such act, the locators acquired exclusive rights over the land, against even 31

the government, without need of any further act such as the purchase of the land or the obtention of a patent over it. As the land had become the private property of the locators, they had the right to transfer the same, as they did, to Benguet and Atok. It is true, as the Court of Appeals observed, that such private property was subject to the "vicissitudes of ownership," or even to forfeiture by non-user or abandonment or, as the private respondents aver, by acquisitive prescription. The Court of Appeals justified this by saying there is "no conflict of interest" between the owners of the surface rights and the owners of the sub-surface rights. Under the aforesaid ruling, the land is classified as mineral underneath and agricultural on the surface, subject to separate claims of title. However, the rights over the land are indivisible and that the land itself cannot be half agricultural and half mineral. The classification must be categorical; the land must be either completely mineral or completely agricultural. In the instant case, as already observed, the land which was originally classified as forest land ceased to be so and became mineral and completely mineral once the mining claims were perfected. As long as mining operations were being undertaken thereon, or underneath, it did not cease to be so and become agricultural, even if only partly so, because it was enclosed with a fence and was cultivated by those who were unlawfully occupying the surface. This is an application of the Regalian doctrine. If a person is the owner of agricultural land in which minerals are discovered, his ownership of such land does not give him the right to extract or utilize the said minerals without the permission of the State to which such minerals belong. Benguet and Atok have exclusive rights to the property in question by virtue of their respective mining claims which they validly acquired before the Constitution of 1935 prohibited the alienation of all lands of the public domain except agricultural lands, subject to vested rights existing at the time of its adoption. The land was not and could not have been transferred to the private respondents by virtue of acquisitive prescription, nor could its use be shared simultaneously by them and the mining companies for agricultural and mineral purposes. The decision is set aside and that of the trial court is reinstated.

32

Custodio v. Court of Appeals G.R. No. 116100, February 9, 1996, 253 SCRA 483 Regalado, J. FACTS: Pacifico Mabasa owns a parcel of land with a two-door apartment. Said property may be described to be surrounded by other immovables owned by petitioner Spouses Custodio, Spouses Santos and Rosalina Morato. From the main street P. Burgos, there are two possible passageways to Mabasas property. One of the tenants of the apartment vacated because an adobe fence was constructed thereby making the first passageway narrower in width. Ma. Cristina Santos testified that she constructed said fence for security reasons. Morato also constructed her fence and even extended it in such a way that the entire passageway was enclosed. It was then that the remaining tenants of the apartment left. Thereafter, Mabasa filed a case for the grant of an easement of right of way against petitioners. The RTC granted the easement of right of way sought by private respondent. On appeal, the CA affirmed the decision of the RTC and furthermore, ordering petitioners to pay private respondent a sum of money for damages. ISSUE: Whether the award of damages to private respondent is proper. HELD: No, the act of petitioners in constructing a fence within their lot is a valid exercise of their right as owners. Article 430 of the Civil Code provides that every owner may enclose or fence his land or tenements by means of walls, ditches, live or dead hedges, or by any other means without detriment to servitudes constituted thereon. The proper exercise of a lawful right cannot constitute a legal wrong for which an action will lie, although the act may result in damage to another. The courts can give no redress for hardship to an individual resulting from action reasonably calculated to achieve a lawful end by lawful means.

33

Abejaron v. Nabasa G.R. No. 84831, June 20, 2001, 359 SCRA 47 Puno, J. FACTS: Petitioner Abejaron avers that he is the actual and lawful possessor and claimant of a 118-square meter portion of a 175-square meter residential lot in Silway, General Santos City. Petitioner Abejaron and his family occupied the 118-square meter land. At that time, the land had not yet been surveyed. They fenced the area and built thereon a family home with nipa roofing and a small store. Petitioner later improved their abode to become a two-storey house. This house, which stands to this day, occupies a portion of Lot 1, Block 5 and a portion of the adjoining Lot 2 of the same Psu. Lot 2 belongs to petitioners' daughter, Conchita. The small store was eventually destroyed and in its stead, petitioner Abejaron another store. He later planted five coconut trees on the property. Knowing that the disputed land was public in character, petitioner declared only his house, and not the disputed land, for taxation purposes. The last two declarations state that petitioners' house stands on Lots 1 and 2, Block 5. Petitioner stated that respondent Nabasa resided on the remaining 57-square meter portion of Lot 1. Nabasa built his house about 4 meters away from petitioner Abejaron's house. Employees of the Bureau of Lands surveyed the area. Abejaron did not apply for title of the land on the belief that he could not secure title over it as it was government property. Without his knowledge and consent, Nabasa applied for and caused the titling in his name the entire Lot 1, including petitioner Abejaron's 118-square meter portion. Nabasa was issued an Original Certificate of Title pursuant to a Free Patent covering Lot 1. As the title included petitioner Abejarons portion of the lot, he filed a protest with the Bureau of Lands against Nabasa's title and application. The protest was dismissed for failure of the petitioner to attend the hearings. Petitioner Abejaron then filed an action for reconveyance with damages against respondent Nabasa before the RTC. The RTC The Regional Trial Court ruled in favor of petitioner in its reconveyance case declaring the possession and occupancy of Abejaron over 118 square meters of lot in good faith and thereby declaring the inclusion of said portion in the OCT issued in the name of Nabasa erroneous. On appeal, the CA reversed the decision of the RTC stating that the only basis for reconveyance is actual fraud which in this case was failed to be substantiated by Abejaron. Without proof of irregularity neither in the issuance of title nor in the proceedings incident thereto nor a claim that fraud intervened in the issuance of the title, the title would become indefeasible. The petitioner hence resorts to the Supreme Court. ISSUE: Whether or not petitioner has acquired title over the disputed land. HELD: An action for reconveyance of a property is the sole remedy of a landowner whose property has been wrongfully or erroneously registered in another's name after one year from the date of the decree so long as the property has not passed to an innocent purchaser for value. The action does not seek to reopen the registration proceeding and set aside the decree of registration but only purports to show that the person who secured the registration of the property in controversy is not the real owner thereof. Fraud is a ground for reconveyance. For an action for reconveyance based on 34

fraud to prosper, it is essential for the party seeking reconveyance to prove by clear and convincing evidence his title to the property and the fact of fraud. Reconveyance is a remedy granted only to the owner of the property alleged to be erroneously titled in another's name. In the case at bench, petitioner does not claim to be the owner of the disputed portion. Admittedly, what he has is only a "preferential right" to acquire ownership thereof by virtue of his actual possession since January 1947. Title to alienable public lands can be established through open, continuous, and exclusive possession for at least 30 years. Not being the owner, petitioner cannot maintain the present suit. Persons who have not obtained title to public lands could not question the titles legally issued by the State.

35

Bachrach Motor Co., Inc. v. Talisay Silay Milling Co. G.R. No. 35223, September 17, 1931, 56 Phil. 117 Romualdez, J. FACTS: On December 22, 1923, the Talisay-Silay Milling Co., Inc., was indebted to the Philippine National Bank. To secure the payment of its debt, it succeeded in inducing its planters, among whom, was Mariano Lacson Ledesma, to mortgage their land to the creditor bank. And in order to compensate those planters for the risk they were running with their property under the mortgage, the aforesaid central, by a resolution passed on that same date, i.e., December 22, 1923, undertook to credit the owners of the plantation thus mortgaged every year with a sum equal to two per centum of the debt secured according to yearly balance, the payment of the bonus being made at once, or in part from time to time, as soon as the central became free of its obligations to the aforesaid bank, and of those contracted by virtue of the contract of supervision, and had funds which might be so used, or as soon as it obtained from said bank authority to make such payment. Bachrach Motor Co., Inc. filed a complaint against the Talisay-Silay Milling Co., Inc., for the delivery of the amount P13,850 or promissory notes or other instruments or credit for that sum payable on June 30, 1930, as bonus in favor of Mariano Lacson Ledesma. The Philippine National Bank filed a third party claim alleging a preferential right to receive any amount which Mariano Lacson Ledesma might be entitled to from the Talisay-Silay Milling Co. as bonus, because that would be civil fruits of the land mortgaged to said bank by said debtor for the benefit of the central referred to, and by virtue of a deed of assignment, and praying that said central be ordered to delivered directly to the intervening bank said sum on account of the latter's credit against the aforesaid Mariano Lacson Ledesma. ISSUE: Whether or not the bonus in question is civil fruits HELD: No. The said bonus bears no immediate, but only a remote accidental relation to the land mentioned, having been granted as compensation for the risk of having subjected one's land to a lien in favor of the bank, for the benefit of the entity granting said bonus. If this bonus be income or civil fruits of anything, it is income arising from said risk, or, if one chooses, from Mariano Lacson Ledesma's generosity in facing the danger for the protection of the central, but certainly it is not civil fruits or income from the mortgaged property. Hence, the amount of the bonus, according to the resolution of the central granting it, is not based upon the value, importance or any other circumstance of the mortgaged property, but upon the total value of the debt thereby secured, according to the annual balance, which is something quite distinct from and independent of the property referred to.

Equatorial Realty Development, Inc. v. Mayfair Theater, Inc. 36

G.R. No. 136221, May 12, 2000, 370 SCRA 56 Pardo, J. FACTS: Carmelo and Bauermann, Inc. use to own a parcel of land, together with two 2-storey buildings constructed thereon. Carmelo entered into a Contract of Lease with Mayfair Theater Inc. for a period of 20 years. The lease covered a portion a portion of the second floor and mezzanine of a 2-storey building which respondent used as a movie house known as Maxim Theater. Two years later, Mayfair entered into a second Contract of Lease with of Carmelo for the lease of another portion of the latters property namely, part of the second floor of the 2-storey building and two store spaces on the ground floor and the mezzanine, on which Mayfair put up another movie house known as Miramar Theater. The contract was likewise for a period of 20 years. Both leases contained a provision granting Mayfair a right of first refusal to purchase the subject properties. However, the subject properties were sold by Carmelo to Equatorial Realty Development, Inc. without offering it first to Mayfair. Mayfair filed a Complaint before the RTC of Manila for the annulment of the Deed of Absolute Sale between Carmelo and Equatorial. The RTC rendered its decision in favour of Carmelo and Equatorial. The Court of Appeals completely reversed and set aside the judgment of the lower court. The Supreme Court denied the petition for review and rescinded the contract of sale between Carmelo and Equatorial and ordered Carmelo to allow Mayfair to buy the lots. However, Carmelo could no longer be located. Thus, following the order of execution of the trial court, Mayfair deposited with the clerk of court a quo its payment to Carmelo. The lower court issued a Deed of Reconveyance in favour of Carmelo and a Deed of Sale in favor of Mayfair. Later, Equatorial filed with the trial court an action for the collection of the sum of money against Mayfair, claiming payment of rentals or reasonable compensation for the defendants use of subject premises after its lease contract had expired. ISSUE: Whether or not Equatorial should be entitled to back rentals. HELD: No. Rescission creates the obligation to return the things which were the object of the contract, together with their fruits, and the price with its interest. It is clear the Equatorial never took actual control and possession of the property sold, in view of Mayfairs timely objection to the sale and continued actual possession of the property. Furthermore, the fact that Mayfair paid rentals to Equatorial during the litigation should not be interpreted to mean actual delivery or ispo facto recognition of Equatorials title. They were made merely to avoid imminent eviction and should not be construed as recognition of Equatorial as new owner.

37

Ignacio v. Hilario G.R. No. L-175, August 30, 1946, 76 Phil. 605 Moran, C. J. FACTS: This case concerns the ownership of a parcel of land, partly rice-land and partly residential. The lower court rendered judgment holding plaintiffs as the legal owners of the whole property but conceding to defendants the ownership of the houses and granaries built by them on the residential portion with the rights of a possessor in good faith, in accordance with article 361 of the Civil Code. Subsequently, the plaintiffs prayed for an order of execution alleging that since they chose neither to pay defendants for the buildings nor to sell to them the residential lot, said defendants should be ordered to remove the structure at their own expense and to restore plaintiffs in the possession of said lot. Defendants objected to this motion which, after hearing, was granted by Judge Natividad. Hence, this petition by defendants praying for (a) a restraint and annulment of the order of execution issued by Judge Natividad; (b) an order to compel plaintiffs to pay them the sum of P2,000 for the buildings, or sell to them the residential lot for P45; or (c), a rehearing of the case for a determination of the rights of the parties upon failure of extra-judicial settlement. ISSUE: Whether the respondent Court erred in its judgment. HELD: Yes. The Civil Code provides: ART. 361. The owner of land on which anything has been built, sown or planted in good faith, shall have the right to appropriate as his own the work, sowing or planting, after the payment of the indemnity stated in articles 453 and 454, or to oblige the one who built or planted to pay the price of the land, and the one who sowed, the proper rent. ART. 453. Necessary expenses shall be refunded to every possessor; but only the possessor in good faith may retain the thing until such expenses are made good to him. Useful expenses shall be refunded to the possessor in good faith with the same right of retention, the person who has defeated him in the possession having the option of refunding the amount of the expenses or paying the increase in value which the thing may have acquired in consequence thereof. The owner of the building erected in good faith on a land owned by another, is entitled to retain the possession of the land until he is paid the value of his building, under article 453. The owner of the land, upon the other hand, has the option, under article 361, either to pay for the building or to sell his land to the owner of the building. But he cannot, as respondents here did, refuse both to pay for the building and to sell the land and compel the owner of the building to remove it from the land where it is erected. He is entitled to another motion only when, after having chosen to sell his land, the other party fails to pay for the same.

38

The Court holds, therefore, that the order of Judge Natividad compelling defendantspetitioners to remove their buildings from the land belonging to plaintiffs-respondents only because the latter chose neither to pay for such buildings not to sell the land, is null and void, for it amends substantially the judgment sought to be executed and is, furthermore, offensive to articles 361 and 453 of the Civil Code.

Ignao v. Intermediate Appellate Court G.R. No. 72876, January 18, 1991, 193 SCRA 17 Fernan, C. J. FACTS: Petitioner Florencio Ignao and his uncles Juan Ignao and Isidro Ignao were coowners of a 534sqm land located in Cavite. Pursuant to an action for partition filed by petitioner, the CFI of Cavite directed the partition of the said land. A total of 133.5 sqm was allotted to the petitioners uncles while the remaining 266.5 was allotted to the petitioner. However, when Juan and Isidro built their houses they encroached upon a portion of land belonging to Florencio. A geodetic engineer surveyed the land and it was found out that Juan and Isidro occupied a total of 101sqm of Florencios lot. The trial court which based its decision on Article 448 of the Civil Code, ruled that Florencio should have the choice to either appropriate to himself that part of the house standing on his lot or to require Juan and Isidro to pay the price of the land. But since the first option seems to be impractical, it ordered to sell to Juan and Isidro those portions occupied by them because it is the workable solution. Upon appeal petitioner contends that Article 448 cannot be applied because they are co-owners of he subject property. However, the appellate court affirmed in toto the decision of the trial court. ISSUE: Whether or not Article 448 of the Civil Code is applicable in the case at bar. HELD: Yes. It is true that Article 448 cannot be applied where a co-owner builds upon a land owned in common. However, in the case at bar, the co-ownership has already been terminated by virtue of the partition, thus, Article 448 now applies since the builder is not anymore considered as an owner of the land where the house was built. As to the workable solution applied by the lower court, the same cannot be upheld because Article 448 clearly states that the right of choice belongs to the land owner and not upon the builder and the courts. Thus, whether it might seem impractical, the landowner may choose to appropriate the improvements.

Filipinas Colleges, Inc. v. Garcia Timbang, et. al., G.R. No. L-12812, September 29, 1989, 164 SCRA 287 Barrera, J.

39

FACTS: After appropriate proceedings, the Court of Appeals held, among other things, that Filipinas Colleges, Inc. are declared to have acquired the rights of the spouses Timbang in the questioned lots, they are ordered to pay the spouses Timbang in the amount of P15,807.90 plus such other amount which said spouses might have paid or had to pay. On the other hand, Maria Gervacio Blas was also declared to be a builder in good faith of the school building constructed in the lot in question and was entitled to be paid the amount of P19,000.00 for the same. Also, in case that Filipinas Colleges, Inc. failed to deposit the value of the land, which after liquidation was fixed at P32,859.34, within the 90-day period set by the Court, Filipinas Colleges would lose all its rights to the land and the spouses Timbang would then become the owners thereof. If that is the case, the Timbangs are ordered to make known to the court their option under Article 448 of the Civil Code whether they would appropriate the building in question, in which even they would have to pay Filipinas Colleges, Inc. the sum of P19,000.00, or would compel the latter to acquire the land and pay the price thereof. Filipinas Colleges, Inc. failed to pay the sum of P32,859.34 so the spouses Timbang made known to the court their decision that they had chosen not to appropriate the building but to compel Filipinas Colleges, Inc., for the payment of the sum of P32,859,34 which was granted by the Court. As a consequence of which, a writ of execution was issued. Meanwhile, Blas filed a motion for execution of her judgment representing the unpaid portion of the price of the house sold to Filipinas which was granted. Levy was made on the house in virtue of the writs of execution. Then, the Sheriff of Manila sold the building in public auction in favor of the spouses Timbang, as the highest bidders. Several motion were the subsequently filed before the lower court wherein the court held that: a) the Sheriff's certificate of sale covering a school building sold at public auction was null and void unless within 15 days from notice of said order spouses Timbang shall pay to Blas the sum of P5,750.00 that the spouses Timbang had bid for the building at the Sheriff's sale; b) that Filipinas is owner of 245.00/32,859.34 undivided interest in Lot No. 2-a on which the building sold in the auction sale is situated; and c) that the undivided interest of the Filipinas in the lot should be sold to satisfy the unpaid portion of the judgment in favor of Blas and against Filipinas in the amount of P8,200.00 minus the sum of P5,750.00. The spouses Timbang contends that because the builder in good faith has failed to pay the price of the land after the owners thereof exercised their option under Article 448 of the Civil Code, the builder lost his right of retention provided in Article 546 and that by operation of Article 445, the spouses Timbang as owners of the land automatically became the owners ipso facto of the school building. ISSUE: Whether or not the spouses Timbang automatically become the owners of the building upon failure of Filipinas to pay the value of the land. HELD: No. Based on Article 448 and 546 of the New Civil Code, the owner of the land has the right to choose between appropriating the building by reimbursing the builder of the value thereof or compelling the builder in good faith to pay for his land. Even this second right cannot be exercised if the value of the land is considerably more than that of the building. In addition to the right of the builder to be paid the value of his improvement, Article 546 gives him the corollary right of retention of the property until he is indemnified by the owner of the land. There is nothing in the language of these 40

two articles, 448 and 546, which would justify the conclusion of appellants that, upon the failure of the builder to pay the value of the land, when such is demanded by the land-owner, the latter becomes automatically the owner of the improvement under Article 445. The case of Bataclan vs Bernardo cannot be applied in this case in the sense that although it is true it was declared therein that in the event of the failure of the builder to pay the land after the owner thereof has chosen this alternative, the builder's right of retention provided in Article 546 is lost, nevertheless there was nothing said that as a consequence thereof, the builder loses entirely all rights over his own building. Also, in the present case, the Court of Appeals has already adjudged that appellee Blas is entitled to the payment of the unpaid balance of the purchase price of the school building. Blas is actually a lien on the school building are concerned. The order of the lower court directing the Timbang spouses, as successful bidders, to pay in cash the amount of their bid in the sum of P5,750.00 is therefore correct.

Manotok Realty v. Tecson G.R. No. L-47475 August 19, 1988, 164 SCRA 287 Gutierrez Jr., J. FACTS: Petitioner Manotok Realty filed a complaint against Nilo Madlangawa for recovery of possession with damages with the Court of First Instance of Manila. Said court rendered judgment declaring Madlangawa as a builder-possessor in good faith; ordering the company to recognize the right of Madlangawa to remain in Lot 345, Block 1, of the Clara Tambunting Subdivision until after he shall have been reimbursed by the company the sum of P7,500.00, without pronouncement as to costs. Not satisfied with the trial courts decision, petitioner appealed to the Court of Appeals and upon affirming the trial courts decision, it elevated the case to the Supreme Court. On July 13, 1977, the Supreme Court issued a resolution denying Manotoks petition for lack of merit. Petitioner then filed with the trial court (Judge Jose H. Tecson), a motion for the approval of the companys exercise of option and for satisfaction of judgment. However, Judge Tecson denied the motion for approval. Hence, this petition is filed. ISSUE: Whether or not respondent Judge Tecson can deny petitioners (landowner) motion to avail of its option. HELD: No. There is, therefore, no basis for the respondent judge to deny the petitioners motion to avail of its option to appropriate the improvements made on its property. Neither can the judge deny the issuance of a writ of execution because the private respondent was adjudged a builder in good faith or on the ground of peculiar circumstances which supervened after the institution of this case, like, for instance, the introduction of certain major repairs of and other substantial improvements because the option given by law belongs to the owner of the land. Under Article 448 of the Civil Code, the right to appropriate the works or improvements or to oblige the one who built or planted to pay the proper price of the land belongs to the owner of the land. The only 41

right given to the builder in good faith is the right of reimbursement of necessary expenses for the preservation of the land; the builder cannot compel the landowner to sell such land to the former.

Bernardo v. Bataclan G.R. No. L-44606, November 28, 1938, 66 Phil. 598 Laurel, J. FACTS: Bernardo bought a parcel of land from Samonte which was located in Cavite. In order that he may take possession and occupy the said land, he filed a case in the CFI for such purpose and the court rendered a favorable decision for Bernardo. However, when he was supposedly set in occupying the said land, he found Bataclan. He was within the premises because he was authorized by the previous owners to clear the land and make the necessary improvements he deems fit, further claiming that such authorization was granted to him ever since 1922. Since Bataclan was not a party in the first case, Bernardo filed against him a separate case. Bernardo was declared owner but the defendant was held to be a possessor in good faith for whom the work done and improvements made by him should be reimbursed. An appeal to the decision of the court was filed by both Bernardo and Bataclan. The decision was modified by lowering the price of the land from P300 to P200 per hectare. Bernardo was given 30 days to exercise his option, whether to sell the land to Bataclan or to buy the improvements from him. Bernardo chose the option which would require Bataclan to pay him the value of the land at the rate of P200 per hectare. However, Bataclan informed the court that he will not be able to pay for the price of the land. The court then gave Bataclan 30 days to pay the price of the property and after the lapse of the period, the land shall be sold in a public auction. After 30 days, the land was sold to Teodoro at a public auction, after failure of Bataclan to pay within the period the purchase price. ISSUE: Whether or not Bataclan has the right of retention over the parcel of land in question. HELD: No. Bataclan no longer has lost the right of retention. The option of the owner was already exercised where he decided that he will just allow the defendant to purchase the land such that Bataclan was to comply with the option if he wants to retain the land. From the moment that he told the courts of his inability to pay for the price of the land, he already lost his right to retain the land.

Heirs of Ramon Durano, Sr. v. Uy G.R. No. 136456 October 24, 2000, 344 SCRA 238 Gonzaga Reyes, J. FACTS: Respondents stated that sometime in August 1970 and months thereafter they 42

received mimeographed notices dated August 2, 1970 and signed by the late Ramon Durano, Sr., informing them that the lands which they are tilling and residing in, formerly owned by the Cebu Portland Cement Company (hereafter, Cepoc), had been purchased by Durano & Co., Inc. The notices also declared that the lands were needed by Durano & Co. for planting to sugar and for roads or residences, and directed respondents to immediately turn over the said lands to the representatives of the company. Simultaneously, tall bamboo poles with pennants at the tops thereof were planted in some areas of the lands and metal sheets bearing the initials RMD were nailed to posts. As early as the first week of August 1970, and even before many of the respondents received notices to vacate, men who identified themselves as employees of Durano & Co. proceeded to bulldoze the lands occupied by various respondents, destroying in their wake the plantings and improvements made by the respondents therein. On September 15, 1970, Durano & Co. sold the disputed property to petitioner Ramon Durano III, who procured the registration of these lands in his name under TCT No. T103 and TCT No. T-104. Respondents contended that the display of force and the known power and prestige of petitioners and their family restrained them from directly resisting this wanton depredation upon their property. Respondents urged the Department of Justice to conduct the preliminary investigation. The RTC found that the case preponderated in favor of respondents, who all possessed their respective portions of the property covered by TCT Nos. T-103 and T-104 thinking that they were the absolute owners thereof. A number of these respondents alleged that they inherited these properties from their parents, who in turn inherited them from their own parents. Some others came into the properties by purchase from the former occupants thereof. They and their predecessors were responsible for the plantings and improvements on the property. They were the ones who sought for the properties to be tax-declared in their respective names, and they continually paid the taxes thereto. Respondents maintained that they were unaware of anyone claiming adverse possession or ownership of these lands until the bulldozing operations in 1970. Dissatisfied, petitioners appealed the RTC decision to the Court of Appeals, which, in turn, affirmed the said decision and ordered the return of the property to all the respondents-claimants. ISSUE: Whether or not the Court of Appeals erred in its decision ordering the petitioners to return the properties to the respondents. HELD: No. The evidence shows that respondents successfully complied with all the requirements for acquisitive prescription to set in. The properties were conveyed to respondents by purchase or inheritance, and in each case the respondents were in actual, continuous, open and adverse possession of the properties. They exercised rights of ownership over the lands, including the regular payment of taxes and introduction of plantings and improvements. They were unaware of anyone claiming to 43

be the owner of these lands other than themselves until the notices of demolition in 1970 --- and at the time each of them had already completed the ten-year prescriptive period either by their own possession or by obtaining from the possession of their predecessors-in-interest. Furthermore, a purchaser of a parcel of land cannot close his eyes to facts which should put a reasonable man upon his guard, such as when the property subject of the purchase is in the possession of persons other than the seller. A buyer who could not have failed to know or discover that the land sold to him was in the adverse possession of another is a buyer in bad faith. In the case, respondents were in open possession and occupancy of the properties when Durano & Co. supposedly purchased the same from Cepoc. Petitioners made no attempt to investigate the nature of respondents possession before they ordered demolition in August 1970. In the same manner, the purchase of the property by petitioner Ramon Durano III from Durano & Co. could not be said to have been in good faith. It is not disputed that Durano III acquired the property with full knowledge of respondents occupancy thereon. There even appears to be undue haste in the conveyance of the property to Durano III, as the bulldozing operations by Durano & Co. were still underway when the deed of sale to Durano III was executed on September 15, 1970. There is not even an indication that Durano & Co. attempted to transfer registration of the property in its name before it conveyed the same to Durano III. Since petitioners knew fully well the defect in their titles, they were correctly held by the Court of Appeals to be builders in bad faith. The Civil Code provides: Art. 449. He who builds, plants or sows in bad faith on the land of another, loses what is built, planted or sown without right of indemnity. Art. 450. The owner of the land on which anything has been built, planted or sown in bad faith may demand the demolition of the work, or that the planting or sowing be removed, in order to replace things in their former condition at the expense of the person who built, planted or sowed; or he may compel the builder or planter to pay the price of the land, and the sower the proper rent. Art. 451. In the cases of the two preceding articles, the landowner is entitled to damages from the builder, planter or sower. Based on these provisions, the owner of the land has three alternative rights: (1) to appropriate what has been built without any obligation to pay indemnity therefor, or (2) to demand that the builder remove what he had built, or (3) to compel the builder to pay the value of the land. In any case, the landowner is entitled to damages under Article 451.

44

The Court sustains the return of the properties to respondents and the payment of indemnity as being in accord with the reliefs under the Civil Code.

Ballatan v. Court of Appeals G.R. No. 125683, March 2, 1999, 304 SCRA 34 Puno, J. FACTS: Ballatan, Martinez and Ling are the owners of adjacent lots in Malabon, Metro Manila. Lot No. 24, 414 square meters in area, is registered in the name of petitioners Eden Ballatan and spouses Betty Martinez and Chong Chy Ling. Lots Nos. 25 and 26 are registered in the name of respondent Gonzalo Go, Sr. On Lot No. 25, respondent Winston Go, son of Gonzalo Go, Sr., constructed his house. Adjacent to Lot No. 26 is Lot No. 27, registered in the name of respondent Li Ching Yao. In 1985, petitioner Ballatan constructed her house on Lot No. 24. During the construction, she noticed that the concrete fence and side pathway of the adjoining house of respondent Winston Go encroached on the entire length of the eastern side of her property. Her building contractor informed her that the area of her lot was actually less than that described in the title. Forthwith, Ballatan informed respondent Go of this discrepancy and his encroachment on her property. Go, however, claimed that his house, including its fence and pathway, were built within the parameters of his father's lot; and that this lot was surveyed by Engineer Jose Quedding, the authorized surveyor of the Araneta Institute of Agriculture (AIA), the owner-developer of the subdivision project. So Ballatan called the attention of the IAI and after another survey of the land, Engineer Quedding found that the lot area of petitioner Ballatan was less by few meters and that of respondent Li Ching Yao, which was three lots away, increased by two meters. Engineer Quedding declared that he made a verification survey of Lots Nos. 25 and 26 of respondents Go in 1983 and allegedly found the boundaries to have been in their proper position. He, however, could not explain the reduction in Ballatan's area since he was not present at the time respondents Go constructed their boundary walls. On 10 June 1985, petitioner Ballatan made a written demand on respondents Go to remove and dismantle their improvements on Lot No. 24 but Go refused. So Ballatan instituted against Go a civil case for recovery of possession the RTC of Malabon decided in favor of Ballatan, ordering the Go's to vacate the subject portion of Lot No. 24, demolish their improvements and pay petitioner Ballatan actual damages, attorney's fees and the costs of the suit. Go appealed. ISSUE: Whether or not Ballatan have a right of remotion. HELD: All the parties have acted in good faith so Article 448 must apply. Petitioners are ordered to exercise within thirty (30) days from finality of the decision their option to either buy the portion of respondents Go's improvement on their Lot No. 24, or sell to said respondents the portion of their land on which the improvement stands. If petitioners elect to sell the land or buy the improvement, the purchase price must be at 45

the prevailing market price at the time of payment. If buying the improvement will render respondents Go's house useless, then petitioners should sell the encroached portion of their land to respondents Go. If petitioners choose to sell the land but respondents Go are unwilling or unable to buy, then the latter must vacate the subject portion and pay reasonable rent from the time petitioners made their choice up to the time they actually vacate the premises. But if the value of the land is considerably more than the value of the improvement, then respondents Go may elect to lease the land, in which case the parties shall agree upon the terms, the lease. Should they fail to agree on said terms, the court of origin is directed to fix the terms of the lease.

46

Spouses Del Ocampo v. Abesia G.R. No. L-49219, April 15, 1998, 160 SCRA 379 Gancayco, J. FACTS: Plaintiffs spouses Concepcion Fernandez and Estanislao Del Campo and defendant Bernarda Fernandez Abesia are co-owners of parcel of land with an area of 45 square meters and divided in the proportion of 2/3 and 1/3 share each, respectively. A commissioner, who is appointed by the court, conducted a survey and recommended that the property be divided into two lots: Lot 1161 A with an area of 30 square meters for the plaintiffs and Lot 1161 B with an area of 15 square meters for the defendants. However, it was shown in the sketch plan that the house of the defendant occupied the portion with an area of 5 square meters of Lot 1161 A of plaintiffs. The parties asked the court to finally settle and adjudicate who among the parties should take possession of the 5 square meters of land. ISSUES: 1.) Whether or not Article 448 of the Civil Code, the rights of a builder in good faith, should be applied to the plaintiff-spouses Del Campo. 2.) Whether or not the house of the defendant Abesia should be removed and demolished at their expense. HELD: 1.) Yes. Article 448 of the Civil Code cannot apply where a co-owner builds, plants or sows on the land owned in common for then, he did not build, plant or sow upon land that exclusively belongs to another but of which he is a co-owner. The coowner is not a third person under the circumstances, and the situation is governed by the rules of co-ownership. However, when, as in this case, the co-ownership is terminated by the partition and it appears that the house of defendants overlaps or occupies a portion of 5 square meters of the land pertaining to plaintiffs which the defendants obviously built in good faith, then the provisions of Article 448 of the new Civil Code should apply. Manresa and Navarro Amandi agree that the said provision of the Civil Code may apply even when there was co-ownership if good faith has been established. 2.) It depends. Applying Article 448 of the Civil Code, the plaintiffs have the right to appropriate said portion of the house of defendants upon payment of indemnity to defendants as provided for in Article 546 of the Civil Code. Otherwise, the plaintiffs may oblige the defendants to pay the price of the land occupied by their house. However, if the price asked for is considerably much more than the value of the portion of the house of defendants built thereon, then the latter cannot be obliged to buy the land. The defendant shall then pay the reasonable rent to the plaintiffs upon such terms and conditions that they may agree. In case of disagreement, the trial court shall fix the terms thereof. Of course, defendants may demolish or remove the said portion of their house, at their own expense, if they so decide.

47

Pacific Farms Inc. v. Esguerra G.R. No. L-21783, November 29, 1969, 30 SCRA 684 Castro, J. FACTS: On October 1, 1956 to March 2, 1957 the Company sold and delivered lumber and construction materials to the Insular Farms Inc. which the latter used in the construction of the si buildings at its compound in Bolinao, Pangasinan, of the total procurement price of P15,000.00, the sum of P4,710.18 has not been paid. Consequently, the Company instituted a civil case to recover the unpaid balance and the court sustained their claim. The defendant sheriff levied th six buildings. The Pacific Farms, Inc. filed a suit against the Company and the sheriff asserting ownership over the levied buildings which it had acquired from the Insular Farms by virtue of absolute sale executed on March 21, 1958. Pacific prays that the judicial sale of the six buildings be declared null and void. The trial court rendered judgment annulling the levy and the certificate of sale. However, it denied the plaintiff's claim for actual and exemplary damages on the ground that it was not "prepared to find there was gross negligence or bad faith on the part of any defendants". ISSUE: Whether or not the application by analogy of the rules of accession would suffice for a just adjudication. HELD: Article 447 of the Civil Code contemplates a principal and an accessory; the land being considered the principal, and the plantings, constructions or works, the accessory. The owner of the land who in good faith - whether personally or through another makes constructions or works thereon, using materials belonging to somebody else, becomes the owner of the said materials with the obligation however of paying for their value. On the other hand, the owner of the materials is entitled to remove them, provided no substantial injury is caused to the landowner. Otherwise, he has the right to reimbursement for the value of his materials, Applying article 447 by analogy, the Court consider the buildings as the principal and the lumber and construction materials that went into their construction as the accessory. Thus the appellee, if it does own the six buildings, must bear the obligation to pay for the values of the said materials; the appellant which apparently has no desire to remove the materials, and, even if it were minded to do so, cannot remove them without necessarily damaging the buildings has the corresponding right to recover the value of the unpaid lumber and construction materials.

Pecson v. Court of Appeals G.R. No. 115814, May 26, 1995, 244 SCRA 407 Davide, Jr. J. FACTS: Pedro Pecson owned a commercial lot situated in Kamias street, Quezon City, on which he built a a four-door, two-storey apartment building. But because of failure to 48

pay realty taxes amounting to P12,000.00, the commercial lot owned was sold at a public auction. It was purchased by Nepomuceno, which later sold the same to the Nuguid spouses for P103,000 on October 12, 1983. Pecson then challenged the sale, alleging that the apartment building, contrary to the claim of the Nuguid spouses, was not included in the sale. The lower court judged in favor of Pecson, declaring that the apartment building was indeed not included in the subject sale. The Court of Appeals affirmed the same. The Spouses Nuguid then filed a motion for delivery of possession of the lot and the apartment building. The lower court ruled in favor of the private respondents, but subject to the reimbursement to Pecson of the cost of constructing the apartment building minus the rents due to the spouses (calculated at P21,000 from June 23, 1993 to September 23, 1993). With the said decision at hand, the spouses then made a move to eject Pecson and as well as the tenants residing therein. However, the spouses have yet to pay Pecson for the construction costs. ISSUE: Whether the Nuguid Spouses can eject Pecson even if reimbursement hasnt been given for the construction costs. HELD: No. The Court ruled that since the spouses still havent reimbursed Pecson for the cost of construction of the building, the latter has the right to retain the property, and along with it, the fruits of which during such possession. The court ruled that though Article 448 do not apply in the case at bar. By its clear language, Article 448 refers to a land whose ownership is claimed by two or more parties, one of whom has built some works, or sown or planted something. The building, sowing or planting may have been made in good faith or in bad faith. As in this case, since the owner himself was the one who constructed the improvement, good faith and bad faith becomes irrelevant. However, by analogy, the indemnity may be applied, considering that the primary intent of Article 448 is to avoid a state of forced coownership and that the parties agree that Articles 448 and 546 of the Civil Code are applicable and indemnity for the improvements may be paid, although they differ as to the basis of the indemnity. Since the spouses have opted to appropriate the apartment building, Pecson is thus entitled to the possession and enjoyment of the apartment building, until he is paid the proper indemnity, as well as of the portion of the lot where the building has been constructed. This is so because the right to retain the improvements while the corresponding indemnity is not paid implies the tenancy or possession in fact of the land on which it is built, planted or sown. The petitioner not having been so paid, he was entitled to retain ownership of the building and, necessarily, the income therefrom.

49

Technogas Philippines Manufacturing Corporation v. Court of Appeals G.R. No. 108894, February, 10, 1997, 268 SCRA 5 Panganiban, J. FACTS: Technogas purchased a parcel of land from Pariz Industries, Inc. In the same year, Eduardo Uy purchased the land adjacent to it. The following year, Uy bought another lot adjoining the lot of Technogas. Portions of the buildings and wall bought by Technogas together with the land from Pariz Industries are occupying a portion of Uys adjoining land. The knowledge of some encroachment was only made known to both parties after their parties of their respective parcels of land. ISSUES: 1.) Whether or not petitioner Technogas Philippines is a possessor in bad faith. 2.) Whether or not petitioner Technogas Philippines has stepped into the shoes of the seller. HELD: 1.) No. Unless one is versed in the science of surveying, no one can determine the precise extent or location of his property by merely examining his paper title. There is no question in that when Technogas purchased the land from Pariz Industries, the buildings and other structures were already in existence. Furthermore, it is not clear as to who actually built these structures but it can be assumed that the predecessor-ininterest of Technogas, Pariz Industries, did so. An article 527 of the New Civil Code presumes good faith. Since no proof exists to show that the builder built the encroaching structures in bad faith, the structures should be presumed to have been built in good faith. Good faith consists in the belief of the builder that the land he is building on is his, and his ignorance of any defect or flaw in his title. Furthermore, possession acquired in good faith does not lose this character except in case and from the moment facts exist which show that the possessor is not aware that he possesses the thing improperly or wrongfully. The good faith ceases from the moment the defects in the title are made known to the possessor, by extraneous evidence or by suit for recovery of the property of the true owner. 2.) Yes. Has been shown, contrary as to the good faith of Technogas has not been overthrown. Similarly, upon delivery of the property to Pariz Industries, as seller, to Technogas, as buyer, the latter acquired ownership of the property. Consequently, Technogas is deemed to have stepped into the shoes of the seller with regard to all the rights of ownership of the property over the immovable sold, including the right to compel Uy to exercise either of the two options under Article 448 of the New Civil Code. Thus, the landowners exercise of his option can only take place after the builder shall have to know the intrusion in short, when both parties shall have become aware of it. Only then will the occasion for exercising the option arise, for it is only then that both parties will have been aware that a problem exists with regard to their property rights.

50

Pleasantville Development Corporation v. Court of Appeals G.R. No. 79688, February 1, 1996, Panganiban, J. FACTS: On March 26, 1974, Wilson Kee on installment Lot 8 from C.T. Torres Enterprises Inc. the exclusive real estate agent of petitioner. Under the Contract to Sell on installment. Kee can exercise possession over the parcel of land even before the completion of installment payments. On January 20, 1975, Kee paid CTTEI relocation fee of Php 50.00 and another on January 27, 1975 for the preparation of lot plan. These amounts were paid by Kee before he took possession of Lot 8. After the preparation of the lot plan and a copy was presented to Kee, Zenaida Octaviano, employee of CTTEI accompanied Donnabelle Kee the wife of Wilson Kee to inspect Lot 8. Unfortuantely, Octaviano pointed Lot 9. Thereafter, Kee constructed his residence on the said Lot 9 together a store, repair shop and other improvements. Edith Robillo purchased from Pleasantville Development Corporation Lot 9. Sometime in 1975, she sold the said parcel of land, Lot 9, to Eldred Jardinico which at that time is vacant. Upon paying completely to Robillo, Jardinico secured from the Register of Deeds of Bacolod City on December 19, 1978 Transfer Certificate of Title No. 106367 in his name. It was only that time that he discovered that Wilson Kee take possession of that lot and that the same have introduced improvements to the same lot. Jardinico confronted Kee and tried to reach for an amicable settlement, but failed. On January 30, 1981, Jardinico, through his lawyer, demanded that Kee vacate Lot 9 and remove all the improvements introduced by the latter. Kee refused which made Jardinico filed with the Municipal Trial Court in Cities, Branch 3, Bacolod City a complaint for ejectment with damages against Kee. Kee, in turn filed a third-party complaint against Pleasantville Development Corporation and CTTEI. The MTCC held that the erroneous delivery was attributable to CTTEI and the Kee has no rights to Lot 9 because of the rescission made by CTTEI of their contract due to Kees failure to pay the installment. MTCC also held that Kee must pay reasonable rental for the use of Lot 9 and furthermore he cannot claim reimbursement for the improvements introduced by him. On appeal, the Regional Trial Court held that Pleasantville and CTTEI were not negligent and that Kee was in bad faith. Kee appealed directly to the Supreme Court which referred the matter to the Court of Appeals. The Appellate Court overturned the ruling of the RTC and held the Kee was a builder in good faith and the erroneous delivery was attributable to the negligence of CTTEI. Hence the instant petition filed by Pleasantville. ISSUES: 1.) Whether or not, Wilson Kee is a builder in good faith. 2.) Whether or not petitioner is liable for the acts of its agent CTTEI. HELD: 1.) Petitioner fails to persuade the Court to abandon the findings and 51

conclusions of the Court of Appeals that Kee was a builder in good faith. Good faith consists in the belief of the builder that the land he is building on is his and his ignorance of any defect or flaw in his title. And as good faith is presumed, petitioner has the burden of proving bad faith on the part of Kee. At the time he built improvements on Lot 8, Kee believed that said lot was what he bought from petitioner. He was not aware that the lot delivered to him was not Lot 8. Thus, Kee is in good faith. Petitioner failed to prove otherwise. To demonstrate Kee's bad faith, petitioner points to Kee's violation of paragraphs 22 and 26 of the Contract of Sale on Installment. It has no merit. Such violations have no bearing whatsoever on whether Kee was a builder in good faith, that is, on his state of mind at the time he built the improvements on Lot 9. These alleged violations may give rise to petitioner's cause of action against Kee under the said contract (contractual breach), but may not be the basis to negate the presumption that Kee was a builder in good faith. 2.) Yes. The rule is that the principal is responsible for the acts of the agent done within the scope of his authority, and should bear the damage caused to third persons. On the other hand, the agent who exceeds his authority is personally liable for the damage. But CTTEI was acting within its authority as the sole real estate representative of petitioner when it made the delivery to Kee, only that in so acting, it was negligent. It is this negligence that is the basis of petitioner's liability, as principal of CTTEI, per Articles 1909 and 1910 of the Civil Code. For such negligence, the petitioner should be held liable for damages. The rights of Kee and Jardinico vis-a-vis each other, as builder in good faith and owner in good faith, respectively, are regulated by law (i.e., Arts. 448, 546 and 548 of the Civil Code). It was error for the Court of Appeals to make a "slight modification" in the application of such law [by holding petitioner and CTTEI solidarily liable], on the ground of "equity".

Germiniano v. Court of Appeals G.R. No. 120303, July 24, 1996, 259 SCRA 344 Davide, Jr., J. FACTS: This is a petition for review on certiorari which has its origins in Civil Case No. 9214 of Branch 3 of the Municipal Trial Court in Cities (MTCC) in Dagupan City for unlawful detainer and damages. During the pre-trial conference, the parties agreed to confine the issues to: (1) whether there was an implied renewal of the lease which expired in November 1985; (2) whether the lessees were builders in good faith and entitled to reimbursement of the value of the house and improvements; and (3) the value of the house. On the first issue, the court held that since the petitioners' mother was no longer the owner of the lot in question at the time the lease contract was executed in 1978, in view of its acquisition by Maria Lee as early as 1972, there was no lease to speak of, much 52

less, a renewal thereof. And even if the lease legally existed, its implied renewal was not for the period stipulated in the original contract, but only on a month-to-month basis pursuant to Article 1687 of the Civil Code. The refusal of the petitioners' mother to accept the rentals starting January 1986 was then a clear indication of her desire to terminate the monthly lease. As regard the petitioners' alleged failed promise to sell to the private respondents the lot occupied by the house, the court held that such should be litigated in a proper case before the proper forum, not an ejectment case where the only issue was physical possession of the property. The court resolved the second issue in the negative, holding that Articles 448 and 546 of the Civil Code, which allow possessors in good faith to recover the value of improvements and retain the premises until reimbursed, did not apply to lessees like the private respondents, because the latter knew that their occupation of the premises would continue only during the life of the lease. Besides, the rights of the private respondents were specifically governed by Article 1678, which allow reimbursement of up to one-half of the value of the useful improvements, or removal of the improvements should the lessor refuse to reimburse. On the third issue, the court deemed as conclusive the private respondents' allegation that the value of the house and improvements was P180,000.00, there being no controverting evidence presented. On appeal by the private respondents, the RTC of Dagupan City reversed the trial court's decision. ISSUE: Whether or not Article 448 or Article 1678 of the Civil Code should apply in the instant case. HELD: In this case, both parties admit that the land in question was originally owned by the petitioners' mother. The land was allegedly acquired later by one Maria Lee by virtue of an extrajudicial foreclosure of mortgage. Lee, however, never sought a writ of possession in order that she gain possession of the property in question. The petitioners' mother therefore remained in possession of the lot. It has been said that while the right to let property is an incident of title and possession, a person may be lessor and occupy the position of a landlord to the tenant although he is not the owner of the premises let. There is no need to apply by analogy the provisions of Article 448 on indemnity as was done in Pecson vs. Court of Appeals, because the situation sought to be avoided and which would justify the application of that provision, is not present in this case. Suffice it to say, "a state of forced co-ownership" would not be created between the petitioners and the private respondents. For, as correctly pointed out by the petitioners, the right of the private respondents as lessees is governed by Article 1678 of the Civil Code which allows reimbursement to the extent of one-half of the value of the useful improvements. It must be stressed, however, that the right to indemnity under Article 1678 of the Civil Code arises only if the lessor opts to appropriate the improvements. Since the 53

petitioners refused to exercise that option the private respondents cannot compel them to reimburse the one-half value of the house and improvements. Neither can they retain the premises until reimbursement is made. The private respondents' sole right then is to remove the improvements without causing any more impairment upon the property leased than is necessary.

54

Agustin v. Intermediate Appellate Court G.R. No. 66075-76, July 5, 1990, 187 SCRA 218 Grino Aquino, J. FACTS: The Cagayan River separates the towns of Solana on the west and Tuguegarao on the east in the province of Cagayan. In 1919 the lands of the east of the river were covered by the Tuguegarao Cadastre. In 1925, OCT 5472 was issued for land east of the Cagayan River owned by Eulogio Agustin. As the years went by, the Cagayan River moved gradually eastward, depositing silt on the west bank. The shifting of the river and siltation continued until 1968. In 1950, all lands west of the river were included in the Solana Cadastre. Among these occupying lands covered by Solana Cadastre were Pablo Binayug and Maria Melad. Through the years, the Cagayan River eroded lands of the Tuguegarao Cadastre on its eastern bank among which was Agustins Lot 8457, depositing the alluvium as accretion on the land possessed by Binayug on the western bank. However, 1968, after a big flood, the Cagayan River changed its course, returned to its 1919 bed and in the process, cut across the lands of Maria Melad, Timoteo Melad, and the spouses Pablo Binayug and Geronima Ubina whose lands were transferred on the eastern, or Tuguegarao, side of the river. To cultivate those lots they had to cross the river. In April 1969, while the Melads, Binayug, Urbina and their tenants were planting corn on their lots located on the easter side of Cagayan River, Agustin, the heirs of Baldomero Langcay, Juan Langcay, and Arturo Balisi, accompanied by the mayor and some policemen of Tuguegarao, claimed the same lands as their own and drove away the Melads, Binayug and Urbina from the premises. ISSUE: Whether or not ownership of accretion is lost upon sudden and abrupt change of the river. HELD: No. The ownership of the accretion to the lands was not lost upon sudden and abrupt change of the course of the river (Cagayan River in 1968 or 1969 when it reverted to its old 1919 bed), and separated or transferred said accretions to the other side (eastern bank) of the river. Articles 459 and 463 of the New Civil Code apply to this situation. Article 459 provides that whenever the current of a river, creek or torrent segregates from an estate on its bank a known portion of land and transfer it to another estate, the owner of the land to which the segregated portion belonged retains the ownership of it, provided that he removes the same within two years. Article 463 provides that, whenever the current of a river divides itself into branches, leaving a piece of land or part thereof isolated, the owner of the land retains his ownership. He also retains it if a portion of land is separated from the estate by the current.

Cureg v. Intermediate Appellate Court G.R. No. 73465, September 7, 1989, 177 SCRA 313 Medialdea, J. 55

FACTS: On November 5, 1982, private respondents Domingo Apostol et al. filed a complaint for quieting of title against petitioners Leonida Cureg et al. The complaint alleged that private respondents, except Apostol, are the legal and/or the forced heirs of the late Domingo Gerardo, and his predecessors-in-interest have been in actual, open, peaceful and continuous possession, under a bona fide claim of ownership of a parcel of land (referred to as their motherland). Subsequently, the heirs verbally sold the motherland to Apostol. The motherland showed signs of accretion caused by the movement of the Cagayan River. When private respondents were about to cultivate their motherland together with its accretion, they were prevented by the petitioners. Petitioners alleged that the motherland claimed by the private respondents is nonexistent, that the subject land is an accretion to their registered land, and that petitioners have been in possession and cultivation of the accretion for many years now. ISSUE: Whether or not the petitioners have the better right of accretion. HELD: Yes. The petitioners are entitled to the accretion. The subject land is an alluvial deposit left by the northward movement of the Cagayan River and pursuant to Article 457 of the New Civil Code: To the owners of land adjoining the banks of river belong the accretion which they gradually receive from the effects of the current of the waters. However, the increase in the area of the petitioners land, being an accretion left by the change of course or the northward movement of the Cagayan River does not automatically become registered land just because the lot which receives such accretion is covered by a Torrens title. As such, it must also be placed under the operation of the Torrens system.

Viajar v. Court of Appeals G.R. No. 77294, December 12, 1988, 168 SCRA 405 Medialdea, J. FACTS: The spouses Ricardo and Leonor Ladrido were the owners of Lot 7511. Spouses Rosendo and Ana Te were also the registered owners of a parcel of land described in their title as Lot 7340 of the Cadastral Survey of Pototan. On 6 September 1973, Rosendo Te, with the conformity of his wife, sold this lot to Angelica F. Viajar and Celso F. Viajar for P5,000. A Torrens title was later issued in the latters names. Later, Angelica Viajar had Lot 7340 relocated and found out that the property was in the possession of Ricardo Y. Ladrido. Consequently, she demanded its return but Ladrido refused. The piece of real property which used to be Lot 7340 of the Cadastral Survey of Pototan was located in barangay Guibuanogan, Pototan, Iloilo; that at the time of the cadastral survey in 1926, Lot 7511 and Lot 7340 were separated by the Suague River; that Lot 7340 has been in the possession of Ladrido; that the area of 14,036 sq.ms., which was formerly the river bed of the Suague River per cadastral survey of 1926, has also been in the possession of Ladrido; and that the Viajars have never been in actual 56

physical possession of Lot 7340. On 15 February 1974, Angelica and Celso Viajar instituted a civil action for recovery of possession and damages against Ricardo Y. Ladrido. The trial court rendered its decision in favor of Ladrido, dismissing the complaint of Angelica and Celso Viajar with costs against them, declaring the Ladridos are entitled to the possession thereof. Not satisfied with the decision, the Viajars appealed to the Court of Appeals. The Court of Appeals affirmed the decision of the court. The Viajars filed a petition for review on certiorari. ISSUE: Whether the respondents are entitled to the land on the ground of accretion. HELD: Article 457 of the New Civil Code provides that to the owners of lands adjoining the banks of rivers belong the accretion which they gradually receive from the effects of the current of the waters." The presumption is that the change in the course of the river was gradual and caused by accretion and erosion. In the present case, the lower court correctly found that the evidence introduced by the Viajars to show that the change in the course of the Suague River was sudden or that it occurred through avulsion is not clear and convincing. The Ladridos have sufficiently established that for many years after 1926 a gradual accretion on the eastern side of Lot 7511 took place by action of the current of the Suague River so that in 1979 an alluvial deposit of 29,912 sq.ms. more or less, had been added to Lot 7511. The established facts indicate that the eastern boundary of Lot 7511 was the Suague River based on the cadastral plan. For a period of more than 40 years (before 1940 to 1980) the Suague River overflowed its banks yearly and the property of the defendant gradually received deposits of soil from the effects of the current of the river. The consequent increase in the area of Lot 7511 due to alluvion or accretion was possessed by the defendants whose tenants plowed and planted the same with corn and tobacco. The quondam river bed had been filled by accretion through the years. The land is already plain and there is no indication on the ground of any abandoned river bed. Under the law, accretion which the banks or rivers may gradually receive from the effects of the current of the waters becomes the property of the owners of the lands adjoining the banks. Therefore, the accretion to Lot 7511 which consists of Lots A and B belong to the Ladridos.

Vda. De Nazareno v. Court of Appeals G.R. No. 98045, June 26, 1996, 257 SCRA 589 Romeo, J. FACTS: The subject of this controversy is a parcel of land formed as a result of sawdust dumped into the dried-up Balacanas Creek and along the banks of the Cagayan river. Private respondents Salasalan and Rabaya leased the subject lots on which their houses stood from Antonio Nazareno, petitioners predessor-in-interest. Private respondents allegedly stopped paying rentals. As a result, Nazareno and petitioners filed a case for ejectment with the MTC of Cagayan de Oro City. The MTC rendered a decision against private respondents which was affirmed by the RTC. After several petitions for annulmentof judgment by private respondents which were all dismissed, the 57

decision of the lower court was finally enforced with the private respondents being ejected from portions of the subject lots they occupied. Before Nazareno died, he caused the approval by the Bureau of lands of the survey plan with a view to perfecting his title over the accretion area being claimed by him. The said petition was protested by private respondents. After conducting a survey of the subject land, land investigator Avelino labis recommended that the survey plan be cancelled and that private respondents be directed to file appropriate public land application covering their respective portions. Nazareno filed a motion for reconsideration with the Undersecretary of the Department of Natural Resources and OIC of the Bureau of lands Ignacio who denied the Motion. Respondent Director of lands Abelardo Palad ordered Nazareno to vacate the portions adjudicated to private respondents and remove whatever improvements they have introduced; he also ordered that private respondents be placed in possession thereof. A petitioner filed a case for annulment of the previous decisions with the RTC but was dismissed. The CA affirmed the RTC decision contending that the approved of the survey plan belongs exclusively to the Director of lands and the same shall be conclusive when approved by the Secretary of Agriculture and Natural Resources. ISSUE: Whether or not petitioners can claim ownership of the subject land by virtue of Art 457 of the Civil Code. HELD: No, accretion as a mode of acquiring property under Art 457 of the NCC requires the concurrence of the requisites mentioned in the Article. These are called rules on alluvion, which if present in a case, give to the owners of lands adjoining the banks of rivers or streams any accretion gradually received from the effects of the current of waters. The word current indicates the participation of the body of water in the flow of waters due to high and low tide. Petitioners, however, admit that the accretion was formed by the dumping of boulders, soil and other filling materials on portions of the Balacanas creek and the Cagayan River. The Bureau of lands classified the subject land as an accretion area which was formed by deposits of sawdust. Petitioners submission not having met the first and second requirements of the rules of alluvion, they cannot claim the rights of a riparian owner. The subject being public land is under the jurisdiction of the Bureau of lands, respondent Palad is authorized to exercise executive control over any form of concession, disposition and management of the lands of public dominion.

Heirs of Navarro v. Intermediate Appellate Court G.R. No. 68166, February 12, 1997, 268 SCRA 589 Hermosisima, J: FACTS: On October 3, 1946, Sinforoso Pascual, filed an application for foreshore lease covering a tract of foreshore land in Sibocon, Balanga, Bataan, having an area of approximately seventeen (17) hectares. Subsequently, petitioners' predecessor-ininterest, Emiliano Navarro, filed a fishpond application with the Bureau of Fisheries 58

covering twenty five (25) hectares of foreshore land also in Sibocon, Balanga, Bataan. Initially, such application was denied by the Director of Fisheries on the ground that the property formed part of the public domain. Sometime in the early part of 1960, Sinforoso Pascual flied an application to register and confirm his title to a parcel of land, situated in Sibocon, Balanga, Bataan, described in Plan Psu-175181 and said to have an area of 146,611 square meters. Pascual claimed that this land is an accretion to his property, situated in Barrio Puerto Rivas, Balanga, Bataan, and covered by Original Certificate of Title No. 6830. It is bounded on the eastern side by the Talisay River, on the western side by the Bulacan River, and on the northern side by the Manila Bay. The Talisay River as well as the Bulacan River flow downstream and meet at the Manila Bay thereby depositing sand and silt on Pascual's property resulting in an accretion thereon. Sinforoso Pascual claimed the accretion as the riparian owner. On March 25, 1960, the Director of Lands, represented by the Assistant Solicitor General, filed an opposition thereto stating that neither Pascual nor his predecessors-ininterest possessed sufficient title to the subject property, the same being a portion of the public domain and, therefore, it belongs to the Republic of the Philippines. ISSUE: Whether or not the land sought to be registered is accretion or foreshore land, or, whether or not said land was formed by the action of the two rivers of Talisay and Bulacan or by the action of the Manila Bay. HELD: Accretion as a mode of acquiring property under said Article 457, requires the concurrence of the following requisites: (1) that the accumulation of soil or sediment be gradual and imperceptible; (2) that it be the result of the action of the waters of the river; and (3) that the land where the accretion takes place is adjacent to the bank of the river. If the accretion were to be attributed to the action of either or both of the Talisay and Bulacan Rivers, the alluvium should have been deposited on either or both of the eastern and western boundaries of petitioners' own tract of land, not on the northern portion thereof which is adjacent to the Manila Bay. Clearly lacking, thus, is the third requisite of accretion, which is, that the alluvium is deposited on the portion of claimant's land which is adjacent to the river bank. The disputed land, thus, is an accretion not on a river bank but on a sea bank, or on what used to be the foreshore of Manila Bay which adjoined petitioners' own tract of land on the northern side. Applicant Pascual has not presented proofs to convince the Court that the land he has applied for registration is the result of the settling down on his registered land of soil, earth or other deposits so as to be rightfully be considered as an accretion [caused by the action of the two rivers]. Article 457 finds no applicability where the accretion must have been caused by action of the bay. The conclusion formed by the trial court on the basis of the aforegoing observation is that the disputed land is part of the foreshore of Manila Bay and therefore, part of the public domain. Thus, the disputed property is an accretion on a sea bank, Manila Bay 59

being an inlet or an arm of the sea; as such, the disputed property is, under Article 4 of the Spanish Law of Waters of 1866, part of the public domain.

60

Del Banco v. Intermediate Appellate Court G.R. No. 72694, December 1, 1987, 156 SCRA 55 Paras, J. FACTS: In a document executed in the Municipality of San Rafael, Bulacan, on February 11, 1859, three brothers, Benedicto Pansacola, Jose Pansacola and Manuel Pansacola (known as Fr. Manuel Pena) entered into an agreement which provided, among others: (1) That they will purchase from the Spanish Government the lands comprising the Island of Cagbalite which is located within the boundaries of the Municipality of Mauban, Province of Tayabas (now Quezon) and has an approximate area of 1,600 hectares; (2) That the lands shall be considered after the purchase as their common property; (3) That the co-ownership includes Domingo Arce and Baldomera Angulo, minors at that time represented by their father, Manuel Pansacola (Fr. Manuel Pena) who will contribute for them in the proposed purchase of the Cagbalite Island; (4) That whatever benefits may be derived from the Island shall be shared equally by the co-owners in the following proportion: Benedicto Pansacola-1/4 share; Jose Pansacola-1/4 share; and, Domingo Arce and Baldomera Angulo-2/4 shares which shall be placed under the care of their father, Manuel Pansacola (Fr. Manuel Pena). On August 14, 1866, co-owners entered into the actual possession and enjoyment of the Island purchased by them from the Spanish Government. On April 11, 1868 they agreed to modify the terms and conditions of the agreement entered into by them on February 11, 1859. About one hundred years later, on November 18, 1968, private respondents brought a special action for partition in the Court of First Instance of Quezon, under the provisions of Rule 69 of the Rules of Court, including as parties the heirs and successors-ininterest of the co-owners of the Cagbalite Island in the second contract of co-ownership dated April 11, 1968. In their answer some of the defendants, petitioners herein, interposed such defenses as prescription, res judicata, exclusive ownership, estoppel and laches. After trial on the merits, the trial court rendered a decision dated November 6, 1981 dismissing the complaint. The motion for reconsideration filed by the plaintiffs, private respondents herein, was denied by the trial court in an order dated February 25, 1982. On appeal, respondent Court reversed and set aside the decision of the lower court .It also denied the motion for reconsideration and the supplement to motion for reconsideration filed by private respondents, in its resolution dated October 15, 1983. ISSUES: 1.) Whether or not Cagbalite Island is still undivided property owned in common by the heirs and successors-in-interest of the brothers, Benedicto, Jose and Manuel Pansacola. 2.) Whether or not a prescription may run in favor of a co-owner against his co- owners or co-heirs.

61

HELD: 1.) On the first issue, there is nothing in all four agreements that suggests that actual or physical partition of the Island had really been made by either the original owners or their heirs or successors-in-interest. The agreement entered into in 1859 simply provides for the sharing of whatever benefits can be derived from the island. The agreement, in fact, states that the Island to be purchased shall be considered as their common property. In the second agreement entered in 1868 the co-owners agreed not only on the sharing proportion of the benefits derived from the Island but also on the distribution of the Island each of the brothers was allocated a 1/4 portion of the Island with the children of the deceased brother, Eustaquio Pansacola allocated a 1/4 portion and the children of Manuel Pansacola (Fr. Manuel Pena) also allocated a 1/4 portion of the Island. With the distribution agreed upon each of the co-owner is a co-owner of the whole, and in this sense, over the whole he exercises the right of dominion, but he is at the same time the sole owner of a portion, in the instant case, a 1/4 portion (for each group of co-owners) of the Island which is truly abstract, because until physical division is effected such portion is merely an Ideal share, not concretely determined (3 Manresa, Codigo Civil, 3rd Ed., page 486, cited in Lopez vs. Cuaycong, 74 Phil. 601; De la Cruz vs. Cruz, 32 SCRA 307 [1970]; Felices vs. Colegado, 35 SCRA 173 [1970],; Dultra vs. CFl 70 SCRA 465 [1976]; Gatchalian vs. Arlegui, 75 SCRA 234 [1977].) In the agreement of January 20, 1907, the heirs that were represented agreed on how the Island was to be partitioned. The agreement of April 18, 1908 which supplements that of January 20, 1907 reveals that as of the signing of the 1908 agreement no actual partition of the Island had as yet been done. The second and fourth paragraphs of the agreement speaks of a survey yet to be conducted by a certain Amadeo and a plan and description yet to be made. Virgilio Pansacola, a son of the surveyor named Amadeo who is referred to in the contract dated April 18, 1908 as the surveyor to whom the task of surveying Cagbalite Island pursuant to said agreement was entrusted, however, testified that said contracts were never implemented because nobody defrayed the expenses for surveying the same. It is not enough that the co-owners agree to subdivide the property. They must have a subdivision plan drawn in accordance with which they take actual and exclusive possession of their respective portions in the plan and titles issued to each of them accordingly (Caro vs. Court of Appeals, 113 SCRA 10 [1982]). The mechanics of actual partition should follow the procedure laid down in Rule 69 of the Rules of Court. Maganon vs. Montejo, 146 SCRA 282 [1986]). Neither can such actual possession and enjoyment of some portions of the Island by some of the petitioners herein be considered a repudiation of the co-ownership. It is undisputed that the Cagbalite Island was purchased by the original co-owners as a common property and it has not been proven that the Island had been partitioned among them or among their heirs. While there is co-ownership, a co-owner's possession of his share is co-possession which is linked to the possession of the other co-owners (Gatchalian vs. Arlegui, 75 SCRA 234 [1977]).

62

2.) On the second issue, no prescription shall run in favor of a co-owner against his coowners or co-heirs so long as he expressly or impliedly recognizes the co-ownership (Valdez vs. Olonga, 51 SCRA 71 [1973], Tero vs. Tero, 131 SCRA 100 [1984]). Coowners cannot acquire by prescription the share of the other co-owners, absent a clear repudiation of the co-ownership clearly communicated to the other co-owners. An action for partition does not prescribe. Article 403 of the Old Civil Code, now Article 497, provides that the assignees of the co-owners may take part in the partition of the common property, and Article 400 of the Old Code, now Article 494 provides that each co-owner may demand at any time the partition of the common property, a provision which implies that the action to demand partition is imprescriptible or cannot be barred by laches (Budlong vs. Pondoc, 79 SCRA 24 [1977]). An action for partition does not lie except when the co-ownership is properly repudiated by the co- owner.

63

Pardell v. Bartolome G.R. No. L-4656, November 18, 1912, 23 Phil. 450 Torres, J. FACTS: Plaintiff Vicenta Ortiz and defendant Matilde Ortiz are the duly recognized natural daughters of the spouses Miguel and Calixta who died in Vigan, Ilocos Sur. Prior to the death of their mother, she executed a will whereby Matilde and Vicenta became the heirs of all her property. Subsequently, defendants, without judicial authorization or extrajudicial agreement took over the administration and enjoyment of the properties as well as collection of the rents, fruits and products thereof. Moreover, Matilde and her husband occupied the upper storey of the house and the room of the lower floor as an office. With this, Vicenta demanded that she be given rental payments by Matilde in occupying the house since she is a co-owner of the property not occupying the same and as such is entitled to its enjoyment and/or fruits. ISSUE: Whether or not Vicenta can collect rentals from Matilde who occupies and enjoy the property alone as a co-owner. HELD: No. The law grants each co-owner the right to use the property for the purpose intended provided that the interest of the co-ownership must not be injured or prejudiced and the other co-owners must not be prevented from using it according to their rights. Matilde occupied the property owned in common in accordance with the purpose for which it is intended. Records show no proof that she neither occasioned any detriment to the interest of the community property nor prevented her sister from utilizing the said property in accordance to her right as a co-owner thereof. Matilde was excercising her right as a co-owner without being prejudicial to Vicenta who could have also occupied her property had she wanted to. Each co-owner of a property has the right pro-indiviso over the whole property and may use and enjoy the same with no other limitation than that he shall not injure the interests of his co-owners, for the reason that until a division is made, the respective part of each holder of a right as a co-owner cannot be determined and every co-owner exercises joint ownership over the pro-indiviso property in addition to his use and enjoyment of the same.

64

Caro v. Court of Appeals G.R. No. L-46001, March 25, 1982, 113 SCRA 10 Guerrero, J. FACTS: Alfredo Benito, Mario Benito and Benjamin Benito were the original co-owners of two parcels of land somewhere in Sorsogon. Sometime in 1957, Mario died. His wife, Basilia Lahorra and his father, Saturnino Benito, were subsequently appointed as joint administrators of Marios estate by the CFI of Sorsogon. On August 26, 1959, Benjamin executed a deed of absolute sale of his one-third undivided portion over said parcels of land in favor of herein petitioner, Luz Caro for the sum of 10,000. Subsequently, with the consent of Saturnino Benito and Alfredo Benito as shown in their affidavits, a subdivision title was issued to petitioner Luz Caro over the lot. Sometime in May 1966, when private respondent Basilia Lahorra learned from a pleading sent to her that petitioner Luz Caro acquired from Benjamin Benito the aforesaid one-third of the undivided share of the subject lands. She sent to petitioner thru counsel, a written offer to redeem the said one-third share. However, this offer was ignored by the petitioner. Hence, private respondent Basilia Lahorra filed a case for legal redemption and sought to prove that as joint administrator of the estate of Mario Benito, she had not been notified of the sale as required by articles 1620 and 1623 of the Civil Code. During the hearing of the case, petitioner presented the following secondary evidence to prove the service of notice of the intended sale to possible redemptioners: (1) affidavit of Benjamin Benito attesting to the fact that the possible redemptioners were formally notified in writing of his intention to sell his undivided share; (2) deposition of Saturninos widow that she received and showed the notice to husband but the latter was not interested to buy the property. The trial court ruled in favor of the petitioner. However, the decision was reversed by the CA. Hence, the case was brought to the SC. ISSUE: Whether or not co-ownership on the lots in question still exist thereby allowing private respondent Basilia Lahorra to exercise the right of legal redemption. HELD: The court held that as early as 1960, co-ownership of the parcels of land covered by TCT Nos. T-609 and T-610 was terminated when Alfredo Benito, Luz Caro and the intestate estate of Mario Benito, represented by administrators Saturnino Benito, as trustee and representative of the heirs of Mario Benito, agreed to subdivide the property. It added that an agreement of partition, though oral, is valid and consequently binding upon the parties.

65

A partition for subdivision was then filed for the purpose. This was accompanied by the affidavits of Alfredo Benito and Saturnino Benito to the effect that they agree to the segregation of the land owned in common by the three amigos. A subdivision plan was made and by common agreement Lot 1-C, with an area of 163 hectares, was ceded to petitioner, to wit, TCT no. T-4978. In addition, notwithstanding the ruling in the Caram case wherein the sale of the property took place after the partition agreement, the court therein saw no difference with respect to a conveyance which took place before the partition agreement. Regarding the contention of private respondent that she was not notified of the sale, the court ruled that since the right of legal redemption does not exist nor apply in this case because admittedly a subdivision title has already been issued in the name of the petitioner on Lot 1-C sold to her, it becomes moot and academic. It becomes unnecessary to decide whether private respondent complied with the requirements for the exercise of legal redemption under Article 1623 of the New Civil Code.

Bailon Casilao v. Court of Appeals G.R. No. 78178, April 15, 1988, 160 SCRA 738 Cortes, J. FACTS: The Roman Catholic Archbishop [sic] of Manila was the owner of a parcel of land (Lot No. 1272, Balanga Cadastre) situated in the Barrio of Puerto Rivas, Municipality of Balanga, Bataan, having an area of 3,368 sq. m., more or less covered by OCT No. 14379 of de Registry of Deeds for the province of Bataan. With respect to its rights over its properties in Bataan (inclusive of Lot No. 1272), the said church was succeeded by the Roman Catholic Bishop of San Fernando, Pampanga which was, likewise, succeeded by Catholic Bishop of Balanga registered as a corporation on 15 December 1975.Prior thereto, or on 23 August 1936, by virtue of the authority given him by the Roman Catholic Archbishop of Manila to donate a portion of Lot No. 1272, the then parish priest and administrator of all the properties of the said church in the Municipality of Balanga Bataan, Rev. Fr. Mariano Sarili, executed an Escritura De Donacion donating an area of 12.40 meters by 21.40 meters or 265.36 sq. m (the subject property) of Lot No. 1272 to Ana de los Reyes and her heirs, as a reward for her long and satisfactory service to the church. Her acceptance of the donation, as well as her possession of the subject property, is indicated in the deed of donation, which deed, for unknown reasons, was refused registration by the Register of Deeds. Six (6) years later, or in 1939, Ana de los Reyes died without issue. Nevertheless, before her death, she had given the subject property to her nephew who had been living with her, the herein defendant-appellant [private respondent]. The latter immediately took possession of the property in the concept of owner, built his house thereon and, through the years, declared the land for taxation purposes as well as paid the taxes due thereon. His possession of the subject property was never disturbed by anybody until plaintiff-

66

appellee [petitioner] filed the instant complaint against him on 5 November 1985, or more than 49 years after the deed of donation was executed. ISSUE: Whether or not petitioner is barred to recover the property by the doctrine of laches. HELD: Yes. Laches means the failure or neglect for an unreasonable and unexplained length of time, to do that which, by exercising due diligence, could or should have been done earlier; it is negligence or omission to assert a right within a reasonable time, warranting the presumption that the party entitled to assert it either has abandoned or declined to assert it. It has also been defined as such neglect or omission to assert a right taken in conjunction with the lapse of time and other circumstances causing prejudice to an adverse party, as will operate as a bar in equity. The following are the essential elements of laches: (1) Conduct on the part of the defendant, or of one under whom he claims, giving rise to the situation complained of; (2) Delay in asserting complainant's right after he had knowledge of the defendant's conduct and after he has an opportunity to sue; (3) Lack of knowledge or notice on the part of the defendant that the complainant would assert the right on which he bases his suit; and (4) Injury or prejudice to the defendant in the event relief is accorded to the complainant. 32 Under the present circumstances, all of the aforegoing elements are attendant in this case. Finally, we agree with the respondent Court of Appeals that, while petitioner is admittedly still the registered owner of the donated property, and jurisprudence is settled as to the imprescriptibility and indefeasibility of a Torrens Title, there is equally an abundance of cases in the annals of our jurisprudence where we categorically ruled that a registered landowner may lose his right to recover the possession of his registered property by reason of laches.

Roque v. Intermediate Appellate Court G.R. No. L-75886, August 30, 1988, 165 SCRA 118 Feliciano, J. FACTS: Petitioner Concepcion Roque, on 6 December 1977, filed a Complaint for "Partition with Specific Performance" (docketed as Civil Case No. 5236-M) with Branch 2 of the then Court of First Instance of Malolos against respondents Emesto Roque and the heirs of Victor Roque. In her complaint, petitioner (plaintiff below) claimed legal ownership of an undivided three-fourths (3/4) portion of Lot No. 1549, by virtue of the 27 November 1961 "Bilihan Lubos at Patuluyan" executed in her favor by Emesto Roque and Victor Roque. In support of this claim, petitioner also presented an undated and unnotarized "Kasulatang Pagkilala sa Bilihan Patuluyan ng Bahagui at Pagmamana sa Labas ng Hukuman at Paghahati-hati at Abuyan ng Bahagui" said to have been signed by the respondents in acknowledgment of the existence and validity of the Bilihan in favor of 67

petitioner. Finally, petitioner alleged that, as a co-owner of Lot No. 1549, she had a right to seek partition of the property, that she could not be compelled to remain in the coownership of the same. Respondents Ernesto Roque and the legal heirs of Victor Roque, however, refused to acknowledge petitioner's claim of ownership of any portion of Lot No. 1549 and rejected the plan to divide the land. ISSUE: Whether or not petitioner can be compelled to remain in the co-ownership. HELD: No. Article 494 of the Civil Code provides that "no co-owner shall be obliged to remain in the co-ownership" and that "each co-owner may demand at any time the partition of the thing owned in common, insofar as his share is concerned." The facts on record clearly show that petitioner Concepcion Roque had been in actual, open and continuous possession of a three-fourths (3/4) portion of Lot No. 1549 ever since execution of the "Bilihan Lubos at Patuluyan" in November of 1961. The Court notes that it was only in their Answer with Compulsory Counterclaim filed with the trial court in December of 1977 more than sixteen (16) years later that respondents first questioned the genuineness and authenticity of the "Bilihan Lubos at Patuluyan." Not once during those sixteen (16) years did respondents contest petitioner's occupation of a three-fourths (3/4) portion of Lot No. 1549. Furthermore, if indeed it is true that respondents, as they claim, are the absolute owners of the whole of Lot No. 1549, it is most unusual that respondents would have allowed or tolerated such prolonged occupation by petitioner of a major portion (3/4) of the land while they, upon the other hand, contented themselves with occupation of only a fourth thereof. This latter circumstance, coupled with the passage of a very substantial length of time during which petitioner all the while remained undisturbed and uninterrupted in her occupation and possession, places respondents here in laches: respondents may no longer dispute the existence of the co-ownership between petitioner and themselves nor the validity of petitioner's claim of a threefourths (3/4) interest in Lot No. 1549, as they are deemed, by their unreasonably long inaction, to have acquiesced in the coownership.

Delima v. Court of Appeals G. R. No. L-46296, September 24, 1991, 201 SCRA 641 Medialdea J. FACTS: Lino Delima acquired a lot from the friar lands. Later, he died, leaving as his only heirs three brothers and sisters namely: Eulalio Delima, Juanita Delima, Galileo Delima and Vicente Delima. Galileo was the caretaker of the property. He was able to execute an affidavit adjusting to himself the parcel of land and was able to secure the issuance of a Transfer Certificate of Title in his name. This prompted the heirs of his siblings to file a action for reconveyance. ISSUE: Whether or not the property is subject to prescription. 68

HELD: Yes. From the moment one of the co-owners claims that he is the absolute and exclusive owner of the properties and denies the others any share therein, the question involved is no longer one of partition but of ownership. In such case, the imprescriptibility of the action for partition can no longer be invoked or applied when one of the co-owners has adversely possessed the property as exclusive owner for a period sufficient to vest ownership by prescription. It is settled that possession by the co-owner or co-heir is that of a trutee. In order that such possession is considered adverse to the cestui que trust amounting to a repudiation of the co-ownership, the following elements must concur: 1) that the trustee has performed unequivocal acts amounting to an ouster of cestui que trust; 2) that such positive acts of repudiation had been made known to the cestui que trust; and 3) that the evidence thereon should be clear and conclusive. When the co-owner of the property executed a deed of partition and on the strength thereof, obtained a cancellation of the title in the name of their predecessor and the issuance of a new title in his name as the owner, the statute of limitations started to run for the purposes of the action instituted by the latter seeking a declaration of the existence of the co-ownership and their rights thereafter. The issuance of a new title constituted a clear act of repudiation of the trust and co-ownership.

Aguilar v. Court of Appeals G.R. No. 76351, October 29, 1993, 227 SCRA 472 Bellosillo, J. FACTS: Petitioner Virgilio and respondent Senen are brothers, and were among the seven (7) children of the late Maximiano Aguilar. In 1969, the two brothers purchased a house and lot in Paraaque where their father could spend and enjoy his remaining years in a peaceful neighborhood. Initially, the brothers agreed that Virgilio's share in the co-ownership was two-thirds while that of Senen was one-third. By virtue of a written memorandum, Virgilio and Senen agreed that henceforth their interests in the house and lot should be equal, with Senen assuming the remaining mortgage obligation of the original owners with the SSS in exchange for his possession and enjoyment of the house together with their father. Since Virgilio was then disqualified from obtaining a loan from SSS, the brothers agreed that the deed of sale would be executed and the title registered in the meantime in the name of Senen. It was further agreed that Senen would take care of their father and his needs since Virgilio and his family were staying in Cebu. After Maximiano Aguilar died in 1974, petitioner demanded from private respondent that the latter vacate the house and that the property be sold and proceeds thereof divided among them. Because of the refusal of respondent to give in to petitioner's demands, the latter filed an action to compel the sale of the house and lot so that the they could divide the proceeds between them. In his complaint, petitioner prayed that the proceeds of the sale, be divided on the basis of two-thirds (2/3) in his favor and one-third (1/3) to 69

respondent. Petitioner also prayed for monthly rentals for the use of the house by respondent after their father died. In his answer with counterclaim, respondent alleged that he had no objection to the sale as long as the best selling price could be obtained; that if the sale would be effected, the proceeds thereof should be divided equally; and, that being a co-owner, he was entitled to the use and enjoyment of the property. Rendering judgment by default against defendant, for failure to appear at pre- trial, the trial court found him and plaintiff to be co-owners of the house and lot, in equal shares on the basis of their written agreement. However, it ruled that plaintiff has been deprived of his participation in the property by defendant's continued enjoyment of the house and lot, free of rent, despite demands for rentals and continued maneuvers of defendants, to delay partition. The trial court also upheld the right of plaintiff as co-owner to demand partition. Since plaintiff could not agree to the amount offered by defendant for the former's share, the trial court held that this property should be sold to a third person and the proceeds divided equally between the parties. The CA set aside the order of the trial court. ISSUE: Whether or not petitioner may demand partition of the property. HELD: Yes. We uphold the trial court in ruling in favor of petitioner, except as to the effectivity of the payment of monthly rentals by respondent as co-owner which we here declare to commence only after the trial court ordered respondent to vacate in accordance with its order. Article 494 of the Civil Code provides that no co-owner shall be obliged to remain in the co-ownership, and that each co-owner may demand at any time partition of the thing owned in common insofar as his share is concerned. Corollary to this rule, Art. 498 of the Code states that whenever the thing is essentially, indivisible and the co-owners cannot agree that it be, allotted to one of them who shall indemnify the others, it shall be sold and its proceeds accordingly distributed. This is resorted to (1) when the right to partition the property is invoked by any of the co-owners but because of the nature of the property it cannot be subdivided or its subdivision would prejudice the interests of the co-owners, and (b) the co-owners are not in agreement as to who among them shall be allotted or assigned the entire property upon proper reimbursement of the co-owners. However, being a co-owner respondent has the right to use the house and lot without paying any compensation to petitioner, as he may use the property owned in common long as it is in accordance with the purpose for which it is intended and in a manner not injurious to the interest of the other co-owners. 9 Each co-owner of property held pro indiviso exercises his rights over the whole property and may use and enjoy the same with no other limitation than that he shall not injure the interests of his co-owners, the reason being that until a division is made, the respective share of each cannot be determined and every co-owner exercises, together with his co-participants joint ownership over the pro indiviso property, in addition to his use and enjoyment of the same. Since petitioner has decided to enforce his right in court to end the co-ownership of the house and lot and respondent has not refuted the allegation that he has been preventing the sale of the property by his continued occupancy of the premises, justice and equity demand that respondent and his family vacate the property so that the sale 70

can be effected immediately. In fairness to petitioner, respondent should pay a rental of P1,200.00 per month, with legal interest; from the time the trial court ordered him to vacate, for the use and enjoyment of the other half of the property appertaining to petitioner. When petitioner filed an action to compel the sale of the property and the trial court granted the petition and ordered the ejectment of respondent, the co-ownership was deemed terminated and the right to enjoy the possession jointly also ceased. Thereafter, the continued stay of respondent and his family in the house prejudiced the interest of petitioner as the property should have been sold and the proceeds divided equally between them. To this extent and from then on, respondent should be held liable for monthly rentals until he and his family vacate.

Tomas Claudio Memorial College v. Court of Appeals G.R. No. 124262, October 12, 1999, 316 SCRA 502 Quisimbing, J. FACTS: Juan De Castro died intestate in 1993 leaving a parcel of land located in Morong, Rizal to his heirs. Mariano De Castro one of the heirs sold the said lot to petitioner Tomas Claudio Memorial College by representing that he is the sole owner of the property. The other heirs filed an action for partition before the Regional Trial Court of Rizal alleging that the sale made by Mariano affected only his undivided share of the lot but not the shares of the other co-owners. Petitioner filed a motion to dismiss the partition for the reason that it has already been barred by prescription. The Regional Trial Court of Rizal dismissed the petitioners motion. The Court of Appeals affirmed the decision. ISSUES: 1.) Whether or not the sale affected only the undivided share of Mariano 2.) Whether or not the action to file for partition has already prescribed. HELD: 1.) Yes. The Court has consistently ruled that even if a co-owner sells the whole property as his, the sale will affect only his own share but not those of the other coowners who did not consent to the sale. The sale of the whole property by a co-owner does not make the sale null and void but it only transfers the rights to the undivided share of the co-owner who made the sale. The proper action in a case like this is not nullification nor recovery but a division or partition of the entire property. 2.) No. As to the issue on prescription, the Civil Code provides that no prescription shall lie in favor of a co-owner or co-heirs as long as he expressly or impliedly recognizes the co-ownership.

Robles v. Court of Appeals 71

GR. No. 123509, March 14, 2000, 328 SCRA 97 Panganiban, J. FACTS: Leon Robles originally owned the land which was inherited by his son Silvino Robles. The latter then took possession of the land and declared it in his name for taxation purposes. Upon his death, the same was inherited by his widow Maria dela Cruz and his children. The plaintiffs entrusted the payment of the land taxes to their coheir and half-brother, Hilario Tobles. For unknown reasons, the tax declaration of the parcel of land in the name of Silvino Robles was cancelled and transferred to one Exequiel Ballena, father of Andres Robles who is the wife of the defendant Hilario Robles. He secured a loan from the Cardona Rural Bank, Inc. which was foreclosed for failure to pay the mortgage debt wherein the defendant bank emerged as the highest bidder during the auction sale. Defendant Rural Bank sold the same to the Spouses Santos. A n action for quieting of title was filed by respondent Santos. The plaintiffs alleged that they had been in possession of the land since 1942 and it was only in 1987 that they knew about the foreclosure of the mortgage. The Court of Appeals ruled that because of the plaintiffs inaction for more than 20 years, prescription had already set in. ISSUE: Whether or not the action has prescribed in favour of Hilario Robles. HELD: Yes. Hilario effected no clear and evident repudiation of the co-ownership. It is a fundamental principle that a co-owner cannot acquire by prescription the share of the other co-owners, absent any clear repudiation of the co-ownership. In order that the title may prescribe in favor of a co-owner, the following requisites must concur: (1) the coowner has performed unequivocal acts of repudiation amounting to an ouster of the other co-owners; (2) such positive acts of repudiation have been made known to the other co-owner; and (3) the evidence thereof is clear and convincing. In the present case, Hilario did not have possession of the subject property; neither did he exclude the petitioners from the use and the enjoyment thereof, as they had indisputably shared in its fruits. Likewise, his act of entering into a mortgage contract with the bank cannot be construed to be a repudiation of the co-ownership. As absolute owner of his undivided interest in the land, he had the right to alienate his share, as he in fact did. Neither should his payment of land taxes in his name, as agreed upon by the co-owners, be construed as a repudiation of the co-ownership. The assertion that the declaration of ownership was tantamount to repudiation was belied by the continued occupation and possession of the disputed property by the petitioners as owners.

Galvez vs. Court of Appeals G.R. No. 157954, March 24, 2006 Chico Nazario, J. FACTS: Timotea F. Galvez died intestate and left a parcel of land in La Union. She left behind her children Ulpiano and petitioner Paz Galvez. Ulpiano who died before Timotea was survived by his son, private respondent, Porfirio Galvez. With regards to 72

the property of Timotea, it is supposed to pass to Paz and Porfirio. However, Porifirio was surprised to discover that Paz executed an affidavit of adjudication stating that she is the true and lawful owner of the said property. Moreover, without the knowledge and consent of Porfirio, Paz sold the property to petitioner Carlos Tam for P10,000.00. Tam thereafter filed an application for registration for said parcel of land. Subsequently, Tam sold the property to Tycoon Properties, Inc. Having knowledge of such sale, Porfirio filed a complaint for Legal Redemption with Damages and Cancellation of documents against petitioner which was affirmed by the lower court and the Court of Appeals. ISSUES: 1.) Whether or not the claim of Porfirio Galvez which is based on an implied trust has already prescribed because the action was filed 24 years after Paz Galvez repudiated the said trust? 2.) Whether or not the claim of Porfirio Galvez which is based on an implied trust is already banned by laches because he failed to assert his alleged right for almost 24 years? 3.) Whether or not Carlos Tam and Tycoon Properties are buyers in good faith and for value and has the right to rely on the face of the title? HELD: 1.) No. Article 494 of the Civil Code provides that "a prescription shall not run in favor of a co-owner or co-heir against his co-owners or co-heirs as long as he expressly or impliedly recognizes the co-ownership." It is a fundamental principle that a co-owner cannot acquire by prescription the share of the other co-owners, absent any clear repudiation of the co-ownership. Prescription, as a mode of terminating a relation of coownership, must have been preceded by repudiation (of the co-ownership). The act of repudiation, in turn, is subject to certain conditions: (1) a co-owner repudiates the coownership; (2) such an act of repudiation is clearly made known to the other co-owners; (3) the evidence thereon is clear and conclusive; and (4) he has been in possession through open, continuous, exclusive, and notorious possession of the property for the period required by law. In this case, we find that Paz Galvez effected no clear and evident repudiation of the co-ownership. The execution of the affidavit of selfadjudication does not constitute such sufficient act of repudiation as contemplated under the law as to effectively exclude Porfirio Galvez from the property. This Court has repeatedly expressed its disapproval over the obvious bad faith of a co-heir feigning sole ownership of the property to the exclusion of the other heirs essentially stating that one who acts in bad faith should not be permitted to profit from it to the detriment of others. 2.) No. On the matter of laches, it is hornbook doctrine that laches is a creation of equity and its application is controlled by equitable considerations. Laches cannot be used to defeat justice or perpetrate fraud and injustice. Neither should its application be used to prevent the rightful owners of a property from recovering what has been fraudulently registered in the name of another. The equitable remedy of laches is, therefore, unavailing in this case.

73

3.) No. As to petitioners Carlos Tam and Tycoon Properties, Inc.s claim that they are buyers in good faith, same fails to persuade. A purchaser in good faith and for value is one who buys the property without notice that some other person has a right to or interest in such property and pays its fair price before he has notice of the adverse claims and interest of another person in the same property. So it is that the "honesty of intention" which constitutes good faith implies a freedom from knowledge of circumstances which ought to put a person on inquiry. "Tam did not exert efforts to determine the previous ownership of the property in question" and relied only on the tax declarations in the name of Paz Galvez. It must be noted that Carlos Tam received a copy of the summons and the complaint on 22 September 1994. This notwithstanding, he sold the property to Tycoon Properties, Inc. on 27 September 1994. Significantly, Carlos Tam is also an owner of Tycoon Properties, Inc. to the extent of 45%. A notice of lis pendens dated 8 July 1997 filed with the Registry of Deeds of the Province of La Union was inscribed on TCT No. T- 40390. Despite the inscription, Tycoon Properties, Inc. mortgaged the land to Far East Bank and Trust Company for the sum of P11,172,600. All these attendant circumstances negate petitioners claim of good faith.

Adille vs. Court of Appeals G.R. No. L-45546, January 29, 1988 Sarmiento, J. FACTS: Felisa Alzul, who owned a parcel of lot in Albay was married twice. The first was with Bernabe Adille whom she had an only child, herein petitioner Rustico Adille. The second was with Procopio Asejo whom she had three children, herein the private respondents. It was alleged that Felisa sold the property in pacto de retro to certain 3 rd persons, for a period of repurchase being 3 years. However, she died without being able to redeem the lot. After her death but during the period of redemption, petitioner Rustico repurchased, by himself alone the said lot. Afterwards, he executed a deed of extrajudicial partition by himself. Efforts to compromise were made but failed. Thus, his halfbrothers and sisters, private respondents filed a present case of partition with accounting on the position that he was only a trustee on an implied trust when he redeemed the lot. Moreover, it turned out that one of the private respondents, Emeteria Asejo was occupying a portion. The lower court was in favor of the petitioner; however, it was reversed by the Court of Appeals. ISSUES: 1.) Whether or not a co-owner can acquire an exclusive ownership over the property held in common. 2.) Whether or not prescription has set in. HELD: 1.) No. The right of repurchase may be exercised by a co-owner with aspect to his share alone. While the records show that the petitioner redeemed the property in its entirety, shouldering the expenses therefore, that did not make him the owner of all of it. In other words, it did not put to end the existing state of co-ownership. Necessary 74

expenses may be incurred by one co-owner, subject to his right to collect reimbursement from the remaining co-owners. There is no doubt that redemption of property entails a necessary expense. Under Article 488 of the Civil Code, it provides that each co-owner shall have a right to compel the other co-owners to contribute to the expenses of preservation of the thing or right owned in common and to the taxes. Any one of the latter may exempt himself from this obligation by renouncing so much of his undivided interest as may be equivalent to his share of the expenses and taxes. No such waiver shall be made if it is prejudicial to the co-ownership. The result is that the property remains to be in a condition of co-ownership. While a vendee a retro, under Article 1613 of the Code, may not be compelled to consent to a partial redemption, the redemption by one co-heir or co-owner of the property in its totality does not vest him ownership over it. Failure on the part of all the co-owners to redeem it entitles the vendee a retro to retain the property and consolidate title thereto in his name. But the provision does not give to the redeeming co-owner the right to the entire property. It does not provide for a mode of terminating a co-ownership. Neither does the fact that the petitioner had succeeded in securing title over the parcel in his name terminate the existing co-ownership. While his half-brothers and sisters are, as we said, liable to him for reimbursement as and for their shares in redemption expenses, he cannot claim exclusive right to the property owned in common. Registration of property is not a means of acquiring ownership. It operates as a mere notice of existing title, that is, if there is one. 2.) We hold in the negative. Prescription, as a mode of terminating a relation of coownership, must have been preceded by repudiation (of the co-ownership). The act of repudiation, in turn is subject to certain conditions: (1) a co-owner repudiates the coownership; (2) such an act of repudiation is clearly made known to the other co-owners; (3) the evidence thereon is clear and conclusive, and (4) he has been in possession through open, continuous, exclusive, and notorious possession of the property for the period required by law. The instant case shows that the petitioner had not complied with these requisites. We are not convinced that he had repudiated the co-ownership; on the contrary, he had deliberately kept the private respondents in the dark by feigning sole heirship over the estate under dispute. He cannot therefore be said to have "made known" his efforts to deny the co-ownership. Moreover, one of the private respondents, Emeteria Asejo, is occupying a portion of the land up to the present; yet, the petitioner has not taken pains to eject her therefrom. As a matter of fact, he sought to recover possession of that portion Emeteria is occupying only as a counterclaim, and only after the private respondents had first sought judicial relief.

Adlawan vs. Adlawan G.R. No. 161916, January 20, 2006 Ynares Santiago, J. FACTS: Petitioner Arnelito Adlawan, the acknowledged illegitimate child of Dominador Adlawan filed an ejejctment suit against the siblings of his father, respondents Narcisa 75

and Emeterio Adlawan. Being the sole heir of Dominador, he executed an affidavit adjudicating the house and lot owned by his father. However, he alleged that out of respect and generosity to respondents, he granted their plea to occupy the subject property provided they would vacate the same should his need for the property arise. Later, when he verbally requested respondents to vacate the house and lot, they refused and filed instead an action for quieting of title. He then also filed a complaint for ejectment. In answer, the respondents, 70 and 59 years of age respectively denied that they begged petitioner to allow them to say on the property since they have been staying there since birth. They claimed that the said lot was originally registered in the name of their deceased parents, Ramon and Oligia Adlawan. Spouses Ramon and Oligia needed money to finance the renovation of their house. Since they were not qualified to obtain a loan, they transferred ownership of the lot to Dominador who was the only one in the family who had a college education. Dominador and his wife, Graciana did not disturb respondents possession of the property until they died. They also argued that even if petitioner is indeed Dominadors acknowledged illegitimate son, his right to succeed is doubtful because Dominador was survived by his wife, Graciana. ISSUE: Whether or not the petitioner can validly maintain the instant case of ejectment. HELD: No. Petitioner averred that he is an acknowledged illegitimate son and the sole heir of Dominador. However, the RTC lost sight of the fact that the theory of succession invoked by petitioner would end up proving that he is not the sole owner of the subject lot. This so because Dominador was survived not only by petitioner but also by his legal wife, Graciana, who died 10 years after the death of Dominador. By intestate succession, Graciana and petitioner became co-owners of the subject lot and house. Petitioner then contended that even granting that he is a co-owner, he can file the instant case pursuant to Article 487 of the Civil Code. This article covers all kinds of actions for the recovery of possession. It includes forcible entry and unlawful detainer (accion interdictal), recovery of possession (accion publiciana) and recovery of ownership (accion de reinvindicacion). A co-owner may bring such action without the necessity of joining all the other co-owners as co-plaintiffs because the suit is presumed to have been filed to benefit his co-owners. It should be stressed, however, that where the suit is for the benefit of the petitioner alone who claims to be the sole owner and entitled to the possession of the litigated property, the action should be dismissed. According to the renowned civilest, Professor Arturo M. Tolentino, he explained that a co-owner may bring such an action, without the necessity of joining all the other coowners as co-plaintiffs, because the suit is deemed to be instituted for the benefit of all. If the action is for the benefit of the plaintiff alone, such that he claims possession for himself and not for the co-ownership, the action will not prosper. In this case, it is not disputed that petitioner brought the suit for unlawful detainer in his name alone and for his own benefit to the exclusion of the heirs of Graciana as he even executed an affidavit of self-adjudication over the disputed property. It is clear therefore that petitioner cannot validly maintain the instant action considering that he does not recognize the co-ownership that necessarily flows from his theory of succession to the property of his father, Dominador. 76

Sumipat v. Banga G.R. No. 155810, August 13, 2004 Tinga, J. FACTS: The spouses Placida Tabo-tabo and Lauro Sumipat acquired three parcels of land. The couple was childless. Lauro Sumipat, however, sired five illegitimate children. They are the petitioners herein. Lauro executed a document denominated Deed of Absolute Transfer and/or Quit-Claim over Real Properties in favor of the petitioners. On the document, it appears that the signature of his wife, Placida which indicates that she gave her marital consent. Moreover, it was alleged that Lauro executed it when he was already very sick and bedridden that upon petitioner Lydias request, their neighbor Benjamin Rivera lifted the body of Lauro whereupon Lydia guided his hand in affixing his signature on the document. Lydia left but later returned on the same day and requested Lauros unlettered wife, Placida to sign on the said document. After Lauros death, his wife, Placida and petitioners jointly administered the properties, 50% of the produce went to his wife. As wifes share in the produce of the properties dwindled, she filed a complaint for declaration of partition disclaiming any partition in the execution of the subject document. ISSUE: Whether or not a co-ownership was formed from the said deed. HELD: No. A perusal of the deed reveals that it is actually a gratuitous disposition of property a donation although Lauro Sumipat imposed upon the petitioners the condition that he and his wife, Placida, shall be entitled to one-half (1/2) of all the fruits or produce of the parcels of land for their subsistence and support. Where the deed of donation fails to show the acceptance, or where the formal notice of the acceptance, made in a separate instrument, is either not given to the donor or else not noted in the deed of donation and in the separate acceptance, the donation is null and void. In this case, the donees acceptance of the donation is not manifested either in the deed itself or in a separate document. Hence, the deed as an instrument of donation is patently void. The Court declared that the deeds of sale questioned therein are not merely voidable but null and void ab initio as the supposed seller declared under oath that she signed the deeds without knowing what they were. The significant circumstance meant, the Court added, that her consent was not merely marred by vices of consent so as to make the contracts voidable, but that she had not given her consent at all.

77

Rizal Cement Co., Inc. v. Villareal G.R. No. L-30272, February 28, 1985, 135 SCRA 15 Cuevas, J. FACTS: Respondents are applicants for the registration of two agricultural lands located in Rizal. They presented testimonial and documentary evidence appearing that the property applied for, designated as Lot Nos. 1 and 2 of Plan Psu-147662, have a total area of 26,015 sq. m.; that these lots originally belong to one Maria Certeza; that upon her death, the property was involved in a litigation between her grandchildren and Gonzalo Certeza, and that the lots were given by the latter to Justice de Joya as the latters attorneys fees; that the lots were then sold by de Joya to Filomeno Sta. Ana, who in turn sold the same to spouses Victoriano Cervo and Ignacia Guillermo in 1939; that sometime in November 1955, the said spouses sold the lots to herein applicants as shown by a duly notarized deed of sale. The spouses Cervo declared the property for taxation purposes in the name of the wife, Ignacia Guillermo, and paid for the realty taxes thereon; that prior to the sale, the spouses Cervo had the two lots surveyed first in 1950 and then in 1955. On the other hand, oppositor (Rizal Cement Company) claims to be the owner of the subject lots, having bought the same from Maria Certeza, and to have been in continuous and adverse possession of the property since 1911. To substantiate this claim, petitioner submitted documentary evidence, one of which is a tax declaration of the said lots. The Court of First Instance denied the application for registration of respondents and ordered the issuance of a decree of registration in the name of Rizal Cement Co., after finality of said decision. On appeal, the Court of Appeals reversed and set aside the decision of the CFI. The CA denied petitioners motion for reconsideration. Hence, this petition was filed. ISSUE: Whether or not respondents had been in actual possession of the land in question. HELD: Yes. The CA gave credence to the testimony of the witnesses for respondents. As a general rule, it is provided in the Civil Code that possession is acquired by the material occupation of a thing or the exercise of a right or by the fact that it is subject to the action of our will, or by the proper acts or legal formalities established for acquiring such right. Petitioners evidence, consisting of tax receipts, tax declaration and survey plan are not conclusive and indisputable basis of ones ownership of the property in question. Assessment alone is of little value as proof of title. Mere tax declaration does not vest ownership of the property upon defendant.

Wong v. Carpio G.R. No. 50264, October 21, 1991, 203 SCRA 118 Bidin, J. FACTS: William Giger sold a parcel of land through a pacto de recto sale to Manuel Mercado. Mercado only began to harvest the coconut fruits but he never placed anyone 78

over the land to watch it. Neither did he reside in the land nor was there any hut constructed thereon to show possession. Thereafter, Ignacio Wong inspected the land to see if whether there was anyone claiming the land. After finding there was none, he bought the land from Giger. He placed workers on the land, constructed a farmhouse, and fenced the boundaries. He couldn't register the sale due to some technicalities. ISSUE: Whether or not the possession of the disputed land belongs to Ignacio Wong. HELD: It should be stressed that "possession is acquired by the material occupation of a thing or the exercise of a right, or by the fact that it is subject to the action of our will, or by the proper acts and legal formalities for acquiring such right." And that the execution of a sale thru a public instrument shall be equivalent to the delivery of the thing, unless there is stipulation to the contrary. If, however, notwithstanding the execution of the instrument, the purchaser cannot have the enjoyment and material tenancy of the thing and make use of it herself, because such tenancy and enjoyment are opposed by another, then delivery has not been effected. Applying the above pronouncements on the instant case, it is clear that possession passed from vendor William Giger to private respondent Manuel Mercado by virtue of the first sale a retro, and accordingly, the later sale a retro in favor of petitioner failed to pass the possession of the property because there is an impediment the possession exercised by private respondent. Possession as a fact cannot be recognized at the same time in two different personalities except in the cases of co-possession. Should a question arise regarding the fact of possession, the present possessor shall be preferred; if there are two possessions, the one longer in possession, if the dates of possession are the same, the one who presents a title; and if these conditions are equal, the thing shall be placed in judicial deposit pending determination of its possession or ownership through proper proceedings.

Somodio v. Court of Appeals G.R. No. 82680, August 15, 1994, 235 SCRA 307 Quiason, J. FACTS: Wilfredo Mabugat and Nicanor Somodio bought a residential lot situated at Rajah Muda, Bula, General Santos. Petitioner and Mabugat partitioned the property into two portions, with petitioner taking the western part. Immediately after the partition, petitioner took possession of his portion and planted thereon ipil-ipil trees, coconut trees and other fruit-bearing trees. In 1976, petitioner began construction of a structure with a dimension of 22-by-18 feet on his lot. His employment, however, took him to Kidapawan, North Cotabato, and he left the unfinished structure to the case of his uncle. He would visit the property every three months or on weekened when he had time. Sometime in October 1977, petitioner allowed respondent Felomino Ayco, to transfer his hut to petitioner's lot. About six years later, petitioner demanded that Ayco vacate the premises but such demand proved futile. Hence, on August 23, 1983, petitioner filed an action for unlawful detainer with damages against respondent Ayco. Meanwhile, on 79

June 26, 1983, respondent Ebenecer Purisima entered the land and constructed a house thereon. Four days later, petitioner filed against respondent Purisima a complaint for forcible entry before the same court docketed as Civil Case No. 2013-I. Said case was later consolidated with Civil Case No. 2032-II. ISSUE: Whether or not Somodio has actual possession of the property. HELD: Yes. Article 531 of the Civil Code of the Philippines provides that possession is acquired by the material occupation of a thing or the exercise of a right, or by the fact that it is subject to the action of our will, or by the proper acts and legal formalities established for acquiring such right. Petitioner took possession of the property sometime in 1974 when he planted the property to coconut trees, ipil- ipil trees and fruit trees. In 1976, he started the construction of a building on the property. It is immaterial that the building was unfinished and that he left for Kidapawan for employment reasons and visited the property only intermittently. Possession in the eyes of the law does not mean that a man has to have his feet on every square meter of ground before it can be said that he is in possession (Ramos v. Director of Lands, 39 Phil. 175 [1918]). It is sufficient that petitioner was able to subject the property to the action of his will.

Maglucot Aw v. Maglucot G.R. No. 132518, March 28, 2000, 329 SCRA 78 Kapunan, J. FACTS: Sometime in 1946 there was a prior oral agreement to tentatively partition Lot No. 1639. By virtue of this agreement, the original co-owners occupied specific portions of Lot No. 1639. It was only in 1952 when the petition to subdivide Lot No. 1639 was filed because two of the co-owners, namely Hermogenes Olis and heirs of Pascual Olis, refused to have said lot subdivided and have separate certificates of title. Significantly, after the 1952 proceedings, the parties in this case by themselves and/or through their predecessors-in-interest occupied specific portions of Lot No. 1639 in accordance with the sketch plan. Sometime in 1963, Guillermo Maglucot rented a portion of the subject lot. Subsequently, Leopoldo and Severo, both surnamed Maglucot, rented portions of subject lot in 1964 and 1969, respectively, and each paying rentals therefor. Said respondents built houses on their corresponding leased lots. They paid the rental amount of P100.00 per annum to Mrs. Ruperta Salma, who represented the heirs of Roberto Maglucot, petitioners predecessor-in-interest. In December 1992, however, said respondents stopped paying rentals claiming ownership over the subject lot alleging they had a right over the land because such was not partitioned and they were co-owners. Manglucot-Aw thus filed a complaint for recovery of possession and damages against Manglucot. ISSUE: Whether or not Manglucot-Aw may recover possession by virtue of a valid partition.

80

HELD: Yes. An order for partition is final and not interlocutory and, hence, appealable because it decides the rights of the parties upon the issue submitted. In this case, both the order of partition and the unconfirmed sketch plan are, thus, interlocutory. Nevertheless, where parties do not object to the interlocutory decree, but show by their conduct that they have assented thereto, they cannot thereafter question the decree, especially, where, by reason of their conduct, considerable expense has been incurred in the execution of the commission. Respondents in this case have occupied their respective lots in accordance with the sketch/subdivision plan. They cannot after acquiescing to the order for more than forty (40) years be allowed to question the binding effect thereof. Under the present rule, the proceedings of the commissioners without being confirmed by the court are not binding upon the parties. However, this rule does not apply in case where the parties themselves actualized the supposedly unconfirmed sketch/subdivision plan. The purpose of court approval is to give effect to the sketch/subdivision plan. In this case, the parties themselves or through their predecessors-in-interest implemented the sketch plan made pursuant to a court order for partition by actually occupying specific portions of Lot No. 1639 in 1952 and continue to do so until the present until this case was filed, clearly, the purpose of the court approval has been met. This statement is not to be taken to mean that confirmation of the commissioners may be dispensed with but only that the parties herein are estopped from raising this question by their own acts of ratification of the supposedly non-binding sketch/subdivision plan.

Cequea v. Bolante G.R. No. 137944, April 6, 2000, 330 SCRA 216 Panganiban, J. FACTS: The petitioners Fernanda Mendoza Cequea and Eduarda Apiado sought for the ownership and possession of the land occupied by the respondent Honorata Bolante. Prior to 1954, the land in Binangonan, Rizal was declared for taxation purposes in the name of Sinforoso Mendoza, the father of respondent. Sinforoso died in 1930. On the basis of an affidavit, the tax declaration in the name of Sinforoso Mendoza of the contested lot was cancelled and subsequently declared in the name of Margarito Mendoza, the father of the petitioners. Margarito and Sinforoso are brothers. During the cadastral survey, respondent Honorata is the present occupant of the land together with Miguel Mendoza, another brother of the petitioners. The trial court rendered the petitioners as the lawful owner and possessors of the land. However, the Court of Appeals reversed the decision because the genuineness and the due execution of the affidavit. It was said to be insufficient to overcome the denial of respondent and her mother. Moreover, the probative value of petitioners tax receipts and declarations paled in comparison with respondents proof of ownership of the disputed parcel. The actual, physical, exclusive and continuous possession by respondent since 1985 gave her a better title under Article 538 of the Civil Code. The petitioners contended otherwise that she came into possession through force and violence, contrary to Article 536 of the Civil Code. 81

ISSUES: 1.) Whether or not the respondent has the actual, physical, exclusive and continuous possession of the land. 2.) Whether or not tax declarations and receipts are conclusive evidence of ownership or possession. HELD: 1.) Yes. Possession by the petitioner before 1985 was not exclusive, as the respondent also acquired it before 1985. The records show that the petitioners father and brother, as well as the respondent and her mother were simultaneously in adverse possession of the land. Based on Article 538 of the Civil Code, the respondent is the preferred possessor because, benefitting from her fathers tax declaration of the subject lot since 1926, she has been in possession thereof for a longer period. On the other hand, petitioners father acquired joint possession only in 1952. 2.) No. Tax declarations and receipts are not conclusive evidence of ownership. At most, they constitute mere prima facie proof of ownership or possession of the property for which taxes have been paid. In the absence of actual public and adverse possession, the declaration of the land for tax purposes does not prove ownership. The petitioners claim of ownership of the whole parcel has no legal basis.

82

Aragon v. Insular Government G.R. No. L-6019, March 25, 1911, 19 Phil. 223 Carson, J. FACTS: The Government of the Philippine Islands, through its proper representatives, objected to the application for registration pursuant to the Land Registration Act of a small lot of parcel of land being instituted by herein petitioner, Juan Aragon on the ground that said land forms part of the public domain applying the provisions of subsection 1 of Article 339 of the old Civil Code, now Article 420, paragraph 1 of the New Civil Code which provides that the following things are property of public dominion: (1) Those intended for public use, such as roads, canals, rivers, torrents, ports and bridges constructed by the State, banks, shores, roadstead, and others of similar character. It appears, however, that possessory title over the land in question was duly registered in favor of petitioner, and that the applicant and their predecessors in interest have been in possession of the parcel of land in question, under an undisputed claim of ownership. That there are strong reasons to believe that the land in question was originally well above the ebb and flow of the tide and only in later years have the waters risen to such a height along the shores of the Bay of Manila at this point as to cover the land in question completely at high tide, though, it cannot be ascertained definitely whether it is due to changes in the current and flow of the waters in the bay, or to the gradual sinking of the land along the coast. ISSUE: Whether or not petitioner is entitled ownership over the land in question. HELD: The Court affirmed the decree entered by the lower court in favor of petitioner applying the provisions of Article 446 of the old Civil Code, Article 539 of the New Civil Code which provides that every possessor has a right to be protected in his possession; and should he be disturbed therein, he shall be protected in or restored to said possession by the means established by the laws and the Rules of Court. Corollary, a possessor may lose his possession under the circumstances provided under Article 555 of the New Civil Code, to wit: (1) By the abandonment of the thing; (2) By an assignment made to another either by onerous or gratuitous title; (3) By the destruction or total loss of the thing, or because it goes out of commerce; and (4) By the possession of another, subject to the provisions of Article 537, if the new possession has lasted longer than one year. But the real right of possession is not lost till after the lapse of ten years. The Court held that since the foregoing enumerations with respect to the loss of possession was not conclusively established by the representatives of the government, and the fact that the owners of the land in question have never intended to abandon the same, then it is just and proper to register said land in their name.

Catholic Vicar Apostolic of the Mountain Province v. Court of Appeals G.R. No. 80294, March 23, 1990, 183 SCRA 639 Gancayco, J.

83

FACTS: CA-G.R. No. 38830-R was a land registration case where petitioner and private respondents were asking for confirmation of their alleged imperfect titles to the lots in question under Section 49 (b) of the Public Land Act. In the said decision, the appellate court found that the petitioner was not entitled to confirmation of its imperfect title to Lots 2 and 3. In separate motions for reconsideration filed by private respondents Heirs of Octaviano and Heirs of Juan Valdez relating to the same decision, they also asked that said two lots be registered in their names. On August 12, 1977, the Court of Appeals denied both motions. Effectively, therefore, in the said decision the appellate court ruled that neither the petitioner nor the private respondents are entitled to the confirmation of imperfect title over said two lots. Pursuant to the said decision in CA-G.R. No. 38830-R, the two lots in question remained part of the public lands. This is the only logical conclusion when the appellate court found that neither the petitioner nor private respondents are entitled to confirmation of imperfect title over said lots. The present actions that were instituted in the Regional Trial Court by private respondents are actions for recovery of possession (accion publiciana) and not for recovery of ownership (accion reivindicatoria). ISSUE: Whether or not petitioner is entitled to the possession of the subject lots. HELD: Yes. Under Article 555 (4) of the Civil Code, it is provided that a possessor may lose his possession by the possession of another, subject to the provisions of Article 537, if the new possession has lasted longer than one year. But the real right of possession is not lost till after the lapse of ten years. In the case at bar, it is clear that the petitioner was in possession of the said property as borrower in commodatum from private respondents since 1906. However, in 1951 petitioner repudiated the trust when it declared the property for tax purposes under its name. Thus, when petitioner filed its application for registration of the said property in 1962, it had been in adverse possession of the same for at least 11 years. Hence, the action for recover of possession of said property filed by private respondents against petitioner must fail. The Court, therefore, finds that the trial court and the Court of Appeals erred in declaring the private respondents to be entitled to the possession thereof. Much less can they pretend to be owners thereof. Said lots are part of the public domain.

84

EDCA Publishing & Distributing Corp. v. Santos, G.R. No. 80298, April 26, 1990, 134 SCRA 614 Cruz, J. FACTS: Jose Cruz ordered by telephone 406 books from EDCA Publishing and Distributing Corp. (EDCA), payable on delivery. EDCA prepared the corresponding invoice and delivered the books as ordered, for which Cruz issued a check. Subsequently, Cruz sold 120 of the books to Leonor Santos who paid him after verifying the seller's ownership from the invoice he showed her. Meanwhile, EDCA having become suspicious over a second order placed by Cruz even before clearing of his first check, made inquiries with the De la Salle College where he had claimed to be a dean and was informed that there was no such person in its employ. Further, Cruz had no account with the Philippine Amanah Bank, against which he had drawn the check. EDCA went to the police, which arrested Cruz whose real name was Tomas de la Pea. EDCA sought the assistance of the police, and forced their way into the store of the Santos and threatened her with prosecution for buying stolen property. They seized the 120 books. Santos sued for recovery of the books after demand for their return was rejected by EDCA. ISSUES: 1.) Whether or not EDCA was unlawfully deprived of the books because the check issued by the impostor in payment therefor was dishonored. 2.) Whether or not EDCA had the right to cease the books that were sold to Santos. HELD: 1.) No. EDCA was not unlawfully deprived of the books. Article 559 of the Civil Code provides that the possession of movable property acquired in good faith is equivalent to a title. Nevertheless, one who has lost any movable or has been unlawfully deprived thereof, may recover it from the person in possession of the same. If the possessor of a movable lost or of which the owner has been unlawfully deprived has acquired it in good faith at a public sale, the owner cannot obtain its return without reimbursing the price paid therefor. A contract of sale is perfected once agreement is reached between the parties on the subject matter and the consideration. Ownership in the thing sold shall not pass to the buyer until full payment of the purchase only if there is a stipulation to that effect. Otherwise, the rule is that such ownership shall pass from the vendor to the vendee upon the actual or constructive delivery of the thing sold even if the purchase price has not yet been paid. Non-payment only creates a right to demand payment or to rescind the contract, or to criminal prosecution in the case of bouncing checks. But absent the stipulation above noted, delivery of the thing sold will effectively transfer ownership to the buyer who can in turn transfer it to another. 2.) No. Actual delivery of the books having been made, Cruz acquired ownership over the books which he could then validly transfer to the private respondents. The fact that he had not yet paid for them to EDCA was a matter between him and EDCA and did not impair the title to the books acquired by the Santos spouses. Therefore, EDCA was not unlawfully deprived of the books and Santos had rights over the books.

85

De Garcia v. Hon. Court of Appeals G.R. No. L-20264, January 30, 1971, 37 SCRA 129 Fernando, J. FACTS: On October 11, 1953, Angelina Guevarra, while talking to Consuelo de Garcia, recognized her ring in the finger of the latter which she lost sometime in February 1952. Guevarra asked where de Garcia bought the ring to which de Garcia answered that she bought it from her comadre. Guevarra explained to de Garcia that that ring was the very same ring stolen from her. De Garcia handed the ring to Guevarra and the ring fitted her finger. Two or three days later, at the request of Guevarra, she, her husband Lt. Col. Juan Guevara, Lt. Cementina of Pasay PD, de Garcia and her attorney proceeded to the store of Mr. Rebullida to whom they showed the ring in question. Mr. Rebullida examined the ring with the aid of high power lens and after consulting the stock card thereon, concluded that it was the very ring that plaintiff bought from him in 1947. The ring was returned to defendant who despite a written request therefor failed to deliver the ring to plaintiff. In trial, de Garcia said that she bought the ring from Mrs. Miranda who got it from Mrs. Angelita Hinahon who in turn got it from, Aling Petring who was boarding in her house. ISSUE: Whether or not de Garcias possession of the ring in good faith confers her title to the said ring. HELD: No. The controlling provision is Article 559 of the Civil Code which provides that possession of movable property acquired in good faith is equivalent to a title. Nevertheless, one who has lost any movable or has been unlawfully deprived thereof may recover it from the person in possession of the same. If the possessor of a movable lost of which the owner has been unlawfully deprived, has acquired it in good faith at a public sale, the owner cannot obtain its return without reimbursing the price paid therefor. Respondent Angelina D. Guevara, having been unlawfully deprived of the diamond ring in question, was entitled to recover it from petitioner Consuelo S. de Garcia who was found in possession of the same. The only exception the law allows is when there is acquisition in good faith of the possessor at a public sale, in which case the owner cannot obtain its return without reimbursing the price. The common law principle that where one of two innocent persons must suffer by a fraud perpetrated by the another, the law imposes the loss upon the party who, by his misplaced confidence, has enabled the fraud to be committed, cannot be applied in a case which is covered by an express provision of the new Civil Code, specifically Article 559. Between a common law principle and statutory provision, the latter must prevail in this jurisdiction. It is thus immediately apparent that there is no merit to the contention raised in the first assigned error that her possession in good faith, equivalent to title, sufficed to defeat respondent Guevara's claim. As the above cases demonstrate, even on that assumption the owner can recover the same once she can show illegal deprivation. Respondent Court of Appeals was so convinced from the evidence submitted that the owner of the ring in litigation is such respondent. 86

Dizon v. Suntay G.R. No. L-30817, September 29, 1972, 47 SCRA 160 Fernando, J. FACTS: Lourdes Suntay is the owner of a 3 carat diamond ring. She entered into a transaction with Clarita Sison, wherein said ring was delivered to the latter for sale on commission. Upon receiving the ring, the receipt was delivered to Suntay. After a lapse of a considerable amount of time, the ring was not yet returned and so Suntay demanded for its return from Sison but the latter could not comply as she had already pledged it with Dizons pawnshop for P 2,600.00. After insistent demands, Sison delivered the pawnshop ticket to Suntay. Suntay through her counsel, wrote to Dizon asking for the delivery of the ring pledged but, the latter refused. She filed an action for recovery with P 500 as attorneys fees and costs. She asked for the remedy of replevin upon filing the requisite bond pending final determination of the action. The CFI of Manila issued the writ and Suntay was able to regain possession during the pendency of the action. The lower court rendered a decision in favor of Suntay. On appeal, Dizon sought the reversal of the lower courts decision and invoking estoppel. CA affirmed the lower courts decision. SC affirmed CA decision. ISSUE: Whether or not the owner of the ring may recover its possession from the pawnshop owner. HELD: Yes. Owner of a diamond ring may recover the possession of the same from a pawnshop where another person had pledged it without authority to do so. Art. 559 of the civil code applies and the defense that the pawnshop acquired possession of the without notice of any defect in the title of the pledgor is unavailing. Neither the promptings of equity nor the mandates of moral right and natural justice come to his rescue. Dizon is engaged in a business where presumably ordinary prudence would manifest itself to ascertain whether or not an individual who is offering a jewelry by way of a pledge is entitled to do so. If no such care be taken he should be the last to complain if thereafter the right of the true owner of such jewelry should be recognized.

Ledesma v. Court of Appeals G.R. No. 86051, September 1, 1992, 213 SCRA 195 Davide, J. FACTS: Two motor vehiclesHonda Gemini and Holden Premiere Modelwere purchased from Citiwide Motors by a person who identified himself as Jojo Consunji. He bought the vehicles purportedly for his father. Upon delivery to him of the vehicles, he paid a managers check drawn against PCIB. The check though was dishonored by the bank on the ground that the checks value has been materially altered. This was 87

reported to the police authorities and it was found out that the person misrepresenting himself was actually Suarez who had a long line of criminal cases against him for his modus operandi. The Holden car was recovered after being abandoned somewhere in Quezon City. The Honda on the other hand, was discovered to be sold to Ledesma. Ledesma averred he purchased the vehicle in good faith from one Neyra, as evidenced by his certificate of registration. Citiwide Motors was able to recover. ISSUE: Whether or not CITIWIDE MOTORS has been unlawfully deprived. HELD: No. There was a perfected unconditional contract of sale between Citiwide Motors and Suarez. The subsequent dishonor of the check merely amounted to failure of consideration which doesn't render a contract of sale void, but merely allows the prejudiced party to sue for specific performance or rescission of the sale. This being the case, Citiwide motors wasn't unlawfully deprived of the property. It is thus not entitled to the return of the vehicle from Ledesma who bought the property in good faith and for consideration.

88

Azarcon and Abobo v. Eusebio G.R. No. L-11977, April 29, 1959, 105 SCRA 569 Labrador, J. FACTS: Victor Eusebio had a dispute over a parcel of land with Leonardo Azarcon, Manuel Azarcon and Esteban Abobo. Eusebio filed a lease application for a parcel of land, a portion thereof was occupied by Azarcon et al. under a homestead application. Before the dispute could be settled, Eusebio filed a complaint in the CFI of Nueva Ecija, alleging that he had acquired a big parcel of land by lease from the Bureau of Lands, and that while he was in possession thereof, Azarcon et al. occupied a portion. The trial court ruled in favor of Eusebio, and a writ of execution ordering Azarcon et al. to restore possession of the land to Eusebio was issued on October 3, 1955. However, in spite of the receipt of the notice of writ of execution, Azarcon et al. nevertheless entered the land to gather palay which was then pending harvest. ISSUE: Whether or not Azarcon and Abobo are entitled to the pending fruits of the land. HELD: Yes. While the court order of October 3, 1955 ordered them to move out of the premises, it did not prohibit them from gathering the crop then existing thereon. Under the law, a person, who is in possession and who is being ordered to leave a parcel of land while products thereon are in pending harvests, has the right to a part of the net harvest, as expressly provided by Article 545 of the Civil Code. Hence, as the order of execution did not expressly prohibit Azarcon et al. from gathering the pending fruits, which fruits were the result of their possession and cultivation of the land, it cannot be said that they committed an act which is clear violation of the courts order.

Cordero v. Cabral G.R. No. L-36789, July 25, 1983, 123 SCRA 532 Abad Santos, J. FACTS: Mr. Gregorio Z. Ocampo of Meycauayan, Bulacan, husband of the plaintiff Felipa Cordero and father of the other plaintiffs surnamed Ocampo, died on May 17, 1958. The said deceased left several properties, which were inherited by the plaintiffs including the land in question which parcel of land was originally registered in accordance with the Land Registration Act on December 14, 1933, and was registered and/or transferred in the name of Mr. Gregorio Z. Ocampo on July 31, 1934. After the death of the said Mr. Gregorio Z. Ocampo, the plaintiffs herein took possession of the said parcel of land which is a riceland, but they found out that the southern portion of the same with an area 4,303 square meters, more or less, upon verification, was possessed by the defendants herein, Victoria P. Cabral, Alejandro Berboso and Dalmacio Montaos. Victoria P. Cabral claimed to be the owner of said portion while her co-defendants co-possessed the same as her tenants. The plaintiffs demanded of the defendants to surrender to the former possession of the portion of land and/or vacate it but they refused and failed to do so, and the defendant Victoria P. Cabral continued 89

claiming to be the owner of the same while her co-defendants continued recognizing her as the owner thereof instead of the plaintiffs. Plaintiffs alleged that because of the defendants' occupancy of the aforementioned plaintiffs' portion of land with the area of 4,303 square meters, more or less, to the exclusion of the latter, the said plaintiffs failed to realize a yearly harvest of at least ten (10) cavanes of palay at the rate of P10.00 per cavan, from the harvest-time of 1958 up to the present. ISSUE: Whether or not the defendants must reimburse the fruits receive. HELD: Yes. The disputed land is included in T.C.T. No. 14513 issued to Gregorio Z. Ocampo, the predecessor of the plaintiffs. The original registration which includes the disputed land was not vitiated by error or fraud. The defendants, by their own admission, are in possession of the disputed land. There is no evidence that they were possessors in bad faith. However, their good faith ceased when they were served with summons to answer the complaint. As possessors in bad faith from the service of the summons they "shall reimburse the fruits received and those which the legitimate possessor could have received.

90

Mendoza and Enriquez v. De Guzman G.R. No. L-28721, October 5, 1928, 52 Phil. 164 Malcolm, J. FACTS: In the cadastral proceedings of the municipality of Sariaya, Tayabas, a piece of land identified as lot No. 687 was adjudicated in favor of Martin Mendoza and Natalio Enriquez in equal parts pro indiviso subject to the right of retention on the part of Manuel de Guzman until he shall have been indemnified for the improvements existing on the land. Mendoza has possessed it since 1916. By virtue of this judgment, De Guzman presented a motion requesting the issuance of a writ of possession for lot No. 687 in his favor which was granted on June 25, 1924. Since then De Guzman has had dominion over the land. Being unable to come to an agreement as to the amount which should be allowed for the improvements made on the land, Martin Mendoza and Natalio Enriquez began an action requesting the court to (a) fix the value of the necessary and useful expenses incurred by Manuel de Guzman in introducing the improvements; (b) require the defendant to render an accounting of the fruits received by him and order that the value of the fruits be applied to the payment of the necessary and useful expenses; and (c) decree the restitution of the possession to the plaintiffs. Max. B. Solis, one of the persons who were ejected from the land, asked leave to intervene, alleging, among other things, that De Guzman had transferred all his rights in the improvements and in the lot to him with the exception of two hundred coconut trees. This petition was granted. At the trial which followed and at the instance of the parties, two commissioners were appinted with instructions to inspect the land and to count the number of coconut trees planted thereon, determining the number of fruit-bearing trees and those that are not fruit-bearing as well as the condition of the same. After trial, Judge of First Instance Gloria rendered judgment declaring (a) that the defendant Manuel de Guzman and the intervenor Bernardo Solis have the right to collect from the plaintiffs Martin Mendoza and Natalio Enriquez the sum of P2,046 as compensation for the necessary and useful expenditures in the proportion of 20 per cent for Manuel de Guzman and 80 per cent for Bernardo Solis; and (b) that Manuel de Guzman and Bernardo Solis are obliged to pay to the plaintiffs the sum of P666.93 per annum from June 25, 1924, one-fifth of this amount to be paid by Manuel de Guzman and the other four-fifths by Bernardo Solis. As on the date when this judgment was rendered, that is on September 23, 1927, the amount that the plaintiffs were required to pay to the defendant and intervenor exceeded the amount that the latter were to pay the former, the defendant and intervenor were ordered to deliver the land and its improvement as soon as the plaintiffs have paid the difference, without special pronouncement as to costs. ISSUE: Whether or not the trial court correctly declared the amount to be paid as "indemnizacion" in the form of necessary and useful expenditures incurred by the defendant. HELD: Yes. Article 361 of the Civil Code in the original Spanish text uses the word "indemnizacion." However one may speculate as to the true meaning of the term "indemnizacion" whether correctly translated as "compensation" or "indemnity," the 91

amount of the "indemnizacion" is the amount of the expenditures mentioned in articles 453 and 454 of the Civil Code, which in the present case is the amount of the necessary and useful expenditures incurred by the defendant. Necessary expenses have been variously described by the Spanish commentators as those made for the preservation of the thing; as those without which the thing would deteriorate or be lost; as those that augment the income of the things upon which they are expanded. Among the necessary expenditures are those incurred for cultivation, production, upkeep, etc. Here the plaintiffs have chosen to take the improvements introduced on the land and are disposed to pay the amount of the necessary and useful expenses incurred by the defendant. Inasmuch as the retentionist, who is not exactly a posessor in good faith with in the meaning of the law, seeks to be reimbursed for the necessary and useful expenditures, it is only just that he should account to the owners of the estate for any rents, fruits, or crops he has gathered from it.

Robles and Martin v. Lizarraga Hermanos G.R. No. L-16736, December 22, 1921, 42 Phil. 584 Romualdez, J. FACTS: Anastasia de la Rama died on the 17th of October, 1916, leaving six children, to wit, Magdalena, Jose, Evarista, Zacarias, Felix, and Purificacion, surnamed Robles, and some properties, among which is house No. 4 on Iznart Street in the city of Iloilo. The children and heirs of Anastasia de la Rama entered into partnership with Lizarraga Hermanos in liquidation and settlement of their accounts, by virtue of which the competent court awarded to said partnership the properties left by the deceased, including the aforesaid house No. 4 on Iznart Street. Evarista Robles, one of the heirs, since before the death of her mother Anastasia de la Rama, has been with her husband occupying the aforesaid house No. 4 on Iznart Street, at the beginning, by permission of her mother, later on by the consent of her coheirs, and lastly by agreement with the partnership, Lizarraga Hermanos, to whom it had been awarded, having made some improvements on the house, the value of which is fixed at four thousand five hundred pesos (P4,500), and paying to said partnership forty pesos (P40) monthly as rent of the upper story. On March 18, 1918, Lizarraga Hermanos notified Evarista Robles (Exhibit J) that beginning April next the rent of the upper story of the house would be raised to sixty pesos (P60) a month, and that, if she did not agree to the new rate of rent, she might vacate the house. Evarista Robles refused to pay such a new rate of rent and to vacate the house, and Lizarraga Hermanos brought suit against her for ejectment. Evarista Robles sued Lizarraga Hermanos afterwards to recover the value of the improvements. ISSUES: 1.) Whether or not Evarista Robles is the owner of the aforesaid improvements and has the right to demand payment of their value. 2.) Whether or not she has any right to retain the building until the said value is paid to her. 92

HELD: 1.) Yes. Robles is the owner of the improvements. The expenditures incurred in these improvements were not necessary inasmuch as without them the house would have continued to stand just as before, but were useful, inasmuch as with them the house better serves the purpose for which it was intended, being used as a residence, and the improvements consisting of the addition of a dining room, kitchen, closet, and bathroom in the lower and upper stories of the house, and a stable, suitable as a coach house and dwelling, it is beyond doubt that such improvements are useful to the building. Since the improvements are useful and Robles possession is in good faith, applying Article 453, it is beyond question that Evarista Robles is the owner of such improvements, and entitled to reimbursement therefor. 2.) Yes. It is a fact that the value of the improvements in question has not as yet been paid by Lizarraga Hermanos. Wherefore, if Evarista Robles and her husband are entitled to retain the building until the value of such improvements is paid them, Lizarraga Hermanos have not yet any right to oust them from the building, nor, therefore, to be indemnified for any damages caused by the refusal of the plaintiffs found on their legitimate rights. Hence, due to the non-reimbursement of the aforesaid useful expenditures, the possessor in good faith has the right of retention until she has been fully reimbursed with the same.

Metropolitan Waterworks and Sewerage System v. Court of Appeals G.R. No. L-54526, August 25, 1986, 143 SCRA 623 Martinez, J. FACTS: Sometime in 1965, petitioner MWSS (then known as NAWASA) leased around one hundred twenty eight (128) hectares of its land (hereafter, subject property) to respondent CHGCCI (formerly the International Sports Development Corporation) for twenty five (25) years and renewable for another fifteen (15) years or until the year 2005, with the stipulation allowing the latter to exercise a right of first refusal should the subject property be made open for sale. The terms and conditions of respondent CHGCCI's purchase thereof shall nonetheless be subject to presidential approval. Pursuant to Letter of instruction (LOI) No. 440 issued on July 29,1976 by then President Ferdinand E. Marcos directing petitioner MWSS to negotiate the cancellation of the MWSS-CHGCCI lease agreement for the disposition of the subject property, Oscar Ilustre, then General Manager of petitioner MWSS, sometime in November of 1980 informed respondent CHGCCI, through its president herein respondent Pablo Roman, Jr., of its preferential right to buy the subject property which was up for sale. Valuation thereof was to be made by an appraisal company of petitioner MWSS' choice, the Asian Appraisal Co., Inc. which, on January 30, 1981, pegged a fair market value of P40.00 per square meter or a total of P53,800,000.00 for the subject property. Upon being informed that petitioner MWSS and respondent CHGCCI had already agreed in principle on the purchase of the subject property, President Marcos expressed his approval of the sale as shown in his marginal note on the letter sent by respondents 93

Jose Roxas and Pablo Roman, Jr. dated December 20, 1982.The Board of Trustees of petitioner MWSS thereafter passed Resolution 36-83, approving the sale of the subject property in favor of respondent SILHOUETTE, as assignee of respondent CHGCCI. The MWSS-SILHOUETTE sales agreement eventually pushed through. Per the Agreement dated May 11, 1983 covering said purchase, the total price for the subject property is P50,925,200, P25 Million of which was to be paid upon President Marcos' approval of the contract and the balance to be paid within one (1) year from the transfer of the title to respondent SILHOUETTE as vendee with interest at 12% per annum. The balance was also secured by an irrevocable letter of credit. A Supplemental Agreement was forged between petitioner MWSS and respondent SILHOUETTE on August 11, 1983 to accurately identify the subject property. Subsequently, respondent SILHOUETTE, under a deed of sale dated July 26, 1984, sold to respondent AYALA about sixty-seven (67) hectares of the subject property at P110.00 per square meter. Of the total price of around P74 Million, P25 Million was to be paid by respondent AYALA directly to petitioner MWSS for respondent SILHOUETTE's account and P2 Million directly to respondent SILHOUETTE. P11,600,000 was to be paid upon the issuance of title in favor of respondent AYALA, and the remaining balance to be payable within one (1) year with 12% per annum interest. Respondent AYALA developed the land it purchased into a prime residential area now known as the Ayala Heights Subdivision. Almost a decade later, petitioner MWSS on March 26, 1993 filed an action against all herein named respondents before the Regional Trial Court of Quezon City seeking for the declaration of nullity of the MWSS-SILHOUETTE sales agreement and all subsequent conveyances involving the subject property, and for the recovery thereof with damages. ISSUE: Whether or not MWSS failed to provide appropriate security measures over its own records; Circumstances led NBI to believe that the fraudulent encashment as an inside job. HELD: Yes. The records likewise show that MWSS failed to provide appropriate security measures over its own records thereby laying confidential records open to unauthorized persons. MWSS's own Fact Finding Committee, in its report submitted to their General Manager underscored this laxity of records control. It observed that the "office of Mr. Ongtengco (Cashier VI of the Treasury Department at the NAWASA) is quite open to any person known to him or his staff members and that the check writer is merely on top of his table. Relying on the foregoing statement of Mr. Ongtengco, the NBI concluded in its Report dated 2 November 1970 that the fraudulent encashment of the 23 checks in question was an "inside job". Thus the NBI believe that the fraudulent act was an inside job or one pulled with inside connivance at NAWASA. The serial numbers of the checks in question conform with the numbers in current use of NAWASA, aside from the fact that these fraudulent checks were found to be of the same kind and design as that of NAWASA's own checks. While knowledge as to such facts may be obtained through the possession of a NAWASA check of current issue, an outsider without information from the inside can not possibly pinpoint which of NAWASA's various accounts has sufficient balance to cover all these fraudulent checks. None of these checks, it should be noted, was dishonored for insufficiency of funds.

94

Bachrach v. Seifert and Elianoff G.R. No. L-2659, October 12, 1950, 87 Phil. 483 Ozaeta, J. FACTS: The deceased E. M. Bachrach, who left no forced heir except his widow Mary McDonald Bachrach, in his last will and testament made various legacies in cash and willed the remainder of his estate. The estate of E. M. Bachrach, as owner of 108,000 shares of stock of the Atok-Big Wedge Mining Co., Inc., received from the latter 54,000 shares representing 50 per cent stock dividend on the said 108,000 shares. On June 10, 1948, Mary McDonald Bachrach, as usufructuary or life tenant of the estate, petitioned the lower court to authorize the Peoples Bank and Trust Company, as administrator of the estate of E. M. Bachrach, to transfer to her the said 54,000 shares of stock dividend by indorsing and delivering to her the corresponding certificate of stock, claiming that said dividend, although paid out in the form of stock, is fruit or income and therefore belonged to her as usufructuary or life tenant. Sophie Seifert and Elisa Elianoff, legal heirs of the deceased, opposed said petition on the ground that the stock dividend in question was not income but formed part of the capital and therefore belonged not to the usufructuary but to the remainderman. While appellants admit that a cash dividend is an income, they contend that a stock dividend is not, but merely represents an addition to the invested capital. ISSUE: Whether or not a dividend is an income and whether it should go to the usufructuary. HELD: Yes. The usufructuary shall be entitled to receive all the natural, industrial, and civil fruits of the property in usufruct. The 108,000 shares of stock are part of the property in usufruct. The 54,000 shares of stock dividend are civil fruits of the original investment. They represent profits, and the delivery of the certificate of stock covering said dividend is equivalent to the payment of said profits. Said shares may be sold independently of the original shares, just as the offspring of a domestic animal may be sold independently of its mother. If the dividend be in fact a profit, although declared in stock, it should be held to be income. A dividend, whether in the form of cash or stock, is income and, consequently, should go to the usufructuary, taking into consideration that a stock dividend as well as a cash dividend can be declared only out of profits of the corporation, for if it were declared out of the capital it would be a serious violation of the law. Under the Massachusetts rule, a stock dividend is considered part of the capital and belongs to the remainderman; while under the Pennsylvania rule, all earnings of a corporation, when declared as dividends in whatever form, made during the lifetime of the usufructuary, belong to the latter. The Pennsylvania rule is more in accord with our statutory laws than the Massachusetts rule.

Hemedes v. Court of Appeals, 95

G.R. No. 107132, October 8, 1999, 316 SCRA 347 Gonzaga Reyes, J. FACTS: Jose Hemedes, father of Maxima Hemedes and Enrique D. Hemedes. Jose Hemedes executed a document entitled "Donation Inter Vivos with Resolutory Conditions" whereby he conveyed ownership over the subject land, together with all its improvements, in favor of his third wife, Justa Kausapin, subject to the following resolutory conditions that upon her death or marriage, the donee shall revert the said property to anyone of Jose Hemedes children. On September 27, 1960 a "Deed of Conveyance of Unregistered Real Property by Reversion" was made conveying to Maxima Hemedes. She had it titled and mortgage it to R & B Insurance with an annotation of Usufruct in favor of her stepmother, Justa Kausapin. Unable to pay the mortgage, R & B Insurance extra-judicially foreclosed the property. However, Justa Kausapin executed another agreement or Kasunduan on May 27, 1971 to his stepson, Enrique D. Hemedes. He obtained tax declarations and pay realty taxes from thereon. The Ministry of Agrarian Reform Office conducted a cadastral survey and indicated Enrique Hemedes as the owner. Enrique Hemedes sold the property to Dominium Realty Const. Corp. (Dominium), a sister company of Asia Brewery. Asia Brewery started to introduce some improvements already when R & B insurance informed them that they are the owners of the property where these improvements are being built. ISSUE: Whether or not the kasunduan executed by Justa Kausapin in favor of Enrique D. Hemedes was valid. HELD: No. The court dismissed the petition and affirmed the decision of the CA. It held that Maxima failed to comply with the requirements of Art. 1332 of the civil code and also failed to repudiate Justa Kausapins allegation that she did not execute such a deed and she never allowed to use the land as security for the loan. It was found that the deed of conveyance to Maxima was spurious and it follows that the original title she had for the property was also null and void so as the mortgage to R & B Insurance. On the other hand, Kausapin executed an affidavit to affirm the authenticity of the the kasundudan in favor of his stepson, Enrique Hemedes whom she is dependent from for her financial support.

96

Fabie v. Gutierrez David G.R. No. L-123, December 12, 1945, 75 Phil. 536 Ozaeta, J. FACTS: The petitioner Josefa Fabie is the usufructuary of the income of certain houses located at 372-376 Santo Cristo, Binondo, and 950-956 Ongpin, Santa Cruz, Manila, under the ninth clause of the will of the deceased Rosario Fabie y Grey. The owner of Santo Cristo property abovementioned is the respondent Juan Grey, while those of the Ongpin property are other person not concern herein. Previous to September 1944 litigation arose between Josefa Fabie as plaintiff and Juan Grey as defendant and the owner of the Ongpin property as intervenors, involving the administration of the houses mentioned. ISSUE: Whether or not the action instituted by the petitioner Josefa Fabie is a purely possessory action and as such within the jurisdiction of said court, or an action founded on property right and therefore beyond the jurisdiction of the municipal court. HELD: Yes. It is admitted by the parties that the petitioner Josefa Fabie is the usufructuary of the income of the property in question and that the respondent Juan Grey is the owner thereof. It is likewise admitted that by virtue of a final judgment entered in Civil Case No. 1659 of the Court of First Instance of Manila between the usufructuary and the owner, the former has the right to collect all the rents of said property for herself with the obligation on her part to pay all the real estate taxes, special assessments, and insurance premiums, and make all necessary repairs thereon, and in case default on her part the owner shall have the right to do all those things, in which event he shall be entitled to collect all subsequent rents of the property concerned until the amount paid by him and the expenses of collection are fully satisfied, after which the usufructuary shall again collect the rents. There is therefore no dispute as to the title to or the respective interests of the parties in the property in question. The naked title to the property is to admittedly in the respondent Juan Grey, but the right to all the rents thereof, with the obligation to pay the taxes and insurance premiums and make the necessary repairs, is, also admittedly, vested in the usufructuary, the petitioner Josefa Fabie, during her lifetime. Construing said judgment in the light of the ninth clause of the will of the deceased Rosario Fabie y Grey, which was quoted in the decision and by which Josefa Fabie was made by the usufructuary during her lifetime of the income of the property in question, we find that the said usufructuary has the right to administer the property in question. All the acts of administration to collect the rents for herself, and to conserve the property by making all necessary repairs and paying all the taxes, special assessments, and insurance premiums thereon were by said judgment vested in the usufructuary

97

Vda. De Aranas v. Aranas G.R. No. L-56249, May 29, 1987, 150 SCRA 415 Paras, J. FACTS: Fr. Teodoro Aranas, a priest of the Roman Catholic Church, died on January 19, 1953. He had executed on June 6, 1946 his Last Will and Testament which was admitted to probate on August 31, 1956. In said Last Will and Testament, Fr. Teodoro Aranas stipulated the special administration of the remainder of his estate (after returning to his brothers Aniceto and Carmelo or their heirs all properties acquired by him including 10 parcels of land inherited by him from his parents) by Vicente Aranas, a faithful and serviceable nephew and designating him also as recipient of 1/2 of the produce of said properties after deducting the expenses for the administration and the other 1/2 of the produce to be given to the Catholic Church for the eternal repose of the testator's soul. Said pertinent provision reads as follows: It is my will that the lands I had bought from other persons should be converged and placed under a special administrator. The special administrator of these lands, for his office, should receive one half of all the produce from which shall be deducted the expenses for the administration, and the other half of the produce should be received by the Roman Catholic Church and should be spent for my soul, Vicente B. Aranas (Tingting), because he is a faithful and serviceable nephew, should be the first special administrator of said properties, without bond, until his death or until he should not want to hold the said office anymore. Anyone of the sons of my brother Carmelo Aranas can hold the said office of special administrator, and none other than they. Their father, my brother Carmelo Aranas shall be the one to decide who among them shall hold the said office, but upon the death of my said brother Carmelo Aranas, his said sons will have power to select the one among them ourselves. The special administration is perpetual. ISSUE: Whether or not perpetual inalienability and administration of the estate of the late Fr. Teodoro Aranas is null and void for being violative of Article 870 of the NCC. HELD: No. Vicente Aranas as a usufructuary has the right to enjoy the property of his uncle with all the benefits which result from the normal enjoyment (or exploitation) of another's property, with the obligation to return, at the designated time, either the same thing, or in special cases its equivalent. This right of Vicente to enjoy the fruits of the properties is temporary and therefore not perpetual as there is a limitation namely his death or his refusal. Likewise his designation as administrator of these properties is limited by his refusal and/or death and therefore it does not run counter to Art. 870 of the Civil Code relied upon by the petitioners. Be it noted that Vicente Aranas is not prohibited to dispose of the fruits and other benefits arising from the usufruct. Neither are the naked owners (the other heirs) of the properties, the usufruct of which has been given to Vicente Aranas prohibited from disposing of said naked ownership without prejudice of course to Vicente's continuing usufruct. To void the designation of Vicente Aranas as usufructuary and/or administrator is to defeat the desire and the dying wish of the testator to reward him for his faithful and unselfish services rendered during the time when said testator was seriously ill or bed-ridden. 98

Locsin v. Valenzuela G.R. No. L-51333, May 18, 1989, 173 SCRA 454 Feliciano, J. FACTS: Petitioners were co-owners of a large tract of agricultural land known as Hacienda Villa Regalado. A portion of this land known as Lot No. 2-C-A-3 was subject to lifetime usufructuary rights of respondent Helen Schon. The bulk of this lot was cultivated by the lessees who customarily delivered the rentals to respondent. In 1972, PD 27 was enacted, decreasing the Emancipation of Tenants. The tract of land owned in common by the petitioners, including the portion thereof subject to petitioners usufructuary rights, fell within the scope of the Operation Land Transfer. Petitioners sought the opinion of the Department of Agrarian Reform(DAR) as to who should be entitled to receive the rental payments which continued to be made by the tenants to respondent. The DAR District Officer rendered the opinion that the rental payments were properly considered as amortization payments for the land and as such should pertain to the landowners and not the usufructuary. ISSUE: Whether or not the usufructuary was extinguished by PD 27 and who, between the naked owner and the usufructuary, should be entitled to the amounts paid by the tenants beginning October 21, 1972. HELD: Yes. The usufruct which had therefore existed as a jus in re aliena in favour of Helen Schon was effectively extinguished by PD 27. To hold, as private respondent apparently urges would obviously defeat the purpose of the land reform statute. PD 27 was enacted to emancipate the tenants from bondage of the soil by giving to the tenant-farmers ownership of the land which they were cultivating. Ownership over the lands subjected to the Operation Land Transfer moved from the registered owner to the tenants. The Court holds that Lot No. 2-C-A-3 having been declared part of the land reform area and subjected to the Operation Land Transfer, the payments made on October 21, 1972 by the tenant-farmers constituted amortization payments on the cost of the land that they were required to pay under PD 27. These payments, therefore, legally pertain to the petitioners as part of the compensation for the dominion over the land of which they were deprived of by operation of PD 27.

99

Valisno v. Adriano G.R. No. L-37409, May 23, 1988, 161 SCRA 398 Grino Aquino, J. FACTS: Plaintiff appellant Nicolas Valisno alleges that he is the owner of a parcel of land in Nueva Ecija which he bought from his sister, Honorata Adriano Francisco. Said land is planted with watermelon, peanuts, corn, tobacco and other vegetables and adjoins the land of Felipe Adriano, on the bank of the Pampanga River. At the time of the sale of the land to Valisno, the land was irrigated by water from the Pampanga River through a canal about 70 meters long, traversing Adrianos land. Later, Adriano levelled a portion of the irrigation canal so that Valisno was deprived of the irrigation water and prevented from cultivating his 57 hectare land. Thus, Valisno filed a complaint for deprivation of waters rights in the Bureau of Public Works and Communications (Bureau PWC). Bureau PWC ruled in favour of Valisno. Instead of restoring the irrigation canal, Adriano asked for a reinvestigation of the case which was granted. In the meantime, Valisno rebuilt the irrigation canal at his own expense due to his urgent need to irrigate his watermelon fields. Valisno then filed a complaint for damages. However, the Secretary of Bureau PWC reversed its decision and dismissed Valisnos complaint. It held that Eladio Adrianos water rights which had been granted in1923 ceased to be enjoyed by him in 1936 or 1937, when his irrigation canal collapsed. His non-use of the water rights since then for a period of more than five years extinguished the grant by operation of law. Hence, the water rights did not form part of his hereditary estate which his heirs partitioned among themselves. Likewise, Valisno, as vendee of the land which Honorata received from her fathers estate did not acquire any water rights with the land purchased. The trial court held that Valisno had no right to pass through the defendant's land to draw water from the Pampanga River. It pointed out that under Section 4 of the Irrigation Law, controversies between persons claiming a right to water from a stream are within the jurisdiction of the Secretary of Bureau-PWC and his decision on the matter is final, unless an appeal is taken to the proper court within thirty days. The court may not pass upon the validity of the decision of the Public Works Secretary collaterally. Furthermore, there was nothing in Valisnos evidence to show that the resolution was not valid. It dismissed the complaint and counterclaim. Valisnos motion for reconsideration was denied, and he appealed to the Court of the Appeals who certified the case to the Supreme Court. ISSUE: Whether the provisions of the Irrigation Act (Act No. 2152) or those of the Civil Code should apply to this case. HELD: The provisions of the Civil Code shall apply. The existence of the irrigation canal on Adrianos land for the passage of water from the Pampanga River to Honorata's land prior to and at the time of the sale of Honorata's land to Valisno was equivalent to a title for the vendee of the land to continue using it as provided in Article 624 of the Civil Code: The existence of an apparent sign of easement between two estates, established or maintained by the owner of both shall be considered, should either of them be alienated, as a title in order that he easement may continue actively and passively, 100

unless at the time, theownership of the two estates is divided, the contrary should be provided in the title of conveyance of either of them, or the sign aforesaid should be removed before the execution of the deed. This provision shall also apply in case of the division of a thing owned in common on by two or more persons (Civil Code). This provision was lifted from Article 122 of the Spanish Law of Waters which provided: Whenever a tract of irrigated land which previously received its waters from a single point is divided through inheritance, sale or by virtue of some other title, between two or more owners, the owners of the higher estates are under obligation to give free passage to the water as an easement of conduit for the irrigation of the lower estates, and without right to any compensation therefore unless otherwise stipulated in the deed of conveyance. The deed of sale in favor of Valisno included the "conveyance and transfer of the water rights and improvements" appurtenant to Honorata Adriano's property. By the terms of the Deed of Absolute Sale, the vendor Honorata Adriano Francisco sold, ceded, conveyed and transferred to Dr. Nicolas Valisno all "rights, title, interest and participations over the parcel of land above- described, together with one Berkely Model 6 YRF Centrifugal Pump G" suction, 6" discharge 500-1500 GPM, with Serial No. 5415812 and one (1) set of suction pipe and discharge of pipe with elbow, nipples, flanges and footvalves," and the water rights and such other improvements appertaining to the property subject of this sale. According to Valisno, the water right was the primary consideration for his purchase of Honorata's property, for without it the property would be unproductive. Water rights, such as the right to use a drainage ditch for irrigation purposes, which are appurtenant to a parcel of land, pass with the conveyance of the land, although not specifically mentioned in the conveyance. The purchaser's easement of necessity in a water ditch running across the grantor's land cannot be defeated even if the water is supplied by a third person. The fact that an easement by grant may also have qualified as an easement of necessity does detract from its permanency as property right, which survives the determination of the necessity. As an easement of waters in favor of Valisno has been established, he is entitled to enjoy it free from obstruction, disturbance or wrongful interference (19 CJ 984), such as Adrianos act of levelling the irrigation canal to deprive him of the use of water from the Pampanga River.

Ronquillo, et. al. v. Roco, et. al. G.R. No. L-10619, February 28, 1958, 103 Phil. 84 Montemayor, J. FACTS: Plaintiff Leogario Ronquillo have been in the continuous and uninterrupted use of a road which traversed the land of the defendants, Rocos, in going to Igualdad Street and the market place of Naga City for more than 20 years and that the Rocos have long recognized and respected the private legal easement of a right of way of said plaintiffs. 101

On May 12, 1953, the defendants along with a number of men maliciously obstructed plaintiffs right of way by constructing a chapel in the middle of the said road and then later, by means of force, intimidation, and threats, illegally and violently planted wooden posts, fenced with barbed wire and closed hermitically the road passage way thereby preventing the plaintiff from using it. The plaintiff claims that he has already acquired the easement of right of way over the land thru prescription by his continuous and uninterrupted use of the narrow strip of land as passage way. However, plaintiffs complaint was dismissed by the CFI. ISSUE: Whether or not an easement of right of way can be acquired by prescription. HELD: No. The Court held than an easement of right of way may not be acquired thru prescription because though it may be apparent, it is nevertheless discontinuous or intermittent, and therefore, under Article 622 of the New Civil Code, can be acquired only by a virtue of a title. Furthermore, a right of way cannot be acquired by prescription because prescription requires that the possession be continuous and uninterrupted.

Taedo v. Bernad G.R. No. L-66520 August 30, 1988, 165 SCRA 86 Padilla, J. FACTS: Private respondent Antonio Cardenas owned Lot 7501-A and Lot 7501-B. On the said two lots, a septic tank was constructed for the common use of the occupants of both lots. Cardenas sold Lot 7501-A to herein petitioner Taedo and the other Lot 7501B was also mortgaged to Taedo as a security for the payment of loan with an agreement that Cardenas would only sell Lot 7501-B to him. However, said Lot 7501-B was sold to herein respondent Spouses Romeo and Pacita Sim. Upon learning of the said sale, Taedo offered to redeem the property from Sim but the latter refused. Instead, Sim blocked the sewage pipe connecting the building of Eduardo Taedo built on Lot 7501-A, to the septic tank in Lot 7501-B. He also asked Taedo to remove that portion of his building enroaching on Lot 7501-B. Taedo was then constrained to file an action for legal redemption and damages invoking Article 1622 of the Civil Code. On the other hand, respondent Spouses claimed they are the absolute owners of Lot 7501-B and that Eduardo Taedo has no right to redeem the land under Art. 1622 of the Civil Code as the land sought to be redeemed is much bigger than the land owned by Taedo. ISSUE: Whether or not the petitioners right to continue to use the septic tank, erected on Lot 7501-B, ceased upon the subdivision of the land and its subsequent sale to different owners who do not have the same interest.

102

HELD: No. Applying Article 631 and 624 of the Civil Code, no statement abolishing or extinguishing the easement of drainage was mentioned in the deed of sale of Lot 7501A to Eduardo Taedo. Nor did Antonio Cardenas stop the use of the drain pipe and septic tank by the occupants of Lot 7501-A before he sold said lot to Eduardo Tafiedo. Hence, the use of the septic tank is continued by operation of law. Accordingly, the spouses Romeo and Pacita Sim the new owners of the servient estate (Lot 7501- B), cannot impair, in any manner whatsoever, the use of the servitude.

Costabella Corporation v. Court of Appeals G.R. No. 80511 January 25, 1991, 193 SCRA 333 Sarmiento, J. FACTS: Petitioner owns the real estate properties situated at Sitio Buyong, Maribago, Lapu-Lapu City, on which it had constructed a resort and hotel. The private respondents, on the other hand, are the owners of adjoining properties. Before the petitioner began the construction of its beach hotel, the private respondents, in going to and from their respective properties and the provincial road, passed through a passageway which traversed the petitioner's property. In 1981, the petitioner closed the aforementioned passageway when it began the construction of its hotel, but nonetheless opened another route across its property through which the private respondents, as in the past, were allowed to pass. Later, or sometime in August, 1982, when it undertook the construction of the second phase of its beach hotel, the petitioner fenced its property thus closing even the alternative passageway and preventing the private respondents from traversing any part of it. Therefore, an action for injunction with damages was filed against the petitioner by the private respondents before the then Court of First Instance of Cebu. The CFI rendered a decision on March 15, 1984 finding that the private respondents had acquired a vested right over the passageway in controversy based on its long existence and its continued use and enjoyment by the private respondents and also by the community at large. On appeal, Appellate Court held as without basis the trial court's finding that the private respondents had acquired a vested right over the passageway in question by virtue of prescription. The appellate court pointed out that an easement of right of way is a discontinuous one which, under Article 622 of the New Civil Code, may only be acquired by virtue of a title and not by prescription. That notwithstanding, the appellate court went on to rule that ". . . in the interest of justice and in the exercise by this Court of its equity jurisdiction, there is no reason for Us in not treating the easement here sought by appellees Katipunan Lumber Co., Inc. and Perfecta Guangco as one that is not dependent upon the claims of the parties but a compulsory one that is legally demandable by the owner of the dominant estate from the owner of the servient estate." ISSUE: Whether or not the easement may be granted to private respondent over the land of Costabella.

103

HELD: No. It is already well-established that an easement of right of way, as is involved here, is discontinuous and as such can not be acquired by prescription. Insofar therefore as the appellate court adhered to the foregoing precepts, it stood correct. Unfortunately, after making the correct pronouncement, the respondent Appellate Court did not order the reversal of the trial court's decision and the dismissal of the complaint after holding that no easement had been validly constituted over the petitioner's property. Instead, the Appellate Court went on to commit a reversible error by considering the passageway in issue as a compulsory easement which the private respondents, as owners of the "dominant" estate, may demand from the petitioner the latter being the owner of the "servient" estate. Based on Articles 649 and 650 of the Civil Code, the owner of the dominant estate may validly claim a compulsory right of way only after he has established the existence of four requisites, to wit: (1) the (dominant) estate is surrounded by other immovables and is without adequate outlet to a public highway; (2) after payment of the proper indemnity; (3) the isolation was not due to the proprietor's own acts; and (4) the right of way claimed is at a point least prejudicial to the servient estate. In the case at bar, there is absent any showing that the private respondents had established the existence of the four requisites mandated by law.

Encarnacion v. Court of Appeals G.R. No. 77628, March 11, 1991, 195 SCRA 74 Fernan, C.J. FACTS: Petitioner owns the dominant estate bounded on north by the servient estate owned by respondents and an estate owned by Magsino, all of which are located in Talisay, Batangas. The servient estate is bound on the north by the national highway. To provide access to the highway, a one meter road path was paved through in which half of its width was taken from the estate of Magsino and the other half from the estate of the respondent. Petitioner started a nursery plant type of business in which pushcarts were used to haul the plants from his estate to and from his nursery and the highway, using the one meter road path. As his business grew, he bought a jeepney to enable him to transport more plants and soil catering to the now bigger demand. The problem however was that the jeepney cannot pass through the road path since its width would not be accommodated by a one meter width. Petitioner made a request upon the respondent to sell to him 1 meters of their property so that the pathway may be widened to enable his jeepney to pass through. The respondents refused. Petitioner went to court praying that he would be granted the additional land to the right of way already constituted but the trial court rendered a decision adverse to the petitioner because there was no such necessity as it was shown that there was the presence of dried river bed only 80 meters away from the property of the petitioner which he may use as an alternative route. The CA affirmed said decision of the trial court.

104

ISSUE: Whether or not petitioner is entitled to be granted his prayer to buy the additional land to increase the existing one meter road path. HELD: Yes. Even with the presence of the dried river bed, upon thorough investigation, it was found to be an inadequate right of way because a concrete bridge traverses it thereby the jeep would have to jump over said bridge which has a height of 5 meters in order to reach the highway. It was also found that during the rainy season, the same was impassable as it became flooded. This right of way could not provide adequate access to the highway thereby when an estate has no access to a public road, it may demand for a right of way. Furthermore, under Article 651 of the Civil Code, it is the needs of the dominant property which ultimately determine the width of the right of way. In this case, since the business of the petitioner grew larger and pushcarts became tedious to transport his nursery plants, it became necessary for him to do so with a jeepney. And in order to efficiently make such transportation of his plants, the right of way had to be widened to accommodate the width of the jeepney of the petitioner. The petitioner thus shall be granted the additional land to the existing right of way.

Case v. Heirs of Tuason G.R. No. L-5044, December 1, 1909, 14 Phil. 521 Torres, J. FACTS: The counsel for the heirs of Pablo Tuason and Leocadia Santibaez alleged that the parties whom he represents are owners in common of the property adjoining that of the petitioner Edwin Case on the southwest. The latter, extended his southwest boundary line to a portion of the lot of the said heirs of Tuason and Santibaez. They alleged that the true dividing line between the property of the petitioner and that of the said heirs is a belonging to the respondents, and that about two years ago, when Case made alterations in the buildings erected on his land, he improperly caused a portion of them to rest on the wall owned by the respondents. ISSUE: Whether or not the wall is the property of the heirs of the late Tuason and Santibaez. HELD: The wall in controversy belongs to the heirs of the late Tuason and Santibaez for the reason, among others, that in the public document by which one of their original ancestors acquired on the 19th of April, 1796, the property now possessed by them, it appears that property was then already inclosed by a stone wall. The wall supports only the property of the respondents and not that of the petitioner, can not be a party wall, one-half of which along its entire length would belong to the adjoining building owned by Mr. Case. There is not sufficient proof to sustain such claim, and besides, the building erected thereon disproves the pretension of the petitioner.

105

Under article 572 of the Civil Code the easement of party walls is presumed, unless there is a title or exterior sign, or proof to the contrary, among others, in dividing walls adjoining buildings up to the common point of elevation. The legal presumption as to party walls is limited to the three cases dealt with in the said article of the code, and is that of juris tantum unless the contrary appear from the title of ownership of the adjoining properties, that is to say, that the entire wall in controversy belongs to one of the property owners, or where there is no exterior sign to destroy such presumption and support a presumption against the party wall. It can not be presumed that the aforesaid portion was a party wall, and that it was not exclusively owned by the respondents, inasmuch as the latter have proven by means of a good title that has not been impugned by the petitioner, that when one of their ancestors and principals acquired the property the lot was already inclosed by the wall on which the building was erected; it must therefore be understood that in the purchase of the property the wall by which the land was inclosed was necessarily included.

Choco v. Santamaria G.R. No. 6076, December 29, 1911, 21 Phil. 132 Mapa, J. FACTS: The defendant in the building of his house, has made several openings and windows in the walls of the house on both sides overlooking then property of the plaintiff; that at the time the defendant was building his house, and the windows and the openings were being made, the plaintiffs protested, and later on and in the year 1905 made written protest and demand on the defendant, and the defendant received the written protest and referred it to his counsel, who, from the evidence, appears to have suggested an amicable and adjustment of the matter, but the adjustment was not made, and this action was brought. The Trial Court rendered judgment in favor of the plaintiffs, Severina and Flora Choco, and against the defendant, Isidro Santamaria, forever prohibiting the opening of the window stated, which must be closed, and forever prohibiting the opening of the windows and openings marked, which must be closed or made to conform to the requirements of law with regard to dimensions and an iron grate embedded in the wall, with the costs of the action. ISSUE: Whether or not the lower court erred by not ordering in his judgment the final and perpetual closing of the large window opened in the balcony of the back part of the appellee's house and that, though the appellant's lot can be seen through the window, it is not contiguous to the latter's property. HELD: To judge from the photographic views, it opens on the boundary line between the said lot and that the appellee and is situated perpendicularly above a part of the wall that belongs to the appellants. This opinion is corroborated by the testimony of the defendant's witness who took the said photographs, in so far as he said that "a part of 106

the window in question is in front of the plaintiffs' property, since between it and the plaintiffs' property there does not intervene the distance required by law that of two meters in the first case, and 60 centimeters in the second, therefore, its opening is a manifest violation of the provisions of article 582 of the Civil Code which reads as follows: Windows with direct views, or balconies or any similar openings projecting over the estate of the neighbor, cannot be made if there is not a distance of, at least, 2 meters between the wall in which they are built and said estate. Neither can side nor oblique views be opened over said property, unless there is a distance of 60 centimeters. Because of the lack of the distance required by law, the window in question must be closed, and consequently the judgment appealed from should be modified in this sense, as regards this window.

Solid Manila Corporation v. Bio Hong Trading Co., Inc. G.R. No. 90596, April 8, 1991, 195 SCRA 748 Sarmiento, J. FACTS: Petitioner Solid Manila Corporation is the owner of the land in Ermita, Manila. The same lies in the vicinity of another parcel, registered in the name of the private respondent Bio Hong Trading Co., Inc. The private respondents title came from a prior owner, and in their deed of sale, the parties thereto reserved as easement of way. As a consequence, there is an annotation which was entered wherein a construction of private alley has been undertaken. However, the petitioner averred that they and their neighbors have been using the private alley and maintained and contributed to its upkeep until sometime in 1983. Due to this, the private respondent constructed steel gates that precluded unhampered used. The petitioner commenced suit for injunction against the private respondent to have the gates removed and to allow full access to the easement. The court a quo issued ex parte an order directing the private respondent to open the gates. However, the Court of Appeals ordered the restoration of the annotation. They ruled that an easement is a mere limitation on ownership and that it does not impair the private respondents title, and that since the private respondent had acquired title to the property, merger brought about an extinguishment of the easement. The petitioner then averred that the very deed of sale executed between the private respondent and the previous owner of the property excluded the alley in question, and that in any event, the intent of the parties was to retain the alley as an easement, notwithstanding the sale. ISSUE: Whether or not an easement had been extinguished by merger. HELD: No. The Court held that no genuine merger took place as a consequence of the sale in favor of the private respondent corporation. According to the Civil Code, a merger exists when ownership of the dominant and servient estates is consolidated in the same person. Merger then, as can be seen, requires full ownership of both estates. One thing ought to be noted here, however. The servitude in question is a personal servitude, that is to say, one constituted not in favor of a particular tenement but rather, 107

for the benefit of the general public as stated in Article 614 of the Civil Code. In personal servitude, there is therefore no owner of a dominant tenement to speak of, and the easement pertains to persons without a dominant estate, in this case, the public at large. Merger, as we said, presupposes the existence of a prior servient-dominant owner relationship, and the termination of that relation leaves the easement of no use. Unless the owner conveys the property in favor of the public, if that is possible, no genuine merger can take place that would terminate a personal easement.

Floro v. Llenado G.R. No. 75723, June 2, 1995, 244 SCRA 713 Romeo, J. FACTS: Petitioner Simeon Floro is the owner of Floro Park Subdivision who has its own egress and ingress to and from the Mac Arthur Highway by means of its Road Lot 4 and the PNR level crossing. On the other hand, Respondent Orlando Llenado, is the registered owner of Llenado Homes Subdivision, adjacent to Floro Park Subdivision. Prior to its purchase by Llenado, the land was known as the Emmanuel Homes Subdivision, a duly licensed and registered housing subdivision in the name of Soledad Ortega. Bounded on the South by the 5 to 6 meter-wide Palanas Creek, which separates it from the Floro Park Subdivision, and on the west by ricelands belonging to Marcial Ipapo. the Llenado Homes does not have any existing road or passage to the Mac Arthur Highway. However, a proposed access road traversing the idle riceland of Marcial Ipapo has been specifically provided in the subdivision plan of the Emmanuel Homes Subdivision which was duly approved by the defunct Human Settlement Regulatory Commission. Meanwhile, the Llenados sought, and were granted permission by the Floros to use Road Lots 4 and 5 of the Floro Park Subdivision as passageway to and from MacArthur Highway. However no contract of easement of right of way was ever perfected by both parties. Later, Floro barricaded Road Lot 5 with a pile of rocks, wooden posts and adobe stones, thereby preventing its use by the Llenados. Llenado instituted a complaint before the RTC of Malolos, Bulacan against Floro for easement of right of way. The RTC granted the prayer for the issuance of a writ of preliminary mandatory injunction and ordered Floro to open the road and pay damages. Thereafter, the trial court rendered another judgment dismissing the case and lifting the writ of preliminary mandatory injunction previously issued and ordered the plaintiff to pay defendant damages and costs. On appeal by Llenado on the CA, the judgment of the RTC was reversed ordering Floro to open roads 4 and 5 and remove all the objects that prevent passage on road 5 and to pay the plaintiff damages with costs and payment of indemnity for the easement of right of way. ISSUE: Whether or not Llenado is entitled to a compulsory easement of right of way. HELD: No. For the Llenados to be entitled to a compulsory servitude of right of way under the Civil Code, the preconditions provided under Articles 649 and 650 thereof must be established. These preconditions are: (1) that the dominant estate is 108

surrounded by other immovables and has no adequate outlet to a public highway (Art. 649, par. 1); (2) after payment of proper indemnity (Art. 649, par. 1); (3) that the isolation was not due to acts of the proprietor of the dominant estate (Art. 649, last par.); and, (4) that the right of way claimed is at the point least prejudicial to the servient estate; and insofar as consistent with this rule, where the distance from the dominant estate to a public highway may be the shortest (Art. 650). The burden of proving the existence of the prerequisites to validly claim a compulsory right of way lies on the owner of the dominant estate. On the past subdivision plans by Emmanuel Homes which is bought by Llenado, there is an indication of an access road through IPAPOs property although it was not properly paved, a dirt road will suffice. Seeing this, Llenado has failed to comply with the first requirement. If the servitude requested by Llenado is allowed, other subdivision developers/owners would be encouraged to hastily prepare a subdivision plan with fictitious provisions for access roads merely for registration purposes. Furthermore, if such practice were tolerated, the very purpose for which Presidential Decree No. 957 was enacted, that is, to protect subdivision buyers from unscrupulous subdivision owners/developers who renege on their duties to develop their subdivisions in accordance with the duly approved subdivision plans, would be defeated. In order to justify the imposition of the servitude of right of way, there must be a real, not a fictitious or artificial necessity for it. Mere convenience for the dominant estate is not what is required by law as the basis for setting up a compulsory easement. Even in the face of a necessity, if it can be satisfied without imposing the servitude, the same should not be imposed. The complaint for easement of right of way filed by Llenado in the lower court did not contain a prayer for the fixing of the amount that he must pay Floro in the event that the easement of right of way is constituted. Thus, the existence of the second requisite has likewise not been established. Private respondent Llenado admitted that the Ipapo riceland was no longer being cultivated. Indications are that it has already been abandoned as a ricefield. There was no reason for private respondent's failure to develop the right of way except the inconvenience and expenses it would cost him. Hence, the third requisite has not been met. Failing to establish the existence of the prerequisites under Articles 649 and 650 of the Civil Code, private respondent Llenado's bid for a compulsory easement of right of way over Road Lots 4 and 5 of the Floro Park Subdivision must fail.

Quimen v. Court of Appeals G.R. No. 112331 May 29, 1996, 257 SCRA 163 Bellosillo, J. FACTS: Petitioner Anastacia Quimen together with her brothers Sotero, Sulpicio, Antonio and sister Rufina inherited a piece of property situated in Pandi, Bulacan. They 109

agreed to subdivide the property equally among themselves, as they did, with the shares of Anastacia, Sotero, Sulpicio and Rufina abutting the municipal road. Located directly behind the lots of Anastacia and Sotero is the share of their brother Antonio designated as Lot No. 1448-B-C which the latter divided into two (2) equal parts, now Lots Nos. 1448-B-6-A and 1448-B-6-B. The latter Lot is behind the property of Sotero, father of private respondent Yolanda Oliveros. Yolanda purchased Lot No. 1448-B-6-A from her uncle Antonio through her aunt Anastacia who was then acting as his administratrix. According to Yolanda, when petitioner offered her the property for sale she was hesitant to buy as it had no access to a public road. But Anastacia prevailed upon her to buy the lot with the assurance that she would give her a right of way on her adjoining property. Thereafter, Yolanda constructed a house on the lot she bought using as her passageway to the public highway a portion of Anastacia's property. But when Yolanda finally offered to pay for the use of the pathway Anastacia refused to accept the payment. In fact she was thereafter barred by Anastacia from passing through her property. Later, Yolanda purchased the other lot of Antonio Quimen, Lot No. 1448-B-6B, located directly behind the property of her parents who provided her a pathway between their house from the lot of Yolanda behind the sari sari store of Sotero, and Anastacia's perimeter fence. The store is made of strong materials and occupies the entire frontage of the lot measuring four (4) meters wide and nine meters (9) long. Although the pathway leads to the municipal road it is not adequate for ingress and egress. The municipal road cannot be reached with facility because the store itself obstructs the path so that one has to pass through the back entrance and the facade of the store to reach the road. Finally, Yolanda filed an action with the proper court praying for a right of way through Anastacia's property. The report was that the proposed right of way was at the extreme right of Anastacia's property facing the public highway, starting from the back of Sotero's sari-sari store and extending inward by one (1) meter to her property and turning left for about five (5) meters to avoid the store. However, the trial court dismissed her complaint. The Court of Appeals reversed the decision declaring that she was entitled to a right of way on petitioners property and that the way proposed by Yoland would cause the least damage and detriment to the servient estate. ISSUE: Whether or not passing through the property of Yolanda's parents is more accessible to the public road than to make a detour to her property and cut down the avocado tree standing thereon. HELD: Yes. The conditions sine quo non for a valid grant of an easement of right of way are: (a) the dominant estate is surrounded by other immovables without an adequate outlet to a public highway; (b) the dominant estate is willing to pay the proper indemnity; (c) the isolation was not due to the acts of the dominant estate; and, (d) the right of way being claimed is at a point least prejudicial to the servient estate. The criterion of least prejudice to the servient estate must prevail over the criterion of shortest distance although this is a matter of judicial appreciation. While shortest distance may ordinarily imply least prejudice, it is not always so as when there are permanent structures obstructing the shortest distance; while on the other hand, the longest distance may be free of obstructions and the easiest or most convenient to pass 110

through. In other words, where the easement may be established on any of several tenements surrounding the dominant estate, the one where the way is shortest and will cause the least damage should be chosen. However, as elsewhere stated, if these two (2) circumstances do not concur in a single tenement, the way which will cause the least damage should be used, even if it will not be the shortest. As between a right of way that would demolish a store of strong materials to provide egress to a public highway, and another right of way which although longer will only require an avocado tree to be cut down, the second alternative should be preferred.

De Jesus, et. al. v. Homart Corporation, et. al. G.R. No. 44191 R, August 28, 1974, 19 CA Rep. 831 FACTS: Jesus and Luz Miranda de Jesus are owners of the building located in Tondo, Manila. They brought an action for damages against Homart Corporation and Howmill Manufacturing Corporation, owners of the land adjoining the plaintiff on the same street where a sixty storey concrete building was constructed. Plaintiffs allege that the defendants failed to observe the necessary care and precautions to protect the construction of the plaintiffs by depriving it of sufficient lateral or subjacent support, thereby causing it to sink in some parts; its walls, ceilings, and floorings to crack in some places; and by the careless manner of handling the cement used the roofings of the building of the plaintiff were damaged with the accumulated debris piled thereon. ISSUE: Whether or not proper precautions had been taken by the defendants in constructing the building in question so as to prevent causing damage to the building of the plaintiff. HELD: No. Article 684 of the New Civil Code provides No property shall make such excavations upon his land as to deprive any adjacent land or building sufficient lateral or subjacent support. A reading of Article 684 shows that the duty of an adjacent owner not to deprive any adjacent land or building of sufficient lateral or subjacent support is an absolute one. It does not depend on the degree of care and precaution made by the proprietor in making the excavation or building on his land. Plaintiffs house which adjoins the seven storey concrete building constructed by the defendants had sunk by about eight inches. The sinking of the left side of the house of the plaintiffs was due to the weakening of subjacent support and to the weight of the seven storey concrete building constructed by the defendant, as the excavation made necessarily disturbed the subjacent soil of the plaintiffs land. Defendants having failed to provide the plaintiffs land and house with sufficient lateral and subjacent support are liable for damages.

La Vista Association, Inc. v. Court of Appeals 111

G.R. No. 95252, September 5, 1997, 278 SCRA 498 Bellosillo, J. FACTS: The Tuasons owned a vast tract of land in Quezon City and Marikina, and when they sold to Philippine Building Corporation a portion of their landholdings, it was expressly provided in the Deed of Sale with Mortgage that the boundary line between the property sold and the adjoining property of the Tuasons shall be a road fifteen (15) meters wide, one-half of which shall be taken from the property sold to the Philippine Building Corporation and the other half from the portion adjoining belonging to the Tuasons. Philippine Building Corporation then sold and assigned with the consent of the Tuasons, the subject parcel of land to ATENEO which assumed the mortgage and the obligation in the seven and one-half roadway. On their part, the Tuasons developed a part of the estate adjoining the portion sold to Philippine Building Corporation into a residential village known as LA VISTA Subdivision. Thus the boundary between LA VISTA and the portion sold to ATENEO was the 15-meter wide roadway known as the Mangyan Road. The Tuasons developed its 7.5-meter share of the 15-meter wide boundary, while ATENEO deferred improvement on its share and erected instead an adobe wall on the entire length of the boundary. ATENEO subsequently sold to Solid Homes Inc. the land which the latter developed into a subdivision now known as LOYOLA Grand Villas. Solid Homes Inc. now claims to have an easement of right-of-way along Mangyan Road through which they could have access to Katipunan Avenue. LA VISTA however instructed its security guards to prohibit agents and assignees of Solid Homes, Inc., from traversing Mangyan Road, and even constructed concrete posts that prevented the residents of LOYOLA from passing through. Solid Homes, Inc., filed a case before the Regional Trial Court and prayed that LA VISTA been joined from preventing and obstructing the use and passage of LOYOLA residents through Mangyan Road. The lower court recognized the easement of right-ofway along Mangyan Road in favor of Solid Homes, Inc., and ordered LA VISTA to pay damages. On appeal by LA VISTA, the decision of the lower court was affirmed. ISSUE: Whether or not there is an easement of right-of-way over Mangyan Road. HELD: Yes. The predecessors-in-interest of both LA VISTA and Solid Homes, Inc., i.e., the Tuasons and the Philippine Building Corporation, respectively, clearly established a contractual easement of right-of-way over Mangyan Road. A voluntary easement is quite evidently manifested in the stipulation in the Deed of Sale with mortgage executed by them. When the easement was established by their contract, the parties unequivocally made provisions for its observance by all whom in the future might succeed them in dominion. It is thus very apparent that the parties and their respective

112

predecessors-in-interest intended to establish an easement of right-of-way over Mangyan Road for their mutual benefit, both as dominant and servient estates. With this, the free ingress and egress along Mangyan Road created by the voluntary agreement between Ateneo and Solid Homes, Inc., is thus legally demandable (Articles 619 and 625, New Civil Code) with the corresponding duty on the servient estate not to obstruct the same. LA VISTA contends that there are other routes to LOYOLA from Mangyan Road, however, this should not be taken into consideration since the opening of an adequate outlet to a highway can extinguish only legal or compulsory easements, not voluntary easements like in the case at bar. The fact that an easement by grant may have also qualified as an easement of necessity does not detract from its permanency as a property right, which survives the termination of the necessity.

Alcantara v. Reta, Jr. G.R. No. 136996, December 14, 2001, 372 SCRA 364 Pardo, J. FACTS: Alcantara and the other petitioners claim that they were tenants or lessees of the land owned by Reta. The land has been converted into a commercial center and Reta is threatening to eject them. They claim that since they are legitimate tenants or lessees of such land, they have the right of first refusal to purchase the land in accordance with Section 3(g) of Presidential Decree No. 1517, the Urban Land Reform Act. They also claimed that the amicable settlement executed between Reta and Ricardo Roble, one of the petitioners, was void ab initio for being violative of PD No. 1517. On the other hand, Reta claimed that the land is question is not within the scope of PD No. 1517 since it was not proclaimed as an Urban Land Reform Zone (ULRZ). Alcantara, among others, then filed complaint for the exercise of the right of first refusal under PD No. 1517 in the Regional Trial Court. However, such complaint was dismissed and such dismissal was affirmed by the Court of Appeals. Hence, this petition was filed. ISSUE: Whether the Alcantara and the other petitioners have the right of first refusal. HELD: No. The land involved has not been proclaimed an Urban Land Reform Zone (ULRZ). In fact, petitioners filed a petition with the National Housing Authority requesting that said land be declared as an ULRZ. Clearly, the request to have the land proclaimed as an ULRZ would not be necessary if the property was an ULRZ. PD No. 1517 pertains to areas proclaimed as ULRZ. Consequently, petitioners cannot claim any right under the said law since the land involved is not an ULRZ. To be able to qualify and avail of the rights and privileges granted by the said decree, one must be: (1) a legitimate tenant of the land for ten (10) years or more; (2) must have 113

built his home on the land by contract; and, (3) has resided continuously for the last ten (10) years. Those who do not fall within the said category cannot be considered "legitimate tenants" and, therefore, not entitled to the right of first refusal to purchase the property should the owner of the land decide to sell the same at a reasonable price within a reasonable time. Reta denies that he has lease agreements with Alcantara and Roble. Alcantara, on the other hand, failed to present evidence of a lease agreement other than his testimony in court. Reta allowed Roble to use sixty-two (62) coconut trees for P186 from where he gathered tuba. This arrangement would show that it is a usufruct and not a lease. Roble was also allowed to construct his house on the land because it would facilitate his gathering of tuba. This would be in the nature of a personal easement under Article 614 of the Civil Code. Whether the amicable settlement is valid or not, the conclusion would still be the same since the agreement was one of usufruct and not of lease. Thus, Roble is not a legitimate tenant as defined by PD No. 1517. With regard to the other petitioners, Reta admitted that he had verbal agreements with them. This notwithstanding, they are still not the legitimate tenants who can exercise the right of first refusal under PD No. 1517. From the moment Reta demanded that the petitioners vacate the premises, the verbal lease agreements, which were on a monthly basis since rentals were paid monthly, ceased to exist as there was termination of the lease. In conclusion, none of the petitioners is qualified to exercise the right of first refusal under PD No. 1517. There was also no intention on the part of Reta to sell the property. Hence, even if the petitioners had the right of first refusal, the situation which would allow the exercise of that right, that is, the sale or intended sale of the land has not happened. PD No. 1517 applies where the owner of the property intends to sell it to a third party.

Prosperity Credit Resources, Inc. v. Court of Appeals G.R. No. 114170, January 15, 1999, 301 SCRA 52 Mendoza, J. FACTS: Private respondent Metropolitan Fabrics, Inc. (MFI) and petitioner Prosperity Credit Resources, Inc. (PCRI) executed a Memorandum of Undertaking (MOU) wherein PCRI acceded to MFIs request to redeem three of the seven lots foreclosed and won by the former in the ensuing public auction. The MOA was conditioned upon the agreement that the petitioner shall be given a right of way on the existing private road which forms part of the area to be redeemed by private respondents. Later, PCRI filed an injunctive suit against MFI alleging, inter alia, that the latter, in violation of the terms of the MOU, refused to allow PCRI to make excavations on one side of the access road for the installation of water. The trial court granted the petition for the issuance of the 114

writ of preliminary mandatory injunction. On appeal, the CA set aside the assailed order of the trial court; hence, this petition for review on certiorari. PCRI contends that it is entitled to the issuance of the writ of preliminary mandatory injunction as may be gleaned from the following provision in the MOU: The above cited lot, being an existing private road, will remain open to ingress and egress for whatever kind of passage in favor of PROSPERITY FINANCIAL RESOURCES, INC. or its successors=in-interest. ISSUE: Whether or not the RTC committed grave abuse of discretion in issuing a writ of preliminary mandatory injunction ordering private respondent to allow petitioner to undertake excavations along the access road for the purpose of installing water pipes. Held: Yes. There is no question as to the meaning of the terms ingress and egress. They give petitioner the right to use the private road as means of entry into and exit from its property on the northwestern side o f the compound. The question concerns the meaning of the phrase for whatever kind of passage. The trial court read this phrase to mean that petitioner had the right to make excavations on the side of the access road in order to install a network of pipes. The word passage does not, however; clearly and unmistakably convey a meaning that includes a right to install water pipes on the access road. The ordinary meaning of the word, as defined in Websters Dictionary, is that act or action of passing: movement or transference from one place or point to another. this legal meaning is not different. It means, according to Blacks Law Dictionary, the act of passing; transit; transition.

Villanueva v. Velasco G.R. No. 130845, November 27, 2000, 346 SCRA 99 Quisumbing, J. FACTS: Petitioner Bryan Villanueva is the registered owner of the parcel of land covered by Transfer Certificate of Title No. 127862 of the Register of Deeds of Quezon City. He bought it from Pacific Banking Corporation, the mortgagee of said property. When petitioner bought the parcel of land there was a small house on its southeastern portion. It occupied one meter of the two-meter wide easement of right of way the Gabriel spouses granted to the Espinolas, predecessors-in-interest of private respondents, in a Contract of Easement of Right of Way. Unknown to petitioner, even before he bought the land, the Gabriels had constructed the aforementioned small house that encroached upon the two-meter easement. Petitioner was also unaware that private respondents, Julio Sebastian and Shirley Lorilla, had filed on May 8, 1991 for easement. As successors-in-interest, Sebastian and Lorilla wanted to enforce the contract of easement. On August 13, 1991, a writ of preliminary mandatory injunction was issued, ordering the Gabriels to provide the right of way and to demolish the small house encroaching on the easement. On January 5, 1995, Judge Tirso Velasco issued an Alias Writ of Demolition. Meanwhile, petitioner filed a Third Party Claim with Prayer to Quash Alias Writ of Demolition. He maintains that the writ of demolition could not apply to his property since he was not a party to the civil case. 115

ISSUE: Whether or not the easement on the property binds petitioner. HELD: Yes. Unlike other types of encumbrance of real property, a servitude like a right of way can exist even if they are not expressly stated or annotated as an encumbrance in a Torrens title because servitudes are inseparable from the estates to which they actively or passively belong. Moreover, Villanueva was bound by the contract of easement, not only as a voluntary easement but as a legal easement. A legal easement is mandated by law, and continues to exist unless its removal is provided for in a title of conveyance or the sign of the easement is removed before the execution of the conveyance conformably with Article 649 in accordance with Article 617 of the Civil Code.

National Irrigation Administration v. Court of Appeals G.R. No. 114348, September 20, 2000, 340 SCRA 661 Pardo, J. FACTS: A free patent over 3 hectares of land in Cagayan was issued and registered in the name of private respondent Dick Manglapus predecessor-in-interest, Vicente Manglapus. The land was granted to the latter subject to the provisions of sections 113, 121, 122 and 124 of Commonwealth Act No. 141 which provide that except in favor of the Government or any of its branches, units, or institutions, the land hereby acquired shall be inalienable and shall not be subject to encumbrance for a period of 5 years from the date of this patent and shall not be liable for the satisfaction of any debt contracted prior to the expiration of that period. Subsequently, private respondent Manglapus acquired the lot from Vicente Manglapus by absolute sale and was later registered 11 years later from the issuance of patent. Meanwhile, petitioner National Irrigation Administration entered into a contract with Villamar Development Construction. Under the contract, petitioner NIA was to construct canals in Cagayan. NIA then entered a portion of petitioners land and made diggings and fillings thereon. Private respondent then filed a complaint for damages alleging that petitioners diggings and fillings destroyed the agricultural use of his land and that no reasonable compensation was paid for its taking. ISSUE: Whether or not the petitioner NIA should pay Manglapus just compensation for the taking of a portion of his property for use as easement of a right of way. HELD: No. We find that NIA is under no obligation. We sustain the appeal. We agree with NIA that the Transfer Certificate of Title and the Original Certificate of Title covering the subject parcel of land contained a reservation granting the government a right of way over the land covered therein. Under the Original Certificate of Title, there was a reservation and condition that the land is subject to to all conditions and public easements and servitudes recognized and 116

prescribed by law, especially thouse mentioned in Sections 109, 110, 111, 112, 113 and 114, Commonwealth Act No. 141, as amended. This reservation, unlike the other provisos imposed on the grant, was not limited by any time period and thus is a subsisting condition. Section 112, Commonwealth Act No. 141, provides that lands granted by patent, shall further be subject to a right of way not exceeding twenty meters in width for public highways, railrods, irrigation, ditches, aqueducts, telegraphs and telephone lines, and similar works as the Government or any public or quasi-public service or enterprises, including mining or forest concessionaires may reasonably require for carrying on their business, with damages for the improvements only. Article 619 of the Civil Code provides that Easements are established either by law or by the will of the owners. The former are called legal and the latter voluntary easements. In the present case, we find and declare that a legal easement of a rightof-way exists in favor of the government. The land was originally public land, and awarded to respondent Manglapus by free patent. The ruling would be otherwise if the land were originally private property, in which case, just compensation must be paid for the taking of a part thereof for public use as an easement of a right of way.

Remman Enterprises, Inc. v. Court of Appeals G.R. No. 125018, April 6, 2000, 330 SCRA 145 Bellosillo, J. FACTS: Petitioner Remman Enterprises, Inc. and private respondent Crispin Lat are adjoining landowners in Lipa City. The land of Lat is agricultural and planted mostly with fruit trees while Remmans land is devoted to its piggery business. The latters land is 1 meters higher in elevation than that of respondent Lat. Meanwhile, respondent noticed that petitioners waste disposal lagoon was already overflowing and inundating of Lats plantation. He made several representations with petitioner but they fell on deaf ears. Consequently, the trees growing on the flooded portion where it was inundated with water containing pig manure, started to wither and die. Private respondent then filed a complaint for damages alleging that the acidity of the soil in his plantation increased because of the overflow of the water heavy with pig manure from petitioners piggery farm. Petitioner denied the allegations and claimed that the construction of additional lagoons was already adopted to contain the waste water coming from its piggery to prevent any damage to the adjoining estate. Petitioner also argued that the damages, if any, were due to a fortuitous event. ISSUE: Whether or not the damages were due to a fortuitous event. HELD: No. We cannot agree with petitioner. We defer instead to the findings opions expressed by the lower courts: Even assuming that the heavy rains constituted an act of God; by reason of their negligence, the fortuitous event became humanized, rendering appellants liable for the ensuing damges. In National Power Corporation v. Court of Appeals, 233 SCRA 649 (1993), the Supreme Court held: Accordingly, petitioners 117

cannot be heard to invoke the act of God or force majeure to escape liability for the loss or damages sustained by private respondents since they, the petitioners, were guilty of negligence. This event then was not occasioned exclusively by an act of God or force majeure; a human factor negligence or imprudence had intervened. The effect tehn of the force majeure in question may be deemed to have, even if only partly, resulted from the participation of man. Thus, the whole occurrence was thereby humanized, as it were, and removed from the rules applicable to acts of God. As regards the alleged natural easement imposed upon the property of appelle, resort to pertinent provisions of applicable law is imperative. Under Article 637 of the Civil Code, it is provided that lower estates are obliged to receive the waters which naturally and without the intervention of man descend from the higher estates, as well as the stones or earth which they carry with them. The owner of the lower estate cannot construct works which will impede this easement; neither can the owner of the higher estate make works which will increase the burden. A similar provion is found under Article 50 of the Water Code of the Philippines (P.D. No. 1067), which provides that lower estates are obliged to receive the water which naturally and without the intervention of man flow from the higher estates, as well as the stone or eath which they carry with them. The owner of the lower estate cannot construct works which will impede this natural flow, unless he provides an alternative method of drainage; neither can the owner of the higher estate make works which will increase this natural flow. As worded, the two aforecited provisions impose a natural easement upon the lower estate to receive the waters which naturally and without the intervention of man descend from higher estates. However, where the waters which flow from a higher estate are those which are artificially collected in man-made lagoons, any damage occasioned thereby entitles the owner of the lower or servient estate to compensation.

Jesus is Lord Christian School Foundation, Inc. v. Municipality (now City) of Pasig, Metro Manila G.R. No. 152230, August 9, 2005, 466 SCRA 235 Callejo, Sr., J. FACTS: Respondent Municipality of Pasig needed an access road from E.R. Santos Street, a municipal road near the Pasig Public Market to Barangay Sto. Tomas Bukid, Pasig where 60 to 70 houses, mostly made of light materials, were located. The road has to be at least three meters in width, as required by the Fire Code, so that fire trucks could pass through in case of conflagration. Likewise, the residents in the area needed the road for water and electrical outlets. The municipality then decided to acquire 51 square meters out of the 1,791 square meter property of Lorenzo Ching Cuanco, Victor Ching Cuanco and Ernesto Ching Cuanco Kho, which is abutting E.R. Santos Street.

118

Meanwhile, the Sangguniang Bayan of Pasig approved an Ordinance authorizing the municipal mayor to initiate expropriation proceedings to acquire the said property and appropriate the fund therefore. The ordinance stated that the property owners were notified of the municipalitys intent to purchase the property for public use as an access road but they rejected the offer. The municipality then filed a complaint against the Cuancos for the expropriation of the property under Section 19 of the Republic Act No. 7160 or otherwise known as the Local Government Code. The Cuancos then contended that they had sold the said property to petitioner Jesus is the Lord Christian School Foundation, Inc. (JILCSFI) as evidenced by a deed of sale. When apprised about the complaint, petitioner JILCSFI filed a motion for leave to intervene as defendant-inintervention which was granted. The petitioner JILCSFI asserted that the respondent must comply with the requirements for the establishment of an easement of right-of-way, more specifically, the road must be constructed at the point lease prejudicial to the servient state, and that there must be no adequate outlet to a public highway. The petitioner also claimed that the portion of the lot sought to be expropriated is located at the middle protion of the petitioners entire parcel of land, thereby splitting the lot into two halves, and making it impossible for the petitioner to put up its school building and worship center. ISSUE: Whether or not the petitioner JILCSFIs contentions are tenable. HELD: No. The subject property is expropriated for the purpose of constructing a road. The respondent is not mandated to comply with the essential requisites for an easement of right-of-way under the New Civil Code. Case law has it that in the absence of legislative restriction, the grantee of the power of eminent domain may determine the location and route of the land to be taken unless such determination is capricious and wantonly injurious. Expropriation is justified so long as it is for the public good and there is genuine necessity of public character. Governmentmay not capriciously choose what private property should be taken. The respondent has demonstrated the necessity for constructing a road from E.R. Santos Street to Sto. Tomas Bukid. The witnesses, who were residents of Sto. Tomas Bukid, testified that although there were other ways through which one can enter the vicinity, no vehicle, however, especially fire trucks, could enter the area except through the newly constructed Damayan Street. This is more than sufficient to establish that there is a genuine necessity for the construction of a road in the area. After all, absolute necessity is not required, only reasonable and practical necessity will suffice. Nonetheless, the respondent failed to show the necessity for constructing the road particularly in the petitioners property and not elsewhere. We note that the wheras clause of the ordinance states that the 51-square meter lot is the shortest and most suitable access road to connect Sto. Tomas Bukid to E.R. Santos Street. The respondents complaint also alleged that the said portion of the petitioners lot has been surveyed as the best possible ingress and egress. However, the respondent failed to adduce a preponderance of evidence.

119

Acap v. Court of Appeals G.R. No. 118114, December 7, 1995, 251 SCRA 30 Padilla, J. FACTS: The title to Lot 1130 of the Cadastral Survey of Hinigaran, Negros Occidental was evidenced by OCT R-12179. The lot has an area of 13,720 sq. m. The title was issued and is registered in the name of spouses Santiago Vasquez and Lorenza Oruma. After both spouses died, their only son Felixberto inherited the lot. In 1975, Felixberto executed a duly notarized document entitled Declaration of Heirship and Deed of Absolute Sale in favor of Cosme Pido. Since 1960, Teodoro Acap had been the tenant of a portion of the said land, covering an area of 9,500 sq. m. When ownership was transferred in 1975 by Felixberto to Cosme Pido, Acap continued to be the registered tenant thereof and religiously paid his leasehold rentals to Pido and thereafter, upon Pidos death, to his widow Laurenciana. The controversy began when Pido died interstate and on 27 November 1981, his surviving heirs executed a notarized document denominated as Declaration of Heirship and Waiver of Rights of Lot 1130 Hinigaran Cadastre, wherein they declared to have adjudicated upon themselves the parcel of land in equal share, and that they waive, quitclaim all right, interests and participation over the parcel of land in favor of Edy de los Reyes. The document was signed by all of Pidos heirs. Edy de los Reyes did not sign said document. It will be noted that at the time of Cosme Pidos death, title to the property continued to be registered in the name of the Vasquez spouses. Upon obtaining the Declaration of Heirship with Waiver of Rights in his favor, de los Reyes filed the same with the Registry of Deeds as part of a notice of an adverse claim against the original certificate of title. Thereafter, delos Reyes sought for Acap to personally inform him that he had become the new owner of the land and that the lease rentals thereon should be paid to him. Delos Reyes alleged that he and Acap entered into an oral lease agreement wherein Acap agreed to pay 10 cavans of palay per annum as lease rental. In 1982, Acap allegedly complied with said obligation. In 1983, however, Acap refused to pay any further lease rentals on the land, prompting delos Reyes to seek the assistance of the then Ministry of Agrarian Reform (MAR) in Hinigaran, Negros Occidental. The MAR invited Acap, who sent his wife, to a conference scheduled on 13 October 1983. The wife stated that the she and her husband did not recognize delos Reyess claim of ownership over the land. On 28 April 1988, after the lapse of four (4) years, delos Reyes filed a complaint for recovery of possession and damages against Acap, alleging that as his leasehold tenant, Acap refused and failed to pay the agreed annual rental of 10 cavans of palay despite repeated demands. On 20 August 1991. ISSUE: Whether or not the subject declaration of heirship and waiver of rights is a recognized mode of acquiring ownership by private respondent over the lot in question. HELD: An asserted right or claim to ownership or a real right over a thing arising from a juridical act, however justified, is not per se sufficient to give rise to ownership over the res. That right or title must be completed by fulfilling certain conditions imposed by law. Hence, ownership and real rights are acquired only pursuant to a legal mode or

120

process. While title is the juridical justification, mode is the actual process of acquisition or transfer of ownership over a thing in question. Under Article 712 of the Civil Code, the modes of acquiring ownership are generally classified into two (2) classes, namely, the original mode (i.e., through occupation, acquisitive prescription, law or intellectual creation) and the derivative mode (i.e., through succession mortis causa or tradition as a result of certain contracts, such as sale, barter, donation, assignment or mutuum). In the case at bench, the trial court was obviously confused as to the nature and effect of the Declaration of Heirship and Waiver of Rights, equating the same with a contract (deed) of sale. They are not the same. In a Contract of Sale, one of the contracting parties obligates himself to transfer the ownership of and to deliver a determinate thing, and the other party to pay a price certain in money or its equivalent. Upon the other hand, a declaration of heirship and waiver of rights operates as a public instrument when filed with the Registry of Deeds whereby the intestate heirs adjudicate and divide the estate left by the decedent among themselves as they see fit. It is in effect an extrajudicial settlement between the heirs under Rule 74 of the Rules of Court. Hence, there is a marked difference between a sale of hereditary rights and a waiver of hereditary rights. The first presumes the existence of a contract or deed of sale between the parties. The second is, technically speaking, a mode of extinction of ownership where there is an abdication or intentional relinquishment of a known right with knowledge of its existence and intention to relinquish it, in favor of other persons who are co-heirs in the succession. Private respondent, being then a stranger to the succession of Cosme Pido, cannot conclusively claim ownership over the subject lot on the sole basis of the waiver document which neither recites the elements of either a sale, or a donation, or any other derivative mode of acquiring ownership.

De Luna v. Abrigo G.R. No. L-57455, January 18, 1990, 181 SCRA 150 Medialdea, J. FACTS: Prudencio de Luna donated a portion of a lot of the Cadastral Survey of Lucena to the Luzonian University Foundation. The donation was embodied in a Deed of Donation Intervivos and made subject to certain terms and conditions and provided for the automatic reversion to the donor of the donated property in case of violation or non-compliance. The foundation failed to comply with the conditions of the donation. De Luna "revived" the said donation in favor of the foundation, in a document entitled "Revival of Donation Intervivos" subject to terms and conditions which among others, required it to construct a chapel, a nursery and a kindergarten school in the donated property within five (5) years from execution. The automatic reversion to the donor of the donated area in case of violation of the conditions was also provided. The foundation, through its president, accepted the donation. A "Deed of Segregation" was 121

later executed by De Luna and the foundation whereby the area donated was adjudicated to the foundation. The heirs of de Luna later filed a complaint with the trial court alleging that the terms and conditions of the donation were not complied with by the foundation. Thus, it prayed for the cancellation of the donation and the reversion of the donated land to the heirs. The foundation invoked, among others, the defense of prescription of action. The court dismissed the complaint. It ruled that under Article 764 of the New Civil Code, actions to revoke a donation on the ground of non-compliance with any of the conditions of the donation shall prescribe in four years (4) counted from such non-compliance. In the instant case, the four-year period for filing the complaint for revocation commenced on April 9, 1976 and expired on April 9, 1980. Since the complaint was brought on September 23, 1980 or more than five (5) months beyond the prescriptive period, it was already barred by prescription. ISSUE: Whether or not the complaint is one for judicial decree of revocation of the donation in question as contemplated in Article 764 of the New Civil Code and which prescribes in four (4) years and not an action to enforce a written contract which prescribes in ten (10) years. HELD: The donation subject of this case is one with an onerous cause. It was made subject to the burden requiring the donee to construct a chapel, a nursery and a kindergarten school in the donated property within five years from execution of the deed of donation. It is true that under Article 764, actions for the revocation of a donation must be brought within for (4) years from the non-compliance of the conditions of the donation. However, the said article does not apply to onerous donations in view of the specific provision of Article 733 providing that onerous donations are governed by the rules on contracts. Therefore, the rules on contracts and the general rules on prescription and not the rules on donations are applicable in the case at bar. Furthermore, while the judicial action for the rescission of a contract is generally not necessary where the contract provides that it may be automatically revoked and cancelled for violation of any of its terms and conditions, however, where one of the parties contests or denies the rescission, judicial intervention is necessary not for purposes of obtaining a judicial declaration rescinding a contract already deemed rescinded by virtue of an agreement providing for rescission even without judicial intervention, but in order to determine whether or not the rescission was proper. Judicial action will be necessary as without it, the extrajudicial resolution will remain contestable and subject to judicial invalidation, unless attack thereon should become barred by acquiescence, estoppel or prescription. In the instant case, trial court was therefore not correct in holding that the complaint is barred by prescription under Article 764 because Article 764 does not apply to onerous donations. As provided in the donation executed on April 9, 1971, compliance with the terms and conditions of the contract of donation, shall be made within five (5) years from its execution. The complaint which was filed on September 23, 1980 was then well within the ten (10) year prescriptive period to enforce a written contract pursuant to Article 1144 par. 1, counted from April 9, 1976. 122

Reyes v. Mosqueda G.R. No. L-45262, July 23, 1990 Gutierrez, Jr., J. FACTS: On May 15, 1969, Dr. Emilio Pascual executed a Deed of Donation of real property located at 1109-1111 R. Papa St. Tondo, Manila in favor of Ofelia Parungao, a minor, with her mother, Rosario Duncil, accepting the gift and donation for and in her behalf. However, Ursula Pascual alleged that Dr. Pascual during his lifetime on November 2, 1966 executed a Donation mortis causa in her favor covering the said property. Parungao, upon reaching the age of majority was able to register the Deed of Donation with the Register of Deeds in Manila and was issued a TCT. On September 23, 1976, Ursula executed a deed of absolute sale over the Tondo property in favor of Benjamin, Oscar, Jose, and Emmanuel Reyes. Benjamin filed a complaint for the declaration of nullity of the TCT of Parungao and/or reconveyance of the deed of title. The CFI of Manila declared the TCT in the name of Parungao null and void and ordered the Register of Deeds to cancel the title. On appeal, the Court of Appeals ruled that the 1966 donation to Ursula was inter vivos, which meant that the property was already transferred to Ursula at that time. ISSUE: Whether or not the donation to Ursula was Inter Vivos or Mortis Causa. HELD: It was a Donation Inter Vivos. The title given by the donor in the deed of donation is not a determinative factor which makes the donation inter vivos or mortis causa. It is the body of the document of donation and the statements contained therein and not the title that should be considered in ascertaining the intent of the donor. In the case, the donor used the term donation Mortis Causa but from the stipulations of the deed, it can be clearly inferred that he was actually executing a donation Inter Vivos to Ursula. The transfer of ownership over the properties donated to Ursula was immediate and independent of the death of Dr. Pascual since it was a donation Inter Vivos. The provision as regards the reservation of properties for the donor's subsistence in relation to the other provisions of the deed of donation confirms the intention of the donor to give the naked ownership of the properties to Ursula immediately after the execution of the deed of donation. Hence, he could not have donated the property again in 1969 in favor of Parungao since the lot was already transferred to Ursula at that time.

Liguez v. Court of Appeals G.R. No. L-11240, December 18, 1957, 102 Phil. 577 Reyes, J.B.L., J. 123

FACTS: The case began upon complaint filed by petitioner-appellant against the widow and heirs of the late Salvador P. Lopez to recover a parcel of land in barrio Davao. Plaintiff averred to be its legal owner, pursuant to a deed of donation of said land, executed in her favor by the late owner, Salvador P. Lopez, on 18 May 1943. The defense interposed was that the donation was null and void for having an illicit causa or consideration, which was the plaintiff's entering into marital relations with Salvador P. Lopez, a married man; and that the property had been adjudicated to the appellees as heirs of Lopez by the court of First Instance, since 1949. It was ascertained by the Court of Appeals that the donated land originally belonged to the conjugal partnership of Salvador P. Lopez and his wife, Maria Ngo; that the latter had met and berated Conchita for living maritally with her husband, sometime during June of 1943; that the widow and children of Lopez were in possession of the land and made improvements thereon; that the land was assessed in the tax rolls first in the name of Lopez and later in that of his widow.; and that the deed of donation was never recorded. Upon these facts, the Court of Appeals held that the deed of donation was inoperative, and null and void (1) because the husband, Lopez, had no right to donate conjugal property to the plaintiff appellant; and (2) because the donation was tainted with illegal cause or consideration, of which donor and donee were participants. ISSUE: Whether or not the donation is valid. HELD: In the present case, it is scarcely disputable that Lopez would not have conveyed the property in question had he known that appellant would refuse to cohabit with him. The cohabitation was an implied condition to the donation, and being unlawful, necessarily tainted the donation itself. The rule that parties to an illegal contract, if equally guilty, will not be aided by the law but will both be left where it finds them, has been interpreted by this Court as barring the party from pleading the illegality of the bargain either as a cause of action or as a defense. Memo auditor propriam turpitudinem allegans. The appellant seeks recovery of the disputed land on the strength of a donation regular on its face. To defeat its effect, the appellees must plead and prove that the same is illegal. But such plea on the part of the Lopez heirs is not receivable, since Lopez, himself, if living, would be barred from setting up that plea; and his heirs, as his privies and successors in interest, can have no better rights than Lopez himself. Appellees, as successors of the late donor, being thus precluded from pleading the defense of immorality or illegal causa of the donation, the total or partial ineffectiveness of the same must be decided by different legal principles. In this regard, the Court of Appeals correctly held that Lopez could not donate the entirety of the property in litigation, to the prejudice of his wife Maria Ngo, because said property was conjugal in 124

character and the right of the husband to donate community property is strictly limited by law

Pershing Tan Queto v. Court of Appeals G.R. No. L-35648, March 27, 1987, 148 SCRA 54 Paras, J. FACTS: Herein private respondent Restituta Tacalinar Guangco de Pombuena received the questioned lot from her mother Basilides Tacalinar either as a purported donation or by way of purchase with P50 as the alleged consideration thereof. The donation or sale was consummated while Restituta was already married to her husband Juan Pombuena. Juan then filed for himself and his supposed co-owner Resitituta an application for a Torrens Title over the land which was later on granted pronouncing him (married to Resitiuta) as the owner of the land. A contract of lease over the lot was entered into between petitioner, Pershing Tan Queto and Restituta with the consent of her husband for a period of 10 years. The lease of contract having expired, Restituta filed for unlawful detainer against Tan Queto. The unlawful detainer case was won by the spouses in the Municipal Court but on appeal in the CFI the entire case was dismissed because of a barter agreement whereby Tan Queto became the owner of the disputed lot and the spouses became the owners of a parcel of land with the house thereon previously owned before the barter by Tan Queto. After the barter agreement, Tan Queto constructed on the disputed land a concrete building without any objection from Restituta. Afterwards Restituta sued both Juan and Tan Queto for reconveyance of the title over the registered but disputed lot, for annulment of the barter, and for recovery of the land with damages. The respondent courts decision which later on was affirmed by the Supreme court led to the reformation of the Contract of Sale of the disputed lot from Basilides to Restituta from a sale to a conveyance of the share of Restituta in the future hereditary estate of her parents. Hence, this petition for a motion for reconsideration. ISSUE: Whether or not the conveyance of the share of Restituta in the future hereditary estate of her parents was valid hence a paraphernal property. HELD: No. The court ruled that the land is conjugal, not paraphernal. The oral donation of the lot cannot be a valid donation intervivos because it was not executed in a public instrument (Art. 749, Civil Code), nor as a valid donation mortis causa for the formalities of a will were not complied with. The allegation that the transfer was a conveyance to RESTITUTA of her hereditary share in the estate of her mother (or parents) cannot be sustained for the contractual transmission of future inheritance is generally prohibited. The fact is ownership was acquired by both JUAN and RESTITUTA by tradition (delivery) as a consequence of the contract of sale (See Art. 712, Civil Code) with 125

P50.00 (then a considerable amount) as the cause or consideration of the transaction. The lot is therefore conjugal, having been acquired by the spouses thru onerous title (the money used being presumably conjugal there being no proof that RESTITUTA had paraphernal funds of her own).

Pajarillo vs. Intermediate Appellate Court G.R. No. 72908, August 11, 1989, 176 SCRA 340 Cruz, J. FACTS: Perfecta Balane de Cordero died intestate in 1945 and leaving a tract of 28 hectares of land with buildings and improvements in the Quezon Province. On May 20, 1946, perfectas siblings Juana and Felipe executed a public instrument entitled Extrajudicial settlement of the estate of the decease Perfecta Balane de Cordero. In it they disposed that in according to Perfectas wishes and in consideration of love and affection, the said property be donated to private respondent Salud Suterio de Matias, Perfectas niece, who will assume the encumbrance/obligation to the Philippine National Bank in the amount of P 1,000. In the same document, the done accepted the donation in a public instrument. The instrument was never registered nor the title transferred to Saluds name although she immediately took possession of the land. Sometime in 1951, Salud transferred the possession of the land to her mother Juana, who was then staying with her brother Claudio and his family. During the period they were occupying the land, Claudio paid realty taxes thereon. On May 25, 1956, Juana executed a deed of absolute sale conveying the land to Claudio. Two years later, Claudio had the land registered in his name. Claudio died in 1961 and his mother in 1963. On June 30, 1965, the private respondents Salud and Pedro Matias filed a complaint for the reconveyance of the property on the ground that the deed of sale in favour of Claudio was fictitious and the registration in his name was null and void. Salud claimed that no compensation was paid by Claudio and that the transaction was deliberately concealed from her by her brother and the defendants. ISSUE: Whether or not the extra-judicial settlement was a donation. HELD: Yes. Felipe and Juana had declared themselves the heirs of Perfecta and the owners of the property in question. As such, they were free to give the land to whomever they pleased and for whatever reason they saw fit. Hence, if they choose to respect Perfectas wishes and carry out her intentions by donating the land to Salud, there was no legal impediment to their doing so. There is no question that Felipe and Juana could have simply disregarded their sisters sentiments and decided not to donate the property to Salud. The fact that they did no do this speaks well of their integrity and their loyalty to their deceased sister. The extra-judicial settlement also reflects their own affection for Salud which constituted the valid consideration for their own act of liberality.

126

Cruz v. Court of Appeals G.R. No. L-58671, November 22, 1985, 140 SCRA 245 Plana, J. FACTS: In 1973, Eduvigis Cruz, a childless widow, donated a 235.5 sq. m. residential lot in San Isidro, Taytay, Rizal together with the two-door apartment erected thereon to her grandnieces (private respondents Teresita, Lydia and Cecilia, all surnamed De Leon). The property was accordingly transferred to the names of private respondents. In 1974, Cruz judicially adopted Cresencia Ocreto, a minor, after which she extrajudicailly tried to revoke the donation, but the donee resisted, alleging that: (1) the property in question was co-owned by Eduvigis Cruz and her brother, the late Maximo Cruz, grandfather of the donees, hence the latter own 1/2 of the property by inheritance; and (2) Eduvigis owns another property, an agricultural land of more than two hectares situated in Barrio Dolores, Taytay, Rizal, hence the donation did not impair the presumptive legitime of the adoptive child. Petitioner filed a complaint against the donees for revocation of donation, invoking Article 760, par. 3 of the NCC. The trial court rendered a decision revoking the donation. On appal, The Court of Appeals reversed the trial court and dismissed the complaint. ISSUE: Whether or not the Court of Appeals correctly dismissed the complaint to annul the subject donation. HELD: Yes. In the case of the subsequent adoption of a minor by one who had previously donated some or all of his properties to another, the donor may sue for the annulment or reduction of the donation within 4 years from the date of adoption, if the donation impairs the legitime of the adopted, taking into account the whole estate of the donor at the time of the donation of the child (Articles 760, 761 and 763 of the NCC). Of course, the burden of proof is on the plaintiff-donor, who must allege and establish the requirements prescribed by law, on the basis of which annulment or reduction of the donation can be adjudged. Unfortunately, in the case at bar, the complaint for annulment does not allege that the subject donation impairs the legitime of the adopted child. Indeed, it contains no indication at all of the total assets of the donor. Nor is there proof of impairment of legitime. On the contrary, there is unrebutted evidence that the donor has another piece of land worth P273,420 in 1977. The legal situation of petitioner-donor is made worse by the factual finding of the Court of Appeals that the grandfather of the donees was the owner pro indiviso of one-half of the donated land, the effect of which is to reduce the value of the donation which can then more easily be taken from the portion of the estate within the free disposal of petitioner.

Roman Catholic Archbishop of Manila v. Court of Appeals 127

G.R. No. 77425, June 19, 1991, 198 SCRA 300 Regalado, J. FACTS: On August 23, 1930, the spouses Eusebio de Castro and Martina Rieta executed a deed of donation in favor of herein petitioner Roman Catholic Archbishop of Manila covering a parcel of land located at Cavite. The deed of donation provides that the donee shall not dispose or sell the property within a period of 100 years from the execution of the deed of donation, otherwise a violation of such condition would render ipso facto null and void the donation and the property would revert to the estate of the donors. However, on June 30, 1980 while within the prohibitive period to dispose, petitioner executed a deed of absolute sale of the property subject of the donation in favor of the petitioner-spouses Florencio and Soledad Ignao in consideration of the sum of P114,000.00. Hence, private respondents filed a complaint for the nullification of the deed of donation. In their answer, the petitioners filed a motion to dismiss based on the grounds that the action has been barred by prescription because the complaint was filed four years after the sale, and that the complaint states no cause of action. ISSUE: Whether or not the deed of donation in favor of the Roman Catholic Archbishop of Manila may be revoked. HELD: No. The complaint in the case at bar cannot be barred by prescription because the applicable prescriptive period is not the 4-year period provided in Article 764 of the New Civil Code, rather it is the 10-year period ordinary prescription shall apply because the deed of donation provides for the automatic reversion of the property to the original owner in case of violation of any condition. The Court in the previous case of De Luna v. Abrigo has already settled such prescriptive period. However, although the action cannot be dismissed on the ground of prescription, the same should be dismissed for lack of cause of action. The cause of action of the private respondents is based on the fact that the petitioner sold the lot during the 50th year of the prohibitive period of 100 years. Such prohibitive period imposed by the respondents was unreasonable because applying in analogy Articles 494 and 870 of the New Civil Code, the donor cannot order a prohibitive period of disposition exceeding 20 years. As such, the said condition regarding the prohibitive period being contrary to law shall be considered as null and void pursuant to Art. 727 of the New Civil Code but the donation shall remain valid and subsisting. Thus, respondents cannot anymore revoke the donation, and the sale of the property by the petitioner to the Ignao spouses shall be valid and with legal effects.

Eduarte v. Court of Appeals G.R. No. 105944, February 9, 1996, 253 SCRA 391 128

Francisco, J. FACTS: Pedro Calapine was the registered owner of a parcel of land with an area of 12,199 square meters. He executed a deed entitled Donation InterVivos ceding onehalf portion thereof to his niece Helen S. Doria. Eventually, the whole parcel of land was ceded to Doria by Calapine. Doria then donated a portion of 157 square meters to the Calauan Christian Reformed Church. He also sold, transferred and conveyed unto the spouses Eduarte the parcel of land, saving the 700 square meters on which Dorias house was erected. However, Pedro Calapine filed a complaint against Doria, the Calauan Christian Reformed Church, Inc. and the spouses Eduarte claiming that his signature to the deed of donation was a forgery. He prays for the revocation of the donation made in favour of Doria, to declare null and void the deeds of donation and sale that she had executed in favor of the Calauan Christian Reformed Church, Inc. and the spouses Eduarte. ISSUE: Whether or not the petitioners are buyers in bad faith of the donated property. HELD: No. The rule is well-settled that mere possession cannot defeat the title of a holder of a registered torrens title to real property. When herein petitioners purchased the subject property from Helen Doria, the same was already covered by TCT No. T23205 under the latter's name. And although Helen Doria's title was fraudulently secured, such fact cannot prejudice the rights of herein petitioners absent any showing that they had any knowledge or participation in such irregularity. Thus, they cannot be obliged to look beyond the certificate of title which appeared to be valid on its fade and sans any annotation or notice of private respondents' adverse claim. Contrary therefore to the conclusion of respondent Court, petitioners are purchasers in good faith and for value as they bought the disputed property without notice that some other person has a right or interest in such property, and paid a full price for the same at the time of the purchase or before they had notice of the claim or interest of some other person in the property. And having established beyond doubt that Helen Doria fraudulently secured her title over the disputed property which she subsequently sold to petitioners, Helen Doria should instead be adjudged liable to private respondents, and not to petitioners as declared by the trial court and respondent Court of Appeals, for the resulting damages to the true owner and original plaintiff, Pedro Calapine. Petition granted.

Quilala v. Alcantara G.R. No. 132681, December 3, 2001, 371 SCRA 311 Ynares Santiago, J. FACTS: On February 20, 1981, Catalina Quilala executed a "Donation of Real Property Inter Vivos" in favor of Violeta Quilala over a parcel of land. The "Donation of Real Property Inter Vivos" consists of two pages. The first page contains the deed of donation itself, and is signed on the bottom portion by Catalina Quilala as donor, Violeta 129

Quilala as donee, and two instrumental witnesses. The second page contains the Acknowledgment, which states merely that Catalina Quilala personally appeared before the notary public and acknowledged that the donation was her free and voluntary act and deed. There appear on the left-hand margin of the second page the signatures of Catalina Quilala and one of the witnesses, and on the right-hand margin the signatures of Violeta Quilala and the other witness The deed of donation was registered with the Register of Deeds and, in due course, TCT No. 17214 was cancelled and TCT No. 143015 was issued in the name of Violeta Quilala. On November 7, 1983, Catalina Quilala died. Violeta Quilala likewise died on May 22, 1984. Petitioner Ricky Quilala alleges that he is the surviving son of Violeta Quilala. Meanwhile, respondents Gliceria Alcantara, Leonora Alcantara, Ines Reyes and Juan Reyes, claiming to be Catalina's only surviving relatives within the fourth civil degree of consanguinity, executed a deed of extrajudicial settlement of estate, dividing and adjudicating unto themselves the above-described property. On September 13, 1984, respondents instituted against petitioner and Guillermo T. San Pedro, the Registrar of Deeds of Manila, an action for the declaration of nullity of the donation inter vivos. The trial court found that the deed of donation, although signed by both Catalina and Violeta, was acknowledged before a notary public only by the donor, Catalina. Consequently, there was no acceptance by Violeta of the donation in a public instrument, thus rendering the donation null and void. On appeal, the Court of Appeals rendered a decision affirming with modification the decision of the trial court by dismissing the complaint for lack of cause of action without prejudice to the filing of probate proceedings of Catalina's alleged last will and testament. ISSUE: Whether or not the deed of donation is void for lack of acceptance on the part of the donee Violeta Quilala. HELD: No. As stated above, the second page of the deed of donation, on which the Acknowledgment appears, was signed by the donor and one witness on the left-hand margin, and by the donee and the other witness on the right hand margin. Surely, the requirement that the contracting parties and their witnesses should sign on the left-hand margin of the instrument is not absolute. The intendment of the law merely is to ensure that each and every page of the instrument is authenticated by the parties. The requirement is designed to avoid the falsification of the contract after the same has already been duly executed by the parties. Hence, a contracting party affixes his signature on each page of the instrument to certify that he is agreeing to everything that is written thereon at the time of signing. Simply put, the specification of the location of the signature is merely directory. The fact that one of the parties signs on the wrong side of the page does not invalidate the document. The purpose of authenticating the page is served, and the requirement in the above-quoted provision is deemed substantially complied with.

130

In the same vein, the lack of an acknowledgment by the donee before the notary public does not also render the donation null and void. The instrument should be treated in its entirety. It cannot be considered a private document in part and a public document in another part. The fact that it was acknowledged before a notary public converts the deed of donation in its entirety a public instrument. The fact that the donee was not mentioned by the notary public in the acknowledgment is of no moment. To be sure, it is the conveyance that should be acknowledged as a free and voluntary act. In any event, the donee signed on the second page, which contains the Acknowledgment only. Her acceptance, which is explicitly set forth on the first page of the notarized deed of donation, was made in a public instrument.

Hemedes v. Court of Appeals G.R. No. 107132, October 8, 1999, 316 SCRA 347 Gonzaga Reyes, J. FACTS: Jose Hemedes, father of Maxima Hemedes and Enrique D. Hemedes. Jose Hemedes executed a document entitled "Donation Inter Vivos With Resolutory Conditions" whereby he conveyed ownership over the subject land, together with all its improvements, in favor of his third wife, Justa Kausapin, subject to the following resolutory conditions that upon her death or marriage, the DONEE shall revert the said property to anyone of Jose Hemedes children. On September 27, 1960 a "Deed of Conveyance of Unregistered Real Property by Reversion" conveying to Maxima Hemedes. She had it titled and mortgage it to R & B Insurance with an annotation of USUFRUCT favor of her stepmother,Justa Kausapin. Unable to pay the mortgage, R & B Insurance extra-judicially foreclosed the property. However, Justa Kausapin executed another agreement or Kasunduan on May 27, 1971 to his stepson, Enrique D. Hemedes. He obtained tax declarations and pay realty taxes from thereon. The Ministry of Agrarian Reform Office conducted a cadastral survey and indicated Enrique Hemedes as the owner. Enrique Hemedes sold the property to Dominium Realty Const. Corp.(Dominium), a sister company of Asia Brewery. Asia Brewery started to introduce some improvements already when R & B insurance informed them that they are the owners of the property where these improvements are being built. ISSUE: Whether or not the kasunduan executed by Justa Kausapin in favor of Enrique D. Hemedes valid. HELD: The court dismissed the petition and affirmed the decision of the CA. It held that Maxima failed to comply with the requirements of Art. 1332 of the civil code and also failed to repudiate Justa Kausapins allegation that she did not execute such a deed and she never allowed to use the land as security for the loan. It was found that the deed of conveyance to Maxima was spurious and it follows that the original title she had for the 131

property was also null and void so as the mortgage to R & B Insurance. On the other hand, Kausapin executed an affidavit to affirm the authenticity of the kasundudan in favor of his stepson, Enrique Hemedes whom she is dependent from for her financial support. Siguan v. Lim G.R. No. 134685, November 19, 1999, 318 SCRA 725 Davide, Jr., C.J. FACTS: On 2 July 1991, a Deed of Donation conveying parcels of land and purportedly executed by LIM on 10 August 1989 in favor of her children, Linde, Ingrid and Neil was registered with the Office of the Register of Deeds of Cebu City. On 23 June 1993, petitioner filed an accion pauliana against LIM and her children to rescind the questioned Deed of Donation and to declare as null and void the new transfer certificates of title issued for the lots covered by the questioned Deed. Petitioner claimed therein that sometime in July 1991, LIM, through a Deed of Donation, fraudulently transferred all her real property to her children in bad faith and in fraud of creditors, including her; that LIM conspired and confederated with her children in antedating the questioned Deed of Donation, to petitioner's and other creditors' prejudice; and that LIM, at the time of the fraudulent conveyance, left no sufficient properties to pay her obligations. The RTC ruled in favor of Siguan and rescinded the Contract, but was reversed by the CA. ISSUE: Whether or not the Deed of Donation executed by respondent may be rescinded for being in fraud of her alleged creditor. HELD: We resolve these issues in the negative. Art. 1381 of the Civil Code enumerates the contracts which are rescissible, and among them are "those contracts undertaken in fraud of creditors when the latter cannot in any other manner collect the claims due them." The action to rescind contracts in fraud of creditors is known as accion pauliana. For this action to prosper, the following requisites must be present: (1) the plaintiff asking for rescission has a credit prior to the alienation, although demandable later; (2) the debtor has made a subsequent contract conveying a patrimonial benefit to a third person; (3) the creditor has no other legal remedy to satisfy his claim; (4) the act being impugned is fraudulent; (5) the third person who received the property conveyed, if it is by onerous title, has been an accomplice in the fraud. The general rule is that rescission requires the existence of creditors at the time of the alleged fraudulent alienation, and this must be proved as one of the bases of the judicial pronouncement setting aside the contract. Without any prior existing debt, there can neither be injury nor fraud. While it is necessary that the credit of the plaintiff in the accion pauliana must exist prior to the fraudulent alienation, the date of the judgment enforcing it is immaterial. Even if the judgment be subsequent to the alienation, it is merely declaratory, with retroactive effect to the date when the credit was constituted. In the instant case, the alleged debt of LIM 132

in favor of petitioner was incurred in August 1990, while the deed of donation was purportedly executed on 10 August 1989. Even assuming arguendo that petitioner became a creditor of LIM prior to the celebration of the contract of donation, still her action for rescission would not fare well because the third requisite was not met. Under Article 1381 of the Civil Code, contracts entered into in fraud of creditors may be rescinded only when the creditors cannot in any manner collect the claims due them. It is, therefore, "essential that the party asking for rescission prove that he has exhausted all other legal means to obtain satisfaction of his claim. 20 Petitioner neither alleged nor proved that she did so. On this score, her action for the rescission of the questioned deed is not maintainable even if the fraud charged actually did exist."

Noceda vs. Court of Appeals G.R. No. 119730, September 2, 1999, 313 SCRA 504 Gonzaga Reyes, J. FACTS: Celestino Arbizo died in 1956 leaving behind a parcel of land having an area of 66,530 square meters. His heirs plaintiff Aurora Directo, defendant Rodolfo Noceda, and Maria Arbizo extrajudicially settled the partition of the land with Directo getting 11,426 square meters, Noceda got 13,294 square meters, and Arbizo got 41,810 square meters. Plaintiff Directo donated 625 square meters of her share to defendant Noceda, who is her nephew being the son of her deceased sister However another extrajudicial settlement-partition was executed. Three fifths of the said land went to Maria Arbizo while plaintiff Directo and defendant Noceda got only one-fifth each. Sometime in 1981, Noceda constructed his house on the land donated to him by Directo. Directo fenced the portion allotted to her in the extrajudicial settlement, excluding the donated portion, and constructed thereon three huts. But in 1985, Noceda removed the fence earlier constructed by Directo, occupied the three huts (3) and fenced the entire land of plaintiff Directo without her consent. Directo demanded from Noceda to vacate her land, but the latter refused. Hence, Directo filed a complaint for the recovery of possession and ownership and rescission/annulment of donation, against defendant Noceda ISSUE: Whether or not the acts of Noceda constitute ingratitude to warrant revocation of the donation. HELD: Yes. It was established that petitioner Noceda occupied not only the portion donated to him by private respondent Aurora Arbizo-Directo but he also fenced the whole area of Lot C which belongs to private respondent Directo, thus petitioner's act of occupying the portion pertaining to private respondent Directo without the latter's knowledge and consent is an act of usurpation which is an offense against the property of the donor and considered as an act of ingratitude of a donee against the donor. The law does not require conviction of the donee; it is enough that the offense be proved in the action for revocation. 133

The action to revoke by reason of ingratitude prescribes within one (1) year to be counted from the time (a) the donor had knowledge of the fact; (b) provided that it was possible for him to bring the action. It is incumbent upon petitioner to show proof of the concurrence of these two conditions in order that the one (1) year period for bringing the action be considered to have already prescribed. No competent proof was adduced by petitioner to prove his allegation.

Heirs of Cesario Velasquez v. Court of Appeals G.R. No. 126996, February 15, 2000, 325 SCRA 552 Gonzaga Reyes, J. FACTS: Spouses Leoncia de Guzman and Cornelio Aquino died intestate sometime in 1945 and 1947, respectively and were childless, leaving 6 parcels of land situated in Pangasinan. Leoncia De Guzman was survived by her sisters Anatalia de Guzman and Tranquilina de Guzman. Sometime in 1989, the Meneses(heirs of Anatalia de Guzman) filed a complaint for annulment, partition and damages against the heirs of Cesario Velasquez(son of Tranquilina de Guzman) for the latters' refusal to partition the properties of the Spouses Aquino. The complaint alleged that before Leoncias death, she told that the documents of donation and partition which she and her husband earlier executed were not signed by them as it was not their intention to give away all the properties to Cesario Velasquez because Anatalia de Guzman who is one of her sisters had several children to support; and that Cesario Velasquez and his mother allegedly promised to divide the properties equally and to give the plaintiffs one-half thereof. Plaintiffs further claim that after the death of Leoncia, defendants forcibly took possession of all the properties and despite plaintiffs' repeated demands for partition, defendants refused. Plaintiffs prayed for the nullity of the documents covering the properties in question since they do not bear the genuine signatures of the Aquino spouses, to order the partition of the properties between plaintiffs and defendants in equal shares and to order the defendants to render an accounting of the produce of the land from the time defendants forcibly took possession until partition shall have been effected. Defendants filed their answer with counterclaim alleging that during the lifetime of spouses Aquino, they had already disposed of their properties in favor of petitioners' predecessors-in-interest, Cesario Velasquez and Camila de Guzman, and petitioners Anastacia and Jose Velasquez in the following manner: (1) The third and sixth parcels were conveyed to defendants' late parents Cesario Velasquez and Camila de Guzman, by virtue of a Escritura de Donation Propter Nuptias dated February 15, 1919; 134

(2) The second parcel was conveyed to defendants' late parents Cesario Velasquez and Camila de Guzman by virtue of a deed of conveyance dated July 14, 1939; (3) The first parcel was likewise conveyed to defendants Jose Velasquez and Anastacia Velasquez by virtue of a deed of conveyance (Donation Intervivos) dated April 10, 1939; (4) As to the fourth and fifth parcels, the same were owned and possessed by third parties. The trial court ruled in favor of the plaintiffs, giving credibility to Santiago Meneses testimony; declaring the Donation Intervivos, the Deed of Sale, the Deed of Donation, the Deed of Sale to third parties over the 4th and 5th parcels as null and void insofar as 1/2 of the 6 parcels are concerned legitimately belong to the plaintiffs; and ordering the defendants to pay damages. Defendants appealed the decision to respondent CA which affirmed the same. A motion for reconsideration was filed by the petitioners but the same was denied. ISSUE: Whether or not the petitioners have acquired absolute and exclusive ownership of the properties in question. HELD: Yes. Private respondent Santiago Meneses failed to prove the nullity of the Deeds of Conveyance executed by the Aquino spouses in favor of petitioners and their predecessors-in-interest Cesario Velasquez and Camila de Guzman since he failed to adduce any evidence to support his claim other than his bare allegations of its nullity. On the other hand, petitioners were able to show by documentary evidence that the Aquino spouses during their lifetime disposed of the four parcels of land subject of the complaint, to wit: (a) Escritura de donation propter nuptias, (b) Deed of donation inter vivos, (c) Escritura de Compreventa with a P500 consideration: (d) Deed of Conveyance with a consideration of P600 and confirming in the same Deed the Escritura de donation propter nuptias and Escritura de compraventa abovementioned. It was reversible error for the court to overlook the probative value of these notarized documents. A donation as a mode of acquiring ownership results in an effective transfer of title over the property from the donor to the donee and the donation is perfected from the moment the donor knows of the acceptance by the donee. Once a donation is accepted, the donee becomes the absolute owner of the property donated. The donation of the first parcel made by the Aquino spouses to petitioners Jose and Anastacia Velasquez who were then 19 and 10 years old respectively was accepted through their father Cesario Velasquez, and the acceptance was incorporated in the body of the same deed of donation and made part of it, and was signed by the donor and the acceptor. Legally speaking there was delivery and acceptance of the deed, and the donation existed perfectly and irrevocably. The donation inter vivos may be revoked only for the reasons provided in Articles 760, 764 and 765 of the Civil Code.

135

The donation propter nuptias in favor of Cesario Velasquez and Camila de Guzman over the third and sixth parcels including a portion of the second parcel became the properties of the spouses Velasquez since 1919. The deed of donation propter nuptias can be revoked by the non-performance of the marriage and the other causes mentioned in Article 86 of the Family Code. The alleged reason for the repudiation of the deed, i.e., that the Aquino spouses did not intend to give away all their properties since Anatalia had several children to support is not one of the grounds for revocation of donation either inter vivos or propter nuptias, although the donation might be inofficious. The Escritura compraventa over another portion of the second parcel and the Deed of conveyance dated July 14, 1939 in favor of Cesario and Camila Velasquez over the remaining portion of the second parcel is also valid. In fact in the deed of sale, the Aquino spouses ratified and confirmed the rights and interests of Cesario Velasquez and Camila de Guzman including the previous deeds of conveyance over the second parcel in the complaint and such deed of sale became the basis for the issuance of TCT in the names of Cesario Velasquez and Camila de Guzman. The best proof of the ownership of the land is the certificate of title and it requires more than a bare allegation to defeat the face value of TCT which enjoys a legal presumption of regularity of issuance. Notably, during the lifetime of Cesario Velasquez, he entered into contracts of mortgage and lease over the property as annotated at the back of the certificate of title which clearly established that he exercised full ownership and control over the property. Petitioners were able to establish that these four parcels of land were validly conveyed to them by the Aquino spouses, hence, they no longer formed part of the conjugal properties of the spouses at the time of their deaths. As regards the fourth and fifth parcels, petitioners alleged that these were also conveyed to third persons and they do not claim any right thereto. In view of the foregoing, the action of partition cannot be maintained. The properties sought to be partitioned by private respondents have already been delivered to petitioners and therefore no longer part of the hereditary estate which could be partitioned. No co-ownership exists between private respondents and petitioners.

Gonzales v. Court of Appeals G.R. No. 110335, June 18, 2001, 358 SCRA 598 Melo, J. FACTS: Deceased spouses Ignacio and Marina Gonzales were registered owners of two parcels of agricultural land. Petitioners are the successors-in-interest or the children and grandchildren of the deceased spouses. On the other hand, private respondents are the farmers and tenants of said spouses who have been cultivating the parcels of land even before World War II either personally or through their predecessors-ininterest. Marina Gonzales died intestate and appointed as administratix was petitioner 136

Lilia Gonzales. Prior to the partition of said estate, Ignacio Gonzales executed a Deed of Donation in favor of his grandchildren but was not registered. When Presidential Decree No. 27 took effect, the landholdings of the said spouses were placed under Operation Land Transfer. Private respondents were then issued the corresponding Certificates of Land Transfer. The administratix of the spouses estate, Lilia Gonzales filed an application for retention requesting that their property be excluded from the Operation Land Transfer. Initially, it was denied but was approved due to the deed of donation. ISSUE: Whether or not the property subject of the deed of donation which was not registered when P.D. No. 27 took effect, should be excluded from the Operation Land Transfer. HELD: No. Article 749 of the Civil Code provides inter alia that in order that the donation of an immovable may be valid, it must be made in a public document, specifying therein the property donated and the value of the charges which the done must satisfy. Corollarily, Article 709 of the same Code explicitly states that the titles of ownership, or other rights over immovable property, which are not duly inscribed or annotated in the Registry of Property shall not prejudice third persons. From the foregoing provisions, it may be inferred that as between the parties to a donation of immovable property, all that is required is for said donation to be contained in a public document. Registration is not necessary for it to be contained in a public document. It is not necessary for it to be considered valid and effective. However, in order to bind third persons, the donation must be registered in the Registry of Property. In the case at bar, the donation executed by Ignacio Gonzales in favor of his grand children, although in writing and duly notarized, has not been registered in accordance with law. For this reason, it shall not be binding upon private respondents who did not participate in said deed nor had no actual knowledge thereof.

Imperial v. Court of Appeals G.R. No. 112483, October 8, 1999, 316 SCRA 393 Gonzaga Reyes, J. FACTS: Leoncio Imperial was the owner of a parcel of land with an area of 32,837 sq. m. and located in Albay. On July 7, 1951, Leoncio sold the lot for Php 1.00 to his acknowledged natural son, petitioner in this case. Petitioner and Victor Imperial, adopted son of Leoncio, agreed that despite the designation of the contract as Deed of Absolute Sale, the transaction is in fact a Donation. Two years after, Leoncio filed a complaint for the Annulment of Donation. It was however resolved through a compromise agreement under the following terms and conditions: (1) Leoncio recognized and agreed the legality and validity of the rights of petitioner; and (2) petitioner agreed to sell a designated 1,000 sq.m. portion of the donated land.

137

Leoncio died leaving only two heirs: petitioner and Victor Imperial. On March 8, 1962, Victor was substituted in the complaint for annulment. He moved for the execution of judgment and it was granted. After 15 years, Victor died and was survived only by his natural father, Ricardo Villalon. Ricardo Villalon is a lessee of the portion of the subject property. Villalon died leaving his heirs, Cesar and Teresa Villalon, respondents in this case. In 1986, respondents filed a complaint for the annulment of the donation. Allegedly, it impairs the legitime of Victor Imperial. ISSUES: 1.) Whether or not the respondents have the right to question the inofficious donation and seek its reduction. 2.) Whether or not the 30-year prescriptive period is applicable in the reduction of the inofficious donation. HELD: 1.) Yes. At the time of the substitution, the judgment approving the compromise agreement has already been rendered. Victor merely participated in the execution of the compromise judgment. He was not a party to the compromise agreement. When Victor substituted Leoncio, he was not deemed to have renounced his legitime. He was therefore not precluded or estopped from subsequently seeking the reduction. Nor are Victors heirs, upon his death, precluded from doing so. This is in accordance with Articles 772 and 1053 of the new Civil Code, to wit: Article 772. Only those who at the time of the donors death have a right to the legitime and their heirs and successors in interest may ask for the reduction of the inofficious donation xxx. and Article 1053. If the heir should die without having accepted or repudiated the inheritance, his rights shall be transmitted to his heirs. 2.) No. Under Article 1144 of the New Civil Code, actions upon an obligation created by law must be brought within ten years from the right of action accrues. Thus, the 10-year prescriptive period applies to the obligation to reduce inofficious donations required under Article 771 of the New Civil Code to the extent that they impair the legitime of compulsory heirs. The cause of action to enforce a legitime accrues upon the death of the donordecedent. Clearly so, since only then that the net estate may be ascertained and on which basis, the legitimes may be determined. It took 24 years since the death of Leoncio to initiate this case. Thus, the action has long prescribed. Not only has prescription set in, they are also guilty of estoppel and laches. Fifteen years after the death of Leoncio, Victor died. Ricardo Villalon, Victors sole heir, died four years later. While Victor was alive, he gave no indication of any interest to contest the donation of his deceased father.

138

Republic of the Philippines v. Silim G.R. No. 140487, April 2, 2001, 356 SCRA 1 Kapunan, J. FACTS: Respondent Spouses Silim and Mangubat donated a 5,600 square meter parcel of land in favor of the Bureau of Public Schools of the Municipality of Malangas, Zamboanga del Sur. In the Deed of Donation, the respondents imposed the condition that the said property should be used exclusively and forever for school purposes only. This donation was accepted by the District Supervisor of the Bureau, through an Affidavit of Acceptance and/or Confirmation of Donation. A school building was thereafter constructed on the donated land. However, another school building that was also supposed to be allocated for the donated parcel of land could not be released since the government required that it be built upon a 1 hectare parcel of land. By reason of this, the District Supervisor and the vice-mayors wife entered into a Deed of Exchange whereby the donated lot was exchanged with a bigger lot owned by the latter. Consequently, the school buildings were constructed on this new school site and the school building previously erected on the donated land was dismantled and transferred to the new location. One day, respondents were surprised when he saw the vice-mayor constructing a house on the donated land. ISSUES: 1.) Whether or not there was a valid donation despite non-notation of the acceptance in the Deed of Donation, as required in Article 749. 2.) Whether or not the condition on the donation was violated. HELD: 1.) Yes. The purpose of the formal requirement for acceptance of a donation is to ensure that such is duly communicated to the donor. In the case at bar, a school building was immediately constructed after the donation was executed. Respondents had knowledge of the existence of the school building put up on the donated lot. The actual knowledge by respondents of the construction and existence of the school building fulfilled the legal requirement that the acceptance of the donation by the donee be communicated to the donor. 2.) No. There was no violation even after the donated lot was exchanged for another one. The purpose of the donation remains the same, which is for the establishment of a school. The exclusivity of the purpose was not altered or affected. In fact, the exchange of the lot for a much bigger one was in furtherance and enhancement of the purpose of the donation. The acquisition of the bigger lot paved the way for the release of funds for the construction of Bagong Lipunan school building which could not be accommodated by the limited area of the donated lot.

139

Gestopa v. Court of Appeals G.R. No. 111904, October 5, 2000, 342 SCRA 105 Quisumbing, J. FACTS: Spouses Danlag were the owners of six parcels of unregistered lands. They executed three deeds of donation mortis causa, two of which were in favor of Mercedes Danlag-Pilapil. All deeds contained the reservation of the rights of the donors to amend, cancel or revoke the donation during their lifetime, and to sell, mortgage, or encumber the properties donated during the donors' lifetime, if deemed necessary. The spouses then executed another deed of donation inter vivos in favor of Mercedes which contained the condition that the donors, Danlag spouses, shall continue to enjoy the fruits of the land during their lifetime and that the donee enjoy the fruits of the land during their lifetime and that the donee cannot sell or dispose of the land during the lifetime of the donors without their prior consent and approval. Consequently, Mercedes caused the transfer of the parcels of land's tax declaration to her name and paid the taxes on them. Spouses Danlag sold two parcels of lots to Spouses Agripino and Isabel Gestopa and executed a deed of revocation recovering the six parcels of land subject to the deed of donation inter vivos. Mercedes Pilapil filed with the Regional Trial Court against the Spouses Danlag and Gestopa, for quieting of title over the parcels of land and alleged that the land was donated to her by Diego Danlag and that she accepted the donation openly and publicly exercised rights of ownership over the donated properties, and transferred the tax declarations to her name. She also alleged that the donation inter vivos was coupled with conditions and, according to Mercedes, since its perfection, she had complied with all of them; that she had not been guilty of any act of ingratitude; and that Diego Danlag had no legal basis to revoke the donation and then in selling the two parcels of land to the Gestopa spouses. In their opposition, the spouses Gestopa and the Danlag averred that the deed of donation was null and void because it was obtained by Mercedes through machinations and undue influence. Even assuming it was validly executed, the intention was for the donation to take effect upon the death of the donor and that the donation was void for it left the donor, Diego Danlag, without any property at all. ISSUE: Whether the donation is a donation inter vivos or a donation mortis causa. HELD: The Court Rules that it was a donation inter vivos. The Court affirmed the Court of Appeals' decision that the reservation by the donor of lifetime usufruct indicated that he transferred to Mercedes the ownership over the donated properties; that the right to sell belonged to the donee, and the donor's right referred to that of merely giving consent; that the donor changed his intention by donating inter vivos properties already donated mortis causa; that the transfer to Mercedes' name of the tax declarations pertaining to the donated properties implied that the donation was inter vivos; and that Mercedes did not purchase two of the six parcels of land donated to her.

140

In ascertaining the intention of the donor, all of the deed's provisions must be read together. The granting clause shows that Diego donated the properties out of love and affection for the donee. This is a mark of a donation inter vivos. Second, the reservation of lifetime usufruct indicates that the donor intended to transfer the naked ownership over the properties. Third, the donor reserved sufficient properties for his maintenance in accordance with his standing in society, indicating that the donor intended to part with the six parcels of land. Lastly, the donee accepted the donation. An acceptance clause is a mark that the donation is inter vivos. Acceptance is a requirement for donations inter vivos. Donations mortis causa, being in the form of a will, are not required to be accepted by the donees during the donors' lifetime. The right to dispose of the properties belonged to the donee. The donor's right to give consent was merely intended to protect his usufructuary interests. The limitation on the right to sell during the donors' lifetime implied that ownership had passed to the donees and donation was already effective during the donors' lifetime. Hence, the moment that it was accepted by Mercedes Danlag-Pilapil, ownership of the properties was transferred.

Sumipat v. Banga G.R. No. 155810, August 13, 2004 Tinga, J. FACTS: The spouses Placida Tabo-tabo and Lauro Sumipat acquired three parcels of land. The couple was childless. Lauro Sumipat, however, sired five illegitimate children. They are the petitioners herein. Lauro executed a document denominated Deed of Absolute Transfer and/or Quit-Claim over Real Properties in favor of the petitioners. On the document, it appears that the signature of his wife, Placida which indicates that she gave her marital consent. Moreover, it was alleged that Lauro executed it when he was already very sick and bedridden that upon petitioner Lydias request, their neighbor Benjamin Rivera lifted the body of Lauro whereupon Lydia guided his hand in affixing his signature on the document. Lydia left but later returned on the same day and requested Lauros unlettered wife, Placida to sign on the said document. After Lauros death, his wife, Placida and petitioners jointly administered the properties, 50% of the produce went to his wife. As wifes share in the produce of the properties dwindled, she filed a complaint for declaration of partition disclaiming any partition in the execution of the subject document. ISSUE: Whether or not the questioned deed by its terms or under the surrounding circumstances has validly transferred title to the disputed properties to the petitioners. HELD: No. A perusal of the deed reveals that it is actually a gratuitous disposition of property a donation although Lauro Sumipat imposed upon the petitioners the condition that he and his wife, Placida, shall be entitled to one-half (1/2) of all the fruits or produce of the parcels of land for their subsistence and support. Where the deed of donation fails to show the acceptance, or where the formal notice of the acceptance, 141

made in a separate instrument, is either not given to the donor or else not noted in the deed of donation and in the separate acceptance, the donation is null and void. In this case, the donees acceptance of the donation is not manifested either in the deed itself or in a separate document. Hence, the deed as an instrument of donation is patently void. The Court declared that the deeds of sale questioned therein are not merely voidable but null and void ab initio as the supposed seller declared under oath that she signed the deeds without knowing what they were. The significant circumstance meant, the Court added, that her consent was not merely marred by vices of consent so as to make the contracts voidable, but that she had not given her consent at all.

142

Você também pode gostar